You are on page 1of 819

2011 PREP SA on CD-ROM

Question: 1

You are called to see a 16-year-old girl who recently was diagnosed with autoimmune hepatitis.
Her therapy includes azathioprine 75 mg/day and prednisone 20 mg/day. Because of complaints
of headaches and low-grade fevers for the past week, she has been taking extra-strength
acetaminophen (500 mg) every 4 hours. Her parents tell you that their daughter is experiencing
difficulty sleeping at night, and she takes frequent naps during the day. Physical examination
demonstrates scleral icterus. Vital signs are normal. She appears drowsy but is arousable. Initial
laboratory data include:

Aspartate aminotransferase, 120 units/L


Alanine aminotransferase, 220 units/L
Total bilirubin, 8.0 mg/dL (136.8 mcmol/L)
Direct bilirubin, 5.0 mg/dL (85.5 mcmol/L)
Prothrombin time, 18 seconds (normal, up to 12 seconds)
Partial thromboplastin time, 30 seconds (normal, up to 26 seconds)
Glucose, 85 mg/dL (4.7 mmol/L)
Albumin, 2.6 g/dL (26 g/L)
Ammonia, 95 mg/dL (normal, 35 to 65 mg/dL)

Of the following, the MOST appropriate next step is

A. albumin infusion

B. consultation with the regional liver transplant center

C. intravenous nutrition with branched-chain amino acids

D. methylprednisolone 1 mg/kg every 12 hours

E. percutaneous liver biopsy

Copyright 2010 by the American Academy of Pediatrics page 1


2011 PREP SA on CD-ROM

Critique: 1 Preferred Response: B

The girl described in the vignette demonstrates symptoms and signs of acute liver failure
(ALF), defined as hepatic decompensation occurring within weeks or a few months after the
diagnosis of liver disease. Her clinical presentation suggests early stage 2 encephalopathy,
manifested by a disturbed sleep-awake cycle (a characteristic of stage 1 encephalopathy) and
drowsiness. These findings, accompanied by deteriorating liver synthetic function
(hypoalbuminemia, hypoglycemia, and coagulopathy) indicate the need for immediate metabolic
support and signal the likelihood that a successful outcome will require a liver transplant. The
therapeutic approach for a child who presents in impending ALF must include management in an
intensive care setting while assessing the need for urgent transplantation. Accordingly,
immediately contacting the regional transplant center is the most appropriate next step in
management for this girl.
Over all age groups, the incidence of ALF in the United States is approximately 10,000 to
20,000 cases per year (17 cases per 100,000 population). Although its frequency in the
pediatric age group is unknown, ALF accounts for 10% to 15% of all pediatric liver transplants
performed in the United States yearly. The most recent overview of children who have ALF
indicates that a specific cause is found in approximately 50% of cases. For those children in
whom a definite cause of hepatic decompensation can be identified, acetaminophen is the most
common precipitating factor (14%); autoimmune liver disease ranks third (6%). The girl in the
vignette has a history of autoimmune hepatitis and recent exposure to acetaminophen, a
potentially hepatotoxic drug. Liver failure due to acetaminophen occurs most commonly in the
setting of intentional overdose, which clearly must be considered in this adolescent, who has a
recently diagnosed chronic and frequently debilitating disease. Therapeutic misadventures
(prolonged or inappropriate dosing) as well as chronic usage in the setting of underlying liver
disease also have been implicated in ALF.
Referral for possible transplant is essential in this case because both encephalopathy and
coagulopathy are independent risk factors for death or the need for liver transplantation in ALF.
When ALF is suspected, assessing aminotransferases may not be helpful because both
aspartate aminotransferase and alanine aminotransferase values may range from normal to
extremely high. Elevated aminotransferases are a marker for active hepatic inflammation, but
normal values may occur in patients who have fulminant hepatic necrosis with loss of more than
80% of functioning hepatocytes. Thus, normal aminotranferases are a potentially ominous sign.
During the period of patient assessment and stabilization, acetaminophen concentrations should
be monitored to determine the need for n-acetylcysteine treatment. Despite the availability of this
specific therapy, stage 2 encephalopathy and coagulopathy are predictive of the requirement for
transplant. Further management includes the use of intravenous 10% dextrose as part of a
supportive fluid regimen to manage hypoglycemia. However, dextrose infusions per se do not
influence the overall outcome. Intravenous mannitol is administered as an osmotic agent to
promote diuresis and ameliorate cerebral edema, and administration of oral/enteral lactulose or
neomycin can aid in reducing ammonia concentrations. Parenteral vitamin K should be provided
in an attempt to correct the liver disease-related coagulopathy.
Expansion of intravascular volume using albumin infusions is not indicated. Fresh frozen
plasma should be administered only to the actively bleeding patient who is unresponsive to

Copyright 2010 by the American Academy of Pediatrics page 2


2011 PREP SA on CD-ROM

vitamin K administration. Platelet administration may be required in the bleeding patient if liver
failure is complicated by profound thrombocytopenia (<50x103/mcL [50x109/L]) as a
consequence of hypersplenism (usually in the setting of chronic cirrhotic liver disease) or
disseminated intravascular coagulation. Administration of branched-chain amino acids (BCAAs)
has been suggested as a potential approach for improving encephalopathy in chronic liver
disease complicated by hepatic decompensation. These nutrients are metabolized in muscle,
where they act as gluconeogenic substrates, and not in the liver. BCAAs also compete with the
aromatic amino acids for uptake across the blood-brain barrier, thus theoretically reducing
synthesis of neurotransmitters that may exacerbate encephalopathy. However, BCAAs have not
been shown to influence the clinical course of ALF. Corticosteroids, although a primary therapy
for autoimmune liver disease, will not improve the outcome for a patient who has ALF and
hepatic encephalopathy. Finally, percutaneous liver biopsy is contraindicated in a patient
presenting with ALF and coagulopathy.

Suggested reading:

Alonso EM, Squires RH, Whittington PF. Acute liver failure in children. In: Suchy FJ, Sokol RJ,
Balistreri WF, eds. Liver Disease in Children. 3rd ed. New York, NY: Cambridge University
Press; 2007:71-96

Bucuvalas J, Yazigi N, Squires RH Jr. Acute liver failure in children. Clin Liver Dis. 2006;10:149-
168. DOI: 10.1016/j.cld.2005.10.006. Abstract available at:
http://www.ncbi.nlm.nih.gov/pubmed/16376799

Polson J, Lee WM; American Association for the Study of Liver Disease. AASLD position paper:
the management of acute liver failure. Hepatology. 2005;41:1179-1197. DOI: 10.1002/hep.20703.
Available at: http://www3.interscience.wiley.com/cgi-bin/fulltext/110472818/HTMLSTART

Roberts EA. Drug-induced liver disease. In: Suchy FJ, Sokol RJ, Balistreri WF, eds. Liver Disease
in Children. 3rd ed. New York, NY: Cambridge University Press; 2007:478-512

Squires RH Jr. Acute liver failure in children. Semin Liver Dis. 2008;28:153-166. DOI: 10.1055/s-
2008-1073115. Abstract available at: http://www.ncbi.nlm.nih.gov/pubmed/18452115

Squires RH Jr, Shneider BL, Bucuvalas J, et al. Acute liver failure in children: the first 348
patients in the pediatric acute liver failure study group. J Pediatr. 2006;148:652-658. DOI:
10.1016/j.jpeds.2005.12.051. Available at:
http://www.ncbi.nlm.nih.gov/pmc/articles/PMC2662127/?tool=pubmed

Copyright 2010 by the American Academy of Pediatrics page 3


2011 PREP SA on CD-ROM

Question: 2

You are evaluating a term infant in the neonatal intensive care unit for a seizure that occurred 1
day after birth. Her neonatal course has been otherwise unremarkable. The only significant
finding on physical examination is an erythematous patch involving the left side of the face,
including the upper and lower eyelids (Item Q2).

Of the following, the MOST common ophthalmologic finding associated with this condition is

A. aniridia

B. cataract

C. coloboma

D. glaucoma

E. microphthalmia

Copyright 2010 by the American Academy of Pediatrics page 4


2011 PREP SA on CD-ROM

Question: 2

(Courtesy of S Freedman)
Erythematous patch, as described for the infant in the vignette.

Copyright 2010 by the American Academy of Pediatrics page 5


2011 PREP SA on CD-ROM

Critique: 2 Preferred Response: D

The newborn described in the vignette has a port wine stain in the distribution of the
trigeminal nerve and seizures that suggest the diagnosis of Sturge-Weber syndrome. The finding
of a port wine stain affecting the upper and lower eyelids should prompt consideration of
associated lesions of the eye and central nervous system. Clinical findings of Sturge-Weber
syndrome include dermal capillary malformations in the distribution of the ophthalmic (V1) and
maxillary (V2) branches of the trigeminal nerve, ipsilateral central nervous system malformations
of the leptomeningeal vessels, and vascular malformations of the choroid of the eye associated
with glaucoma. Affected infants may have seizures, and some children have developmental
delay, learning disabilities, and intellectual disability. Evaluation of all infants who have port wine
stains in the distribution of the trigeminal nerve should include an examination by a pediatric
ophthalmologist and neuroimaging.
Glaucoma is an uncommon condition in the newborn and young infant, affecting fewer than
0.05% of children. In glaucoma, the examination may reveal corneal clouding or an increase in
corneal diameter (normal is approximately 10 mm) (Item C2A). However, these findings may not
be present at birth; they may develop over the first postnatal weeks to months. Symptoms of
epiphora (excessive tearing), photophobia, and blepharospasm (eyelid squeezing) may alert the
clinician to the development of congenital glaucoma. This clinical triad occurs because corneal
edema is accompanied by breaks in the basement membrane of the cornea. Enlargement of the
eye, or buphthalmos, may be seen as the aqueous drainage of the anterior chamber is blocked
and abnormally high pressure leads to stretching of the cornea and the entire eye. Congenital
glaucoma may be isolated or associated with conditions such as rubella syndrome, Axenfeld-
Rieger anomaly, Lowe syndrome, and Sturge-Weber syndrome.
Other ophthalmologic conditions affecting the newborn include aniridia, neonatal cataract,
coloboma, and microphthalmia. However, none of these entities is characterized by clouding of
the cornea or an increase in its diameter. Aniridia (absence of the visible iris) is a rare condition
that may be an autosomal dominant trait or occur sporadically (Item C2B). Up to one third of
sporadic cases occur with Wilms tumor. Neonatal cataracts (opacified lenses) (Item C2C) may
be caused by viral infections, inborn errors of metabolism, or trauma; associated with
syndromes or other eye malformations; genetic; or idiopathic. Coloboma (a defect in one of the
structures of the eye, including the eyelid, iris, retina, choroid, or optic disc) (Item C2D) is a more
common congenital abnormality of the eye that may be isolated, autosomal dominant, or
associated with a syndrome such as CHARGE or cat eye syndrome. Microphthalmia is a term
used to describe a neonatal eye whose diameter is less than two thirds of the normal 16 mm. It
may be isolated, although it is seen with chromosomal disorders and syndromes. All of these
conditions should be evaluated by a pediatric ophthalmologist.

Suggested reading:

Gupta BK, Hamming NA, Miller MT. Neonatal eye disease. In: Martin RJ, Fanaroff AA, Walsh MC,
eds. Fanaroff and Martins Neonatal-Perinatal Medicine: Diseases of the Fetus and Infant. 8th
ed. Philadelphia, Pa: Mosby Elsevier: 2006:1726-1746

Copyright 2010 by the American Academy of Pediatrics page 6


2011 PREP SA on CD-ROM

Olitsky SE, Hug D, Smith LP. Childhood glaucoma. In: Kliegman RM, Behrman RE, Jenson HB,
Stanton BF, eds. Nelson Textbook of Pediatrics. 18th ed. Philadelphia, Pa: Saunders Elsevier;
2007:2608-2610

Tallman B, Tan OT, Trainor S, et al. Location of port-wine stains and the likelihood of ophthalmic
and/or central nervous system complications. Pediatrics. 1991;87:323-327. Available at:
http://pediatrics.aappublications.org/cgi/content/abstract/87/3/323

Copyright 2010 by the American Academy of Pediatrics page 7


2011 PREP SA on CD-ROM

Critique: 2

(Courtesy of S Freedman)
Advanced congenital glaucoma: There are enlarged, hazy corneas resulting from long-standing
elevated intraocular pressure.

Copyright 2010 by the American Academy of Pediatrics page 8


2011 PREP SA on CD-ROM

Critique: 2

(Courtesy of the Media Lab at Doernbecher)


Aniridia is an absence of the iris.

Copyright 2010 by the American Academy of Pediatrics page 9


2011 PREP SA on CD-ROM

Critique: 2

(Courtesy of M Rimsza)
Opacification of the lens (ie, cataract) resulting from congenital rubella syndrome.

Copyright 2010 by the American Academy of Pediatrics page 10


2011 PREP SA on CD-ROM

Critique: 2

(Courtesy of M Rimsza)
Keyhole-shaped pupil consistent with an iris coloboma.

Copyright 2010 by the American Academy of Pediatrics page 11


2011 PREP SA on CD-ROM

Question: 3

During a health supervision visit, a mother expresses concern that her 2-year-old sons
foreskin does not retract easily. He has not indicated any discomfort with voiding, has frequent
wet diapers, and is otherwise well. On physical examination, the appropriately grown child,
who is not circumcised, has no redness, edema, or tenderness of the foreskin. The foreskin
can be retracted slightly beyond the meatal opening only.

Of the following, the MOST appropriate next step is to

A. encourage daily washing of the area with gentle soap and water

B. encourage the mother to retract the foreskin fully several times weekly

C. obtain urinalysis and urine culture

D. prescribe a 1-month course of topical steroids

E. refer the child to a urologist for circumcision

Copyright 2010 by the American Academy of Pediatrics page 12


2011 PREP SA on CD-ROM

Critique: 3 Preferred Response: A

Phimosis, a tight foreskin opening that prevents the foreskin from being retracted over the
glans penis, is a common, normal finding during infancy. At birth, 96% of infant boys have
physiologic phimosis, but as adhesions between the inner foreskin and glans gradually
separate, the condition resolves. The prepuce is retractable in 15% of 6-month-old, 50% of 1-
year-old, and 90% of 3-year-old uncircumcised boys. Phimosis persists in 8% of 6 to 7 year olds
and 1% of 16 to 18 year olds. Primary (physiologic) phimosis has no underlying abnormalities;
secondary (pathologic) phimosis frequently is due to scarring from infection or dermatologic
conditions such as lichen sclerosis et atrophicus (balanitis xerotica obliterans). Smegma, a pasty
or cheesy material largely made up of desquamated epithelial cells and sebaceous secretions,
commonly is found under the foreskin. This is not infectious and actually is involved in the
physiologic separation of the phimotic prepuce.
The mother in the vignette should be counseled about appropriate care of the uncircumcised
foreskin, which includes external cleansing with gentle soap and water. The foreskin never
should be retracted forcefully because this can cause bleeding, inflammation, and often stronger
adherence to the glans. Paraphimosis occurs when the foreskin is retracted beyond the coronal
sulcus, leading to venous stasis, pain, and the inability to reduce the foreskin. Paraphimosis is a
urologic emergency. Although manual reduction often can resolve the immediate paraphimosis,
emergency circumcision sometimes is required.
Traditionally, treatment for persistent phimosis has been surgical, and parent preference still
guides some treatment decisions. However, topical corticosteroid applied to the phimotic ring for
30 days has proven effective in two thirds or more cases. Circumcision or other surgical
approaches are indicated, particularly when the condition does not respond to topical treatment
or if the phimosis leads to difficulty voiding, the child is approaching puberty, there is an
underlying pathologic cause for the phimosis, or the child has recurrent urinary tract infections.
Because the child in the vignette has no symptoms suggestive of a urinary tract infection,
urinalysis and urine culture are not necessary.

Suggested reading:

Ashfield JE, Nickel KR, Siemens DR, MacNeily AE, Nickel JC. Treatment of phimosis with topical
steroids in 194 children. J Urol. 2003;169:1106-1108. DOI: 10.1097/01.ju.0000048973.26072.eb.
Abstract available at: http://www.ncbi.nlm.nih.gov/pubmed/12576863

Elder JS. Anomalies of the penis and urethra. In: Kliegman RM, Behrman RE, Jenson HB, Stanton
BF, eds. Nelson Textbook of Pediatrics. 18th ed. Philadelphia, Pa: Saunders Elsevier; 2007:2253-
2259

Lawless MR, Serwint JR. In brief: the foreskin. Pediatr Rev. 2006;27:477-478. DOI:
10.1542/10.1542/pir.27-12-477. Available at:
http://pedsinreview.aappublications.org/cgi/content/full/27/12/477

Tekgul S, Riedmiller H, Gerharz E, et al. Phimosis. In: Guidelines on Paediatric Urology. Arnhem,

Copyright 2010 by the American Academy of Pediatrics page 13


2011 PREP SA on CD-ROM

The Netherlands: European Association of Urology, European Society for Paediatric Urology;
2009. Available at: http://www.guideline.gov/summary/summary.aspx?doc_id=12529

Copyright 2010 by the American Academy of Pediatrics page 14


2011 PREP SA on CD-ROM

Question: 4

A 7-year-old girl presents for the second time in a month with an eyelid nodule. Initially, there was
tenderness and erythema along the upper lid at the base of the lashes. She responded to a short
course of oral antistaphylococcal antibiotics. Now there is a new nodule without erythema or
induration (Item Q4). She has no fever, visual disturbance, or conjunctival injection.

Of the following, the MOST appropriate management is

A. application of warm compresses

B. injection of dexamethasone into the lesion

C. prompt incision and drainage

D. topical ophthalmic antibiotic drops

E. urgent referral to ophthalmology

Copyright 2010 by the American Academy of Pediatrics page 15


2011 PREP SA on CD-ROM

Question: 4

(Courtesy of D Krowchuk)
Nodule, as described for the girl in the vignette.

Copyright 2010 by the American Academy of Pediatrics page 16


2011 PREP SA on CD-ROM

Critique: 4 Preferred Response: A

Small chalazia, such as described for the girl in the vignette, often resolve with warm
compresses and expectant management over several days to weeks. Larger ones, those
interfering with vision, or those failing to resolve must be excised by an ophthalmologist. A
chalazion represents a subacute or chronic inflammation of the meibomian glands and is a
lipogranulomatous nodule (Item C4A). If the nodule occurs on the upper lid, it may irritate the
cornea as the lid is opened and closed and possibly obstruct vision. A complete eye examination
should be undertaken, with a fluorescein examination indicated for children who have signs or
symptoms of corneal abrasion. If corneal abrasion is suspected, the affected eye should be
patched and topical ophthalmic antibiotics instilled pending urgent referral to an ophthalmologist.
A stye (external hordeolum) is inflammation, usually representing bacterial infection, of the
sebaceous gland (gland of Zeis) or sweat gland (gland of Moll) of the eyelash follicle and
occurs at the lid margin (Item C4B). It is generally self-limited and often resolves spontaneously.
An internal hordeolum frequently occurs in the sweat or meibomian gland. If this long gland duct
becomes obstructed, the swelling may occur on the tarsal conjunctiva (under the eyelid) and
may benefit from use of topical ophthalmic antibiotics.
Intralesional injection of steroids for chalazion has met with some success in older children
and adults, but sedation or anesthesia may be required for such treatment in younger children.
Steroid use may cause depigmentation of the lid in children who have darker skin.

Suggested reading:

Ehrenhaus MP, Sturridge KA. Hordeolum. eMedicine Specialties, Ophthalmology, Lid. 2009.
Available at: http://emedicine.medscape.com/article/1213080-overview

Fansler JL, Schraga ED, Santen S. Chalazion. eMedicine Specialties, Emergency Medicine,
Ophthalmology. 2009. Available at: http://emedicine.medscape.com/article/797763-overview

Lederman C, Miller M. Hordeola and chalazia. Pediatr Rev. 1999; 20:283-284. DOI:
10.1542/10.1542/pir.20-8-283. Available at:
http://pedsinreview.aappublications.org/cgi/content/full/20/8/283

Wald ER. Periorbital and orbital infections. Pediatr Rev. 2004;25:312-320. DOI:
10.1542/10.1542/pir.25-9-312. Available at:
http://pedsinreview.aappublications.org/cgi/content/full/25/9/312

Copyright 2010 by the American Academy of Pediatrics page 17


2011 PREP SA on CD-ROM

Critique: 4

(Courtesy of D Krowchuk)
A chalazion is caused by granulomatous inflammation of meibomian glands. The obstruction
leads to obstruction of the gland orifice, resulting in a nodule involving the upper or lower lid.

Copyright 2010 by the American Academy of Pediatrics page 18


2011 PREP SA on CD-ROM

Critique: 4

(Courtesy of K Woodin)
A stye (or hordeolum) presents as a tender erythematous papule or pustule on the lid margin.

Copyright 2010 by the American Academy of Pediatrics page 19


2011 PREP SA on CD-ROM

Question: 5

An 18-year-old girl presents with a history of occasional mild chest pain of 1 weeks duration.
The episodes occur at rest and have not affected her performance as a competitive long-
distance swimmer. On physical examination, her heart rate is 48 beats/min and blood pressure is
105/65 mm Hg. Electrocardiography demonstrates left ventricular hypertrophy, which is confirmed
by echocardiography.

Of the following, the MOST likely cause of these findings is

A. aortic stenosis

B. athletes heart

C. cardiac conduction disturbance

D. coronary artery anomaly

E. hypertrophic cardiomyopathy

Copyright 2010 by the American Academy of Pediatrics page 20


2011 PREP SA on CD-ROM

Critique: 5 Preferred Response: B

Chest pain is a common complaint in the general pediatric practice, and no significant
cardiac abnormality is found in most cases. Musculoskeletal pain is the most common cause of
chest pain in the child and adolescent and is the most likely source of the mild pain reported for
the girl in the vignette. Rarely, however, chest pain is a sign of cardiac pathology such as
pericarditis, myocarditis, or myocardial infarction. The lack of pain during exercise (when the
individual would be expected to be in a high myocardial demand state) for this girl is reassuring,
suggesting that the pain is of a benign, noncardiac origin. In addition, the lack of associated
symptoms such as radiation of the pain to the face, arms, or back; associated dizziness or
syncope; and physical examination findings suggestive of cardiovascular disease are all
reassuring.
The low blood pressure and heart rate reported for this girl are not of particular concern
and most likely reflect athletes heart. The well-trained athlete exhibits findings on physical
examination and laboratory assessment that differ from the general population. Specifically, the
resting heart rate typically is 20% to 30% lower than the average for age in the population, and
blood pressure is at the lower range of normal. Often, a physiologic flow murmur can be
detected, which is caused by a high cardiac output state. Findings on electrocardiography can
include left ventricular hypertrophy. Echocardiographic findings can include left ventricular wall
thickening/hypertrophy associated with normal or hyperdynamic left ventricular contractility.
Such physiologic left ventricular hypertrophy can be distinguished from pathologic hypertrophic
cardiomyopathy based upon features of diastolic function, mitral valve disease, and assessment
of left ventricular outflow tract obstruction. The patient who has hypertrophic cardiomyopathy
exhibits mitral regurgitation, obstruction to blood flow due to the narrowed left ventricular
outflow tract, and diastolic dysfunction. The individual who has athletes heart manifests none of
these pathologic echocardiographic features. Novel imaging methods (including cardiac magnetic
resonance imaging) can differentiate hypertrophic cardiomyopathy from left ventricular
hypertrophy associated with athletes heart. In addition, when the well-trained individual who
has athletes heart is deconditioned for several months, echocardiography demonstrates
resolution of the hypertrophy.
Aortic valve stenosis of such severity to cause chest pain always is associated with a
pronounced systolic ejection-type murmur that is heard most commonly at the left midsternal
border, with radiation to the right infraclavicular region. Cardiac conduction abnormalities can
manifest as chest pain in the young child who has difficulty distinguishing pain from other
unusual chest symptoms such as palpitations. However, the teenager is unlikely to describe
palpitations or other rhythm disturbances as evoking chest pain. Coronary artery anomalies are
rare and include acquired conditions such as Kawasaki disease and congenital lesions such as
anomalous left coronary arising from the pulmonary artery (ALCAPA) and coronary fistulae.
Coronary anomalies do not cause left ventricular hypertrophy and should be detected by
detailed echocardiography. As noted previously, hypertrophic cardiomyopathy can be
distinguished from the left ventricular hypertrophy associated with athletes heart by the
presence of a pathologic murmur caused by either mitral regurgitation or left ventricular outflow
tract obstruction, findings on echocardiography, and chest pain that is more likely to occur during
exercise and have associated dizziness, syncope, and radiation of the pain.

Copyright 2010 by the American Academy of Pediatrics page 21


2011 PREP SA on CD-ROM

As a result of reviewing this information, do you intend to make a change in practice


to provide better patient care?
Yes No

Suggested reading:

Cava JR, Sayger PL. Chest pain in children and adolescents. Pediatr Clin North Am.
2004;51:1553-1568. DOI: 10.1016/j.pcl.2004.07.002. Abstract available at:
http://www.ncbi.nlm.nih.gov/pubmed/15561173

Maron BJ. Distinguishing hypertrophic cardiomyopathy from athlete's heart physiological


remodeling: clinical significance, diagnostic strategies and implications for preparticipation
screening. Br J Sports Med. 2009;43:649-656. Abstract available at:
http://www.ncbi.nlm.nih.gov/pubmed/19734498

Copyright 2010 by the American Academy of Pediatrics page 22


2011 PREP SA on CD-ROM

Question: 6

A 17-year-old girl presents with complaints of recurring headaches that are becoming more
frequent. She was diagnosed with migraine headaches 2 years ago. Previously she had an
average of one migraine per month, but she now has one per week. She has no double vision,
vomiting, or awakening from sleep with headaches. Headaches can occur at any time of day, are
throbbing and bifrontal, and are associated with nausea and phonophobia. They are relieved by
nonsteroidal anti-inflammatory drugs and by lying down in the dark. They rarely last more than 4
hours. The adolescent and her mother are concerned that the increased headache frequency
could be due to a brain tumor or aneurysm.

Of the following, the MOST appropriate next step is

A. brain magnetic resonance angiography to rule out aneurysm

B. brain magnetic resonance imaging to rule out brain tumor

C. head computed tomography scan with contrast to rule out brain tumor

D. lumbar puncture with manometry to rule out elevated intracranial pressure

E. perform no diagnostic testing at this time

Copyright 2010 by the American Academy of Pediatrics page 23


2011 PREP SA on CD-ROM

Critique: 6 Preferred Response: E

When evaluating a child who has a headache, the first decision point is whether to perform
medical diagnostic testing, particularly neuroimaging such as head computed tomography (CT)
scan or brain magnetic resonance imaging (MRI). This decision is based on an assessment of
the likelihood of a primary headache (eg, migraine, tension, cluster) or a secondary headache
(due to disease). Primary headaches are painful paroxysmal events occurring in isolation or,
more commonly, as part of a chronic, recurring pain disorder. No underlying lesion is causing the
pain. Most children, adolescents, and adults presenting to the primary care physician with
headache, including the girl described in the vignette, have primary headaches, and no medical
diagnostic testing is needed as part of validated standard of care. The advantages and
disadvantages of neurodiagnostic tests for a variety of conditions are noted in Item C6.
A family history of aneurysms may increase the risk for aneurysms (and anxiety), but it is
not an indication for neuroimaging testing for aneurysms in a patient younger than age 30 years.
However, because cigarette smoking increases the risk for aneurysmal rupture, this is a good
opportunity to remind families about the dangers of smoking.
Most brain tumors are sporadic primary tumors or metastatic, not familial, and, therefore, a
family history of brain tumors is not an indication for neuroimaging in most cases. In the absence
of symptoms and signs of elevated intracranial pressure or meningitis, lumbar puncture is not
needed.
Imaging studies (CT scan, MRI, and occasionally magnetic resonance angiography),
sometimes followed by lumbar puncture with manometry, are indicated when there is suspicion
of intracranial disease (eg, tumor, aneurysm) or other medical processes, particularly processes
that increase intracranial pressure. Symptoms that may herald the presence of secondary
headache include pain awakening from sleep, pain relieved by vomiting, and pain increased by
the Valsalva maneuver. Additional symptoms such as new binocular double vision, any focal
numbness or weakness, or loss of coordination should prompt an investigation. Secondary
headaches are more likely in the presence of abnormal signs such as papilledema, acquired
(new) ocular malalignment with double vision, stiff neck, limb weakness, or ataxia.

Suggested reading:

Evans RW. Diagnostic testing for migraine and other primary headaches. Neurol Clin.
2009;27:393-415. DOI: 10.1016/j.ncl.2008.11.009. Abstract available at:
http://www.ncbi.nlm.nih.gov/pubmed/19289222

Gunner KB, Smith HD. Practice guideline for diagnosis and management of migraine headaches
in children and adolescents: part one. J Pediatr Health Care. 2007;21:327-332. DOI:
10.1016/j.pedhc.2007.06.004

Lewis DW. Pediatric migraine. Neurol Clin. 2009;27:481-501. DOI: 10.1016/j.ncl.2008.11.003.


Abstract available at: http://www.ncbi.nlm.nih.gov/pubmed/19289227

Lewis DW. Pediatric migraine. Pediatr Rev. 2007;28:43-53. DOI: 10.1542/10.1542/pir.28-2-43.

Copyright 2010 by the American Academy of Pediatrics page 24


2011 PREP SA on CD-ROM

Available at: http://pedsinreview.aappublications.org/cgi/content/full/28/2/43

Strine TW, Okoro CA, McGuire LC, Balluz LS. The associations among childhood headaches,
emotional and behavioral difficulties, and health care use. Pediatrics. 2006;117:1728-1735. DOI:
10.1542/peds.2005-1024. Available at:
http://pediatrics.aappublications.org/cgi/content/full/117/5/1728

Copyright 2010 by the American Academy of Pediatrics page 25


2011 PREP SA on CD-ROM

Critique: 6

Copyright 2010 by the American Academy of Pediatrics page 26


2011 PREP SA on CD-ROM

Question: 7

You are meeting with a couple who moved to your area recently. They explain that their first baby
died suddenly and unexpectedly after birth. The baby was born vaginally at 34 weeks gestation
after a pregnancy complicated by reduced fetal movement and polyhydramnios. They were told
that the umbilical cord was "too short" and broke off during delivery. They did not want to see the
baby after she died, but they did request an autopsy. They would like to have another baby and
want to know if this is likely to happen again. You obtain permission to send for medical records
and the autopsy report and arrange to see them again after you have reviewed these.

Of the following, the baby MOST likely had

A. achondroplasia

B. arthrogryposis

C. trisomy 13

D. trisomy 18

E. trisomy 21

Copyright 2010 by the American Academy of Pediatrics page 27


2011 PREP SA on CD-ROM

Critique: 7 Preferred Response: B

The combination of reduced-to-absent fetal movement, polyhydramnios, and short umbilical


cord described for the infant in the vignette is suggestive of fetal akinesia sequence, which is
associated with arthrogryposis. The term "arthrogryposis" describes multiple congenital
contractures affecting two or more different regions of the body. It is important to note that
arthrogryposis does not represent a single entity and is not a diagnosis unto itself. Instead, it is
a clinical description that is part of more than 300 syndromes. Thus, it is incumbent upon the
diagnostician to define the cause of arthrogryposis in any given individual to determine
inheritance pattern, natural history, and options for management.
The arthrogryposes may be due to genetic factors, nongenetic factors, or both. Among the
potential causes are neurologic abnormalities (involving brain, spine, or peripheral nerve),
muscular defects (including muscular dystrophies and mitochondrial myopathies), and maternal
diseases (eg, myasthenia gravis, multiple sclerosis).
Management of arthrogryposis is multidisciplinary and includes geneticists, neurologists,
orthopedists, physiatrists, and physical and occupational therapists. Every effort is made to
promote increased range of motion at the sites of contracture. The potential success of such
efforts depends on the underlying cause of the arthrogryposis, but intervention should be early
and aggressive.
Achondroplasia, the most common dwarfing condition, is not associated with
arthrogryposis. Although trisomies 13, 18, and 21 may be associated with reduced fetal
movement, they are not typically associated with arthrogryposis.

Suggested reading:

Bamshad M, Van Heest AE, Pleasure D. Arthrogryposis: a review and update. J Bone Joint
Surg. 2009;91(suppl 4):40-46

Bernstein RM. Arthrogryposis and amyoplasia. J Am Acad Orthop Surg. 2002;10:417-424.


Abstract available at: http://www.ncbi.nlm.nih.gov/pubmed/12470044

Bevan WP, Hall, JG, Bamshad M, Staheli LT, Jaffe KM. Arthrogryposis multiplex congenita
(amyoplasia): an orthopaedic perspective. J Pediatr Orthop. 2007;27:594-600

Hall JG. Arthrogryposis multiplex congenita: etiology, genetics, classification, diagnostic


approach, and clinical aspects. J Pediatr Orthop B. 1997;6:159-166. Abstract available at:
http://www.ncbi.nlm.nih.gov/pubmed/9260643

Copyright 2010 by the American Academy of Pediatrics page 28


2011 PREP SA on CD-ROM

Question: 8

A 17-year-old sexually active girl returns to your office with complaints of dysuria of 5 days
duration. At the onset of her symptoms, urinalysis showed 8 white blood cells/high-power field,
but the urine culture yielded negative results, so you did not treat her. She has no findings of note
on past medical history. The physical and genital examinations performed by a resident working
with you today yield normal results, and she asks for guidance on further testing.

Of the following, the MOST helpful next diagnostic test(s) is(are)

A. endocervical swab for Neisseria gonorrhoeae and Chlamydia cultures

B. repeat urinalysis and urine culture

C. serologic testing for Chlamydia

D. vaginal swabs for N gonorrhoeae and Chlamydia cultures

E. vaginal swabs for N gonorrhoeae and Chlamydia nucleic acid amplification test (NAAT) testing

Copyright 2010 by the American Academy of Pediatrics page 29


2011 PREP SA on CD-ROM

Critique: 8 Preferred Response: E

Adolescents who have dysuria and pyuria on urinalysis but negative urine cultures, such
as the girl described in the vignette, should be evaluated for urethritis. The causes of urethritis in
both males and females include sexually transmitted infections, especially those caused by
Neisseria gonorrhoeae and Chlamydia trachomatis. Previously, diagnosing these infections
required obtaining material for culture from the endocervix in females and the urethra in males.
However, newer tests that employ nucleic acid amplification (NAAT) of DNA or RNA from these
organisms are more sensitive than cultures and are specific. Because these are not culture
tests, NAATs do not require viable organisms. The increased sensitivity of NAATs is attributable
to their ability to produce a positive signal from as little as a single copy of the target DNA or
RNA. The addition of hybrid capture technology allows for testing for C trachomatis and N
gonorrhoeae from one sample.
Commercial tests differ in their amplification methods and their target nucleic acid
sequences. The three types of available NAAT tests use polymerase chain reaction (PCR),
strand displacement amplification (SDA), or transcription-mediated amplification (TMA). Urine in
males and females and cervical or vaginal swabs in females may be used for testing. Vaginal
swabs may be obtained either by the clinician or the patient, making them the preferable test
source. Vaginal swabs are as sensitive as cervical swabs and are more sensitive than urine
specimens when using TMA. The ability to obtain specimens without a pelvic examination allows
for testing of asymptomatic individuals and use in nontraditional screening sites. However,
symptomatic patients should undergo a complete urogenital examination.
Currently, NAAT tests are not recommended for tests of cure (the test result may remain
positive for 3 or more weeks after treatment) or for medicolegal detection in sexual abuse
victims. However, this recommendation is under review.
Repeat urinalysis and culture are unlikely to be useful for this patient because the previous
specimen had no growth. Serologic testing is not useful for the diagnosis of C trachomatis
infections.

As a result of reviewing this information, do you intend to make a change in practice


to provide better patient care?
Yes No

Suggested reading:

Association of Public Health Laboratories. Laboratory Diagnostic Testing for Chlamydia


trachomatis and Neisseria gonorrhoeae. Expert Consultation Meeting Summary Report.
Atlanta, Ga: January 13-15, 2009 Available at:
http://www.aphl.org/aphlprograms/infectious/std/documents/ctgclabguidelinesmeetingreport.pdf

Gaydos CA. Nucleic acid amplification tests for gonorrhea and chlamydia: practice and
applications. Infect Dis Clin North Am. 2005;19:367-386. DOI: 10.1016/j.idc.2005.03.006.
Abstract available at: http://www.ncbi.nlm.nih.gov/pubmed/15963877

Copyright 2010 by the American Academy of Pediatrics page 30


2011 PREP SA on CD-ROM

Hobbs MM, van der Pol B, Totten P, et al. From the NIH: proceedings of a workshop on the
importance of self-obtained vaginal specimens for detection of sexually transmitted infections.
Sex Transm Dis. 2008;35:8-13. DOI: 10.1097/OLQ.0b013e31815d968d. Available at:
http://journals.lww.com/stdjournal/Fulltext/2008/01000/From_the_NIH__Proceedings_of_a_Works
hop_on_the.3.aspx

Masek BJ, Arora N, Quinn N, et al. Performance of three nucleic acid amplification tests for
detection of Chlamydia trachomatis and Neisseria gonorrhoeae by use of self-collected vaginal
swabs obtained via an Internet-based screening program. J Clin Microbiol. 2009;47:1663-1667.
DOI: 10.1128/JCM.02387-08. Available at:
http://jcm.asm.org/cgi/content/full/47/6/1663?view=long&pmid=19386838

Miller CA, Shafer MAB. Chlamydia trachomatis. In: Neinstein LS, Gordon CM, Katzman DK,
Rosen DS, Woods ER, eds. Adolescent Health Care: A Practical Guide. 5th ed. Philadelphia Pa:
Lippincott Williams & Wilkins, a Wolters Kluwer business; 2008:805-818

Shafer MA, Moncada J, Boyer CB, Betsinger K, Flinn SD, Schachter J. Comparing first-void urine
specimens, self-collected vaginal swabs, and endocervical specimens to detect Chlamydia
trachomatis and Neisseria gonorrhoeae by a nucleic acid amplification test. J Clin Microbiol.
2003;41:4395-4359. DOI: 10.1128/JCM.41.9.4395-4399.2003. Available at:
http://jcm.asm.org/cgi/content/full/41/9/4395?view=long&pmid=12958275

Copyright 2010 by the American Academy of Pediatrics page 31


2011 PREP SA on CD-ROM

Question: 9

You are evaluating a 5-year-old boy in the emergency department for lethargy. His mother reports
that the boy has been lethargic since awakening this morning. She adds that he has been
complaining of headaches and has been having morning emesis for several weeks. She also
reports that he has become increasingly "clumsy" over the past 4 to 6 weeks. On physical
examination, the patient is difficult to arouse; has sluggish pupillary responses; and has rapid,
deep, sustained breaths at a rate of 35 breaths/min.

Of the following, the next BEST step in the initial management of this patient is

A. administration of dexamethasone

B. administration of furosemide

C. endotracheal intubation

D. noncontrast computed tomography scan of the head

E. urine toxicology screen

Copyright 2010 by the American Academy of Pediatrics page 32


2011 PREP SA on CD-ROM

Critique: 9 Preferred Response: C

The clinical presentation and medical history of the child described in the vignette is very
concerning for an underlying brain tumor, based on the several week history of headaches,
morning emesis, and disturbance in motor balance. In addition, he has clinical evidence of
acutely elevated intracranial pressure, as evidenced by his abnormal mental status, delayed
pupillary response, and his rapid and deep respiratory pattern. Emergent intubation and control
of the airway is indicated before obtaining diagnostic imaging because the child is at significant
risk for brainstem herniation and respiratory arrest. Intubation should be performed by
experienced personnel to avoid hypoventilation and worsening of intracranial pressure.
Increased intracranial pressure can result from a variety of disease processes, including
head trauma, central nervous system infections, hypoxic-ischemic injuries, and hydrocephalus.
Clinical signs of increased intracranial pressure can include altered levels of consciousness,
bradycardia, elevated blood pressure, and abnormal breathing patterns.
Normal breathing is controlled by a complex interaction of the central nervous system,
peripheral chemoreceptors, and mechanical receptors. Abnormal breathing patterns often are
seen in association with central nervous system diseases and may include hyperventilation,
apneustic breathing (prolonged inspiration), or Cheynes-Stokes respirations (alternating apnea
and hyperventilation). Once the child in the vignette is intubated and ventilation is controlled,
additional diagnostic and therapeutic interventions are indicated, including contrast-enhanced
computed tomography scan of the head (Item C9). Although furosemide has been administered
for increased intracranial pressure in the past, administration of hypertonic saline or mannitol is
safer and more effective. Dexamethasone may be useful to decrease vasogenic edema if a
brain tumor is confirmed, but it is less effective in acutely lowering intracranial pressure. The
childs history and physical examination findings are not suggestive of an overdose, obviating
the need for a toxicology screen.

Suggested reading:

Avner JR. Altered states of consciousness. Pediatr Rev. 2006:27:331-338. DOI:


10.1542/10.1542/pir.27-9-331. Available at:
http://pedsinreview.aappublications.org/cgi/content/full/27/9/331

Frankel LR. Neurologic emergencies and stabilization. In: Kliegman RM, Behrman RE, Jenson HB,
Stanton BF, eds. Nelson Textbook of Pediatrics. 18th ed. Philadelphia, Pa: Saunders Elsevier;
2007:405-410

Sarnair AP, Heidemann SM, Respiratory pathophysiology and regulation. In: Kliegman RM,
Behrman RE, Jenson HB, Stanton BF, eds. Nelson Textbook of Pediatrics. 18th ed. Philadelphia,
Pa: Saunders Elsevier; 2007:1719-1730

Copyright 2010 by the American Academy of Pediatrics page 33


2011 PREP SA on CD-ROM

Critique: 9

(Courtesy of B Poss)
Computed tomography scan of the patient described in the vignette demonstrating a large
posterior fossa mass (medulloblastoma) with associated hemorrhage, obstruction of the 4th
ventricle, and dilation of the lateral and 3rd ventricles.

Copyright 2010 by the American Academy of Pediatrics page 34


2011 PREP SA on CD-ROM

Question: 10

A 9-year-old boy is admitted to the hospital because of altered consciousness and fever. A
lumbar puncture, performed because of a stiff neck and altered mentation, reveals cerebrospinal
fluid protein of 100 mg/dL; cytologic findings of 100 white cells, 95% lymphocytes, and 10 red
blood cells; and glucose of 40 mg/dL. Additional serum laboratory findings include:

Glucose, 70 mg/dL (3.9 mmol/L)


Sodium, 124 mEq/L (124 mmol/L)
Potassium, 3.2 mEq/L (3.2 mmol/L)
Chloride, 93 mEq/L (93 mmol/L)
Bicarbonate, 18 mEq/L (18 mmol/L)

Results of serum cortisol and adrenocorticotrophic hormone concentrations are pending. Broad-
spectrum antibiotic therapy is initiated.

Of the following, the MOST appropriate next step is to

A. administer an initial bolus dose of hydrocortisone hemisuccinate 2 mg/kg intravenously

B. administer 0.9% saline with 10% dextrose 10 mL/kg intravenously over 4 hours

C. administer 3% saline 10 mL/kg intravenously over 4 hours

D. initiate sodium polystyrene phosphonate per rectum every 8 hours

E. restrict fluids to half of the maintenance requirement

Copyright 2010 by the American Academy of Pediatrics page 35


2011 PREP SA on CD-ROM

Critique: 10 Preferred Response: E

The boy described in the vignette has meningitis, with low serum sodium and potassium and
normal glucose concentrations. The serum electrolyte findings are typical for the syndrome of
inappropriate antidiuretic hormone release (SIADH) associated with central nervous system
inflammation or injury. Another possibility in the differential diagnosis is adrenal insufficiency.
However, primary adrenal insufficiency leads to hyponatremia and hyperkalemia. Isolated
cortisol deficiency due to adrenocorticotrophic hormone (ACTH) deficiency can present with
hyponatremia and a normal potassium concentration but generally is associated with
hypoglycemia. Urinary electrolyte assessment could clarify this picture because urine osmolality
and sodium are increased (urine osmolality > serum osmolality, and urine sodium >20 mEq/L [20
mmol/L]) in SIADH, whereas a dilute urine is excreted in adrenal insufficiency.
Fluid restriction is the most important first step in management of SIADH. If there is reason to
suspect cortisol deficiency (blood pressure instability or evidence of vascular instability),
administration of hydrocortisone hemisuccinate is appropriate, but if it is possible to await
laboratory results of cortisol and ACTH measurements, glucocorticoid therapy can follow
laboratory confirmation of the condition. Isolated ACTH deficiency is a rare finding in meningitis.
Administering additional saline is not useful in SIADH unless hypertonic saline is administered
with a loop diuretic such as furosemide to help to retain salt and excrete water. Decreasing fluid
intake is almost always a better option because acute correction may lead to pontine
demyelination. Sodium polystyrene phosphonate resin binds potassium and is useful in the
treatment of hyperkalemia. In the absence of hyperkalemia, rectal administration of this drug
represents an unnecessary and risky therapy.

Suggested reading:

Albanese A, Hindmarsh P, Stanhope R. Management of hyponatraemia in patients with acute


cerebral insults. Arch Dis Child. 2001;85:246-251. DOI: 10.1136/adc.85.3.246. Available at:
http://www.ncbi.nlm.nih.gov/pmc/articles/PMC1718897/?tool=pubmed

Brierley J, Carcillo JA, Choong K, et al. Clinical practice parameters for hemodynamic support of
pediatric and neonatal septic shock: 2007 update from the American College of Critical Care
Medicine. Crit Care Med. 2009;37:666-688. DOI: 10.1097/CCM.0b013e31819323c6. Abstract
available at: http://www.ncbi.nlm.nih.gov/pubmed/19325359

Rivkees SA. Differentiating appropriate antidiuretic hormone secretion, inappropriate antidiuretic


hormone secretion and cerebral salt wasting: the common, uncommon, and misnamed. Curr
Opin Pediatr. 2008;20:448-452. DOI: 10.1097/MOP.0b013e328305e403. Abstract available at:
http://www.ncbi.nlm.nih.gov/pubmed/18622203

Verive MJ. Hyperkalemia: treatment & medication. eMedicine Specialties, Pediatrics: Cardiac
Disease and Critical Care Medicine, Critical Care. 2010. Available at:
http://emedicine.medscape.com/article/907543-treatment

Copyright 2010 by the American Academy of Pediatrics page 36


2011 PREP SA on CD-ROM

Question: 11

You are examining a 5-year-old boy who is unable to ambulate independently and uses a
specialized walker. He speaks in full sentences, although he has an articulation disorder that
affects the intelligibility of his speech. His developmental skills have been delayed, but no skills
have been lost. When you lift him onto the examination table, his legs scissor. He has no
dysmorphisms, and his skin is free of neurocutaneous stigmata. His head circumference is at the
50th percentile, and his weight and height are at the 10th percentile. On neurologic examination,
his eyes intermittently deviate outward. He has brisk relexes and sustained clonus. He has good
eye contact and smiles at his parents while reaching toward a picture of a car on the wall. The
nurse practitioner asks what other manifestations are associated with his underlying disorder.

Of the following, the MOST likely additional manifestation is

A. absence seizures

B. dystonia

C. gastrointestinal symptoms

D. high-frequency hearing loss

E. tethered cord

Copyright 2010 by the American Academy of Pediatrics page 37


2011 PREP SA on CD-ROM

Critique: 11 Preferred Response: C

The child described in the vignette has cerebral palsy (CP), the third most common major
developmental disability after autism and intellectual disorder. The overall prevalence of CP is
between 1.5 and 2.0 cases per 1,000 live births. The diagnosis of CP refers to the static motor
impairment exhibited by affected individuals.
A range of conditions is associated with CP, including cognitive disability (about 50%),
sensory impairment, limitation of movement, scoliosis, joint instability, bladder dysfunction, altered
growth and nutrition, difficulties with communication, altered appearance, and chronic illness
such as seizures (30%).
Individuals may have deficits of the cranial nerves, which lead to oculomotor abnormalities
and uncoordinated movements that affect swallowing and speech. Gastrointestinal problems
are particularly common. Patients may have gastroesophageal reflux or constipation, with the
latter resulting from altered smooth muscle and sphincter tone as well as effects of medications,
decreased activity, and varying levels of fluid intake. Feeding problems occur in about 50% of
patients and may lead to significant undernutrition.
Seventy-five percent of children have spasticity, but only 10% to 15% have a dyskinetic
form of CP, presenting with dystonia. Seizures occur in more than 25% of affected individuals,
with the greatest risk among those who have spastic quadriplegia and hemiplegia. The seizures
typically are generalized tonic-clonic, not absence. High-frequency hearing loss is typical of
athetoid CP caused by bilirubin encephalopathy; the frequency of sensorineural hearing loss is
about 30% in other cases. The presence of a tethered cord is closely linked to spina bifida, not
CP.

Suggested reading:

Cooley WC and Committee on Children with Disabilities. Providing a primary care medical home
for children and youth with cerebral palsy. Pediatrics. 2004;114:1106-1113. DOI:
10.1542/peds.2004-1409. Available at:
http://pediatrics.aappublications.org/cgi/content/full/114/4/1106

Eicher PS. Nutrition and feeding. In: Dormans JP, Pellegrino L, eds. Caring for Children with
Cerebral Palsy: A Team Approach. Baltimore, Md: Paul H. Brookes Publishing Co; 1998:243-282

Johnson MW, Hoon AH Jr, Kaufman WE. Neurobiology, diagnosis and management of cerebral
palsy. In: Accardo PJ, ed. Capute & Accardos Neurodevelopmental Disabilities in Infancy and
Childhood. 3rd ed. Baltimore, Md: Paul H. Brookes Publishing Co; 2008:61-82

Nield LS, Nanda S, Someshwar J, et al. Cerebral palsy: a multisystem review. Consultant for
Pediatricians. 2007;6:337-340. Available at:
http://www.consultantlive.com/display/article/10162/11570

Pelligrino L. Well-child care and health maintenance. In: Dormans JP, Pellegrino L. Caring for
Children with Cerebral Palsy: A Team Approach. Baltimore, Md: Paul H. Brookes Publishing Co;

Copyright 2010 by the American Academy of Pediatrics page 38


2011 PREP SA on CD-ROM

1998:79-80

Copyright 2010 by the American Academy of Pediatrics page 39


2011 PREP SA on CD-ROM

Question: 12

A 16-year-old boy presents to the emergency department with a 5-day history of congestion and
headache. Today he developed right eye swelling, severe right-sided frontal headache, and
vomiting. On physical examination, the lethargic but arousable boy has proptosis of the right eye,
nasal congestion, a clear pharynx, and a supple neck. The remainder of his evaluation yields no
findings of note.

Of the following, the BEST test for establishing the diagnosis is

A. lumbar puncture

B. magnetic resonance imaging of the brain and sinuses

C. nasopharyngeal aspirate for bacterial culture

D. toxicology screen

E. transillumination of the sinuses

Copyright 2010 by the American Academy of Pediatrics page 40


2011 PREP SA on CD-ROM

Critique: 12 Preferred Response: B

Brain abscesses can arise by direct spread from a contiguous focus or hematogenous
spread. The prodromal congestion and headache followed by right eye swelling described for
the boy in the vignette suggest extension from sinusitis. In adolescents, epidural extension from
frontal sinuses is an important complication to consider. The eye swelling also raises concern
for preseptal or orbital cellulitis. The best test for assessing the extent of infection and
identifying a brain abscess is neuroimaging, such as magnetic resonance imaging or computed
tomography scan (Item C12).
Lumbar puncture in the face of asymmetric neurologic findings is relatively contraindicated
due to a risk for herniation. Further, although there may be evidence of inflammation in the
cerebrospinal fluid, this procedure would not define the extent of infection or confirm the
diagnosis of an abscess. Sinusitis typically is caused by organisms entering from the
nasopharynx (Streptococcus pneumoniae, Haemophilus influenzae, oral anaerobes, and
occasionally Staphylococcus aureus). However, cultures from the nasopharynx are not useful
in determining the etiologic agents of sinusitis or the extent of disease. Transillumination of the
sinuses has not been shown to be reliable in diagnosing sinusitis and certainly would not help in
diagnosing a brain abscess. The sudden onset of obtundation in an adolescent may raise
concern for intoxication or other ingestion, but the other clinical findings for this boy argue
against such a cause.
Bacterial cultures from brain abscesses reflect the mechanism by which the infection arises
and frequently are polymicrobial. For abscesses developing as a complication of sinusitis, the
microbiology reflects the sinus infection, as cited previously. Other potential sources that could
lead to brain abscesses are head trauma, postneurosurgical complications, and direct extension
from dental infection. Brain abscesses seeded by hematogenous spread may complicate
endocarditis or congenital heart disease with right-to-left shunting. In these cases, S aureus and
S viridans are the most common pathogens. Nocardia may enter the bloodstream from the lungs
and subsequently seed the cerebral cortex. Aspergillus and Cryptococcus can cause brain
abscesses via a similar route of infection in immunocompromised hosts.

Suggested reading:

Al Masalma M, Armougom F, Scheld WM, et al. The expansion of the microbiological spectrum of
brain abscesses with use of multiple 16S ribosomal DNA sequencing. Clin Infect Dis.
2009;48:1169-1178. DOI: 10.1086/597578. Available at:
http://www.journals.uchicago.edu/doi/full/10.1086/597578

Goodkin HP, Harper MB, Pomeroy SL. Intracerebral abscess in children: historical trends at
Childrens Hospital Boston. Pediatrics. 2004;113:1765-1770. Available at:
http://pediatrics.aappublications.org/cgi/content/full/113/6/1765

Hunter JV, Morriss MC. Neuroimaging of central nervous system infections. Semin Pediatr Infect
Dis. 2003;14:140-164. Abstract available at: http://www.ncbi.nlm.nih.gov/pubmed/12881802

Copyright 2010 by the American Academy of Pediatrics page 41


2011 PREP SA on CD-ROM

Southwick FS. Pathogenesis, clinical manifestations, and diagnosis of brain abscess. UpToDate
Online 17.3. 2009. Available at:
http://www.utdol.com/online/content/topic.do?topicKey=cns_infe/7193&view=print

Copyright 2010 by the American Academy of Pediatrics page 42


2011 PREP SA on CD-ROM

Critique: 12

(Courtesy of D Mulvihill)
Brain abscess: Sagittal T1-weighted magnetic resonance imaging demonstrates a low-attenuation
lesion in the parietal lobe with a contrast-enhancing rim and surrounding edema.

Copyright 2010 by the American Academy of Pediatrics page 43


2011 PREP SA on CD-ROM

Question: 13

A 9-year-old girl who was diagnosed with influenza 3 days ago presents with fever
(temperature of 40.0C), cough, and lethargy. On physical examination, the ill-appearing girl has
tachypnea, subcostal retractions, and crackles on auscultation of the right lung field. She is
intubated for respiratory distress. Tracheal secretions are purulent, and a blood culture grows
gram-positive cocci in clusters.

Of the following, the MOST appropriate antimicrobial therapy for this girl is

A. azithromycin

B. ceftazidime

C. nafcillin

D. penicillin

E. tobramycin

Copyright 2010 by the American Academy of Pediatrics page 44


2011 PREP SA on CD-ROM

Critique: 13 Preferred Response: C

The result of the Gram stain of the blood culture described for the girl in the vignette
suggests a staphylococcal infection. Most likely, she has Staphylococcus aureus bacteremia
complicating an influenza infection. Most staphylococci produce penicillinases, enzymes that
hydrolyze and inactivate penicillins and, therefore, are resistant to penicillin and ampicillin.
Accordingly, a penicillinase-resistant penicillin such as nafcillin or oxacillin is the treatment of
choice for this girl.
Nafcillin is a semisynthetic penicillin derivative and beta-lactam antibiotic that has
antibacterial activity against susceptible staphylococci. It inhibits bacterial wall synthesis by
binding to penicillin-binding proteins and impeding peptidoglycan synthesis. Penicillinase-resistant
penicillins are used to treat infections caused by penicillin-resistant strains of staphylococci.
They also can be used to treat streptococcal species, although they have less activity against
streptococci than does penicillin. Penicillin is appropriate therapy for mild-to-severe infections
due to susceptible gram-positive organisms such as Streptococcus pyogenes, some gram-
negative organisms such as Neisseria meningitidis, some anaerobes, and spirochetes.
First- and second-generation cephalosporins have activity against susceptible
staphylococci, but the expanded-spectrum cephalosporins (third- and fourth-generation) such
as ceftazidime do not and are not appropriate for treatment of staphylococcal bacteremia. Some
strains of S aureus, both in the community and in the hospital, are resistant to numerous
antibiotics, including methicillin, and require treatment with vancomycin. Aminoglycosides such
as tobramycin are used to treat infections caused by gram-negative organisms. Although
macrolides such as azithromycin may have activity against methicillin-susceptible strains of
staphylococci, they are used more appropriately to treat community-acquired pneumonia caused
by Mycoplasma and Chlamydia.

Suggested reading:

American Academy of Pediatrics. Staphylococcal infections. In: Pickering LK, Baker CJ, Kimberlin
DW, Long SS, eds. Red Book: 2009 Report of the Committee on Infectious Diseases. 28th ed.
Elk Grove Village, Ill: American Academy of Pediatrics; 2009:601-615

Kaplan SL, Hulten KG, Mason EO. Staphylococcus aureus infections (coagulase-positive
staphylococci). In: Feigin RD, Cherry JD, Demmler-Harrison GJ, Kaplan SL, eds. Feigin &
Cherrys Textbook of Pediatric Infectious Diseases. 6th ed. Philadelphia, Pa: Saunders Elsevier;
2009:1197-1212

Malik ZA, Litman N. In brief: the penicillins. Pediatr Rev. 2006;27:471-473. DOI:
10.1542/10.1542/pir.27-12-471. Available at:
http://pedsinreview.aappublications.org/cgi/content/full/27/12/471

Todd JK. Staphylococcal infections. Pediatr Rev. 2005;26:444-450. DOI: 10.1542/10.1542/pir.26-


12-444. Available at: http://pedsinreview.aappublications.org/cgi/content/full/26/12/444

Copyright 2010 by the American Academy of Pediatrics page 45


2011 PREP SA on CD-ROM

Question: 14

A 14-year-old boy who has cystic fibrosis presents with fever, cough, and respiratory distress.
On physical examination, his temperature is 38.9C, respiratory rate is 28 breaths/minute, heart
rate is 90 beats/minute, blood pressure is 116/74 mm Hg, and pulse oximetry is 91% on room air.
Chest radiography reveals bilateral infiltrates. After collecting blood and sputum specimens for
culture, you initiate treatment with intravenous ceftazidime and tobramycin.

Of the following, the MOST likely adverse effect that can occur with this treatment regimen is

A. Achilles tendonitis

B. aplastic anemia

C. gallbladder sludge

D. interstitial pneumonitis

E. ototoxicity

Copyright 2010 by the American Academy of Pediatrics page 46


2011 PREP SA on CD-ROM

Critique: 14 Preferred Response: E

The patient described in the vignette, who has cystic fibrosis, has been prescribed a
combination of a cephalosporin and an aminoglycoside with antipseudomonal activity to treat
pneumonia caused by Pseudomonas aeruginosa. Cephalosporins, like other beta-lactam
antibiotics, can be associated with rashes, Clostridium difficile-induced diarrhea,
thrombocytopenia, and leukopenia. Aminoglycosides such as gentamicin, tobramycin, and
amikacin are notable for their associated nephrotoxicity and ototoxicity. The mechanism of
nephrotoxicity is believed to be antibiotic accumulation within the kidney during its uptake and
retention in the proximal convoluted tubule, resulting in cellular damage. Aminoglycoside-induced
ototoxicity can result in permanent, severe, bilateral, high-frequency sensorineural hearing loss,
with some temporary vestibular dysfunction. The mechanism of ototoxicity is degeneration of
hair cells and neurons within the cochlea. Accordingly, the most likely adverse effect to expect
with the antibiotic regimen administered to this patient is ototoxicity. Like aminoglycosides,
vancomycin has been associated with nephrotoxicity, and less commonly, ototoxicity; the
nephrotoxic effects can occur with vancomycin alone or be additive to those seen with
aminoglycosides when the antibiotics are used in combination.
Tendonitis (including Achilles tendonitis) has been associated with fluoroquinolones such as
ciprofloxacin. Aplastic anemia was notoriously associated with the use of chloramphenicol, but
use of this agent has been rare in the United States in the past 2 decades. Gallbladder sludging
or pseudocholethiasis is a reversible condition associated with ceftriaxone. Finally, interstitial
pneumonitis or fibrosis has been associated with nitrofurantoin.

Suggested reading:

Aminoglycosides. In: The Merck Manuals Online Medical Library. 2009. Available at:
http://www.merck.com/mmpe/sec14/ch170/ch170b.html?qt=aminoglycosides&alt=sh819sec14-
ch170-ch170b-256

Contopoulos-Ioannidis DG, Giotis ND, Baliatsa DV, Ioannidis JPA. Extended-interval


aminoglycoside administration for children: a meta-analysis. Pediatrics. 2004;114:e111-e118.
Available at: http://pediatrics.aappublications.org/cgi/content/full/114/1/e111

Guthrie OW. Aminoglycoside induced ototoxicity. Toxicology. 2008;249:91-96. DOI:


10.1016/j.tox.2008.04.015. Abstract available at: http://www.ncbi.nlm.nih.gov/pubmed/18514377

Nagai J, Takano M. Molecular aspects of renal handling of aminoglycosides and strategies for
preventing the nephrotoxicity. Drug Metab Pharmacokinet. 2004;19:159-170. DOI:
10.2133/dmpk.19.159. Available at: http://www.jstage.jst.go.jp/article/dmpk/19/3/19_159/_article

Vancomycin. In: The Merck Manuals Online Medical Library. 2009. Available at:
http://www.merck.com/mmpe/sec14/ch170/ch170q.html

Copyright 2010 by the American Academy of Pediatrics page 47


2011 PREP SA on CD-ROM

Question: 15

A 10-year-old boy was discharged from the pediatric intensive care unit 6 weeks ago following
treatment for a severe asthma exacerbation. At the time of admission, he required endotracheal
intubation and mechanical ventilation for 6 days. Today his mother expresses concern that the
boy has become increasingly hoarse when speaking over the past 2 weeks and that his
breathing is noisier. His current medications include a twice-daily fluticasone metered dose
inhaler, oral leukotriene antagonist, and oral antihistamine. On physical examination, you notice
audible biphasic stridor and hoarseness during phonation. He is currently afebrile, his respiratory
rate is 30 breaths/min, and his heart rate is 90 beats/min.

Of the following, the MOST likely cause for this boys symptoms is

A. asthma exacerbation

B. bacterial tracheitis

C. gastroesophageal reflux

D. subglottic stenosis

E. vocal cord nodule

Copyright 2010 by the American Academy of Pediatrics page 48


2011 PREP SA on CD-ROM

Critique: 15 Preferred Response: D

When evaluating a child who has recurrent or chronic respiratory symptoms, an appropriate
history and physical examination should provide clues to the cause. Biphasic stridor, as heard in
the child described in the vignette, is a high-pitched inspiratory and expiratory noise that is
specific for the subglottic space. Biphasic stridor and hoarseness can be complications after
intubation and are due to mechanical trauma from the endotracheal tube and secondary edema.
Recognizing that the subglottic space is the narrowest aspect of the airway during intubation
and ensuring a small air leak can help decrease the risk of tissue trauma, necrosis, and the
incidence of subglottic stenosis (Item C15A).
Wheezing is another high-pitched noise, but it typically occurs during expiration and
represents pathology in the intrathoracic trachea, bronchi, or bronchioles. An asthma
exacerbation occurs when the lower airways develop inflammation, mucus production, and
airway narrowing. Common triggers in children include viral infections, allergic rhinitis, and
exercise. Although this boy has asthma, biphasic stridor is rare during an asthma exacerbation.
Hoarseness can be a complication from inhaled corticosteroids prescribed for asthma treatment
due to direct irritation of the upper airway; other common causes of hoarseness include viral
upper respiratory tract infections and vocal cord nodules (Item C15B). Vocal cord nodules
represent one of the most common causes of hoarseness, but such nodules usually are due to
chronic voice abuse or misuse, which is not evident for this boy.
Gastroesophageal reflux (GER) can present with regurgitation, atypical chest pain, vomiting,
cough, hoarseness, or wheezing. There usually are few or no clinical findings on physical
examination, and biphasic stridor is not associated with GER. Further, a Cochrane review could
not demonstrate the effectiveness of antireflux treatment for hoarseness.
Bacterial tracheitis is a potentially serious infection of the upper airway that generally
occurs in children younger than 7 years of age. The rapid onset of high fevers in a toxic-
appearing child following a viral laryngotracheobronchitis infection should prompt immediate
intervention. Staphylococcus aureus is the most common identified cause. The lack of fever and
the gradual onset of symptoms described in the vignette make bacterial tracheitis unlikely.
Similar to the evaluation of stridor or wheezing, the approach to hoarseness involves a
careful history, physical examination, and differential diagnosis (Item C15C). When interventions
directed at a specific cause do not improve symptoms, flexible or direct laryngoscopy can aid by
allowing direct viewing of the larynx and vocal cord region.

Suggested reading:

Hastriter EV, Olsson JM. In brief: hoarseness. Pediatr Rev. 2006;27:e47-e48. DOI:
10.1542/10.1542/pir.27-6-e47. Available at:
http://pedsinreview.aappublications.org/cgi/content/full/27/6/e47

Hopkins C, Yousaf U, Pedersen M. Acid reflux treatment for hoarseness. Cochrane Database
Syst Rev. 2006;1:CD005054. DOI: 10.1002/14651858.CD005054.pub2. Available at:
http://www.mrw.interscience.wiley.com/cochrane/clsysrev/articles/CD005054/frame.html

Copyright 2010 by the American Academy of Pediatrics page 49


2011 PREP SA on CD-ROM

Critique: 15

(Courtesy of D Kirse)
Acquired subglottic stenosis: There is a narrow opening in the airway (arrow).

Copyright 2010 by the American Academy of Pediatrics page 50


2011 PREP SA on CD-ROM

Critique: 15

(Courtesy of D Kirse)
Vocal cord nodules are located anteriorly (arrows).

Copyright 2010 by the American Academy of Pediatrics page 51


2011 PREP SA on CD-ROM

Critique: 15

Copyright 2010 by the American Academy of Pediatrics page 52


2011 PREP SA on CD-ROM

Question: 16

A 2-year-old boy is brought to the emergency department because he has been having episodes
in which his arms and legs become stiff, his head turns to the side, and his jaw becomes
clenched. The episodes, which occurred in the past hour, last for 30 seconds and resolve
spontaneously. After the episodes, he is fussy but awake and alert. His mother reports that
earlier today he had fallen off the bed but subsequently acted normally. He is otherwise healthy
and takes no medications. His maternal grandmother has type 2 diabetes and gastroesophageal
reflux, and his mother suffers from an anxiety disorder. On physical examination, his heart rate is
140 beats/min, respiratory rate is 24 breaths/min, blood pressure is 100/60 mm Hg, and oxygen
saturation is 98%. All of his other physical findings are within normal parameters.

Of the following, the MOST likely cause of this childs signs and symptoms is:

A. epidural hematoma

B. glyburide ingestion

C. lorazepam ingestion

D. metoclopramide ingestion

E. new-onset seizure disorder

Copyright 2010 by the American Academy of Pediatrics page 53


2011 PREP SA on CD-ROM

Critique: 16 Preferred Response: D

The child described in the vignette is exhibiting signs of an acute dystonic reaction, one of
many neurologic manifestations that can result from toxic ingestions. Acute dystonia is a well-
recognized adverse effect of metoclopramide as well as typical and atypical antipsychotic
medications. Glyburide, an oral hypoglycemic agent, may cause sedation and seizures related to
hypoglycemia, and lorazepam, a benzodiazepine, causes sedation and coma in overdose. An
epidural hematoma is likely to cause sedation and focal motor deficits. Finally, although
differentiating an acute dystonic reaction from a seizure may be challenging, the stereotypical
muscular activity and lack of unresponsiveness during the episodes reported for this boy are
hallmarks of acute dystonia.
More than 50% of all poisonings reported to the American Association of Poison Control
Centers involve children younger than 5 years of age, and although most of these exposures
are not serious, those that result in neurologic toxicity frequently require medical intervention.
Common neurologic signs and symptoms following significant toxic exposures include seizures,
coma, acute dystonia, ataxia, and muscular abnormalities (Item C16). These neurologic effects
may be related to a direct effect on the central or peripheral nervous system or to resultant
metabolic abnormalities (eg, hypoglycemia, acidosis).

Suggested reading:

Lamkin S, Buhl D. Extrapyramidal side effects: could you identify them in the emergency
department? J Emerg Nurs. 2009;35:72-73. DOI: 10.1016/j.jen.2008.10.014

Rodgers GC Jr, Condurache T, Reed M, Gal P, Bestic M. Poisonings. In: Kleigman RM, Behrman
RE, Jenson HB, Stanton BF, eds. Nelson Textbook of Pediatrics. 18th ed. Philadelphia, Pa:
Saunders Elsevier; 2007:339-356

Copyright 2010 by the American Academy of Pediatrics page 54


2011 PREP SA on CD-ROM

Critique: 16

Copyright 2010 by the American Academy of Pediatrics page 55


2011 PREP SA on CD-ROM

Question: 17

A 6-month-old infant presented in the newborn period with intestinal malrotation and mid-gut
volvulus. Emergency surgery resulted in resection of his entire small bowel, except for 7 cm of
duodenum. He has been maintained on parenteral nutrition since then and is listed for small bowel
transplantation at the regional transplant center. His parenteral nutrition regimen provides 120
kcal/kg per day and includes 20% dextrose, 3 g/120 kcal per day amino acids, and 3 g/kg per day
lipids. He has had increasing jaundice over the past month. On physical examination, the alert,
afebrile, and icteric infant has a firm liver edge palpable 3.5 cm below the right costal margin.
Laboratory data include:

Total bilirubin, 12.5 mg/dL (213.8 mcmol/L)


Direct bilirubin, 8.0 mg/dL (136.8 mcmol/L)
Alanine aminotransferase, 200 units/L
Aspartate aminotransferase, 150 units/L
Gamma-glutamyl transpeptidase, 180 units/L

Of the following, the MOST appropriate treatment is to

A. add phenobarbital 5 mg/kg per day orally

B. add ursodeoxycholic acid 20 mg/kg per day orally

C. decrease amino acids to 1.5 g/120 kcal per day

D. decrease dextrose to 10%

E. decrease lipids to 1.0 g/kg per day

Copyright 2010 by the American Academy of Pediatrics page 56


2011 PREP SA on CD-ROM

Critique: 17 Preferred Response: E

Parenteral nutrition (PN) is a life-sustaining therapy for patients whose limited


gastrointestinal function does not permit adequate enteral absorption of nutrients. Despite its
ability to maintain nutritional status and support growth, long-term PN has been associated with
numerous serious and sometimes life-threatening complications, including bloodstream infections
due to bacterial and fungal contamination of central venous catheters, metabolic derangements,
and PN-associated liver disease (PNALD). The infant described in the vignette has PNALD, an
extremely challenging problem that is especially common in patients who have short-bowel
syndrome, for reasons that are not completely understood. Although the causes of
hepatocellular dysfunction and cholestasis (indicated by direct hyperbilirubinemia and elevated
gamma-glutamyl transpeptidase values) in this clinical setting may be multifactorial, current data
suggest that excess intravenous lipids or specific elements in lipid emulsions play a major role in
the pathogenesis of PNALD. Accordingly, the most reasonable approach for the infant in the
vignette is to decrease the amount of infused lipid.
For the patient who has an extremely short bowel, PN serves as a "bridge therapy."
Because the infant in the vignette retains only a short length of duodenum, the prognosis is dire
for achieving any significant small bowel adaptation that can lead to recovery of intestinal
absorptive function. Therefore, long-term survival clearly depends upon a successful small
intestinal transplant. For patients in whom residual bowel length is sufficient to undergo adaptive
change and support some level of fluid and nutrient absorption, early reintroduction of enteral
feedings remains the best option for limiting the occurrence and severity of PNALD.
Careful ongoing clinical assessment is essential for all patients receiving PN to identify and
treat potential complications of this therapy. In addition to routine biochemical monitoring (Item
C17), scrupulous attention must be paid to strict adherence to aseptic technique, both in
parenteral fluid preparation and central line care. Ideally, central venous catheter access,
including care required for routine dressing changes, should be carried out only by personnel
trained and credentialed in line management. Such precautions should limit PN-associated
complications, although PNALD remains a vexing and often unavoidable problem.
All major nutrients infused in PN solutions, as well as other factors (eg, infection, hepatic
accumulation of bile acids) have been implicated in the pathogenesis of PNALD. However,
advances in the composition and delivery of nutrients to infants receiving PN have resulted in
amino acid profiles that mimic those during breastfeeding, and control of carbohydrate intake
avoids hyperglycemia and wide swings in blood glucose values. Recently, increased focus has
been placed on the role of intravenous lipids in the pathogenesis of PNALD. Both excess lipid
administration and phytosterols, present in soybean emulsions used for PN, have been
postulated as causative factors. Reducing infused lipids to 1.0 g/kg per day in this infant should
ameliorate PNALD at least partially while preventing the onset of essential fatty acid deficiency.
One recent study in two infants reported dramatic improvement in PNALD following replacement
of the standard omega-6 fatty acid-based lipid infusate with a phytosterol-free omega-3 fatty
acid emulsion.
Control of carbohydrate and protein intake, as demonstrated in the vignette, should limit PN-
associated metabolic derangements. Accordingly, these major nutrients are much less likely to be
implicated in the pathogenesis of PNALD in this infant, and reduction in parenteral glucose or

Copyright 2010 by the American Academy of Pediatrics page 57


2011 PREP SA on CD-ROM

amino acid intake would not be expected to ameliorate cholestasis. Early studies suggested that
phenobarbital enhances bile salt-independent biliary flow. However, its efficacy in this condition
has not been demonstrated, and this therapy is not recommended for use in any pediatric
cholestatic state. Ursodeoxycholic acid is a bile acid that does not form micelles, undergoes
enterohepatic circulation, and increases hepatocellular bile excretion. However, the extreme
short bowel with absent ileum (the site of active bile salt reabsorption) described for the infant in
the vignette precludes its effectiveness.

Suggested reading:

Clayton PT, Whitfield P, Iyer K. The role of phytosterols in the pathogenesis of liver complications
of pediatric parenteral nutrition. Nutrition. 1998;14:158-164. Abstract available at:
http://www.ncbi.nlm.nih.gov/pubmed/9437703

Colomb V, Jobert-Giraud A, Lacaille F, Goulet O, Fournet JC, Ricour C. The role of lipid emulsions
in cholestasis associated with long-term parenteral nutrition in children. JPEN J Parenter Enteral
Nutr. 2000;24:345-350. Abstract available at: http://www.ncbi.nlm.nih.gov/pubmed/11071594

Gura KM, Duggan CP, Collier SB, et al. Reversal of parenteral nutritionassociated liver disease
in two infants with short bowel syndrome using parenteral fish oil: implications for future
management. Pediatrics. 2006;118:e197-e201. DOI: 10.1542/peds.2005-2662. Available at:
http://pediatrics.aappublications.org/cgi/content/full/118/1/e197

Kleinman RE. Parenteral nutrition. In: Pediatric Nutrition Handbook. 6th ed. Elk Grove Village, Ill:
American Academy of Pediatrics; 2009:519-540

Copyright 2010 by the American Academy of Pediatrics page 58


2011 PREP SA on CD-ROM

Critique: 17

Copyright 2010 by the American Academy of Pediatrics page 59


2011 PREP SA on CD-ROM

Question: 18

During the 1-week health supervision visit, a mother who is exclusively breastfeeding asks about
vitamin and iron supplementation for her healthy term infant. She explains that her previous child,
who was born at 30 weeks gestation, was discharged with an oral iron supplement and
vitamins.

Of the following, the MOST appropriate oral supplement to initiate for this infant at this visit is

A. calcium

B. folic acid

C. iron

D. vitamin D

E. vitamin K

Copyright 2010 by the American Academy of Pediatrics page 60


2011 PREP SA on CD-ROM

Critique: 18 Preferred Response: D

The most appropriate oral supplement to initiate at 1 week of age for an exclusively
breastfeeding term infant is vitamin D. In 2008, new guidelines from the American Academy of
Pediatrics recommended that breastfed and partially breastfed infants be supplemented with 400
IU of vitamin D daily within days of birth. This change in the recommended amount of vitamin D
supplementation arose because of continued reports of rickets in breastfed infants. Exclusive
breastfeeding without adequate sun exposure or vitamin D supplementation is a risk factor for
vitamin D deficiency and rickets.
Term newborns are assumed to have adequate iron stores for the first 4 to 6 months after
birth. The adequate intake of iron for the first 6 postnatal months is 0.27 mg/day and is estimated
from the content of iron in human milk and its high bioavailability. Recent studies have shown that
exclusively breastfed infants who received iron supplementation between 1 and 6 months of
age had improved hemoglobin concentrations at 6 months of age as well as better visual acuity
and higher Bayley Psychomotor Indices at 13 month of age when compared with
unsupplemented peers. The AAP Committee on Nutrition recently recommended that exclusively
breastfed term infants receive a supplement of elemental iron at 1 mg/kg per day, starting at 4
months of age and continuing until appropriate iron-containing foods have been introduced. The
preterm infant has lower iron content than the term infant and requires initiation of iron
supplementation between 2 and 4 weeks of age and extending through 12 months of age.
The term infant has adequate calcium stores, in contrast to the preterm infant, who is born
during the period when 80% of calcium, phosphorus, and magnesium are accrued. The term
infant also has adequate folate stores. The routine use of a standard dose of intramuscular
vitamin K at birth minimizes the risk of hemorrhagic disease of the newborn, which results from
vitamin K deficiency.

As a result of reviewing this information, do you intend to make a change in practice


to provide better patient care?
Yes No

Suggested reading:

Baker RD, Greer FR, and the Committee on Nutrition. Diagnosis and prevention of iron deficiency
and iron-deficiency anemia in infants and young children (03 years of age). Pediatrics.
2010;126:1040-1050. Available at:
http://pediatrics.aappublications.org/cgi/content/abstract/peds.2010-2576v1

Dee DL, Sharma AJ, Cogswell ME, Grummer-Strawn LM, Fein SB, Scanlon KS. Sources of
supplemental iron among breastfed infants during the first year of life. Pediatrics.
2008;122(suppl 2):S98-S104. DOI: 10.1542/peds.2008-1315m. Available at:
http://pediatrics.aappublications.org/cgi/content/full/122/Supplement_2/S98

Kleinman RE. Iron. In: Pediatric Nutrition Handbook. 6th ed. Elk Grove Village, Ill: American
Academy of Pediatrics; 2009:403-422

Copyright 2010 by the American Academy of Pediatrics page 61


2011 PREP SA on CD-ROM

Wagner CL, Greer FR, and the Section on Breastfeeding and Committee on Nutrition. Prevention
of rickets and vitamin D deficiency in infants, children, and adolescents. Pediatrics.
2008;122:1142-1152. DOI: 10.1542/peds.2008-1862. Available at:
http://pediatrics.aappublications.org/cgi/content/full/122/5/1142

Copyright 2010 by the American Academy of Pediatrics page 62


2011 PREP SA on CD-ROM

Question: 19

A 5-year-old previously healthy boy has developed a limp and right hip pain over the past week.
There is no history of trauma, fever, rashes, or other systemic symptoms. On physical
examination, he has limited internal rotation and abduction of the right hip; other findings are within
normal parameters. A lateral, anteroposterior, and frog leg radiograph series demonstrates a
crescentic subchondral lucency in the medial aspect of the epiphysis (Item Q19).

Of the following, the MOST likely diagnosis is

A. developmental hip dysplasia

B. Legg-Calv-Perthes disease

C. septic hip

D. slipped capital femoral epiphysis

E. toxic synovitis of the hip

Copyright 2010 by the American Academy of Pediatrics page 63


2011 PREP SA on CD-ROM

Question: 19

(Reprinted with permission from Gough-Palmer A, McHugh K. Investigating hip pain in a well child.
BMJ. 2007;334:1216-1217)
Lateral hip radiograph showing a subchondral lucency (arrow), as described for the boy in the
vignette.

Copyright 2010 by the American Academy of Pediatrics page 64


2011 PREP SA on CD-ROM

Critique: 19 Preferred Response: B

The differential diagnosis of limp and hip pain in children is extensive and varies with age
and findings on history, physical examination, and radiographic evaluation. Legg-Calv-Perthes
disease should be in the differential diagnosis of hip pain for any prepubertal child between 3
and 10 years of age. The prevalence of the condition is 0.2 to 29.0 per 100,000 children younger
than 14 years, with the incidence peaking between 5 and 7 years. Boys account for about 80%
of cases, although the severity may be as bad or worse for girls. Typically, the child has an
insidious onset of leg pain or limp not associated with fever. Radiographs may appear normal or
show subtle abnormalities early in the course of the disease. They subsequently become
diagnostic and progress through four phases: 1) narrowing and sclerosis of the femoral
epiphysis, 2) fragmentation of the femoral head, 3) reossification, and 4) healing. The crescent
sign seen on lateral radiographs appears as subchondral lucency, as described for the boy in
the vignette, and represents a fracture of the femoral epiphysis (Item C19A). This finding
frequently can be seen when the child initially becomes symptomatic.
Legg-Calv-Perthes disease represents avascular necrosis of the capital femoral epiphysis
of uncertain cause. Passive exposure to smoking both prenatally and during childhood has been
associated in some studies, possibly by affecting vascular development. Thrombophilia
associated with abnormalities of protein C and protein S also has been implicated.
Slipped capital femoral epiphysis also often presents with limp and hip and knee pain and
affects boys more often than girls, but it occurs in children between 10 and 16 years of age.
The classic presentation is in obese teens. The typical radiographic finding is failure of a line
drawn along the superior femoral neck (Klein line) to intersect the femoral head (Item C19B).
Septic hip is the most urgent condition in the differential diagnosis of childhood hip pain.
Classically, the patient appears ill, refuses to bear weight, has fever, and has a tender hip joint
with extremely limited range of motion. Radiographs show widening of the joint space because
of purulent effusion (Item C19C).
Differentiating toxic (transient) synovitis from Legg-Calv-Perthes disease may be difficult
because toxic synovitis also most frequently affects 3- to 8-year-old children. The onset is
sudden, and the patient is afebrile or has a low-grade fever. He or she may limp or refuse to
bear weight and often has limited internal rotation at the hip. Radiographs may appear normal or
show joint effusion; the femoral epiphysis appears normal.
Developmental dysplasia of the hip has a chronic course and usually is diagnosed in the
first few postnatal months. Girls are affected more commonly than boys. If the diagnosis is
missed during the first postnatal year, the child may have few symptoms until the onset of early
degenerative hip arthritis. Radiographs show inadequate acetabular development (Item C19D).

Suggested reading:

Bahmanyar S, Montgomery SM, Rudiger JW, Ekbom A. Maternal smoking during pregnancy, other
prenatal and perinatal factors, and the risk of Legg-Calv-Perthes disease. Pediatrics.
2008;122:e459-e464. DOI: 10.1542/peds.2008-0307. Available at:
http://pediatrics.aappublications.org/cgi/content/full/122/2/e459

Copyright 2010 by the American Academy of Pediatrics page 65


2011 PREP SA on CD-ROM

Eldridge J, Dilley A, Austin H, et al. The role of protein C, protein S, and resistance to activated
protein C in Legg-Perthes disease. Pediatrics. 2001;107:1329-1334. Available at:
http://pediatrics.aappublications.org/cgi/content/full/107/6/1329

Frick SL. Evaluation of the child who has hip pain. Orthop Clin North Am. 2006;37:133-140. DOI:
10.1016/j.ocl.2005.12.003. Abstract available at: http://www.ncbi.nlm.nih.gov/pubmed/16638444

Goldberg MJ. Early detection of developmental hip dysplasia: synopsis of the AAP clinical
practice guideline. Pediatr Rev. 2001;22:131-134. DOI: 10.1542/10.1542/pir.22-4-131. Available
at: http://pedsinreview.aappublications.org/cgi/content/full/22/4/131

Gough-Palmer A, McHugh K. Investigating hip pain in a well child. BMJ. 2007;334:1216-1217.


DOI: 10.1136/bmj.39188.515741.47.

Lindsley CB, Asher MA. Legg-Calv-Perthes and other hip diseases in children. In: Hochberg MC,
Silman AJ, Smolen JS, Weinblatt ME, Weisman MH, eds. Rheumatology. 4th ed. Philadelphia, Pa:
Mosby Elsevier; 2007:995-1002

Sawyer JR, Kapoor M. The limping child: a systematic approach to diagnosis. Am Fam
Physician. 2009;79:215-224. Abstract available at:
http://www.ncbi.nlm.nih.gov/pubmed/19202969

Tse SML, Laxer RM. Approach to acute limb pain in childhood. Pediatr Rev. 2006;27:170-180.
DOI: 10.1542/10.1542/pir.27-5-170. Available at:
http://pedsinreview.aappublications.org/cgi/content/full/27/5/170

Copyright 2010 by the American Academy of Pediatrics page 66


2011 PREP SA on CD-ROM

Critique: 19

(Reprinted with permission from Gough-Palmer A, McHugh K. Investigating hip pain in a well child.
BMJ. 2007;334:1216-1217)
Lateral hip radiograph showing the classic subchondral lucency of early avascular necrosis
(arrow), not visible on the initial film, indicating early Legg-Calv-Perthes disease.

Copyright 2010 by the American Academy of Pediatrics page 67


2011 PREP SA on CD-ROM

Critique: 19

(Courtesy of D Krowchuk)
Slipped capital femoral epiphysis is characterized by an upward anterior movement of the femoral
neck on the capital epiphysis, which becomes displaced posteriorly and inferiorly. In the normal
hip (right), a line along the superior margin of the femoral neck transects a portion of the ossified
epiphysis. This does not occur on the affected side (left).

Copyright 2010 by the American Academy of Pediatrics page 68


2011 PREP SA on CD-ROM

Critique: 19

(Courtesy of C Barnes)
Increased soft-tissue density and swelling around the left hip with widening of the joint space
caused by a septic hip. There is destruction of the medial proximal femur due to coexisting
osteomyelitis.

Copyright 2010 by the American Academy of Pediatrics page 69


2011 PREP SA on CD-ROM

Critique: 19

(Courtesy of D Krowchuk)
Anteroposterior radiograph of the hip in a 12-month-old who has delayed recognition of
developmental dysplasia of the hip: The right acetabulum is shallow and steep in orientation. The
hip is dislocated superolaterally, and the ossification center is smaller than the left.

Copyright 2010 by the American Academy of Pediatrics page 70


2011 PREP SA on CD-ROM

Question: 20

The parents of a 2-month-infant who you are seeing for a health supervision visit relate that a red
mass on the infants eyelid has grown and no longer is the flat birthmark seen in the newborn
nursery. Physical examination reveals a hemangioma located on the upper lid (Item Q20). The
visual axis is not obstructed, but the infant is unable to open her eyelid completely. The remainder
of the examination findings are normal, with no evidence of other hemangiomas.

Of the following, the MOST important reason for urgent pediatric ophthalmologic referral for this
infant is

A. concern for Kasabach-Merritt syndrome

B. need for urgent intralesional injection of corticosteroids

C. need for urgent surgical excision

D. prevention of amblyopia

E. prevention of blindness in the contralateral eye

Copyright 2010 by the American Academy of Pediatrics page 71


2011 PREP SA on CD-ROM

Question: 20

(Courtesy of D Krowchuk)
Hemangioma, as described for the infant in the vignette.

Copyright 2010 by the American Academy of Pediatrics page 72


2011 PREP SA on CD-ROM

Critique: 20 Preferred Response: D

The infant described in the vignette is at risk for amblyopia or diminished visual acuity
because of potential obstruction of vision by a hemangioma. Thus, urgent ophthalmologic referral
is indicated. Amblyopia is the consequence of diminished visual input into the visual cortex and
consequent impairment of diminished neuro-ophthalmologic pathways.
The causes of amblyopia include intrinsic and extrinsic diseases that impair passage of light
through the pupil and into the visual pathways. Strabismus is the most common cause of
amblyopia and results from imbalanced extraocular muscle strength. Strabismus may be
detected by routine visual examination by the "cover test" and "corneal light reflex" test.
Tumors of the eye and face, which diminish the amount of light and clear visual signals that
enter through the pupil also may cause amblyopia. Thus, hemangiomas and other tumors of the
eyelids or face that may grow in size and obstruct or impede vision must be managed urgently.
Other causes of amblyopia include cataracts and orbital tumors. An abnormally white pupil
(leukocoria) or mass seen on retinal examination are clues to these conditions. Congenital ptosis
may also cause amblyopia.
Kasabach-Merritt syndrome is a platelet consumption disorder that is associated with large
or multiple hemangiomas. Although bleeding and ulceration are potential complications of any
hemangioma, these complications are more likely to occur if the hemangioma is large and in a
location that is likely to be injured. If ophthalmologic evaluation reveals evidence of amblyopia,
laser therapy or intralesional corticosteroids may be used to reduce the size of the hemangioma.
Surgical excision is contraindicated due to bleeding, scarring, and regrowth of the hemangioma.
Although diminished input from either eye may affect neuro-ophthalmic tracts and visual cortex
development, diminished visual input would not cause blindness in the contralateral eye.

Suggested reading:

Schwartz SR, Blei F, Ceisler E, Steele M, Furlan L, Kodsi S. Risk factors for amblyopia in children
with capillary hemangiomas of the eyelids and orbit. J AAPOS. 2006;10:262-268. Abstract
available at: http://www.ncbi.nlm.nih.gov/pubmed/16814181

Schwartz SR, Kodsi SR, Blei F, Ceisler E, Steele M, Furlan L. Treatment of capillary hemangiomas
causing refractive and occlusional amblyopia. JAAPOS. 2007;11:577-583. DOI:
10.1016/j.jaapos.2007.04.022. Abstract available at:
http://www.ncbi.nlm.nih.gov/pubmed/17720571

Tingley DH. Vision screening essentials: screening today for eye disorders in the pediatric
patient. Pediatr Rev. 2007;28:54-61. doi:10.1542/10.1542/pir.28-2-54. Available at:
http://pedsinreview.aappublications.org/cgi/content/full/28/2/54

Waner M, North PE, Scherer KA, Frieden IJ, Waner A, Mihm MC Jr. The nonrandom distribution of
facial hemangiomas. Arch Dermatol. 2003;139:869875. Available at: http://archderm.ama-
assn.org/cgi/content/full/139/7/869

Copyright 2010 by the American Academy of Pediatrics page 73


2011 PREP SA on CD-ROM

Question: 21

During your examination of a 7-year-old boy at his health supervision visit, conducted with a
pediatric resident, you determine that his weight is greater than the 97th percentile for age. His
mother is obese, his father has type 2 diabetes mellitus, and one grandfather died of a myocardial
infarction at 51 years of age. You counsel the family about improvements they can make in the
boys diet and level of exercise.

Of the following, you are MOST likely to advise the resident that this childs risk of developing
metabolic syndrome

A. can be predicted by a determination of hemoglobin A1c values

B. is close to that of the general population because there is no family history of hyperlipidemia or
systemic hypertension

C. is reduced if he begins to develop a healthy lifestyle as a child

D. is the same as the general population if cholesterol-lowering agents are started, even without
lifestyle changes

E. is the same as the general population if his fasting lipid profile is currently normal

Copyright 2010 by the American Academy of Pediatrics page 74


2011 PREP SA on CD-ROM

Critique: 21 Preferred Response: C

The findings on physical examination combined with the family history for the boy described
in the vignette suggest that he is at risk of metabolic syndrome, a combination of medical
disorders that increase the risk of developing cardiovascular disease and diabetes. Metabolic
syndrome affects one in five people, the prevalence increases with age, and some studies
estimate the prevalence in the United States to be up to 25% of the population. Metabolic
syndrome also is known as metabolic syndrome X, syndrome X, and insulin resistance
syndrome. The term "metabolic syndrome" dates back to at least the late 1950s but came into
common usage in the late 1970s to describe various associations of risk factors with diabetes.
The term "metabolic syndrome" for associations of obesity, diabetes mellitus,
hyperlipoproteinemia, and hyperuricemia describes the additive effects of risk factors on
atherosclerosis. The terms "metabolic syndrome," "insulin resistance syndrome," and "syndrome
X" now are used specifically to define a constellation of abnormalities that is associated with
increased risk for the development of type 2 diabetes and atherosclerotic vascular disease (eg,
heart disease and stroke).
Very little is known about the development of metabolic syndrome in children, and the term is
not used in pediatrics. However, clinicians are becoming increasingly cognizant of the risk
factors in the pediatric population, which include obesity, family predisposition to early
cardiovascular disease, systemic hypertension, type 2 diabetes, and an unhealthy dietary and
exercise-related lifestyle. Criteria have been determined for treating childhood hyperlipidemia,
with the first line of therapy being diet modification and exercise programs. Adoption of such
lifestyle changes in childhood can reduce the risk of developing metabolic syndrome. Cholesterol-
lowering agents never are used in the absence of concomitant recommendations for institution
of lifestyle changes. Although an elevated hemoglobin A1c value does predict diabetes, data are
insufficient in the pediatric population to make predictions regarding the use of this value alone to
predict risk for the eventual development of the metabolic syndrome. The same holds for fasting
lipid profiles: an abnormal panel predicts the development of hyperlipidemia during adulthood but
does not predict the development of the metabolic syndrome. A normal fasting lipid profile does
not reduce this risk. The risk for the development of metabolic syndrome does not require the
presence of all components of the definition. The absence of several risk factors (ie, family
history of hyperlipidemia/hypertension) does not reduce this childs risk to that of the normal
population because of the presence of other risk factors.
The exact mechanisms of the complex pathways of metabolic syndrome are not yet
completely known. Most patients are older, obese, sedentary, and have a degree of insulin
resistance. Stress also can be a contributing factor. The most important factors are: obesity,
genetic predisposition, aging, and sedentary lifestyle (ie, low physical activity and excess
caloric intake).
There is debate regarding whether obesity or insulin resistance is the cause of the
metabolic syndrome or if they are consequences of a more far-reaching metabolic derangement.
A number of markers of systemic inflammation, including C-reactive protein, often are increased,
as are fibrinogen, interleukin-6, tumor necrosis factor-alpha, and others. Central adiposity is a
key feature of the syndrome. However, despite the importance of obesity, patients who are of
normal weight also may be insulin-resistant and have the syndrome. The metabolic syndrome

Copyright 2010 by the American Academy of Pediatrics page 75


2011 PREP SA on CD-ROM

affects 44% of the United States population older than age 50, and a greater percentage of
women older than age 50 have the syndrome than do men. It is estimated that 75% of patients
who have type 2 diabetes have the metabolic syndrome. With appropriate cardiac rehabilitation
and changes in lifestyle (eg, nutrition, physical activity, weight reduction, and, in some cases,
medications), the prevalence of the syndrome can be reduced.
The International Diabetes Federation consensus worldwide definition of the metabolic
syndrome (2006) includes central obesity (defined by waist circumference), AND any two of
the following:
Elevated triglycerides
Low high-density lipoprotein (HDL) cholesterol
Hypertension
Elevated fasting plasma glucose
Various strategies have been proposed to prevent the development of metabolic syndrome,
including increased physical activity (such as walking 30 minutes every day) and a healthy,
reduced-calorie diet. However, these measures are effective in only a minority of people,
primarily due to a lack of compliance. Drug treatment frequently is required. Diuretics and
angiotensin-converting enzyme inhibitors may be used to treat hypertension. Cholesterol drugs
may be used to lower low-density lipoprotein cholesterol and triglyceride concentrations, if they
are elevated, and to raise HDL concentrations, if they are low. Use of drugs that decrease
insulin resistance such as metformin is controversial; this treatment is not approved by the
United States Food and Drug Administration. Cardiovascular exercise has been shown to be
therapeutic in approximately 30% of cases. The most probable benefit is reduction in triglyceride
concentrations, but fasting plasma glucose and insulin resistance in most patients did not
improve.

Suggested reading:

Daniels SR, Greer FR; Committee on Nutrition. Lipid screening and cardiovascular health in
childhood. Pediatrics. 2008;122:198-208. DOI: 10.1542/peds.2008-1349. Available at:
http://pediatrics.aappublications.org/cgi/content/full/122/1/198

Ford ES, Giles WH, Dietz WH. Prevalence of metabolic syndrome among US adults: findings from
the Third National Health and Nutrition Examination Survey. JAMA. 2002;287:356-359. Available
at: http://jama.ama-assn.org/cgi/content/full/287/3/356

Katzmaryk PT, Leon AS, Wilmore JH, et al. Targeting the metabolic syndrome with exercise:
evidence from the HERITAGE Family Study. Med Sci Sports Exerc. 2003;35:17031709.
Abstract available at: http://www.ncbi.nlm.nih.gov/pubmed/14523308

Copyright 2010 by the American Academy of Pediatrics page 76


2011 PREP SA on CD-ROM

Question: 22

A 14-year-old boy who was diagnosed with migraines at age 11 presents to the emergency
department with a severe migraine. For the past 2 months, he has had two to three such
headaches per week. His mother asks about using stronger pain medications. You are concerned
about the possible complications of medication overuse.

Of the following, the class of abortive medications that is MOST likely to induce chronic
headaches is

A. caffeine-containing medications

B. isometheptene compounds

C. nonsteroidal anti-inflammatory drugs

D. opiates

E. triptans

Copyright 2010 by the American Academy of Pediatrics page 77


2011 PREP SA on CD-ROM

Critique: 22 Preferred Response: D

Opiates and barbiturates are more likely than caffeine-containing medications,


isometheptene compounds, nonsteroidal anti-inflammatory drugs (NSAIDs), or triptans to cause
chronic headaches due to overuse of medications. Medication overuse headache is the third
most common type of chronic headache after migraine and tension headache. Most often, it
results from NSAID use because NSAIDs are the most widely used agents for headaches.
However, the probability of medication overuse headache is higher with opiates.
Medication withdrawal headache is a huge public health problem and a factor involved in
chronic daily headache for many adolescents. The only effective treatment is withdrawal of the
medication. Although less common than with opiates, medication overuse headache may occur
with over-the-counter analgesics, triptans, ergotamines, and combination analgesics with
caffeine.
Other well-known complications of medications used to treat headache include constipation
with opiates, tolerance with benzodiazepines, and gastrointestinal distress with NSAIDs.

As a result of reviewing this information, do you intend to make a change in practice


to provide better patient care?
Yes No

Suggested reading:

Damen L, Bruijn JKJ, Verhagen AP, Berger MY, Passchier J, Koes BW. Symptomatic treatment of
migraine in children: a systematic review of medication trials. Pediatrics. 2005;116:e295-e302.
DOI: 10.1542/peds.2004-2742. Available at:
http://pediatrics.aappublications.org/cgi/content/full/116/2/e295

Gunner KB, Smith HD. Practice guideline for diagnosis and management of migraine headaches
in children and adolescents: part one. J Pediatr Health Care. 2007;21:327-332. DOI:
10.1016/j.pedhc.2007.06.004

Lewis DW. Pediatric migraine. Pediatr Rev. 2007;28:43-53. DOI: 10.1542/10.1542/pir.28-2-43.


Available at: http://pedsinreview.aappublications.org/cgi/content/full/28/2/43

Copyright 2010 by the American Academy of Pediatrics page 78


2011 PREP SA on CD-ROM

Question: 23

A 16-year-old girl who is new to your practice comes to the clinic for a physical examination prior
to enrollment in a summer volleyball camp. She is generally healthy, and she does well
academically. On physical examination, you note that she is unusually tall and slender, and she
appears to have long fingers (Item Q23) and toes. You are concerned that she could have Marfan
syndrome, and you refer her for a clinical genetics evaluation.

Of the following, the additional finding that would MOST strongly suggest the diagnosis of Marfan
syndrome for this girl is

A. high myopia

B. long, narrow face

C. mitral valve prolapse

D. narrow palatal contour

E. spontaneous pneumothorax

Copyright 2010 by the American Academy of Pediatrics page 79


2011 PREP SA on CD-ROM

Question: 23

(Courtesy of M Rimsza)
Long fingers, as described for the girl in the vignette.

Copyright 2010 by the American Academy of Pediatrics page 80


2011 PREP SA on CD-ROM

Critique: 23 Preferred Response: E

Marfan syndrome (MS) is an autosomal dominant connective tissue disorder that has a
prevalence of 1 in 10,000 and usually is caused by alterations in the fibrillin 1 (FBN1) gene. MS
primarily involves the skeletal, cardiovascular, and ocular systems. Skeletal features include
pectus carinatum, pectus excavatum, reduced upper-to-lower segment ratio, scoliosis of
greater than 20 degrees or spondylolisthesis, reduced elbow extension, joint hypermobility, and
others. Major cardiovascular features are dilation or dissection of the ascending aorta, and
minor features include mitral valve prolapse, dilation of the main pulmonary artery, and
calcification of the mitral annulus. Ocular features include ectopia lentis, flat cornea, hypoplastic
iris, and increased axial length of the globe. Although high myopia; long, narrow face; mitral
valve prolapse; and narrow palatal contour all are associated with MS, they are also relatively
common findings in other syndromes and in the general population. Spontaneous pneumothorax
occurring in a teenager, however, is unusual and is one of the minor criteria for the diagnosis of
MS. Therefore, the history of spontaneous pneumothorax associated with this girls physical
features should increase the pediatricians suspicion for MS.
Due to the risks for aortic root dilatation and dissection associated with MS, affected
individuals are asked not to participate in contact or competitive sports or isometric exercise. In
addition, they should avoid activities placing them at increased risk for joint injury or pain.

Suggested reading:

De Paepe A, Devereaux RB, Dietz HC, Hennekam RCM, Pyeritz RE. Revised diagnostic criteria for
the Marfan syndrome. Am J Med Genet. 1996;62:417-426. Abstract available at:
http://www.ncbi.nlm.nih.gov/pubmed/8723076

Dietz HC. Marfan syndrome. GeneReviews. 2009. Available at:


http://www.ncbi.nlm.nih.gov/bookshelf/br.fcgi?book=gene&part=marfan

Judge DP, Dietz HC. Marfans syndrome. Lancet. 2005;366:1965-1976. DOI: 10.1016/S0140-
6736(05)67789-6. Abstract available at: http://www.ncbi.nlm.nih.gov/pubmed/16325700

Shirley ED, Sponsellar PD. Marfan syndrome. J Am Acad Orthoped Surg. 2009;9:572-581.
Abstract available at: http://www.ncbi.nlm.nih.gov/pubmed/19726741

Copyright 2010 by the American Academy of Pediatrics page 81


2011 PREP SA on CD-ROM

Question: 24

A 14-year-old girl is referred to your office for further management following an ultrasonographic
report showing a 2-cm left ovarian cyst. She had presented to the emergency department 1
week ago, following 2 days of left lower quadrant pain. She had no associated nausea or
vomiting, vaginal discharge, or dysuria. She is not sexually active, and her pregnancy test result
was negative. She began menstruating about 3 years ago and has regular cycles with mild pain.
Her physical examination today yields normal findings.

Of the following, the MOST appropriate next step is to

A. prescribe oral contraceptive pills for cyst resolution

B. reassure the girl and advise her to return if she has recurrent pain

C. recommend a surgical consultation for cyst removal

D. recommend abdominal magnetic resonance imaging

E. repeat pelvic ultrasonography in 3 months

Copyright 2010 by the American Academy of Pediatrics page 82


2011 PREP SA on CD-ROM

Critique: 24 Preferred Response: B

Ovarian "cysts" that are smaller than 3 cm, as described for the girl in the vignette, usually
are normal follicles and do not require any further intervention or evaluation. The girl can be
reassured and advised to return if she has recurrent pain.
Physiologic (functional) ovarian cysts include follicular, corpus luteum, and theca lutein
cysts. Most often, they are incidental findings on pelvic ultrasonography. They result from
disordered function of the pituitary-ovarian-axis, with increased fluid accumulation within the
preovulatory follicle (follicular cyst) or corpus luteum. Theca lutein cysts are caused by
pregnancy-related human chorionic gonadotropin production, usually are asymptomatic, and do
not require surgical intervention.
Follicular cysts are the most common and can vary in size from 3 to 15 cm. In adolescents,
they typically resolve spontaneously in 4 to 8 weeks. They may be followed ultrasonographically
at monthly intervals. Combined oral contraceptives do not speed resolution, although they may
prevent the development of new cysts and make ultrasonographic follow-up evaluations easier
to interpret. Adolescents using progestin-only contraceptives (such as depot
medroxyprogesterone acetate) may have slower follicular atresia and cyst resolution than
adolescents using combined oral contraceptives. If a fluid-filled cyst is enlarging or causing
symptoms (pelvic heaviness, urinary frequency, constipation) or undergoes torsion, with the
development of sudden acute pain and nausea, vomiting, low-grade fever, and pallor, surgical
removal of the cyst is recommended.
Corpus luteum cysts also often are asymptomatic but may undergo bleeding into the cyst or
rupture with intraperitoneal hemorrhage. Bleeding into the cyst increases cyst weight and size
and, thus, the risk for torsion. If ultrasonography reveals a complex mass rather than a simple
cyst, further evaluation by magnetic resonance imaging may be indicated for possible ovarian
tumors/malignancies.

As a result of reviewing this information, do you intend to make a change in practice


to provide better patient care?
Yes No

Suggested reading:

Hillard PJA. Pelvic masses. In: Neinstein LS, Gordon CM, Katzman DK, Rosen DS, Woods ER,
eds. Adolescent Health Care A Practical Guide. 5th ed. Philadelphia, Pa: Lippincott Williams &
Wilkins, a Wolters Kluwer business; 2008:706-713

Laufer MR, Goldstein DP. Benign and malignant ovarian masses. In: Emans SJH, Laufer MR,
Goldstein DP, eds. Pediatric and Adolescent Gynecology. 5th ed. Philadelphia, Pa: Lippincott
Williams & Wilkins; 2005:685-728

Paul RI. Ovarian torsion: salvage possible despite prolonged symptoms. AAP Grand Rounds.
2005;14:32-33. Available at: http://aapgrandrounds.aappublications.org/cgi/content/full/14/3/32

Copyright 2010 by the American Academy of Pediatrics page 83


2011 PREP SA on CD-ROM

Question: 25

You are treating a 20-kg, 4-year-old boy who was admitted to the pediatric intensive care unit 3
days ago after being endotracheally intubated and resuscitated in the emergency department for
septic shock. Over the past 2 days, he has required increasing ventilator settings, progressive
opacification of his lung fields has been apparent on chest radiography (Item Q25), and he now
fulfills the clinical criteria for acute respiratory distress syndrome (ARDS). His current ventilator
settings are a respiratory rate of 25 breaths/min, tidal volume of 300 mL, positive end-expiratory
pressure (PEEP) of 12 mm Hg with an FiO2 of 0.6, and a resultant oxygen saturation (SpO2) of
90%. His most recent arterial blood gas assessment shows a pH of 7.30, a PacO2 of 50 mm Hg,
and a PaO2 of 65 mm Hg.

Of the following, the MOST effective treatment strategy to decrease ARDS-associated mortality
risk in this patient is

A. adjustment of the respiratory tidal volume to 6 mL/kg

B. administration of corticosteroids

C. administration of surfactant

D. decrease of PEEP to 5 mm Hg

E. increase in oxygen to ensure an SpO2 greater than 95%

Copyright 2010 by the American Academy of Pediatrics page 84


2011 PREP SA on CD-ROM

Question: 25

(Courtesy of B Poss)
Chest radiograph of the child described in the vignette. Acute respiratory distress syndrome is
characterized by diffuse bilateral pulmonary infiltrates and the absence of pulmonary edema.

Copyright 2010 by the American Academy of Pediatrics page 85


2011 PREP SA on CD-ROM

Critique: 25 Preferred Response: A

Acute respiratory failure remains a significant cause of morbidity and mortality in


hospitalized pediatric patients. The most severe forms of acute respiratory failure are acute lung
injury (ALI) and acute respiratory distress syndrome (ARDS), which are defined by: acute
onset, severe arterial hypoxemia (Pao2/Fio2 ratio <200 mm Hg for ARDS and Pao2/Fio2 ratio
<300 mm Hg for ALI), bilateral infiltrates on chest radiography, and absence of left atrial
hypertension. Risk factors for the development of ARDS include sepsis, trauma, aspiration,
pneumonia, near-drowning, and multiorgan dysfunction. ALI/ARDS affects an estimated 2,500 to
9,000 children in the United States annually, with 500 to 2,000 deaths.
Various treatment strategies have been studied for adult patients who have ARDS, and
results have been extrapolated to the pediatric population. The use of low tidal volumes (4 to 6
mL/kg) has been the only approach demonstrated to decrease mortality in several adult studies.
The improvement in mortality is believed to be due to the avoidance of high tidal volume
strategies, which have been shown to produce secondary lung injury. In addition, goals of
respiratory support include a Pao2 of 60 to 80 mm Hg, Spo2 of 90% or greater, and pH of 7.30 to
7.45. Therefore, decreasing the PEEP (with subsequent loss of lung volume and increased
oxygen requirement) or increasing the oxygen to ensure an Spo2 greater than 95% are not
indicated. Early use of corticosteroids in adult patients who have ARDS has shown conflicting
results and is not recommended routinely. Surfactant has been shown to improve oxygenation,
but it has not been demonstrated to have an effect on long-term outcome.

Suggested reading:

Frankel LR. Respiratory distress and failure. In: Kliegman RM, Behrman RE, Jenson HB, Stanton
BF, eds. Nelson Textbook of Pediatrics. 18th ed. Philadelphia, Pa: Saunders Elsevier; 2007:421-
423

Randolph AG. Management of acute lung injury and acute respiratory distress syndrome in
children. Crit Care Med. 2009;37:2448-2454. DOI: 10.1097/CCM.0b013e3181aee5dd. Abstract
available at: http://www.ncbi.nlm.nih.gov/pubmed/19531940

Vish M, Shanley TP. Acute lung injury and acute respiratory distress syndrome. In: Wheeler DS,
Wong HR, Shanley TP, eds. Pediatric Critical Care Medicine: Basic Science and Clinical
Evidence. New York, NY: Springer-Verlag; 2007:395-411

Copyright 2010 by the American Academy of Pediatrics page 86


2011 PREP SA on CD-ROM

Question: 26

A 15-year-old obese boy (body mass index of 32 kg/M2) is brought to the emergency department
by ambulance. He was home alone over the weekend and was discovered semicomatose and
poorly responsive when his parents returned. They state that he had been urinating and drinking
frequently for a few days. The mother, who is also obese, had gestational diabetes, and the
father "takes pills for his sugar." Laboratory studies demonstrate:

Plasma glucose, 1,000 mg/dL (55.5 mmol/L)


Sodium, 132 mEq/L (132 mmol/L)
Potassium, 3.8 mEq/L (3.8 mmol/L)
Chloride, 95 mEq/L (95 mmol/L)
Bicarbonate, 21 mEq/L (21 mmol/L)

Drug screening studies yield negative results. Ketones in the urine are trace positive.

Of the following, a TRUE statement about treatment for this boy is that

A. adding insulin to treatment only after the first 6 hours of rehydration prevents symptomatic
cerebral edema

B. permitting the blood glucose to decrease no more than 75 mg/dL (4.2 mmol/L) per hour
prevents symptomatic cerebral edema

C. rehydrating with intravenous 0.9% saline, adding glucose only when blood glucose decreases
to less than 300 mg/dL (16.7 mmol/L), prevents symptomatic cerebral edema

D. replacing fluid and electrolyte losses from dehydration over 36 hours prevents cerebral edema

E. the possibility of symptomatic cerebral edema can be reduced but not prevented entirely

Copyright 2010 by the American Academy of Pediatrics page 87


2011 PREP SA on CD-ROM

Critique: 26 Preferred Response: E

The physical and laboratory findings combined with the family history described for the boy
in the vignette suggest that he is experiencing a hyperosmolar nonketotic coma (HNKC). Cerebral
edema has been reported anecdotally in adolescents who have HNKC. The cause of this
condition is poorly understood, and appropriate preventive treatment is unclear. Cerebral edema
is a known complication of the treatment of diabetic ketoacidosis and diabetic coma. The more
severe and prolonged the acidosis, the greater the likelihood of this complication.
Presently, taking guidance from the treatment of diabetic ketoacidosis, it is generally
considered prudent to administer insulin judiciously to patients who have HNKC to assess insulin
sensitivity, which may be variable, and to rehydrate the patient cautiously with a solution
containing sodium ranging from 0.45% to 0.9% to correct dehydration. Some have suggested
that allowing the blood glucose to decrease during rehydration and only starting insulin after 4 to
6 hours have passed is appropriate management, but there is no evidence that this will prevent
the rare cases of cerebral edema complicating HNKC. The recommendation for adults who have
HNKC is rehydration with dilute solutions more rapidly than over 36 hours because of concern
about vascular and cardiac complications of dehydration and the infrequency of cerebral
edema. At the time of this writing, no data suggest that this is reasonable in young people who
have HNKC. Therefore, the general approach is to treat HNKC like diabetic ketoacidosis but to
remain constantly alert to the possibility of altered consciousness or changes in blood pressure
or pulse that might suggest the development of cerebral edema so that treatment can commence
before there is severe damage from brain swelling. In addition, severe dehydration might be
considered an indication for more rapid rehydration if a patient is in shock or clearly exceedingly
dehydrated. This may be hard to discern clinically because of the hyperosmolar state. The
options offered are reasonable approaches to management, but there is no evidence to suggest
that they will reduce cerebral edema.

Suggested reading:

Glaser N. Cerebral injury and cerebral edema in children with diabetic ketoacidosis: could
cerebral ischemia and reperfusion injury be involved? Pediatr Diabetes. 2009;10:534-541. DOI:
10.1111/j.1399-5448.2009.00511.x

Glaser NS, Marcin JP, Wootton-Gorges SL, et al. Correlation of clinical and biochemical findings
with diabetic ketoacidosis-related cerebral edema in children using magnetic resonance diffusion-
weighted imaging. J Pediatr. 2008;153:541-546. DOI: 10.1016/j.jpeds.2008.04.048. Abstract
available at: http://www.ncbi.nlm.nih.gov/pubmed/18589447

Jeha GS, Haymond MW. Cerebral edema in children with diabetic ketoacidosis. UpToDate Online
17.3. 2009. Available at:
http://www.utdol.com/online/content/topic.do?topicKey=pediendo/21063&selectedTitle=3%7E150
&source=search_result819

Kitabchi A, Rose BD. Treatment of diabetic ketoacidosis and hyperglycemic hyperosmolar state

Copyright 2010 by the American Academy of Pediatrics page 88


2011 PREP SA on CD-ROM

in adults. UpToDate Online 17.3. 2009. Available at:


http://www.utdol.com/online/content/topic.do?topicKey=diabetes/25334&selectedTitle=1%7E150
&source=search_result819

Rosenbloom AL. Hyperglycemic hyperosmolar state: an emerging pediatric problem. J Pediatr.


2010;156:180-184. DOI: 10.1016/j.jpeds.2009.11.057

Wolfsdorf J, Craig ME, Daneman D, et al. Diabetic ketoacidosis in children and adolescents with
diabetes. Pediatr Diabetes. 2009;10(suppl 12):118-133

Yuen N, Anderson SE, Glaser N, Tancredi DJ, O'Donnell ME. Cerebral blood flow and cerebral
edema in rats with diabetic ketoacidosis. Diabetes. 2008;57:2588-2594. Available at:
http://diabetes.diabetesjournals.org/content/57/10/2588.long

Copyright 2010 by the American Academy of Pediatrics page 89


2011 PREP SA on CD-ROM

Question: 27

A 9-year-old boy comes to your office for a health supervision visit. He has difficulty seeing large
print and recently began learning Braille in school. As an infant, he had severe retinopathy of
prematurity, and he has had long-standing difficulty seeing, even with corrective lenses. His
parents have been told that their son is considered legally blind. They ask what this designation
means.

Of the following, the BEST response is that legal blindness means

A. ability to detect only the direction of a light source

B. corrected vision of 20/70 or worse in the best eye

C. corrected vision of 20/200 or worse in the best eye

D. no residual visual function in either eye

E. uncorrected vision of 20/100 or worse in the best eye

Copyright 2010 by the American Academy of Pediatrics page 90


2011 PREP SA on CD-ROM

Critique: 27 Preferred Response: C

The child described in the vignette is legally blind, defined as a corrected vision of 20/200 or
worse in the best eye. Approximately 75% of legally blind individuals have some residual visual
ability. Students may be classified as educationally visually impaired if their corrected vision is
20/70 or worse in the best eye.
Children who have visual impairment need to learn in specialized environments to motivate
them to move and improve their abilities to interpret sensory information accurately. Young
infants should be encouraged to be mobile and orient themselves to the environment using
touch, smell, and auditory input. Beginning in preschool, the goal is to teach skills that promote
independence in the performance of daily activities. Depending on the degree of any associated
impairments, the child may attend a regular school program with input from a teacher of the
visually handicapped. The educational program should emphasize sensory experiences and
auditory programs. The school-age child should have an Individualized Educational Plan. Again,
depending on any associated impairments, the child may participate in regular community-based
classroom activities, focusing on reading, writing, travel needs, and eventually vocational
training and independent living. Braille is used for nonvisual communication and audio recordings
supplement reading.

Suggested reading:

American Foundation for the Blind web site. Available at: http://www.afb.org

Msall ME. Visual impairment. In: Parker S, Zuckerman B, Augustyn M, eds. Developmental and
Behavioral Pediatrics: A Handbook for Primary Care. 2nd ed. Philadelphia, Pa: Lipppincott
Williams & Wilkins; 2005:366-369

National Library Service for the Blind and Physically Handicapped web site. Available at:
http://www.loc.gov/nls/

Copyright 2010 by the American Academy of Pediatrics page 91


2011 PREP SA on CD-ROM

Question: 28

An 18-year-old college student reports to the student health service with a 2-day history of
swelling on the right side of his face, tactile fever, malaise, and headache. Physical examination
reveals swelling above the angle of the jaw and anterior to the ear on the right (Item Q28).

Of the following, the MOST likely cause of the boys illness is infection with

A. human immunodeficiency virus

B. influenza A virus

C. mumps virus

D. Staphylococcus aureus

E. Streptococcus pyogenes

Copyright 2010 by the American Academy of Pediatrics page 92


2011 PREP SA on CD-ROM

Question: 28

(Courtesy of M Rimsza)
Findings, as described for the patient in the vignette.

Copyright 2010 by the American Academy of Pediatrics page 93


2011 PREP SA on CD-ROM

Critique: 28 Preferred Response: C

Acute swelling over the parotid glands (parotitis) accompanied by headache, fever, and
malaise is the classic presentation of infection with mumps virus. With widespread use of
mumps vaccine in infants and young children, the incidence of this disease has decreased
greatly, but occasional outbreaks among college students or other groups of teenagers or young
adults continue to occur. In these settings, even the recommended two doses of vaccine are not
absolutely protective, although attack rates are higher among those who are unvaccinated or
who received only one dose of vaccine. Mumps is generally a self-limited infection, but
complications may include orchitis, meningoencephalitis, pancreatitis, myocarditis, and oophoritis.
Other respiratory viruses (eg, influenza A, parainfluenza, enteroviruses) have been
associated with parotitis, although the illness caused by these agents typically is milder and
frequently involves respiratory symptoms. Parotitis occurs in human immunodeficiency virus-
infected children but is typically chronic, causes fewer symptoms, and is characterized by firm
and nontender swelling of both parotid glands.
Bacterial parotitis presents with high fever, pain in the involved gland, and purulent
discharge from the Stensen duct. Infected children typically appear toxic and have a marked
leukocytosis. Staphylococcus aureus is the most common pathogen. Rarely, mixed bacterial
infection with anaerobes or gram-negative organisms has been found in bacterial parotitis.
Streptococcus pyogenes has not been described in this entity.
Sialolithiasis (or stones in the parotid ducts) is a rare cause of parotid gland enlargement
that occurs most commonly in adults. The pain and swelling of the involved gland is exacerbated
with eating. Sialolithiasis is not associated with systemic disease.
Recurrent idiopathic parotitis is an uncommon condition characterized by recurrent episodes
of parotid inflammation. A single episode may resemble mumps infection, although the child
generally is not systemically ill during the episode. This syndrome occurs most often in children 5
to 15 years of age and cannot be diagnosed with a single episode. Parotitis also rarely is
associated with systemic autoimmune disease such as Sjgren syndrome.

Suggested reading:

American Academy of Pediatrics. Mumps. In: Pickering LK, Baker CJ, Kimberlin DW, Long SS,
eds. Red Book: 2009 Report of the Committee on Infectious Diseases. 28th ed. Elk Grove
Village, Ill: American Academy of Pediatrics; 2009:468-472

Centers for Disease Control and Prevention. Update: multistate outbreak of mumps United
States, January 1 May 2, 2006. MMWR Morb Mortal Wkly Rep. 2006;55:559-563. Available
at: http://www.cdc.gov/mmwr/preview/mmwrhtml/mm5520a4.htm

Templer JW, Liess BD. Parotitis. eMedicine Specialties, Otolaryngology and Facial Plastic
Surgery, Salivary Glands. 2009. Available at: http://emedicine.medscape.com/article/882461-
overview

Copyright 2010 by the American Academy of Pediatrics page 94


2011 PREP SA on CD-ROM

Question: 29

You are examining a 5-year-old boy who was bitten on the hand by a raccoon. The raccoon
cannot be found. You clean the wound with soap and water, but sutures are not required.
According to his medical record, the boy received diphtheria-tetanus-acellular pertussis
immunization 1 year ago.

Of the following, the MOST appropriate next step in management is administration of

A. immune globulin and a tetanus booster

B. immune globulin and rabies vaccine

C. rabies immune globulin and a tetanus booster

D. rabies immune globulin and rabies vaccine

E. rabies vaccine and a tetanus booster

Copyright 2010 by the American Academy of Pediatrics page 95


2011 PREP SA on CD-ROM

Critique: 29 Preferred Response: D

Rabies exists in wild animals throughout the United States, except Hawaii, and most often is
transmitted to humans and domestic animals from infected bats, raccoons, skunks, foxes, and
coyotes. Rabies in small rodents and rabbits is unusual. Rarely, the virus is transmitted by
contamination of mucosa or skin lesions by saliva or neural tissue from the infected animal. The
incubation period of the virus ranges from a few days to years but usually is 4 to 6 weeks.
In cases where prompt testing of the animal for rabies is not possible, such as in the
vignette, rabies immune globulin (RIG) and rabies vaccine for prophylaxis should be administered
as soon as possible, ideally within 24 hours of exposure. Human RIG is the only product
available in the United States for passive immunization. Intravenous immune globulin contains
pooled immunoglobulin G extracted from the plasma of more than 1,000 blood donors and is not
appropriate for rabies postexposure prophylaxis because it does not contain much, if any,
antibody against rabies. Equine RIG is as safe and effective as human RIG and is available
outside of the United States. RIG should be administered within 7 days of exposure and should
be given with the first dose of vaccine to protect against rabies between the time of exposure
and antibody production from vaccine. A dose of 20 IU/kg of RIG should be infiltrated around the
wound(s), with the remainder administered intramuscularly (IM) at a distant site from the
vaccine.
Two rabies vaccines are available for use in the United States: human diploid cell vaccine
(HDCV) and purified chicken embryo cell vaccine (PCECV). Rabies vaccine adsorbed is licensed
but no longer distributed in the United States. Rabies vaccine is administered as a 1.0-mL IM
dose in the deltoid muscle or the anterolateral thigh on the day of exposure or first day of
postexposure prophylaxis (day 0) and is repeated on days 3, 7, and 14 after the initial dose in
immunocompetent individuals. For immunosuppressed individuals, an additional 1.0-mL dose of
vaccine should be provided on day 28 of postexposure prophylaxis. Documentation of
seroconversion 1 to 2 weeks after the completion of prophylaxis is reserved for
immunosuppressed persons.
Adverse reactions to rabies vaccines are more common in adults than children. Local
reactions such as pain, swelling, induration, or erythema occur in 60% to 90% of adults
receiving HDCV and 11% to 57% of adults receiving PCECV. Mild systemic reactions such as
fever, headache, dizziness, and gastrointestinal symptoms have been reported in 0% to 56%.
One study documented mild-to-moderate clinical reactions in only 7% of children receiving
PCECV. All suspected serious systemic, anaphylactic, or neuroparalytic reactions to a rabies
vaccine should be reported to the Vaccine Adverse Event Reporting System at
http://www.vaers.hhs.gov/
All wounds resulting from an animal bite should be flushed and washed with soap and
water. A tetanus booster is recommended for contaminated wounds if 5 years have elapsed
since receipt of a tetanus toxoid-containing vaccine. The 5-year-old boy in the vignette does not
require a tetanus booster because he received a diphtheria-tetanus-acellular pertussis
immunization only 1 year ago.

Suggested reading:

Copyright 2010 by the American Academy of Pediatrics page 96


2011 PREP SA on CD-ROM

American Academy of Pediatrics. Rabies. In: Pickering LK, Baker CJ, Kimberlin DW, Long SS,
eds. Red Book: 2009 Report of the Committee on Infectious Diseases. 28th ed. Elk Grove
Village, Ill: American Academy of Pediatrics; 2009:552-559

Centers for Disease Control and Prevention. Use of a Reduced (4-Dose) Vaccine Schedule for
Postexposure Prophylaxis to Prevent Human Rabies. Atlanta, Ga: Centers for Disease Control
and Prevention; 2010 at:
http://www.cdc.gov/mmwr/preview/mmwrhtml/rr5902a1.htm?s_cid=rr5902a1_e

Mani CS, Murray DL. Rabies. Pediatr Rev. 2006;27:129-136. DOI: 10.1542/10.1542/pir.27-4-129.
Available at: http://pedsinreview.aappublications.org/cgi/content/full/27/4/129

Copyright 2010 by the American Academy of Pediatrics page 97


2011 PREP SA on CD-ROM

Question: 30

A 5-year-old boy presents with fever, cough, and body aches. Physical examination reveals a
temperature of 38.6C, heart rate of 86 beats/minute, respiratory rate of 20 breaths/minute, and
blood pressure of 106/60 mm Hg. He has rhinorrhea and tonsillar erythema. Urinalysis
demonstrates a specific gravity of 1.020, pH of 6, 2+ protein, and no blood.

Of the following, the MOST likely cause for this patients urinary findings is

A. Alport nephritis

B. false-positive effect of urine pH

C. fever

D. minimal change disease

E. poststreptococcal glomerulonephritis

Copyright 2010 by the American Academy of Pediatrics page 98


2011 PREP SA on CD-ROM

Critique: 30 Preferred Response: C

The urine dipstick test is the mainstay for detection of proteinuria, but this technique has
some limitations. The urine dipstick is impregnated with tetrabromophenol blue, which changes
from yellow to greenish-blue when in contact with varying concentrations of albumin. False-
negative results can occur because of the specificity of these sticks for albumin, which means
that low-molecular weight proteins are not detected. Furthermore, because the dipsticks detect
albumin concentrations, "true" albuminuria is missed if the urine sample is dilute. Conversely,
false-positive results for proteinuria occur in the presence of alkaline urine or contaminating
substances such as chlorhexidine antiseptic or radiocontrast (eg, immediately following voiding
cystourethrography).
With regard to interpretation of results, 1+ protein can be normal if the urine is reasonably
concentrated (urine specific gravity >1.015). On the other hand, 2+ protein, as described for the
child in the vignette, always is abnormal. The patients young age and isolated proteinuria make
the likelihood of an underlying nephritis unlikely because most forms of glomerulonephritis
present with hematuria plus proteinuria. Young boys affected by Alport nephritis may have
isolated hematuria initially, but they never have proteinuria without hematuria. Poststreptococcal
glomerulonephritis (PSGN) occurs 7 to 21 days after clinical pharyngitis. Furthermore, PSGN is
characterized by hematuria/proteinuria and varying degrees of azotemia, hypertension, and
edema. Even if the boy in the vignette had a mild or subclinical case of PSGN, he would not have
proteinuria without hematuria and would not show such symptoms at the time of acute
pharyngitis. Children who have minimal change disease present with nephrotic syndrome (3 to
4+ proteinuria and edema). A false-positive test result for proteinuria may occur when the urine
pH is 7.5 or greater. Accordingly, the most likely cause for this boys urinary findings is fever.
Evaluation of proteinuria in the pediatric patient requires the practitioner to determine if the
proteinuria is transient or persistent (fixed). These two varieties can be distinguished by
determining if the proteinuria is orthostatic. Orthostatic proteinuria is rare before 6 years of age
but can be common in adolescence. If suspected, a first morning urine specimen should be
collected for testing. Patients are instructed to empty their bladders before retiring to bed, void
upon awakening into the collection cup, and have these samples tested for proteinuria. Fixed
proteinuria (present on random and first morning samples) warrants an evaluation for a renal
disorder, especially a glomerulopathy.
Transient proteinuria can occur with stress, dehydration, exercise, or fever. Proteinuria
associated with vigorous exercise is mild (usually 2+) and resolves after resting for 48 hours.
Proteinuria associated with fever typically occurs when the temperature is more than 38.3C and
resolves when the fever abates. However, findings of greater than 2+ protein on a dipstick
suggest an alternate diagnosis. The mechanism for proteinuria in the setting of fever is
unknown. Patients who have proteinuria associated with fever, such as the boy in the vignette,
require a follow-up urine sample in 2 to 4 weeks to check for resolution of the proteinuria when
they are afebrile.

Suggested reading:

Bergstein JM. A practical approach to proteinuria. Pediatr Nephrol. 1999;13:697-700. Abstract

Copyright 2010 by the American Academy of Pediatrics page 99


2011 PREP SA on CD-ROM

available at: http://www.ncbi.nlm.nih.gov/pubmed/10502130

Hogg RJ, Portman RJ, Milliner D, Lemley KV, Eddy A, Ingelfinger J. Evaluation and management of
proteinuria and nephrotic syndrome in children: recommendations from a pediatric nephrology
panel established at the National Kidney Foundation Conference on Proteinuria, Albuminuria, Risk,
Assessment, Detection, and Elimination (PARADE). Pediatrics. 2000;105:1242-1249. Available at:
http://pediatrics.aappublications.org/cgi/content/full/105/6/1242

Omokaro SO. Nephrology. In: Robertson J, Shilkofski N, eds. The Harriet Lane Handbook: A
Manual for Pediatric House Officers. 17th ed. Philadelphia, Pa: Elsevier Mosby; 2005:477-506

Copyright 2010 by the American Academy of Pediatrics page 100


2011 PREP SA on CD-ROM

Question: 31

You are seeing a 6-year-old girl who has persistent asthma and rhinitis for a follow-up evaluation
after ordering serum immunoglobulin (Ig)E testing to evaluate for aeroallergens. The only positive
test on the panel was house dust mite, which is consistent with her perennial rhinorrhea and
congestion. You discuss possible lifestyle modifications and treatment options regarding dust
mites.

Of the following, the action that provides the BEST reduction in house dust mite exposure is to

A. encase the pillow, mattress, and box spring

B. maintain the room humidity higher than 70%

C. purchase a high-efficiency particulate air (HEPA) filter for the room

D. remove stuffed animals from the bed

E. vacuum the carpets and upholstery regularly

Copyright 2010 by the American Academy of Pediatrics page 101


2011 PREP SA on CD-ROM

Critique: 31 Preferred Response: A

Dust mites are eight-legged insects of the family Acaridae. Found throughout the world in
regions that have humidity levels consistently above 50%, the major species associated with
atopy (ie, atopic dermatitis, allergic rhinitis, and asthma) are Dermatophagoides pteronyssinus,
Dermatophagoides farinae, and Blomia tropicalis. Because their primary food source is skin
scales, dust mites generally accumulate in the areas in which people spend significant time (eg,
bed, sofa). The major allergen associated with dust mites is contained in the feces and falls
quickly in undisturbed air. Studies measuring Der p 1, a major dust mite allergen, have
consistently found concentrations of 2 to 10 mcg/g of dust to be associated with sensitization
and asthma in infants and children at risk for atopy.
Although the therapeutic choices for the child in the vignette include medication and
immunotherapy, effective avoidance measures can lower dust mite allergen concentrations and
improve clinical symptoms. The most effective avoidance measures include encasing the
mattress, box spring, and pillows in impermeable covers and washing the bedding weekly in hot
water (55 to 60oC). Vacuuming regularly may reduce dust mite concentrations in carpet or
upholstered furniture, but this action does not address the sleeping environment where dust mite
allergen is the most abundant. Removing stuffed animals from the bed is recommended because
they can be a source for allergen accumulation, but such removal is not as effective as
encasing the mattress, box spring, and pillow. Maintaining the humidity above 50%, which often
is seen in homes containing humidifiers or "swamp coolers," encourages dust mite growth.
Many hardware stores sell hygrometers, which measure relative humidity and can aid families
who are actively trying to maintain the humidity below 50%. Finally, although high-efficiency
particulate air (HEPA) filters can reduce airborne concentrations of allergens, dust mite allergen
falls relatively quickly compared with other airborne allergens, making HEPA filters less effective
for this allergen. In addition, HEPA filters do not reduce the concentrations of house dust mite in
areas such as bedding or upholstered furniture.

As a result of reviewing this information, do you intend to make a change in practice


to provide better patient care?
Yes No

Suggested reading:

Gtzsche PC, Johansen HK. House dust mite control measures for asthma. Cochrane Database
Syst Rev. 2008;2:CD001187. DOI: 10.1002/14651858.CD001187.pub3. Available at:
http://www.mrw.interscience.wiley.com/cochrane/clsysrev/articles/CD001187/frame.html

Sheikh A, Hurwitz B, Shehata YA. House dust mite avoidance measures for perennial allergic
rhinitis. Cochrane Database Syst Rev. 2006;4:CD001563. DOI:
10.1002/14651858.CD001563.pub2. Available at:
http://www.mrw.interscience.wiley.com/cochrane/clsysrev/articles/CD001563/frame.html

Wood PR, Hill VL. Practical management of asthma. Pediatr Rev. 2009;30:375-385. DOI:

Copyright 2010 by the American Academy of Pediatrics page 102


2011 PREP SA on CD-ROM

10.1542/10.1542/pir.30-10-375. Available at:


http://pedsinreview.aappublications.org/cgi/content/full/30/10/375

Copyright 2010 by the American Academy of Pediatrics page 103


2011 PREP SA on CD-ROM

Question: 32

A mother brings her 3-year-old boy to the emergency department because of increased
sleepiness. She explains that his activity has decreased over the day, and when she tried to
awaken him from his nap 1 hour ago, she could not arouse him. He has had no fever or other
signs of illness and no findings of note on his past history. His 6-month-old sister has a seizure
disorder, and his maternal grandmother has diabetes. On physical examination, the child moans to
sternal rub. His temperature is 36.5C, heart rate is 72 beats/min, respiratory rate is 20
breaths/min, and blood pressure is 60/30 mm Hg. You administer oxygen by a nonrebreather
mask and obtain a bedside glucose determination, which is 120 mg/dL (6.7 mmol/L).

Of the following, the MOST appropriate next step in this childs treatment is to

A. administer activated charcoal

B. administer atropine

C. infuse 20 mL/kg of 0.9% saline

D. initiate external rewarming

E. obtain a urine toxicology screen

Copyright 2010 by the American Academy of Pediatrics page 104


2011 PREP SA on CD-ROM

Critique: 32 Preferred Response: C

The boy described in the vignette most likely has taken his sisters anticonvulsant medication
and is exhibiting signs and symptoms consistent with a barbiturate overdose, including coma,
bradycardia, and hypotension. Although an overdose of oral hypoglycemic agents may cause
similar symptoms, they would be due to hypoglycemia, which is not present in this patient.
Treatment of a barbiturate overdose is largely supportive and involves restoration of vital
functions and decontamination. The initial approach should focus on ensuring adequate
oxygenation and ventilation with supplemental oxygen and assisted ventilation and correcting
hypotension with fluid boluses and vasoactive pressor agents. The boy requires an initial saline
bolus to correct hypotension. Decontamination with activated charcoal should follow once
resuscitative measures are underway, although in a patient who has altered mental status, care
must be taken to ensure a protected airway prior to activated charcoal administration. Passive
external rewarming with warm blankets may be initiated and a urine toxicology screen may be
obtained as the patient is being stabilized. Atropine is not indicated because the bradycardia in
this setting is not vagally mediated.
Although the overall incidence of barbiturate overdose has decreased as safer
anticonvulsants and sleep aids have been introduced, the illicit use and abuse of barbiturates
has seen a resurgence among adolescents. This is of particular concern because these
medications are highly addictive and produce tolerance within a narrow therapeutic range. As a
result, teens who become addicted require increasingly higher doses that can extend into the
lethal dose range.

Suggested reading:

Greene JP, Ahrendt D, Stafford EM. Adolescent abuse of other drugs. Adolesc Med Clin.
2006;17:283-318. DOI: 10.1016/j.admecli.2006.03.007. Abstract available at:
http://www.ncbi.nlm.nih.gov/pubmed/16814695

Lafferty KA. Toxicity, barbiturate. eMedicine Specialties, Emergency Medicine, Toxicology.


2008. Available at: http://www.emedicine.com/emerg/topic52.htm

Copyright 2010 by the American Academy of Pediatrics page 105


2011 PREP SA on CD-ROM

Question: 33

A previously healthy, 3 1/2-year-old girl presents following 2 days of diarrhea, vomiting, and a
low-grade fever. Her symptoms began shortly after the family dined at a local fast-food
restaurant. She has had four to six watery-mucoid stools per day. Her parents are very
concerned because they have begun to see some blood in her stool. On physical examination,
the alert, somewhat irritable child has a temperature of 38.6C, heart rate of 100 beats/min, and
respiratory rate of 16 breaths/min. Her oral mucous membranes are dry, and capillary refill is 2
seconds. Her abdomen is diffusely tender and without distention. Initial laboratory results include:

Hemoglobin, 11.5 g/dL (115 g/L)


White blood cells, 14.5x10 3/mcL (14.5x109/L)
Sodium, 136 mEq/L (136 mmol/L)
Potassium, 4.5 mEq/L (4.5 mmol/L)
Bicarbonate, 18 mEq/L (18 mmol/L)

Of the following, the MOST appropriate treatment for this child is

A. a glucose-electrolyte solution

B. cholestyramine

C. loperamide

D. metronidazole

E. trimethoprim-sulfamethoxazole

Copyright 2010 by the American Academy of Pediatrics page 106


2011 PREP SA on CD-ROM

Critique: 33 Preferred Response: A

The clinical findings of fever, diarrhea (often with mucus and blood), and leukocytosis
appearing soon after dining at a restaurant described for the girl in the vignette are typical for
gastroenteritis caused by nontyphoidal Salmonella (NTS). Such findings generally appear within
72 hours and as rapidly as 6 hours following exposure. NTS is the most common organism
implicated in foodborne gastroenteritis in the United States. S enteritidis (serogroup D), S
typhimurium (B), and S newport (C2) are the most common NTS species, accounting for
approximately 50% of the culture-confirmed isolates. The principal reservoirs for NTS are
poultry, livestock, reptiles, and household pets (especially turtles). Foods of animal origin
comprise the most common modes of transmission, especially poultry, dairy products, beef, and
eggs. Ingestion of contaminated water and contact with infected animals (reptiles are a common
source) represent other infecting vehicles. As with many foodborne illnesses, transmission also
may occur via the fecal-oral route.
In most cases, infection causes a relatively mild, self-limited illness. However, in young
infants (<6 months of age), the elderly, and debilitated and immunocompromised individuals,
infection with NTS may be associated with a wide spectrum of serious complications, including
bacteremia, enterocolitis, and other localized invasive infections such as meningitis and
osteomyelitis. Therapy for the previously healthy toddler in the vignette should focus on
maintaining fluid homeostasis with a glucose-electrolyte solution provided either orally or
intravenously, depending upon the childs hydration status and oral tolerance.
Antibiotics are not indicated for uncomplicated cases of NTS gastroenteritis because they
do not shorten the duration of illness and may prolong the period of fecal bacterial excretion.
Antibiotics should be reserved for patients who have severe NTS disease or who are at a high
risk for invasive disease. For example, treatment with antimicrobial agents are indicated for all
infants younger than 3 months of age and for infants younger than 12 months of age who have
temperatures exceeding 39.0C pending blood culture results. Others for whom antibiotic
treatment may be recommended include those who have hemoglobinopathies (especially sickle
cell disease), human immunodeficiency virus infection and other immunocompromised states,
neoplastic diseases, and chronic gastrointestinal illnesses.
In suspected bacterial enterocolitis, antidiarrheal agents such as loperamide may inhibit
gastrointestinal motility, leading to third space fluid accumulation; mask symptoms, and prolong
the duration of illness. Cholestyramine is a bile acid-binding resin that may be efficacious in
certain enterotoxigenic diarrheas (eg, as an adjunctive, toxin-binding therapy for Clostridium
difficile enterocolitis), but it has no role in the management of NTS infection.

Suggested reading:

American Academy of Pediatrics. Salmonella infections. In: Pickering LK, Baker CJ, Kimberlin
DW, Long SS, eds. Red Book: 2009 Report of the Committee on Infectious Diseases. 28th ed.
Elk Grove Village, Ill: American Academy of Pediatrics; 2009:584-589

Chatterjee A, OKeefe C, Varman M. Salmonella infection. eMedicine Specialties, Pediatrics:


General Medicine, Infectious Disease. 2009. Available at:

Copyright 2010 by the American Academy of Pediatrics page 107


2011 PREP SA on CD-ROM

http://www.emedicine.com/PED/topic2033.htm

Crum-Cianflone NF. Salmonellosis and the gastrointestinal tract: more than just peanut butter.
Curr Gastroenterol Rep. 2008;10:424-431. Available at:
http://www.ncbi.nlm.nih.gov/pmc/articles/PMC2753534/?tool=pubmed

Hohmann EL. Nontyphoidal salmonellosis. Clin Infect Dis. 2001;32:263-269. DOI:


10.1086/318457. Available at:
http://www.journals.uchicago.edu/doi/full/10.1086/318457?cookieSet=1

Ogle JW, Anderson MS. Salmonella gastroenteritis. In: Hay WW Jr, Levin MJ, Deterding RR,
Sondheimer JM, eds. Current Diagnosis & Treatment: Pediatrics. 19th ed. New York, NY: The
McGraw-Hill Companies, Inc; 2008, 1152-1154

Copyright 2010 by the American Academy of Pediatrics page 108


2011 PREP SA on CD-ROM

Question: 34

A mother in your pediatric practice recently delivered a 28 weeks gestation infant who is in the
neonatal intensive care unit. The woman exclusively breastfed her previous child, who was born
at 36 weeks gestation. She is concerned that something is wrong with her milk for this infant
because it is being combined with human milk fortifier before being given to her infant. You
reassure her by explaining that fortification helps to meet the additional needs of her preterm
infant.

Of the following, the MOST important role of such fortification is to

A. augment the immunologic properties of human milk

B. boost the carbohydrate content of human milk

C. decrease the osmolality of human milk

D. enhance the absorption of iron from human milk

E. increase the protein content of human milk

Copyright 2010 by the American Academy of Pediatrics page 109


2011 PREP SA on CD-ROM

Critique: 34 Preferred Response: E

The protein content of human milk must be increased to meet the requirements of a preterm
infant. The estimated protein requirement for a preterm infant is 3.0 to 4.0 mg/kg per day
compared to 1.5 to 2.0 mg/kg per day for the term infant. The requirement for the preterm infant
does not include additional allowances for catch-up growth that are due to losses of lean body
weight prior to the infant surpassing birthweight. If these allowances are included, the
recommended protein content increases to 3.4 to 4.2 mg/kg per day.
Milk supplied by the preterm infants mother is the preferred enteral feeding. Although milk
produced by a mother who delivers prematurely may have increased protein content compared
to term milk, the protein content declines in the first weeks of lactation to that of term milk. Human
milk fortifier supplies additional protein, which has been shown to increase concentrations of
blood urea nitrogen and weight gain in the enterally feeding preterm infant.
Human milk fortifier does not augment the immunologic properties of human milk. Human milk
contains secretory immunoglobulin A as well as other factors such as lactoferrin and lysozyme
that confer immunologic protection upon the infant. The carbohydrate content of preterm human
milk may be slightly lower than term human milk, but it is well tolerated in spite of the intestinal
lactase functioning at 30% of the level of a term infant. The iron content of human milk is lower
than that of iron-fortified formula, but nearly 50% is absorbed. Human milk fortifier does not
significantly increase the carbohydrate content or enhance the absorption of iron. Of note, the
addition of human milk fortifier does increase the osmolality of enteral human milk feedings.

Suggested reading:

Adamkin DH. Nutrition management of the very-low birthweight infant: II. Optimizing enteral
nutrition and postdischarge nutrition. NeoReviews. 2006;7:e608-e614. Available at:
http://neoreviews.aappublications.org/cgi/content/full/7/12/e608

Kleinman RE. Nutritional needs of the preterm infant. In: Pediatric Nutrition Handbook. 6th ed. Elk
Grove Village, Ill: American Academy of Pediatrics; 2009:79-112

Kuschel CA, Harding JE. Multicomponent fortified human milk for promoting growth in preterm
infants. Cochrane Database Syst Rev. 2004;1:CD000343. DOI:
10.1002/14651858.CD000343.pub2. Available at:
http://www.mrw.interscience.wiley.com/cochrane/clsysrev/articles/CD000343/frame.html

Premji SS, Fenton TR, Sauve RS. Higher versus lower protein intake in formula-fed low birth
weight infants. Cochrane Database Syst Rev. 2006;1:CD003959.
DOI:10.1002/14651858.CD003959.pub2. Available at:
http://www.mrw.interscience.wiley.com/cochrane/clsysrev/articles/CD003959/frame.html

Copyright 2010 by the American Academy of Pediatrics page 110


2011 PREP SA on CD-ROM

Question: 35

You are examining a healthy 4-year-old boy who has complained of intermittent left anterior thigh
pain over the past 1 to 2 months. In the past week, his parents have begun to notice that he is
limping slightly on that side. He has had no fever, malaise, other complaints of pain, or change in
appetite or activity. He is taking no medications. Physical examination reveals an afebrile child
who has no redness, warmth, or swelling over his legs or feet and no point tenderness over his
spine, pelvis, thigh, lower leg, feet, or abdomen. He does exhibit decreased internal rotation at the
left hip. His peripheral white blood cell count is 9.6x103/mcL (9.6x109/L), erythrocyte
sedimentation rate is 8 mm/hr, and C-reactive protein is 0.4 mg/dL. Radiographic evaluation of the
hips shows flattening and sclerosis of the femoral head on the left (Item Q35).

Of the following, the MOST appropriate management includes

A. observation at home and follow-up evaluation in 1 week

B. orthopedics consultation

C. parenteral antibiotics for 2 to 4 weeks

D. physical therapy referral

E. rheumatology consultation

Copyright 2010 by the American Academy of Pediatrics page 111


2011 PREP SA on CD-ROM

Question: 35

(Reprinted with permission from Scherl SA. Common lower extremity problems in children.
Pediatr Rev. 2004;25:52-62)
Anteroposterior radiograph of the hips reveals sclerosis and flattening of the femoral head, as
described for the boy in the vignette.

Copyright 2010 by the American Academy of Pediatrics page 112


2011 PREP SA on CD-ROM

Critique: 35 Preferred Response: B

Legg-Calv-Perthes disease is one of the common causes of hip pain and limp in children 3
to 10 years of age, occurring more frequently in boys (80%) than girls (20%). Often, the initial
interruption in vascular supply is asymptomatic, with limp or pain developing during collapse of
the femoral head or revascularization. On physical examination, the child appears well and is
afebrile. The most constant findings are limited abduction and internal rotation of the hip, as
described for the boy in the vignette. The findings are most prominent at the extremes of the
range of motion, with less pain in midrange motion.
Legg-Calv-Perthes disease is diagnosed using bilateral hip or frog leg and lateral
radiographs. Earliest findings include joint space widening due to narrowing of the proximal
femoral epiphysis and sclerosis of the femoral epiphysis. The next phase is fragmentation in
which lateral films may show a crescent sign resulting from a subchondral fracture.
Subsequently, films may show reossification and eventual healing. Some authors believe that
magnetic resonance imaging is more sensitive in detecting early disease. Laboratory findings are
normal and are only useful for evaluation of other entities in the differential diagnosis.
Appropriate treatment of Legg-Calv-Perthes disease is controversial and depends on
patient age and extent of necrosis. Consultation with an orthopedic specialist is vital as soon as
the diagnosis is made. The prognosis is best when the child is younger than 6 years at
presentation or there is less than 50% necrosis of the femoral head. For these patients, physical
therapy and close monitoring may be sufficient to maintain the femoral head in appropriate
position in the acetabulum, but orthopedics consultation must precede such therapy. Orthoses
that hold the hip in abduction have been used for patients who have more severe disease, but
not all studies have shown benefit with this treatment, and some authors recommend
abandoning this modality. A variety of surgical approaches is available, but operative intervention
usually is reserved for children who have the most severe involvement. In a recent study,
osteotomy resulted in a significantly better outcome than orthosis or physical therapy for
children older than 6 years of age who had more than 50% femoral head necrosis.
Observation with close follow-up is appropriate for a limping child who has transient
synovitis, but radiographs for such a patient would appear normal or show only a small hip
effusion. Surgical evacuation of the hip effusion and a prolonged course of antibiotics are
required for a septic hip and osteomyelitis. However, a child who has that diagnosis would be
febrile, have severe pain, and have radiographs showing a hip effusion. If a rheumatologic
condition such as juvenile idiopathic arthritis is suspected, a rheumatology consultation is
appropriate, but flattening and sclerosis of the femoral head would not be seen on radiography.

Suggested reading:

Canale ST, Beaty JH. Osteochondrosis or epiphysitis and other miscellaneous affections: Legg-
Calv-Perthes disease. In: Campbell's Operative Orthopaedics. 11th ed. Philadelphia, Pa: Mosby
Elsevier; 2007: Chapter 29

Canavese F, Dimeglio A. Perthes disease: prognosis in children under six years of age. J Bone
Joint Surg (Br). 2008;90:940-945. Abstract available at:

Copyright 2010 by the American Academy of Pediatrics page 113


2011 PREP SA on CD-ROM

http://www.ncbi.nlm.nih.gov/pubmed/18591607

Frick SL. Evaluation of the child who has hip pain. Orthop Clin North Am. 2006;37:133-140. DOI:
10.1016/j.ocl.2005.12.003. Abstract available at: http://www.ncbi.nlm.nih.gov/pubmed/16638444

Gough-Palmer A, McHugh K. Investigating hip pain in a well child. BMJ. 2007;334:1216-1217.


DOI: 10.1136/bmj.39188.515741.47.

Sawyer JR, Kapoor M. The limping child: a systematic approach to diagnosis. Am Fam
Physician. 2009;79:215-224. Abstract available at:
http://www.ncbi.nlm.nih.gov/pubmed/19202969

Tse SML, Laxer RM. Approach to acute limb pain in childhood. Pediatr Rev. 2006;27:170-180.
DOI: 10.1542/10.1542/pir.27-5-170. Available at:
http://pedsinreview.aappublications.org/cgi/content/full/27/5/170

Wiig O, Terjesen T, Svenningsen S. Prognostic factors and outcome of treatment in Perthes


disease: a prospective study of 368 patients with five-year follow-up. J Bone Joint Surg Br.
2008;90:1364-1371. Abstract available at: http://www.ncbi.nlm.nih.gov/pubmed/18827249

Copyright 2010 by the American Academy of Pediatrics page 114


2011 PREP SA on CD-ROM

Question: 36

A 10-year-old girl has been participating in competitive gymnastics since she was 4 years of age
and has been very successful. You have seen her several times over the past few years for
pain in her wrist. She now presents with persistent pain, even at rest. Physical examination
reveals generalized tenderness without a mass or deformity over the wrist. Radiographs of the
wrist reveal chronic changes consistent with epiphyseal inflammation (epiphysitis).

Of the following, the MOST concerning potential outcome of this overuse injury is

A. avascular necrosis of the navicular bone

B. carpal tunnel syndrome

C. development of a ganglion cyst

D. radial epiphyseal damage with growth arrest

E. radial epiphyseal fracture with nonunion

Copyright 2010 by the American Academy of Pediatrics page 115


2011 PREP SA on CD-ROM

Critique: 36 Preferred Response: D

Overuse injuries are common in child athletes and are characterized by repetitive
microtrauma to bone and tendon. Gymnasts, such as the girl described in the vignette, sustain
foot and hand injuries due to load bearing during handstands and similar activities. In the overuse
injury of the radius described for the girl in the vignette, the major concern is radial epiphysitis,
which may result in impaired linear growth of the affected bone. Necrosis of the navicular bone
also may cause impaired growth of the wrist in children and limit movement in patients of all
ages, but it usually is seen with fracture (often occult). The child in the vignette does not have
evidence of overt fracture of the epiphysis, but clinicians should be alert for Salter Harris
fractures involving the epiphysis of long bones and growth impairment. A ganglion cyst or carpal
tunnel syndrome would be an unlikely result of overuse in a gymnast. A ganglion cyst normally
presents over the dorsal aspect of the hand as a mass that is slightly tender to palpation. Carpal
tunnel syndrome generally involves paresthesias and pain and results from restriction of the
carpal ligaments, usually in middle-aged adults.
Overuse injuries, by definition, result from repetitive strain at cartilaginous junctions. The
recommended therapeutic approach is rest, ice, and anti-inflammatory medications. Exercise that
does not aggravate the injury but preserves conditioning may be continued during recovery,
especially for the competitive athlete. This exercise program can be prescribed for the athlete by
the coach and trainer, often in consultation with a sports medicine specialist. Other common
overuse injuries are Osgood-Schlatter disease of the tibial tuberosity, often seen in athletes who
engage in repetitive running and jumping motions, and Sever disease of the calcaneus.

Suggested reading:

Atanda A Jr, Reddy D, Rice JA, Terry MA. Injuries and chronic conditions of the knee in young
athletes. Pediatr Rev. 2009;30:419-430. DOI: 10.1542/10.1542/pir.30-11-419. Available at:
http://pedsinreview.aappublications.org/cgi/content/short/30/11/419

Brenner JS and the Council on Sports Medicine and Fitness. Overuse injuries, overtraining, and
burnout in child and adolescent athletes. Pediatrics. 2007;119:1242-1245. DOI:
10.1542/peds.2007-0887. Available at:
http://pediatrics.aappublications.org/cgi/content/full/119/6/1242

Frush TJ, Lindenfeld TN. Peri-epiphyseal and overuse injuries in adolescent athletes. Sports
Health: A Multidisciplinary Approach. 2009;1:201-211. DOI: 10.1177/1941738109334214.
Abstract available at: http://sph.sagepub.com/content/1/3/201.abstract

Copyright 2010 by the American Academy of Pediatrics page 116


2011 PREP SA on CD-ROM

Question: 37

A 5-month-old infant who had been born at 37 weeks gestation presents to the emergency
department with a 3-day history of cough, fever, and increased work of breathing. On physical
examination, the infants heart rate is 140 beats/min, respiratory rate is 55 beats/min, blood
pressure is 82/55 mm Hg, and oxygen saturation is 82%. Auscultation of the chest reveals rales
over the right back and a II/VI vibratory systolic ejection murmur at the left sternal border. Chest
radiography demonstrates a right lower lobe consolidation. Laboratory evaluation reveals normal
electrolyte concentrations except for a bicarbonate of 29 mEq/L (29 mmol/L), hematocrit of 29%
(0.29), pH of 7.30, PO2 of 60 mm Hg, and PcO2 of 50 mm Hg.

Of the following, the MOST likely reason for this childs desaturation is

A. anemia of prematurity

B. bacterial pneumonia

C. methemoglobinemia

D. myocarditis

E. ventricular septal defect

Copyright 2010 by the American Academy of Pediatrics page 117


2011 PREP SA on CD-ROM

Critique: 37 Preferred Response: B

The child who has cyanosis can present either acutely ill or with chronic, compensated
hypoxemia. The hypoxemia can be caused by cardiopulmonary or noncardiopulmonary disease.
The most common cause of hypoxemia in childhood is a right-to-left shunt caused by a V-Q
mismatch as a result of pulmonary disease. The possible intrapulmonary causes include
bacterial or viral upper or lower respiratory tract infection, bronchiolitis, bronchospasm, pleural
effusion, pneumothorax, pulmonary sequestration, tumors, and congenital cystic adenomatoid
malformation. The findings of rales on auscultation of the lungs, fever, respiratory compromise,
and focal pulmonary infiltrate on chest radiography, as described for the infant in the vignette,
suggest the diagnosis of pneumonia. Childhood viral causes include influenza virus A and B,
respiratory syncytial virus, parainfluenza virus, adenovirus, herpes simplex virus (primarily in
newborns), and cytomegalovirus (primarily in the immunocompromised patient). The most
common causes of pediatric bacterial pneumonia are the gram-positive organisms
Streptococcus pneumoniae and Staphylococcus aureus. Gram-negative bacteria are seen less
frequently and include Haemophilus influenzae, Klebsiella pneumoniae, Escherichia coli,
Pseudomonas aeruginosa, and Moraxella catarrhalis.
Methemoglobinemia can cause generalized cyanosis in a young infant. Affected infants
appear ill and exhibit desaturation on pulse oximetry. Of note, co-oximetric analysis of arterial
blood is required for the diagnosis, which allows quantitation of the true amount of
oxyhemoglobin and determination of the percentage of methemoglobin in the blood (15% to 30%
results in clinically apparent cyanosis). Normally, methemoglobin values are less than 1% on co-
oximetry. Methemoglobin concentrations in the blood are elevated when the mechanisms that
defend against oxidative stress within the red blood cell are overwhelmed and the oxygen-
carrying ferrous ion (Fe2+) of the heme group of the hemoglobin molecule is oxidized to the
ferric state (Fe3+). This converts hemoglobin to methemoglobin, which is a nonoxygen-binding
form of hemoglobin that binds a water molecule instead of oxygen. Another mechanism of
methemoglobinemia is caused by toxicity due to the nitrite-based antidote to cyanide poisoning.
Cyanide exposure can occur from smoke inhalation, ingestion of toxic household and workplace
substances, and ingestion of cyanogenic foods. The vitamin B12 precursor hydroxocobalamin
may be a safer alternative therapy for cyanide poisoning.
Preterm infants often exhibit anemia and typically experience heavy blood losses from
frequent laboratory testing in the first few postnatal weeks. Although their anemia is
multifactorial, repeated blood sampling and reduced erythropoiesis with extremely low serum
concentrations of erythropoietin are major determining factors. During the first weeks after birth,
all infants experience a decline in hemoglobin concentration. Normally, this stimulates significantly
increased production of erythropoietin, but this response is diminished in preterm infants. Fever,
rales on physical examination, and changes on chest radiography are not associated with
anemia of prematurity, and the hematocrit of the infant in the vignette is too high to suggest
symptomatic anemia of prematurity.
Myocarditis most commonly is viral in origin and can be accompanied by fever, respiratory
distress, rales on examination, and acidemia. Myocarditis results in cardiomyopathy, with left
ventricular dilation and dysfunction. The lack of an increased cardiac silhouette on chest
radiograph argues against this diagnosis for this infant.

Copyright 2010 by the American Academy of Pediatrics page 118


2011 PREP SA on CD-ROM

A ventricular septal defect (VSD) causes a left-to-right shunt and congestive heart failure.
An intrapulmonary V-Q mismatch that causes cyanosis does not occur until the child suffers
from severe resultant pulmonary overcirculation and parenchymal congestion. The cardiac
examination findings for a child who has a hemodynamically significant VSD include a high-
grade holosystolic murmur accompanied by a diastolic rumble. In addition, fever itself is not a
part of the symptom complex.

Suggested reading:

Aly H. Respiratory disorders in the newborn: identification and diagnosis. Pediatr Rev.
2004;25:201-208. DOI: 10.1542/10.1542/pir.25-6-201. Available at:
http://pedsinreview.aappublications.org/cgi/content/full/25/6/201

Geller RJ, Barthold C, Saiers JA, Hall AH. Pediatric cyanide poisoning: causes, manifestations,
management, and unmet needs. Pediatrics. 2006;118:2146-2158. DOI: 10.1542/peds.2006-1251.
Available at: http://pediatrics.aappublications.org/cgi/content/full/118/5/2146?

Copyright 2010 by the American Academy of Pediatrics page 119


2011 PREP SA on CD-ROM

Question: 38

A 3-year-old child presents with 24 hours of episodes of inconsolable crying. His parents state
that his sleep/wake pattern is altered and he seems uncharacteristically "out of it" at times.
Physical examination shows fluctuation in attention and arousal, with diffuse hyperreflexia.
Emergency head computed tomography scan shows no abnormalities. Laboratory testing
documents markedly elevated serum transaminases.

Of the following, the MOST likely cause for this childs mental status change is

A. acetaminophen ingestion

B. antiphospholipid antibody syndrome

C. carbon monoxide poisoning

D. diphenhydramine intoxication

E. lead poisoning

Copyright 2010 by the American Academy of Pediatrics page 120


2011 PREP SA on CD-ROM

Critique: 38 Preferred Response: A

The child described in the vignette has acute confusional state/encephalopathy,


characterized by fluctuating arousal (sleep/wake) and attention ("out of it") plus irritability. An
ingestion is a prime consideration for acute encephalopathy in a toddler, and the laboratory
results for this child support acetaminophen ingestion.
It is important to approach the problem of altered mental status systematically. One approach
is to consider three broad causative categories, using the mnemonic F-I-T, representing focal,
ictal, and toxic/metabolic.
Focal disease processes responsible for encephalopathy can include trauma, abscess, and
stroke. Physical examination of the child in the vignette reveals hyperreflexia, but it is diffuse. No
findings on examination, such as focal weakness or asymmetric motor responses, support a
focal lesion. Focal diseases should be considered first because generally emergency imaging
with head computed tomography scan should be obtained in the presence of positive findings. In
this case, it was nondiagnostic.
The second category is ictal (seizure-related) or postictal states. New-onset or recurrent
epileptic seizures leading to postictal confusion are a common cause of self-limited
encephalopathy in childhood. However, confusion for the duration described for this child, if
seizure-related, likely would represent nonconvulsive status epilepticus. Such a diagnosis is
rare in children of this age who are not known to have epilepsy. If clinically apparent seizures
are observed, however, the child should be treated with a loading dose of antiseizure
medication. In the proper clinical setting, infection and inflammatory causes also can be
considered.
Emergent laboratory testing for toxic/metabolic causes should be automatic and includes
complete blood count, glucose, electrolytes, liver panel, and toxicology screen. Ingestion is a
particular consideration in a toddler. The markedly elevated liver transaminases reported for this
child are consistent with an acute hepatic process causing a hepatic encephalopathy. The early
stages of acute liver failure present with encephalopathy, irritability, and hyperreflexia. The most
common cause for such findings in children younger than age 3 is acetaminophen ingestion.
Ingestion of a sufficient quantity of lead to produce acute encephalopathy is rare. Such an
ingestion produces elevated intracranial pressure, seizures, and possibly hemolytic anemia and
multiorgan dysfunction.
Carbon monoxide is an odorless gas, and poisoning is a common cause of intoxication,
leading to death. The source in the home is usually a furnace that produces unstable carbon
monoxide, which enters the lungs and robs them of oxygen. Carbon monoxide poisoning can
present with encephalopathy, headache, and flulike symptoms and generally is more indolent
than described in the vignette. Carbon monoxide poisoning that produces symptoms this severe
in a toddler also should cause symptoms in other family members.
Diphenhydramine is an antihistamine sold over-the-counter in liquid or pill form and included
in combination cold medications. Overdose commonly causes an acute encephalopathy,
seizures, and anticholinergic symptoms and signs. Elevated transaminases are not
characteristic.
Antiphospholipid antibody syndrome is a rare autoimmune condition diagnosed using clinical
and laboratory criteria. It can present with one or more episodes of arterial or venous

Copyright 2010 by the American Academy of Pediatrics page 121


2011 PREP SA on CD-ROM

thrombosis, which could lead to stroke and encephalopathy. The examination and normal head
computed tomography scan results for this child do not support this diagnosis.

Suggested reading:

Bucuvalas J, Yazigi N, Squires RH Jr. Acute liver failure in children. Clin Liver Dis. 2006;10:149-
68. DOI: 10.1016/j.cld.2005.10.006. Abstract available at:
http://www.ncbi.nlm.nih.gov/pubmed/16376799

Eicher T, Avery E. Toxic encephalopathies. Neurol Clin. 2005;23:353-376. DOI:


10.1016/j.ncl.2004.12.004. Abstract available at: http://www.ncbi.nlm.nih.gov/pubmed/15757789

Eldridge DL, Van Eyk J, Kornegay C. Pediatric toxicology. Emerg Med Clin North Am.
2007;25:283-308. DOI: 10.1016/j.emc.2007.02.011. Abstract available at:
http://www.ncbi.nlm.nih.gov/pubmed/17482021

Squires RH Jr, Shneider BL, Bucuvalas J, et al. Acute liver failure in children: the first 348
patients in the Pediatric Acute Liver Failure Study Group. J Pediatr. 2006;148:652-658. DOI:
10.1016/j.jpeds.2005.12.051. Available at:
http://www.ncbi.nlm.nih.gov/pmc/articles/PMC2662127/?tool=pubmed

Copyright 2010 by the American Academy of Pediatrics page 122


2011 PREP SA on CD-ROM

Question: 39

A 3-year-old boy is brought to the clinic by his parents due to concerns about how easily he
bruises. They say that since he began walking at 18 months, he frequently has large, purple
bruises that appear with no known history of trauma. They do not believe that he falls more
frequently than other children his age, and they deny a family history of easy bruising. On
physical examination, the normally grown child has prominent eyes, a delicate and narrow nose,
and numerous bruises (Item Q39A) in various stages of healing, primarily overlying his shins but
also scattered elsewhere on his body. He has translucent skin over the chest, with prominent
vascular markings, and his fingers are slender and hypermobile (Item Q39B).

Of the following, the condition that is MOST consistent with this boys features is

A. Ehlers-Danlos syndrome

B. hemophilia A

C. Marfan syndrome

D. Stickler syndrome

E. von Willebrand disease

Copyright 2010 by the American Academy of Pediatrics page 123


2011 PREP SA on CD-ROM

Question: 39

(Courtesy of M Rimsza)
Bruises, as described for the boy in the vignette.

Copyright 2010 by the American Academy of Pediatrics page 124


2011 PREP SA on CD-ROM

Question: 39

(Courtesy of M Rimsza)
Hypermobility of the fingers, as exhibited by the boy in the vignette.

Copyright 2010 by the American Academy of Pediatrics page 125


2011 PREP SA on CD-ROM

Critique: 39 Preferred Response: A

Ehlers-Danlos syndrome (EDS) comprises a group of heritable connective tissue disorders


characterized by the triad of hyperextensible skin (Item C39), hyperextensible joints, and tissue
fragility. EDS is divided into six primary types: 1) classic, 2) hypermobility, 3) vascular fragility, 4)
kyphoscoliotic, 5) arthrochalasia, and 6) dermatosparaxis, based on specific major and minor
diagnostic criteria. Most types of EDS are autosomal dominant, but autosomal recessive and X-
linked forms also are described. Although some of the genes that, when altered, can cause EDS
are known, many are not, making clinical acumen critical for diagnosis.
The child described in the vignette has the characteristic facial features, easy bruising,
prominent vasculature, and distal hypermobility of EDS, vascular fragility type. Individuals who
have this form of EDS are at increased risk for spontaneous rupture of medium-sized arteries,
and many succumb to their disease by age 40. Management is aimed at avoiding activities that
can place affected individuals at increased risk for arterial rupture, such as collision sports,
weight training, and heavy lifting. Elective surgeries and arteriography generally are
discouraged. Complaints of sudden pain or bleeding should prompt immediate evaluation in
consultation with a physician familiar with EDS because emergent intervention for significant
vascular injury is critical for the best possible outcome.
Hemophilia A is an X-linked condition that is not associated with dysmorphic features or
hypermobility. Excessive bleeding is not due to vascular rupture but to deficiency of Factor VIII.
Marfan and Stickler syndromes are autosomal dominant connective tissue disorders that are
associated with hypermobility, but affected individuals typically do not have easy bruising or
prominent vasculature. von Willebrand disease represents a group of typically autosomal
dominant clotting disorders due to abnormal quantity or quality of von Willebrand factor; it is not
associated with hypermobility or unusual physical features.

Suggested reading:

Beighton P, De Paepe A, Steinmann B, Tsipouras P, Wenstrup RJ. Ehlers-Danlos: revised


nosology, Villefranche, 1997. Ehlers-Danlos National Foundation (USA) and Ehlers-Danlos
Support Group (UK). Am J Med Genet. 1998;77:31-37. DOI: 10.1002/(SICI)1096-
8628(19980428)77:1<31::AID-AJMG8>3.0.CO;2-O. Abstract available at:
http://www.ncbi.nlm.nih.gov/pubmed/9557891

Wenstrup R, De Paepe A. Ehlers Danlos syndrome, classic type. GeneReviews. 2008. Available
at: http://www.ncbi.nlm.nih.gov/bookshelf/br.fcgi?book=gene&part=eds

Copyright 2010 by the American Academy of Pediatrics page 126


2011 PREP SA on CD-ROM

Critique: 39

(Courtesy of T Jewett)
Hyperextensibility of the skin in a patient who has Ehlers-Danlos syndrome.

Copyright 2010 by the American Academy of Pediatrics page 127


2011 PREP SA on CD-ROM

Question: 40

A 14-year-old girl who is not sexually active presents with a yellow vaginal discharge that has
had an increasingly foul odor over the past 2 weeks. She had a normal menstrual period 3 weeks
ago. She currently has no other symptoms, has a negative past medical history, and is not
receiving any medications.

Of the following, the MOST likely diagnosis is

A. herpes simplex infection

B. nonspecific vulvovaginitis

C. trichomonal vaginitis

D. vaginal candidiasis

E. vaginal foreign body

Copyright 2010 by the American Academy of Pediatrics page 128


2011 PREP SA on CD-ROM

Critique: 40 Preferred Response: E

The girl described in the vignette is not sexually active and has a foul-smelling, yellowish
discharge that is highly suggestive of a foreign body. A retained tampon may have such a
presentation without the development of toxic shock syndrome.
Young adolescent females who present with vulvar or vaginal symptoms need to be
evaluated for both sexually transmitted infections (STIs) and nonsexually related causes. The
condition may involve the vulva, vagina, or cervix.
When compared with postpubertal females, those who are prepubertal are more likely to
have vulvar irritation due to the absence of labial fat and pubic hair and the lack of protective
effects of estrogenization of the mucosa. In this age group, irritation can result from tight
clothing, nonabsorbent underwear, obesity, poor hygiene, allergy, or pinworms. Contact of the
vaginal mucosa with certain chemicals, such as soaps or bubble baths, may cause complaints
of genital burning or swelling. Physical examination may reveal slight redness of the vaginal
mucosa and an increase in normal discharge.
In postpubertal females, vulvovaginitis may result from candidiasis, trichomoniasis, or
bacterial vaginosis. Candidiasis may present with skin irritation and itching, dysuria from urine
flow over the irritated skin areas (external dysuria), a change in the usual vaginal discharge,
dyspareunia, and a mild odor. Trichomoniasis is a sexually transmitted infection that presents
with a white or yellow discharge, occasionally associated with pruritus. Bacterial vaginosis may
not be sexually transmitted and presents as a malodorous, grayish discharge. Abnormal vaginal
discharge also may result from cervicitis caused by Neisseria gonorrhoeae, Chlamydia
trachomatis, or herpes simplex virus. Herpes simplex virus infections, like N gonorrhoeae and C
trachomatis, may be asymptomatic. However, herpes simplex virus infections also may be
characterized by multiple small, painful ulcers on the vulva.

Suggested reading:

Hwang LY, Schaffer MAB. Vaginitis and vaginosis. In: Neinstein LS, Godron CM, Katzman DK,
Rosen DS, Woods ER, eds. Adolescent Health Care A Practical Guide. 5th ed. Philadelphia, Pa:
Lippincott Williams & Wilkins, a Wolters Kluwer business; 2008:723-732

Kokotos F, Adam HM. In brief: vulvovaginitis. Pediatr Rev. 2006;27:116117. DOI:


10.1542/10.1542/pir.27-3-116. Available at:
http://pedsinreview.aappublications.org/cgi/content/full/27/3/116

Sugar NF, Graham EA. Common gynecologic problems in prepubertal girls. Pediatr Rev.
2006;27:213-223. DOI: 10.1542/10.1542/pir.27-6-213. Available at:
http://pedsinreview.aappublications.org/cgi/content/full/27/6/213

Copyright 2010 by the American Academy of Pediatrics page 129


2011 PREP SA on CD-ROM

Question: 41

You are discussing with a mother the upcoming sedated echocardiography for her 6-month-old
infant to evaluate the size of a ventricular septal defect. She asks you why the cardiologist had
said that her child needed to stop breastfeeding for 4 hours prior to the procedure.

Of the following, the MOST appropriate reason for following the nothing per mouth guidelines
prior to sedation is to

A. decrease the risk of human milk and medication interactions

B. decrease the risk of hyperglycemia during the procedure

C. increase the sedative effect of the medication

D. minimize the risk of pulmonary aspiration

E. minimize the size of the stomach to allow adequate imaging

Copyright 2010 by the American Academy of Pediatrics page 130


2011 PREP SA on CD-ROM

Critique: 41 Preferred Response: D

The goal of procedural sedation is to provide anxiolysis and analgesia and to minimize
patient movement. Loss of airway protective reflexes during sedation is possible, and clinicians
must be able to recognize and respond to potential airway compromise. Preoperative fasting
guidelines are designed to reduce the rare risk of pulmonary aspiration of gastric contents.
Current guidelines published by The American Academy of Pediatrics and the American Academy
of Pediatric Dentistry recommend fasting periods as shown in Item C41.
The optimal duration of fasting is unknown for each individual patient, and the risk/benefit
ratio of emergent procedures must be assessed prior to proceeding. A recent study
demonstrated that gastric fluid volumes were similar in infants after either a 4- to 6-hour fast
from infant formula or 2-hour fast after clear liquids.
Prolonged fasting in infants should be avoided because it may increase the risk of
hypoglycemia and dehydration in some patients. The increased irritability associated with
prolonged fasting seen in some infants may necessitate increased doses of sedation and
subsequent risk of adverse events. Formula and human milk may have variable effects on the
gastric absorption of oral sedative medicines, although these generally are minimal. Finally, air in
the stomach may interfere with optimal imaging during echocardiography, but the presence of
small amounts of fluid is unlikely to have the same effect.

Suggested reading:

American Academy of Pediatrics, American Academy of Pediatric Dentistry, Cot CJ, Wilson S,
AAP Work Group on Sedation. Guidelines for monitoring and management of pediatric patients
during and after sedation for diagnostic and therapeutic procedures: an update. Pediatrics.
2006;118:2587-2602. DOI: 10.1542/peds.2006-2780. Available at:
http://pediatrics.aappublications.org/cgi/content/full/118/6/2587

Cook-Sather SD, Harris KA, Chiavacci R, Gallagher PR, Schreiner MS. A liberalized fasting
guideline for formula-fed infants does not increase average gastric fluid volume before elective
surgery. Anesth Analg. 2003;96:965969. Available at:
http://www.anesthesiaanalgesia.org/cgi/reprint/96/4/965

Wetzel R. Anesthesia and perioperative care. In: Kliegman RM, Behrman RE, Jenson HB, Stanton
BF, eds. Nelson Textbook of Pediatrics. 18th ed. Philadelphia, Pa: Saunders Elsevier; 2007:460-
474

Copyright 2010 by the American Academy of Pediatrics page 131


2011 PREP SA on CD-ROM

Critique: 41

Copyright 2010 by the American Academy of Pediatrics page 132


2011 PREP SA on CD-ROM

Question: 42

You are seeing a 12-year-old boy who has missed his health supervision visits for 2 years. His
parents say he has been well and they could not afford to take time off from work to bring him in
for his yearly checkups, but they are worried about him now because he has had headaches
that have caused him to miss several days of school for the past month. The headaches have
awakened him at night once or twice in the past 2 weeks. When you evaluate his visual fields, he
appears to have trouble with vision on the upper outer third of his visual fields that is worse on
the right.

Of the following, the additional physical finding that is MOST contributory to your suspected
diagnosis is

A. axillary hair without other signs of puberty

B. decreased linear growth rate

C. decreased weight gain

D. gynecomastia

E. unilateral testicular enlargement

Copyright 2010 by the American Academy of Pediatrics page 133


2011 PREP SA on CD-ROM

Critique: 42 Preferred Response: B

The boy described in the vignette has evidence of a mass pressing on his optic chiasm and
causing a visual field disturbance. Masses in this area in children are often cranipharyngiomas
arising from the hypothalamus or pituitary, pituitary tumors, or, more rarely, other hypothalamic-
pituitary lesions. Tumors in this area are commonly diagnosed because of headache and visual
field disturbance, but many are identified because of changes in growth velocity leading to the
diagnosis of hypopituitarism. Changes in body composition, including increased fat, decreased
muscle mass, and infantile body proportions are more subtle and often missed. Decreased linear
growth rate is an important sign of hypopituitarism in the growing child. It may represent growth
hormone deficiency, thyroid-stimulating hormone deficiency leading to secondary or tertiary
hypothyroidism, or a combination of the two. It is reported in 7% to 93% of children who have
craniopharyngioma. Some children who have damage to the hypothalamus or pituitary because
of a craniopharyngioma may develop diabetes insipidus, but this is rare before there is other
evidence of tumor such as growth attenuation, headache, or visual field abnormalities.
Axillary hair without other signs of puberty is not uncommon in children who have
adrenarche or adrenal puberty. Such findings also may be seen after exposure to other,
nontesticular sources of androgen. Decreased weight gain rarely is a manifestation of tumors
affecting the central nervous system. Both growth hormone deficiency and hypothyroidism
usually are associated with somewhat increased fat mass. Craniopharyngiomas may extend to
the appetite centers of the hypothalamus and increase hunger, leading to weight gain. Only very
rare tumors affecting the appetite center decrease food intake, leading to weight loss (eg,
diencephalic syndrome). Gynecomastia is a response to low concentrations of circulating
estrogen, as may be seen when androgen of either adrenal or testicular origin is converted to
estrogen in the peripheral fat or if there is exposure to another exogenous or endogenous
source of estrogen. It is not a sign of pituitary hypofunction. Unilateral testicular enlargement
should increase concern for a local testicular tumor but is not expected in hypopituitarism.

Suggested reading:

Chung TT, Monson JP. Hypopituitarism. Endotext.org. Available at:


http://www.endotext.org/neuroendo/neuroendo12/neuroendoframe12.htm

Diamantopoulos S, Bao Y. Gynecomastia and premature thelarche: a guide for practitioners.


Pediatr Rev. 2007;28:e57-e68. DOI: 10.1542/10.1542/pir.28-9-e57. Available at:
http://pedsinreview.aappublications.org/cgi/content/full/28/9/e57

Geffner ME. Hypopituitarism in childhood: diagnosis of hypopituitarism. Medscape Today.


Available at: http://www.medscape.com/viewarticle/438527_9

Halac I, Zimmerman D. Endocrine manifestations of craniopharyngioma. Childs Nerv Syst.


2005;21:640-648. DOI: 10.1007/s00381-005-1246-x. Abstract available at:
http://www.ncbi.nlm.nih.gov/pubmed/16047216

Copyright 2010 by the American Academy of Pediatrics page 134


2011 PREP SA on CD-ROM

Karavitaki N, Cudlip S, Adams CB, Wass JA. Craniopharyngiomas. Endocr Rev. 2006;27:371-
397. DOI: 10.1210/er.2006-0002. Available at:
http://edrv.endojournals.org/cgi/content/full/27/4/371

May JA, Krieger MD, Bowen I, Geffner ME. Craniopharyngioma in childhood. Adv Pediatr.
2006;53:183-209. Abstract available at: http://www.ncbi.nlm.nih.gov/pubmed/17089867

Ross JH. Prepubertal testicular tumors. Urology. 2009;74:94-99. DOI:


10.1016/j.urology.2008.12.036

Copyright 2010 by the American Academy of Pediatrics page 135


2011 PREP SA on CD-ROM

Question: 43

The parents of an 18-month-old boy contact you after he has two episodes of holding his breath
and fainting. Most recently, he was upset when he had to leave the playground and began to
scream and cry. He turned blue while holding his breath prior to losing consciousness. He had a
similar event 1 month ago when he cut his finger and saw that it was bleeding. The anxious
parents ask what they should do.

Of the following, the MOST appropriate intervention is to

A. obtain a complete blood count

B. pick up the child quickly and comfort him when he starts to cry

C. reassure the parents that this is a benign event

D. refer the child for behavioral therapy

E. refer the child for electroencephalography

Copyright 2010 by the American Academy of Pediatrics page 136


2011 PREP SA on CD-ROM

Critique: 43 Preferred Response: C

The child described in the vignette is having a breath-holding spell (BHS), which is the
involuntary stopping of breathing due to a painful or frustrating event. They occur in almost 5%
of children, typically between the ages of 6 and 18 months. A BHS of sufficient duration may
lead to loss of consciousness or a seizure. There are two major types of BHSs. The cyanotic
spell occurs when the child is upset. The child may give a cry, followed by forced expiration,
apnea, cyanosis, and loss of consciousness. Pallid spells are less common and occur following
a painful experience or when the child is startled. The child stops breathing, becomes pale and
hypotonic, and may have a tonic seizure. These seizures do not imply a diagnosis of epilepsy
because they are provoked by the BHS. Electroencephalography may show slowing but not
epileptiform activity and, therefore, is not indicated.
The parents should be advised that while frightening to witness, a BHS is benign and will
not lead to epilepsy, brain damage, or death. Picking up or comforting a child who has become
angry or frustrated in an attempt to avert a BHS should be discouraged because it may reinforce
an unwanted behavior. Similarly, parents should be consistent and not reinforce the behavior
after the child recovers.
Because BHSs are involuntary, behavioral therapy for the child is not indicated. Parents and
caregivers, however, do need to be instructed in appropriate behavioral modification
approaches.
Hematologic conditions such as iron deficiency and transient erythroblastopenia have been
reported in conjunction with a BHS. Because anemia can exacerbate BHS, obtaining a complete
blood count is indicated in severe cases.

Suggested reading:

Johnston MV. Conditions that mimic seizures. In: Kliegman RM, Behrman RE, Jenson HB, Stanton
BF, eds. Nelson Textbook of Pediatrics. 18th ed. Philadelphia, Pa: Saunders Elsevier; 2007:2476-
2477

Rubenstein JE, Vining EPG, Kossoff EHW. Epilepsy and developmental disabilities. In: Accardo PJ,
ed. Capute & Accardos Neurodevelopmental Disabilities in Infancy and Childhood. 3rd ed.
Baltimore, Md: Paul H. Brookes Publishing Co; 2008:399-412

Zuckerman B. Breath holding. In: Parker S, Zuckerman B, Augustyn M, eds. Developmental and
Behavioral Pediatrics: A Handbook for Primary Care. 2nd ed. Philadelphia, Pa: Lippincott
Williams & Wilkins; 2005:139-140

Copyright 2010 by the American Academy of Pediatrics page 137


2011 PREP SA on CD-ROM

Question: 44

When an 11-year-old boy attempts to pet a stray dog in the park, he is bitten on the dorsum of his
right hand, sustaining several breaks in the skin. The parents clean the wounds with soap and
water. The dog is located and captured by animal control officers. Forty-eight hours later, the
child presents to your office with painful swelling of the hand, erythema, and purulent discharge.
The boy is allergic to penicillin.

Of the following, the BEST choice for antibiotic therapy for this child is

A. amoxicillin-clavulanate

B. azithromycin and trimethoprim-sulfamethoxazole

C. cefdinir

D. clindamycin and trimethoprim-sulfamethoxazole

E. doxycycline

Copyright 2010 by the American Academy of Pediatrics page 138


2011 PREP SA on CD-ROM

Critique: 44 Preferred Response: D

Antibiotic therapy for infected dog bites should have a broad spectrum to cover the oral
flora of the animal, including Pasteurella, Staphylococcus aureus, streptococci, and oral
anaerobes. Clindamycin and trimethoprim-sulfamethoxazole is the recommended regimen to
cover this spectrum in the penicillin-allergic child. Amoxicillin-clavulanate is otherwise considered
the drug of choice. Methicillin-resistant S aureus infections have not been reported in animal
bites to date. Azithromycin and trimethoprim-sulfamethoxazole, cefdinir, and doxycycline do not
provide adequate coverage for anaerobes or Pasteurella.
Antibiotic prophylaxis at the time of an animal bite might be considered for hand bites or
deep wounds because they are at the highest risk of infection. Cat bites transmit similar flora,
and treatment recommendations are the same as for dog bite infections.
Rabies exposure may be another major concern after an animal bite. As in this vignette, if
the animal seems well and can be captured and observed for 10 days, rabies prophylaxis is not
necessary. With dog, cat, and ferret bites, if the animal is suspected of being rabid, it should be
sacrificed immediately and tested for rabies. If the animal escaped, consultation with the local
health department is advised for bites from dogs, cats, or ferrets. Skunks, raccoons, foxes, and
most other carnivores as well as bats should be considered rabid unless laboratory tests prove
the animal is negative. Rodents and lagomorphs (rabbits, hares) are unlikely to carry or transmit
rabies.
If rabies prophylaxis is indicated, recent Centers for Disease Control and Prevention
guidelines recommend a four-dose vaccine series (rather than the previously recommended five
doses) plus an initial dose of rabies immune globulin. In all instances, the patients tetanus
immunization status should be reviewed. If it is not up to date, tetanus immune globulin or tetanus
vaccine should be administered.

As a result of reviewing this information, do you intend to make a change in practice


to provide better patient care?
Yes No

Suggested reading:

American Academy of Pediatrics. Bite wounds. In: Pickering LK, Baker CJ, Kimberlin DW, Long
SS, eds. Red Book: 2009 Report of the Committee on Infectious Diseases. 28th ed. Elk Grove
Village, Ill: American Academy of Pediatrics; 2009:187-191

Manning SE, Rupprecht CE, Fishbein D, et al; Advisory Committee on Immunization Practices,
Centers for Disease Control and Prevention. Human rabies prevention United States, 2008:
recommendations of the Advisory Committee on Immunization Practices. MMWR Recomm Rep.
2008;57(RR-3):1-28. Available at: http://www.cdc.gov/mmwr/preview/mmwrhtml/rr5703a1.htm

Copyright 2010 by the American Academy of Pediatrics page 139


2011 PREP SA on CD-ROM

Question: 45

A 6-month-old boy who lives in a foster home presents with progressively increased work of
breathing and poor feeding. His past medical history is unknown, except that his biological mother
used illicit drugs during pregnancy. On physical examination, his temperature is 37.8C, heart rate
is 110 beats/minute, respiratory rate is 60 breaths/minute, oxygen saturation is 70% in room air,
weight is 7.3 kg (10th percentile), and length and head circumference are at the 95th percentile
for age. He has mild-to-moderate subcostal retractions, and his lungs are clear to auscultation. A
chest radiograph shows diffuse bilateral interstitial infiltrates (Item Q45). The white blood cell
count is 2.0x103/mcL (2.0x109/L), with 51% polymorphonuclear leukocytes, 43% lymphocytes,
and 6% monocytes. Serum lactate dehydrogenase is elevated at 700 units/L.

Of the following, the MOST likely test to yield the diagnosis is

A. blood culture

B. bronchoalveolar lavage

C. chest computed tomography scan

D. serologic study

E. sweat chloride test

Copyright 2010 by the American Academy of Pediatrics page 140


2011 PREP SA on CD-ROM

Question: 45

(Courtesy of D Palazzi)

Copyright 2010 by the American Academy of Pediatrics page 141


2011 PREP SA on CD-ROM

Critique: 45 Preferred Response: B

The boy described in the vignette has an elevated lactate dehydrogenase concentration,
tachypnea, oxygen desaturations, low-grade fever, and diffuse bilateral pulmonary infiltrates,
which are consistent with pneumonia caused by Pneumocystis jiroveci (previously P carinii).
Pneumocystis pneumonia (PCP) is an opportunistic infection that occurs almost exclusively in
immunocompromised patients. Although this 6-month-old infant who has lymphopenia and poor
growth may have either a primary or secondary immune deficiency, the history of maternal illicit
drug use is suspicious for perinatally acquired human immunodeficiency virus (HIV) infection.
The clinical features of PCP are not sufficiently specific to differentiate it from other
opportunistic pulmonary infections in high-risk children. A definitive diagnosis can be made only
by demonstration of P jiroveci organisms in lung tissue or respiratory tract secretions. Open-
lung biopsy and transbronchial biopsy are the most sensitive and specific procedures for
diagnosing PCP. However, bronchoalveolar lavage, deep endotracheal aspiration with intubation,
and induced sputum in older children are sensitive and less invasive procedures for patients
who have a high burden of organisms causing disease.
The clinical presentation of PCP can be subacute, acute, or fulminant and life-threatening.
Trimethoprim-sulfamethoxazole is effective and generally well tolerated and is the drug of choice
for both treatment and prevention. It should be initiated promptly in an immunocompromised
patient who has presumed Pneumocystis infection. The drug is administered intravenously for at
least 2 weeks to patients who do not have acquired immunodeficiency syndrome (AIDS) and for
3 weeks to those who have AIDS. Prophylaxis against the first episodes of PCP is recommended
for all infants born to HIV-infected women beginning at 4 to 6 weeks of age and until HIV
infection has been excluded definitively. Children who cannot tolerate trimethoprim-
sulfamethoxazole can be given intravenous pentamidine.
Pneumocystis organisms cannot be grown in blood culture. Patients who have PCP can
exhibit a variety of chest computed tomography scan findings, including diffuse interstitial or
reticulogranular infiltrates (most common), nodules, cavities, cystic spaces, bullae,
pneumothorax, adenopathy, and pleural effusions. No single radiographic presentation is
diagnostic of PCP. Serologic studies reveal that most humans are infected with this ubiquitous
organism during the first 2 to 4 years after birth, making serologic findings of no use in
diagnosing symptomatic infection. Neither immunoglobulin M nor immunoglobulin G tests have
been helpful in diagnosing acute PCP.
A sweat chloride test is used to diagnose cystic fibrosis. Affected children may present
with chronic sinopulmonary infections, bronchiectasis, and poor growth. The boy in the vignette
has an oxygen saturation that is strikingly lower than what is expected by symptomatology,
which would be unusual in infantile cystic fibrosis.

Suggested reading:

American Academy of Pediatrics. Pneumocystis jiroveci infections. In: Pickering LK, Baker CJ,
Kimberlin DW, Long SS, eds. Red Book: 2009 Report of the Committee on Infectious Diseases.
28th ed. Elk Grove Village, Ill: American Academy of Pediatrics; 2009:536-540

Copyright 2010 by the American Academy of Pediatrics page 142


2011 PREP SA on CD-ROM

Hughes WT. Pneumocystis pneumonia. In: Feigin RD, Cherry JD, Demmler-Harrison GJ, Kaplan
SL, eds. Feigin & Cherrys Textbook of Pediatric Infectious Diseases. 6th ed. Philadelphia, Pa:
Saunders Elsevier; 2009:2971-2980

Kovacs JA, Masur H. Evolving health effects of Pneumocystis: 100 years of progress in
diagnosis and treatment. JAMA. 2009;301:2578-2585. Abstract available at:
http://www.ncbi.nlm.nih.gov/pubmed/19549975

Copyright 2010 by the American Academy of Pediatrics page 143


2011 PREP SA on CD-ROM

Question: 46

An 8-year-old African American boy presents with gross hematuria. His mother describes his
urine as bright red with clots. He has had no trauma or recent infections. His 4-year-old brother
has sickle cell disease. On physical examination, this boys temperature is 37.3C, heart rate is 76
beats/min, respiratory rate is 16 breaths/min, and blood pressure is 110/64 mm Hg. He has no
costovertebral tenderness, bruising, or edema.

Results of urinalysis include:


Specific gravity, 1.010
pH, 7
3+ blood
1+ protein
Negative nitrite
Negative leukocyte esterase
More than 100 red blood cells/high-power field (hpf)
Fewer than 5 white blood cells/hpf

Other laboratory results include:


Blood urea nitrogen, 14 mg/dL (5.0 mmol/L)
Creatinine, 0.5 mg/dL (44.2 mcmol/L)
White blood cell count, 6.5x10 3/mcL (6.5x109/L)
Hemoglobin, 12.0 g/dL (120 g/L)
Platelet count, 405x10 3/mcL (405x109/L)

Of the following, the MOST appropriate next diagnostic test is

A. complement component 3 assessment

B. hemoglobin electrophoresis

C. prothrombin time

D. urine calcium and urine creatinine assessment

E. urine culture

Copyright 2010 by the American Academy of Pediatrics page 144


2011 PREP SA on CD-ROM

Critique: 46 Preferred Response: B

The differential diagnosis for a child who has gross hematuria must be separated into an
upper tract (kidney) or a lower tract (ureters, bladder, urethra) origin. Patients who have classic
upper tract disease have cola-colored urine without clots. In contrast, those who have lower
tract disease may have bright red urine with or without clots and sometimes with localizing
symptoms such as flank pain, frequency, or dysuria.
The child described in the vignette has painless, gross hematuria with clots, no history of
trauma, and a positive family history of sickle cell disease in a sibling. Findings on the physical
examination are negative, and the patient is normotensive. Preliminary laboratory evaluation
shows a urine specific gravity of 1.010 with hematuria and proteinuria, normal renal function,
and normal complete blood count. The term used to describe a situation in which the urine is
neither dilute nor concentrated relative to serum (eg, urine specific gravity of 1.010 and urine
osmolality of 300 mOsm/L) is isosthenuria. The combination of the family history, the absence of
symptoms, and isosthenuria makes sickle cell trait most likely. Furthermore, because this child
has a normal hemoglobin value and is phenotypically "normal," he either has sickle cell trait (67%
likelihood) or is genotypically normal (33% likelihood) at the sickle cell gene locus. Hemoglobin
electrophoresis can confirm the diagnosis.
Complement component 3 assessment is useful for diagnosing glomerulonephritis. Urine
calcium and urine creatinine frequently are assessed in the child who has visible blood in the
urine as a screen for stone disease (nephrolithiasis or urolithiasis). However, isolated
hypercalciuria (eg, without associated renal stones) as a cause of gross hematuria is rare. A
bleeding diathesis, which is assessed via prothrombin time, rarely presents with isolated
hematuria without associated bruising or bleeding from the gums. Similarly, hemorrhagic cystitis
can present with gross hematuria, but the clinical presentation of this patient and the absence of
urgency, frequency, and dysuria make this unlikely. Furthermore, a bacterial cause for
hemorrhagic cystitis is very unlikely in a male child.

As a result of reviewing this information, do you intend to make a change in practice


to provide better patient care?
Yes No

Suggested reading:

Ataga KI, Orringer EP. Renal abnormalities in sickle cell disease. Am J Hematol. 2000;63:205-
211. Abstract available at: http://www.ncbi.nlm.nih.gov/pubmed/10706765

Bruno D, Wigfall DR, Zimmerman SA, Rosoff PM, Wiener JS. Genitourinary complications of sickle
cell disease. J Urol. 2001;166:803-811. Abstract available at:
http://www.ncbi.nlm.nih.gov/pubmed/11490223

Driscoll MC. Sickle cell disease. Pediatr Rev. 2007;28:259-268. DOI: 10.1542/10.1542/pir.28-7-
259. Available at: http://pedsinreview.aappublications.org/cgi/content/full/28/7/259

Copyright 2010 by the American Academy of Pediatrics page 145


2011 PREP SA on CD-ROM

Pham PT, Pham PC, Wilkinson AH, Lew SQ. Renal abnormalities in sickle cell disease. Kidney Int.
2000;57:1-8. DOI: 10.1046/j.1523-1755.2000.00806.x. Available at:
http://www.nature.com/ki/journal/v57/n1/full/4491289a.html

Scheinman JI. Sickle cell disease and the kidney. Nat Clin Pract Nephrol. 2009;5:78-88. DOI:
10.1038/ncpneph1008. Abstract available at: http://www.ncbi.nlm.nih.gov/pubmed/19048000

Copyright 2010 by the American Academy of Pediatrics page 146


2011 PREP SA on CD-ROM

Question: 47

A 12-year-old boy presents with a 4-year history of sneezing, nasal pruritus, nasal congestion,
rhinorrhea, and ocular pruritus. His symptoms seem to resolve during colder months but worsen
in the spring and fall. His parents also are concerned that he complains of oral itching after eating
bananas. Oral antihistamines have provided some symptomatic relief, but he still has symptoms
on most days during these months.

Of the following, the MOST likely reason for this boys symptoms is

A. chronic sinusitis

B. nonallergic rhinitis

C. perennial allergic rhinitis

D. recurrent upper respiratory tract infections

E. seasonal allergic rhinitis

Copyright 2010 by the American Academy of Pediatrics page 147


2011 PREP SA on CD-ROM

Critique: 47 Preferred Response: E

The boy described in the vignette has classic symptoms of seasonal allergic rhinitis.
Symptoms occurring in spring and fall likely represent tree and weed pollen, respectively.
Interestingly, 10% to 40% of patients who have seasonal allergic rhinitis also have oral allergy
syndrome, as seen in this child. Certain raw fruits and vegetables contain heat-labile proteins
that are similar to seasonal allergic proteins. When ingested, they cause oral pruritus that
generally is self-limited, resolving within a few minutes.
Other conditions mimicking seasonal allergic rhinitis are chronic sinusitis, nonallergic rhinitis,
and viral upper respiratory tract infections (URIs). Chronic sinusitis can complicate allergic
rhinitis but generally has other clinical features such as headache, facial pain, and discolored
rhinorrhea, which this boy does not have. The symptoms of nonallergic rhinitis are similar to
those of allergic rhinitis, but nonallergic rhinitis is more common in adults and involves triggers
such as cold air, tobacco smoke, perfumes, colognes, strong odors, and spicy food. Allergy skin
prick or immunoglobulin E serum testing often is needed to distinguish allergic from nonallergic
rhinitis.
Viral URIs can cause sneezing, rhinorrhea, and nasal congestion, but they generally do not
result in the ocular pruritus described in the vignette. Also, the duration of a viral URI is brief (ie,
a few days to a couple of weeks) rather than consistently for months. Perennial allergic rhinitis
can be a distinct diagnosis or occur in conjunction with seasonal allergic rhinitis. Triggers for
perennial symptoms include house dust mite, cockroach, and pet dander, although some children
who live in temperate climates may have perennial rhinitis symptoms due to year-round grass.
Because three to four seasons of exposure generally are required to develop seasonal
allergic rhinitis, the onset of symptoms usually peaks between the ages of 4 and 10 years.
Symptoms occurring within the first year after birth should prompt consideration of recurrent
viral URIs, chronic sinusitis, and perennial allergic rhinitis. In contrast to aeroallergen
sensitization, immunoglobulin E-mediated food allergy to common food triggers such as milk, egg,
wheat, soy, or peanut usually occurs before age 4 years.

Suggested reading:

Tatachar P, Kumar S. Food-induced anaphylaxis and oral allergy syndrome. Pediatr Rev.
2008;29:e23-e27. DOI: 10.1542/10.1542/pir.29-4-e23. Available at:
http://pedsinreview.aappublications.org/cgi/content/full/29/4/e23

Taylor A, Adam HM. In brief: sinusitis. Pediatr Rev. 2006;27:395-397. DOI: 10.1542/10.1542/pir.27-
10-395. Available at: http://pedsinreview.aappublications.org/cgi/content/full/27/10/395

Copyright 2010 by the American Academy of Pediatrics page 148


2011 PREP SA on CD-ROM

Question: 48

A 16-year-old boy is brought to the emergency department by friends because he started


screaming uncontrollably at a party. On physical examination, he is agitated, talking to himself, and
appears to be hallucinating. His temperature is 38.1C, heart rate is 135 beats/min, respiratory
rate is 28 breaths/min, and blood pressure is 140/92 mm Hg. No other findings on the evaluation
are of note. Although you place the boy in a quiet room, his agitation continues.

Of the following, the MOST appropriate next step is to

A. administer activated charcoal

B. administer labetalol

C. administer lorazepam

D. begin external cooling

E. place the patient in physical restraints

Copyright 2010 by the American Academy of Pediatrics page 149


2011 PREP SA on CD-ROM

Critique: 48 Preferred Response: C

The boy described in the vignette is exhibiting the signs and symptoms of acute psychosis,
most likely due to hallucinogen intoxication. Although the differential diagnosis for new-onset
acute psychosis in adolescents is broad, the temporal relationship of his symptoms to his
presence at a party is highly suggestive of a drug-related condition. Commonly abused
hallucinogens include lysergic acid diethylamide (LSD), mescaline, phencyclidine,
dextromethorphan, and Salvia divinorum. Use and abuse of hallucinogens are associated with
the development of acute psychosis as well as the other signs the boy is exhibiting, such as
agitation, tachycardia, hypertension, and hyperthermia. Among the other behavioral effects are
synesthesia (sense of "seeing" sounds or "hearing" colors), time distortion, out-of-body
sensation, and euphoria or dysphoria. Physiologic addiction does not occur with most
hallucinogens; rather, the abuse potential is related largely to the euphoric effects of the drugs.
Management of hallucinogen exposure is largely supportive. For most patients,
hypertension, tachycardia, and mild hyperthermia are the result of psychomotor agitation, so
specific treatment for these symptoms with external cooling or labetalol is not indicated. Agitation
should be treated by initiating calming measures, placing the patient in a low-stimulation
environment, and administering benzodiazepines rather than with physical restraints, which are
likely to increase agitation. Activated charcoal rarely is indicated because hallucinogen
absorption is rapid and symptoms are self-limited. If, however, the patients vital signs are
severely abnormal or do not improve with sedation, other more serious causes should be
considered, including serotonin syndrome, neuroleptic malignant syndrome, and barbiturate
withdrawal.

Suggested reading:

Babu KM, Boyer EW. Emergency department evaluation of acute onset psychosis in children.
UpToDate Online 17.3. 2009. Available at:
http://www.utdol.com/online/content/topic.do?topicKey=ped_symp/13395&selectedTitle=3~11&s
ource=search_result

Delgado J. Intoxication from LSD and other common hallucinogens. UpToDate Online 17.3. 2009.
Available at:
http://www.utdol.com/online/content/topic.do?topicKey=ad_tox/19168&selectedTitle=1~11&sour
ce=search_result

Greene JP, Ahrendt D, Stafford EM. Adolescent abuse of other drugs. Adolesc Med Clin.
2006;17:283-318. DOI: 10.1016/j.admecli.2006.03.007. Abstract available at:
http://www.ncbi.nlm.nih.gov/pubmed/16814695

Jenkins RR, Adger H. Substance abuse. In: Kleigman RM, Behrman RE, Jenson HB, Stanton BF,
eds. Nelson Textbook of Pediatrics. 18th ed. Philadelphia, Pa: Saunders Elsevier; 2007:824-833

Copyright 2010 by the American Academy of Pediatrics page 150


2011 PREP SA on CD-ROM

Question: 49

A previously healthy 4-year-old boy presents with a 1-week history of intermittent, crampy
abdominal pain; watery diarrhea; and a 0.5-kg weight loss. His symptoms started after the family
returned from a day of apple picking at a local farm-based orchard. He is receiving no
medications.

Of the following, the MOST likely cause of his symptoms is

A. Blastocystis hominis

B. Clostridium difficile

C. enteropathogenic Escherichia coli

D. rotavirus

E. Yersinia enterocolitica

Copyright 2010 by the American Academy of Pediatrics page 151


2011 PREP SA on CD-ROM

Critique: 49 Preferred Response: C

Although rotavirus is the most common cause of diarrheal illness in infants and toddlers,
foodborne bacterial pathogens remain an important reservoir of infection. The 1 week of watery
diarrhea that began after a visit to an apple orchard described for the boy in the vignette most
likely is caused by enteropathogenic Escherichia coli (EPEC). This rod-shaped, gram-negative
bacterium has been isolated from unfiltered, unpasteurized apple juice.
At least five E coli pathotypes have been associated with diarrheal illness in children. Each
possesses a distinct set of somatic (O) and flagellar (H) antigens (Item C49). Unfortunately, most
laboratories are equipped to identify only E coli O157:H7 (STEC), a Shiga toxin-producing strain
associated with hemolytic-uremic syndrome in all age groups and with postinfectious thrombotic
thrombocytopenic purpura in adults.
Similar to most other gastrointestinal pathogens, many reservoirs of EPEC infection have
been identified. These include animals, symptomatic or asymptomatic infants or children, and
asymptomatic adult carriers. Transmission probably occurs via the fecal-oral route. Accordingly,
close contact with animals or individuals harboring the organism, either as commensal flora or as
a pathogen during diarrheal illness; sharing of food items; and food itself can be sources of
infection. Because of its wide carriage in food items (one recent study identified 19 EPEC
isolates out of 400 E coli isolates found in a variety of foods), proper hygiene in food handling
and processing is essential to prevent infection. Although widespread diarrheal disease caused
by EPEC is a problem confined largely to the developing world, outbreaks in nurseries resulting in
severe chronic diarrhea have been reported in the United States. Symptomatic infection in the
United States, in which EPEC has been identified as the offending organism, generally is
confined to children younger than 5 years of age.
Blastocystis hominis is a commensal, protozoal organism commonly isolated from the stools
of asymptomatic individuals. Its role in diarrheal illness is controversial. Symptomatic Clostridium
difficile infection most frequently occurs in the setting of antibiotic-associated diarrhea and most
often presents with hematochezia. This organism is not considered a foodborne pathogen.
Rotavirus also is not transmitted predominantly via food items, and a 1-week duration of illness in
a 4-year-old would be unusual. Yersinia enterocolitica is a known foodborne pathogen,
especially associated with the consumption of chitterlings in the southern United States.
However, the incidence of disease caused by this organism is very low in young children in the
United States (estimated at 1.4 per 100,000). In older children, Y enterocolitica typically causes
a pseudoappendicitis syndrome; younger children may present with fever and mucoid, bloody
diarrhea.

Suggested reading:

Afset JE, Bevanger L, Romundstad P, Bergh K. Association of atypical enteropathogenic


Escherichia coli (EPEC) with prolonged diarrhoea. J Med Microbiol. 2004;53:1137-1144. DOI:
10.1099/jmm.0.45719-0. Available at: http://jmm.sgmjournals.org/cgi/content/full/53/11/1137

American Academy of Pediatrics. Escherichia coli diarrhea (including hemolytic-uremic


syndrome). In: Pickering LK, Baker CJ, Kimberlin DW, Long SS, eds. Red Book: 2009 Report of

Copyright 2010 by the American Academy of Pediatrics page 152


2011 PREP SA on CD-ROM

the Committee on Infectious Diseases. 28th ed. Elk Grove Village, Ill: American Academy of
Pediatrics; 2009:294-298

Cohen MB, Nataro JP, Bernstein DI, Hawkins J, Roberts N, Staat MA. Prevalence of diarrheagenic
Escherichia coli in acute childhood enteritis: a prospective controlled study. J Pediatr.
2005;146:54-61. DOI: 10.1016/j.jpeds.2004.08.059. Abstract available at:
http://www.ncbi.nlm.nih.gov/pubmed/15644823

Ochoa TJ, Salazar-Lindo E, Cleary TG. Management of children with infection-associated


persistent diarrhea. Semin Pediatr Infect Dis. 2004;15:229-236. DOI: 10.1053/j.spid.2. Abstract
available at: http://www.ncbi.nlm.nih.gov/pubmed/15494946

Pawlowski SW, Warren CA, Guerrant R. Diagnosis and treatment of acute or persistent diarrhea.
Gastroenterology. 2009;136: 1874-1886. DOI: 10.1053/j.gastro.2009.02.072. Abstract available
at: http://www.ncbi.nlm.nih.gov/pubmed/19457416

Copyright 2010 by the American Academy of Pediatrics page 153


2011 PREP SA on CD-ROM

Critique: 49

Copyright 2010 by the American Academy of Pediatrics page 154


2011 PREP SA on CD-ROM

Question: 50

A 27 weeks gestation preterm male infant who weighs 900 g is delivered at a community hospital
by emergent cesarean section due to abruptio placentae. After intubation in the delivery room, he
is taken to the nursery for stabilization, including umbilical venous line placement, prior to transfer
to a tertiary center.

Of the following, the MOST appropriate initial solution for parenteral administration should include

A. 5% dextrose

B. 5% dextrose and 0.2% sodium chloride

C. 10% dextrose

D. 10% dextrose and 0.2% sodium chloride

E. 0.9% sodium chloride

Copyright 2010 by the American Academy of Pediatrics page 155


2011 PREP SA on CD-ROM

Critique: 50 Preferred Response: C

After the resuscitation and stabilization of a very low-birthweight (VLBW) infant


(birthweight <1,500 g), the initial assessment should include the measurement of blood glucose
and the initiation of parenteral fluids with 10% dextrose solution. The VLBW infant requires a
constant glucose infusion to prevent hypoglycemia due to reduced endogenous glycogen and
fat stores and a limited ability to perform gluconeogenesis. The goal should be to maintain the
blood glucose concentration greater than 50 mg/dL (2.8 mmol/L) in the first 24 hours after birth.
This value is consistent with the normal fetal glucose concentration in the second half of
gestation, which is not less than 54 mg/dL (3.0 mmol/L).
Administration of 10% dextrose solution without electrolytes should be initiated at a rate of
80 mL/kg per day in the VLBW infant, providing a glucose infusion rate of 5.5 mg/kg per minute.
This supports the glucose requirement for VLBW infants, which generally is 4 to 6 mg/kg per
minute. For the preterm infant, the blood glucose values must be monitored closely to avoid either
hypoglycemia or hyperglycemia. Hyperglycemia is defined as a blood glucose concentration
greater than 120 mg/dL (6.7 mmol/L). Some preterm infants manifest hyperglycemia due to the
high infusion rates needed to support insensible fluid losses or endogenous glucose production
in response to stress-reactive hormones such as epinephrine.
The initial parenteral fluids for VLBW infants do not need to contain electrolytes. With the
exception of calcium, electrolytes rarely are needed until after the first 24 hours after birth.
Administering parenteral fluids with 5% dextrose puts the VLBW infant at risk for hypoglycemia
because this provides a glucose infusion rate of only 2.8 mg/kg per minute when run at 80 mL/kg
per day.

Suggested reading:

American Academy of Pediatrics. Sugar and safe care. In: Karlsen KA, ed. The S.T.A.B.L.E.
Program: Post Resuscitation/Pre-Transport Stabilization of Sick Infants: Guidelines for
Neonatal Healthcare Providers. 5th ed. Elk Grove Village, Ill: American Academy of Pediatrics;
2006:5-42

Angert R, Adam HM. In brief: care of the very low-birthweight infant. Pediatr Rev. 2009;30:32-
35. DOI: 10.1542/10.1542/pir.30-1-32. Available at:
http://pedsinreview.aappublications.org/cgi/content/full/30/1/32

Hay WW Jr. Early postnatal nutritional requirements of the very preterm infant based on a
presentation at the NICHD-AAP workshop on research in neonatology. J Perinatol.
2006;26(suppl 2):s13-s18. DOI: 10.1038/sj.jp.7211426. Abstract available at:
http://www.ncbi.nlm.nih.gov/pubmed/16801962

Copyright 2010 by the American Academy of Pediatrics page 156


2011 PREP SA on CD-ROM

Question: 51

A 10-year-old boy has stopped playing soccer because of heel pain of 3 weeks duration. He
now sometimes walks on his toes to avoid pressure over his posterior feet. He cannot recall any
specific traumatic event, has no other areas of pain, and otherwise feels well. Physical
examination reveals no swelling, redness, bruising, or skin lesions. He has full range of motion in
his hips, knees, ankles, and toes. However, he has significant pain with compression
("squeezing") medially and laterally over the calcaneus.

Of the following, the MOST appropriate recommendation for this child is

A. heel stretching exercises

B. magnetic resonance imaging of the calcaneus

C. non-weight-bearing activities for the next 6 months

D. referral to a rheumatologist

E. referral to an orthopedic surgeon

Copyright 2010 by the American Academy of Pediatrics page 157


2011 PREP SA on CD-ROM

Critique: 51 Preferred Response: A

The boy described in the vignette has typical findings of Sever disease, which often is
referred to as calcaneal apophysitis, although it is not an inflammatory condition but rather an
overuse syndrome. It occurs in children 8 to 13 years of age, affects boys twice as often as
girls, and frequently occurs in athletes, particularly those participating in soccer, basketball,
gymnastics, track, or running. The patient complains of pain over the heel, often bilaterally, that is
worse with running and may toe-walk to avoid putting pressure on the heel. Physical
examination reveals no redness, heat, swelling, or deformity. Occasionally, the child has a tight
Achilles tendon. The most specific finding is a positive calcaneal compression test, with pain
produced by medial and lateral compression of the heel.
Sever disease is a clinical diagnosis; radiographs or further studies are indicated only if
additional diagnoses are being considered. In the past, many practitioners believed that
fragmentation of the calcaneal apophysis on plain radiographs was a specific finding for Sever
disease. However, further studies have shown that such fragmentation is seen frequently in
children who had radiographs for reasons other than heel pain. A research study of magnetic
resonance imaging (MRI) in Sever disease showed that patients had bone bruising (increased
signal intensity and edema) within the metaphyseal portion of the calcaneus. Signal intensity
was increased within the apophysis, but it did not correspond to the fissures seen on plain
radiographs. With resolution of symptoms, signal intensity in the metaphysis decreased,
confirming that Sever disease is the result of microtrauma. Clinically, MRI is indicated only for
patients for whom the diagnosis is uncertain or who fail to respond to conservative treatment.
Treatment for Sever disease is supportive and may include stretching and heat. Rest,
although often recommended, is not mandatory, and if used, should only be for brief periods.
Nonsteroidal anti-inflammatory drugs may be used but are not consistently effective, again
reflecting the noninflammatory nature of the condition. Appropriately supportive shoes with arch
support to prevent foot pronation should be recommended. Heel cups or 1/4- to 5/8-inch heel lifts
may relax the insertion of the Achilles tendon, thus relieving pain. The condition and pain usually
resolve in 6 weeks to 3 months.
Rheumatologic evaluation is not indicated for patients who have typical findings of Sever
disease but should be considered for those who have findings of arthritis or inflammation.
Surgical intervention is not indicated for this self-limited condition.

Suggested reading:

Crawford AH, Al-Sayyad MJ, Mehlman CT. Fractures and dislocations of the foot and ankle. In:
Green NE, Swiontkowski MF, eds. Skeletal Trauma in Children. 4th ed. Philadelphia, Pa:
Elsevier; 2008: Chapter 16

Ogden JA, Ganey TM, Hill JD, Jaakkola JI. Severs injury: a stress fracture of the immature
calcaneal metaphysic. J Pediatr Orthop. 2004;24:488-492. Abstract available at:
http://www.ncbi.nlm.nih.gov/pubmed/15308897

Volpon JB, de Carvalho Filho G. Calcaneal apophysitis: a quantitative radiographic evaluation of

Copyright 2010 by the American Academy of Pediatrics page 158


2011 PREP SA on CD-ROM

the secondary ossification center. Arch Orthop Trauma Surg. 2002;122:338-341. DOI:
10.1007/s00402-002-0410-y. Abstract available at:
http://www.ncbi.nlm.nih.gov/pubmed/12136298

Weiner DS, Morscher M, Dicintio MS. Calcaneal apophysitis: simple diagnosis, simpler treatment.
J Fam Pract. 2007;56:352-355. Available at: http://www.jfponline.com/Pages.asp?AID=4993

Copyright 2010 by the American Academy of Pediatrics page 159


2011 PREP SA on CD-ROM

Question: 52

A grandmother brings in her 9-year-old grandson because she is concerned that he is behaving
aggressively toward his siblings and cousins and recently destroyed some property. He is
exhibiting aggressive behavior at school, with the teacher reporting that he is very hyperactive
and distractible. In addition, his grades have fallen over the past year. The child has been placed
in the grandmothers guardianship voluntarily by his mother while she seeks employment and
divorce after years of domestic violence. The grandmother states that she has tried a variety of
discipline techniques, including spankings, which seem to make things worse. She asks your
opinion about this boys prognosis and your recommendations for intervention.

Of the following, the MOST appropriate next step in the management of this childs behavioral
problems is

A. administration of an atypical antipsychotic medication

B. emergent evaluation for possible child sexual abuse at a specialty center

C. prescription for stimulant medication

D. referral for counseling that includes anger management strategies

E. transfer of the child to a new school

Copyright 2010 by the American Academy of Pediatrics page 160


2011 PREP SA on CD-ROM

Critique: 52 Preferred Response: D

The child described in the vignette has many risk factors for aggressive or violent behavior.
Early identification and intervention for children who exhibit aggressive behaviors may help
prevent the development of violent behaviors. Accordingly, the most appropriate course of action
is referral for counseling to develop anger management strategies.
The risk factors for violence or aggression for the child described in the vignette include
exposure to domestic violence, corporal punishment, divorce, and maternal unemployment. Many
of these factors may result in the child developing the perception that the world is generally
hostile. Referral for further evaluation and counseling that includes anger management,
therapeutic interventions for school and home, and behavior modification is most likely to result in
long-term improvement. Among suggested strategies are school-based counseling interventions,
peer group therapies, structured after-school activities, and home and family-based counseling.
These strategies may provide the child with opportunities for conflict resolution. Communities
differ in services provided; most have parenting classes to help parents and grandparents with
behavior modification and discipline techniques. In the United Kingdom, successful strategies
have included intensive home and family-based efforts.
Empiric prescription of psychoactive medications is not appropriate for this child who has
not been appropriately evaluated for a psychiatric disorder. However, a complete physical
examination, input from the school in the form of Vanderbilt questionnaires for attention-
deficit/hyperactivity disorder and comorbidities, parental input via questionnaire, and office
evaluation of the child for depression are rational steps for a pediatrician to undertake to
determine if psychoactive medications may be needed. Transfer to a new school to escape the
difficulties at the current school would represent one more stressor and change in the boys life.
In addition, the current school teacher and counselor may have some beneficial insights into his
behaviors, which can assist in diagnosis and treatment of his aggressive behavior. Also, a
stable school environment may enhance this boys academic achievement, and children who are
successful academically are less likely to engage in violent behavior. Furthermore, enhanced
early peer relationships are relatively protective against violent behavior. Although it is
appropriate to question this boy regarding possible sexual abuse, unless there are physical
examination findings or history to suggest the possibility of sexual abuse, referral for evaluation
is unhelpful and may have the unintended consequence of creating more stress.

Suggested reading:

Committee on Injury, Violence, and Poison Prevention. Role of the pediatrician in youth violence
prevention. Pediatrics. 2009;124:393-402. DOI: 10.1542/peds.2009-0943. Available at:
http://pediatrics.aappublications.org/cgi/content/full/124/1/393

Cowden JD, Smith S, Pyle S, Dowd MD. Connected kids at Head Start: taking office-based
violence prevention to the community. Pediatrics. 2009;124:1094-1099. DOI: 10.1542/peds.2009-
0312. Available at: http://pediatrics.aappublications.org/cgi/content/full/124/4/1094

Cowell K, Horstmann S, Linebarger J, Meaker P, Aligne CA. Pediatrics in the community: a

Copyright 2010 by the American Academy of Pediatrics page 161


2011 PREP SA on CD-ROM

"vaccine" against violence: coping power. Pediatr Rev. 2008;29:362-363. DOI:


10.1542/10.1542/pir.29-10-362. Available at:
http://pedsinreview.aappublications.org/cgi/content/full/29/10/362

Gini G, Pozzoli T. Association between bullying and psychosomatic problems: a meta-analysis.


Pediatrics. 2009;123:1059-1065. DOI: 10.1542/peds.2008-1215. Available at:
http://pediatrics.aappublications.org/cgi/content/full/123/3/1059

Shepperd S, Doll H, Gowers S, et al. Alternatives to inpatient mental health care for children and
young people. Cochrane Database Syst Rev. 2009;2:CD006410. DOI:
10.1002/14651858.CD006410.pub2. Available at:
http://www.mrw.interscience.wiley.com/cochrane/clsysrev/articles/CD006410/frame.html

Wagman Borowsky I, Mozayeny S, Stuenkel K, Ireland M. Effects of a primary care-based


intervention on violent behavior and injury in children. Pediatrics. 2004;114:e392-e399. DOI:
10.1542/peds.2004-0693. Available at:
http://pediatrics.aappublications.org/cgi/content/full/114/4/e392

Ybarra ML, Diener-West M, Markow D, Leaf PJ, Hamburger M, Boxer P. Linkages between
Internet and other media violence with seriously violent behavior by youth. Pediatrics.
2008;122:929-937. DOI: 10.1542/peds.2007-3377. Available at:
http://pediatrics.aappublications.org/cgi/content/full/122/5/929

Copyright 2010 by the American Academy of Pediatrics page 162


2011 PREP SA on CD-ROM

Question: 53

You are evaluating a 2-year-old child at a health supervision visit. His mother states that he has a
chronic cough and occasional difficulty with swallowing of solid foods, but she has not seen any
problems with liquids. Physical examination of the 15-kg child reveals no findings of note. Barium
swallow test reveals a posterior indentation of the esophagus, suggesting the possibility of a
vascular ring (Item Q53).

Of the following, the MOST appropriate test to confirm the diagnosis is

A. chest radiography

B. computed tomography angiography

C. echocardiography

D. esophageal endoscopy

E. upper gastrointestinal radiographic series

Copyright 2010 by the American Academy of Pediatrics page 163


2011 PREP SA on CD-ROM

Question: 53

(Courtesy of B Carter)
Barium swallow test, as described for the child in the vignette, revealing an indentation on the
posterior wall of the esophagus (arrow).

Copyright 2010 by the American Academy of Pediatrics page 164


2011 PREP SA on CD-ROM

Critique: 53 Preferred Response: B

The possibility of a vascular ring must be considered in any young child who has findings of
coughing, stridor, and gagging with associated difficulty in swallowing solids. However, feeding
disorders in children may have many different causes, necessitating diagnostic testing in many
cases. A barium swallow study is used to assess esophageal mechanics and to evaluate for
gastroesophageal reflux and associated microaspiration. The finding of a posterior indentation of
the esophagus in the region of the mediastinum, as described for the child in the vignette, is
highly suggestive of vascular compression phenomena. Most patients in whom this condition is
suspected have vascular rings. A vascular ring is a general term applied to a variety of vascular
anomalies involving aortic arch branching. Vascular rings are formed when an aortic arch
anomaly, with or without a patent ductus arteriosus, produces a ring that completely encircles
the trachea and esophagus, leading to symptoms of tracheal and esophageal constriction. The
two most common forms of a vascular ring are: 1) a right aortic arch with an abnormal origin of
the left subclavian artery from the proximal descending aorta and either a patent ductus
arteriosus or a ligamentum arteriosus completing the ring, and 2) a double aortic arch caused by
persistence of both paired embryonic arches, thus encircling the trachea and esophagus and
entrapping these structures (Item C53A).
After a barium swallow allows a presumptive diagnosis, numerous methods can be
undertaken for definitive identification of the nature of the vascular ring. The most common is
computed tomography angiography. Using contrast to enhance viewing of the vascular
structures, the three-dimensional relationships between the aortic vessels, the trachea, and the
esophagus can be established (Item C53B). The compressive effect of the high-pressure
vascular structure as it impinges upon the distensible trachea and esophagus can be
determined. Complex vascular malformations can be mapped, rarely requiring sedation due to the
rapid acquisition of imaging sequences using the most modern pediatric imaging tools.
Echocardiography is an important part of the evaluation and often can provide a definitive
diagnosis, but the vascular components forming the ring can be very small or atretic, making
them difficult or impossible to define and requiring the use of additional imaging modalities. For
many years, chest radiography and esophageal endoscopy were the primary diagnostic
modalities used to confirm or exclude vascular rings in patients who had compatible histories.
Although these tests offer useful information, they cannot provide a definitive diagnosis. Cardiac
catheterization with angiography can define the cardiovascular anatomy. Although angiography
does not image the esophagus and trachea, it can define the relationships in space and provide
definitive evidence of the lesions consistent with a physiologically important vascular ring.
However, the invasive nature of this test precludes its use as the standard approach to
diagnosis. A complete upper gastrointestinal radiographic series offers no additional information
beyond that provided by the barium swallow and, therefore, is not indicated.

Suggested reading:

Hernanz-Schulman M. Vascular rings: a practical approach to imaging diagnosis. Pediatr Radiol.


2005;35:961-979. DOI: 10.1007/s00247-005-1529-0. Abstract available at:
http://www.ncbi.nlm.nih.gov/pubmed/16052335

Copyright 2010 by the American Academy of Pediatrics page 165


2011 PREP SA on CD-ROM

Humphrey C, Duncan K, Fletcher S. Decade of experience with vascular rings at a single


institution. Pediatrics. 2006;117:e903-e908. DOI: 10.1542/peds.2005-1674. Available at:
http://pediatrics.aappublications.org/cgi/content/full/117/5/e903

Copyright 2010 by the American Academy of Pediatrics page 166


2011 PREP SA on CD-ROM

Critique: 53

(Courtesy of A Johnson)
In a double aortic arch, persistent paired embryonic arches encircle the trachea and esophagus.
RSA=right subclavian artery, RCCA=right common carotid artery, LCCA=left common carotid
artery, LSA=left subclavian artery.

Copyright 2010 by the American Academy of Pediatrics page 167


2011 PREP SA on CD-ROM

Critique: 53

(Courtesy of D Krowchuk)
Posterior view from a computed tomography angiographic study of the chest with three-
dimensional reformatted imaging reveals a double aortic arch, with the right being smaller than the
left. H=head, L=left, R=right, F=foot, P=posterior view.

Copyright 2010 by the American Academy of Pediatrics page 168


2011 PREP SA on CD-ROM

Question: 54

A 4-year-old boy presents with recurrent otitis media (OM) with persistent effusion. He is
otherwise healthy. His mother is concerned about possible hearing loss and also wants to know
if her son is at risk for any other neurologic complications because of his recurrent OM.

Of the following, the MOST likely neurologic complication for which this boy is at risk is

A. autistic spectrum disorders

B. balance difficulties

C. nystagmus

D. speech apraxia

E. tic disorders

Copyright 2010 by the American Academy of Pediatrics page 169


2011 PREP SA on CD-ROM

Critique: 54 Preferred Response: B

The boy described in the vignette has a persistent middle ear effusion after recurrent bouts
of otitis media. In addition to dampened conductance of sound to the inner ear, balance problems
may occur, due to differences in pressure in the middle ear. However, "balance problems" in a 4-
year-old also can be due to serious diseases affecting the brainstem and cerebellum. Therefore,
any toddler who presents with such problems warrants a careful neurologic examination.
Autistic spectrum disorders involve impairments in language and communication. Although
chronic conductive hearing loss can affect speech articulation, the other problems in autism with
communication, social skills, and need for sameness do not occur. Similarly, speech apraxia, a
neurologic problem affecting the childs ability to coordinate production of sounds to produce
words, is not related to conductive hearing loss.
Nystagmus (ie, rhythmic oscillations of the eyes) is an important neurologic finding that
should be sought in any person who has balance problems or subjective dizziness because it
can indicate serious disease. Nystagmus may be peripheral, due to pathology in the inner ear or
vestibulocochlear nerve, or central, due to pathology in the brainstem or cerebellum. Middle ear
effusions do not produce nystagmus. Most consultations for "dizziness" do not reveal any
nystagmus, and neuroimaging, although often ordered, rarely is needed. Balance problems
accompanied by nystagmus require urgent neurologic evaluation in the emergency department, if
necessary. The differential diagnosis is very large, but some immediate considerations include
toxins/ingestions, acute cerebellar ataxia, acute disseminated encephalomyelitis, opsoclonus
myoclonus ataxia syndrome, cerebellar strokes, and posterior fossa tumors.
Tics are repetitive, nonrhythmic, patterned movements or sounds produced involuntarily or
in response to premonitory urges. They are neurologic symptoms and are not produced directly
by infections or middle ear effusions. Upper respiratory tract infections may act as a precipitant
for tics in susceptible children.

Suggested reading:

Fenichel GM. Ataxia. Clinical Pediatric Neurology: A Signs and Symptoms Approach. 6th ed.
Philadelphia, Pa: Saunders Elsevier; 2009:227-248

Gilbert DL. Ataxia. In: Singer HS, Kossoff EH, Hartman AL, Crawford TO, eds. Treatment of
Pediatric Neurologic Disorders. Boca Raton, Fla: Informa Healthcare; 2005:415-422

Pelton SI. Otitis media: re-evaluation of diagnosis and treatment in the era of antimicrobial
resistance, pneumococcal conjugate vaccine, and evolving morbidity. Pediatr Clin North Am.
2005;52:711-728. DOI: 10.1016/j.pcl.2005.02.013. Abstract available at:
http://www.ncbi.nlm.nih.gov/pubmed/15925659

Phillips JO, Backous DD. Evaluation of vestibular function in young children. Otolaryngol Clin
North Am. 2002;35:765-790. Abstract available at:
http://www.ncbi.nlm.nih.gov/pubmed/12487080

Copyright 2010 by the American Academy of Pediatrics page 170


2011 PREP SA on CD-ROM

Question: 55

You are seeing a 3-year-old boy for a health supervision visit. His mother is pregnant, and when
you ask how she is feeling, she bursts into tears. She tells you that recent prenatal
ultrasonography has revealed multiple physical anomalies in their unborn child. She has days
when she is hopeful and days when she is terrified; her husband has buried himself in his work.

Of the following, the approach that is MOST likely to address this couples concerns is

A. encouraging their extended family to be supportive

B. providing specific medical knowledge pertaining to fetal anomalies/outcomes

C. recommending they see a religious leader for spiritual guidance

D. recounting success stories relating to special needs children in your practice

E. suggesting they meet with another couple who has faced similar issues

Copyright 2010 by the American Academy of Pediatrics page 171


2011 PREP SA on CD-ROM

Critique: 55 Preferred Response: B

Learning that there are serious problems in their fetus or newborn creates a crisis for the
parents that is likened to experiencing the death of a loved one. In fact, such news brings about
the death of the dream of a healthy child. Parents experience shock, followed by the grieving
stages that are associated with a death in the family: denial, bargaining, confusion, anger, and
depression. This journey is complex and unique to each individual and family. Parents typically
work through this process differently from each other, which may cause further pain and
suffering. In addition, they may be faced with extraordinarily difficult decisions such as whether
to terminate the pregnancy or whether to withdraw life support. In situations when fetal or
neonatal outcome is uncertain, there is the added strain of balancing hope with the potential for
loss.
In a scenario such as the one described in the vignette, it is important for the pediatrician to
be a good listener and to offer support. It also has been shown repeatedly that providing
information about the fetal/newborn diagnosis and potential outcomes allows parents to regain
some control of the situation. They may choose to pursue further research, seek another
opinion, or simply put the information to use in considering options and forming opinions.
Although it is wise to ask parents about their sources of support, clinicians must be careful
not to recommend a course of action that may not be appropriate for the couple. They may not
have told their extended family about their situation, or there may be conflicts among family
members, in which cases recommending discussion with family is not helpful. Even if parents
consider themselves to be religious, they might prefer not to discuss this particular topic with a
religious leader. Recounting anecdotes about families in similar situations may feel cold and
impersonal to the parents. Finally, making an offer to meet with another couple who has had a
similar experience may be helpful, but this is a personal preference and may not be desired or
acceptable.

Suggested reading:

Allen JS, Mulhauser LC. Genetic counseling after abnormal prenatal diagnosis: facilitating coping
in families who continue their pregnancies. J Genet Couns. 1995;4:251-265

Mayes LC. Child mental health consultation with families of medically compromised infants. Child
Adolesc Psychiatr Clin North Am. 2003;12:401-421

Starke M, Albertson Wikland K, Mller A. Parents experiences of receiving the diagnosis of


Turner syndrome: an explorative and retrospective study. Patient Educ Couns. 2002;47:347-
354. Abstract available at: http://www.ncbi.nlm.nih.gov/pubmed/12135826

Walker AP. Genetic counseling. In: Rimoin DL, Connor JM, Pyeritz RE, Korf BR, eds. Emery and
Rimoins Principles and Practice of Medical Genetics. 4th ed. London, United Kingdom:
Churchill Livingstone; 2002:842-874

Copyright 2010 by the American Academy of Pediatrics page 172


2011 PREP SA on CD-ROM

Question: 56

The mother of an 8-year-old girl is concerned because she has noticed the recent onset of
yellowish staining on her daughters underwear. The mother requests an antibiotic. The girl is
embarrassed, says she feels fine, and states that no one has ever touched her genital area. She
is at Sexual Maturity Rating 2 of pubertal development and she has a clear, scant, mucoid
discharge at her introitus, with normal hymenal tissue. She has no other findings of note on
physical examination.

Of the following, the MOST appropriate next step is

A. a course of an oral antibiotic

B. pelvic ultrasonography

C. reassurance of both the mother and child

D. recommendation that the child douche regularly

E. vaginoscopy

Copyright 2010 by the American Academy of Pediatrics page 173


2011 PREP SA on CD-ROM

Critique: 56 Preferred Response: C

The increased estrogen concentration that accompanies the onset of puberty results in
thickening of the vaginal epithelium, change in the pH from alkaline to acidic, and production of a
mucoid discharge (leukorrhea). During established menstrual cycles, the discharge changes in
character from mucoid or watery in mid-cycle to a stickier, scantier discharge as a result of
increasing progesterone concentrations in the second half of the cycle and finally to an
increased quantity just before menses. There are no associated symptoms such as itching or
odor. However, parents may note staining of the underwear when the discharge air dries,
raising concerns about a possible infection, as described for the girl in the vignette. Treatment is
reassurance, education on good hygiene, including wiping from front to back, and wearing of
cotton underwear. The use of sitz baths with room temperature water followed by air drying
may be suggested if the child continues to be bothered by the discharge. With the development
of the labia majora as puberty progresses, leukorrhea becomes less of an issue. A physiologic
discharge may increase in quantity with the wearing of tight clothing or underwear made from
nonabsorbent material. If a wet mount is obtained, it shows epithelial cells, normal flora, and few,
if any, white blood cells.
Bubble baths and douching should be discouraged, and there is no role for antibiotics or
vaginal creams for leukorrhea. The presence of a foreign body would result in a foul-smelling,
yellow vaginal discharge that may be blood-stained. If a foreign body is suspected and not
clearly visible, vaginoscopy under anesthesia for removal may be required in a young child.
There is no role for a pelvic ultrasonography in this situation.

Suggested reading:

Freeto JP, Jay MS. "What's really going on down there?" A practical approach to the adolescent
who has gynecologic complaints. Pediatr Clin North Am. 2006;53:529-545.
DOI:10.1016/j.pcl.2006.02.005. Abstract available at:
http://www.ncbi.nlm.nih.gov/pubmed/16716795

Hwang LY, Shafer M-AB. Vaginitis and vaginosis. In: Neinstein LS, Gordon CM, Katzman DK,
Rosen DS, Woods ER, eds. Adolescent Health Care: A Practical Guide. 5th ed. Philadelphia, Pa:
Lippincott Williams & Wilkins, a Wolters Kluwer business; 2008:729-731

Rosen DS. Physiologic growth and development during adolescence. Pediatr Rev. 2004;25:194-
200. DOI: 10.1542/10.1542/pir.25-6-194. Available at:
http://pedsinreview.aappublications.org/cgi/content/full/25/6/194

Woods ER, Emans SJ. Vulvovaginal complaints in the adolescent. In: Emans SJH, Laufer MR,
Goldstein DP. eds. Pediatric and Adolescent Gynecology. 5th ed. Philadelphia, Pa: Lippincott
Williams & Wilkins; 2005:525-564

Copyright 2010 by the American Academy of Pediatrics page 174


2011 PREP SA on CD-ROM

Question: 57

You are called to evaluate a 16-year-old boy on the pediatric ward for a brief, new-onset,
generalized tonic-clonic seizure. The adolescent was transferred yesterday from the pediatric
intensive care unit (PICU) after nonoperative management of traumatic brain injury that resulted
from a motor vehicle crash 4 days ago. The patient had been reported to be improving during his
PICU course, but the nurse reports that he was increasingly sleepy overnight. On physical
examination, the young man is sleepy but can be aroused and knows his name. He has nonfocal
neurologic examination findings, and his pupils are equal and reactive. He has a temperature of
37.0C, heart rate of 80 beats/min, respiratory rate of 15 breaths/min, and blood pressure of
120/80 mm Hg. Results of recent laboratory studies include:

Serum sodium, 120 mEg/L (120 mmol/L)


Serum potassium, 4 mEq/L (4 mmol/L)
Serum chloride, 95 mEq/L (95 mmol/L)
Hematocrit, 45.0% (0.45)
Blood urea nitrogen, 10 mg/dL (3.6 mmol/L)
Serum creatinine, 1.0 mg/dL (88.4 mcmol/L)

Of the following, the MOST likely reason for the patients change in mental status is

A. diabetes insipidus

B. posttraumatic epilepsy

C. syndrome of inappropriate antidiuretic hormone

D. temporal lobe (uncal) herniation

E. transtentorial herniation

Copyright 2010 by the American Academy of Pediatrics page 175


2011 PREP SA on CD-ROM

Critique: 57 Preferred Response: C

Endocrine disorders following head trauma occur in approximately 5% of traumatic brain


injuries and can be challenging in both diagnosis and treatment. The syndrome of inappropriate
secretion of antidiuretic hormone (SIADH) is the most commonly seen disorder of sodium
homeostasis following head injury. It is characterized by decreased urine output, hyponatremia,
low serum osmolality, and high urine osmolality that is out of proportion to the serum osmolality.
The altered mental status, hyponatremia, and seizure exhibited by the boy in the vignette likely
are the result of SIADH. Clinical symptoms include lethargy, changes in mental status, and
seizures. Treatment consists of fluid restriction and close monitoring of fluid volume and
electrolytes. Patients who have acute neurologic changes such as seizures or coma that are
due to severe hyponatremia (<120 mEq/L [120 mmol/L]) should be treated with 3% hypertonic
saline to raise their serum sodium concentrations approximately 6 to 10 mEq/L (6 to 10 mmol/L).
Diabetes insipidus also can be seen in patients who suffer head injury, especially those
who have a history of severe brain injury and basilar skull fracture. However, it is characterized
by increased urine output, hypernatremia, high serum osmolality, and low urine osmolality that is
out of proportion to the serum osmolality. Clinical symptoms reflect severe dehydration and
include hemodynamic instability and irritability.
Posttraumatic epilepsy is defined as recurrent posttraumatic seizures believed to be due to
brain tissue injury. Patients who have seizures following trauma should have electrolytes
measured, and if values are abnormal, SIADH is the more likely cause. Therefore, sodium should
be corrected before undertaking further evaluation and assigning a diagnosis of posttraumatic
epilepsy. Cerebral edema can result from hyponatremia, but findings on this patients clinical
examination are not consistent with signs of uncal or transtentorial herniation.

Suggested reading:

Atabaki SM. Pediatric head injury. Pediatr Rev. 2007;28;215-224. DOI: 10.1542/10.1542/pir.28-6-
215. Available at: http://pedsinreview.aappublications.org/cgi/content/full/28/6/215

Klein MJ. Post head injury endocrine complications. eMedicine Specialties, Physical Medicine
and Rehabilitation, Traumatic Brain Injury. 2009. Available at:
http://emedicine.medscape.com/article/326123-overview

Moritz ML, Ayus JC. Disorders of water metabolism in children: hyponatremia and hypernatremia.
Pediatr Rev. 2002;23;371-380. DOI: 10.1542/10.1542/pir.23-11-371. Available at:
http://pedsinreview.aappublications.org/cgi/content/full/23/11/371

Copyright 2010 by the American Academy of Pediatrics page 176


2011 PREP SA on CD-ROM

Question: 58

A 3-year-old boy is brought to the emergency department by emergency medical services after
being found unresponsive and twitching by his parents. The emergency medical technicians
determined that the boys blood glucose was 25 mg/dL (1.4 mmol/L) on the scene and started an
intravenous infusion with dextrose. When you see him in the emergency department, he is
beginning to awaken and recognize his parents. Repeat blood glucose measures 73 mg/dL (4.1
mmol/L).

Of the following, the MOST useful additional laboratory test is

A. serum C-peptide assessment

B. serum insulin assessment

C. serum proinsulin assessment

D. serum tryptophan synthetase assessment

E. urine dipstick for ketones

Copyright 2010 by the American Academy of Pediatrics page 177


2011 PREP SA on CD-ROM

Critique: 58 Preferred Response: E

It is important to determine the cause of childhood hypoglycemia, as described for the boy in
the vignette, because management varies, depending upon the cause of the episode. The most
common cause of hypoglycemia in early childhood is ketotic hypoglycemia, which seems to
result from an imbalance between glucose utilization and production through hepatic, and to a
lesser extent, renal glycogenolysis and gluconeogenesis. It commonly manifests as fasting
hypoglycemia noted in the morning hours after sleep that has followed poor food intake the day
before. Affected children often are thin and have decreased muscle and fat mass. This disorder
should not persist beyond the age of 7 or 8 years because hepatic glucose production capacity
from glucose precursors produced from muscle and fat should meet fasting glucose needs of
the brain and other obligate glucose-using tissues after that time.
Measurement of a "critical" blood sample at the time of hypoglycemia may help to determine
if hyperinsulinism is the cause of low blood glucose concentrations. Measurement of serum
insulin and C-peptide can determine whether insulin values are elevated at the time of
hypoglycemia. If C-peptide is not elevated when insulin values are high, the hyperinsulinism likely
is from an insulin medication vial and, therefore, free of the C-peptide released from the
pancreatic beta cell in equimolar amounts with insulin. Proinsulin also is released from the beta
cell but in relatively small amounts compared with insulin. In some rare disorders of insulin
cleavage, proinsulin is released in relatively larger amounts compared with insulin. Once
hypoglycemia has been treated, as it has been for this boy, measurement of insulin and related
compounds are not useful diagnostic tests.
Because insulin suppresses ketogenesis, the finding of a large amount of acetoacetate
(ketones) in a dipstick urine sample shortly after hypoglycemia strongly suggests that insulin
excess is not the cause of hypoglycemia and supports the diagnosis of ketotic hypoglycemia.
However, small amounts of ketones have been found in the urine of some children who have
documented hyperinsulinism. Other disorders that involve an inability to use ketones or
ineffective metabolism of carbohydrate may lead to hypoglycemia with ketonuria. Examples
include endocrine deficiency disorders, some glycogen storage diseases, defects in
gluconeogenesis, and organic acidurias.
Tryptophan synthetase is an enzyme found in plants and bacteria, but not in animals, that
catalyzes the final step in the synthesis of tryptophan. Tryptophan is an essential amino acid
that cannot be synthesized by humans.

Suggested reading:

Hoe FM. Hypoglycemia in infants and children. Adv Pediatr. 2008;55:367-384. Abstract available
at: http://www.ncbi.nlm.nih.gov/pubmed/19048739

Huidekoper HH, Duran M, Turkenburg M, Ackermans MT, Sauerwein HP, Wijburg FA. Fasting
adaptation in idiopathic ketotic hypoglycemia: a mismatch between glucose production and
demand. Eur J Pediatr. 2008;167:859-865. DOI: 10.1007/s00431-007-0598-5. Abstract available
at: http://www.ncbi.nlm.nih.gov/pubmed/17934759

Copyright 2010 by the American Academy of Pediatrics page 178


2011 PREP SA on CD-ROM

Kleinman RE, ed. Hypoglycemia in infants and children. In: Pediatric Nutrition Handbook. 6th ed.
Elk Grove Village, Ill: Academy of Pediatrics; 2009:701-718

Copyright 2010 by the American Academy of Pediatrics page 179


2011 PREP SA on CD-ROM

Question: 59

A 2-year-old boy is mildly delayed in reaching his gross motor milestones, but his language
development is appropriate for his age. He can speak in three- to four-word sentences. His
parents are concerned that he will have academic difficulties due to his motor deficiencies. His
neurologic examination yields nonfocal results.

Of the following, the MOST appropriate response is that

A. gross motor development is an accurate predictor for school success

B. his delays are associated with learning disabilities

C. his delays will result in poor handwriting skills

D. his mildly delayed gross motor skills suggest a diagnosis of cerebral palsy

E. language development is an accurate predictor of intellectual function

Copyright 2010 by the American Academy of Pediatrics page 180


2011 PREP SA on CD-ROM

Critique: 59 Preferred Response: E

A childs language development in infancy and early childhood correlates closely with
cognitive development and, thus, is a better predictor of cognitive function than gross motor
development. The acquisition and use of language is critical to a childs development in the areas
of cognitive and social development. Phonemic awareness is the ability to attend to phonemes
(speech sounds) that are used in syllables and words and is a critical language skill in the
development of reading.
The delay in achieving gross motor milestones described for the boy in the vignette has the
weakest correlation with general intellectual functioning and, therefore, is not a strong predictor
of school success. He has mildly delayed gross motor skills and a nonfocal neurologic
examination. As a result, he does not meet the criteria for a diagnosis of cerebral palsy.
Handwriting skills are affected by weakness in fine (not gross) motor development.

Suggested reading:

Coplan L. Language delays. In: Parker S, Zuckerman B, Augustyn M, eds. Developmental and
Behavioral Pediatrics: A Handbook for Primary Care. 2nd ed. Philadelphia, Pa: Lippincott
Williams & Wilkins; 2005:222-226

Feldman HM. Evaluation and management of language and speech disorders in preschool
children. Pediatr Rev. 2005;26:131-142. DOI: 10.1542/10.1542/pir.26-4-131. Available at:
http://pedsinreview.aappublications.org/cgi/content/full/26/4/131

Shaywitz SE, Shaywitz BA. Dyslexia (specific reading disability). Pediatr Rev. 2003;24;147-153.
DOI: 10.1542/10.1542/pir.24-5-147. Available at:
http://pedsinreview.aappublications.org/cgi/content/full/24/5/147

Copyright 2010 by the American Academy of Pediatrics page 181


2011 PREP SA on CD-ROM

Question: 60

You diagnose Kawasaki disease in a 2-year-old child who has fever, conjunctival injection, rash,
and mucositis. You recommend therapy with intravenous immune globulin (IVIG) and aspirin.
The parents ask you about potential adverse reactions to IVIG.

Of the following, the MOST common adverse event associated with IVIG is

A. aseptic meningitis

B. hepatitis B infection

C. hepatitis C infection

D. human immunodeficiency virus infection

E. intravenous site phlebitis

Copyright 2010 by the American Academy of Pediatrics page 182


2011 PREP SA on CD-ROM

Critique: 60 Preferred Response: A

Adverse reactions to intravenous immune globulin (IVIG) are primarily inflammatory or


hypersensitivity. Although not seen commonly, aseptic meningitis may occur, beginning within
several hours to 2 days after IVIG administration and presenting with severe headache,
nuchal rigidity, drowsiness, fever, photophobia, nausea, and vomiting. It may occur more
commonly in recipients of high-dose (2 g/kg) treatment and is self-limited, resolving over several
days.
Infusion of IVIG, as with other blood products, should be monitored for hypersensitivity
and anaphylactic reactions, including fever, nausea, vomiting, hypotension, stridor, respiratory
distress, urticaria, and pruritus. Reducing the rate of infusion may ameliorate these symptoms,
but epinephrine should be available to address potential life-threatening reactions. Preparations
of IVIG are more than 90% IgG, but they may contain small amounts of IgA, which could
prompt potential anaphylactic reactions if administered to IgA-deficient individuals. IgA-free
preparations are available for use in such patients.
Although derived from pooled human plasma (>1,000 donors per lot), IVIG undergoes
multiple procedures to prevent transmission of infectious organisms, including hepatitis B,
hepatitis C, and human immunodeficiency virus (HIV). Prior to standardization of present
purification procedures, rare cases of hepatitis C infection were reported after IVIG infusion.
Current purification processes include cold alcohol fractionation from plasma, followed by
ultrafiltration, ion exchange chromatography, treatment at an acid pH in the presence of
detergents, and lyophilization.
Intravenous site phlebitis is not specifically associated with IVIG infusion (as opposed to
infusion of drugs such as nafcillin and chemotherapeutic agents). Other adverse reactions
associated with IVIG include acute renal failure, fluid volume overload, transient decrease in
white blood cell count, hemolytic anemia, myalgias, and arthralgias.
The clinician should be aware that IVIG can interfere with immune responses to live viral
vaccines. Therefore, administration of live vaccines (measles, mumps, and rubella; varicella)
must be delayed for up to 11 months, based on the dose of IVIG administered.
Pediatric indications for the use of IVIG include primary immunodeficiencies, Kawasaki
disease, pediatric HIV infection, recent stem cell transplantation, and immune-mediated
thrombocytopenia.

Suggested reading:

American Academy of Pediatrics. Active immunization of people who recently received immune
globulin. Passive immunization, Immune globulin intravenous. In: Pickering LK, Baker CJ, Kimberlin
DW, Long SS, eds. Red Book: 2009 Report of the Committee on Infectious Diseases. 28th ed.
Elk Grove Village, Ill: American Academy of Pediatrics; 2009:37

American Academy of Pediatrics. Passive immunization: immune globulin intravenous. In:


Pickering LK, Baker CJ, Kimberlin DW, Long SS, eds. Red Book: 2009 Report of the Committee
on Infectious Diseases. 28th ed. Elk Grove Village, Ill: American Academy of Pediatrics; 2009:58-
61

Copyright 2010 by the American Academy of Pediatrics page 183


2011 PREP SA on CD-ROM

Silvergleid AJ. Berger M. General principles of the use of intravenous immune globulin. UpToDate
Online 17.3. 2009. Available at:
http://www.uptodate.com/online/content/topic.do?topicKey=lab_med/2914&selectedTitle=4%7E1
50&source=search_result819

Copyright 2010 by the American Academy of Pediatrics page 184


2011 PREP SA on CD-ROM

Question: 61

A previously healthy 5-year-old girl presents with fever, abdominal pain, and frequent nonbloody
diarrhea 1 week after swimming at a newly opened water park. On physical examination, her
temperature is 38.0C, her mucous membranes are moist, and her abdomen is diffusely but mildly
tender to palpation. She has no rebound or guarding. Microscopic examination of the stool reveals
oocysts (Item Q61).

Of the following, the BEST treatment for this patient is

A. azithromycin

B. ciprofloxacin

C. nitazoxanide

D. paromomycin

E. trimethoprim-sulfamethoxazole

Copyright 2010 by the American Academy of Pediatrics page 185


2011 PREP SA on CD-ROM

Question: 61

(Courtesy of the Public Health Image Library, Centers for Disease Control and Prevention,
Division of Parasite Diseases)
Oocysts (arrows), as described in the vignette.

Copyright 2010 by the American Academy of Pediatrics page 186


2011 PREP SA on CD-ROM

Critique: 61 Preferred Response: C

Exposure to recreational water can result in illness of the gastrointestinal tract, respiratory
tract, ears, eyes, skin, and central nervous system. The most common recreational water illness
is gastroenteritis, which tends to occur more during the summer months and often is associated
with inadequately disinfected or treated recreational water.
The child described in the vignette has diarrhea due to Cryptosporidium, the most common
cause of gastrointestinal illness associated with treated water venues. Although the infection
can be asymptomatic, most children have frequent, nonbloody, watery diarrhea. Fever, vomiting,
anorexia, and abdominal pain also can occur. The incubation period usually is 7 days.
Documenting oocysts in a stool specimen is diagnostic (Item C61). Nitazoxanide can be
administered for 3 days, although immunocompetent people generally do not require specific
therapy because the illness is self-limited.
Paromomycin is used to treat mild-to-moderate amebiasis. In combination with azithromycin,
it is considered appropriate for the treatment of cryptosporidiosis in human immunodeficiency
virus-infected patients. Azithromycin, ciprofloxacin, or trimethoprim-sulfamethoxazole can be
used to treat susceptible isolates of Shigella, Salmonella, and Escherichia coli.
Chlorination is the most common treatment used to prevent transmission of infectious
diseases at recreational water venues. Relatively chlorine-sensitive pathogens include Shigella,
Giardia, and norovirus, although both Giardia and norovirus can survive in chlorinated water
for up to 60 minutes and have been associated with swimming pool outbreaks. Cryptosporidia
are protozoa that form oocysts that are highly resistant to chlorination and can remain infectious
for days in chlorinated pools.
People who have diarrhea should not participate in recreational water activities. Because
prolonged excretion can occur with Cryptosporidium infection, avoidance of water venues is
recommended for an additional 2 weeks after the cessation of diarrhea. Those infected with
other diarrheagenic pathogens should refrain from swimming for an additional 1 week after
symptoms cease. Proper hand hygiene is critical in preventing the spread of infection caused by
organisms transmitted via the fecal-oral route. Children who are not toilet-trained should be
checked often for soiling at recreational water venues and should be taken for regular restroom
breaks. Avoiding the ingestion of water is recommended to prevent acquisition of infection.

Suggested reading:

American Academy of Pediatrics. Cryptosporidiosis. In: Pickering LK, Baker CJ, Kimberlin DW,
Long SS, eds. Red Book: 2009 Report of the Committee on Infectious Diseases. 28th ed. Elk
Grove Village, Ill: American Academy of Pediatrics; 2009:272-273

American Academy of Pediatrics. Prevention of illnesses associated with recreational water


use. In: Pickering LK, Baker CJ, Kimberlin DW, Long SS, eds. Red Book: 2009 Report of the
Committee on Infectious Diseases. 28th ed. Elk Grove Village, Ill: American Academy of
Pediatrics; 2009:196-197

Centers for Disease Control and Prevention. Notice to readers: revised recommendations for

Copyright 2010 by the American Academy of Pediatrics page 187


2011 PREP SA on CD-ROM

responding to fecal accidents in disinfected swimming venues. MMWR Morb Mortal Wkly Rep.
2008;57:151-152. Available at: http://www.cdc.gov/mmwr/preview/mmwrhtml/mm5706a5.htm

Ochoa TJ, White AC Jr. Cryptosporidiosis. In: Feigin RD, Cherry JD, Demmler-Harrison GJ, Kaplan
SL, eds. Feigin & Cherrys Textbook of Pediatric Infectious Diseases. 6th ed. Philadelphia, Pa:
Saunders Elsevier; 2009:2869-2880

Yoder J, Roberts V, Craun GF, et al. Surveillance for waterborne disease and outbreaks
associated with drinking water and water not intended for drinking United States, 2005-2006.
MMWR Surveill Summ. 2009;57(SS-9):39-62. Available at:
http://www.cdc.gov/mmwr/preview/mmwrhtml/ss5709a4.htm

Copyright 2010 by the American Academy of Pediatrics page 188


2011 PREP SA on CD-ROM

Critique: 61

(Courtesy of the Public Health Image Library, Centers for Disease Control and Prevention,
Division of Parasitic Diseases)
Cryptosporidium parvum oocysts (arrows) appear red when stained with a modified acid-fast
method.

Copyright 2010 by the American Academy of Pediatrics page 189


2011 PREP SA on CD-ROM

Question: 62

A 3-year-old girl presents with a 4- to 5-day history of diarrhea, increased fussiness, and
decreased urine output over the previous day. On physical examination, her temperature is
37.5C, heart rate is 120 beats/min, respiratory rate is 24 breaths/min, and blood pressure is
126/84 mm Hg. In addition, she has slightly pale, moist mucous membranes and a II/VI flow
murmur, but no gallop or edema. Laboratory evaluation shows:

White blood cell count, 24.2x10 3/mcL (24.2x109/L)


Hemoglobin, 6.1 g/dL (61 g/L)
Hematocrit, 18.5% (0.185)
Platelet count, 68x10 3/mcL (68x109/L)
Blood urea nitrogen, 60 mg/dL (21.4 mmol/L)
Creatinine, 2.9 mg/dL (256.4 mcmol/L)

Of the following, the MOST useful diagnostic test in this clinical setting is

A. ADAMTS 13 assessment

B. anti-double-stranded DNA assessment

C. complement component 3 assessment

D. direct Coombs test

E. peripheral blood smear

Copyright 2010 by the American Academy of Pediatrics page 190


2011 PREP SA on CD-ROM

Critique: 62 Preferred Response: E

Any child who has a diarrheal prodrome of several days duration and subsequently
develops oliguria, such as the girl described in the vignette, needs to be screened for hemolytic-
uremic syndrome (HUS). HUS is characterized by the triad of hemolytic anemia,
thrombocytopenia, and acute renal failure and most often is preceded by gastroenteritis caused
by Escherichia coli O157:H7. Most cases (90%) of childhood HUS are associated with diarrhea
(D+). It is estimated that this strain of E coli is responsible for more than two thirds of cases of
D+ HUS. Infection from this strain of E coli often presents clinically with watery diarrhea for the
first few days, followed later by bloody diarrhea. The Shiga toxin produced by E coli O157:H7 is
absorbed through the inflamed gastrointestinal tract. Neutrophils are believed to transport the
Shiga toxin to renal endothelial cells, where cellular injury occurs. The kidneys are targeted by
Shiga toxin because of receptor expression on glomerular endothelial and renal tubular cells.
The receptors are membrane glycosphingolipid receptors [globotriaosyl ceramide (GB3)] and
have been shown to be expressed in higher numbers in young children compared with
adolescents and adults. After Shiga toxin binding occurs, cellular injury results. The endothelial
injury leads to fibrin accumulation and initiates microangiopathic hemolytic anemia with
thrombocytopenia and subsequent renal injury from the thrombotic microangiopathy. In one
study, antibiotic treatment was associated with the development of HUS in children younger than
10 years of age who have gastrointestinal infections from E coli O157:H7. However, a
subsequent meta-analysis did not confirm this finding. In addition to hematologic and renal
involvement, the central nervous system (CNS) can be involved in 20% to 30% of cases.
The clinician who suspects HUS in a patient should perform a complete blood count (CBC).
The most useful component of the CBC is the peripheral blood smear, which should demonstrate
schistocytes and burr cells and minimal platelets in patients who have HUS. Many patients
exhibit oliguria and some have anuria during the course of their disease. Those producing urine
have hematuria and proteinuria on urinalysis. Other laboratory features are consistent with a
microangiopathic hemolytic anemia, including elevated lactate dehydrogenase and reticulocyte
count and depressed haptoglobin, hemoglobin, and platelet counts. A Coombs test result would
be expected to be negative because this is a nonimmune-mediated hemolytic anemia and,
therefore, the Coombs test is not a useful diagnostic evaluation for HUS.
Other diseases that can be associated with schistocytes on a blood smear are thrombotic
thrombocytopenic purpura (TTP) and systemic lupus erythematosus (SLE). Patients who have
TTP have similar symptoms to HUS but typically are older and have more central nervous system
involvement. TTP is another thrombotic microangiopathic process, but its pathogenesis has been
traced to a defect in the ADAMTS (A Disintegrin And Metalloprotease with ThromboSpondin type
1 repeats) 13 protein, a metalloproteinase responsible for the proteolytic cleavage of large von
Willebrand factor (vWF) multimers. The congenital deficiency of ADAMTS 13 or an antibody
directed at this protein leads to extraordinarily large vWF multimers and resultant platelet
aggregation, with end-organ injury. The girl in the vignette does not fit the clinical picture of TTP
and does not warrant assessment of ADAMTS 13 activity. Similarly, screening for SLE with anti-
double-stranded DNA serology is not indicated. Complement component 3 is a useful
assessment when screening for immune complex-mediated diseases such as
glomerulonephritis, including postinfectious glomerulonephritis and SLE.

Copyright 2010 by the American Academy of Pediatrics page 191


2011 PREP SA on CD-ROM

Treatment for HUS is primarily supportive. Meticulous management of fluid and electrolytes is
essential to avoid complications of hyponatremia, hyperkalemia, hyperphosphatemia, or fluid
overload. Consideration should be given to a fluid prescription of insensible water loss (500
mL/m2 per day) plus urine and stool output replacement (mL for mL) in those who exhibit
euvolemia. It is essential to maintain euvolemia to avoid the risks of fluid overload (congestive
heart failure and pulmonary edema) or volume depletion (exacerbation of azotemia). Severe
oligoanuria is difficult to manage without dialysis because of overt fluid overload or the need for
severe fluid restriction to avoid volume overload. Other conditions that warrant dialysis are
hyperkalemia or severe uremia (blood urea nitrogen >100 mg/dL [35.7 mmol/L]). Packed red blood
cells are administered for anemia; platelet transfusions usually are avoided to prevent the
theoretical risk of exacerbating the thrombotic microangiopathic process and microthrombi
deposits. Whether to administer antibiotics to children who have E coli O157:H7-associated
diarrhea remains controversial, but most experts do not initiate antimicrobial therapy because no
benefit has been proven. Studies have failed to confirm that children who have E coli O157:H7-
associated diarrhea and are treated with antibiotics have a greater risk of developing HUS.

Suggested reading:

Corrigan JJ Jr, Boineau FG. Hemolytic-uremic syndrome. Pediatr Review. 2001;22:365-369. DOI:
10.1542/10.1542/pir.22-11-365. Available at:
http://pedsinreview.aappublications.org/cgi/content/full/22/11/365

Loirat C, Girma JP, Desconclois C, Coppo P, Veyradier A. Thrombotic thrombocytopenic purpura


related to severe ADAMTS13 deficiency in children. Pediatr Nephrol. 2009;24:19-29. Abstract
available at: http://www.ncbi.nlm.nih.gov/pubmed/18574602

Oakes RS, Kirkham JK, Nelson RD, Siegler RL. Duration of oliguria and anuria as predictors of
chronic renal-related sequelae in post-diarrheal hemolytic uremic syndrome. Pediatr Nephrol.
2008;23:1303-1308. Abstract available at: http://www.ncbi.nlm.nih.gov/pubmed/18465151

Ray PE, Liu XH. Pathogenesis of Shiga toxin-induced hemolytic uremic syndrome. Pediatr
Nephrol. 2001;16:823-839. Abstract available at: http://www.ncbi.nlm.nih.gov/pubmed/11605791

Safdar N, Said A, Gangnon RE, Maki DG. Risk of hemolytic uremic syndrome after antibiotic
treatment of Escherichia coli O157:H7 enteritis: a meta-analysis. JAMA. 2002;288:996-1001.
Available at: http://jama.ama-assn.org/cgi/content/full/288/8/996

Wong CS, Jelacic S, Habeeb RL, Watkins SL, Tarr PI. The risk of the hemolytic-uremic syndrome
after antibiotic treatment of Escherichia coli O157:H7 infections. N Engl J Med. 2000;342:1930-
1936. Available at: http://content.nejm.org/cgi/content/full/342/26/1930

Copyright 2010 by the American Academy of Pediatrics page 192


2011 PREP SA on CD-ROM

Question: 63

You are evaluating a 14-year-old girl for allergic rhinitis. Her current medical diagnoses include
moderate persistent asthma, gastroesophageal reflux, and migraine headaches. You discuss that
before aeroallergen skin testing, some medications need to be stopped temporarily because they
can suppress skin test results.

Of the following, the drug that is MOST likely to suppress aeroallergen skin tests is

A. a short-acting beta2 agonist

B. amitriptyline

C. an inhaled corticosteroid

D. montelukast

E. ranitidine

Copyright 2010 by the American Academy of Pediatrics page 193


2011 PREP SA on CD-ROM

Critique: 63 Preferred Response: B

Because approximately 60% of children who have persistent asthma may have coexisting
allergic rhinitis, it is important to consider evaluating and treating this underlying condition. Allergy
testing typically is performed via skin or blood testing.
Allergy skin testing generally is more sensitive and specific than blood testing, and results
are available within 15 to 20 minutes. Further, because allergy skin testing typically is performed
by an allergist, specific interpretation of the relevant positive tests can be applied to the
individual patient. The disadvantages of skin testing include discomfort for some children and the
need to cease antihistamines prior to testing (Item C63). Of the medications listed in the vignette,
only amitriptyline can significantly suppress immunoglobulin (Ig) E-mediated wheal and flare
responses. Ranitidine and cimetidine are H2-antihistamines but do not block the H1-histamine
receptor and, therefore, do not alter allergy skin testing results. Long-term oral corticosteroids
may affect skin testing, but inhaled corticosteroids have not been demonstrated to have such an
effect. Montelukast is a leukotriene inhibitor approved for allergic rhinitis and asthma that does
not suppress skin wheal and flare responses. Short-acting beta2-agonists also do not suppress
allergy skin testing.
Serum or in vitro IgE testing has the advantage of being performed at most commercial
laboratories. Further, serum testing generally is preferred for patients who have severe atopic
dermatitis or dermatographism or those who have experienced life-threatening reactions to the
allergen.

Suggested reading:

Calabria CW, Dietrich J, Hagan L. Comparison of serum-specific IgE (ImmunoCAP) and skin-prick
test results for 53 inhalant allergens in patients with chronic rhinitis. Allergy Asthma Proc.
2009;30:386-96. Abstract available at: http://www.ncbi.nlm.nih.gov/pubmed/19772760

Cartwright RC, Dolen WK. Consultation with the specialist: who needs allergy testing and how to
get it done. Pediatr Rev. 2006;27:140-146. DOI: 10.1542/10.1542/pir.27-4-140. Available at:
http://pedsinreview.aappublications.org/cgi/content/full/27/4/140

Demoly P, Bousquet J, Romano A. In vivo methods for the study of allergy. In: Adkinson NF Jr,
Bochner BS, Busse WW, Holgate ST, Lemanske RF Jr, Simons FER, eds. Middletons Allergy:
Principles & Practice. 7th ed. Philadelphia, Pa: Mosby Elsevier; 2009:1267-1279

Copyright 2010 by the American Academy of Pediatrics page 194


2011 PREP SA on CD-ROM

Critique: 63

Copyright 2010 by the American Academy of Pediatrics page 195


2011 PREP SA on CD-ROM

Question: 64

A 16-year-old boy presents to the emergency department with headache, dizziness, and chest
pain. He is agitated and has occasional ticlike movements. On physical examination, his
temperature is 37.5C, heart rate is 120 beats/min, respiratory rate is 20 breaths/min, and blood
pressure is 130/86 mm Hg. His pupils are mildly dilated and briskly reactive. Other findings on the
remainder of his examination are within normal parameters.

Of the following, the MOST likely explanation for this boys symptoms is abuse of

A. alcohol

B. cocaine

C. dextromethorphan

D. LSD

E. marijuana

Copyright 2010 by the American Academy of Pediatrics page 196


2011 PREP SA on CD-ROM

Critique: 64 Preferred Response: B

The patient described in the vignette is exhibiting signs of stimulant intoxication. Cocaine is a
commonly abused stimulant among adolescents. Alcohol and marijuana are sedatives, and the
absence of hallucinations makes dextromethorphan or lysergic acid diethylamide (LSD) ingestion
unlikely.
Signs and symptoms of acute cocaine overdose include tachycardia, hypertension,
hyperthermia, agitation, headache, and restlessness. Cocaine ingestion should be considered in
any adolescent who presents with altered mental status, new-onset seizures, chest pain,
dysrhythmias, myocardial ischemia or infarction, shortness of breath, intracranial hemorrhage,
epistaxis, or myoglobinuria. Three phases are identified in severe toxicity (Item C64A), with
progression through the phases occurring over minutes to approximately 1 hour, depending on
the dose and route of exposure. Cocaine overdose may resemble serotonin syndrome,
neuroleptic malignant syndrome, thyroid storm, and other hyperadrenergic states.
Signs and symptoms in chronic cocaine users often depend on the route of exposure (Item
C64B). Risk for dependence also is related to route of exposure, with intravenous users and
those who smoke cocaine at higher risk because of the rapid onset of psychological effects.

Suggested reading:

Burnett LB, Roldan CJ, Adler J. Toxicity, cocaine. eMedicine Specialties, Emergency Medicine,
Toxicology. 2008. Available at: http://emedicine.medscape.com/article/813959-overview

Gorelick DA. Cocaine abuse in adults. UpToDate 17.3. 2009. Available at:
http://www.uptodate.com/patients/content/topic.do?topicKey=~Eu55ZYzxXoeX6

Greene JP, Ahrendt D, Stafford EM. Adolescent abuse of other drugs. Adolesc Med Clin.
2006;17:283-318. DOI: 10.1016/j.admecli.2006.03.007. Abstract available at:
http://www.ncbi.nlm.nih.gov/pubmed/16814695

Jenkins RR, Adger H. Substance abuse. In: Kleigman RM, Behrman RE, Jenson HB, Stanton BF,
eds. Nelson Textbook of Pediatrics. 18th ed. Philadelphia, Pa: Saunders Elsevier; 2007:824-833

Nelson L, Odujebe O. Cocaine: acute intoxication. UpToDate 17.3. 2009. Available at:
http://www.utdol.com/online/content/topic.do?topicKey=ad_tox/12972&selectedTitle=1~150&sou
rce=search_result

Copyright 2010 by the American Academy of Pediatrics page 197


2011 PREP SA on CD-ROM

Critique: 64

Copyright 2010 by the American Academy of Pediatrics page 198


2011 PREP SA on CD-ROM

Critique: 64

Copyright 2010 by the American Academy of Pediatrics page 199


2011 PREP SA on CD-ROM

Question: 65

A 16-year-old previously healthy girl presents with a complaint of leg weakness. Two weeks
ago, she had a 5-day episode of watery diarrhea, abdominal pain, and fever. She had been
gradually improving until this morning, when she fell while trying to get out of bed and could not
stand or walk without support. She denies headaches, visual problems, vertigo, difficulty
swallowing, or incontinence. She has been taking acetaminophen and ibuprofen for fever during
her recent illness. Her immunizations are up to date. Physical examination demonstrates an alert
but anxious girl who has normal vital signs. Other findings of note on her examination include:

Head, eyes, ears, nose, throat


-Pupils equal and reactive to light
-Normal extraocular movements
-No papilledema
-No facial weakness or asymmetry
-Cardiac, respiratory, abdominal examination results unremarkable
Neurologic
-Normal cranial nerves
-Upper extremities: Strength 4/5, normal deep tendon reflexes and sensation
-Lower extremities: Strength 2/5, deep tendon reflexes absent, sensation intact

Of the following, the MOST likely cause of this childs illness is

A. Acinetobacter baumannii

B. Campylobacter jejuni

C. enterotoxigenic Escherichia coli

D. Haemophilus influenza

E. Neisseria meningitidis

Copyright 2010 by the American Academy of Pediatrics page 200


2011 PREP SA on CD-ROM

Critique: 65 Preferred Response: B

Relatively mild infectious disorders may be associated with serious clinical sequelae. The
girl described in the vignette presents with an ascending neuropathic syndrome occurring
approximately 2 weeks after the onset of a symptomatically resolved diarrheal illness. The
physical findings suggest the onset of a symmetric polyneuropathy affecting the lower
extremities. Based upon these data, the presumptive diagnosis is Guillain-Barr syndrome
(GBS), now the most common cause of acute neuromuscular paralysis. Prior studies suggest
that two thirds of patients who have GBS present following an undiagnosed and often mild
respiratory or gastrointestinal illness. The organism that has been linked most closely with GBS
is Campylobacter jejuni. Acinetobacter baumannii, enterotoxigenic Escherichia coli,
Haemophilus influenzae, and Neisseria meningitidis have not been associated with this
disorder. In addition, of these four organisms, only Shiga toxin-producing E coli is a significant
gastrointestinal pathogen.
Campylobacter are motile, gram-negative bacilli that are the most common cause of acute
bacterial gastroenteritis in the developed world, with the most prevalent pathotypes being C
jejuni and C coli. These organisms are harbored in the gastrointestinal tracts of both wild and
domestic birds and animals (isolates have been found in up to 100% of feces from chickens and
turkeys), and outbreaks are common in children visiting dairy farms, particularly among those
who drink unpasteurized milk. Following ingestion of Campylobacter and an incubation period of
1 to 7 days, an acute diarrheal syndrome may develop, often with visible or occult blood,
accompanied by fever, malaise, and abdominal pain. Recovery usually occurs within 1 week of
onset, although up to 20% of patients experience either a relapse or a prolonged illness. Severe
infection with bleeding may mimic inflammatory bowel disease. Unlike the therapeutic approach
to other foodborne pathogens, in which antibiotic therapy generally is not recommended,
treatment of Campylobacter gastroenteritis with either erythromycin or azithromycin can
eradicate the organism within 2 to 3 days, shorten the duration of illness, and prevent relapse
when administered early in the course of infection.
With the virtual eradication of poliomyelitis in the developed world, GBS has become the
most common cause of acute neuromuscular paralysis. It is an autoimmune disorder of the
peripheral nervous system that is characterized by symmetric ascending weakness progressing
over several days. Bacteriologic or serologic evidence of recent C jejuni infection has been
found in 26% of those who have GBS compared with 1% of age-matched controls. Among
patients who have clinical histories and serologic findings consistent with antecedent
Campylobacter enteritis, the mean time between diarrheal illness and onset of neuropathic
symptoms is 9 days. These patients appear to have a significantly worse outcome, with greater
disability, than do patients who have GBS without evidence of a recent C jejuni infection. In
addition to GBS, C jejuni also has been associated with other immunoreactive complications,
including Miller-Fisher syndrome (ophthalmoplegia, areflexia, ataxia), Reiter syndrome (arthritis,
urethritis, bilateral conjunctivitis), erythema nodosum, and reactive arthritis, all presenting during
the convalescent phase of infection.

Suggested reading:

Copyright 2010 by the American Academy of Pediatrics page 201


2011 PREP SA on CD-ROM

Allos BM, Lippy FT, Carlsen A, Washburn RG, Blaser MJ. Campylobacter jejuni strains from
patients with Guillain-Barr syndrome. Emerg Infect Dis. 1998;4(2). Available at:
http://www.cdc.gov/ncidod/EID/vol4no2/allos.htm

American Academy of Pediatrics. Campylobacter infections. In: Pickering LK, Baker CJ, Kimberlin
DW, Long SS, eds. Red Book: 2009 Report of the Committee on Infectious Diseases. 28th ed.
Elk Grove Village, Ill: American Academy of Pediatrics; 2009:243-245

Dzieciatkowska M, Liu X, Heikema AP, et al. Rapid method for sensitive screening of
oligosaccharide epitopes in the lipooligosaccharide from Campylobacter jejuni strains isolated
from Guillain-Barr syndrome and Miller Fisher syndrome patients. J Clin Microbiol.
2008;46:3429-3436. DOI: 10.1128/JCM.00681-08. Available at:
http://jcm.asm.org/cgi/content/full/46/10/3429?view=long&pmid=18753342

Hughes RA, Cornblath DR. Guillain-Barr syndrome. Lancet. 2005;366:1653-1666. DOI:


10.1016/S0140-6736(05)67665-9. Abstract available at:
http://www.ncbi.nlm.nih.gov/pubmed/16271648

Rees JH, Soudain SE, Gregson NA, Hughes RA. Campylobacter jejuni infection and Guillain-
Barr syndrome. N Engl J Med. 1995;333:1374-1379. Available at:
http://content.nejm.org/cgi/content/full/333/21/1374

Copyright 2010 by the American Academy of Pediatrics page 202


2011 PREP SA on CD-ROM

Question: 66

An infant is delivered at 35 weeks gestation by cesarean section because of worsening


maternal pregnancy-induced hypertension. The infant is initially vigorous but develops worsening
respiratory distress with an increasing oxygen requirement and an escalation of her work of
breathing over the first 4 hours after birth. You place an umbilical venous line and obtain a chest
radiograph (Item Q66). Because she is requiring 60% hood oxygen to maintain saturations of
93%, you elect to intubate her endotracheally.

Of the following, the treatment that is MOST likely to decrease her respiratory distress is

A. chest tube placement

B. increased oxygen delivery (to 100%)

C. inhaled nitric oxide

D. prostaglandin therapy

E. surfactant

Copyright 2010 by the American Academy of Pediatrics page 203


2011 PREP SA on CD-ROM

Question: 66

(Courtesy of B Carter)

Copyright 2010 by the American Academy of Pediatrics page 204


2011 PREP SA on CD-ROM

Critique: 66 Preferred Response: E

The infant described in the vignette has respiratory distress syndrome (RDS), which is
treated best by the administration of surfactant into the lungs via an endotracheal tube. RDS is
caused by a deficiency of pulmonary surfactant and is seen most commonly in the preterm
infant. Surfactant is produced by the type II pneumocytes in the lung, with production beginning
around 23 weeks gestation and increasing as the lungs evolve from the saccular to alveolar
phase, during which time the type II pneumocytes proliferate and mature. The presence of
surfactant in the lungs improves alveolar stability by decreasing surface tension, leading to
increased pulmonary compliance. Surfactant deficiency is seen in 60% of infants born before 28
weeks gestation and affects fewer than 5% of infants born after 34 weeks gestation.
The clinical symptoms of RDS include tachypnea, grunting, flaring, retractions, central
cyanosis, and apnea. Initially, the work of breathing is increased because of the need to distend
the alveoli to prevent the development of atelectasis. If atelectasis occurs, the functional residual
capacity of the lung decreases, leading to lung injury, protein exudation, and edema. The
characteristic chest radiograph of an infant who has RDS demonstrates a ground-glass
appearance, with air bronchograms and diffuse atelectasis (Item C66).
Treatment strategies for RDS are preventive, supportive, and interventional. Antenatal
steroids have been shown to decrease RDS, intraventricular hemorrhage, and death when
administered to women at high risk of preterm delivery prior to 34 weeks gestation. Infants
manifesting mild RDS may receive supplemental hood oxygen or continuous positive airway
pressure. Intubation and delivery of exogenous surfactant may be used prophylactically in
infants younger than 28 weeks gestation or as rescue therapy in older neonates manifesting
the clinical and radiologic findings of RDS. Multiple studies have demonstrated decreases in
death and chronic lung disease with surfactant administration.
Pneumothorax requiring chest tube placement is a complication of surfactant deficiency due
to alveolar mismatch during ventilation, but it is not apparent on the chest radiograph for this
infant. The hypoxia found in the infant who has RDS is improved with increased oxygen
delivery, but such delivery does not improve the underlying cause of the respiratory distress or
the ability to ventilate. It is essential to consider additional diagnoses in a late preterm infant
experiencing respiratory distress, including pneumonia, persistent pulmonary hypertension, and
cardiac disease. The apparent improvement in saturation with oxygen therapy reported for this
infant makes cyanotic heart disease less likely. If the infant does not improve after surfactant
administration, echocardiography should be undertaken to rule out noncyanotic heart disease
that requires prostaglandin therapy or coexisting pulmonary hypertension that requires inhaled
nitric oxide.

Suggested reading:

Jobe AH. Pharmacology review: why surfactant works for respiratory distress syndrome.
NeoReviews. 2006;7:e95-e106. Available at:
http://neoreviews.aappublications.org/cgi/content/full/7/2/e95

Rodriquez RJ, Martin RJ, Fanaroff AA. Respiratory distress syndrome and its management. In:

Copyright 2010 by the American Academy of Pediatrics page 205


2011 PREP SA on CD-ROM

Martin RJ, Fanaroff AA, Walsh MC, eds. Fanaroff and Martins Neonatal-Perinatal Medicine:
Diseases of the Fetus and Infant. 8th ed. Philadelphia, Pa: Mosby Elsevier; 2006:1097-1107

Warren JB, Anderson JM. Core concepts: respiratory distress syndrome. NeoReviews.
2009;10:e351-e361. Available at:
http://neoreviews.aappublications.org/cgi/content/full/10/7/e351

Copyright 2010 by the American Academy of Pediatrics page 206


2011 PREP SA on CD-ROM

Critique: 66

(Courtesy of B Carter)
Underinflation, a "ground glass" appearance, and air bronchograms (arrows) are characteristic of
respiratory distress syndrome.

Copyright 2010 by the American Academy of Pediatrics page 207


2011 PREP SA on CD-ROM

Question: 67

Upon returning from a camping trip, a 12-year-old boy notices a painful lesion on his leg that he
thinks might be a spider bite. He has had no fever and has seen no drainage from the wound. He
feels otherwise well but is concerned about the persistent pain. He has no chronic medical
problems, and his immunizations are up to date for his age. Physical examination reveals a 2-cm
slightly red lesion that has a thick, adherent crust over a moderately deep ulcer (Item Q67). There
is no fluctuance. The remainder of his physical examination findings are normal.

Of the following, the MOST appropriate next step is

A. application of a topical corticosteroid

B. consultation with a toxicologist for envenomation care

C. hospital admission for administration of parenteral ceftazidime

D. incision and drainage followed by oral cephalexin

E. local wound care plus oral penicillin

Copyright 2010 by the American Academy of Pediatrics page 208


2011 PREP SA on CD-ROM

Question: 67

(Courtesy of WW Tunnessen, Jr)


Lesions, as described for the boy in the vignette.

Copyright 2010 by the American Academy of Pediatrics page 209


2011 PREP SA on CD-ROM

Critique: 67 Preferred Response: E

The boy described in the vignette has ecthyma, one of several common superficial skin
infections seen in children. Ecthyma appears similar to nonbullous impetigo, but it deepens as it
becomes more persistent, affecting not only the epidermis but the dermis. It is characterized by
an ulcer covered with a tightly adherent crust and surrounded by elevated margins (Item C67A).
It appears most commonly on the legs and is more frequently painful than impetigo. Ecthyma is
caused by Streptococcus pyogenes, in contrast to impetigo, which is caused most commonly by
Staphylococcus aureus with or without S pyogenes present. If S aureus is cultured from a
lesion of ecthyma, it is likely a secondary pathogen.
Treatment for ecthyma is directed toward the pathogen; oral penicillin with local wound care
is usually sufficient. Of note, ecthyma gangrenosum is an entirely different entity most often
caused by systemic Pseudomonas aeruginosa infection. It begins as a red macule that enlarges
and develops into a tender nodule with a grey or hemorrhagic center (Item C67B). The center
becomes necrotic, resulting in a deep ulcer with a gray-black eschar. Ecthyma gangrenosum
occurs almost exclusively in immunocompromised children, who typically have systemic
symptoms of infection in addition to the local skin findings. Parenteral antibiotics to which
Pseudomonas are susceptible, such as ceftazidime, would be appropriate therapy for ecthyma
gangrenosum but are unnecessary for ecthyma caused by S pyogenes.
Superficial skin infections in children are very common and range from impetigo to cellulitis to
abscess. Impetigo is the most common, accounting for 50% to 60% of bacterial skin infections,
and takes two forms: bullous and nonbullous. Nonbullous impetigo comprises 70% of these
infections and is described classically as superficial erosion with a red base and a honey-
colored crust. Systemic symptoms are absent, but regional lymphadenopathy is present in about
90% of cases. Although S pyogenes can be causative, S aureus is the most common pathogen.
The organism responsible for nonbullous impetigo cannot be discerned based on the clinical
appearance of the lesion. Bullous impetigo is caused exclusively by S aureus, usually phage
group 2. Lesions result from localized toxin production and appear as single to multiple flaccid
bullae, some containing purulent fluid (Item C67C).
Cellulitis is a bacterial infection affecting the subcutaneous tissues and dermis. It presents
with erythema, heat, swelling, and pain, usually without purulence unless it is associated with
an open wound. Abscess is a collection of pus; when it is associated with a hair shaft, it is
called a furuncle. A carbuncle is a coalescence of several furuncles. S aureus is the causative
organism in about 70% of all superficial skin infections; S pyogenes is present in 30%. Very
rarely, other organisms, including S epidermidis, Escherichia coli, and group B Streptococcus,
are cultured from superficial skin infections.
In the last decade, the treatment of superficial skin infections has changed because of the
dramatic increase in community-acquired methicillin-resistant S aureus (MRSA). Topical
antibiotics such as mupirocin typically are sufficient for impetigo, although bacterial resistance to
this antibiotic is increasing. Retapamulin is a newer topical agent licensed for children 9 months
of age or older, but in vitro resistance has been detected. The choice of systemic antibiotics for
nonpurulent cellulitis should be based on the prevalence of MRSA in the community. Clindamycin
generally is effective for community-acquired MRSA, although erythromycin resistance may be a
marker for inducible clindamycin resistance. Trimethoprim-sulfamethoxazole and tetracyclines

Copyright 2010 by the American Academy of Pediatrics page 210


2011 PREP SA on CD-ROM

usually are active against MRSA but do not treat group A streptococcal infections. Whenever
possible, culture from the wound or aspiration of the cellulitis should be obtained to guide
therapy. For an abscess, adequate drainage is the most important aspect of treatment; for small
abscesses that are adequately drained, additional systemic antibiotics add little to the treatment.
Large abscesses (>5 cm), the presence of systemic signs (fever), or infections in young
children often require additional antibiotics effective against the identified organism. Incision and
drainage is not indicated in this patient due to the absence of fluctuance.
Topical steroids are used to treat a number of skin conditions, most frequently atopic
dermatitis, but are of no use for isolated superficial skin infections. If impetigo or cellulitis is seen
as a complication of atopic dermatitis, topical steroids might be used in combination with an
appropriate antibiotic. However, the child in the vignette has no underlying atopic dermatitis.
Consultation with a toxicologist would be helpful in the unusual circumstance of a brown spider
envenomation. These bites can result in skin findings ranging from minimal redness to rapidly
progressive, full-thickness necrosis. Neither the time course nor appearance of the lesion
described for this patient is consistent with this diagnosis.

Suggested reading:

Abrahamian FM, Talan DA, Moran GJ. Management of skin and soft-tissue infections in the
emergency department. Infect Dis Clin North Am. 2008;22:89-116. DOI:
10.1016/j.idc.2007.12.001. Abstract available at: http://www.ncbi.nlm.nih.gov/pubmed/18295685

Browning J, Levy M. Cellulitis and superficial skin infections. In: Long SS, Pickering LK, Prober
CG, eds. Principles and Practice of Pediatric Infectious Diseases. 3rd ed. Philadelphia, Pa:
Churchill Livingstone; 2008:434-441

Daum RS. Skin and soft-tissue infections caused by methicillin-resistant Staphylococcus aureus.
N Engl J Med. 2007;357:380-390. Extract available at:
http://content.nejm.org/cgi/content/extract/357/4/380

Gorwitz RJ. A review of community-associated methicillin-resistant Staphylococcus aureus skin


and soft tissue infections. Pediatr Infect Dis J. 2008;27:1-7. DOI:
10.1097/INF.0b013e31815819bb. Abstract available at:
http://www.ncbi.nlm.nih.gov/pubmed/18162929

Sjogren R, McGregor RS, Zenel J. Visual diagnosis: an infant who has a red papule on a
swollen, tender arm. Pediatr Rev. 2004;25:182-185. DOI: 10.1542/10.1542/pir.25-5-182.
Available at: http://pedsinreview.aappublications.org/cgi/content/full/25/5/182

Wasserzug O, Valinsky L, Klement E, et al. A cluster of ecthyma outbreaks caused by a single


clone of invasive and highly infective Streptococcus pyogenes. Clin Infect Dis. 2009;48:1213-
1219. DOI: 10.1086/597770. Abstract available at:
http://www.ncbi.nlm.nih.gov/pubmed/19331587

Copyright 2010 by the American Academy of Pediatrics page 211


2011 PREP SA on CD-ROM

Critique: 67

(Courtesy of D Krowchuk)
Ecthyma produces ulcers that may have adherent crusts.

Copyright 2010 by the American Academy of Pediatrics page 212


2011 PREP SA on CD-ROM

Critique: 67

(Courtesy of Chris Ha, MD; DermAtlas; www. DermAtlas.org)


Ecthyma gangrenosum begins as a red macule that develops a hemorrhagic center.

Copyright 2010 by the American Academy of Pediatrics page 213


2011 PREP SA on CD-ROM

Critique: 67

(Reprinted with permission from Krowchuk DP, Mancini AM, eds. Pediatric Dermatology. A Quick
Reference Guide. Elk Grove Village, Ill: American Academy of Pediatrics; 2007)
In bullous impetigo, the fragile blisters rupture rapidly, forming round, crusted erosions with a
peripheral rim of scale (the remnant of the blister roof).

Copyright 2010 by the American Academy of Pediatrics page 214


2011 PREP SA on CD-ROM

Question: 68

The nurse practitioner who works in your hospital nursery brings in her 6-month-old son for a
health supervision visit. She and her husband ask your opinion about stimulating toys, books, and
television programs for infants and toddlers. She shows you some advertisements from
parenting magazines of proprietary products that claim to be recommended by pediatricians and
teachers.

Of the following, you are MOST likely to advise the mother to restrict his television viewing

A. to 1 hour per day

B. to 2 hours per day

C. to commercial television programs

D. to nonviolent programs

E. until he is 2 years old

Copyright 2010 by the American Academy of Pediatrics page 215


2011 PREP SA on CD-ROM

Critique: 68 Preferred Response: E

Many parents seek to provide rich opportunities for learning to their young children, including
older infants. Programs that improve literacy and language have been proven to be successful
and depend on warm, close interaction between parent and child. However, there is no
evidence to support educational television or computer programs to improve literacy and
language development for the very young child. Thus, television viewing should be deferred until
the child is at least 2 years of age.
The parents of the child described in the vignette can be reassured that no research data
show that specific television programs have increased childrens academic readiness and that
interactive play as well as traditional manipulative toys are preferred. In addition to television
avoidance until age 2 years, solitary television viewing should be discouraged in young children.
Children of all ages should have television, Internet, video game, and other electronic device
"screen time" limited to no more than 2 hours per day. Many children watch television 3 to 4
hours per day on weeknights and 6 to 12 hours per day on weekends.

Suggested reading:

Bijvank MN, Konijn EA, Bushman BJ, Roelofsma PHMP. Age and violent-content labels make video
games forbidden fruits for youth. Pediatrics. 2009;123:870-876. DOI: 10.1542/peds.2008-0601.
Available at: http://pediatrics.aappublications.org/cgi/content/full/123/3/870

Chandra A, Martino SC, Collins RL, et al. Does watching sex on television predict teen
pregnancy? Findings from a national longitudinal survey of youth. Pediatrics. 2008;122:1047-
1054. DOI: 10.1542/peds.2007-3066. Available at:
http://pediatrics.aappublications.org/cgi/content/full/122/5/1047

Christakis, DA, Zimmerman FJ. Violent television viewing during preschool is associated with
antisocial behavior during school age. Pediatrics. 2007;120: 993-999. DOI: 10.1542/peds.2006-
3244. Available at: http://pediatrics.aappublications.org/cgi/content/full/120/5/993

Council on Communications and Media. Policy statement: media violence. Pediatrics.


2009;124:1495-1503. DOI: 10.1542/peds.2009-2146. Abstract available at:
http://pediatrics.aappublications.org/cgi/content/full/124/5/1495

Funk JB, Brouwer J, Curtiss K, McBroom E. Parents of preschoolers: expert media


recommendations and ratings knowledge, media-effects beliefs, and monitoring practices.
Pediatrics. 2009;123:981-988. DOI: 10.1542/peds.2008-1543. Available at:
http://pediatrics.aappublications.org/cgi/content/full/123/3/981

Sargent JD, Beach ML, Dalton MA, et al. Effect of parental R-rated movie restriction on
adolescent smoking initiation: a prospective study. Pediatrics. 2004;114:149-156. Available at:
http://pediatrics.aappublications.org/cgi/content/full/114/1/149

Copyright 2010 by the American Academy of Pediatrics page 216


2011 PREP SA on CD-ROM

Zuckerman B. Promoting early literacy in pediatric practice: 20 years of Reach Out and Read.
Pediatrics. 2009;124:1660-1665. DOI: 10.1542/peds.2009-1207. Available at:
http://pediatrics.aappublications.org/cgi/content/full/124/6/1660

Copyright 2010 by the American Academy of Pediatrics page 217


2011 PREP SA on CD-ROM

Question: 69

A 14-year-old boy loses consciousness while playing basketball. He regains consciousness in 30


seconds and is transported to a pediatric emergency department. Results of head computed
tomography scan, electroencephalography, and echocardiography are within normal limits.
Electrocardiography results are interpreted as abnormal, with a heart rate of 90 beats/min, PR
interval of 150 msec, and QTc interval of 550 (Item Q69).

Of the following, the MOST likely explanation for this patients syncopal episode is

A. complete atrioventricular block

B. first-degree atrioventricular block

C. hypertrophic cardiomyopathy

D. long QT syndrome

E. supraventricular tachycardia due to Wolff-Parkinson-White syndrome

Copyright 2010 by the American Academy of Pediatrics page 218


2011 PREP SA on CD-ROM

Question: 69

(Courtesy of M Lewin)
Lead II from the electrocardiogram, as described for the boy in the vignette.

Copyright 2010 by the American Academy of Pediatrics page 219


2011 PREP SA on CD-ROM

Critique: 69 Preferred Response: D

The young child or adolescent who experiences an episode of syncope must undergo 12-
lead electrocardiography (ECG) as a part of his or her evaluation. Findings on ECG may indicate
the possibility of a rhythm disturbance or conduction disorder. However, the corrected QT
interval must be measured to assess for the possible diagnosis of long QT syndrome. Findings
on ECG almost always are abnormal in the patient who has symptomatic long QT syndrome. In
addition to a prolonged corrected QT interval, there may be bizarre or notched T waves and
prominent U waves, as shown for the boy in the vignette. Exercise testing may elicit
abnormalities not seen on resting ECG. Patients who have long QT syndrome are at risk for life-
threatening ventricular tachycardia, torsades de pointes, and ventricular fibrillation. The
syndrome may have an autosomal dominant or autosomal recessive inheritance pattern or may
be a new mutation. Many of the mutations causing long QT syndrome demonstrate ion channel
abnormalities. Clinical laboratory testing is available. Pharmacologic therapy and implantation of
automatic cardiovertor-defibrillators are the currently employed treatment modalities. Affected
patients may present with cardiac arrest, syncope, seizures, or palpitations. Any patient who
presents with suspicious symptoms in whom ECG identifies a corrected QT interval greater than
450 msec warrants specialty evaluation.
Patients who have complete atrioventricular block also may present with syncope.
However, the teenager who has complete atrioventricular block would have a resting heart rate
dramatically lower than 70 beats/min (typically in the range of 40 to 60 beats/min), and the ECG
would demonstrate a profound conduction disturbance characterized by a lack of relationship
between the atrial and ventricular rates. It is atypical for first-degree atrioventricular block to
result in syncope. In addition, first-degree block is identified easily on baseline ECG by the PR
interval exceeding approximately 180 msec.
Hypertrophic cardiomyopathy can present with syncope due to either obstruction of left
ventricular outflow and resultant hypotension or ventricular arrhythmias caused by the
disturbance to repolarization. However, the corrected QT interval is not markedly prolonged. In
addition, the findings of hypertrophic cardiomyopathy are readily discerned by
echocardiography. Supraventricular tachycardia due to Wolff-Parkinson-White syndrome can
result in syncope because patients are at risk for degeneration of their arrhythmia to atrial and
subsequently ventricular fibrillation. However, baseline ECG should demonstrate the classic
features of a short PR interval and a delta wave.

Suggested reading:

Cardiac dysrhythmias and sports. American Academy of Pediatrics Committee on Sports


Medicine and Fitness. Pediatrics. 1995;95:786-788. Abstract available at:
http://pediatrics.aappublications.org/cgi/content/abstract/95/5/786

Clarke CJ, McDaniel GM. The risk of long QT syndrome in the pediatric population. Curr Opin
Pediatr. 2009;21:573-578. DOI: 10.1097/MOP.0b013e3283307ae2. Abstract available at:
http://www.ncbi.nlm.nih.gov/pubmed/19617828

Copyright 2010 by the American Academy of Pediatrics page 220


2011 PREP SA on CD-ROM

Wren C. Screening for potentially fatal heart disease in children and teenagers. Heart.
2009;95:2040-2046;. DOI: 10.1136/hrt.2009.172858. Extract available at:
http://heart.bmj.com/content/95/24/2040.extract

Copyright 2010 by the American Academy of Pediatrics page 221


2011 PREP SA on CD-ROM

Question: 70

A 7-year-old girl who has chronic sinusitis presents to the emergency department with 1 to 2
weeks of headaches that are worse at night. On physical examination, she is afebrile but seems
uncharacteristically sleepy and exhibits hyperreflexia. Funduscopic examination shows
papilledema (Item Q70).

Of the following, the MOST appropriate next diagnostic test is

A. electroencephalography

B. head computed tomography scan with contrast

C. lumbar puncture

D. magnetic resonance venography

E. visual evoked potentials

Copyright 2010 by the American Academy of Pediatrics page 222


2011 PREP SA on CD-ROM

Question: 70

(Courtesy of RG Weaver, Jr)

Copyright 2010 by the American Academy of Pediatrics page 223


2011 PREP SA on CD-ROM

Critique: 70 Preferred Response: B

The girl described in the vignette has a common problem, sinusitis with headache, but she
has developed the critical symptom of nocturnal awakening due to head pain that requires
thoughtful investigation. A complaint of headache that is worse after a recumbent period raises
the possibility of elevated intracranial pressure due to a focal intracranial process. During sleep,
intracranial pressure is naturally higher due to increased pooling of cerebrospinal fluid (CSF) and
blood in the cranium. Therefore, an intracranial mass may present initially with worsening pain at
night. The girls physical examination reveals mental status changes, hyperreflexia, and
papilledema. This constellation of findings raises concerns for elevated intracranial pressure due
to an abscess, a known complication of sinusitis, and a head computed tomography (CT) scan
with contrast should be performed.
Electroencephalography is useful in situations where mental status changes are suspected
to be caused by ongoing nonconvulsive seizures. However, in this case, the headache and
signs of increased intracranial pressure mandate structural, not functional, diagnostic testing
initially. A lumbar puncture should not be obtained prior to imaging because removing CSF and
reducing pressure below the foramen magnum can cause expanded intracranial contents to
herniate downward.
Magnetic resonance imaging (MRI) with venography (MRV) is the study of choice to rule out
an intracranial venous sinus thrombosis. Although findings in this case are consistent with that
diagnosis, it is less likely than a brain abscess. More importantly, the need for sedation, time,
cost, and availability make CT scan preferred over MRI/MRV. Visual evoked potentials are used
occasionally to assess the physiologic integrity of visual pathways from the retina to occipital
cortex. This test is not used emergently to assess for structural causes of mental status
changes, headache, or hyperreflexia.

Suggested reading:

Cochrane DD. Consultation with the specialist: brain abscess. Pediatr Rev. 1999;20:209-215.
DOI: 10.1542/10.1542/pir.20-6-209. Available at:
http://pedsinreview.aappublications.org/cgi/content/full/20/6/209

Goodkin HP, Harper MB, Pomeroy SL. Intracerebral abscess in children: historical trends at
Children's Hospital Boston. Pediatrics. 2004;113:1765-1770. Available at:
http://pediatrics.aappublications.org/cgi/content/full/113/6/1765

Haslam RHA. Brain abscess. In: Kliegman RM, Behrman RE, Jenson HB, Stanton BF, eds. Nelson
Textbook of Pediatrics. 18th ed. Philadelphia, Pa: Saunders Elsevier; 2007:2524

Kan L, Nagelberg J, Maytal J. Headaches in a pediatric emergency department: etiology, imaging,


and treatment. Headache. 2000;40:25-29. DOI: 10.1046/j.1526-4610.2000.00004.x. Abstract
available at: http://www.ncbi.nlm.nih.gov/pubmed/10759899

Piatt JH Jr. Recognizing neurosurgical conditions in the pediatrician's office. Pediatr Clin North

Copyright 2010 by the American Academy of Pediatrics page 224


2011 PREP SA on CD-ROM

Am. 2004;51:237-270. DOI: 10.1016/S0031-3955(03)00213-X. Abstract available at:


http://www.ncbi.nlm.nih.gov/pubmed/15062671

Copyright 2010 by the American Academy of Pediatrics page 225


2011 PREP SA on CD-ROM

Question: 71

The parents of one of your patients, who has spina bifida, would like to have a second child.
They want to do everything possible to reduce the risk of having another baby affected by a
neural tube defect. You discuss the benefits of maternal folic acid supplementation, citing the
recommendations from the Centers for Disease Control and Prevention.

Of the following, the percent reduction in neural tube defects that has occurred since
implementation of these recommendations is CLOSEST to

A. 5% to 15%

B. 15% to 25%

C. 25% to 50%

D. 50% to 70%

E. 70% to 90%

Copyright 2010 by the American Academy of Pediatrics page 226


2011 PREP SA on CD-ROM

Critique: 71 Preferred Response: C

Neural tube defects (NTDs) are among the most common of the birth defects, with a
worldwide incidence of 1 per 1,000 births. They are caused by both environmental and genetic
factors. Of the environmental factors that play a role in neural tube closure, folic acid has
proven to be the most significant thus far.
Folic acid is a water-soluble B vitamin (vitamin B9) that is essential to humans and is
abundant in leafy, green vegetables; legumes (such as peas and beans); and some seeds (such
as sunflower). Folate, the naturally occurring form of folic acid, plays an important role in
conditions involving rapid cell division, such as pregnancy. Folate also is important in purine
production, which is required for DNA and RNA synthesis and repair, as well as other
biochemical pathways.
Numerous well-designed studies have shown that oral supplementation with 0.4 mg/day of
folic acid from 1 month prior to conception through the first trimester in women who have not
had a previous child affected by an NTD reduces the current fetus NTD risk by up to 75%. For
women who have had a previously affected child, a dose of 4.0 mg/day is recommended for the
same time period and reduces the recurrence risk by up to 75%.
Despite the previously cited recommendations for supplementation, the expected decline in
NTD rate did not occur, and in 1998, mandatory fortification of cereal grain products with folic
acid went into effect in the United States. Unfortunately, the hoped-for reduction in NTD rate has
not occurred since that time. Instead, the reduction in rate of NTDs has been approximately 25%
to 50%. Factors contributing to this lower reduction rate include lack of patient education,
reduced patient compliance, and barriers to prenatal care.
NTD research continues, with a focus on genes and epigenetic (factors that influence gene
expression) variables that play a role in NTDs. The role(s) of folic acid in these processes
largely remain(s) a mystery.

Suggested reading:

Blom HJ. Folic acid, methylation, and neural tube closure in humans. Birth Defects Res A Clin
Mol Teratol. 2009;85:295-302. DOI: 10.1002/bdra.20581. Abstract available at:
http://www.ncbi.nlm.nih.gov/pubmed/19301298

Hunter AGW. Disorders of neural tube closure. In: Stevenson RE, Hall JG, eds. Human
Malformations and Related Anomalies. 2nd ed. Oxford, United Kingdom: Oxford University
Press; 2006:715-756

Melvin EC, George TM, Worley G, et al. Genetic studies in neural tube defects. NTD Collaborative
Group. Pediatr Neurosurg. 2000;32:1-9. DOI: 10.1159/000028889. Abstract available at:
http://www.ncbi.nlm.nih.gov/pubmed/10765131

Spina bifida and anencephaly before and after folic acid mandateUnited States, 1995-1996 and
1999-2000. MMWR Morbid Mortal Wkly Rep. 2004;53:362-365. Available at:
http://www.cdc.gov/mmwr/preview/mmwrhtml/mm5317a3.htm

Copyright 2010 by the American Academy of Pediatrics page 227


2011 PREP SA on CD-ROM

Question: 72

You receive a call from the radiologist, who states that one of your 17-year-old female patients
had radiography to rule out a fracture on which he noted osteopenia. She was in your office 1
month ago with complaints of tiredness. She denied any excessive exercise, body image
concerns, or weight loss. At that time, her body mass index was 20.2 kg/m2, and physical
examination findings were normal. In reviewing her chart, you note that she achieved menarche
at age 15 years but has had only two light bleeding episodes since then.

Of the following, the MOST likely cause of this girls osteopenia is

A. decreased overall caloric intake

B. increased caffeine and soda intake

C. lack of weightbearing exercise

D. primary ovarian insufficiency

E. rickets from vitamin D deficiency

Copyright 2010 by the American Academy of Pediatrics page 228


2011 PREP SA on CD-ROM

Critique: 72 Preferred Response: D

Osteopenia in a previously healthy female usually is the result of a hypoestrogenic state.


Common causes include malnutrition, as in anorexia nervosa and female athlete triad (disordered
eating, amenorrhea, and osteoporosis), or medication-induced, as with depot
medroxyprogesterone acetate. Primary ovarian insufficiency, previously referred to as
premature ovarian failure, is another cause of a hypoestrogenic state that may be more common
than initially recognized. It represents a continuum of impaired ovarian function whose cause is
uncertain in most cases. Not all patients have profound estrogen deficiency and, therefore,
symptoms may be variable, as in this patient, with her complaints of tiredness but otherwise
normal examination findings. The girl experienced a relatively late onset of menarche (median
age of menarche in the United States is 12.43 years) and now has secondary amenorrhea.
Patients who have at least 4 months of amenorrhea, often preceded by a prodrome of irregular
menstrual cycles that may be infrequent or more frequent, warrant measurement of follicle-
stimulating hormone to rule out this condition before being placed on any hormonal medications to
regulate their menses or for contraception.
Imbibing cola and caffeine-containing drinks, lack of weightbearing exercise, and vitamin D
deficiency may affect bone density, but they do not alter menstrual cycles. The lack of weight
loss and a normal body mass index reported for this girl make decreased caloric intake unlikely.

Suggested reading:

Bloomfield D. In brief: secondary amenorrhea. Pediatr Rev. 2006;27:113-114. DOI:


10.1542/10.1542/pir.27-3-113. Available at:
http://pedsinreview.aappublications.org/cgi/content/full/27/3/113

Emans SJ. Delayed puberty. In Emans SJH, Laufer MR, Goldstein DP, eds. Pediatric and
Adolescent Gynecology. 5th ed. Philadelphia, Pa: Lippincott Williams & Wilkins; 2005:181-213

Nelson LM. Clinical practice. Primary ovarian insufficiency. N Engl J Med. 2009;360:606-614.
DOI: 10.1056/NEJMcp0808697. Available at:
http://www.ncbi.nlm.nih.gov/pmc/articles/PMC2762081/?tool=pubmed

Popat VB, Calis KA, Vanderhoof VH, et al. Bone mineral density in estrogen-deficient young
women. J Clin Endocrinol Metab. 2009;94:2277-2283. Abstract available at:
http://www.ncbi.nlm.nih.gov/pubmed/19401379

Copyright 2010 by the American Academy of Pediatrics page 229


2011 PREP SA on CD-ROM

Question: 73

You are evaluating a 2-year-old girl in the emergency department whose mother found her in the
bathroom 6 hours ago with an open bottle of drain cleaner. According to the mother, the girl initially
was coughing and had two episodes of emesis, but she has improved and is only "drooling and
gagging a little bit." On physical examination, the patient has no visible mouth or pharyngeal burns
and no stridor, hoarseness, or other signs of respiratory distress.

Of the following, the next BEST step in management is

A. administration of a neutralizing agent

B. administration of corticosteroids

C. administration of oral fluids

D. observation for additional symptoms

E. upper gastrointestinal tract endoscopy

Copyright 2010 by the American Academy of Pediatrics page 230


2011 PREP SA on CD-ROM

Critique: 73 Preferred Response: E

Ingestion of caustic agents remains a significant pediatric medical issue. Most ingestions are
seen in the 1- to 3-year age group, have a small male predominance, and usually are accidental
and small volume. Symptoms following ingestion include dysphagia, drooling, abdominal pain,
hematemesis, and respiratory distress. Esophageal injuries account for most complications and
are estimated to occur in up to 45% of caustic ingestions. Unfortunately, neither the presence of
oral burns nor specific symptoms predicts esophageal injury. Therefore, children who have a
history of ingestion and symptoms such as those described for the child in the vignette should
be referred for endoscopy. Ideally, endoscopy should be performed more than 6 hours after the
ingestion to document the full extent of injury but before 4 days postingestion to minimize the risk
of perforation.
Administration of neutralizing agents such as sodium bicarbonate or vinegar should be
avoided because the heat resulting from the chemical reaction may damage tissue further. In
addition, attempted dilution with fluids should be avoided because of the potential for induced
emesis. It is estimated that approximately 20% of patients develop esophageal strictures, but
corticosteroid administration has not been shown to reduce their formation and is not
recommended routinely. Because the child in the vignette has a clear history and symptoms
consistent with a caustic ingestion, simple observation is not appropriate.

Suggested reading:

Ferry GD. Caustic esophageal injury in children. UpToDate Online 17.3. 2008. Available at:
http://www.utdol.com/online/content/topic.do?topicKey=pedigast/11441&selectedTitle=2~25&so
urce=search_result

Powers KS, Cholette KM, Abboud P. Toxic exposures: diagnosis and management. In: Wheeler
DS, Wong HR, Shanley TP, eds. Pediatric Critical Care Medicine: Basic Science and Clinical
Evidence. New York, NY: Springer-Verlag; 2007:1621-1641

Copyright 2010 by the American Academy of Pediatrics page 231


2011 PREP SA on CD-ROM

Question: 74

A 9-month-old girl is brought to the emergency department by her middle eastern immigrant
parents, who have observed an episode of twitching of her extremities. On physical examination,
the infant has prominent wrists (Item Q74) and ankles and an open fontanelle. The parents tell
you through an interpreter that she is exclusively breastfed and neither she nor her mother takes
vitamins. You note that the mother is partially veiled.

Of the following, the MOST likely cause of the twitching is

A. hypercalcemia

B. hypocalcemia

C. hypomagnesemia

D. hypophosphatemia

E. vitamin D deficiency

Copyright 2010 by the American Academy of Pediatrics page 232


2011 PREP SA on CD-ROM

Question: 74

(Courtesy of M Rimsza)
Prominent wrists, as described for the infant in the vignette.

Copyright 2010 by the American Academy of Pediatrics page 233


2011 PREP SA on CD-ROM

Critique: 74 Preferred Response: B

The child described in the vignette has clinical signs of rickets, and her mother is protected
from sunlight by veiling. Neither mother nor child takes supplemental vitamins. Therefore, the
child likely has vitamin D deficiency as a result of poor stores at birth and continued poor vitamin
D intake and production. However, vitamin D deficiency alone does not cause the twitching
reported for the girl. Twitching is a sign of hypocalcemia caused by vitamin D deficiency.
Hypocalcemia induces neuromuscular irritability that can manifest as a positive Chvostek
sign, carpopedal spasm, or a positive Trousseau sign. Approximately 10% of individuals who
have normal calcium concentrations have positive Chvostek signs. A positive Trousseau sign is
induced by the tissue hypoxia caused by a tight blood pressure cuff and causes enough
discomfort that this test rarely is performed when a laboratory assessment of calcium is so
easily confirmatory. Severe hypocalcemia induces paresthesias (oral, finger, and toe tingling),
twitching, and seizures. Hypocalcemia also can lead to diarrhea. One of the most common
causes of hypocalcemia is vitamin D deficiency rickets, either when it is very severe or during
the initial phases of recovery when calcium is being taken up rapidly by healing bone.
Hypercalcemia can cause slowed mentation, stupor, constipation, polyuria, renal calculi, and
extreme thirst but does not cause twitching. Hypomagnesemia may cause neuromuscular
irritability similar to that seen in hypocalcemia but is much less common and is accompanied by
nausea and loss of appetite. Magnesium interferes with release of stored parathyroid hormone
and, therefore, can cause hypocalcemia. The signs and history typical for vitamin D deficiency
rickets reported for this girl make this less probable. Hypophosphatemia causes muscle
weakness and changes in mental status. It also may be seen in rickets but is not associated
with neuromuscular irritability.

Suggested reading:

Agus Z. Clinical manifestations of hypercalcemia. UpToDate Online 17.3. 2009. Available at:
http://www.utdol.com/online/content/topic.do?topicKey=calcium/5007&selectedTitle=4%7E150&s
ource=search_result819

Agus ZS. Diagnosis and treatment of hypophosphatemia. UpToDate Online 17.3. 2009. Available
at:
http://www.uptodateonline.com/online/content/topic.do?topicKey=calcium/5885&selectedTitle=37
%7E150&source=search_result

Agus ZS. Diagnosis and treatment of hypomagnesemia. UpToDate Online 17.3. 2008. Available
at:
http://www.uptodateonline.com/online/content/topic.do?topicKey=calcium/5547&selectedTitle=9
%7E150&source=search_result

Fitzpatrick LA. Hypocalcemia: diagnosis and treatment. Endotext.org. 2002. Available at:
http://www.endotext.org/parathyroid/parathyroid7/parathyroid7.htm

Copyright 2010 by the American Academy of Pediatrics page 234


2011 PREP SA on CD-ROM

Misra M, Pacaud D, Petryk A, Collett-Solberg PF, Kappy M; Drug and Therapeutics Committee of
the Lawson Wilkins Pediatric Endocrine Society. Vitamin D deficiency in children and its
management: review of current knowledge and recommendations. Pediatrics. 2008;122:398-
417. DOI: 10.1542/peds.2007-1894. Available at:
http://pediatrics.aappublications.org/cgi/content/full/122/2/398

Singhai A, Campbell DE. Hypocalcemia. eMedicine Specialties, Pediatrics: General Medicine,


Endocrinology. 2009. Available at: http://emedicine.medscape.com/article/921844-overview

Wagner CL, Greer FR; Section on Breastfeeding and Committee on Nutrition. Prevention of
rickets and vitamin D deficiency in infants, children, and adolescents. Pediatrics. 2008;122:1142-
1152. DOI: 10.1542/peds.2008-1862. Available at:
http://pediatrics.aappublications.org/cgi/content/full/122/5/1142

Copyright 2010 by the American Academy of Pediatrics page 235


2011 PREP SA on CD-ROM

Question: 75

Results of a cognitive test given to a 7-year-old girl indicate functioning in the moderate intellectual
disabilities range (intelligence quotient between 40 and 50). Her parents ask what the future
holds for their child academically and vocationally.

Of the following, the MOST likely expectation is that the child

A. can achieve a third- to sixth-grade reading level by adolescence

B. can be employed in a competitive unskilled or semiskilled job

C. can work in a sheltered workshop that provides close supervision

D. will only be able to learn to read simple signs such as stop and exit

E. will require help with activities of daily living from caregivers when an adult

Copyright 2010 by the American Academy of Pediatrics page 236


2011 PREP SA on CD-ROM

Critique: 75 Preferred Response: C

The American Association on Intellectual and Developmental Disabilities (AAIDD) defines


intellectual disability (ID) as a disability "characterized by significant limitations both in intellectual
function and in adaptive behavior as expressed in conceptual, social, and practical adaptive skill.
This disability originates before the age of 18."
Children who have mild ID (IQ, 50-55 to 70) and no comorbid disorders are self-sufficient in
activities of daily living (ADL) and communication. They can be expected to learn at one half to
two thirds normal velocity. They can reach a third- to sixth-grade reading level by late
adolescence and can be expected to be employed in competitive unskilled, semiskilled, or in
some cases, skilled jobs. The prevalence rate of mild ID is 20 to 30 per 1,000, and affected
children often are identified in kindergarten and early elementary school.
Individuals who have moderate ID (IQ, 35-40 to 50-55), such as the girl described in the
vignette, are able to do ADL and express basic needs. They learn at one third to one half
velocity. They can be expected to achieve a first- to third-grade reading level. They may be able
to function in a supportive employment setting, but more often they work in sheltered workshops
that provide constant supervision. In adulthood, they often live in group homes. The prevalence
rate of moderate ID is 5 per 1,000, and the condition most often is identified in preschool.
Children who have severe ID (IQ, 20-25 to 35-40) have limited language skills and need
support with ADL. They might be able to learn to read simple signs such as stop and exit. They
will continue to need help with ADL from caregivers as adults. The prevalence rate of severe ID
is 3 per 1,000, and affected children are identified before age 3 years. These individuals often
have identified genetic, medical, and neurologic causes.
Children who have profound ID (IQ, <20-25) require assistance for ADL. They have the
highest rates of identified genetic, medical, and neurologic causes. The prevalence rate of
profound ID is 1 to 2 per 1,000, and affected children are identified prior to age 2 years.

Suggested reading:

Acharya K, Msall ME. The spectrum of cognitive-adaptive developmental disorders in intellectual


disability. In: Accardo PJ, ed. Capute & Accardos Neurodevelopmental Disabilities in Infancy
and Childhood. 3rd ed. Baltimore, Md: Paul H. Brookes Publishing Co; 2008:241-260

American Association on Intellectual and Developmental Disabilities web site. Available at:
http://www.aaidd.org

Johnson CP, Walker WO Jr. Mental retardation: management and prognosis. Pediatr Rev.
2006;27:249-256. DOI: 10.1542/10.1542/pir.27-7-249. Available at:
http://pedsinreview.aappublications.org/cgi/content/full/27/7/249

Walker WO Jr, Johnson CP. Mental retardation: overview and diagnosis. Pediatr Rev.
2006;27:204-212. DOI: 10.1542/10.1542/pir.27-6-204. Available at:
http://pedsinreview.aappublications.org/cgi/content/full/27/6/204

Copyright 2010 by the American Academy of Pediatrics page 237


2011 PREP SA on CD-ROM

Question: 76

A 1-year-old boy is hospitalized following 10 days of temperatures to 40.6C. Physical


examination is unrevealing. His white blood cell count is 32x103/mcL (32x109/L), with 82%
neutrophils, 6% bands, and 12% lymphocytes. Computed tomography scan of the chest shows a
left upper lobe infiltrate and enlarged mediastinal lymph nodes. Surgical biopsy reveals necrotizing
granuloma, and cultures of the biopsy specimen grow Acinetobacter anitratus.

Of the following, the BEST test for confirming the diagnosis of immunodeficiency in this child is

A. human immunodeficiency virus serology

B. immunoglobulin E concentration

C. immunoglobulin G concentration

D. oxidative burst test

E. T-cell subset assessment

Copyright 2010 by the American Academy of Pediatrics page 238


2011 PREP SA on CD-ROM

Critique: 76 Preferred Response: D

The severe infection with an unusual, low-grade pathogen described for the child in the
vignette suggests an immune deficiency. Acinetobacter anitratus is a catalase-positive enteric
gram-negative rod. Infection with a catalase-positive organism and the presence of necrotizing
granulomas on pathology is most consistent with chronic granulomatous disease (CGD). CGD is
characterized by a defect in the phagocytic NADPH oxidase system that prevents phagocytes
from adequately killing catalase-positive organisms, including Staphylococcus aureus and a
number of enteric gram-negative rods (including Salmonella, Serratia marcescens,
Burkholderia cepacia), Aspergillus, and Nocardia. In the oxidative burst test, the ability of the
patients white blood cells (WBCs) to oxidize a rhodamine dye is assessed by flow cytometry.
Failure of the WBCs to perform this reaction is indicative of CGD.
Human immunodeficiency virus (HIV) infection may present with an opportunistic infection,
but the infection described in the vignette is not typical of symptomatic HIV infection. In HIV, the
primary immune defects are in T-cell and immunoglobulin function. Opportunistic infections
reflecting these immune deficits include severe infections with encapsulated bacteria (eg,
Streptococcus pneumoniae), Pneumocystis jiroveci, viruses (eg, cytomegalovirus, varicella,
herpes simplex virus), Mycobacterium avium complex, and Candida albicans.
Hyperimmunoglobulin E (Job syndrome) is characterized by recurrent staphylococcal
abscesses, sinopulmonary infections, and severe eczema. Facial anomalies also may be
present. Abnormalities in immunoglobulin G concentrations can be seen in a spectrum of
disorders ranging from severe combined immunodeficiency disorder to transient
hypogammaglobulinemia of infancy. Infections associated with immunoglobulin G deficiencies
generally are caused by encapsulated bacteria (eg, S pneumoniae, Haemophilus influenzae).
Abnormalities in T-cell subsets, including HIV infection, present with viral, fungal, or opportunistic
infections.

Suggested reading:

Lekstrom-Himes JA, Gallin JI. Immunodeficiency diseases caused by defects in phagocytes. N


Engl J Med. 2000;343:1703-1714. Extract Available at:
http://content.nejm.org/cgi/content/extract/343/23/1703

Opsimos H, Dadiz R, Schroeder SA, et al. Ten-month-old boy with persistent fever and a chest
mass. J Pediatr. 2005;146:267272. DOI: 10.1016/j.jpeds.2004.10.027

Roberts RL, Bonilla FA. Primary disorders of phagocytic function: an overview. UpToDate Online
17.3. 2009. Available at:
http://www.uptodate.com/online/content/topic.do?topicKey=immunnon/13290&selectedTitle=1%7
E6&source=search_result819

Copyright 2010 by the American Academy of Pediatrics page 239


2011 PREP SA on CD-ROM

Question: 77

A 6-year-old girl presents to the emergency department with fever, dysuria, and vomiting.
Physical examination reveals a temperature of 39.0C, heart rate of 110 beats/minute, dry mucous
membranes, and mild left-sided flank pain. The white blood cell count is 15.0x103/mcL
(15.0x109/L), with 55% polymorphonuclear leukocytes, 30% lymphocytes, and 15% monocytes.
A urine culture and urinalysis were obtained at the pediatricians office yesterday and oral
trimethoprim-sulfamethoxazole was prescribed. The urinalysis was positive for leukocytes and
nitrites, and the laboratory reports today that the urine culture is growing a gram-positive
organism.

Of the following, the MOST appropriate antibiotic to treat the girls infection is

A. ampicillin

B. cefazolin

C. ceftriaxone

D. ciprofloxacin

E. gentamicin

Copyright 2010 by the American Academy of Pediatrics page 240


2011 PREP SA on CD-ROM

Critique: 77 Preferred Response: A

Urinary tract infections (UTIs) are among the most common serious bacterial infections
encountered in pediatrics. Empiric antimicrobial therapy for children who have UTIs depends on
the antimicrobial resistance patterns of the isolates in the community, the severity of the patients
illness, and whether the patient has an underlying medical or urologic condition. Definitive
antimicrobial therapy is based on the susceptibility pattern of the urine culture isolate.
Gram-negative organisms of the gastrointestinal tract are the most common pathogens of
UTI. Escherichia coli cause 75% to 90% of UTIs in children. Other gram-negative pathogens
include Citrobacter, Enterobacter, Klebsiella, Proteus, and Pseudomonas. Gram-positive
causative bacteria include Enterococcus and, in sexually active females, Staphylococcus
saprophyticus. Staphylococcus aureus is rare and usually is associated with complicated UTIs.
The gram-positive pathogen reported on urine culture for the young girl described in the
vignette suggests that her pyelonephritis is caused by Enterococcus. Intravenous ampicillin is
appropriate antimicrobial therapy because most community-acquired isolates of Enterococcus
are susceptible. Addition of an aminoglycoside would be appropriate for patients who have
invasive enterococcal infections (eg, endocarditis). Ampicillin and amoxicillin are semisynthetic
beta-lactam antibiotics that can have activity against some gram-negative organisms such as
Escherichia coli, Haemophilus influenzae, Proteus mirabilis, Salmonella, and Shigella. In
addition, they are active against gram-positive penicillin-susceptible bacteria. Resistance to
ampicillin and amoxicillin occurs via beta-lactamase production or alterations in penicillin-binding
proteins.
About 50% of all E coli are resistant to amoxicillin or ampicillin, but susceptibility patterns
can differ greatly among communities. Increasing rates of E coli resistance to first-generation
cephalosporins, such as cefazolin, and trimethoprim-sulfamethoxazole have been reported.
Some organisms, usually in children receiving prophylactic antibiotics, demonstrate resistance to
multiple antibiotics because of extended-spectrum beta-lactamase production. In general,
second- and third-generation cephalosporins (eg, ceftriaxone) and aminoglycosides such as
gentamicin are appropriate empiric antibiotics for the treatment of UTI because most infections
are caused by gram-negative organisms. However, cephalosporins have no activity against
Enterococcus. Ciprofloxacin is appropriate therapy for treating UTIs caused by Pseudomonas or
other multidrug-resistant gram-negative pathogens but should not be used as a first-line agent.

Suggested reading:

Chang SL, Shortliffe LD. Pediatric urinary tract infections. Pediatr Clin North Am. 2006;53:379-
400. DOI: 10.1016/j.pcl.2006.02.011. Abstract available at:
http://www.ncbi.nlm.nih.gov/pubmed/16716786

Committee on Infectious Diseases. The use of systemic fluoroquinolones. Pediatrics.


2006;118:1287-1292. DOI: 10.1542/peds.2006-1722. Available at:
http://pediatrics.aappublications.org/cgi/content/full/118/3/1287

Michelow IC, McCracken GH Jr. Antibacterial therapeutic agents. In: Feigin RD, Cherry JD,

Copyright 2010 by the American Academy of Pediatrics page 241


2011 PREP SA on CD-ROM

Demmler-Harrison GJ, Kaplan SL, eds. Feigin & Cherrys Textbook of Pediatric Infectious
Diseases. 6th ed. Philadelphia, Pa: Saunders Elsevier; 2009:3178-3226

Raszka WV Jr, Khan O. Pyelonephritis. Pediatr Rev. 2005;26;364-370. DOI:


10.1542/10.1542/pir.26-10-364. Available at:
http://pedsinreview.aappublications.org/cgi/content/full/26/10/364

Copyright 2010 by the American Academy of Pediatrics page 242


2011 PREP SA on CD-ROM

Question: 78

A previously well 6-year-old boy presents with a rash over his lower extremities. He has had
knee and ankle pain for 3 days that has caused difficulty ambulating since this morning. On
physical examination, he is afebrile, and his heart rate is 80 beats/min, respiratory rate is 16
breaths/min, and blood pressure is 108/60 mm Hg. You note bilateral ankle swelling and a purpuric
rash over his lower extremities (Item Q78).

Laboratory findings include:


White blood cell count,10x10 3/mcL (10x109/L)
Hemoglobin, 11.5 g/dL (115 g/L)
Hematocrit, 35% (0.35)
Platelet count, 410x10 3/mcL (410x109/L)

Urinalysis shows:
Specific gravity, 1.025
pH, 6
3+ blood
Trace protein
20 to 50 red blood cells/high-power field

The random urine protein-to-creatinine ratio is 0.15.

Of the following, the MOST appropriate next step in the management of this patient is

A. administration of corticosteroids because of his multisystem involvement

B. administration of ibuprofen as the first-line treatment of joint symptoms

C. follow-up evaluation only if he develops gross hematuria

D. referral for renal biopsy

E. weekly screening for the development of significant proteinuria

Copyright 2010 by the American Academy of Pediatrics page 243


2011 PREP SA on CD-ROM

Question: 78

(Courtesy of A Bousvaros)
Rash, as described for the child in the vignette.

Copyright 2010 by the American Academy of Pediatrics page 244


2011 PREP SA on CD-ROM

Critique: 78 Preferred Response: E

The boy described in the vignette presents with a purpuric rash, a 3-day history of
musculoskeletal pain with ankle swelling, and laboratory findings of microscopic hematuria and a
normal complete blood count. The clinical diagnosis suggested by these findings is Henoch-
Schnlein purpura (HSP). HSP is classified as a small-vessel vasculitis and features involvement
of the joints, skin, gastrointestinal (GI) tract, and kidneys. The characteristic dermatologic
manifestation is a purpuric rash over the extensor surfaces (often the buttocks and lower
extremities) (Item C78). The GI involvement includes submucosal bleeding, which can present
clinically as abdominal pain, hematochezia, or intussusception. The musculoskeletal involvement
is characterized by arthralgias and arthritis, which can be severe enough to keep the patient
from walking. The renal manifestations are those of an immune complex-mediated
glomerulonephritis, which can vary clinically from asymptomatic hematuria with or without
proteinuria to more severe involvement, including nephrotic syndrome or acute
glomerulonephritis (AGN). Symptoms in patients who have AGN can range from cola-colored
urine with normal renal function to a severe, crescentic nephritis with marked azotemia at
presentation. Patients who have HSP and overt renal disease (nephrotic syndrome or a severe
nephritis) carry a worse renal prognosis. Although end-stage renal disease is uncommon in
children who have HSP, those whose renal involvement at the outset is more severe carry the
worst prognosis.
The goal of management for HSP is to recognize renal involvement early to reduce the
likelihood of long-term renal morbidity. Approximately one third of affected patients develop
urinary abnormalities (hematuria or proteinuria). However, it is common for children to have
normal urinalysis results at the time of clinical presentation and eventually exhibit
hematuria/proteinuria weeks later. Most patients who have renal involvement (85%) develop
urinary abnormalities within 4 weeks of disease onset and nearly all (97%) within 12 weeks.
Accordingly, children who have HSP and normal urinalysis results should undergo urinalysis
weekly until the systemic symptoms abate. Others have suggested obtaining a urinalysis every
other week for at least 2 months after disease onset, which, in some cases, could be a few
weeks after symptoms have resolved.
Microscopic hematuria with HSP is not concerning, but the development of proteinuria (2+ or
greater on a urinalysis or urine protein/creatinine ratio >0.20) is cause for concern and warrants
timely consultation with a pediatric nephrologist. Such consultation is necessary because
children who have HSP associated with hematuria and significant proteinuria require a renal
biopsy to assess the severity of glomerulonephritis and need for immunosuppressant
medication. The patient in the vignette does not require renal biopsy at this point because the
urinalysis does not yet show evidence of significant proteinuria. Screening for the development
of subclinical nephritis (microscopic hematuria and proteinuria) should be undertaken for all
patients who have HSP. Corticosteroid therapy is not indicated for joint symptoms or purpura.
One investigator demonstrated benefit with a 2-week course of corticosteroids as "prophylaxis"
against the development of renal disease among patients who had HSP and normal urinalysis
findings, but these results have not been confirmed in subsequent studies by other
investigators. There is no role for prophylactic corticosteroids for the patient in the vignette, who
already has abnormal urinalysis results. Nonsteroidal anti-inflammatory drugs, such as

Copyright 2010 by the American Academy of Pediatrics page 245


2011 PREP SA on CD-ROM

ibuprofen, should be used with caution in children at risk of renal disease, and they should be
avoided in the patient in the vignette, who already has microscopic hematuria, to prevent further
progression of early renal involvement.

Suggested reading:

Meadow SR, Glasgow EF, White RH, Moncrieff MW, Cameron JS, Ogg CS. Schnlein-Henoch
nephritis. Q J Med. 1972;41:241-258

Mollica F, Li Volti S, Garozzo R, Russo G. Effectiveness of early prednisone treatment in


preventing the development of nephropathy in anaphylactoid purpura. Eur J Pediatr.
1992;151:140-144. Abstract available at: http://www.ncbi.nlm.nih.gov/pubmed/1343079

Sano H, Izumida M, Shimizu H, Ogawa Y. Risk factors of renal involvement and significant
proteinuria in HenochSchnlein purpura. Eur J Pediatr. 2002;161:196201. Abstract available
at: http://www.ncbi.nlm.nih.gov/pubmed/12014385

Zaffanello M, Fanos V. Treatment-based literature of HenochSchnlein purpura nephritis in


childhood. Pediatr Neph 2009;24:1901-1911. Abstract available at:
http://www.ncbi.nlm.nih.gov/pubmed/19066976

Copyright 2010 by the American Academy of Pediatrics page 246


2011 PREP SA on CD-ROM

Critique: 78

(Courtesy of D Krowchuk)
Henoch-Schnlein purpura often involves the buttocks and lower extremities.

Copyright 2010 by the American Academy of Pediatrics page 247


2011 PREP SA on CD-ROM

Question: 79

An 18-year-old boy presents with a 3- to 4-year history of receiving antibiotics every other
month. The infections primarily have been sinusitis and pneumonia, but he has had one episode of
diarrhea due to Giardia lamblia. Computed tomography scan of his sinuses and lungs last month
showed pansinusitis and central bronchiectasis, respectively. His initial complete cell count, liver
function tests, and renal panel yield normal results, but he has low immunoglobulin A, M, and G
concentrations. T- and B-cell flow cytometry demonstrates normal counts for T and B cells.

Of the following, the MOST likely diagnosis is

A. common variable immunodeficiency

B. severe combined immunodeficiency

C. specific antibody deficiency

D. Wiskott-Aldrich syndrome

E. X-linked (Bruton) agammaglobulinemia

Copyright 2010 by the American Academy of Pediatrics page 248


2011 PREP SA on CD-ROM

Critique: 79 Preferred Response: A

Children or adolescents who experience recurrent, severe, persistent, or unusual infections


should be evaluated for a possible primary or secondary immunodeficiency. The history of
recurrent sinusitis, pneumonia, and decreased immunoglobulin concentrations described for the
boy in the vignette should prompt evaluation for a possible humoral immunodeficiency,
specifically, common variable immunodeficiency (CVID).
Although the specific pathophysiology for CVID still is not understood for most cases,
mutations in B-lymphocyte growth and regulation have been identified in 10% of affected
patients. Despite abnormalities in the B lymphocyte, patients generally have normal quantities of
B cells. Patients experience fewer infections after starting intravenous (IVIG) or subcutaneous
(SCIG) immunoglobulin, but they still may develop bronchiectasis and are at increased risk for
malignancies.
Bruton or X-linked agammaglobulinemia also presents with low-to-absent IgG, A, and M
concentrations, but in contrast to CVID, B cell concentrations also are low. The defect in Bruton
agammaglobulinemia, a mutation affecting the Btk tyrosine kinase, halts B cell development at the
pro- and pre-B cell in the bone marrow, with very few (usually <1%) B cells moving out of the
bone marrow. Patients classically present in early infancy (ie, 4 to 6 months of age), when
passive maternal antibody wanes and the B-cell defect prohibits mature B-cell and
immunoglobulin production in the infant.
Severe combined immunodeficiency most often is due to a defect in the common gamma
chain that is vital for intracellular signaling for interleukin (IL)-2, -4, -7, -9, -15, and -21 receptors.
The defect impairs cellular function due to low concentrations of T lymphocytes and natural killer
cells. Patients typically present within the first 6 months after birth with failure to thrive, chronic
diarrhea, and an eczematous rash that often is mistaken for atopic dermatitis. Immediate
recognition and diagnosis is crucial because bone marrow transplantation prior to age 4 months
of age is associated with improved survival.
Specific antibody deficiency (SAD) presents similarly to CVID, with sinusitis, pneumonia,
and impaired antibody responses to pneumococcal polysaccharide antigens. However, in
contrast to CVID, concentrations of IgA, IgM, and IgG are normal. The diagnosis of SAD is
controversial because clinicians differ in their interpretation of an appropriate vaccine response
to pneumococcal polysaccharide antigens. Consultation with an immunologist prior to initiation of
IVIG is strongly encouraged.
Wiscott-Aldrich syndrome (WAS) classically presents with the triad of severe eczema,
sinopulmonary infections, and thrombocytopenia, although only 5% to 10% of patients have all
three features at presentation. Concentrations of IgM may be low, although IgG, IgA, B-cell, and T-
cell concentrations are normal. WAS, an X-linked disease, is due to defects in the WAS protein
that is a regulator in actin polymerization and hematopoietic cell development. Treatment includes
stem cell transplantation, IVIG, management of bleeding diathesis, and antibiotics.
When evaluating low immunoglobulin values, clinicians also should screen for protein-losing
disorders (eg, nephrotic syndrome, liver failure, protein-losing enteropathy) as secondary
causes for hypogammaglobulinemia.

Suggested reading:

Copyright 2010 by the American Academy of Pediatrics page 249


2011 PREP SA on CD-ROM

Bonagura VR, Marchlewski R, Cox A, Rosenthal DW. Biologic IgG level in primary
immunodeficiency disease: the IgG level that protects against recurrent infection. J Allergy Clin
Immunol. 2008;122:210-211. DOI: 10.1016/j.jaci.2008.04.044. Available at:
http://www.jacionline.org/article/PIIS0091674908009421/fulltext

Boyle RJ, Le C, Balloch A, Tang ML. The clinical syndrome of specific antibody deficiency in
children. Clin Exp Immunol. 2006;146:486-492. DOI: 10.1111/j.1365-2249.2006.03242.x.
Available at: http://www3.interscience.wiley.com/cgi-bin/fulltext/118580087/HTMLSTART

International Union of Immunological Societies Expert Committee on Primary Immunodeficiencies,


Notarangelo LD, Fischer A, Geha RS, et al. Primary immunodeficiencies: 2009 update. J Allergy
Clin Immunol. 2009;124:1161-1178. DOI:10.1016/j.jaci.2009.10.013. Abstract available at:
http://www.ncbi.nlm.nih.gov/pubmed/20004777

Urschel S, Kayikci L, Wintergerst U, Notheis G, Jansson A, Belohradsky BH. Common variable


immunodeficiency disorders in children: delayed diagnosis despite typical clinical presentation. J
Pediatr. 2009;154:888-894. DOI: 10.1016/j.jpeds.2008.12.020. Available at:
http://www.jpeds.com/article/PIIS0022347608010809/fulltext

Copyright 2010 by the American Academy of Pediatrics page 250


2011 PREP SA on CD-ROM

Question: 80

Paramedics bring a 1-year-old child to the emergency department after her mother found her
under water in the bathtub. The mother had stepped away to answer the phone, and when she
returned, the child was face down in the tub. The mother administered three rescue breaths, and
when the paramedics arrived, the child was awake and alert and had normal vital signs. Physical
examination in the emergency department reveals no abnormal findings. You admit the child to the
hospital for observation, and 4 hours after the episode, she has increasing respiratory distress
and hypoxia.

Of the following, the MOST likely explanation for the childs respiratory signs and symptoms is

A. acute respiratory distress syndrome

B. aspiration pneumonia

C. bronchiolitis obliterans

D. chemical pneumonitis

E. pneumothorax

Copyright 2010 by the American Academy of Pediatrics page 251


2011 PREP SA on CD-ROM

Critique: 80 Preferred Response: A

Drowning is the second leading cause of unintentional injury-related death in children


between the ages of 1 and 14 years. In addition, for every drowning death, four children are
hospitalized and 14 are treated and released from emergency departments for near-drowning, a
condition defined as survival for 24 hours following a submersion event. Patients whose
experiences ultimately are characterized as near-drownings may present to medical attention
with no signs or symptoms, such as the girl described in the vignette, or be severely impaired.
Close observation for and early treatment of signs and symptoms as they develop are key to
favorable outcomes.
The pathophysiology of submersion injury is primarily related to the systemic effects of
hypoxemia that occur during the submersion event. Attempts to breathe may lead to aspiration
("wet drowning") or laryngospasm ("dry drowning"), either of which results in hypoxia and
impaired oxygen delivery to tissues. The tissue hypoxia can lead to anoxic injury in multiple
organ systems, including central nervous, pulmonary, kidney, muscle, and cardiovascular.
Pulmonary injury typically results from surfactant washout in the setting of aspiration,
neurogenic or postobstructive pulmonary edema, or airway obstruction due to inhaled debris.
The resultant atelectasis and ventilation-perfusion mismatch may lead to mild, self-limited
tachypnea and hypoxia or to profound respiratory distress and respiratory failure from acute
respiratory distress syndrome (ARDS). ARDS is a well-recognized consequence of submersion
injury and is characterized by worsening dyspnea, oxygen-resistant hypoxemia, and bilateral
pulmonary infiltrates caused by the host inflammatory response to an inciting stimulus. As in the
girl in the vignette, ARDS develops over several hours as the inflammatory cascade causes
cellular damage to the alveolar-capillary unit, surfactant inactivation, and microvascular
thrombosis. Because of the risk for delayed-onset severe pulmonary pathology and ARDS, any
patient who has respiratory signs and symptoms following a submersion event should be
observed in the hospital to allow early intervention if the respiratory status worsens. For
patients who are asymptomatic upon hospital arrival, an observation period of at least 6 hours is
recommended prior to discharge.
Despite the aspiration event that frequently accompanies near-drowning, aspiration
pneumonia is not common, and prophylactic antibiotics are not indicated. Chemical pneumonitis is
also not a common sequela of near-drowning. Bronchiolitis obliterans is a rare inflammatory
condition of the bronchioles and lung parenchyma seen in the setting of collagen-vascular
disease or after radiation therapy and bone marrow transplants. A pneumothorax may be seen
in patients who require significant ventilatory support for ARDS following near-drowning, but it is
not a typical isolated complication of near-drowning.

Suggested reading:

Chandy D, Weinhouse GL. Submersion injuries (near-drowning). UpToDate 17.3. 2008. Available
at:
http://www.utdol.com/online/content/topic.do?topicKey=ad_emerg/5474&selectedTitle=1~43&so
urce=search_result

Copyright 2010 by the American Academy of Pediatrics page 252


2011 PREP SA on CD-ROM

Huang LH. Acute respiratory distress syndrome. eMedicine Specialties, Pediatrics: Cardiac
Disease and Critical Care Medicine, Critical Care. 2009. Available at:
http://emedicine.medscape.com/article/906653-overview

Kallas HJ. Drowning and submersion injury. In: Kleigman RM, Behrman RE, Jenson HB, Stanton
BF, eds. Nelson Textbook of Pediatrics. 18th ed. Philadelphia, Pa: Saunders Elsevier; 2007:438-
449

Shepherd SM, Shoff WH. Drowning. eMedicine Specialties, Emergency Medicine,


Environmental. 2009. Available at: http://emedicine.medscape.com/article/772753-overview

Verive MJ. Near drowning. eMedicine Specialties, Emergency Medicine, Environmental. 2009.
Available at: http://emedicine.medscape.com/article/908677-overview

Copyright 2010 by the American Academy of Pediatrics page 253


2011 PREP SA on CD-ROM

Question: 81

A previously healthy 7-year-old boy presents to your office following 5 days of crampy
abdominal pain, tactile fevers, and watery diarrhea. For the past 2 days, the diarrhea has been
bloody. Symptoms began shortly after his family visited a local petting zoo. He recently completed
a 10-day course of amoxicillin for otitis media. Physical examination demonstrates a well-
developed, well-nourished child who is in moderate distress because of diffuse, moderate, direct
abdominal tenderness. Vital signs are normal, mucous membranes are dry, and capillary refill time
is 2 seconds. Initial laboratory studies show:

Hemoglobin, 10.5 g/dL (105 g/L)


Platelet count, 70x10 3/mcL (70x109/L)
Prothrombin time, 12 seconds (normal, up to 12 seconds)
Partial thromboplastin time, 23 seconds (normal, up to 26 seconds)
Urinalysis positive for blood, protein, and ketones

Of the following, the MOST likely cause of this childs illness is

A. Campylobacter jejuni

B. Clostridium difficile

C. Escherichia coli O157:H7

D. Salmonella type D

E. Shigella dysenteriae

Copyright 2010 by the American Academy of Pediatrics page 254


2011 PREP SA on CD-ROM

Critique: 81 Preferred Response: C

The clinical history and available laboratory data described for the boy in the vignette
indicate development of anemia, thrombocytopenia, and urinary abnormalities in association with
acute enteritis characterized by the development of hematochezia. These findings strongly
suggest a diagnosis of hemolytic-uremic syndrome (HUS) as a consequence of infection with
the Shiga toxin-producing bacterium Escherichia coli O157:H7 (STEC). Further studies likely will
demonstrate evidence of both microangiopathic hemolytic anemia and renal dysfunction that,
along with thrombocytopenia, define the HUS triad. In the United States, HUS occurs in
approximately 8% of children who have E coli O157:H7 diarrhea, an incidence that is
significantly greater than observed with other STEC serotypes.
Pathologic E coli species are transmitted most commonly via contaminated food products.
Particular reservoirs of STEC include raw or undercooked ground beef, unpasteurized milk, and
any food or beverage (eg, unpasteurized apple cider) that may be contaminated with bovine
feces. Because all STEC are excreted in the feces of cattle, sheep, deer, and other ruminants,
direct contact with these animals is another significant source of contamination. Several human
outbreaks of E coli O157:H7 enteritis have been linked to animals in petting zoos.
STEC is an important cause of acute renal injury and accounts for 70% to 90% of all HUS
cases in countries where these bacteria are endemic. HUS typically develops 5 to 9 days
(mean, 7 days) after the onset of gastrointestinal symptoms and 2 to 6 days (mean, 4 days)
after the appearance of bloody stools. Accordingly, when acute enteritis is associated with the
passage of blood, clinical assessment must include both a hematologic profile (including
examination of a blood smear for fragmented cells) and an evaluation of renal function.
The previous treatment with amoxicillin for otitis media reported for the boy in the vignette
raises the question of a possible role for antibiotics in the development of HUS. Although earlier
studies suggested that antibiotic usage in hemorrhagic colitis increased the risk of developing
HUS, a meta-analysis failed to confirm these findings. Nevertheless, available evidence clearly
indicates that antimicrobial agents do not alter the duration or outcome of illness, and the use of
antibiotics for E coli O157:H7 infection is not recommended. Furthermore, because most clinical
microbiology laboratories can culture and identify this organism, as well as the other common
bacterial pathogens that cause acute enterocolitis (Salmonella, Shigella dysenteriae,
Campylobacter jejuni, Clostridium difficile), decisions about specific therapy generally should
await bacteriologic identification.
Although Campylobacter, Salmonella type D, and C difficile may cause acute
gastroenteritis with hematochezia, they are not associated with HUS. Campylobacter are the
most common cause of acute bacterial gastroenteritis in the developed world, and nearly 50% of
all Salmonella infections in the United States are caused by Salmonella serotypes B, D, and
Newport. C difficile is a common cause of antibiotic-associated enterocolitis, and the organism
has been linked to exacerbations of inflammatory bowel disease. Although S dysenteriae (type
1) has been associated with HUS, the incidence of this disorder is far greater following infection
with either E coli O157:H7 or another STEC species.

Suggested reading:

Copyright 2010 by the American Academy of Pediatrics page 255


2011 PREP SA on CD-ROM

American Academy of Pediatrics. Escherichia coli diarrhea (including hemolytic-uremic


syndrome). In: Pickering LK, Baker CJ, Kimberlin DW, Long SS, eds. Red Book: 2009 Report of
the Committee on Infectious Diseases. 28th ed. Elk Grove Village, Ill: American Academy of
Pediatrics; 2009:294-298

Bitzan M. Treatment options for HUS secondary to Escherichia coli O157:H7. Kidney Int Suppl.
2009;75:S62S66. Abstract available at: http://www.ncbi.nlm.nih.gov/pubmed/19180140

Goode B, OReilly C, Dunn J, et al. Outbreak of Escherichia coli O157:H7 infections after petting
zoo visits, North Carolina State Fair, October-November 2004. Arch Pediatr Adolesc Med.
2009;163:42-48. Abstract available at: http://www.ncbi.nlm.nih.gov/pubmed/19124702

Holtz LR, Neill MA, Tarr PI. Acute bloody diarrhea: a medical emergency for patients of all ages.
Gastroenterology. 2009;136:1887-1898. DOI: 10.1053/j.gastro.2009.02.059. Abstract available
at: http://www.ncbi.nlm.nih.gov/pubmed/19457417

Shilkofski N, Cheng TL. In brief: Escherichia coli O157:H7. Pediatr Rev. 2004;25:75-76. DOI:
10.1542/10.1542/pir.25-2-75. Available at:
http://pedsinreview.aappublications.org/cgi/content/full/25/2/75

Copyright 2010 by the American Academy of Pediatrics page 256


2011 PREP SA on CD-ROM

Question: 82

A 17-year-old young woman in your practice recently learned she is pregnant. She was
diagnosed with systemic lupus erythematosus 3 years ago. Both she and her parents are very
worried about the health of the baby during the pregnancy.

Of the following, the MOST appropriate serial method to assess fetal growth in this pregnancy is

A. fundal height measurement

B. glucose tolerance testing

C. nonstress testing

D. stress testing

E. ultrasonography

Copyright 2010 by the American Academy of Pediatrics page 257


2011 PREP SA on CD-ROM

Critique: 82 Preferred Response: E

The most appropriate method for monitoring the growth of a fetus at risk for intrauterine
growth restriction (IUGR) due to maternal chronic systemic illness is serial ultrasonography.
IUGR describes a reduction in the expected fetal growth pattern, regardless of whether the
infant is small for gestational age (SGA) at the time of birth. The two events that have a
significant negative impact on fetal growth are uterine ischemia and hypoxia. Poorly controlled
chronic hypertension, which can be essential or associated with renal disease, affects fetal
growth. This effect is most severe in the fetuses of mothers who have end-organ disease,
such as diabetic retinopathy or lupus nephritis. Hypoxia related to maternal cyanotic cardiac
disease or severe sickle cell disease also is associated with growth restriction.
Although careful measurement of fundal height at each obstetric visit may allow
identification of fetuses whose growth is beginning to lag, serial ultrasonography is the primary
method for recognizing and following IUGR. Ultrasonographic biometry using measurements of
the biparietal diameter, head circumference, abdominal circumference, and femur length can be
correlated to established standards. Ultrasonographic evaluation also allows for potential
identification of associated anomalies and oligohydramnios, such as congenital cardiac
anomalies in infants of diabetic mothers. Glucose tolerance testing in pregnancy may identify
mothers at risk for diabetes, but it does not aid in monitoring serial growth of the fetus.
Fetuses affected by IUGR have a five- to sixfold risk of perinatal death and require careful
monitoring. Disagreement still exists in the obstetric literature about the optimal method for timing
delivery when IUGR due to uteroplacental insufficiency is suspected. Nonstress testing consists
of observing a fetus for 20 minutes to determine the presence of at least two fetal heart rate
accelerations, which is considered reactive. If the nonstress test is nonreactive, a contraction
stress test may be performed to assess the fetal response to the stress of a uterine
contraction. The test is considered negative if there are no late or significant variable
decelerations with at least three moderate uterine contractions and positive if there are late
decelerations with at least 50% of the contractions. The nonstress test and stress test may be
used to determine fetal well-being and timing of delivery, but they are not used to monitor fetal
growth directly.

Suggested reading:

American College of Obstetricians and Gynecologists. ACOG Practice Bulletin 101:


Ultrasonography in pregnancy. Obstet Gynecol. 2009;113:451-461. DOI:
10.1097/AOG.0b013e31819930b0. Extract available at:
http://journals.lww.com/greenjournal/Citation/2009/02000/ACOG_Practice_Bulletin_No__101__Ult
rasonography_in.33.aspx

Devoe LD. Antenatal fetal assessment: contraction stress test, nonstress test, vibroacoustic
stimulation, amniotic fluid volume, biophysical profile, and modified biophysical profile-an
overview. Semin Perinatol. 2008;32:247-252. DOI: 10.1053/j.semperi.2008.04.005. Abstract
available at: http://www.ncbi.nlm.nih.gov/pubmed/18652922

Copyright 2010 by the American Academy of Pediatrics page 258


2011 PREP SA on CD-ROM

Kliegman RM. Intrauterine growth restriction. In: Martin RJ, Fanaroff AA, Walsh MC, eds. Fanaroff
and Martins Neonatal-Perinatal Medicine: Diseases of the Fetus and Infant. 8th ed.
Philadelphia, Pa: Mosby Elsevier; 2006:271-306

Martinez A, Simmons R. Abnormalities of fetal growth. In: Taeusch HW, Ballard RA, Gleason CA,
eds. Averys Diseases of the Newborn. 8th ed. Philadelphia, Pa: Elsevier Saunders; 2005:32-45

Copyright 2010 by the American Academy of Pediatrics page 259


2011 PREP SA on CD-ROM

Question: 83

A 7-year-old girl presents in September with an intensely itchy rash of several weeks duration.
During the summer she had many mosquito bites and one area of ringworm, but otherwise she
has had no prior skin conditions. She has had no fever, joint pain, myalgias, fatigue, or change in
appetite or activity. Antihistamines helped when she had the insect bites, but now they have little
effect. No other family members have a rash. On physical examination, the rash is apparent on
exposed areas and consists of papules, vesicles, and wheals, some in a linear array or in
triangular clusters (Item Q83). There are also numerous hyperpigmented macules. The scalp is
involved, but the palms and soles are spared.

Of the following, the MOST likely cause of this rash is

A. atopic dermatitis

B. hypersensitivity reaction to insect bites

C. id reaction to a recent fungal infection

D. recurrent impetigo with postinflammatory hyperpigmentation

E. scabies

Copyright 2010 by the American Academy of Pediatrics page 260


2011 PREP SA on CD-ROM

Question: 83

(Reprinted with permission from Hernandez RG, Cohen BA. Pediatrics. 2006;118:e189-e196)
Papules, some in groups, as described for the girl in the vignette.

Copyright 2010 by the American Academy of Pediatrics page 261


2011 PREP SA on CD-ROM

Critique: 83 Preferred Response: B

The girl described in the vignette has papular urticaria, an annoying, pruritic eruption that is
the result of an insect bite-induced hypersensitivity reaction. It typically affects children between
2 and 10 years of age, who experience chronic or recurrent eruptions of papules, vesicles, and
wheals, often with excoriations because of their persistent itchiness. The age distribution is
presumed to be because children younger than 1 year of age are unable to mount the
hypersensitivity reaction, and most children have developed tolerance to insects by age 10
years. The lesions typically are symmetrically distributed and grouped in linear or triangular
clusters (Item C83A) that are concentrated on exposed surfaces, including the scalp, waist, and
sock lines. As described for the girl in the vignette, the palms and soles are spared. Often,
hyperpigmented or violaceous macules persist as the original eruption heals. Although most
cases are related to flea and mosquito bites, any biting insect may be involved, including bed
bugs and mites. The rash can be frustrating for families and clinicians alike because of its
chronicity, the limited treatment options (primarily symptomatic care), and the paradox that only
one family member may be affected, even though it is due to insect bites.
Papular urticaria differs from atopic dermitis by its lack of family history, distribution,
configuration, and absence of scale. Although id reactions are typically symmetric and itchy,
they often involve hands and feet and usually are preceded by a worsening of the underlying
condition, which is often a fungal infection such as tinea capitis (Item C83B). Recurrent impetigo
is characterized by crusting and responds well to antibiotic treatment. Like papular urticaria,
scabies is insect-related and involves an immune reaction. However, scabies produces smaller
papules or vesicles, frequently involves the hands and intertriginous areas, and is characterized
by burrows that are not present with papular urticaria. Also, scabies often affects multiple family
members.

Suggested reading:

Habif TP. Urticaria and angioedema. In: Clinical Dermatology: A Color Guide to Diagnosis and
Therapyy. 5th ed. Philadelphia, Pa: Mosby Elsevier; 2010:181-216

Hernandez RG, Cohen BA. Insect bite-induced hypersensitivity and the SCRATCH principles: a
new approach to papular urticaria. Pediatrics. 2006;118:e189-e196. doi:10.1542/peds.2005-
2550. Available at: http://pediatrics.aappublications.org/cgi/content/full/118/1/e189

Howard R, Frieden IJ. Papular urticaria in children. Pediatr Dermatol. 1996;13:246-249. Abstract
available at: http://www.ncbi.nlm.nih.gov/pubmed/8806128

Copyright 2010 by the American Academy of Pediatrics page 262


2011 PREP SA on CD-ROM

Critique: 83

(Reprinted with permission from Hernandez RG, Cohen BA. Insect bite-induced hypersensitivity
and the SCRATCH principles: a new approach to papular urticaria. Pediatrics. 2006;118:e189-
e196)
This 4-year-old has uniform 2- to 4-mm hyperpigmented macules and papules with
hypopigmented centers grouped in a linear pattern. Note the collarette of scale and central crust
on some lesions.

Copyright 2010 by the American Academy of Pediatrics page 263


2011 PREP SA on CD-ROM

Critique: 83

(Courtesy of D Krowchuk)
An id or autosensitization reaction produces an often generalized and symmetric eruption
composed of small pruritic papules. The hands frequently are involved.

Copyright 2010 by the American Academy of Pediatrics page 264


2011 PREP SA on CD-ROM

Question: 84

A frequent patient in the emergency department is an 18-month-old boy whose mother brings him
in with seemingly trivial complaints and almost always on weekends. Tonight, she expresses
concern that he should be admitted to the hospital. When you allay her fears that his common cold
symptoms do not require hospitalization, she says that she cannot go home at this hour because
the city buses are not running. She also discloses that she is pregnant and not feeling well. When
you begin to probe deeper about other sources of transportation home, she begins to cry and
relates her fear of going home because her husband becomes abusive to her following illicit drug
and alcohol use.

Of the following, the MOST appropriate next step in the management of this mothers concerns is
to

A. arrange transportation to a domestic violence shelter

B. give her the address of local domestic violence shelters

C. have the hospital social worker contact her at home in the morning

D. report the family to child protective services

E. report the family to law enforcement

Copyright 2010 by the American Academy of Pediatrics page 265


2011 PREP SA on CD-ROM

Critique: 84 Preferred Response: A

The astute clinician may recognize subtle signs of domestic violence during a pediatric visit,
which should initiate further questioning of a parent regarding possible domestic abuse. The
mother described in the vignette demonstrates many of these cues, including frequently seeking
medical attention, hesitation in leaving the office, repeated presentations to the emergency
department, and requests for support with transportation or other social concerns. Whenever
domestic violence is discovered, action should be taken to prevent further abuse. The
appropriate action needs to be individualized and will vary with the availability of services in the
physicians community. For the mother described in the vignette, arranging transportation to a
domestic violence shelter is likely to be the most effective approach to preventing further abuse.
However, emergency social work or child protective services intervention may be possible in
some locales, and brief hospital admission may sometimes be needed if there are concerns
regarding more serious medical problems in the child, postpartum depression, child abuse or
neglect, or exposure of the child to dangerous drugs.
Unfortunately, attempts by the mother to distance herself from the abuser, such as by
seeking a separation or divorce or moving to a shelter, is a known precipitant or provocation for
more domestic violence. Many women are caught in a cycle such that they become further
victimized by attempting to stop the violence by leaving for a shelter, reporting the violence, or
seeking help. A component of the emotional abuse in domestic violence is the belief by abused
women that they are responsible for the violence and, therefore, they feel guilty about leaving or
reporting it. The mothers emotional bond to her abusive partner and her childrens attachment to
the violent partner additionally make leaving the home difficult. In addition, the mothers financial
dependence on the perpetrator may make it difficult for her to leave. Thus, referring the mother
to a shelter is less likely to be effective than arranging transportation for her. The mother is most
likely to agree to a referral if it includes specific plans for financial support, medical and
educational follow-up plans, and treatment for comorbid conditions such as maternal depression
or substance abuse. In many, if not most communities, the mother must report domestic violence
herself unless the clinician can document suspicion of child abuse or imminent risk to the child.
The effects of growing up in a home where domestic violence is perpetrated against the
mother (or occasionally the father) and where child abuse occurs is harmful for the child. These
children are more likely to exhibit aggressive behaviors than other children.

Suggested reading:

Berkowitz CD. Domestic violence: a pediatric concern. Pediatr Rev. 2004;25:306-311. DOI:
10.1542/10.1542/pir.25-9-306. Available at:
http://pedsinreview.aappublications.org/cgi/content/full/25/9/306

Cahill L, Sherman P. Child abuse and domestic violence. Pediatr Rev. 2006;27:339-345. DOI:
10.1542/10.1542/pir.27-9-339. Available at:
http://pedsinreview.aappublications.org/cgi/content/full/27/9/339

Chung EK, Mathew L, Rothkopf AC, Elo IT, Coyne JC, Culhane JF. Parenting attitudes and infant

Copyright 2010 by the American Academy of Pediatrics page 266


2011 PREP SA on CD-ROM

spanking: the influence of childhood experiences. Pediatrics. 2009;124:e278-e286. DOI:


10.1542/peds.2008-3247. Available at:
http://pediatrics.aappublications.org/cgi/content/full/124/2/e278

Melchior M, Caspi A, Howard LM, et al. Mental health context of food insecurity: a representative
cohort of families with young children. Pediatrics. 2009;124:e564-e572. DOI: 10.1542/peds.2009-
0583. Available at: http://pediatrics.aappublications.org/cgi/content/full/124/4/e564

Copyright 2010 by the American Academy of Pediatrics page 267


2011 PREP SA on CD-ROM

Question: 85

At 60 minutes of age, a term 3.3-kg female infant appears cyanotic but is otherwise well. Her
oxygen saturation is 79%, she has widespread cyanosis, and you can hear a faint low-pitched
murmur diffusely across the chest. The remainder of findings on her physical examination are
within normal limits. After placing her on nasal cannula oxygen at 2 L/min, you note no change in
saturation.

Of the following, the MOST likely cause of this childs findings is

A. anemia

B. hypoplastic left heart syndrome

C. neonatal sepsis

D. retained fetal lung liquid syndrome

E. tracheoesophageal fistula

Copyright 2010 by the American Academy of Pediatrics page 268


2011 PREP SA on CD-ROM

Critique: 85 Preferred Response: B

Hypoplastic left heart syndrome (HLHS) occurs in approximately 2 of every 1,000 live births
and represents approximately 3% of congenital heart diseases. The spectrum of HLHS ranges
from hypoplasia of the mitral or aortic valves to valvar atresia. In addition, there is virtually
always coarctation of the aorta, and the left ventricle is markedly hypoplastic and dysfunctional.
Although HLHS often can be detected prenatally via fetal echocardiography, most cases are
unsuspected at birth. Affected infants may appear initially well prior to ductal closure, and the
only clinical finding may be cyanosis in the initial hours after birth. There may be a soft murmur,
as described for the infant in the vignette, or no murmur because the blood flow typically is not
turbulent. Spontaneous closing of the ductus arteriosus results in tachypnea, respiratory
distress, mottling, pallor, hypotension, weak pulses, cool extremities, and eventually acidosis and
shock due to systemic hypoperfusion. Management of shock due to cyanotic congenital heart
disease consists of the administration of prostaglandin E1 to maintain patency of the ductus
arteriosus.
Other causes of cyanotic congenital heart disease include complete transposition of the
great arteries, single ventricle lesions such as tricuspid atresia, pulmonary atresia, severe
tetralogy of Fallot, truncus arteriosus, interrupted aortic arch, and total anomalous pulmonary
venous return. All of these manifest only a minimal or no response to oxygen administration.
The "hyperoxia test" has been used for decades to help determine if a neonate who has
cyanosis is likely to have a cyanotic congenital cardiac defect. An Fio2 is delivered via head
hood, and a "preductal" right radial arterial blood gas is obtained. Pao2 values in excess of 150
mm Hg during a hyperoxia test are not consistent with the diagnosis of cyanotic congenital heart
disease. Pulmonary disease and primary pulmonary hypertension (if not sufficiently severe to be
unresponsive to oxygen) are more likely in this scenario. Pao2 values less than 150 mm Hg may
indicate the presence of cyanotic congenital heart disease. Patients who have complete
transposition of the great arteries have the lowest Pao2 findings, with typical values of 30 to 60
mm Hg. The patient who has HLHS usually has a Pao2 in the range of 50 to 120 mm Hg, with the
severity dependent upon the degree of obstruction across the mitral and aortic valves as well
as the presence or absence of obstruction to left-to-right shunting across the patent foramen
ovale.
The neonate who has severe anemia can manifest cyanosis, but the hyperoxia test should
elicit a dramatic improvement in Pao2 and oxygen saturation. The same holds true for the
neonate who has sepsis, retained fetal lung fluid syndrome, and tracheoesophageal fistula. All
of these conditions may have associated pulmonary parenchymal disease but should respond to
administered oxygen.

Suggested reading:

Dorfman AT, Marino BS, Wernovsky G, et al. Critical heart disease in the neonate: presentation
and outcome at a tertiary care center. Pediatr Crit Care Med. 2008;9:193-202. DOI:
10.1097/PCC.0b013e318166eda5. Abstract available at:
http://www.ncbi.nlm.nih.gov/pubmed/18477933

Copyright 2010 by the American Academy of Pediatrics page 269


2011 PREP SA on CD-ROM

Silberbach M, Hannon D. Presentation of congenital heart disease in the neonate and young
infant. Pediatr Rev. 2007;28:123-131. DOI: 10.1542/10.1542/pir.28-4-123. Available at:
http://pedsinreview.aappublications.org/cgi/content/full/28/4/123

Copyright 2010 by the American Academy of Pediatrics page 270


2011 PREP SA on CD-ROM

Question: 86

A 10-year-old previously healthy girl presents with subacute low back pain. In the past 24 hours,
she had "pins and needles" tingling in her left leg and subsequently her right leg. Six hours after
the onset of the tingling, she was unable to void urine. On physical examination, the girl is not
somnolent or confused, but she has flaccid weakness in the legs, absent reflexes in the legs, a
sensory level in the lower back at T9-10, diminished sensation in the feet to pinprick and vibration,
and reduced rectal tone.

Of the following, the MOST appropriate next diagnostic test is

A. brain magnetic resonance imaging with contrast

B. computed tomography scan of the spine

C. electroencephalography

D. lumbar puncture

E. spinal magnetic resonance imaging with contrast

Copyright 2010 by the American Academy of Pediatrics page 271


2011 PREP SA on CD-ROM

Critique: 86 Preferred Response: E

The symptom complex of escalating back pain, leg numbness, leg weakness, and urinary
retention described for the girl in the vignette mandates urgent evaluation. The loss of leg
strength and rectal tone as well as a sensory level documented on the physical examination are
pathognomonic for a spinal cord lesion. Emergency imaging of the spine with magnetic
resonance imaging (MRI) is warranted (Item C86).
Computed tomography scan of the spine is appropriate for detecting lesions in bone, but it is
less sensitive than MRI, particularly for intramedullary (intracord) processes. MRI is the preferred
imaging modality for inflammatory conditions such as infectious/postinfectious/inflammatory
myopathies; vascular conditions such as strokes, hematomas, vascular malformations;
neoplasms; and trauma. If an inflammatory lesion is seen, brain MRI with contrast should be
obtained to look for demyelination in brain, cerebellum, brainstem, and optic nerves.
A problem in the brain, specifically hydrocephalus, can cause leg weakness, but it would
not cause a sensory loss at and below a dermatomal level. Therefore, imaging of the head is not
needed initially and likely would be helpful only if a demyelinating spinal lesion is identified on MRI.
Lumbar puncture is not indicated initially and could cause problems for this child. Because the
problem clearly localizes to the spine, it is necessary to identify a structural problem in the spine
first. If there is a mass lesion in the spinal cord, shifting of the cord could cause more extensive
problems. Electroencephalography would not be helpful for this patient because the symptoms
and signs are not consistent with a seizure disorder.

Suggested reading:

Fenichel GM. Paraplegia and quadriplegia. Clinical Pediatric Neurology: A Signs and Symptoms
Approach. 6th ed. Philadelphia, Pa: Saunders Elsevier; 2009:267-284

Pidcock FS, Krishnan C, Crawford TO, Salorio CF, Trovato M, Kerr DA. Acute transverse myelitis
in childhood: center-based analysis of 47 cases. Neurology. 2007;68:1474-1480. Abstract
available at: http://www.ncbi.nlm.nih.gov/pubmed/17470749

Willoughby RE Jr. Cerebellar ataxia, transverse myelitis and myelopathy, GuillainBarr


syndrome, neuritis, and neuropathy. In: Long SS, Pickering LK, Prober CG, eds. Principles and
Practice of Pediatric Infectious Diseases. 3rd ed. Philadelphia, Pa: Churchill Livingstone;
2008:318-323

Copyright 2010 by the American Academy of Pediatrics page 272


2011 PREP SA on CD-ROM

Critique: 86

(Courtesy of D Gilbert)
T2 magnetic resonance imaging of the spine demonstrating swelling and signal change (yellow
arrows) in the distal thoracic and lumbar spinal cord consistent with transverse myelitis. The
affected areas of the cord (yellow arrows) are brighter than expected (compare with the normal
cervical cord, red arrow).

Copyright 2010 by the American Academy of Pediatrics page 273


2011 PREP SA on CD-ROM

Question: 87

A 4-year-old girl presents to your clinic for evaluation of developmental delays and dysmorphic
features. On physical examination, you document unusual facial features and multiple major
anomalies. Review of a recent developmental evaluation reveals significant global delays. Family
history is negative for any individuals who have birth defects or intellectual deficits. You consult a
medical geneticist, who does not recognize a pattern to this childs features and recommends
genetic testing. The high-resolution chromosome analysis yields normal results.

Of the following, the testing strategy that is MOST likely to be helpful in determining the cause of
this childs anomalies is

A. comparative genomic hybridization

B. DNA testing for fragile X syndrome

C. fluorescence in situ hybridization

D. urine organic acid analysis

E. X-inactivation studies

Copyright 2010 by the American Academy of Pediatrics page 274


2011 PREP SA on CD-ROM

Critique: 87 Preferred Response: A

Genetic testing for individuals who have intellectual disabilities, multiple anomalies, and
dysmorphic features is evolving rapidly. High-resolution chromosome analysis, which requires
dividing cells, has a resolution of 5 to 19 megabases (Mb), making it impossible to detect
deletions, duplications, or rearrangements that could contain hundreds of genes. Therefore,
additional testing is required for the girl described in the vignette.
Fluorescence in situ hybridization (FISH), which uses fluorescently labeled DNA probes that
hybridize to dividing or resting cells, has a higher resolution than chromosome analysis and is
excellent in detecting missing, extra, or relocated chromosomal regions. However, it does not
detect single-gene mutations, and it requires that the physician request testing for a specific
region, or regions, of interest. Without a recognized pattern to the features for the child in the
vignette, a specific request is not possible. FISH is useful for identifying contiguous gene
deletions, as can be seen in Prader-Willi, Angelman, Williams, Smith-Magenis, and Miller-Dieker
syndromes. The combined diagnostic yield of chromosome analysis and targeted FISH testing
approaches 10% in children who have intellectual disabilities and dysmorphisms.
Microarray-based comparative genomic hybridization (CGH) is a revolutionary testing
strategy for the detection of DNA copy number variations throughout the human genome that
may be associated with disease. Depending on the platforms used, which vary largely
according to the size of the probes used (the smaller the probe, the greater the resolution) and
the amount of the genome covered, microarray CGH can detect clinically significant abnormalities
in up to 20% of individuals who have unexplained intellectual disability. Therefore, it is the most
helpful choice for determining the cause of this childs anomalies. It is important to note,
however, that this modality also detects variations of unclear clinical significance as well as
variations of no clinical significance.
The array CGH procedure employs thousands to millions of probes (synthetic DNA
sequences that hybridize with genomic DNA) that are affixed to glass or silicon wafers (chips)
or beads. Added to this matrix are a sample of the patients DNA labeled with a green dye, for
example, and a sample of reference DNA labeled with a red dye, for example. Following
hybridization of the samples to the chip, sophisticated equipment is used to interpret the results.
Simply speaking, wherever there are equal amounts of patient DNA and reference DNA on the
chip, a yellow color is present. If there is excess patient DNA on a probe, the region appears
green, and if there is excess reference sample on a probe, the region appears red.
Microarray CGH does NOT replace high-resolution chromosome analysis for identifying all
genetic abnormalities of clinical significance. Abnormalities that can be detected on chromosome
analysis but not on microarray CGH at the time of this writing include balanced chromosomal
rearrangements (that theoretically can transect a gene and may have reproductive significance
for parents of the proband who are potential carriers of the rearrangement), free-standing
trisomy versus translocation-related trisomy, and low-level chromosomal mosaicism.
Fragile X syndrome typically is not associated with multiple major anomalies and is seen
most commonly in boys. Organic acid analysis is most helpful in the evaluation of children who
have metabolic derangements, which are not part of this childs presentation. Finally, X-
inactivation studies generally are used if a girl is believed to have a presentation typically only
seen in a boy who has an X-linked disorder, in which case she may have preferential

Copyright 2010 by the American Academy of Pediatrics page 275


2011 PREP SA on CD-ROM

inactivation of the X chromosomes containing the normal copy of the gene in question.

Suggested reading:

Bejjani BA, Shaffer LG. Application of array-based comparative genomic hybridization to clinical
diagnostics. J Mol Diagn. 2006;8:528-533. Available at:
http://jmd.amjpathol.org/cgi/content/full/8/5/528

Stankiewicz P, Beaudet AL. Use of array CGH in the evaluation of dysmorphology,


malformations, developmental delay, and idiopathic mental retardation. Curr Opin Genet Dev.
2007;17:182-192. DOI: 10.1016/j.gde.2007.04.009. Abstract available at:
http://www.ncbi.nlm.nih.gov/pubmed/17467974

Veltman JA. Genomic microarrays in clinical diagnosis. Curr Opin Pediatr. 2006;18:598-603. DOI:
10.1097/MOP.0b013e3280105417. Abstract available at:
http://www.ncbi.nlm.nih.gov/pubmed/17099357

Copyright 2010 by the American Academy of Pediatrics page 276


2011 PREP SA on CD-ROM

Question: 88

A 13-year-old girl who is late for her menstrual period has a positive result on a urine pregnancy
test in your office. She is a healthy girl who has a negative past medical history. She wishes to
continue with the pregnancy and feels she is ready to be a mother. Her 19-year-old sister has a 1-
year-old healthy infant and lives in the same household. Her mother asks you if the girl is at higher
risk for medical complications because of her age.

Of the following, the MOST appropriate response is that

A. good nutritional care will eliminate risk

B. good prenatal care will eliminate risk

C. she has the same risk as her older sister had

D. she is at increased risk for preterm delivery

E. there is no increased risk related to her young age

Copyright 2010 by the American Academy of Pediatrics page 277


2011 PREP SA on CD-ROM

Critique: 88 Preferred Response: D

The primary concern with outcomes of adolescent pregnancy is among females who are
either unaware that they are pregnant or in complete denial that they could be pregnant and,
therefore, do not access prenatal care. The most frequently cited medical complications of
adolescent pregnancy are similar to those experienced by adult women and include anemia,
pregnancy-induced hypertension, low birthweight, prematurity, intrauterine growth restriction,
and neonatal mortality. The risk for these outcomes in adolescent mothers is related
predominantly to the social, economic, and behavioral factors that predispose such young
women to pregnancy.
With good prenatal care, adolescents have the same outcomes as adult women, but among
those younger than age 15 years, some studies point to a modest increase in prematurity, low
birthweight, and neonatal death, despite good prenatal and nutritional care. Biologic factors are
considered to be the cause of the negative outcomes and include low prepregnancy weight and
height, poor pregnancy weight gain, and parity.

Suggested reading:

Berenson AB, Wiemann CM, McCombs SL. Adverse perinatal outcomes in young adolescents. J
Reprod Med. 1997;42:559-564. Abstract available at:
http://www.ncbi.nlm.nih.gov/pubmed/9336751

Cox JE. Teenage pregnancy. In: Neinstein LS, Gordon CM, Katzman D, Rosen DS, Woods ER,
eds. Adolescent Health Care: A Practical Guide. 5th ed. Philadelphia, Pa: Lippincott Williams &
Wilkins, a Wolters Kluwer business; 2008;41: 565-580

Fraser AM, Brockert JE, Ward RH. Association of young maternal age with adverse reproductive
outcomes. N Engl J Med. 1995;332:17:1113-1118. Available at:
http://content.nejm.org/cgi/content/full/332/17/1113

Klein JD; American Academy of Pediatrics Committee on Adolescence. Adolescent pregnancy:


current trends and issues. Pediatrics. 2005;116:281-286. DOI: 10.1542/peds.2005-0999.
Available at: http://pediatrics.aappublications.org/cgi/content/full/116/1/281

Menacker F, Martin JA, MacDorman MF, Ventura SJ. Births to 10-14 year-old mothers, 1990-
2002: trends and health outcomes. Natl Vital Stat Rep. 2004;53(7):1-18. Abstract available at:
http://www.ncbi.nlm.nih.gov/pubmed/15581020

Nicoletti AM. Teen pregnancy. In Emans SJH, Laufer MR, Goldstein DP, eds. Pediatric and
Adolescent Gynecology. 5th ed. Philadelphia, Pa: Lippincott Williams & Wilkins; 2005:844-878

Phipps MG, Sowers M. Defining early adolescent childbearing. Am J Public Health. 2002;92:125-
128. Available at:
http://ajph.aphapublications.org/cgi/content/full/92/1/125?view=long&pmid=11772775

Copyright 2010 by the American Academy of Pediatrics page 278


2011 PREP SA on CD-ROM

Question: 89

You are evaluating a 2-year-old boy in the emergency department who reportedly swallowed a
small amount of kerosene approximately 4 hours ago. On physical examination, the boy has a
temperature of 38.0C, heart rate of 130 beats/min, blood pressure of 85/50 mm Hg, and
respiratory rate of 36 breaths/min. He has mild nasal flaring, and his breath sounds are equal with
bilateral crackles. His oxygen saturation by pulse oximetry is 90% on room air.

Of the following, the next BEST step is administration of

A. antibiotics

B. corticosteroids

C. isoproterenol

D. oxygen

E. syrup of ipecac

Copyright 2010 by the American Academy of Pediatrics page 279


2011 PREP SA on CD-ROM

Critique: 89 Preferred Response: D

Hydrocarbons represent about 2% of all pediatric ingestions. Ingestion of agents such as


gasoline, kerosene, turpentine, and lighter fluid are generally accidental in young children and
occur in small amounts due to the foul taste. However, aspiration of hydrocarbons and
subsequent respiratory failure can be a significant medical challenge. Hydrocarbons can
produce both destruction of the respiratory components (airway epithelium, alveolar septae, and
pulmonary capillaries) and dissolution of the lipid surfactant layer. These processes result in a
chemical pneumonitis that can progress to subsequent necrosis in severe cases.
Symptoms from hydrocarbon ingestion can be limited to the respiratory system or systemic
(due to gastrointestinal and respiratory absorption). Respiratory symptoms include coughing,
choking, tachypnea, and cyanosis. Systemic symptoms and clinical findings include central
nervous system depression, cardiac dysrhythmias, fever, leukocytosis, hemolysis, and
hemoglobinuria.
Management is primarily supportive. Therefore, administration of oxygen is most appropriate
for the child in the vignette. Progression to respiratory failure should prompt intubation and
administration of positive-pressure ventilation. The pneumonitis associated with hydrocarbon
ingestion is chemical in nature; antibiotics are indicated only if there are signs of secondary
infection (fever or progression of infiltrates on chest radiography after 48 hours). There is no
evidence that corticosteroids reduce respiratory symptoms or damage. Patients may develop
bronchospasm and should be treated with beta2 selective agents. Isoproterenol and epinephrine
should be avoided because of myocardial sensitization and increased risk of fatal dysrhythmias.
Administration of syrup of ipecac should be avoided because emesis may increase the risk of
aspiration.

Suggested reading:

Colombo JL. Aspiration syndromes. In Kliegman RM, Behrman RE, Jenson HB, Stanton BF, eds.
Nelson Textbook of Pediatrics. 18th ed. Philadelphia, Pa: Saunders Elsevier; 2007:1789

Lewander WJ, Aleguas A. Hydrocarbon poisoning. UpToDate Online 17.3. 2009. Available at:
http://www.utdol.com/online/content/topic.do?topicKey=ped_tox/11453&selectedTitle=8%7E150
&source=search_result819

Copyright 2010 by the American Academy of Pediatrics page 280


2011 PREP SA on CD-ROM

Question: 90

You are caring for a 15-year-old girl who has been hospitalized for 3 months for treatment of
burns over 65% of her body suffered in a house fire. For several weeks, she has required
hyperalimentation because of problems in consuming food. Routine laboratory studies reveal a
serum calcium of 12.1 mg/dL (3.0 mmol/L) and phosphorus of 4.6 mg/dL (1.5 mmol/L). Serum
electrolyte values are normal.

Of the following, the MOST likely cause of this girls hypercalcemia is

A. hyperparathyroidism

B. hypervitaminosis A

C. hypervitaminosis D

D. immobilization

E. low-magnesium hyperalimentation formula

Copyright 2010 by the American Academy of Pediatrics page 281


2011 PREP SA on CD-ROM

Critique: 90 Preferred Response: D

Although hyperparathyroidism, hypervitaminosis A, and hypervitaminosis D can cause


hypercalcemia, the most likely cause of a high serum calcium value for the girl described in the
vignette, who has been unable to move and has been in a hospital bed, is immobilization.
Immobilization leads to osteoclast activation and relative bone resorption. High doses of vitamin A
act transcriptionally to stimulate osteoclast activity. Vitamin D encourages calcium uptake from
the gut and, in high doses, causes hypercalcemia. A low-magnesium hyperalimentation formula
could lead to hypocalcemia because magnesium is necessary for the release of preformed
parathyroid hormone from the parathyroid gland.

Suggested reading:

Agus ZS. Diagnosis and treatment of hypomagnesemia. UpToDate Online 17.3. 2008. Available
at:
http://www.uptodateonline.com/online/content/topic.do?topicKey=calcium/5547&selectedTitle=9
%7E150&source=search_result

Agus ZS. Etiology of hypercalcemia. UpToDate Online 17.3. 2008. Available at:
http://www.uptodateonline.com/online/content/topic.do?topicKey=calcium/6872&selectedTitle=16
%7E150&source=search_result

Claudius IA, Fattal O, Nakamoto J, Pitukcheewanont P. Hypercalcemia. eMedicine Specialties,


Pediatrics: General Medicine, Endocrinology. 2009. Available at:
http://emedicine.medscape.com/article/920955-overview

Goltzman D. Approach to hypercalcemia. Endotext.org. 2008. Available at:


http://www.endotext.com/parathyroid/parathyroid4/parathyroidframe4.htm

Stewart AF, Adler M, Byers CM, Segre GV, Broadus AE. Calcium homeostasis in immobilization:
an example of resorptive hypercalciuria. N Engl J Med. 1982;306:1136-1140. Abstract available
at: http://www.ncbi.nlm.nih.gov/pubmed/6280047

Copyright 2010 by the American Academy of Pediatrics page 282


2011 PREP SA on CD-ROM

Question: 91

During a health supervision visit, a mother reports that her child is only understood about 75% of
the time when she speaks to other adults.

Of the following, this finding is MOST expected for a typically developing child who is

A. 18 months old

B. 24 months old

C. 30 months old

D. 36 months old

E. 48 months old

Copyright 2010 by the American Academy of Pediatrics page 283


2011 PREP SA on CD-ROM

Critique: 91 Preferred Response: D

A helpful rule regarding the development of articulation is the Rule of Fourths. The average 2-
year-old child should be understood by strangers half the time (2/4), the average 3-year-old
child should be understood 3/4 of the time, and the average 4-year-old should be understood all
the time (4/4). The girl in the vignette is described as being understood by a stranger 75% of the
time, which is typical for a child of 3 years.
Children may have difficulty with the pronunciation of certain sounds until they are 7 years
of age, but 100% of speech should be understood by age 4 years. In normal speech
development, the first eight consonant sounds that develop are m, b, y, n, w, d, p, and h. The last
eight consonants are sh, th (as in thirty), s, z, th (as in the), zh sound typically spelled as s (as
in pleasure), l, and r, which develop by age 7.

Suggested reading:

Feldman HM. Evaluation and management of language and speech disorders in preschool
children. Pediatr Rev. 2005;26:131-142. DOI: 10.1542/10.1542/pir.26-4-131. Available at:
http://pedsinreview.aappublications.org/cgi/content/full/26/4/131

Kelly DP, Sally JI. Disorders of speech and language. In: Levine MD, Carey WB, Crocker AC, eds.
Developmental-Behavioral Pediatrics. 3rd ed. Philadelphia, Pa: WB Saunders Company; 199:621-
631

Copyright 2010 by the American Academy of Pediatrics page 284


2011 PREP SA on CD-ROM

Question: 92

A family in your practice has adopted an 11-month-old girl from China. They bring the childs
immunization record from the adoption agency to the first visit and ask if they can accept these
vaccinations as accurate. You explain that such records may not be reliable, but you can attempt
to determine whether at least partial immunization has been achieved.

Of the following, the MOST appropriate approach is to measure antibody concentrations to

A. Haemophilus influenzae type b

B. measles

C. pertussis

D. Streptococcus pneumoniae

E. tetanus

Copyright 2010 by the American Academy of Pediatrics page 285


2011 PREP SA on CD-ROM

Critique: 92 Preferred Response: E

The adequacy of vaccines administered to children adopted internationally has been


questioned, based on possible problems with potency and storage as well as inadequate
responses due to malnourishment. However, the Committee on Infectious Diseases of the
American Academy of Pediatrics has stated that written records of immunization may be
accepted if the number of doses, interval between doses, and age at immunization are
appropriate. Serologic testing can determine if protective antibody concentrations exist. Of the
vaccines available abroad and based on the age of the child described in the vignette,
measurement of tetanus antibodies may be of greatest assistance in determining whether age-
appropriate responses were achieved. Similarly, serologic responses to diphtheria, polio, and
hepatitis B can be measured.
Measles vaccine should not be administered routinely to children younger than 1 year of
age, and even if the vaccine is administered, provision of two doses after age 1 year is advised.
Pertussis titers do not reliably predict protection against this organism and generally are not
recommended. Testing for Haemophilus influenzae type b and Streptococcus pneumoniae
antibodies are not recommended because these vaccines are not available to children in
developing countries.

As a result of reviewing this information, do you intend to make a change in practice


to provide better patient care?
Yes No

Suggested reading:

American Academy of Pediatrics. Active immunization: immunizations received outside the United
States. In: Pickering LK, Baker CJ, Kimberlin DW, Long SS, eds. Red Book: 2009 Report of the
Committee on Infectious Diseases. 28th ed. Elk Grove Village, Ill: American Academy of
Pediatrics; 2009:36

American Academy of Pediatrics. Medical evaluation of internationally adopted children for


infectious diseases. In: Pickering LK, Baker CJ, Kimberlin DW, Long SS, eds. Red Book: 2009
Report of the Committee on Infectious Diseases. 28th ed. Elk Grove Village, Ill: American
Academy of Pediatrics; 2009:177-179

Lee PJ. Vaccines for travel and international adoption. Pediatr Infect Dis J. 2008;27:315-354.
DOI: 10.1097/INF.0b013e31816f1368. Abstract available at:
http://www.ncbi.nlm.nih.gov/pubmed/18379374

Copyright 2010 by the American Academy of Pediatrics page 286


2011 PREP SA on CD-ROM

Question: 93

A 10-year-old boy is brought to the emergency department in July complaining of fever,


headache, and confusion. He returned from a camping trip to Wisconsin yesterday. He spent
time hiking and swimming but had no injuries or animal exposures. Physical examination reveals a
febrile child who is confused and lethargic. Kernig and Brudzinski signs are present. He has
many mosquito bites on his trunk and extremities. Cerebrospinal fluid evaluation reveals 100 white
blood cells/mm3 with 80% lymphocytes, 10% polymorphonuclear leukocytes, and 10%
monocytes; protein of 70 mg/dL; and glucose of 60 mg/dL.

Of the following, you are MOST likely to advise the parents of this child that

A. antiviral therapy can hasten recovery from this infection

B. mortality caused by this infection is uncommon

C. myocarditis is a clinical feature of this infection

D. seizures and focal neurologic findings will occur in their child

E. the infection could have been prevented with appropriate vaccination

Copyright 2010 by the American Academy of Pediatrics page 287


2011 PREP SA on CD-ROM

Critique: 93 Preferred Response: B

The child described in the vignette has meningoencephalitis due to an arthropod-borne virus
transmitted through the bite of a mosquito. Because of his age and recent travel to Wisconsin,
the most likely cause of his illness is La Crosse (LAC) virus, a California serogroup virus. Cache
Valley and Jamestown Canyon viruses also are California serogroup viruses, but they rarely
cause encephalitis. Although much less common than illness caused by LAC in Wisconsin,
disease due to West Nile (greatest in Texas, California, Colorado), St. Louis encephalitis
(greatest in Illinois, Indiana, Ohio, Texas), Western equine (central and western states), rarely
Eastern equine encephalitis (eastern and Gulf states), and Powassan (tick vector, northeast and
north central states) viruses can occur.
Most arboviruses cause asymptomatic infection. Patients who have nonspecific febrile
illness caused by an arbovirus often exhibit myalgias, arthralgias, nausea, vomiting, headache,
fatigue, and sometimes rash. Neuroinvasive disease can include meningitis, encephalitis, acute
flaccid paralysis, focal neurologic changes, and seizures. LAC encephalitis is most common in
children younger than age 16 years and can result in a variety of neurologic sequelae. Mortality
caused by LAC occurs in fewer than 1% of those who have clinical disease. Therefore, the
parents of the child in the vignette can be told that mortality caused by this infection is
uncommon. The child may not develop seizures or focal neurologic findings. Myocarditis is not a
clinical feature of this infection.
Humans are accidental hosts of arboviruses. Most viruses cycle between birds or small
mammals and their arthropod vectors. With the exception of dengue, anthroponotic transmission
(person-vector-person) of arboviruses generally does not occur. However, blood transfusion
and organ transplantation can transmit the viruses. Infants can acquire infection in utero, but it is
uncertain if transmission through human milk occurs.
In the northern United States, arboviral disease occurs in the summer and fall, when
arthropod activity is highest. This is in contrast to infections in the southern United States, which
occur throughout the year. The number of cases reported annually in the United States varies
greatly, depends on the year and specific arbovirus, and likely is an underestimation of the
burden of disease because virus-specific antibody testing of blood or cerebrospinal fluid rarely
is performed. There is no vaccine against LAC virus, and no antiviral drug effective against LAC
virus exists at this time.

Suggested reading:

American Academy of Pediatrics. Arboviruses. In: Pickering LK, Baker CJ, Kimberlin DW, Long
SS, eds. Red Book: 2009 Report of the Committee on Infectious Diseases. 28th ed. Elk Grove
Village, Ill: American Academy of Pediatrics; 2009:214-220

Centers for Disease Control and Prevention. La Crosse Encephalitis. Atlanta, Ga: Centers for
Disease Control and Prevention; 2009. Available at: http://www.cdc.gov/LAC/index.html

Division of Vector-borne Infectious Diseases. Arboviral Encephalitides. Atlanta, Ga: Centers for
Disease Control and Prevention. 2009. Available at:

Copyright 2010 by the American Academy of Pediatrics page 288


2011 PREP SA on CD-ROM

http://www.cdc.gov/ncidod/dvbid/arbor/arbocase.htm

Copyright 2010 by the American Academy of Pediatrics page 289


2011 PREP SA on CD-ROM

Question: 94

A 14-year-old girl who is dependent on dialysis because of end-stage renal disease presents to
the emergency department with status epilepticus. She has gained 8 lb in 2 days. On physical
examination, her temperature is 37.3C, heart rate is 76 beats/min, respiratory rate is 16
breaths/min, and blood pressure is 200/120 mm Hg. She has mild facial swelling, pretibial edema,
and nonfocal findings on neurologic examination.

Of the following, the MOST likely cause for this patients seizures is

A. hypertension

B. hypocalcemia

C. hypoglycemia

D. hypomagnesemia

E. hyponatremia

Copyright 2010 by the American Academy of Pediatrics page 290


2011 PREP SA on CD-ROM

Critique: 94 Preferred Response: A

Seizures in the pediatric patient undergoing dialysis can have a number of causes, including
electrolyte abnormalities. Serum sodium abnormalities are uncommon in dialysis patients.
Hypermagnesemia is common due to diminished excretion and clearance, but this condition
typically is subclinical and seizures do not occur. Hypoglycemia can be associated with
seizures, but dialysis patients are no more prone to this abnormality than other children.
Hypocalcemia can occur in patients who have end-stage renal disease due to associated
hyperphosphatemia and reduced concentrations of 1,25 (OH)2 vitamin D and can be a plausible
explanation for seizures in a dialysis patient. However, this is less likely for the girl in the
vignette, who has severe hypertension and a history and physical findings consistent with
volume overload, a feature that is not unusual in dialysis patients, especially those who have
oliguria.
The girl in the vignette best fits the clinical picture of hypertensive encephalopathy (HE),
which is a result of severe hypertension with life-threatening organ damage. Under physiologic
circumstances, increased systemic blood pressure is associated with autoregulation of the
cerebral circulation, resulting in vasoconstriction. However, with severe hypertension, the
cerebral vasculature is unable to vasoconstrict sufficiently to prevent hyperperfusion and
edema of the brain. The most vulnerable region of the brain is the white matter of the parieto-
occipital areas, probably because of reduced sympathetic innervation to the posterior circulation
relative to the carotid circulation. Other terms now used interchangeably with HE are reversible
posterior leukoencephalopathy and posterior reversible encephalopathy syndrome (PRES). The
clinical manifestations of HE or PRES are related to brain swelling in those regions and include
vomiting, altered mental status, seizures, and visual changes, including blindness. HE or PRES is
believed to occur more in children who have acute increases in blood pressure, especially those
who do not have a longstanding history of hypertension, such as with acute glomerulonephritis.
Treatment of HE or PRES should focus on gradual reduction of blood pressure, aiming for
normalization over a few days. Those who do not have longstanding hypertension have a better
ability to autoregulate cerebral blood flow. For those who have chronic hypertension, ischemic
central nervous system insults such as cerebral infarction, transverse ischemic myelopathy, and
blindness can occur if the hypertension is corrected too quickly.

Suggested reading:

Adelman RD, Coppo R, Dillon MJ. The emergency management of severe hypertension. Pediatr
Nephrol. 2000;14:422-427. Abstract available at: http://www.ncbi.nlm.nih.gov/pubmed/10805473

Jones BV, Egelhoff JC, Patterson RJ. Hypertensive encephalopathy in children. AJNR Am J
Neuroradiol. 1997;18:101-106. Available at: http://www.ajnr.org/cgi/reprint/18/1/101

McNiece KL, Portman RJ. Hypertension: epidemiology and evaluation. In: Kher KK, Schnaper HW,
Makker SP, eds. Clinical Pediatric Nephrology. 2nd ed. London, United Kingdom: Informa
Healthcare; 2007:461-480

Copyright 2010 by the American Academy of Pediatrics page 291


2011 PREP SA on CD-ROM

Pavlakis SG, Frank Y, Chusid R. Hypertensive encephalopathy, reversible occipitoparietal


encephalopathy, or reversible posterior leukoencephalopathy: three names for an old syndrome.
J Child Neurol. 1999;14:277-281. Abstract available at:
http://www.ncbi.nlm.nih.gov/pubmed/10342593

Copyright 2010 by the American Academy of Pediatrics page 292


2011 PREP SA on CD-ROM

Question: 95

During clinical rounds, a resident describes a 2-month-old boy who was admitted for failure to
thrive and lymphopenia. A repeat complete blood count has been ordered, but a medical student
asks if this could be a case of severe combined immunodeficiency (SCID).

Of the following, additional historical information that would be MOST suggestive for SCID is

A. acute otitis media

B. chronic diarrhea

C. delayed umbilical cord separation

D. preterm delivery

E. urticaria

Copyright 2010 by the American Academy of Pediatrics page 293


2011 PREP SA on CD-ROM

Critique: 95 Preferred Response: B

Severe combined immunodeficiency (SCID) is a primary immunodeficiency resulting from one


of many described defects (Item C95). Typically presenting in the first 6 months after birth, the
most common clinical signs are chronic diarrhea, chronic respiratory viral infections, and chronic
dermatitis. Chronic diarrhea frequently is due to viral or bacterial gastrointestinal infection and
leads to failure to thrive.
Although urticaria is uncommon in infants who have SCID, recurrent Candida infection and a
persistent atopic dermatitis-like rash are common. The dermatitis is due to maternal T cells
entering the infants circulation and reacting against the infants tissues, resulting in a graft
versus host presentation. Otitis media can occur in SCID, but infants generally present with more
severe infections, such as pneumonia, sepsis, and meningitis. Delayed umbilical cord separation
is a recognized presentation for leukocyte adhesion defect type 1(LAD-1) and is due to impaired
neutrophil adhesion. Although delayed umbilical cord separation (ie, >30 days after birth) is more
likely a normal variant or due to poor umbilical care, a screening complete blood cell count or
flow cytometry for CD18 may be required to identify patients in whom LAD-1 is suspected.
Although infants who have underlying SCID are not more likely to be delivered preterm, certain
ethnic populations are at increased risk for specific SCID types (eg, defect in the Artemis protein
in Athabaskan Native Americans).
Current research has shown promise for SCID screening via the absence of T-cell receptor
excision circles (TRECs) on neonatal Guthrie cards.

Suggested reading:

Fleisher TA. Back to basics: primary immune deficiencies: windows into the immune system.
Pediatr Rev. 2006;27;363-372. DOI: 10.1542/10.1542/pir.27-10-363. Available at:
http://pedsinreview.aappublications.org/cgi/content/full/27/10/363

Morinishi Y, Imai K, Nakagawa N, et al. Identification of severe combined immunodeficiency by T-


cell receptor excision circles quantification using neonatal Guthrie cards. J Pediatr.
2009;155:829-833. DOI: 10.1016/j.jpeds.2009.05.026. Abstract available at:
http://www.ncbi.nlm.nih.gov/pubmed/19628217

Copyright 2010 by the American Academy of Pediatrics page 294


2011 PREP SA on CD-ROM

Critique: 95

Copyright 2010 by the American Academy of Pediatrics page 295


2011 PREP SA on CD-ROM

Question: 96

You are seeing a 15-year-old soccer player who complains of intermittent knee pain. He reports
frequent knee pain after practice and an episode of severe knee pain after a fall during a game 1
month ago. He recalls that his kneecap looked "out of place" at the time, but the deformity
resolved after he was helped off the field. On physical examination, his knee is not swollen, red,
or warm; he has no medial, lateral, or joint line tenderness; and there is no joint instability. He has
a positive patellar apprehension test (Item Q96).

Of the following, the MOST appropriate initial treatment recommendation for this patient is

A. corticosteroid injection

B. knee immobilization for 6 weeks

C. quadriceps strengthening exercises

D. referral to an orthopedist for surgery

E. use of a knee brace during practice

Copyright 2010 by the American Academy of Pediatrics page 296


2011 PREP SA on CD-ROM

Question: 96

(Courtesy of D Krowchuk)
Patellar apprehension test: Application of medial-to-lateral pressure to the patella causes the
patient to be apprehensive, feeling that the patella will dislocate.

Copyright 2010 by the American Academy of Pediatrics page 297


2011 PREP SA on CD-ROM

Critique: 96 Preferred Response: C

Patellar dislocation is the clinical condition in which the patella has moved from its central
position in the trochlear femoral groove between the distal femur and proximal tibia. The patella
most often is dislocated laterally as the result of a twisting or pivoting movement of the knee in
gymnastics, basketball, while batting, or while playing soccer. It also may occur from a direct
blow to the knee. Patellar dislocation occurs more commonly in adolescent girls and in patients
who have patellar hypermobility, generalized joint hyperlaxity, history of patellar subluxation, or
patella alta (high-riding patella). Patellar subluxation is the clinical situation in which the history is
consistent with patellar dislocation ("my kneecap looked out of place"), but the deformity
resolves spontaneously, as described for the boy in the vignette.
In either subluxation or dislocation, the patient reports acute, severe knee pain, often in
association with an audible pop or tearing sound. In the case of dislocation, the patient presents
to medical attention with the knee in 20 to 30 degrees of flexion and an obvious knee deformity,
with the patella lateral to the distal femur. The patella should be reduced expeditiously by
applying lateral to medial pressure to the patella while extending the knee. Reduction results in
immediate pain relief. Following reduction, or in a patient whose history is consistent with
patellar subluxation, knee radiographs, including anteroposterior, lateral, and Merchant (patellar)
views, should be obtained to evaluate the knee joint for fracture. The patient should be placed in
a knee immobilizer and on crutch walking. Pain and swelling should be managed with
nonsteroidal anti-inflammatory medications and ice.
As many as 70% of patients younger than 15 years have recurrent dislocations.
Postdislocation rehabilitation, specifically aimed at quadriceps strengthening and improving range
of motion, appears equivalent to surgery in preventing recurrence. Surgery now is reserved for
those patients who have recurrent dislocations that are refractory to quadriceps strengthening.
Use of corticosteroid injections and prolonged immobilization are not indicated. Use of a brace
while playing sports is recommended when the athlete returns to play following rehabilitation at
least 4 to 6 weeks after the injury.

Suggested reading:

Andrews S, Davies M, Donatelli R, Whiteside JA. Patellofemoral instability and malalignment. In:
Griffin LY, ed. Essentials of Musculoskeletal Care. 3rd ed. Rosemont, Ill: American Academy of
Orthopaedic Surgeons; 2005:541-545

Hosalkar HS, Wells L. The knee. In: Kleigman RM, Behrman RE, Jenson HB, Stanton BF, eds.
Nelson Textbook of Pediatrics. 18th ed. Philadelphia, Pa: Saunders Elsevier; 2007:2796-2799

Moore BR, Bothner J. Patellar dislocation in children and adolescents. UpToDate Online 17.3.
2008. Available at:
http://www.utdol.com/online/content/topic.do?topicKey=ped_proc/10100&selectedTitle=1~15&so
urce=search_result

Palmu S, Kallio PE, Donell ST, Helenius I, Nietosvaara Y. Acute patellar dislocation in children and

Copyright 2010 by the American Academy of Pediatrics page 298


2011 PREP SA on CD-ROM

adolescents: a randomized clinical trial. J Bone Joint Surg Am. 2008;90:463-470. DOI:
10.2106/JBJS.G.00072. Abstract available at: http://www.ncbi.nlm.nih.gov/pubmed/18310694

Wheeless CR III. Subluxation/dislocation of the patella. In: Wheeless Textbook of Orthopaedics.


2008. Available at:
http://www.wheelessonline.com/ortho/subluxation_dislocation_of_the_patella

Copyright 2010 by the American Academy of Pediatrics page 299


2011 PREP SA on CD-ROM

Question: 97

You are asked to see a 14 month-old boy who has rectal bleeding. The child was seen at another
practice 6 months ago because of a history of intermittent constipation, and a barium enema
examination was performed (Item Q97). For the past week, he has experienced several loose-to-
watery and occasionally explosive bowel movements per day, which now contain streaks of
blood. Over the past 24 hours, he has developed a low-grade fever. Physical examination
demonstrates a well-developed, thin child whose height is 77 cm, weight is 9.2 kg, temperature is
38.1C, heart rate is 110 beats/min, and blood pressure is 90/60 mm Hg. His abdomen is
moderately distended and diffusely tender, with bowel sounds present. Initial laboratory data
demonstrate:

Hemoglobin, 10.4 g/dL (104 g/L)


White blood cell count, 16.0x10 3/mcL (16.0x109/L)
Sodium, 136 mEq/L (136 mmol/L)
Potassium, 4.8 mEq/L (4.8 mmol/L)
Bicarbonate, 16 mEq/L (16 mmol/L)
Erythrocyte sedimentation rate, 32 mm/hr

Of the following, the childs symptoms are MOST likely caused by

A. Clostridium difficile infection

B. cow milk protein allergy

C. Escherichia coli O157:H7 infection

D. Hirschsprung disease

E. inflammatory bowel disease

Copyright 2010 by the American Academy of Pediatrics page 300


2011 PREP SA on CD-ROM

Question: 97

(Courtesy of S Schwarz)
Barium enema demonstrating a narrower caliber of the rectosigmoid (small arrow) than the
proximal colon (large arrow).

Copyright 2010 by the American Academy of Pediatrics page 301


2011 PREP SA on CD-ROM

Critique: 97 Preferred Response: D

Rectal bleeding in infants and young children can be alarming for both parents and
physicians. The symptoms described for the 14-month-old child in the vignette appear to be
consistent with an acute bacterial colitis, and the childs clinical history indicates a problem with
constipation. The lateral projection of the previously performed barium enema demonstrates a
relatively narrow diameter rectosigmoid colonic segment compared with the more proximal colon
(Item C97A), a common radiographic finding in Hirschsprung disease (HD), also known as
colonic aganglionosis. The present illness is consistent with Hirschsprung enterocolitis, a
serious complication that, in this case, likely developed because of the delay in diagnosing HD.
Although laboratory evaluation for potential infectious causes of enterocolitis, including
Clostridium difficile and Escherichia coli O157:H7, must be performed, this childs clinical
history and presentation make a primary infectious process unlikely. The enterocolitis of cow milk
protein allergy rarely, if ever, presents after the first few postnatal months. Inflammatory bowel
disease, particularly ulcerative colitis, has been described in infants and toddlers, but the history
and radiographic findings for this boy clearly suggest an alternate diagnosis.
HD is the most common cause of lower intestinal obstruction in infants, with an incidence of
approximately 1 per 5,000 live births. Trisomy 21 is the most frequently associated anomaly. HD
also may present as intractable constipation in toddlers and school-age children. The disorder is
characterized by the absence of ganglion cells in the myenteric and submucous colonic
plexuses. The abnormality in colonic innervation results in sustained contraction of the
aganglionic segment.
More than 90% of healthy newborns and fewer than 10% of infants who have HD pass
meconium during the first 24 hours after birth. One exception to this characteristic clinical history
is the child who has short-segment HD, in which only the most distal rectal segment is
aganglionic. Short-segment HD may present early in the newborn period with signs of low
intestinal obstruction or the condition may not be diagnosed until childhood, when children are
evaluated because of intractable constipation that often is marked by the passage of ribbonlike
stools, abdominal distension, and failure to thrive. Overall, 8% to 20% of patients who have HD
are not diagnosed until they are 3 years of age or older.
A digital rectal examination in an infant or child who has a history of constipation typically
demonstrates increased anal sphincter tone and an empty rectal vault for the patient who has
HD. When HD is suspected, and before diagnostic studies are undertaken, patients should be
referred to a center where further evaluation may be conducted by a pediatric
gastroenterologist and pediatric surgeon. Any delay in diagnosis increases the risk of developing
enterocolitis.
A barium enema (BE) frequently is the first diagnostic test obtained. In infants who have HD,
the typical BE finding is a relatively narrow-diameter rectosigmoid colonic segment. A clear
transition zone, demarcating the junction between a contracted rectosigmoid and a dilated
proximal colon, is more commonly found in toddlers and older children who have HD (Item C97B).
A BE should not be performed following the recent use of enemas because rectal manipulation
may dilate the distal colonic segment mechanically and obscure the findings consistent with HD.
Although a BE usually is not required to diagnose HD beyond infancy, it may be useful in
determining the length of the aganglionic colonic region prior to surgery.

Copyright 2010 by the American Academy of Pediatrics page 302


2011 PREP SA on CD-ROM

Rectal biopsy and anorectal manometry are the only studies that may establish or rule out
HD reliably. Histopathologic examination of suction rectal biopsy specimens, usually obtained
approximately 2 and 5 cm from the anal verge, demonstrates absence of ganglion cells in the
submucosal plexus. This finding is diagnostic, and the presence of ganglion cells rules out HD. In
some cases of inconclusive suction biopsy results, a surgically obtained full-thickness biopsy
may be required. In the cooperative child and in equivocal situations, particularly when short-
segment HD is suspected, anorectal manometry may be recommended. This test examines the
response to balloon inflation in the region of the internal and external anal sphincters. In HD,
absence of normal internal sphincter relaxation following balloon inflation is noted. Properly
conducted, anorectal manometry is both sensitive and specific for HD.
Enterocolitis is the most serious complication related to HD. It usually presents with watery,
explosive stools, with or without gross bleeding, and may be the initial presentation of HD.
Hirschsprung enterocolitis most commonly occurs at 2 to 3 months of age in previously
undiagnosed patients and is associated with a 20% mortality rate. However, it may occur at any
age prior to a diagnosis of HD. Enterocolitis also has been reported to occur up to 2 years
following an endorectal pull-through surgical correction.

Suggested reading:

Constipation Guideline Committee of the North American Society for Pediatric Gastroenterology,
Hepatology and Nutrition. Evaluation and treatment of constipation in infants and children:
recommendations of the North American Society for Pediatric Gastroenterology, Hepatology and
Nutrition. J Pediatr Gastroenterol Nutr. 2006;43:e1-e13. DOI:
10.1097/01.mpg.0000233159.97667.c3. Available at:
http://journals.lww.com/jpgn/Fulltext/2006/09000/Evaluation_and_Treatment_of_Constipation_in.2
8.aspx

Croffie JM, Davis MM, Faught PR, et al. At what age is a suction rectal biopsy less likely to
provide adequate tissue for identification of ganglion cells? J Pediatr Gastroenterol Nutr.
2007;44:198-202. DOI: 10.1097/01.mpg.0000252188.12793.ee. Available at:
http://journals.lww.com/jpgn/Fulltext/2007/02000/At_What_Age_Is_a_Suction_Rectal_Biopsy_Le
ss_Likely.8.aspx

Dasgupta R, Langer JC. Evaluation and management of persistent problems after surgery for
Hirschsprung disease in a child. J Pediatr Gastroenterol Nutr. 2008;46:13-19. DOI:
10.1097/01.mpg.0000304448.69305.28. Available at:
http://journals.lww.com/jpgn/Fulltext/2008/01000/Evaluation_and_Management_of_Persistent_Pr
oblems.5.aspx

de Lorijn F, Kremer LC, Reitsma JB, Benninga MA. Diagnostic tests in Hirschsprung disease: a
systematic review. J Pediatr Gastroenterol Nutr. 2006;42:496-505. DOI:
10.1097/01.mpg.0000214164.90939.92. Available at:
http://journals.lww.com/jpgn/Fulltext/2006/05000/Diagnostic_Tests_in_Hirschsprung_Disease__

Copyright 2010 by the American Academy of Pediatrics page 303


2011 PREP SA on CD-ROM

A.6.aspx

Engum SA, Grosfeld JL. Long-term results of treatment of Hirschsprung's disease. Semin
Pediatr Surg. 2004;13:273-285. doi:10.1053/j.sempedsurg.2004.10.015. Abstract available at:
http://www.ncbi.nlm.nih.gov/pubmed/15660321

Thapar N. New frontiers in the treatment of Hirschsprung disease. J Pediatr Gastroenterol Nutr.
2009;48:S92-S94. DOI: 10.1097/MPG.0b013e3181a15d62. Available at:
http://journals.lww.com/jpgn/Fulltext/2009/04002/New_Frontiers_in_the_Treatment_of_Hirschspr
ung.17.aspx

Copyright 2010 by the American Academy of Pediatrics page 304


2011 PREP SA on CD-ROM

Critique: 97

(Courtesy of S Schwarz)
Barium enema demonstrating a narrower caliber of the rectosigmoid (small arrow) than the
proximal colon (large arrow). This is a common radiographic finding in Hirschsprung disease.

Copyright 2010 by the American Academy of Pediatrics page 305


2011 PREP SA on CD-ROM

Critique: 97

(Courtesy of J Amodio)
Long segment Hirschsprung disease: There is a somewhat longer segment of contracted
rectosigmoid colon, transition zone (line), and dilated proximal colon.

Copyright 2010 by the American Academy of Pediatrics page 306


2011 PREP SA on CD-ROM

Question: 98

You are evaluating an infant in the nursery who was born vaginally to an 18-year-old girl who
had no prenatal care. Ballard examination reveals the infant to be 36 weeks gestation, with a
weight of 2,000 g (5th percentile), head circumference of 29 cm (5th percentile), and length of 43
cm (5th percentile). Physical examination reveals diffuse petechiae on the back,
hepatosplenomegaly, and mild jaundice. Upon visiting the mother to update her on her infants
status, you note that she has a generalized macular eruption that involves her palms (Item Q98).

Of the following, the MOST appropriate therapy for the infants underlying condition is

A. acyclovir

B. ganciclovir

C. isoniazid

D. penicillin G

E. sulfadiazine

Copyright 2010 by the American Academy of Pediatrics page 307


2011 PREP SA on CD-ROM

Question: 98

(Courtesy of the Red Book Online)


Rash, as described for the girl in the vignette.

Copyright 2010 by the American Academy of Pediatrics page 308


2011 PREP SA on CD-ROM

Critique: 98 Preferred Response: D

The finding of a maternal palmar rash combined with a small for gestational age infant who
has hepatosplenomegaly, petechiae, and jaundice described in the vignette suggests the
diagnosis of congenital syphilis. The preferred therapy for syphilis is penicillin G. Congenital
syphilis may be due to transplacental transmission of Treponema pallidum at any time during
pregnancy. Prenatal screening for syphilis using nontreponemal testing (rapid plasma reagin or
Venereal Disease Research Laboratory [VDRL]) should be undertaken early in pregnancy and
preferably again near the time of delivery. No infant should be discharged from the hospital after
birth without documentation of maternal serologic status for syphilis at least once in pregnancy.
If the mother is treated for syphilis, the titers must be monitored to determine treatment efficacy
and the need to treat the newborn.
Only one third of infants who have congenital syphilis are symptomatic at birth, with most
identified by maternal screening studies during pregnancy or at the time of delivery. Clinical
findings of congenital syphilis include jaundice, petechiae, hepatosplenomegaly, desquamation of
the hands and feet (Item C98A), mucous patches, rhinitis, and osteochondritis. Radiologic
evaluation may reveal periostitis. Affected infants also may present with severe luetic
pneumonia. If not treated, the symptoms develop further and evolve. Late findings of congenital
syphilis include bony changes (Saber shin, saddle nose), dental anomalies (Hutchinson teeth)
(Item C98B), and neurologic abnormalities (juvenile paresis, tabes dorsalis).
Acquired syphilis almost always is contracted by direct exposure to ulcerative lesions of
the skin or mucous membranes during sexual activity. Primary acquired syphilis presents as a
chancre (a painless ulcer) located on the genitalia, mouth, breast, or rectum, with localized
lymphadenopathy. If untreated, secondary syphilis develops 2 to 10 weeks after the chancre
heals. A nonpruritic, maculopapular rash may erupt that involves the entire body, including the
palms and soles (Item C98C). One to two months after the onset of the rash, the infection
becomes latent, with intermittent relapses and some affected individuals developing tertiary
syphilis. Sexual abuse must be considered when acquired syphilis is diagnosed in children.
A detailed algorithm for evaluating and treating infants whose mothers have had positive
results on serologic screening for syphilis is available (Item C98D). Evaluation of the
cerebrospinal fluid (CSF) is recommended, although interpretation of the results is difficult.
Neurosyphilis may be present in an infant who has a normal cell count and negative CSF VDRL
result. A 10-day course of parenteral penicillin G continues to be the preferred treatment for
congenital syphilis and covers potential central nervous system involvement. Alternative
treatment strategies, including penicillin G benzathine, may be used in select situations (Item
C98E).
Agents used for other congenital infections include acyclovir for congenital herpes,
ganciclovir for congenital cytomegalovirus, isoniazid for congenital tuberculosis, and
sulfadiazine for congenital toxoplasmosis.

Suggested reading:

American Academy of Pediatric. Syphilis. In: Pickering LK, Baker CJ, Kimberlin DW, Long SS, eds.
Red Book: 2009 Report of the Committee of Infectious Diseases. 28th ed. Elk Grove Village, Ill:

Copyright 2010 by the American Academy of Pediatrics page 309


2011 PREP SA on CD-ROM

American Academy of Pediatrics; 2009:638-651

Hyman EL, Adam HM. In brief: syphilis. Pediatr Rev. 2006;27:37-39. DOI: 10.1542/10.1542/pir.27-
1-37. Available at: http://pedsinreview.aappublications.org/cgi/content/full/27/1/37

Copyright 2010 by the American Academy of Pediatrics page 310


2011 PREP SA on CD-ROM

Critique: 98

(Courtesy of P Treadwell)
Desquamation of the hands and feet may occur in congenital syphilis.

Copyright 2010 by the American Academy of Pediatrics page 311


2011 PREP SA on CD-ROM

Critique: 98

(Courtesy of the Centers for Disease Control Public Health Image Library)
Hutchinson teeth are notched permanent central incisors.

Copyright 2010 by the American Academy of Pediatrics page 312


2011 PREP SA on CD-ROM

Critique: 98

(Courtesy of T Badgett)
The eruption of secondary syphilis is composed of ham-colored macules that affect the palms
and soles.

Copyright 2010 by the American Academy of Pediatrics page 313


2011 PREP SA on CD-ROM

Critique: 98

(Courtesy of the Red Book Online)


Algorithm for evaluation and treatment of infants born to mothers with reactive serologic tests for
syphilis.

Copyright 2010 by the American Academy of Pediatrics page 314


2011 PREP SA on CD-ROM

Critique: 98

Courtesy of Redbook Online, Elk Grove Village, IL: American Academy of Pediatrics; 2009

Copyright 2010 by the American Academy of Pediatrics page 315


2011 PREP SA on CD-ROM

Question: 99

A 10-month-old infant has developed a diffuse, pruritic rash over the past 2 weeks. She is
otherwise well and is receiving no medications. She lives with her mother and three older
siblings, one of whom also has itchy hands and arms. The infants rash involves the trunk (Item
Q99A), arms, palms, and soles (Item Q99B) and is composed of papules and vesiculopustules.

Of the following, the MOST appropriate management is topical

A. diphenhydramine for the affected children

B. hydrocortisone for the affected children

C. mupirocin for the infant

D. permethrin for the entire family

E. permethrin for the infant

Copyright 2010 by the American Academy of Pediatrics page 316


2011 PREP SA on CD-ROM

Question: 99

(Courtesy of D Krowchuk)

Copyright 2010 by the American Academy of Pediatrics page 317


2011 PREP SA on CD-ROM

Question: 99

(Courtesy of D Krowchuk)

Copyright 2010 by the American Academy of Pediatrics page 318


2011 PREP SA on CD-ROM

Critique: 99 Preferred Response: D

The infant described in the vignette demonstrates a typical presentation of scabies for her
age group. Scabies is a parasitic infection of the skin caused by Sarcoptes scabiei var hominis.
It usually is transmitted from person to person by direct skin contact, including by sexual contact.
The mite can survive on room surfaces for 24 to 36 hours at typical room temperatures and
humidity, permitting the rare possibility of disease transmission by fomites. Because symptoms
are due to an immunologic response to the infestation, they usually do not occur until 3 to 6
weeks after initial infestation, but they can develop within days of a reinfestation.
The most prominent symptom of scabies is intense pruritus, especially at night. Adults and
older children frequently have inflammatory papules affecting the finger webs, wrists, elbows,
axillae, buttocks, female breasts, and genitalia. Burrows, characterized as grey, serpiginous
ridges on the skin surface of hands, feet, wrists, and digital web spaces (Item C99A), as well
as nodules are specific but not universal findings. In infants, the characteristic rash is
papulovesicular and involves the face, scalp, postauricular area, palms, and soles (Item C99B).
Secondary eczematization and impetigo can occur. Often, multiple family members are affected.
The diagnosis usually is clinical, but it can be confirmed by microscopic examination of scrapings
under mineral oil or saline. Potassium hydroxide dissolves fecal material and decreases the
chance of confirming the diagnosis. The presence of the female mite, egg (Item C99C), or fecal
material (Item C99D) is diagnostic. However, the technique can be difficult for practitioners who
do not perform it often. Recently, the use of an epiluminescent dermatoscope has been shown
to be a sensitive alternative to microscopy for diagnosis by dermatologists.
Atypical presentations occur in specific clinical settings. Norwegian or crusted scabies
affects immunocompromised hosts, particularly those who have human immunodeficiency
virus/acquired immune deficiency syndrome. It is an often diffuse outbreak, appears crusted,
and frequently resembles psoriasis. Because the number of mites involved in this type of
infestation is very large, this is a particularly contagious form.
In the elderly, pruritus is common, but the rash is less inflamed; in bedridden patients, the
rash may be located primarily on the back. Bullous scabies affects adults older than 65 years
and appears similar to bullous pemphigoid. Nodular scabies presents with a few violaceous,
firm, pruritic nodules on covered body parts and is believed to be due to an exaggerated
hypersensitivity reaction to the mite.
Treatment of scabies is recommended for the infested patient and close contacts, especially
household members, even if they are asymptomatic. In the United States, the current treatment
of choice is topical 5% permethrin applied as a single 10-hour application. A second treatment
about 1 week later generally is warranted, particularly for crusted scabies. Adverse reactions
are uncommon but include burning or worsening pruritus. Resistance to this medication does not
appear to be common but may be increasing. Topical lindane 1% lotion has been relegated to
second-line status because of the potential for central nervous system effects with overuse,
misuse, or accidental oral exposure. It should not be used in infants, pregnant or breastfeeding
women, those who have seizure disorders, the elderly, or people weighing less than 110
pounds. Additional topical treatments that have been used include: benzyl benzoate, allethrin
(neither available in the United States), crotamiton, and precipitated sulfur.
Ivermectin is an oral antiparasitic medication administered as two doses 2 weeks apart that

Copyright 2010 by the American Academy of Pediatrics page 319


2011 PREP SA on CD-ROM

is not approved for scabies treatment in the United States but has been used in Europe and
Mexico. Its primary advantage is its ease of oral administration, and it may be particularly useful
for patients who have Norwegian (crusted) scabies. Topical corticosteroids or diphenhydramine
can be used adjunctively for pruritus resulting from scabies but are not effective for the mite
infestation. Similarly, topical mupirocin might be used for secondary impetigo but not primarily for
scabies.

Suggested reading:

Centers for Disease Control and Prevention. Scabies: Medications. 2008. Available at:
http://www.cdc.gov/scabies/hcp/meds.html

Chosidow O. Scabies. N Engl J Med. 2006;354:1718-1727. Available at:


http://content.nejm.org/cgi/content/full/354/16/1718

Curry BJ, McCarthy JS. Permethrin and ivermectin for scabies. N Engl J Med. 2010;362:717-715.
Available at: http://www.nejm.org/doi/full/10.1056/NEJMct0910329

Neynaber S, Wolff H. Diagnosis of scabies with dermoscopy. CMAJ. 2008;178:1540-1541. DOI:


10.1503/cmaj.061753. Available at: http://www.cmaj.ca/cgi/content/full/178/12/1540

Orion E, Marcos B, Davidovici B, Wolf R. Itch and scratch: scabies and pediculosis. Clin
Dermatol. 2006;24:168-175. DOI: 10.1016/j.clindermatol.2005.11.001. Abstract available at:
http://www.ncbi.nlm.nih.gov/pubmed/16714197

Strong M, Johnstone P. Interventions for treating scabies. Cochrane Database Syst Rev.
2009;3:CD00320. DOI: 10.1002/14651858.CD000320.pub2. Available at:
http://www.mrw.interscience.wiley.com/cochrane/clsysrev/articles/CD000320/frame.html

Copyright 2010 by the American Academy of Pediatrics page 320


2011 PREP SA on CD-ROM

Critique: 99

(Courtesy of D Krowchuk)
Burrow (arrow) and papules on the heel of an infant who has scabies.

Copyright 2010 by the American Academy of Pediatrics page 321


2011 PREP SA on CD-ROM

Critique: 99

(Courtesy of D Krowchuk)
In infants, scabies produces a generalized eruption composed of papules, nodules, or vesicles.

Copyright 2010 by the American Academy of Pediatrics page 322


2011 PREP SA on CD-ROM

Critique: 99

(Courtesy of D Krowchuk)
Mineral oil preparation in scabies: Note the mite and eggs (arrow).

Copyright 2010 by the American Academy of Pediatrics page 323


2011 PREP SA on CD-ROM

Critique: 99

(Courtesy of D Krowchuk)
Mineral oil preparation in scabies demonstrating mite fecal material (scybala).

Copyright 2010 by the American Academy of Pediatrics page 324


2011 PREP SA on CD-ROM

Question: 100

You are evaluating an 18-month-old child who has an obvious limp in a clinic for adoptees from
foreign countries. Perinatal and other medical history are not available. The childs gait is
characterized by a "drop" in stance when walking on one side, with the affected femur
appearing shorter than the other. Physical examination reveals no other abnormalities. A
radiograph of the hips is obtained (Item Q100).

Of the following, the MOST likely cause of this childs findings is

A. congenital hip dysplasia

B. coxa vara deformity

C. femoral anteversion

D. healed fracture

E. Legg-Calv-Perthes disease

Copyright 2010 by the American Academy of Pediatrics page 325


2011 PREP SA on CD-ROM

Question: 100

(Courtesy of B Wood)
Radiograph, as described for the child in the vignette.

Copyright 2010 by the American Academy of Pediatrics page 326


2011 PREP SA on CD-ROM

Critique: 100 Preferred Response: B

The radiograph of the child in the vignette demonstrates coxa vara deformity (Item C100A),
a decrease in the normal angle of 120 to 135 degrees between the femoral head and shaft (Item
C100B). This deformity may be congenital or acquired. Congenital coxa vara deformity results
from abnormal ossification of the femoral neck, and acquired coxa vara deformity is theorized to
result from ischemia of the femoral neck following trauma or infection or due to conditions such
as Legg-Calv-Perthes or sickle cell disease. Affected children often present with a waddling
gait and may have leg length discrepancy. On physical examination there may be limited internal
rotation and abduction of the affected hip.
Although a healed fracture or osteomyelitis could have damaged the femoral shaft, neither
condition is apparent on radiography. Congenital hip dysplasia is associated with coxa valga
deformity (increase in the angle between the femoral head and shaft of greater than 135
degrees). It can best be distinguished from coxa vara deformity by magnetic resonance imaging
of the hip. Legg-Calv-Perthes disease is an avascular necrosis of the femoral head. Femoral
anteversion is a rotational deformity that generally presents as intoeing and not with a limp.
Coxa valga deformity also is associated with disorders that cause spasticity of the hip
adductors such as cerebral palsy or meningomyelocele.
Both coxa vara and coxa valga can result in osteoarthritis of the hip, if undetected.
Treatment is aimed at any underlying disease and may include surgery.

Suggested reading:

Letts RM, Kontio KK. Congenital coxa vara. eMedicine Specialties, Orthopedic Surgery,
Pediatrics. 2009. Available at: http://emedicine.medscape.com/article/1259556-overview

Scherl SA. Common lower extremity problems in children. Pediatr Rev. 2004;25:52-62. DOI:
10.1542/10.1542/pir.25-2-52. Available at:
http://pedsinreview.aappublications.org/cgi/content/full/25/2/52

Copyright 2010 by the American Academy of Pediatrics page 327


2011 PREP SA on CD-ROM

Critique: 100

(Courtesy of B Wood)
Bilateral congenital coxa vara: The angle between the femoral neck and femoral shaft (which
should measure between 120 degrees and 135 degrees) is reduced.

Copyright 2010 by the American Academy of Pediatrics page 328


2011 PREP SA on CD-ROM

Critique: 100

(Courtesy of A Johnson)
Femoral abnormalities: Normally, the angle between the femoral head and shaft is 120 degrees to
135 degrees (center). In coxa vara, this angle is reduced (left); in coxa valga, the angle is
increased (right).

Copyright 2010 by the American Academy of Pediatrics page 329


2011 PREP SA on CD-ROM

Question: 101

You discover a heart murmur in a term 3.1-kg male infant immediately after birth. The infant
demonstrates cyanosis, and his oxygen saturation is 65%. His breathing is unlabored, there is no
hepatomegaly or splenomegaly, and the distal pulses are excellent. A chest radiograph
demonstrates clear lung fields and a normal cardiac silhouette but a narrowed mediastinal
shadow (Item Q101). You order echocardiography.

Of the following, the MOST likely diagnosis is

A. coarctation of the aorta

B. complete atrioventricular canal defect

C. hypertrophic cardiomyopathy

D. large ventricular septal defect

E. transposition of the great arteries

Copyright 2010 by the American Academy of Pediatrics page 330


2011 PREP SA on CD-ROM

Question: 101

(Courtesy of M Lewin)
Chest radiograph, as described for the infant in the vignette.

Copyright 2010 by the American Academy of Pediatrics page 331


2011 PREP SA on CD-ROM

Critique: 101 Preferred Response: E

The diagnosis of transposition of the great arteries (TGA) should be suspected in the infant
exhibiting profound cyanosis without respiratory distress. As for the infant described in the
vignette, chest radiography typically demonstrates a narrow mediastinal shadow resulting from
the anteroposterior alignment of the aorta and pulmonary artery rather than the normal side-by-
side configuration of the vessels. Echocardiography is considered the gold standard for
diagnosis. Most patients undergo an arterial switch operation based on echocardiographic
diagnosis alone without the need for cardiac catheterization or alternative imaging modalities.
Currently, many neonates undergo balloon atrial septostomy at the bedside under
echocardiographic procedural guidance.
Complete TGA represents approximately 5% of all cases of congenital heart disease. The
hallmark of this malformation is the origin of the pulmonary artery from the left ventricle and the
aorta from the right ventricle (Item C101). With this parallel circulation, desaturated blood travels
to the body and saturated blood travels to the lungs, resulting in marked systemic circulatory
cyanosis. Affected neonates have oxygen saturations that average 50% to 70%. Prostaglandin
is administered to maintain ductal patency and allow for adequate mixing of better oxygenated
blood with poorly saturated blood. Most children who have TGA require a balloon atrial
septostomy, which allows free egress of highly saturated blood returning to the left atrium from
the pulmonary veins to shunt left-to-right across the patent foramen ovale to the right atrium and
subsequently to the aorta via the right ventricle. Once adequate atrial-level mixing is achieved,
the prostaglandin administration can be discontinued. Most children who have TGA undergo an
arterial switch procedure during the first 7 to 10 days after birth. The operation entails
transection of the aorta and pulmonary artery above their respective valves and transferring to
their appropriate positions over the left ventricle and right ventricle, respectively. The coronary
arteries also are transferred so that they arise off of the "neo-aorta." After successful repair, a
normal circulatory pattern is re-established. Children rarely develop complications associated
with kinking of the coronary arteries, ventricular dysfunction, and outflow tract obstruction at
the anastomotic sites. However, most patients have excellent short- and long-term outcomes.
Prior to acceptance of the arterial switch procedure, patients underwent the atrial switch
procedure (Senning or Mustard) in which the pulmonary and systemic venous returns were
baffled to their appropriate atria. Although highly successful, patients who had this procedure
(many of whom are now in mid-adulthood) suffer from atrial arrhythmias and systemic (right)
ventricular dysfunction.
The findings in this case are not consistent with a complete atrioventricular canal defect or
a ventricular septal defect, both of which represent acyanotic congenital heart disease.
Although patients who have shunting lesions can exhibit mild cyanosis in the early neonatal
period prior to the natural decrease in pulmonary vascular resistance, they would not be
expected to manifest oxygen saturations as low as 65%. Coarctation of the aorta may be
associated with mild lower extremity desaturation due to right-to-left shunting via the ductus
arteriosus to maintain adequate systemic arterial circulation. However, lower extremity pulses
would be diminished. Hypertrophic cardiomyopathy is not associated with cyanosis. All of these
lesions would be excluded by the physical examination, with confirmation obtained by
echocardiography.

Copyright 2010 by the American Academy of Pediatrics page 332


2011 PREP SA on CD-ROM

Suggested reading:

de Koning WB, van Osch-Gevers M, Ten Harkel AD, et al. Follow-up outcomes 10 years after
arterial switch operation for transposition of the great arteries: comparison of cardiological
health status and health-related quality of life to those of a normal reference population. Eur J
Pediatr. 2008;167:995-1004. DOI: 10.1007/s00431-007-0626-5. Abstract available at:
http://www.ncbi.nlm.nih.gov/pubmed/17987315

Skinner J, Hornung T, Rumball E. Transposition of the great arteries: from fetus to adult. Heart.
2008;94:1227-1235. DOI: 10.1136/hrt.2006.104737. Extract available at:
http://heart.bmj.com/content/94/9/1227.extract

Warnes CA. Transposition of the great arteries. Circulation. 2006;114:2699-2709. DOI:


10.1161/CIRCULATIONAHA.105.592352. Available at:
http://circ.ahajournals.org/cgi/content/full/114/24/2699

Copyright 2010 by the American Academy of Pediatrics page 333


2011 PREP SA on CD-ROM

Critique: 101

(Courtesy of A Johnson)
In transposition of the great arteries, the pulmonary artery originates from the left ventricle and
the aorta from the right ventricle.

Copyright 2010 by the American Academy of Pediatrics page 334


2011 PREP SA on CD-ROM

Question: 102

An 8-year-old girl who has intellectual disability and dysmorphic facial features presents to the
emergency department with reduced alertness of 24 hours duration. For the past hour, she has
been crying inconsolably and holding her head. At triage she makes poor eye contact and is
banging her head. Physical examination of the irritable child reveals dysmorphic features and
abnormal head shape. On eye examination, her pupils are equally reactive and appear normally
aligned. It is unclear whether her neck is supple, due to a lack of cooperation, but she has a
subcutaneous tube connected to a shunt bulb over her right parietal skull.

Of the following, the MOST appropriate next step is to

A. insert a needle into the shunt bulb

B. order brain magnetic resonance imaging with contrast

C. order head computed tomography scan without contrast

D. order stat electroencephalography

E. perform a lumbar puncture

Copyright 2010 by the American Academy of Pediatrics page 335


2011 PREP SA on CD-ROM

Critique: 102 Preferred Response: C

The girl described in the vignette is manifesting symptoms of elevated intracranial pressure,
including head pain and, ominously, mental status changes. The most likely cause for her
problems is a malfunction of her ventriculoperitoneal shunt, resulting in acute hydrocephalus.
The diagnostic modality of choice is head computed tomography (CT) scan, which should be
obtained emergently, along with a routine radiographic shunt series to assess for
disconnections in tubing.
Brain magnetic resonance imaging is less available than CT scan, requires more time for
study completion, and involves a greater need for sedation in an agitated patient. Moreover, in
this clinical setting, if there is hydrocephalus, it should be readily apparent on CT scan.
Electroencephalography is not necessary emergently. Although nonconvulsive status
epilepticus could explain mental status changes, this condition typically would not produce the
pain syndrome this child is experiencing.
In the setting of elevated intracranial pressure, it is important to determine initially if an
intracranial mass lesion is present. Such a lesion may be a contraindication to the use of lumbar
puncture. Withdrawing fluid through the shunt generally is not performed as an initial intervention
before obtaining radiologic confirmation of the shunt malfunction.

Suggested reading:

Dias MS, Skaggs DL. Neurosurgical management of myelomeningocele (spina bifida). Pediatr
Rev. 2005;26:5060. DOI: 10.1542/10.1542/pir.26-2-50. Available at:
http://pedsinreview.aappublications.org/cgi/content/full/26/2/50

Tabassi K, Silvestri CC. Management of increased intracranial pressure and intracranial shunts.
In: Roberts JR, Hedges JR, eds. Clinical Procedures in Emergency Medicine. 5th ed.
Philadelphia, Pa: Saunders Elsevier; 2009:1185-1196

Copyright 2010 by the American Academy of Pediatrics page 336


2011 PREP SA on CD-ROM

Question: 103

A couple comes to you with questions following the diagnosis of trisomy 21 in their now 18-week
fetus. They ask what medical problems may be anticipated for a child who has Down syndrome.

Of the following, the medical problem that is encountered MOST frequently in infants who have
trisomy 21 is

A. a cardiac defect

B. duodenal atresia

C. hypotonia

D. polycythemia

E. strabismus

Copyright 2010 by the American Academy of Pediatrics page 337


2011 PREP SA on CD-ROM

Critique: 103 Preferred Response: C

A syndrome is a recognizable pattern of features owing to a specific cause. Down


syndrome (DS) is caused by trisomy for all, or a critical portion of, chromosome 21. Almost 99%
of affected individuals have extra chromosome 21 material in every cell in their bodies;
approximately 1% have a mosaic chromosome complement (46,XX/47,XX+21 or
46,XY/47,XY+21). Chromosome 21 is composed of about 400 genes. An extra copy of some or
all of these genes causes a variety of potential medical problems, which should be discussed
with the parents who are expecting an affected baby. It is important to note that each child who
has DS is unique, and there is a spectrum of potential outcomes. Of note, individuals who have
mosaic DS are anticipated to have similar features/cognitive ability to those who have full trisomy
21 because it is not possible to know which tissues contain the abnormal chromosome
complement. If these individuals are affected more mildly than the typical person who has DS,
this fact becomes evident over time.
Hypotonia is one of the most common abnormalities in babies who have DS, occurring in up
to 90%. The degree of hypotonia varies from child to child, but usually it is not so severe as to
interfere with feeding in the newborn period, and most babies who have DS are able to
breastfeed effectively (assuming there is not a cardiac abnormality that precludes this). Early
intervention programs should address hypotonia by offering appropriate therapies. The natural
history of low muscle tone in DS is that it improves over time.
Approximately 12% of children who have DS are born with anomalies of the gastrointestinal
tract, including Hirschsprung disease, esophageal atresia, tracheoesophageal fistula, and
pyloric stenosis. Duodenal atresia occurs in approximately 4%.
Cardiac abnormalities are present in approximately 45% of newborns who have DS. Most
are atrioventricular canal defects (almost 50%), followed by ventricular septal defects (~33%),
but a variety of anomalies can occur. All newborns who have DS should undergo
echocardiography, regardless of their physical examination findings.
Polycythemia is a complication during the newborn period in approximately 18% of
individuals who have DS. Polycythemia can interfere with oxygenation, and sometimes partial
exchange transfusion is recommended for treatment. Otherwise, this problem typically is self-
limiting.
Problems affecting the eye occur in approximately 60% of children who have DS and
include cataracts, lacrimal duct stenosis, nystagmus, and refractive errors. Strabismus occurs
in up to 50%. Direct ophthalmoscopic examination should be performed regularly during pediatric
visits, with referral to a pediatric ophthalmologist for baseline evaluation between 6 and 12
months of age. Ophthalmology follow-up is tailored to the childs needs but should take place at
least every 2 years.

Suggested reading:

Cohen WI. Down syndrome preventive medical check list. Down Syndrome Quarterly. 1999;4:(3).
Available at: http://www.ndsccenter.org/resources/healthcare.pdf

Committee on Genetics. American Academy of Pediatrics: Health supervision for children with

Copyright 2010 by the American Academy of Pediatrics page 338


2011 PREP SA on CD-ROM

Down syndrome. Pediatrics. 2001;107:442-449. Available at:


http://pediatrics.aappublications.org/cgi/content/full/107/2/442

Freeman SB, Torfs CP, Romitti PA, et al. Congenital gastrointestinal defects in Down syndrome: a
report from Atlanta and the National Down Syndrome Projects. Clin Genet. 2009;75:180-184.
DOI: 10.1111/j.1399-0004.2008.01110.x. Abstract available at:
http://www.ncbi.nlm.nih.gov/pubmed/19021635

Vis JC, Duffels MGJ, Winter MM, et al. Down syndrome: a cardiovascular perspective. J Intellect
Disabil Res. 2009;53;5:419-425. DOI: 10.1111/j.1365-2788.2009.01158.x. Abstract available at:
http://www.ncbi.nlm.nih.gov/pubmed/19228275

Copyright 2010 by the American Academy of Pediatrics page 339


2011 PREP SA on CD-ROM

Question: 104

As an adjunct to abstinence education, you are asked about the value of starting a "virginity
pledge" program in your neighborhood school. You meet with the school staff to educate them on
the pros and cons.

Of the following, evidence suggests that the MOST likely outcome of such programs is that

A. formal pledges are more effective than informal pledges

B. most pledgers abstain from oral sex

C. pledgers and nonpledgers have similar sexually transmitted infection rates if sexually active

D. pledgers are more likely than nonpledgers to abstain from vaginal intercourse

E. pledgers are more likely than nonpledgers to use condoms when they become sexually active

Copyright 2010 by the American Academy of Pediatrics page 340


2011 PREP SA on CD-ROM

Critique: 104 Preferred Response: C

The virginity pledge movement began in 1993, and initial reports indicated that adolescents
who took such pledges were more likely to delay initiation of sexual intercourse than those who
did not. Later studies suggested that the outlook was not as positive, noting that 61% of young
adults who took pledges reported breaking their vows. Also, the effect of virginity pledges did
not extend to other sexual behaviors. Pledging adolescents were equally likely to engage in oral
sex as those who did not make pledges. Longitudinal studies examining sexually transmitted
infections among young adults indicated that rates of such infections among pledgers did not
differ from nonpledgers. Pledging adolescents were less likely to use condoms at first
intercourse and less likely to worry about and get tested for sexually transmitted infections.
Studies examining the association between formal and nonformal virginity pledges and the
initiation of genital play, oral sex, and vaginal intercourse found that adolescents who made
private pledges or promises to themselves (nonformal pledges) to wait to have sexual
intercourse until older had reduced likelihoods of engaging in sexual intercourse and oral sex.
These findings suggest that a more effective approach may be to encourage young people to
make personal commitments to delay the onset of sex and raise their awareness of how early
sexual initiation is associated with risks that may threaten future plans.

Suggested reading:

Bersamin MM, Walker S, Waiters ED, Fisher DA, Grube JW. Promising to wait: virginity pledges
and adolescent sexual behavior. J Adolesc Health. 2005;36:428-436. DOI:
10.1016/j.jadohealth.2004.09.016. Abstract available at:
http://www.ncbi.nlm.nih.gov/pubmed/15837347

Brckner H, Bearman P. After the promise: the STD consequences of adolescent virginity
pledges. J Adolesc Health. 2005;36:271-278. DOI: 10.1016/j.jadohealth.2005.01.005. Available
at: http://www.jahonline.org/article/PIIS1054139X05000558/fulltext

Martino SC, Elliott MN, Collins RL, Kanouse DE, Berry SH. Virginity pledges among the willing:
delays in first intercourse and consistency of condom use. J Adolesc Health. 2008;43:341-348.
DOI: 10.1016/j.jadohealth.2008.02.018. Abstract available at:
http://www.ncbi.nlm.nih.gov/pubmed/18809131

Rosenbaum JE. Patient teenagers? A comparison of the sexual behavior of virginity pledgers and
matched nonpledgers. Pediatrics. 2009;123:e110-e120. DOI: 10.1542/peds.2008-0407. Available
at: http://pediatrics.aappublications.org/cgi/content/full/123/1/e110

Copyright 2010 by the American Academy of Pediatrics page 341


2011 PREP SA on CD-ROM

Question: 105

You are evaluating an 18-month-old boy who is admitted to the pediatric intensive care unit (PICU)
following bronchoscopy. He was diagnosed with croup 4 days ago and treated as an outpatient
with dexamethasone. His mother reports that he did well, with near resolution of his fever and
cough until last night, when he developed a temperature to 40.0C and the onset of increased
difficulty breathing. She brought him to the emergency department, and physicians there referred
him to otolaryngology, where he underwent emergent bronchoscopy and was given a dose of
intravenous ceftriaxone. The bronchoscopy revealed purulent secretions (Item Q105). On
physical examination in the PICU, the child is lying supine, does not appear to be drooling, and has
audible stridor only when crying. His heart rate is 140 beats/min, respiratory rate is 40
breaths/min, blood pressure is 90/60 mm Hg, and oxygen saturation on room air is 94% by pulse
oximetry.

Of the following, the MOST appropriate next step in the treatment of this patient is administration
of

A. cool mist

B. fluconazole

C. nafcillin

D. penicillin

E. racemic epinephrine

Copyright 2010 by the American Academy of Pediatrics page 342


2011 PREP SA on CD-ROM

Critique: 105 Preferred Response: C

Bacterial tracheitis has emerged as a more common life-threatening airway infection than
epiglottitis over the past 10 to 15 years. Patients who have bacterial tracheitis usually present
with a history of a recent upper respiratory tract infection, stridor, cough, fever, and failure to
respond to nebulized racemic epinephrine or corticosteroids. Bacterial tracheitis is confirmed
with bronchoscopic or laryngoscopic viewing of laryngotracheal erythema, edema, and thick
purulent secretions, as described for the boy in the vignette.
Staphylococcus aureus is responsible for more than 50% of infections, and initial antibiotic
therapy should be directed against this and other common organisms, including Haemophilus
influenzae, Moraxella catarrhalis, Streptococcus pneumoniae, and beta-hemolytic
Streptococcus. A third-generation cephalosporin agent combined with a penicillinase-resistant
penicillin such as nafcillin is appropriate for first-line therapy. Vancomycin should be substituted
if methicillin-resistant S aureus is locally prevalent. Clindamycin maybe be substituted for
vancomycin if local methicillin-resistant S aureus resistance to clindamycin is low. Although
yeast and fungal infections have been reported, these are less common and should be
considered only if the patient fails to respond to initial therapy or culture results reveal fungi or
yeast. Therefore, fluconazole is not indicated initially. Cool mist and racemic epinephrine would
be of minimal benefit in this patient.

Suggested reading:

Hopkins A, Lahiri T, Salerno R, Heath B. Changing epidemiology of life-threatening upper airway


infections: the reemergence of bacterial tracheitis. Pediatrics. 2006;118;1418-1421. DOI:
10.1542/peds.2006-0692. Available at: http://www.pediatrics.org/cgi/doi/10.1542/peds.2006-
0692

Woods CR. Clinical features, evaluation, and diagnosis of croup. UpToDate Online 17.3. 2009.
Available at:
http://www.utdol.com/online/content/topic.do?topicKey=pedi_id/22768&selectedTitle=1~13&sour
ce=search_result

Copyright 2010 by the American Academy of Pediatrics page 343


2011 PREP SA on CD-ROM

Question: 106

You diagnose familial hypophosphatemic rickets in a boy who presents with rickets and whose
mother had rickets as a child and required osteotomies as an adult. You explain to the parents
that treatment can help the boy reach normal height and lessen his rachitic bone changes.

Of the following, the MOST appropriate treatment is

A. daily injections of human growth hormone and oral calcium twice daily

B. oral calcitriol once daily

C. oral calcium twice daily with oral cholecalciferol once daily

D. oral neutral phosphate salts every 6 hours with calcitriol once or twice daily

E. oral neutral phosphate salts once daily

Copyright 2010 by the American Academy of Pediatrics page 344


2011 PREP SA on CD-ROM

Critique: 106 Preferred Response: D

The rickets seen in children who have hypophosphatemic rickets is due to phosphate loss,
but when phosphate is replaced in the form of oral neutral phosphate salts, hypocalcemia can
develop because the phosphate takes calcium with it into bone. Therefore, patients such as the
boy in the vignette need additional rapid-acting vitamin D (1,25 hydroxyvitamin D or calcitriol) in
the regimen to increase calcium availability. With this combination, they tend to grow fairly well,
although maintaining normal phosphate concentrations between meals and during the night can
be difficult, and large amounts of oral phosphate may cause gastric distress. Many children,
therefore, eventually require leg osteotomies once growth is complete because of long bone
bowing.
Any preparation of vitamin D alone is not sufficient to treat these children, who have a
primary disorder of phosphate loss. Treatment with human growth hormone has been shown to
increase serum phosphate concentrations slightly and may increase growth rate. However,
human growth hormone is not approved by the United States Food and Drug Administration for
this purpose. Recently, studies have suggested that some of the newer calcimimetic drugs may
play a role in future treatment.

Suggested reading:

Makitie O, Toiviainen-Salo S, Marttinen E, Kaitila I, Sochett E, Sipila I. Metabolic control and growth
during exclusive growth hormone treatment in X-linked hypophosphatemic rickets. Horm Res.
2008;69:212-220. DOI: 10.1159/000113021. Abstract available at:
http://www.ncbi.nlm.nih.gov/pubmed/18204268

Rauch F, Scheinman SJ, Agus ZS, Drezner MK. Hereditary hypophosphatemic rickets and tumor-
induced osteomalacia. UpToDate Online 17.3. 2008. Available at:
http://www.uptodateonline.com/online/content/topic.do?topicKey=pediendo/11337&selectedTitle=
1%7E87&source=search_result

Roth KS, Chan JCM. Hypophosphatemic rickets. eMedicine Specialties, Pediatrics: General
Medicine, Endocrinology. 2009. Available at: http://emedicine.medscape.com/article/922305-
overview

Roth KS, Chan JCM. Hypophosphatemic rickets: treatment & medication. eMedicine Specialties,
Pediatrics: General Medicine, Endocrinology. 2009. Available at:
http://emedicine.medscape.com/article/922305-treatment

Copyright 2010 by the American Academy of Pediatrics page 345


2011 PREP SA on CD-ROM

Question: 107

A 7-year-old boy has mild intellectual impairment and an autism spectrum disorder. He has a
pleasant disposition and no aggressive behaviors, but he has trouble paying attention in the
classroom and difficulty staying in his seat. Both teacher report and parent rating forms document
highly significant hyperactivity and difficulty paying attention. His parents are concerned that his
activity level is affecting his ability to learn. They ask if any medication can help him attain school
success.

Of the following, the MOST appropriate response is to

A. begin an atypical antipsychotic

B. begin a trial of a serotonin reuptake inhibitor

C. begin a trial of stimulant medication

D. explain that medication would be ineffective due to his intellectual disability

E. reassure the family that he is doing his best and have them return in 6 months

Copyright 2010 by the American Academy of Pediatrics page 346


2011 PREP SA on CD-ROM

Critique: 107 Preferred Response: C

The child described in the vignette has an autism spectrum disorder, symptoms of
hyperactivity, and a short attention span. According to his teacher and parents, these symptoms
are affecting his ability to learn at school. Therefore, a trial of a stimulant medication is indicated
to target the symptoms. Stimulants can be effective, even in individuals who have intellectual
disabilities. In 2005, the Research Units on Pediatric Psychopharmacology Autism Network
studied methylphenidate in the treatment of children who had autism spectrum disorders. The
results indicated drug efficacy, but with reduced effectiveness and a less satisfactory adverse
effect profile. The most reported adverse effect was decreased appetite.
The boy is having significant difficulty, and simply reassuring the family will not help his
functioning at school. An atypical antipsychotic is not indicated in this situation because the child
is not exhibiting aggressive or self-injurious behaviors, irritability, or explosive outbursts.
Serotonin reuptake inhibitors are indicated for the treatment of obsessive-compulsive behaviors,
anxiety, or symptoms of low mood, symptoms not exhibited by the boy in the vignette.

Suggested reading:

American Academy of Pediatrics Autism Expert Panel. Autism: Caring for Children with Autism
Spectrum Disorders: A Resource Toolkit for Clinicians. Elk Grove, Ill: American Academy of
Pediatrics; 2007

Leskovec TJ, Rowles BM, Findling RL. Pharmacological treatment options for autism spectrum
disorders in children and adolescents. Harv Rev Psychiatry. 2008;16:97-112. DOI:
10.1080/10673220802075852. Abstract available at:
http://www.ncbi.nlm.nih.gov/pubmed/18415882

Meyers SM. Management of autism spectrum disorders in primary care. Pediatr Ann. 2009;18:42-
49

Myers SM, Johnson CP and the Council on Children with Disabilities. Management of children
with autism spectrum disorders. Pediatrics. 2007;120:1162-1182. DOI: 10.1542/peds.2007-
2362. Available at: http://pediatrics.aappublications.org/cgi/content/full/120/5/1162

Copyright 2010 by the American Academy of Pediatrics page 347


2011 PREP SA on CD-ROM

Question: 108

A 3-year-old boy developed a petechial rash (Item Q108) beginning on his wrists and ankles 5
days after a family camping trip in the Chesapeake Bay area. A latex agglutination assay confirms
the diagnosis of Rocky Mountain spotted fever.

Of the following, the MOST appropriate antibiotic choice for treating this child is

A. amoxicillin

B. azithromycin

C. chloramphenicol

D. doxycycline

E. trimethoprim-sulfamethoxazole

Copyright 2010 by the American Academy of Pediatrics page 348


2011 PREP SA on CD-ROM

Question: 108

(Courtesy of D Krowchuk)
Rash, as described for the child in the vignette.

Copyright 2010 by the American Academy of Pediatrics page 349


2011 PREP SA on CD-ROM

Critique: 108 Preferred Response: D

Tetracyclines generally are not recommended for children younger than 8 years of age due
to concerns about staining of dental enamel. However, if the drug is the best option for
treatment, the risk should be weighed against the potential benefits of therapy. Tetracycline in
the form of doxycycline is the treatment of choice for Rocky Mountain spotted fever (RMSF).
Furthermore, available data suggest that doxycycline (versus tetracycline) appears not to cause
discoloration of permanent teeth.
Chloramphenicol has been considered an alternative agent for treatment for RMSF, although
the drug no longer is available in the United States. When used, chloramphenicol serum
concentrations must be monitored to prevent bone marrow toxicity. Chloramphenicol may be less
effective than doxycycline in treating RMSF. Furthermore, doxycycline provides coverage
against other tickborne organisms (Ehrlichia, Anaplasma) that chloramphenicol does not.
Amoxicillin, azithromycin, and trimethoprim-sulfamethoxazole are not active against
Rickettsia rickettsii, the etiologic agent of RMSF.
In addition to their use in rickettsial infections, tetracyclines may have a role in the treatment
of chlamydial infections (eg, nongonococcal urethritis, pelvic inflammatory disease), Lyme
disease, community-acquired methicillin-resistant Staphylococcus aureus skin and soft-tissue
infections, Legionnaires disease, Mycoplasma infections, leptospirosis, chloroquine-resistant
malaria, and travelers diarrhea.

As a result of reviewing this information, do you intend to make a change in practice


to provide better patient care?
Yes No

Suggested reading:

American Academy of Pediatrics. Rocky Mountain spotted fever. In: Pickering LK, Baker CJ,
Kimberlin DW, Long SS, eds. Red Book: 2009 Report of the Committee on Infectious Diseases.
28th ed. Elk Grove Village, Ill: American Academy of Pediatrics; 2009:573-575

American Academy of Pediatrics. Tetracyclines. In: Pickering LK, Baker CJ, Kimberlin DW, Long
SS, eds. Red Book: 2009 Report of the Committee on Infectious Diseases. 28th ed. Elk Grove
Village, Ill: American Academy of Pediatrics; 2009:739.

May DB. Tetracyclines. UpToDate Online 17.3. 2009. Available at:


http://www.uptodate.com/online/content/topic.do?topicKey=antibiot/9555&selectedTitle=5%7E15
0&source=search_result819

Copyright 2010 by the American Academy of Pediatrics page 350


2011 PREP SA on CD-ROM

Question: 109

A 9-year-old boy presents with fever, headache, and mild confusion in August. His mother tells
you she found several dead birds in their backyard. Physical examination reveals a febrile child
who knows his name and age but is not oriented to place. He has mild meningismus. You note
numerous mosquito bites on his arms and legs. Cerebrospinal fluid examination reveals 30 white
blood cells/mm3, with 70% lymphocytes, 20% polymorphonuclear leukocytes, and 10%
monocytes; protein and glucose values are normal.

Of the following, the MOST appropriate information to share with the patients mother is that

A. application of diethyltoluamide (DEET) could have prevented this infection

B. appropriate antibiotics will cure the infection

C. appropriate antiviral therapy can shorten the duration of illness

D. blood culture may confirm the diagnosis

E. long-term neurologic sequelae are to be expected

Copyright 2010 by the American Academy of Pediatrics page 351


2011 PREP SA on CD-ROM

Critique: 109 Preferred Response: A

The findings described for child in the vignette, coupled with the report of dead birds,
suggest that the boy has meningitis caused by West Nile Virus (WNV). WNV is transmitted to
humans primarily from the bite of infected Culex mosquitoes, which acquire the virus by feeding
on infected birds and transmit it to humans and other mammals. WNV transmission has been
reported in all areas of the United States except Alaska and Hawaii. It also has been described in
Canada, Mexico, Central America, the Caribbean, Europe, the Middle East, Africa, India, Asia, and
Australia. The risk of infection is higher in warmer months, when mosquitoes are abundant.
Intrauterine transmission, transmission through human milk, and transmission through blood
transfusion and organ donation are rare. WNV infection can be prevented by avoiding being
outdoors from dusk to dawn, using effective insect repellents (such as diethyltoluamide [DEET],
picaridin, and oil of eucalyptus), and screening blood and organ donors.
Fewer than 1% of people infected with WNV develop neuroinvasive disease. Most people
who develop WNV neuroinvasive disease are older than 60 years of age. Meningitis,
encephalitis, seizures, flaccid paralysis, focal neurologic deficits, Guillain-Barr syndrome, and
movement disorders can occur. Most patients who have meningitis recover completely, but some
who have encephalitis may have residual neurologic sequelae. Flaccid paralysis caused by
WNV may persist. The boy in the vignette has meningitis but does not exhibit overt
encephalopathy. Therefore, long-term neurologic sequelae would not be expected.
Most people who have WNV infection are asymptomatic, but about 20% develop a systemic
febrile illness called West Nile fever. West Nile fever is a flulike illness manifested by fever,
headache, myalgias, and weakness and often is accompanied by abdominal pain, nausea,
vomiting, or diarrhea. Occasionally, a transient maculopapular rash occurs. Rare complications
of WNV infection include optic neuritis, chorioretinitis, uveitis, myocarditis and cardiac
arrhythmias, rhabdomyolysis, hepatitis, pancreatitis, and orchitis.
WNV infection is diagnosed by testing serum or cerebrospinal fluid (CSF) for WNV-specific
immunoglobulin (Ig) M antibody or by documenting a fourfold increase in WNV-specific IgG
antibody titer between acute and convalescent serum samples collected at least 2 weeks apart.
State public health laboratories and the Centers for Disease Control and Prevention can perform
viral culture and nucleic acid amplification tests on serum, CSF, and tissue specimens, but these
tests routinely do not return positive results during the acute illness. Routine blood cultures do
not detect the virus.
The treatment of WNV disease is supportive; no antiviral therapy has shown specific
benefit. Antibiotic therapy is not appropriate for treating viral meningitis.

Suggested reading:

American Academy of Pediatrics. West Nile virus. In: Pickering LK, Baker CJ, Kimberlin DW, Long
SS, eds. Red Book: 2009 Report of the Committee on Infectious Diseases. 28th ed. Elk Grove
Village, Ill: American Academy of Pediatrics; 2009:730-733

Division of Vector-borne Infectious Diseases. West Nile Virus. Atlanta, Ga: Centers for Disease
Control and Prevention; 2009. Available at: http://www.cdc.gov/ncidod/dvbid/westnile/index.htm

Copyright 2010 by the American Academy of Pediatrics page 352


2011 PREP SA on CD-ROM

Romero JR. West Nile virus. In: Feigin RD, Cherry JD, Demmler-Harrison GJ, Kaplan SL, eds.
Feigin & Cherrys Textbook of Pediatric Infectious Diseases. 6th ed. Philadelphia, Pa: Saunders
Elsevier; 2009:2331-2342

Copyright 2010 by the American Academy of Pediatrics page 353


2011 PREP SA on CD-ROM

Question: 110

An 8-year-old boy who has intellectual disability and who had a history of hypercalcemia in his
first postnatal year presents for a health supervision visit. Physical examination reveals a
temperature of 36.8C, heart rate of 80 beats/min, respiratory rate of 16 breaths/min, and blood
pressure of 144/90 mm Hg. You note a II/VI systolic murmur over the left upper sternal border
radiating into the neck, an abdominal bruit to the right of the midline, and good pulses over the
radial arteries and femoral arteries without radial-femoral delay. A dipstick urinalysis yields
negative results.

Of the following, the MOST likely cause for this patients findings is

A. coarctation of the aorta

B. hyperthyroidism

C. reflux nephropathy

D. renal artery stenosis

E. renal dysplasia

Copyright 2010 by the American Academy of Pediatrics page 354


2011 PREP SA on CD-ROM

Critique: 110 Preferred Response: D

The boy described in the vignette most likely has Williams syndrome (WS), based on his past
medical history of hypercalcemia and intellectual disability. He now presents with hypertension,
a systolic murmur, and an abdominal bruit. The most likely diagnosis in this clinical setting is renal
artery stenosis (RAS). In addition to RAS, more common causes for hypertension in the pediatric
patient include renal causes such as scarring from reflux nephropathy and renal parenchymal
disease (glomerulonephritis, polycystic kidney disease). This patients normal urinalysis results
make renal parenchymal disease unlikely. Reflux nephropathy would not be expected to be
associated with a heart murmur and an abdominal bruit. Renal dysplasia can be associated with
azotemia, but it is an uncommon cause of hypertension because most patients have associated
renal salt wasting. Hyperthyroidism is a cause for hypertension in children, but it often is
associated with tachycardia and thyromegaly and not the physical findings seen in the patient in
the vignette. Coarctation of the aorta is part of the midaortic syndrome that can be seen in
children who have WS, but this patients good femoral pulses, absence of radial-femoral delay,
and unilateral bruit to the right of the midline makes this diagnosis less likely.
WS, also known as Williams-Beuren syndrome, has an incidence of 1 in 10,000 and affects
children worldwide across all ethnicities. It was described initially in 1961, characterizing four
children who had supravalvular aortic stenosis. Further understanding of this disorder
documented widespread vascular involvement, including pulmonary artery stenosis, coarctation
of the aorta, aortic hypoplasia, and renal artery stenosis. The very similar vascular findings to
those in familial supravalvular aortic stenosis (familial SVAS) led to identification of the
involvement of the elastin gene as the genetic locus for this disorder. Because WS also has a
number of associated abnormalities not seen in familial SVAS, further characterization
demonstrated a microdeletion disorder of approximately 1.5-Mb pairs of DNA on chromosome 7
(7q11.23). This microdeletion involves the elastin gene and some 25 to 27 other genes, which
may explain the many other features of WS.
In addition to the vascular involvement, the spectrum of associated findings in WS begins
with infantile hypercalcemia, which can lead to hypercalciuria and nephrocalcinosis that usually
resolves by 1 to 2 years of age. Developmental problems have been well described. Affected
children have some degree of intellectual disability, with intelligence quotient (IQ) scores ranging
from 40 to 90. Language is relatively preserved, with fluent speech and better-than-expected
vocabulary than predicted by the IQ. Children who have WS struggle with visuospatial
relationships and understanding math concepts. Although they function well socially and have
friendly demeanors, they do exhibit some phobias and anxiety. The phenotypic physical features
of WS include broad forehead, small nose, long philtrum, wide mouth, and full lips. Some of these
features soften somewhat into adulthood, but others persist.

Suggested reading:

Bouchireb K, Boyer O, Bonnet D, et al. Clinical features and management of arterial hypertension
in children with Williams-Beuren syndrome. Nephrol Dial Transplant. 2009;25:434-438. Abstract
available at: http://www.ncbi.nlm.nih.gov/pubmed/19815602

Copyright 2010 by the American Academy of Pediatrics page 355


2011 PREP SA on CD-ROM

Lashkari A, Smith AK, Graham JM Jr. Williams-Beuren syndrome: an update and review for the
primary physician. Clin Pediatr. 1999;38:189-208. Abstract available at:
http://www.ncbi.nlm.nih.gov/pubmed/10326175

Copyright 2010 by the American Academy of Pediatrics page 356


2011 PREP SA on CD-ROM

Question: 111

A 1-year-old boy presents with a 2-month history of "wet" coughing. He was delivered at term
but had delayed passage of meconium due to a meconium plug. Over the past 3 to 6 months, he
has been treated each month for acute otitis media. His parents are concerned that, despite a
good appetite, their son has been losing weight and has four to six loose, foul-smelling stools per
day.

Of the following, the MOST appropriate next test or study is

A. 24-hour pH probe monitoring

B. pulmonary function testing

C. serum immunoglobulins (IgG, IgA, and IgM)

D. sweat chloride measurement

E. tuberculin skin test

Copyright 2010 by the American Academy of Pediatrics page 357


2011 PREP SA on CD-ROM

Critique: 111 Preferred Response: D

An infant or toddler who presents with loose, foul-smelling stools; poor weight gain; and
recurrent ear, sinus, or lung infections, as described for the boy in the vignette, should be
evaluated for cystic fibrosis. The classic initial screening for cystic fibrosis is measurement of
sweat chloride via the quantitative pilocarpine iontophoresis test. Values of 60 mmol/L or greater
are consistent with cystic fibrosis and should be repeated at least once, 1 month after the first
test.
Chronic cough or poor weight gain may represent extraesophageal manifestations of
gastroesophageal reflux disease (GERD), although the more common symptoms of vomiting or
epigastric pain are not present in this child. If GERD is suspected, intraesophageal pH probe
monitoring can aid in determining the frequency of reflux. One index, termed the reflux index, is
considered consistent for GERD if the pH is less than 4.0 more than 12% of the time in children
older than 1 year or more than 6% of the time in children younger than 1 year of age.
Primary humoral immunodeficiency (eg, Bruton agammaglobulinemia, common variable
immunodeficiency), transient hypogammaglobulinemia of infancy, or secondary
immunodeficiency due to protein-losing states may present in infancy with diarrhea and
recurrent infections. Common variable immunodeficiency also has pulmonary manifestations (eg,
bronchiectasis) similar to those of cystic fibrosis. However, the diarrhea is due to viral or
bacterial infections, in contrast to the fat malabsorption in cystic fibrosis. Assessment of
immunoglobulin G, A, and M concentrations may be warranted for this child, but the initial
presentation, coupled with the foul-smelling stools and poor weight gain, make cystic fibrosis
more likely.
Tuberculosis may present with chronic cough, poor weight gain, fever, and decreased
energy. Common risk factors for children include foreign travel, association with a person(s)
who has tuberculosis, or growing up in an area that has high tuberculosis prevalence. Although
the Mantoux purified protein derivative is used as a screening tool for latent tuberculosis
infection, the gold standard for diagnosing pulmonary tuberculosis is identification of the
organism from respiratory tract specimens.
Pulmonary function testing is used to measure inspiratory and expiratory airflow. Three
patterns typically are identified: normal, obstructive, and restrictive. Patients who have cystic
fibrosis or moderate-to-severe asthma generally have obstructive lung disease. Measuring lung
function for the child in the vignette would be helpful, but children typically are not able to
perform the coordinated expiration and inspiration required for pulmonary function testing until 5
to 7 years of age.
When evaluating a chronic cough in a child, a thorough history and physical examination
often suggest the most likely diagnosis. Empiric therapy frequently is started and if the cough
resolves, no further diagnostic studies may be required. However, if an initial diagnostic
procedure is desired or if empiric treatment strategies fail, chest radiography, pH probe, or sinus
imaging should be considered. Depending on the clinical history and age of the patient, pertussis,
foreign body, or vascular ring may be additional diagnoses to consider.

Suggested reading:

Copyright 2010 by the American Academy of Pediatrics page 358


2011 PREP SA on CD-ROM

Dranove JE. Focus on diagnosis: new technologies for the diagnosis of gastroesophageal reflux
disease. Pediatr Rev. 2008;29:317-320. DOI: 10.1542/10.1542/pir.29-9-317. Available at:
http://pedsinreview.aappublications.org/cgi/content/full/29/9/317

Montgomery GS, Howenstine M. Cystic fibrosis. Pediatr Rev. 2009;30:302-310. DOI:


10.1542/10.1542/pir.30-8-302. Available at:
http://pedsinreview.aappublications.org/cgi/content/full/30/8/302

Copyright 2010 by the American Academy of Pediatrics page 359


2011 PREP SA on CD-ROM

Question: 112

A 2-year-old girl is brought to the emergency department by paramedics after being found at the
bottom of a backyard swimming pool. She had apnea and no pulse at the scene, but after
cardiopulmonary resuscitation, intubation, and assisted ventilation, physical examination in the
emergency department reveals a temperature of 31.5C, heart rate of 100 beats/min, blood
pressure of 100/60 mm Hg, and the beginning of spontaneous movement.

Of the following, the MOST appropriate next step in management is to

A. activate the extracorporeal membrane oxygenation team

B. administer peritoneal lavage with warm fluid

C. infuse intravenous fluids heated to 37.0C

D. place bilateral chest tubes and initiate warm pleural lavage

E. provide heated, humidified oxygen via the endotracheal tube

Copyright 2010 by the American Academy of Pediatrics page 360


2011 PREP SA on CD-ROM

Critique: 112 Preferred Response: E

Hypothermia is a common complication of near-drowning, even among children who drown


in relatively warm water. Hypothermia may develop during submersion, when conductive and
convective heat loss occurs very quickly in water, or during resuscitation, when the patient is
exposed to ambient temperatures. The methods used for rewarming depend on the severity of
hypothermia, as determined by the core body temperature (Item C112).
All patients who have hypothermia should have their core temperatures monitored using a
bladder, esophageal, or rectal probe and a low-register thermometer. Wet clothes must be
removed and vital functions assessed and monitored. For patients who have moderate-to-
severe hypothermia, care must be taken to avoid precipitation of ventricular dysrhythmias by
vigorous stimulation or "afterdrop," the initial drop in core temperature in early rewarming as cold
blood is mobilized from the extremities. Rapid rewarming is indicated once the patient is in a
health-care facility. In the field, aggressive rewarming should not be initiated to avoid
"afterdrop."
There is debate in the literature over the resuscitation of profoundly hypothermic patients
who have experienced prolonged cardiac arrest in the setting of uncontrolled, accidental
hypothermia associated with submersion. Rare cases of neurologically intact survival have been
reported following submersion in icy water (<5.0C). In circumstances other than this,
hypothermia is a poor prognostic indicator for intact survival.
The patient described in the vignette is suffering from moderate hypothermia. The
therapeutic goal for patients who have moderate-to-severe hypothermia is to raise and maintain
the core temperature to more than 35.0C. Administration of warmed, heated oxygen (42.0 to
46.0C) via the endotracheal tube and infusion of intravenous fluids warmed to more than 40.0C
are effective rewarming methods in this setting. Aggressive interventions such as pleural or
peritoneal lavage or extracorporeal membrane oxygenation typically are not necessary.

Suggested reading:

American Heart Association. ECC Guidelines. Part 8: Advanced challenges in resuscitation.


Section 3. Special challenges in ECC. Circulation. 2000;102:I-229. Available at:
http://circ.ahajournals.org/cgi/content/short/102/suppl_1/I-229

Antoon AY, Donovan MK. Cold injuries. In: Kleigman RM, Behrman RE, Jenson HB, Stanton BF,
eds. Nelson Textbook of Pediatrics. 18th ed. Philadelphia, Pa: Saunders Elsevier; 2007:458-459

Kallas HJ. Drowning and submersion injury. In: Kleigman RM, Behrman RE, Jenson HB, Stanton
BF, eds. Nelson Textbook of Pediatrics. 18th ed. Philadelphia, Pa: Saunders Elsevier; 2007:438-
449

Copyright 2010 by the American Academy of Pediatrics page 361


2011 PREP SA on CD-ROM

Critique: 112

Copyright 2010 by the American Academy of Pediatrics page 362


2011 PREP SA on CD-ROM

Question: 113

You are evaluating a 7-week-old infant for persistent jaundice. She was born at term, following
an uncomplicated pregnancy and delivery, weighed 3.2 kg at birth, and has been exclusively
breastfed. She was first evaluated for jaundice at 3 weeks of age. The total bilirubin
concentration was 16.0 mg/dL (273.7 mcmol/L), and abdominal ultrasonography demonstrated "a
collapsed gall bladder without dilatation of the intrahepatic or extrahepatic bile ducts." You
diagnosed breast milk jaundice and on follow-up visits every few days you noted a gradual
reduction in total bilirubin. Physical examination demonstrates an alert, icteric infant whose weight
is 4.2 kg. She has a firm liver edge palpable 1.5 cm below the right costal margin and a spleen tip
palpable 2 cm below the left costal margin. You obtain the following laboratory data:

Hemoglobin, 9.5 mg/dL (95 g/L)


White blood cell count, 10.5x10 3/mcL (10.5x109/L)
Total bilirubin, 8.5 mg/dL (145.4 mcmol/L)
Direct bilirubin, 4.5 mg/dL (77.0 mcmol/L)
Alanine aminotransferase, 140 units/L
Aspartate aminotransferase, 70 units/L
Alkaline phosphatase, 450 units/L

Of the following, the MOST appropriate next diagnostic test is

A. abdominal computed tomography scan

B. alpha-1-antitrypsin assessment

C. intraoperative cholangiography

D. percutaneous liver biopsy

E. urine succinylacetone measurement

Copyright 2010 by the American Academy of Pediatrics page 363


2011 PREP SA on CD-ROM

Critique: 113 Preferred Response: D

The infant described in the vignette has direct hyperbilirubinemia, which is defined by a
serum direct bilirubin concentration of more than 1.0 mg/dL (17.1 mcmol/L) with total bilirubin
values of less than 5.0 mg/dL (85.5 mcmol/L) or greater than 20% of the total bilirubin for values
greater than 5.0 mg/dL (85.5 mcmol/L). Direct hyperbilirubinemia indicates cholestasis and is an
abnormal finding that requires additional evaluation. Immediate evaluation of hepatobiliary integrity
in this infant is critical to determine if the direct hyperbilirubinemia is a consequence of
extrahepatic biliary atresia. Recent studies have shown the best surgical outcomes for infants
who have biliary atresia when the diagnosis is established by 30 to 45 days of age. By 7
weeks, as seen for the infant in the vignette, physical examination may show findings
consistent with hepatic fibrosis (firm liver edge) and portal hypertension (splenomegaly).
Therefore, even while additional tests are obtained to determine alternate causes, biliary atresia
must be ruled out urgently, and a liver biopsy is essential. Histologic sections demonstrate portal
tract bile ductular proliferation (Item C113), a pathognomonic sign of extrahepatic biliary tract
obstruction.
Other causes of neonatal cholestasis include infections and metabolic diseases.
Measurement of urinary succinylacetone (to rule out tyrosinemia) and alpha-1-antitrypsin (to
assess for possible alpha-1-antitrypsin deficiency-associated liver disease) are part of the
standard evaluation for neonatal cholestasis and, in this infant, are indicated if the liver biopsy
rules out biliary atresia.
Although the "gold standard" for identifying extrahepatic biliary tract obstruction remains
percutaneous liver biopsy, recent data indicate that biliary tract ultrasonography, conducted by
an experienced ultrasonographer, may be helpful. The finding of a "triangular cord sign," which
represents the fibrous remnant of an obliterated extrahepatic biliary tree, recently was shown to
be highly specific for biliary atresia. Other diagnostic modalities, including magnetic resonance
cholangiopancreatography and endoscopic retrograde cholangiopancreatography, currently are
being investigated. Abdominal computed tomography scan may suggest an absent or atretic
gallbladder, but this imaging study has not proved to be of significant value in the evaluation of
biliary atresia.
Hepatobiliary scintigraphy, using a radiolabeled derivative of iminodiacetic acid, has long
been touted as an important adjunct to the evaluation of neonatal cholestasis. Although definitive
evidence of radiolabel excretion into the small bowel may confirm biliary tract patency, the study
has a significant incidence of both false-positive and false-negative results.
At surgery, the diagnosis of extrahepatic biliary atresia can be confirmed via intraoperative
cholangiography, which is followed by dissection of the atretic, extrahepatic biliary tree to the
liver hilum. A portoenterostomy (Kasai procedure) is performed, with anastomosis of a Roux-en-
Y jejunal limb to the porta hepatis.
A number of medical therapies, including corticosteroids and ursodeoxycholic acid, have
been suggested as means to enhance bile flow in the postoperative period. For those patients in
whom bile flow is achieved, cholangitis remains a significant cause of both short- and long-term
morbidity. As a preventive measure, prophylactic antimicrobial therapy has been used widely
after portoenterostomy, but the efficacy of this therapeutic approach has yet to be confirmed.
Early diagnosis greatly enhances the likelihood of successful portoenterostomy that

Copyright 2010 by the American Academy of Pediatrics page 364


2011 PREP SA on CD-ROM

establishes bile flow and allows patients a prolonged survival period with their native livers. Any
formula-fed infant who is noted to be jaundiced at the 2-week health supervision visit should be
assessed for cholestasis by initially obtaining a fractionated (total and direct reacting) bilirubin
value. For nursing infants, in whom breast milkassociated jaundice may result in a prolonged
period of indirect hyperbilirubinemia, this evaluation may be delayed until 3 weeks of age, if
findings on physical examination are normal, the infant has no history of dark-colored urine or
light-colored stools, and the infant can be monitored reliably.

Suggested reading:

Bassett MD, Murray KF. Biliary atresia: recent progress. J Clin Gastroenterol 2008;42:720-729.
DOI: 10.1097/MCG.0b013e3181646730. Abstract available at:
http://www.ncbi.nlm.nih.gov/pubmed/18496390

Moyer V, Freese DK, Whitington PF, et al. Guideline for the evaluation of cholestatic jaundice in
infants: recommendations of the North American Society for Pediatric Gastroenterology,
Hepatology and Nutrition. J Pediatr Gastroenterol Nutr. 2004;39:115-128. Available at:
http://journals.lww.com/jpgn/Fulltext/2004/08000/Guideline_for_the_Evaluation_of_Cholestatic.1.
aspx

Roach JP, Bruny JL. Advances in the understanding and treatment of biliary atresia. Curr Opin
Pediatr. 2008;20:315-319. DOI: 10.1097/MOP.0b013e3282ffdc43. Abstract available at:
http://www.ncbi.nlm.nih.gov/pubmed/18475102

Sokol RJ, Shepherd RW, Superina R, et al. Screening and outcomes in biliary atresia: summary of
a National Institutes of Health workshop. Hepatology. 2007;46:566-581. DOI: 10.1002/hep.21790.
Available at: http://www3.interscience.wiley.com/cgi-bin/fulltext/114297836/HTMLSTART

Suchy FJ. Neonatal cholestasis. Pediatr Rev. 2004;25:388-396. doi:10.1542/10.1542/pir.25-11-


388. Available at: http://pedsinreview.aappublications.org/cgi/content/full/25/11/388

Utterson EC, Shepherd RW, Sokol RJ, et al. Biliary atresia: clinical profiles, risk factors, and
outcomes of 755 patients listed for liver transplantation. J Pediatr. 2005;147:180-185.
doi:10.1016/j.jpeds.2005.04.073. Abstract available at:
http://www.ncbi.nlm.nih.gov/pubmed/16126046

Wadhwani SI, Turmelle YP, Nagy R, Lowell J, Dillon P, Shepherd RW. Prolonged neonatal jaundice
and the diagnosis of biliary atresia: a single-center analysis of trends in age at diagnosis and
outcomes. Pediatrics. 2008;121:e1438-e1440. DOI: 10.1542/peds.2007-2709. Available at:
http://pediatrics.aappublications.org/cgi/content/full/121/5/e1438

Copyright 2010 by the American Academy of Pediatrics page 365


2011 PREP SA on CD-ROM

Critique: 113

(Courtesy of S Schwarz)
Percutaneous liver biopsy specimen (hematoxylin and eosin stain) showing a portal area with bile
duct proliferation (ie, an increased number of ducts) (yellow arrows). Areas of hepatocellular bile
staining are also evident (brown pigment, demonstrated by the white arrows).

Copyright 2010 by the American Academy of Pediatrics page 366


2011 PREP SA on CD-ROM

Question: 114

You are evaluating a 28-day-old infant in the special care nursery who was born at 29 weeks
gestation. He is tolerating fortified human milk via a nasogastric tube with no "spit-ups." The only
medication he is receiving is iron. He has been breathing room air for 2 weeks, and caffeine
therapy was discontinued 4 days ago. Last night, he had several respiratory pauses lasting 20
seconds that were associated with mild desaturations and bradycardia that required gentle
stimulation to resolve. His hemoglobin is 12 g/dL (120 g/L), and his white blood cell count and
differential count are normal.

Of the following, the MOST appropriate initial intervention to decrease his respiratory pauses is to

A. administer metoclopramide

B. initiate continuous positive airway pressure

C. place an endotracheal tube

D. restart the caffeine

E. transfuse packed red blood cells

Copyright 2010 by the American Academy of Pediatrics page 367


2011 PREP SA on CD-ROM

Critique: 114 Preferred Response: D

The preterm infant described in the vignette is having apnea of prematurity and would
benefit from restarting caffeine. Apnea of prematurity is seen commonly in infants born at less
than 34 weeks gestation, with resolution generally occurring by 37 to 40 weeks corrected
gestational age. It is defined most commonly by the absence of respiratory flow for more than 20
seconds, often accompanied by color change and bradycardia. Apnea of prematurity usually is
a mixed apnea, with both central and obstructive components. The immature brainstem
respiratory centers of the preterm infant inconsistently produce excitatory impulses to stimulate
respiratory effort, leading to central apnea. The obstructive apnea is due to the underdeveloped
airway, which has a tendency to collapse when the neck is hyperextended or hyperflexed.
During sleep, newborns of all gestational ages have periods when regular respiratory effort
is interrupted by regular, recurring cycles (each of 10 to 15 seconds duration) consisting of
brief pauses of 3 to 10 seconds followed by bursts of rapid breathing (50 to 60 breaths/min).
This pattern of breathing is known as periodic breathing and is of no concern unless associated
with color change or bradycardia, which are signals for further evaluation.
The preterm or late preterm infant who has new-onset or increasing apnea must be
assessed for underlying causes. Infection and severe anemia must be ruled out. If clinical
examination or laboratory studies support infection, antibiotics should be initiated. A transfusion
of packed red blood cells may be considered if symptomatic anemia is suspected. The initiation
of caffeine is most appropriate if the initial evaluation is reassuring. Caffeine therapy reduces
the frequency and intensity of apnea of prematurity and lowers the rate of bronchopulmonary
dysplasia in very low-birthweight infants. Respiratory support with continuous positive airway
pressure or intubation may be needed if the apnea is severe, based on the frequency of the
events or the degree of intervention required. Treatment for gastroesophageal reflux (eg, with
metoclopramide) should be initiated cautiously because the relationship between
gastroesophageal reflux and apnea continues to be debated.

Suggested reading:

AAP Committee on Fetus and Newborn. Apnea, sudden infant death syndrome, and home
monitoring. Pediatrics. 2003:111:914-917. Available at:
http://pediatrics.aappublications.org/cgi/content/full/111/4/914

Di Fiore JM, Arko M, Whitehouse M, Kimball A, Martin RJ. Apnea is not prolonged by acid
gastroesophageal reflux in preterm infants. Pediatrics 2005;116:1059-1063. DOI:
10.1542/peds.2004-2757. Available at:
http://pediatrics.aappublications.org/cgi/content/full/116/5/1059

Dudell GG, Stoll BJ. Respiratory tract disorders. In: Kleigman RM, Behrman RE, Jenson HB,
Stanton BF, eds. Nelson Textbook of Pediatrics. 18th ed. Philadelphia, Pa: Saunders Elsevier;
2007:728-730

Martin RJ, Abu-Shaweesh JM, Baird TM. Apnoea of prematurity. Paediatr Respir Rev.

Copyright 2010 by the American Academy of Pediatrics page 368


2011 PREP SA on CD-ROM

2004;5(suppl A):S377-S382

Matiz A, Roman EA, Adam HM. In brief: apnea. Pediatr Rev. 2003;24:32-34. DOI:
10.1542/10.1542/pir.24-1-32. Available at:
http://pedsinreview.aappublications.org/cgi/content/full/24/1/32

Schmidt B, Roberts RS, Davis P, et al; Caffeine for Apnea of Prematurity Trial Group. Caffeine
therapy for apnea of prematurity. N Engl J Med. 2006;354:2112-2121. Abstract available at:
http://content.nejm.org/cgi/content/abstract/354/20/2112

Copyright 2010 by the American Academy of Pediatrics page 369


2011 PREP SA on CD-ROM

Question: 115

You are evaluating an otherwise healthy 3-year-old boy who has developed a cluster of 5 to 10
flesh-colored, pearly papules, some of which have central umbilication (Item Q115). He has no
prior skin conditions.

Of the following, the MOST appropriate management is

A. cimetidine orally

B. laser therapy

C. observation

D. permethrin cream

E. triamcinolone cream

Copyright 2010 by the American Academy of Pediatrics page 370


2011 PREP SA on CD-ROM

Question: 115

(Courtesy of D Krowchuk)
Eruption, as described for the boy in the vignette.

Copyright 2010 by the American Academy of Pediatrics page 371


2011 PREP SA on CD-ROM

Critique: 115 Preferred Response: C

Molluscum contagiosum are small (1 mm to 1 cm) shiny, pearly, round papules with central
depressions (umbilications) (Item C115A) that occur in 2% to 8% of all children. They are caused
by a DNA pox virus called molluscum contagiosum virus (MCV). MCV1 and MCV1v are the
common agents causing lesions in healthy children; MCV2 is the primary cause of human
immunodeficiency virus (HIV)-related lesions and sexually transmitted infection. Infection is
passed from person to person by direct contact with an infected person or contaminated
fomites; it also may occur after sports contact. Infection may be present for 6 months before
lesions are evident. The condition is rare in children younger than 6 months of age, and although
it may occur at any age, is most frequent in those 2 to 5 years old. It occurs more commonly in
people who have defects in cellular immunity, and the frequency may increase in the presence
of atopic dermatitis. Without treatment, individual lesions usually resolve in 6 to 9 months, and the
infection clears within 1 to 4 years. Molluscum contagiosum often behaves differently in patients
who have HIV and resists treatment more than in healthy children.
Molluscum contagiosum is a benign condition, but it has potential for complications and
adverse effects that have led to a search for effective treatments. Reasons to treat generally
fall into five categories: 1) discomfort and pruritus, 2) cosmetic and psychological implications, 3)
bleeding from lesions, 4) potential for autoinoculation and transmission to others, and 5)
superinfection. Unfortunately, little data other than small series and anecdotal reports support the
use of any of the myriad treatments that have been proposed. Therefore, a recent evidence-
based review of treatments for molluscum contagiosum concludes that there is insufficient
evidence to support any of the proposed treatments. Because of its benign nature and ultimate
self-resolution, observation is frequently the best approach for healthy children, such as the boy
described in the vignette.
When treatment is considered, four types of approaches have been used: physical
destruction, provocation of local inflammatory response, enhancement of immune response, and
antiviral treatment (Item C115B). Many of these treatments involve off-label use of medications.
For patients who have underlying atopic dermatitis, the most important treatment is appropriate
care of the underlying condition, which may include triamcinolone cream. Permethrin cream is
used in the treatment of scabies but is not indicated for treatment of molluscum contagiosum.

Suggested reading:

Cathcart S, Coloe J, Morrell DS. Parental satisfaction, efficacy, and adverse events in 54
patients treated with cantharidin for molluscum contagiosum infection. Clin Pediatr (Phila).
2009;48:161-165. Abstract available at: http://www.ncbi.nlm.nih.gov/pubmed/18936288

Maronn M, Salm C, Lyon V, Galbraith S. One-year experience with Candida antigen


immunotherapy for warts and molluscum. Pediatr Dermatol. 2008;25:189-192. DOI:
10.1111/j.1525-1470.2008.00630.x. Abstract available at:
http://www.ncbi.nlm.nih.gov/pubmed/18429776

Sidbury R. Whats new in pediatric dermatology: update for the pediatrician. Curr Opin Pediatr.

Copyright 2010 by the American Academy of Pediatrics page 372


2011 PREP SA on CD-ROM

2004;16:410-414. Abstract available at: http://www.ncbi.nlm.nih.gov/pubmed/15273502

Silverberg NB. Pediatric molluscum contagiosum: optimal treatment strategies. Paediatr Drugs.
2003;5:505-512. Abstract available at: http://www.ncbi.nlm.nih.gov/pubmed/12895133

Smith KJ, Skelton H. Molluscum contagiosum: recent advances in pathogenic mechanisms, and
new therapies. Am J Clin Dermatol. 2002;3:535-545. Abstract available at:
http://www.ncbi.nlm.nih.gov/pubmed/12358555

van der Wouden JC, Van der Sandr R, van Suijlekom-Smit LW, Berger MY, Butler CC, Koning S.
Interventions for cutaneous molluscum contagiosum. Cochrane Database Syst Rev.
2009;4:CD004767. DOI: 10.1002/14651858.CD004767.pub3. Available at:
http://www.mrw.interscience.wiley.com/cochrane/clsysrev/articles/CD004767/frame.html

Copyright 2010 by the American Academy of Pediatrics page 373


2011 PREP SA on CD-ROM

Critique: 115

(Courtesy of D Krowchuk)
The lesions of molluscum contagiosum are shiny, translucent papules. Some lesions (arrows)
have central depressions (umbilications).

Copyright 2010 by the American Academy of Pediatrics page 374


2011 PREP SA on CD-ROM

Critique: 115

Copyright 2010 by the American Academy of Pediatrics page 375


2011 PREP SA on CD-ROM

Question: 116

An 8-year-old girl presents for follow-up assessment for obesity management with a body mass
index (BMI) of 24 kg/m2, only slightly improved from 25 kg/m2 at her visit 6 months ago. At that
visit, a consultation with a nutritionist was arranged, and the girls parents report successful
reduction in her consumption of sweets, juice, and fast food. Because of bullying at school and
neighborhood crime, she is not allowed to play outside alone. She spends 8 hours each day
watching television.

Of the following, the next BEST step in anticipatory guidance is to recommend

A. removal of the television from the childs bedroom

B. restriction of television and video games to no more than 4 hours per day

C. substitution of video games for passive television viewing

D. watching television only when eating with the family

E. watching videos rather than commercial television

Copyright 2010 by the American Academy of Pediatrics page 376


2011 PREP SA on CD-ROM

Critique: 116 Preferred Response: A

The girl and her family described in the vignette have made modest progress with dietary
changes in addressing her obesity, but she remains significantly overweight. Therefore, further
steps must be taken to reduce her weight. Risk factors for obesity include limited outdoor play
and social isolation. Most importantly, excessive television viewing and a television in her
bedroom place her at risk not only for continued obesity but also for the psychosocial risks of
potentially unsupervised television watching and exposure to substance use and sexuality in
television.
Although technology affords opportunities for young children to develop early computer
skills, the independent use of this technology may be harmful in excess. In particular, there are
concerns that independent television and computer use may replace valuable time spent in
outdoor play and socialization that the traditional playground has offered. The impact of
television viewing on children has been of concern for the last 2 decades, along with other
"screen time" elements of computer and video games. Substituting other forms of "screen time"
(eg, videos, computer) for television time will not address the need for exercise or reduce the
social isolation associated with excessive screen time. Negative effects of television viewing
include increased aggressive behavior and greater acceptance of violence as well as trivializing
sexual activity. Television viewing also blurs the distinction between fantasy and reality. Specific
television programming also can increase the risk of obesity, lead to early onset of sexual
activity and smoking. Increased screen time also results in less time spent in healthier activities,
including unstructured play. Results of numerous studies, including those that are prospective
and randomized in a variety of settings in industrialized countries, suggest that television has
significant effects on child behavior. However, confounders in the studies include ethnicity,
poverty, parental education, and parental health habits. In general, more television viewing
occurs in households that have many of those factors.
Parents should be made aware of the recommendations to limit television viewing to 2 hours
per day or less for all children; should be discouraged from placing a television in a childs
bedroom; and should be made aware of links between television viewing, obesity, and
diminished academic performance.
There is no evidence that television viewing in young children provides any benefit over
reading and playing with parents and other children. Further, television viewing may contribute to
obesity through both the time spent in the sedentary activity and exposure to advertising for
food and largely sedentary toys. Watching television while eating decreases family
communication during meals.
The recommendation for ensuring less than 2 hours of total screen time per day still allows
for sharing of family-oriented programming, limited amounts of computer experience and skill
development, and very limited access to "educational" programming. Older children may find
limiting such activity to be a challenge because of homework requirements using the computer,
use of social networking sites, casual television watching, and playing video games, some of
which may have benefits for social interaction.
Placing a television in the childs bedroom is especially problematic in terms of adult
supervision for both the time and quality of programming. Research has demonstrated that
children who have televisions in their rooms have increased risks of obesity, sleep disturbance

Copyright 2010 by the American Academy of Pediatrics page 377


2011 PREP SA on CD-ROM

(especially sleep latency prolongation), and lower academic performance.

Suggested reading:

Dworak M, Schierl T, Bruns T, Strder HK. Impact of singular excessive computer game and
television exposure on sleep patterns and memory performance of school-aged children.
Pediatrics. 2007;120:978-985. DOI: 10.1542/peds.2007-0476. Available at:
http://pediatrics.aappublications.org/cgi/content/abstract/120/5/978

Funk JB, Brouwer J, Curtiss K, McBroom E. Parents of preschoolers: expert media


recommendations and ratings knowledge, media-effects beliefs, and monitoring practices.
Pediatrics. 2009;123:981-988. DOI: 10.1542/peds.2008-1543. Available at:
http://pediatrics.aappublications.org/cgi/content/abstract/123/3/981?rss=1

Sargent JD, Beach ML, Adachi-Mejia AM, et al. Exposure to movie smoking: its relation to smoking
initiation among us adolescents. Pediatrics. 2005;116:1183-1191. DOI: 10.1542/peds.2005-0714.
Available at: http://pediatrics.aappublications.org/cgi/content/abstract/116/5/1183

Schmidt ME, Rich M, Rifas-Shiman SL, Oken E, Taveras EM. Television viewing in infancy and
child cognition at 3 years of age in a US cohort. Pediatrics. 2009;123:e370-e375. DOI:
10.1542/peds.2008-3221. Available at:
http://pediatrics.aappublications.org/cgi/content/full/123/3/e370

Copyright 2010 by the American Academy of Pediatrics page 378


2011 PREP SA on CD-ROM

Question: 117

A 3-year-old girl presents for a health supervision visit. At birth, she was diagnosed with
hypoplastic left heart syndrome and underwent uncomplicated staged surgical palliation. Today
her mother asks you if her daughters heart disease could affect her development.

Of the following, you are MOST likely to advise the mother that

A. a small percentage of children who undergo neonatal heart surgery may develop transient
motor delay

B. cognitive problems are rare with neonatal heart surgery

C. evaluation for developmental delay should wait until after kindergarten

D. speech and behavioral disorders are common among those who undergo neonatal heart
surgery

E. the only children who suffer developmental delay after neonatal heart surgery are those who
have pre- or postoperative complications

Copyright 2010 by the American Academy of Pediatrics page 379


2011 PREP SA on CD-ROM

Critique: 117 Preferred Response: D

Over the past decade, outcomes associated with neonatal cardiovascular surgery have
improved dramatically. The expectation is that overall survival for pediatric open heart surgery
will exceed 96%. The arterial switch procedure performed for transposition of the great arteries
(TGA) is expected to achieve survival of 95% or greater, and the Norwood operation for
hypoplastic left heart syndrome (HLHS) should maintain mortality at less than 15%. However,
morbidity associated with these procedures continues to exist. End-organ complications may
result from associated congenital anomalies, cardiovascular collapse in the patient who
presents for care at the time of ductal closure, and complications at the time of surgery or
postoperatively (eg, embolic phenomena).
Clinicians are just beginning to understand the neurodevelopmental impact on the child who
has cyanotic heart disease. A wide range of outcomes exists in this population, and screening
programs only recently have been established. The screening is modeled on that performed in
the long-term neonatology follow-up clinics that have been developed for preterm infants. The
most robust programs include expertise in assessing motor milestone attainment, physical and
occupational therapy, neurocognitive evaluation and management, psychological testing, and
educational assessment.
The data surrounding outcomes in the population of patients who have cyanotic congenital
heart disease are raising concerns about various areas of development. The mean full-scale
intelligence quotient (IQ) score has been shown to be 93 in patients who have TGA and have
undergone arterial switch operations. A number of studies have shown that the population of
patients who have undergone Norwood procedures for HLHS have IQ scores between 71 and
90. Overall outcomes for the arterial switch operation demonstrate that 55% of patients have
some degree of impaired cognitive, behavioral, motor, or speech function. A Childrens Hospital of
Philadelphia study showed that 69% of patients who had HLHS had attention-deficit disorder, but
only one patient was receiving medication. This highlights the poor recognition of impairment.
Among patients who have cyanotic heart disease and have undergone staged "single ventricle"
palliation, 45% have required speech therapy, 40% have required physical therapy, 40% have
required special education, and 30% have been followed by a neurologist. Accordingly, the
mother in the vignette should be told that speech and behavioral disorders are common among
those who undergo neonatal heart surgery.
Most patients who undergo neonatal heart surgery experience transient motor delay as they
recover from the stress of intervention and stabilize their hemodynamics. However, only a small
number have long-standing profound gross or fine motor dysfunction unless there is associated
brain injury.
It is important to initiate neurodevelopmental evaluation early for these children. For those in
whom assessment is delayed until the school years, institution of therapy specific to their needs
is less successful, and academic milestones are more difficult to achieve. Assessment should
begin during the toddler years to allow directed therapy. The issues of risk of
neurodevelopmental and physical deficits should be addressed thoroughly as a part of ongoing
and early discussions with families. The discussion should delineate a strategy for obtaining
screening assessments at specific time intervals throughout childhood. Of course, the child who
has more obvious deficits should be referred for consultation with the appropriate specialist at

Copyright 2010 by the American Academy of Pediatrics page 380


2011 PREP SA on CD-ROM

the first sign of limitations.

Suggested reading:

Mahle WT, Visconti KJ, Freier MC, et al. Relationship of surgical approach to neurodevelopmental
outcomes in hypoplastic left heart syndrome. Pediatrics. 2006;117:e90-e97. DOI:
10.1542/peds.2005-0575. Available at:
http://pediatrics.aappublications.org/cgi/content/full/117/1/e90

Mitchell ME, Ittenbach RF, Gaynor JW, Wernovsky G, Nicolson S, Spray TL. Intermediate
outcomes after the Fontan procedure in the current era. J Thorac Cardiovasc Surg.
2006;131:172-180. DOI: 10.1016/j.jtcvs.2005.08.047. Abstract available at:
http://www.ncbi.nlm.nih.gov/pubmed/16399309

Copyright 2010 by the American Academy of Pediatrics page 381


2011 PREP SA on CD-ROM

Question: 118

A 5-year-old boy who is new to your practice presents for a health supervision visit. His mother
and preschool teachers are concerned about the possibility of attention-deficit/hyperactivity
disorder or learning disabilities. On questioning, you learn that he achieved motor and language
milestones within broad normal limits but has always been clumsy. However, his mother thinks his
motor skills have not worsened over time. Physical examination reveals mild dolichocephaly
(anterior/posterior skull elongation), hyperreflexia at the knees and ankles, and elevated tone at
the ankles.

Of the following, this constellation of findings is MOST consistent with

A. elevated bilirubin in infancy

B. hypoxic-ischemic injury at term birth

C. lead intoxication in early childhood

D. periventricular injury after second trimester birth

E. stroke in the third trimester

Copyright 2010 by the American Academy of Pediatrics page 382


2011 PREP SA on CD-ROM

Critique: 118 Preferred Response: D

The boy described in the vignette presents with classic, though mild, findings for a form of
cerebral palsy termed spastic diplegia (Item C118). He has upper motor neuron findings of
spasticity and hyperreflexia in the legs as well as an elongated head, which is characteristic of
children who have loss of brain volume in periventricular white matter. This pattern of brain
injury most commonly occurs in children born preterm, in the second trimester. In addition to the
motor findings in the legs, motor clumsiness and frontal lobe/executive function difficulties are
common.
Brain injury due to elevated bilirubin in infancy, also known as kernicterus or chronic bilirubin
encephalopathy, presents with hearing loss, a hyperkinetic movement disorder (dystonic
cerebral palsy or choreoathetosis), and impaired upgaze. In addition, repetitive tongue thrusting
and truncal hypotonia are common.
Hypoxic-ischemic injury in a term infant usually does not present with spastic diplegia.
Severe injury results in spastic quadriparesis and global intellectual impairment. Milder injuries,
which may occur in white matter and basal ganglia, tend to result in spastic cerebral palsy or
mixed spastic and dystonic cerebral palsy, but the symptoms do not typically affect legs more
than arms.
Stroke occurring in the third trimester tends to follow a large vessel vascular distribution and
most commonly is unilateral, resulting in hemiplegia (one side of face, arm, and/or leg). Most lead
intoxication in children is chronic and associated with behavioral and intellectual deficits, not
focal motor deficits in the legs.

Suggested reading:

Ashwal S, Russman BS, Blasco PA, et al. Practice parameter: diagnostic assessment of the
child with cerebral palsy: report of the Quality Standards Subcommittee of the American
Academy of Neurology and the Practice Committee of the Child Neurology Society. Neurology.
2004;62:851-863. Available at: http://www.neurology.org/cgi/content/full/62/6/851

Dodge NN. Cerebral palsy: medical aspects. Pediatr Clin North Am. 2008;55:1189-1207. DOI:
10.1016/j.pcl.2008.07.003. Abstract available at: http://www.ncbi.nlm.nih.gov/pubmed/18929060

Kernicterus in full-term infants--United States, 1994-1998. MMWR Morb Mortal Wkly Rep.
2001;50:491-494. Available at: http://www.cdc.gov/mmwr/preview/mmwrhtml/mm5023a4.htm

Papavasiliou AS. Management of motor problems in cerebral palsy: a critical update for the
clinician. Eur J Paediatr Neurol. 2009;13:387-396. Abstract available at:
http://www.ncbi.nlm.nih.gov/pubmed/18778959

Russman BS, Ashwal S. Evaluation of the child with cerebral palsy. Semin Pediatr Neurol.
2004;11:47-57. Abstract available at: http://www.ncbi.nlm.nih.gov/pubmed/15132253

Sanger TD. Is cerebral palsy a wastebasket diagnosis? J Child Neurol. 2008;23:726-728

Copyright 2010 by the American Academy of Pediatrics page 383


2011 PREP SA on CD-ROM

Critique: 118

Copyright 2010 by the American Academy of Pediatrics page 384


2011 PREP SA on CD-ROM

Question: 119

You have sent one of your patients, who has achondroplasia, for follow-up evaluation in the
genetics clinic. When discussing achondroplasia with the medical student after the evaluation, the
medical geneticist points out that approximately 80% of individuals who have this diagnosis have
spontaneous mutations in the fibroblast growth factor receptor 3 (FGFR3) gene.

Of the following, the mutation that causes achondroplasia MOST commonly is a(n)

A. autosomal recessive trait

B. contiguous gene deletion

C. deletion of FGFR3 only

D. duplication of FGFR3

E. single base-pair substitution

Copyright 2010 by the American Academy of Pediatrics page 385


2011 PREP SA on CD-ROM

Critique: 119 Preferred Response: E

Achondroplasia, the most common form of dwarfism, is an autosomal dominant condition. In


99% of affected individuals, it is caused by a single base-pair change (guanine-to-adenine or
guanine-to-cytosine) at nucleotide 1138 in the fibroblast growth factor receptor 3 gene
(FGFR3). Both of these changes result in the substitution of an arginine for a glycine in the
encoded protein, leading to increased activation of the growth factor receptor. Because the
normal action of the receptor is to reduce bone growth, its increased activity leads to shorter
bones and smaller stature.
It is important to note that DNA point mutations (substitutions, deletions, or insertions of a
single base-pair) can cause a wide range of outcomes, some of which are dramatic. Point
mutations can result in the production of nonfunctional proteins (nonsense mutations), different
amino acids than expected (missense mutations), and alterations in the amino acid or protein that
have no functional significance (silent mutations).
One of the best known examples of a point mutation causing severe disease is that seen in
sickle cell disease (SCD). SCD is caused by a single base-pair substitution in the beta-globin
gene, resulting in the production of a valine instead of a glutamic acid in the protein. The resultant
hemoglobin molecule is malformed, such that it cannot carry oxygen effectively.
Although contiguous gene deletions, as well as gene deletions and duplications, are
associated with numerous genetic disorders, they do not play a significant role in causing
achondroplasia.

Suggested reading:

Bender MA, Hobbs W. Sickle cell disease. GeneReviews. 2009. Available at:
http://www.ncbi.nlm.nih.gov/bookshelf/br.fcgi?book=gene&part=sickle

Francomano CA. Achondroplasia. GeneReviews. 2006. Available at:


http://www.ncbi.nlm.nih.gov/bookshelf/br.fcgi?book=gene&part=achondroplasia

Trotter TL, Hall JG; American Academy of Pediatrics Committee on Genetics. Health supervision
for children with achondroplasia. Pediatrics. 2005;116:771-783. DOI: 10.1542/peds.2005-1440.
Available at: http://pediatrics.aappublications.org/cgi/content/full/116/3/771

Copyright 2010 by the American Academy of Pediatrics page 386


2011 PREP SA on CD-ROM

Question: 120

You are seeing a 16-year-old girl who is new to your practice. Screening questions reveal that
she began sexual activity 1 year ago, has had unprotected sex with four partners in the last
year, and occasionally smokes marijuana. Her physical and gynecologic examination results are
normal today. She opts to use condoms as her sole birth control method. She asks when she
should return.

Of the following, the MOST appropriate visit schedule for testing for this girl is

A. a Papanicolaou test annually

B. Chlamydia trachomatis screening every 6 months

C. human immunodeficiency virus screening every 6 months

D. syphilis screening every 6 months

E. yearly serologic testing for herpes simplex virus

Copyright 2010 by the American Academy of Pediatrics page 387


2011 PREP SA on CD-ROM

Critique: 120 Preferred Response: B

All sexually active females younger than 25 years of age should be screened for
Chlamydia trachomatis at least yearly. Screening every 6 months is recommended for those in a
higher-risk category, usually defined by behavioral factors such as younger age, the number of
sex partners, new or more than one sex partner, partner at least 2 years older, substance use,
lack of condom use, a previous history of C trachomatis infection, homelessness, in detention,
and being paid for sex.
Papanicolaou testing should begin at age 21 years. The Centers for Disease Control and
Prevention recommend routine human immunodeficiency virus testing for all sexually active
adolescents; it should be repeated yearly for higher-risk youth. Syphilis testing is recommended
for pregnant adolescents or when other infections are diagnosed. There is no current
recommendation for routine screening for herpes simplex virus. Screening for Neisseria
gonorrhoeae infection also should be conducted in sexually active adolescents.

As a result of reviewing this information, do you intend to make a change in practice


to provide better patient care?
Yes No

Suggested reading:

American Academy of Pediatrics. Chlamydia trachomatis. In: Pickering LK, Baker CJ, Kimberlin
DW, Long SS, eds. Red Book: 2009 Report of the Committee on Infectious Diseases. 28th ed.
Elk Grove Village, Ill: American Academy of Pediatrics; 2009:25-269

Cervical cytology screening. ACOG Practice Bulletin No. 109. American College of Obstetricians
and Gynecologists. Obstet Gynecol. 2009;114:1409-1420. Available at:
http://journals.lww.com/greenjournal/documents/PB109_Cervical_Cytology_Screening.pdf

Kohl KS, Markowitz LE, Koumans EH. Developments in the screening for Chlamydia trachomatis:
a review. Obstet Gynecol Clin North Am. 2003;30:637-658

Miller CA, Shafer M-AB. Chlamydia trachomatis. In: Neinstein LS, Gordon CM, Katzman D, Rosen
DS, Woods ER, eds. Adolescent Health Care: A Practical Guide. 5th ed. Philadelphia Pa:
Lippincott Williams & Wilkins, a Wolters Kluwer business; 2008:805-818

National Chlamydia Coalition web site. Available at: http://www.ncc.prevent.org/

Shrier LA. Bacterial sexually transmitted infections: gonorrhea, chlamydia, pelvic inflammatory
disease, and syphilis: In: Emans SJH, Laufer MR, Goldstein DP, eds. Pediatric and Adolescent
Gynecology. 5th ed. Philadelphia, Pa: Lippincott Williams & Wilkins; 2005:565-614

Copyright 2010 by the American Academy of Pediatrics page 388


2011 PREP SA on CD-ROM

Question: 121

You are talking with the mother of one of your clinic patients, an 18-year-old boy who is ready to
be discharged after a 30-day hospitalization for acute respiratory distress syndrome (ARDS).
She reports that he is very weak and has lost almost 15% of his body weight. She is concerned
about how much recovery he will be able to make.

Of the following, the MOST accurate statement regarding recovery following hospital discharge
for most ARDS patients is that

A. their pulmonary function is usually normal by 12 months

B. their recovery is unaffected by the degree of their illness

C. they recover completely by 1 month

D. they regain their strength by 3 months

E. they require supplemental oxygen for 12 months

Copyright 2010 by the American Academy of Pediatrics page 389


2011 PREP SA on CD-ROM

Critique: 121 Preferred Response: A

Acute respiratory distress syndrome (ARDS) is a significant cause of morbidity and


mortality in critically ill pediatric patients and can result in significant long-term health effects.
Although the exact incidence of ARDS in the pediatric population has been difficult to quantify, it
is estimated to account for approximately 2% to 4% of admissions, 8% of total patient days, and
33% of deaths in the pediatric intensive care unit. Complete resolution may take 6 to 12 months,
with most patients reaching normal lung status. A small number of patients can develop lifelong
restrictive lung disease, lung cysts, or decreased exercise tolerance.
A recent longitudinal study of adult ARDS survivors showed that they had functional
limitations 12 months after discharge that were believed to be due to weakness and fatigue.
Survivors had prolonged intensive care unit stays (median, 25 days) and significant weight loss
(18% of their baseline body weight) during their critical illness. Those patients who had rapid
resolution of lung injury and multiorgan dysfunction had better functional outcomes at 12 months.
In this group of patients, lung volume and spirometric measurements were normal by 6 months,
with no patients requiring supplemental oxygen at 12 months.

Suggested reading:

Frankel LR. Respiratory distress and failure. In: Kliegman RM, Behrman RE, Jenson HB, Stanton
BF, eds. Nelson Textbook of Pediatrics. 18th ed. Philadelphia, Pa: Saunders Elsevier; 2007:421-
423

Herridge MS, Cheung AM, Tansey CM, et al. One-year outcomes in survivors of acute respiratory
distress syndrome. N Engl J Med. 2003;348:683-693. Available at:
http://content.nejm.org/cgi/content/full/348/8/683

Vish M, Shanley TP. Acute lung injury and acute respiratory distress syndrome. In: Wheeler DS,
Wong HR, Shanley TP, eds. Pediatric Critical Care Medicine: Basic Science and Clinical
Evidence. New York, NY: Springer-Verlag; 2007:395-411

Copyright 2010 by the American Academy of Pediatrics page 390


2011 PREP SA on CD-ROM

Question: 122

You are seeing a 6-year-old girl for a health supervision visit. On physical examination, you note
Sexual Maturity Rating (SMR) 3 pubic hair and SMR 1 breast tissue. You noted no pubic hair last
year. She has had a growth spurt in the past 2 years and is presently at the 75th percentile for
height (Item Q122). Her weight is at the 50th percentile for age. Her blood pressure is 90/60 mm
Hg. The remainder of her evaluation is within normal parameters except for possible
clitoromegaly. The radiologist interprets a bone age radiograph as 8 years.

Of the following, the MOST helpful diagnostic laboratory blood test is measurement of

A. androstenedione

B. dehydroepiandrosterone sulfate

C. electrolytes

D. 17-hydroxyprogesterone

E. testosterone

Copyright 2010 by the American Academy of Pediatrics page 391


2011 PREP SA on CD-ROM

Question: 122

(Courtesy of L Levitsky)

Copyright 2010 by the American Academy of Pediatrics page 392


2011 PREP SA on CD-ROM

Critique: 122 Preferred Response: D

The girl described in the vignette has an advanced bone age, rapid growth rate over 2
years, pubic hair, and clitoromegaly. The most likely explanation for these findings in a girl is
congenital adrenal hyperplasia (CAH). Because the degree and the timing of onset of virilization
in children who have CAH depends upon the degree of enzyme activity of the most severely
affected of the two inherited cyp21 genes, there is a spectrum of presentations in this disorder.
The presentations can range from almost complete enzyme deficiency resulting in
masculinization of female fetuses and the rapid development of a salt-losing crisis to very mild
virilization of adult females, which may be confused with polycystic ovary syndrome in adult
women. Within this spectrum, children have been classified as having classic salt-losing CAH,
non-salt-losing, and nonclassic CAH identified at various ages to adulthood.
The incidence of classic CAH in the United States is about 1 in 14,000, but the incidence of
later-onset forms is reported to be 1 in 100 to 1 in 1,000 among whites, in whom it is more
common than other racial groups. Classic CAH most commonly results from 21-hydroxylase
deficiency (95%). More than 70% of children present with a salt-losing crisis within the first
several weeks after birth. Girls who have this condition exhibit masculinization of genital
development at birth (Item C122). Some children can produce enough mineralocorticoid
(aldosterone) (non-salt losers) and, therefore, are identified only because of masculinization of
genital development in baby girls and isosexual precocity in boys. Children who have the classic
form of CAH usually are identified via prenatal screening for 17-hydroxyprogesterone.
The diagnosis of CAH resulting from 21-hydroxylase deficiency (the most common type) is
based on the finding of an elevated 17-hydroxyprogesterone concentration in response to an
adrenocorticotrophic hormone stimulus or in a first morning specimen, when adrenal steroid
release is at its highest. Dehydroepiandrosterone sulfate concentrations are elevated to pubertal
ranges in CAH, but such findings also are seen in children who have premature pubarche.
Although androstenedione may be elevated in children who have CAH, such a finding is not
diagnostic. Because most children have greater elevations in 17-hydroxyprogesterone, the end
product just before the enzymatic block in adrenal biosynthesis, this is taken as the gold
standard for diagnosing 21-hydroxylase deficiency, although evaluation of other steroid
precursors or genetic analysis is necessary for confirmation. Serum electrolyte values are
abnormal in decompensated classic CAH associated with aldosterone deficiency. In this
situation, low serum sodium and elevated serum potassium values might be expected, but
electrolyte abnormalities are not found in late-onset CAH. The testosterone value is somewhat
elevated in late-onset CAH, but this finding is not diagnostic.

Suggested reading:

Antal Z, Zhou P. Congenital adrenal hyperplasia: diagnosis, evaluation and management. Pediatr
Rev. 2009;30:e49-e57. DOI: 10.1542/10.1542/pir.30-7-e49. Available at:
http://pedsinreview.aappublications.org/cgi/content/full/30/7/e49

Armengaud JB, Charkaluk ML, Trivin C, et al. Precocious pubarche: distinguishing late-onset
congenital adrenal hyperplasia from premature adrenarche. J Clin Endocrinol Metab.

Copyright 2010 by the American Academy of Pediatrics page 393


2011 PREP SA on CD-ROM

2009;94:2835-2840. Abstract available at: http://www.ncbi.nlm.nih.gov/pubmed/19454583

Niemann LK. Genetics and clinical presentation of nonclassic (late-onset) congenital adrenal
hyperplasia due to 21-hydroxylase deficiency. UpToDate Online 17.3. 2009. Available at:
http://www.uptodateonline.com/online/content/topic.do?topicKey=adrenal/20734&selectedTitle=9
%7E86&source=search_result

Nimkarn S, New MI. 21-hydroxylase-deficient congenital adrenal hyperplasia. GeneReviews.


2009. Available at: http://www.ncbi.nlm.nih.gov/bookshelf/br.fcgi?book=gene&part=cah

Copyright 2010 by the American Academy of Pediatrics page 394


2011 PREP SA on CD-ROM

Critique: 122

(Reprinted with permission from Antal Z, Zhou P. Congenital adrenal hyperplasia: diagnosis,
evaluation and management. Pediatr Rev. 2009;30:e49-e57)
Virilization of the external genitalia of a female infant affected with congenital adrenal hyperplasia
due to 21-hydroxylase deficiency. Note complete fusion of the labioscrotal folds and a male penile
appearance without testes.

Copyright 2010 by the American Academy of Pediatrics page 395


2011 PREP SA on CD-ROM

Question: 123

A 13-year-old boy who has an autism spectrum disorder is displaying severely aggressive
behavior to his parents and teachers. When frustrated, he has tantrums, and when upset, he hits
himself and throws objects. This behavior has continued even after intensive behavioral and
educational interventions. His parents wish to begin medication to help handle him safely at home.

Of the following, the MOST appropriate medication with which to begin a trial is

A. atypical antipsychotic

B. lithium

C. melatonin

D. serotonin reuptake inhibitor

E. stimulant medication

Copyright 2010 by the American Academy of Pediatrics page 396


2011 PREP SA on CD-ROM

Critique: 123 Preferred Response: A

The significant behaviors exhibited by the boy described in the vignette include explosive
outbursts, aggression, and self-injurious behaviors. Behavioral and educational approaches are
the starting point for intervention. Because the childs behavior places him at risk for harming
himself or others, pharmacologic therapy should be initiated. Medications that target irritability,
aggression, explosive outbursts, and self-injurious behaviors include atypical antipsychotics and
anticonvulsant mood stabilizers.
An atypical antipsychotic is an appropriate choice for this boy. The most common adverse
effects noted with these medications are weight gain, increase in appetite, and fatigue.
Anticonvulsant mood stabilizers also may be used to treat target behaviors as well as bipolar
symptoms and repetitive behaviors. Alpha-2 agonists such as long-acting guanfacine may be
used to target attention-deficit/hyperactivity disorder (ADHD) symptoms, aggression, and sleep
dysfunction.
Stimulant medications and selective norepinephrine reuptake inhibitors are used for ADHD
symptoms of hyperactivity, impulsivity, inattention, and distractibility. They are not used to target
primarily aggressive behaviors or self-injurious behaviors. Lithium is used for bipolar disorder
and has a low therapeutic-to-toxic ratio. Selective serotonin reuptake inhibitors are used to treat
repetitive behaviors, anxiety, and depressive symptoms. Many selective serotonin reuptake
inhibitors are used off-label. Melatonin is a naturally occurring hormone that is not regulated by
the United States Food and Drug Administration, but it is available over the counter in the United
States. Several studies have indicated its usefulness in promoting sleep but not for aggression
or self-injurious behaviors.

Suggested reading:

American Academy of Pediatrics Autism Expert Panel. Autism: Caring for Children with Autism
Spectrum Disorders: A Resource Toolkit for Clinicians. Elk Grove, Ill: American Academy of
Pediatrics; 2007

Leskovec TJ, Rowles BM, Findling RL. Pharmacological treatment options for autism spectrum
disorders in children and adolescents. Harv Rev Psychiatry. 2008;16:97-112. DOI:
10.1080/10673220802075852. Abstract available at:
http://www.ncbi.nlm.nih.gov/pubmed/18415882

Meyers SM. Management of autism spectrum disorders in primary care. Pediatr Ann. 2009;38:42-
49

Myers SM, Johnson CP and the Council on Children with Disabilities. Management of children
with autism spectrum disorders. Pediatrics. 2007;120:1162-1182. DOI: 10.1542/peds.2007-
2362. Available at: http://pediatrics.aappublications.org/cgi/content/full/120/5/1162

Copyright 2010 by the American Academy of Pediatrics page 397


2011 PREP SA on CD-ROM

Question: 124

A 17-year-old boy comes to your office seeking international health information. He will be
participating in a 3-week charity mission to El Salvador in 2 months. He has read about malaria
and asks your advice regarding preventive medication.

Of the following, the MOST appropriate choice for malaria prophylaxis in this situation is

A. atovaquone-proguanil

B. chloroquine

C. doxycycline

D. mefloquine

E. primaquine

Copyright 2010 by the American Academy of Pediatrics page 398


2011 PREP SA on CD-ROM

Critique: 124 Preferred Response: B

Malaria prophylaxis is recommended when traveling to endemic areas of the world, which
include much of Southeast Asia, Africa, the Middle East, and Central and South America. In areas
of Central and South America, where pathogens remain sensitive to chloroquine, it is the drug of
choice for prophylaxis. The drug is administered weekly, beginning 1 to 2 weeks before travel
and continuing through the period of travel to an endemic area and for 4 weeks after return.
Chloroquine is generally well tolerated, but reported adverse effects include gastrointestinal
upset, headache, dizziness, blurred vision, insomnia, and pruritus. Chloroquine also may
exacerbate psoriasis.
Atovaquone-proguanil, doxycycline, and mefloquine are indicated for the treatment or
prevention of malaria in areas where chloroquine-resistant strains have emerged. These drugs
also may be used for chloroquine-sensitive malaria in the individual who cannot tolerate
chloroquine. Primaquine is used for treating the exoerythrocytic stage of nonfalciparum malaria.
It is not used in routine malaria prophylaxis.
Travelers also should be aware of methods to prevent mosquito bites in an effort to
minimize risk of acquiring malaria and other vector-borne infections. Up-to-date information on
malaria and other travel-related concerns can be found on the Centers for Disease Control and
Prevention web site: http://www.cdc.gov

Suggested reading:

American Academy of Pediatrics. Malaria. In: Pickering LK, Baker CJ, Kimberlin DW, Long SS,
eds. Red Book: 2009 Report of the Committee on Infectious Diseases. 28th ed. Elk Grove
Village, Ill: American Academy of Pediatrics; 2009: 438-444

Arguin PM, Steele SF. The pre-travel consultation: malaria. In: Travelers Health Yellow Book.
Atlanta, Ga: Centers for Disease Control and Prevention. 2009. Available at:
http://wwwnc.cdc.gov/travel/yellowbook/2010/chapter-2/malaria.aspx

Copyright 2010 by the American Academy of Pediatrics page 399


2011 PREP SA on CD-ROM

Question: 125

A 6-month-old boy has had rhinorrhea and cough for 3 days. Today he presents with wheezing
and increased work of breathing. Physical examination reveals a temperature of 38.0C, heart
rate of 120 beats/minute, respiratory rate of 40 breaths/min, oxygen saturation of 94% in room air,
mild subcostal retractions, and diffuse expiratory wheezing. Chest radiography shows
symmetrically hyperinflated lungs with bilateral interstitial markings but no focal consolidation (Item
Q125). The result of a rapid direct fluorescent antibody assay for respiratory syncytial virus is
negative.

Of the following, the MOST likely cause of the patients symptoms is

A. cytomegalovirus

B. Haemophilus influenzae

C. human metapneumovirus

D. Mycoplasma pneumoniae

E. respiratory syncytial virus

Copyright 2010 by the American Academy of Pediatrics page 400


2011 PREP SA on CD-ROM

Question: 125

(Courtesy of B Wood)
Chest radiograph demonstrating hyperinflation and prominent interstitial markings, as described
for the infant in the vignette.

Copyright 2010 by the American Academy of Pediatrics page 401


2011 PREP SA on CD-ROM

Critique: 125 Preferred Response: C

The 6-month-old boy described in the vignette has bronchiolitis. Causes of bronchiolitis in
infants and children include respiratory syncytial virus (RSV), rhinovirus, parainfluenza viruses,
influenza, and human metapneumovirus (hMPV). hMPV has been shown to cause acute
respiratory tract illness such as pneumonia, croup, and upper respiratory tract infections with
concomitant acute otitis media in infants, older children, and adults. It can be clinically
indistinguishable from bronchiolitis caused by RSV. Most children have at least one infection due
to hMPV by the age of 5 years. Concurrent infection with other viruses can occur. Otherwise
healthy children tend to have mild-to-moderate illness, but those who have immunodeficiency
may experience more severe illness that requires hospitalization.
Humans are the only source of hMPV infection. In temperate climates, infection usually
overlaps with the RSV season and occurs during late winter and early spring. In healthy infants,
viral shedding lasts for 1 to 2 weeks but may be prolonged in severely immunocompromised
individuals.
hMPV infection can be diagnosed by commercially available rapid diagnostic
immunofluorescent assays. Reverse transcriptase polymerase chain reaction testing is available
in research and some clinical laboratories. Current routine viral culture techniques may grow the
virus, but prolonged incubation periods are required. Measuring acute and convalescent serum
antibody titers may be useful for the first episode of infection. Treatment of hMPV infection is
supportive.
Because commercially available rapid RSV assays that use antigen capture technology
generally are more than 90% sensitive and specific, the negative findings on such assay for this
patient make hMPV more likely than RSV bronchiolitis. Confirmation of RSV infection by culture
can take from 4 days to 2 weeks. Cytomegalovirus infection in young children usually results in
a nonspecific febrile illness or in asymptomatic infection and does not cause bronchiolitis.
Infection due to Haemophilus influenzae, usually not type b, can cause otitis media, sinusitis,
and pneumonia. Acute bronchitis, upper respiratory tract infections, and pneumonia due to
Mycoplasma pneumoniae occur in school-age and older children.

Suggested reading:

American Academy of Pediatrics. Human metapneumovirus. In: Pickering LK, Baker CJ, Kimberlin
DW, Long SS, eds. Red Book: 2009 Report of the Committee on Infectious Diseases. 28th ed.
Elk Grove Village, Ill: American Academy of Pediatrics; 2009:463-464

Cherry JD. Human metapneumovirus. In: Feigin RD, Cherry JD, Demmler-Harrison GJ, Kaplan SL,
eds. Feigin & Cherrys Textbook of Pediatric Infectious Diseases. 6th ed. Philadelphia, Pa:
Saunders Elsevier; 2009:2487-2493

Williams JV, Wang CK, Yang CF, et al. The role of human metapneumovirus in upper respiratory
tract infections in children: a 20-year experience. J Infect Dis. 2006;193:387-395. DOI:
10.1086/499274. Available at: http://www.journals.uchicago.edu/doi/full/10.1086/499274

Copyright 2010 by the American Academy of Pediatrics page 402


2011 PREP SA on CD-ROM

Question: 126

A 1-month-old male infant presents with fever and poor feeding. His prenatal history is
uneventful. He has no gross hematuria or foul-smelling urine. On physical examination, his
temperature is 39.0C, heart rate is 120 beats/min, respiratory rate is 30 breaths/min, and blood
pressure is 98/60 mm Hg. He has nonfocal findings on neurologic evaluation.

Laboratory results include:


White blood cell (WBC) count,16.5x10 3/mcL (16.5x109/L) (70% neutrophils, 25%
lymphocytes, 5% monocytes)
Hemoglobin, 10.0 g/dL (100 g/L)
Platelet count, 410x10 3/mcL (410x109/L)
Cerebrospinal fluid, normal
Electrolytes, normal
Blood urea nitrogen, 10 mg/dL (3.57 mmol/L)
Creatinine, 0.3 mg/dL (26.5 mcmol/L)

Urinalysis shows:
Specific gravity, 1.010
pH, 6
2+ blood
Negative protein, leukocyte esterase, and nitrite
5 to 10 red blood cells/high-power field (hpf)
Fewer than 5 WBCs/hpf

Findings on renal/bladder ultrasonography are unremarkable.

Of the following, the MOST likely cause for the infants urinalysis findings is

A. acute glomerulonephritis

B. fever

C. obstructive uropathy

D. renal vein thrombosis

E. urethral injury

Copyright 2010 by the American Academy of Pediatrics page 403


2011 PREP SA on CD-ROM

Critique: 126 Preferred Response: E

As part of a "rule out sepsis" evaluation, the 1-month-old febrile infant described in the
vignette has undergone evaluation for serious bacterial infection. Because a urinary tract
infection needs to be considered in all infants who present with fever, a sterile urine sample for
culture has been obtained by catheterization. Urinalysis reveals 2+ blood on dipstick and 5 to 10
red blood cells/high-power field. This combination of findings indicates hematuria (excluding
myoglobinuria or hemoglobinuria). Because the urine collections performed in infants and non-
toilet-trained children should be obtained via catheterization of the urinary bladder, the clinician
needs to be cognizant of the potential complications of this procedure. In addition to local pain
and potentially emotional trauma from the procedure, urethral trauma with bleeding and urinary
tract infection can occur. More rarely, urethral strictures, urethral false passage with or without
fistula formation, and knotting of the catheter within the bladder have been reported. The infant
in the vignette most likely suffered urethral trauma during catheterization.
Renal vein thrombosis is characterized by enlarged kidneys, thrombocytopenia, gross
hematuria, and reduced urine output. This infants normal creatinine value, platelet count, and
renal ultrasonography reading make this diagnosis unlikely. Acute glomerulonephritis is very
uncommon in infants, and the absence of proteinuria, azotemia, edema, and hypertension in this
infant make this very unlikely. Obstructive uropathy (most likely due to posterior urethral valves)
occurs in 1 in 5,000 males and is characterized by bilateral hydronephrosis with or without a
thickened/trabeculated bladder on ultrasonography. Fever has been associated with proteinuria
in children but is not a cause for microscopic hematuria.

Suggested reading:

Farhat W, McLorie G. Urethral syndromes in children. Pediatr Rev. 2001;22:17-21. DOI:


10.1542/10.1542/pir.22-1-17. Available at:
http://pedsinreview.aappublications.org/cgi/content/full/22/1/17

Foster H, Ritchey M, Bloom D. Adventitious knots in urethral catheters: report of 5 cases. J Urol.
1992;148:1496-1498. Abstract available at: http://www.ncbi.nlm.nih.gov/pubmed/1433556

Robson WL, Leung AK, Thomason MA. Catheterization of the bladder in infants and children.
Clin Pediatr. 2006;45:795-800. Abstract available at:
http://www.ncbi.nlm.nih.gov/pubmed/17041166

Copyright 2010 by the American Academy of Pediatrics page 404


2011 PREP SA on CD-ROM

Question: 127

A 4-year-old boy who has a history of atopic dermatitis has had intermittent wheezing and
coughing for the past 6 months. His parents report that viral upper respiratory tract infections,
vigorous activity, and visits to his grandmother who smokes and has a cat trigger his symptoms.
Most nights he awakes with coughing, and he missed 3 days of school last month due to
shortness of breath. On physical examination, the healthy-appearing boy has normal vital signs,
appropriate height and weight, normal findings on cardiac and pulmonary evaluation, and no
digital clubbing.

Of the following, the MOST appropriate initial therapy is an

A. inhaled anticholinergic

B. inhaled corticosteroid

C. inhaled short-acting beta2 agonist

D. oral antihistamine

E. oral leukotriene

Copyright 2010 by the American Academy of Pediatrics page 405


2011 PREP SA on CD-ROM

Critique: 127 Preferred Response: B

For the child in the vignette, who has symptoms consistent with moderate persistent
asthma, an inhaled corticosteroid is the initial medication of choice. Level A evidence has
demonstrated that inhaled corticosteroids reduce asthma symptoms, decrease asthma
exacerbations, and improve lung function. Inhaled corticosteroids also are considered controller
or preventive medications for asthma, in contrast with rescue medications such as inhaled
anticholinergic and short-acting beta2 agonist (SABA) medications.
An inhaled anticholinergic or SABA would be prescribed for this boy, although based on the
frequency of his symptoms, neither would be initial therapy. Patients who have intermittent
asthma (defined as daytime symptoms less than twice weekly or nighttime symptoms less than
twice monthly) could be managed initially with one of these medications.
Oral leukotrienes are approved for asthma in children 6 months of age and older, but current
Expert Panel Report-3 Asthma Guidelines recommend the addition of this medication only when
medium-dose inhaled corticosteroids have failed to control asthma. Oral antihistamines may
improve asthma when allergic triggers are identified, but they are considered an adjunctive
therapy for asthma rather than initial therapy.
Asthma is one the most common causes of recurrent wheezing in children, but other
causes include congenital anomalies, bronchiolitis, foreign body aspiration, and
gastroesophageal reflux.

Suggested reading:

Adams NP, Bestall JC, Lasserson TJ, Jones P, Cates CJ. Fluticasone versus placebo for chronic
asthma in adults and children. Cochrane Database Syst Rev. 2008;4:CD003135. DOI:
10.1002/14651858.CD003135.pub4. Available at:
http://www.mrw.interscience.wiley.com/cochrane/clsysrev/articles/CD003135/frame.html

National Asthma Education and Prevention Program. Expert Panel Report 3 (EPRS): Guidelines
for the Diagnosis and Management of Asthma. Full Report 2007. NIH Publication 07-4051.
Bethesda, Md: National Heart, Lung, and Blood Institute; 2007. Available at:
http://www.nhlbi.nih.gov/guidelines/asthma/asthgdln.htm

Wood PR, Hill VL. Practical management of asthma. Pediatr Rev. 2009;30:375-385. DOI:
10.1542/10.1542/pir.30-10-375. Available at:
http://pedsinreview.aappublications.org/cgi/content/full/30/10/375

Copyright 2010 by the American Academy of Pediatrics page 406


2011 PREP SA on CD-ROM

Question: 128

After being struck by a car while riding her bicycle, a 5-year-old girl is transported to the
emergency department by paramedics, who report that she was awake and crying at the scene,
complaining of right leg pain. On physical examination, she is awake and her heart rate is 160
beats/min, respiratory rate is 28 breaths/min, and blood pressure is 78/40 mm Hg. Her lungs are
clear, her abdomen is mildly distended and diffusely tender, and she has an obvious deformity of
her right thigh. After two 20-mL/kg boluses of 0.9% saline, her heart rate is 130 beats/min and
blood pressure is 100/60 mm Hg.

Of the following, the radiographic test that is MOST likely to identify the cause of her initial
hypotension is

A. abdominal computed tomography scan

B. cervical spine radiograph

C. head computed tomography scan

D. retrograde urethrography

E. right femur radiograph

Copyright 2010 by the American Academy of Pediatrics page 407


2011 PREP SA on CD-ROM

Critique: 128 Preferred Response: A

Blunt abdominal trauma in children is second only to brain injury as a cause of injury-related
mortality. The most commonly injured intra-abdominal organs are the liver and spleen, accounting
for 60% of childhood blunt abdominal injuries. Intra-abdominal trauma must be suspected in
pediatric patients who have blunt trauma and hypotension in the absence of obvious
hemorrhage if the primary survey does not reveal evidence of tension pneumothorax,
hemothorax, or cardiac tamponade.
A number of modalities are employed to facilitate the identification of intra-abdominal injuries.
Because of high diagnostic sensitivity and specificity, abdominal and pelvic computed
tomography (CT) scans are the radiographic studies of choice in most trauma centers (Item
C128A) and (Item C128B). Most centers perform these studies using intravenous contrast only
because of the risk for oral contrast aspiration and the relative lack of additional diagnostic
accuracy provided by the addition of oral contrast during the acute evaluation.
The quantity of blood lost into the subdural or epidural spaces in the setting of head trauma
or into the thigh following a femur fracture almost never is significant enough to cause
hemorrhagic shock. For this reason, a head CT or femur radiograph might reveal injuries but not
those that would cause hypotension. A cervical spine injury would not cause hypotension in the
absence of other neurologic deficits. Retrograde urethrography is used to detect urethral
injuries, which also are not likely to cause hypotension.
Abdominal ultrasonography is being used with increasing frequency in pediatric trauma
evaluations. The focused assessment with sonography in trauma (FAST) examination has
moderate sensitivity for identifying the presence of free fluid in the peritoneum that may be
indicative of intra-abdominal injury. Although some suggest that such a finding can help identify
patients who may require immediate laparotomy, most pediatric intra-abdominal injuries in stable
patients are managed nonoperatively. Further, because FAST may not detect small peritoneal
fluid collections, and there is a need to define any intra-abdominal injuries, CT remains the
diagnostic procedure of choice.
With the exception of identifying free intra-abdominal air, the lack of sensitivity and
specificity of plain abdominal radiographs limit their utility.

Suggested reading:

Holmes JF, Sokolove PE, Brant WE, et al. Identification of children with intra-abdominal injuries
after blunt trauma. Ann Emerg Med. 2002;39:500509. DOI: 10.1067/mem.2002.122900.
Abstract available at: http://www.ncbi.nlm.nih.gov/pubmed/11973557

Reyes Mendez D. Overview of blunt abdominal trauma in children. UpToDate Online 17.3. 2008.
Available at:
http://www.utdol.com/online/content/topic.do?topicKey=ped_trau/2434&selectedTitle=1~48&sou
rce=search_result

Richards JR, Knopf NA, Wang L, McGahan JP. Blunt abdominal trauma in children: evaluation
with emergency US. Radiology. 2002;222:749-754. Available at:

Copyright 2010 by the American Academy of Pediatrics page 408


2011 PREP SA on CD-ROM

http://radiology.rsna.org/content/222/3/749.long

Saxena AK, Nance ML, Lutz N, Stafford PW. Abdominal trauma. eMedicine Specialties,
Pediatrics: Surgery, General Surgery. 2008. Available at:
http://emedicine.medscape.com/article/940726-overview

Wegner S, Colletti JE, Van Wie D. Pediatric blunt abdominal trauma. Pediatr Clin North Am.
2006;53:243256. DOI: 10.1016/j.pcl.2006.02.002. Abstract available at:
http://www.ncbi.nlm.nih.gov/pubmed/16574524

Copyright 2010 by the American Academy of Pediatrics page 409


2011 PREP SA on CD-ROM

Critique: 128

(Courtesy of M Wright)
Computed tomography scan demonstrating a liver laceration (arrows).

Copyright 2010 by the American Academy of Pediatrics page 410


2011 PREP SA on CD-ROM

Critique: 128

(Courtesy of M Wright)
Computed tomography scan demonstrating a pancreatic laceration (arrows).

Copyright 2010 by the American Academy of Pediatrics page 411


2011 PREP SA on CD-ROM

Question: 129

A 14-year-old boy in the 9th grade presents with a complaint of abdominal pain. He describes the
pain as being in both the right and left upper quadrants and explains that it occurs at any time of
the day or night. He also complains of recent left hip and leg pain. Physical examination
demonstrates a cooperative, alert adolescent whose height is 137 cm and weight is 28 kg. A firm
liver edge is palpable 4 cm below the right costal margin, and the spleen is palpable 6 cm below
the left costal margin. Initial laboratory data include:

Hemoglobin, 10.4 g/dL (104 g/L)


White blood cell count, 2.5x10 3/mcL (2.5x109/L)
Platelet count, 75x10 3/mcL (75x109/L)
Aspartate aminotransferase, 110 units/L
Alanine aminotransferase, 85 units/L
Alkaline phosphatase, 650 units/L

Of the following, the study that is MOST likely to provide additional diagnostic information is

A. Epstein-Barr virus serology

B. glucocerebrosidase assay

C. serum amino acids measurement

D. succinylacetone assay

E. urine organic acids measurement

Copyright 2010 by the American Academy of Pediatrics page 412


2011 PREP SA on CD-ROM

Critique: 129 Preferred Response: B

Depending upon the age at presentation, simultaneous hepatomegaly and splenomegaly in


the pediatric patient may be the consequence of hematologic, infectious, metabolic, neoplastic,
obstructive, or storage disorders. The organomegaly associated with abdominal complaints,
bone/joint pain, and slow growth (height and weight <5th percentile) described for the 14-year-
old boy in the vignette indicate chronic illness. Screening laboratory results demonstrate hepatic
involvement, and pancytopenia suggests the possibility of either bone marrow failure or
hypersplenism. The most likely unifying diagnosis, based on these findings, is Gaucher disease,
a multisystem lipidosis characterized by anemia, thrombocytopenia, visceromegaly, growth
failure, and musculoskeletal abnormalities. This diagnosis may be confirmed by demonstrating a
deficiency in the activity of blood leukocyte glucocerebrosidase (acid beta-glucosidase).
Depending on the nature and severity of liver disease, the spleen may be enlarged because
of infiltrative, infectious, and storage disorders that directly affect both the liver and spleen or
may result from portal blood flow obstruction to a congested/fibrotic liver. In the neonate, the
initial evaluation of hepatosplenomegaly should focus on the presence or absence of
hyperbilirubinemia. Direct hyperbilirubinemia indicates cholestasis and, among other disorders of
hepatobiliary function, suggests the possibility of biliary atresia. Prompt assessment must
exclude this diagnosis. Visceromegaly associated with elevations in the indirect bilirubin fraction
suggests congenital infection (including human immunodeficiency virus) or extramedullary
hematopoiesis, although hyperbilirubinemia is not a universal finding in these disorders. In the
absence of hyperbilirubinemia, infiltrative and vascular obstructive causes must be evaluated.
In the older child and adolescent patient, the diagnostic approach to hepatosplenomegaly
also begins with screening laboratory studies, including a complete blood count, comprehensive
metabolic profile, and fractionated bilirubin assessment. An elevated direct bilirubin value
suggests chronic cholestatic liver disease with portal hypertension and passive splenic
congestion. Physical examination may demonstrate a firm, nodular liver edge as evidence of
cirrhosis. Chronic viral hepatitis B and C, autoimmune hepatitis, primary sclerosing cholangitis
(often in association with ulcerative colitis), and Wilson disease all may progress to hepatic
fibrosis and portal venous obstruction. In these disorders, jaundice frequently is the first clinical
sign that prompts a diagnostic evaluation.
Chronic hematologic diseases, including the hemolytic anemias, often are associated with
hepatosplenomegaly, with or without hyperbilirubinemia. Splenomegaly is usually predominant,
and laboratory studies should include careful examination of a blood smear and often a bone
marrow aspiration. These studies also aid in evaluating the patient for infiltrative processes
caused by both malignant and nonmalignant conditions. Parasitic infections and infection with
Epstein-Barr virus also are characterized by hepatosplenomegaly in the presence or absence of
elevated bilirubin values. However, the growth retardation, presence of musculoskeletal
symptoms, and absence of a fever history make an infectious cause unlikely for the boy in the
vignette, pointing more directly to a chronic disease state.
Organ enlargement is a common feature of many storage diseases, (eg, sphingolipidoses,
mucopolysaccharidoses). Other inborn errors of metabolism, including the organic acidemias
and the amino acidopathies, also may be associated with organomegaly. However, these
illnesses become symptomatic during infancy. Accordingly, neither amino acid assessment nor a

Copyright 2010 by the American Academy of Pediatrics page 413


2011 PREP SA on CD-ROM

urine organic acid profile (including a succinylacetone measurement to evaluate for tyrosinemia)
is likely to establish a diagnosis in this adolescent.
A careful review of this patients history, physical examination, and screening laboratory
results demonstrates several of the major clinical features of type 1 Gaucher disease (non-
neuropathic, "adult-type"), characterized by the accumulation of glucocerebroside in various
tissues. The combination of hepatosplenomegaly and bone pain is a unique feature of this
disorder, which is the most common glycolipid storage disease. Hematologic abnormalities
(pancytopenia), organomegaly, musculoskeletal abnormalities (avascular necrosis of the hip,
infiltration of the bone marrow, fractures), and growth retardation are typical findings. Affected
patients usually present by the end of the first decade of life, and the disorder is particularly
frequent in Ashkenazi Jews, with an incidence of 1 in 855 births (the overall incidence is 1 in
50,000 to 100,000 births). Children who have type 1 disease comprise 99% of affected patients,
and they usually are in good health for the first few postnatal years. Progressive engorgement
of macrophages over time leads to organomegaly and bony infiltration.

Suggested reading:

Drelichman G, Ponce E, Basack N, et al. Clinical consequences of interrupting enzyme


replacement therapy in children with type 1 Gaucher disease. J Pediatr. 2007;151:197-201. DOI:
10.1016/j.jpeds.2007.02.057. Abstract available at:
http://www.ncbi.nlm.nih.gov/pubmed/17643778

Grabowski GA. Recent clinical progress in Gaucher disease.

Curr Opin Pediatr. 2005;17:519-524. Abstract available at:


http://www.ncbi.nlm.nih.gov/pubmed/16012266

Larsen EC, Connolly SA, Rosenberg AE. Case 20-2003A nine-year-old girl with
hepatosplenomegaly and pain in the thigh. N Engl J Med. 2003;348:2669-2677. Extract available
at: http://content.nejm.org/cgi/content/extract/348/26/2669

MacElwee M. Index of suspicion: case 1. Pediatr Rev. 2001;22:388-393. DOI:


10.1542/10.1542/pir.22-11-388. Available at:
http://pedsinreview.aappublications.org/cgi/content/full/22/11/388

Wolf AD, Lavine JE. Hepatomegaly in neonates and children. Pediatr Rev. 2000;21:303-310. DOI:
10.1542/10.1542/pir.21-9-303. Available at:
http://pedsinreview.aappublications.org/cgi/content/full/21/9/303

Copyright 2010 by the American Academy of Pediatrics page 414


2011 PREP SA on CD-ROM

Question: 130

You are evaluating a 5-week-old infant who had been born at 26 weeks gestation. She is
breathing room air and experiences infrequent apnea and bradycardia of prematurity that
requires caffeine therapy. She is tolerating full feedings of fortified human milk via nasogastric
tube and averaging 20 g/day weight gain. Physical examination shows a resting heart rate of 180
beats/min, respiratory rate of 50 breaths/min, blood pressure of 60/40 mm Hg, and oxygen
saturation of 94%. Because of the infrequent apnea, you order laboratory studies that reveal a
hemoglobin of 8.0 g/dL (80 g/L) and a caffeine concentration of 15 mcg/mL. You order a
transfusion of packed red blood cells.

Of the following, the MOST likely effect of the blood transfusion in this patient is

A. significantly decreased apnea

B. significantly decreased resting heart rate

C. significantly improved oxygen saturation

D. significantly increased blood pressure

E. unchanged respiratory rate

Copyright 2010 by the American Academy of Pediatrics page 415


2011 PREP SA on CD-ROM

Critique: 130 Preferred Response: B

The preterm infant described in the vignette has symptoms of anemia, and blood transfusion
should decrease her resting heart rate. Anemia of prematurity affects more than 50% of infants
born at less than 32 weeks gestation. The nadir in the preterm infant generally occurs between
4 and 10 weeks after birth, with a hemoglobin value as low as 7.0 to 8.0 g/dL (70 to 80 g/L).
Anemia of prematurity is due to repeated phlebotomy, inadequate red blood cell production, and
shortened red blood cell life span. Affected infants present with symptoms that include pallor,
decreased activity, poor weight gain, poor feeding, apnea, tachypnea, and tachycardia,
reflecting the role that red blood cells play in oxygen delivery.
The preterm infant cannot respond independently to severe anemia by activation of the
erythropoietin system until 34 to 36 weeks corrected gestational age. Activation does not occur
until the hemoglobin concentration reaches its nadir. Thus, transfusion during this time
suppresses the release of the erythropoietin and postpones recovery. Accordingly, the
hemoglobin value, in combination with clinical symptoms and corrected gestational age, guides
transfusion practices in preterm infants.
Transfusion of infants who have anemia of prematurity has been demonstrated to decrease
resting heart rate and respiratory rates. The role of transfusion in reducing apnea and
desaturation events is less clear. Some studies have reported no differences in apnea after
transfusion; others reported statistically significant differences, acknowledging that the absolute
decrease in frequency of apnea was less than one event per day. Arterial oxygen content, but
not oxygen saturation, is improved by transfusion. Blood pressure is not affected significantly
unless the transfusion is administered in response to acute blood loss.

Suggested reading:

Bell EF, Strauss RG, Widness JA, et al. Randomized trial of liberal versus restrictive guidelines
for red blood transfusion in preterm infants. Pediatrics. 2005;115:1685-1691. DOI:
10.1542/peds.2004-1884. Available at:
http://pediatrics.aappublications.org/cgi/content/full/115/6/1685

Kirpalani H, Whyte RK, Andersen C, et al. The Premature Infants in Need of a Transfusion (PINT)
study: a randomized, controlled trial of a restrictive (low) versus liberal (high) transfusion
threshold for extremely low birth weight infants. J Pediatr. 2006;149:301-307. DOI:
10.1016/j.jpeds.2006.05.011. Available at: http://www.ncbi.nlm.nih.gov/pubmed/16939737

Ohls RK. Anemia in the newborn infant. In: Polin RA, Yoder MC, eds. Workbook in Practical
Neonatology. 4th ed. Philadelphia, Pa: Saunders Elsevier; 2007:120-122

Westkamp E, Soditt V, Adrian S, Bohnhorst B, Groneck P, Poets CF. Blood transfusion in anemic
infants with apnea of prematurity. Biol Neonate. 2002;82:228-232. DOI: 10.1159/000065891.
Abstract available at: http://www.ncbi.nlm.nih.gov/pubmed/12381929

Widness JA. Pathophysiology of anemia during the neonatal period, including anemia of

Copyright 2010 by the American Academy of Pediatrics page 416


2011 PREP SA on CD-ROM

prematurity. NeoReviews. 2008;9:e520-e525. Available at:


http://neoreviews.aappublications.org/cgi/content/full/9/11/e520

Copyright 2010 by the American Academy of Pediatrics page 417


2011 PREP SA on CD-ROM

Question: 131

A 10-year-old boy has had patchy hair loss for the past month. He has a history of atopic
dermatitis and allergic rhinitis that has been treated intermittently, but he is not using any
medications at this time. He has been otherwise well, is growing normally, and is doing well in
school. He reports no new exposures and has not changed or added any hair care or skin
products. On physical examination, he has several oval patches of hair loss without scaling (Item
Q131A). Gently pulling hair at the periphery of a lesion frees a small number of hairs. You also
note pitting of his fingernails (Item Q131B).

Of the following, the MOST likely diagnosis is

A. alopecia areata

B. nevus sebaceus of Jadassohn

C. tinea capitis

D. traction alopecia

E. trichotillomania

Copyright 2010 by the American Academy of Pediatrics page 418


2011 PREP SA on CD-ROM

Question: 131

(Reprinted with permission from Krowchuk DP, Mancini AJ, eds. Pediatric Dermatology. A Quick
Reference Guide. Elk Grove Village, Ill: American Academy of Pediatrics; 2007)
Areas of hair loss, as described for the boy in the vignette.

Copyright 2010 by the American Academy of Pediatrics page 419


2011 PREP SA on CD-ROM

Question: 131

(Courtesy of D Krowchuk)
Pitting of the fingernails, as described for the boy in the vignette.

Copyright 2010 by the American Academy of Pediatrics page 420


2011 PREP SA on CD-ROM

Critique: 131 Preferred Response: A

Alopecia areata is a frequent cause of hair loss among children and adults, with a 1.7%
lifetime risk for individuals in the general population. About 60% of patients have their first
episode before age 20 years. Alopecia areata commonly presents as a sudden onset of one or
more oval patches of complete hair loss (Item C131A). Typically, the skin is smooth and flesh-
colored, although short hair stubs may be seen. Firmly pulling on a small clump of about 60 hairs
at the periphery of the lesion usually results in loss of six or more hairs. A positive hair pull test
also is seen with hair loss from telogen effluvium and sometimes with systemic disease (eg,
thyroid disease, iron deficiency anemia), but it can help differentiate alopecia areata from other
causes of hair loss such as trichotillomania or androgenic alopecia. The pattern of hair loss
varies widely; patchy loss is called alopecia areata, complete scalp hair loss is alopecia totalis,
and loss of all scalp and body hair is termed alopecia universalis. Nail changes, including pitting,
transverse grooves, roughened surface, or spooning, occurs in many patients (10% to 66%) at
some point during the course of the disease and are a marker for worse prognosis.
The cause of alopecia areata is uncertain, but heredity, stress, and an immune-mediated
mechanism have been proposed. A family history is noted frequently, with 37% of patients who
first present before 30 years of age and 7.1% of those presenting after 30 years having other
affected family members. Between 20% and 30% of patients (adults and children combined)
have other autoimmune diseases, including thyroiditis, type 1 diabetes mellitus, lupus
erythematosus, celiac disease, and vitiligo. A recent study of children found that 3.2% already
had a personal history of autoimmune disease (not different from controls), but an additional 5%
had Hashimoto thyroiditis discovered at the time of their initial diagnosis of alopecia areata.
Atopic disease is present in as many as 40% of those who have alopecia areata. Histologic
studies show an increase in T lymphocytes, particularly CD8(+) and CD57(+) cells, and
Langerhans cells within the areas of alopecia; some studies also report an increase of
eosinophils. Proposed psychological factors have not been consistently detectable when
studied.
History and physical findings differentiate alopecia areata from other forms of hair loss.
Trichotillomania typically consists of irregular areas of incomplete hair loss with underlying
normal scalp and remaining hairs of various lengths (Item C131B). Nevus sebaceus is present at
birth in two thirds of cases and occurs as a single oval-to-linear plaque (Item C131C). During
infancy, the lesion is smooth-to-papillated, waxy, and hairless; during puberty, sebaceous glands
hypertrophy, producing a more elevated and verrucous appearance. Tinea capitis presents as
one to several round patches of scale and alopecia with short stubs of broken hair. Traction
alopecia appears as circumscribed patches of alopecia at scalp margins (Item C131D), with
broken hairs and inflammatory follicular papules. Lymphadenopathy may be present.

Suggested reading:

Cetin ED, Savk E, Uslu M, Eskin M, Karul A. Investigation of the inflammatory mechanisms in
alopecia areata. Am J Dermatopathol. 2009;31:53-60. DOI: 10.1097/DAD.0b013e318185a66e.
Abstract available at: http://www.ncbi.nlm.nih.gov/pubmed/19155726

Copyright 2010 by the American Academy of Pediatrics page 421


2011 PREP SA on CD-ROM

Dombrowski NC, Bergfeld WF. Alopecia areata: what to expect from current treatments. Clev
Clin J Med. 2005;72:758-768. Available at: http://www.ccjm.org/content/72/9/758.long

Habif TP. Hair diseases. In: Clinical Dermatology: A Color Guide to Diagnosis and Therapy. 5th
ed. Philadelphia, Pa: Mosby Elsevier; 2010:913-990

Kakourou T, Karachristou K, Chrousos G. A case series of alopecia areata in children: impact of


personal and family history of stress and autoimmunity. J Eur Acad Dermatol Venereol.
2007;21:356-359. DOI: 10.1111/j.1468-3083.2006.01931.x. Abstract available at:
http://www.ncbi.nlm.nih.gov/pubmed/17309458

Kar BR, Handa S, Dogra S, Kumar B. Placebo-controlled oral pulse prednisolone therapy in
alopecia areata. J Am Acad Dermatol. 2005;52:287-290. DOI: 10.1016/j.jaad.2004.10.873.
Abstract available at: http://www.ncbi.nlm.nih.gov/pubmed/15692475

Morelli JC. Disorders of hair. In: Kliegman RM, Behrman RE, Jenson HB, Stanton BF, eds. Nelson
Textbook of Pediatrics. 18th ed. Philadelphia, Pa: Saunders Elsevier; 2007:2726-2730

Tosti A, Bellavista S, Iorizzo M. Alopecia areata: a long term follow-up study of 191 patients. J
Am Acad Dermatol. 2006;55:438-441. DOI: 10.1016/j.jaad.2006.05.008. Abstract available at:
http://www.ncbi.nlm.nih.gov/pubmed/16908349

Copyright 2010 by the American Academy of Pediatrics page 422


2011 PREP SA on CD-ROM

Critique: 131

(Reprinted with permission from Krowchuk DP, Mancini AJ, eds. Pediatric Dermatology. A Quick
Reference Guide. Elk Grove Village, Ill: American Academy of Pediatrics; 2007)
Alopecia areata presents with one or more patches of hair loss.

Copyright 2010 by the American Academy of Pediatrics page 423


2011 PREP SA on CD-ROM

Critique: 131

(Courtesy of D Krowchuk)
Trichotillomania is characterized by an area of relative alopecia within which hairs of differing
lengths may be seen. Erosions may be present.

Copyright 2010 by the American Academy of Pediatrics page 424


2011 PREP SA on CD-ROM

Critique: 131

(Courtesy of D Krowchuk)
A nevus sebaceus appears as a yellow, orange, or tan hairless plaque.

Copyright 2010 by the American Academy of Pediatrics page 425


2011 PREP SA on CD-ROM

Critique: 131

(Courtesy of Kimberly Horii, MD. Reprinted with permission from Krowchuk DP, Mancini AJ, eds.
Pediatric Dermatology. A Quick Reference Guide. Elk Grove Village, Ill: American Academy of
Pediatrics; 2007)
Marked thinning of the temporal scalp hair due to traction.

Copyright 2010 by the American Academy of Pediatrics page 426


2011 PREP SA on CD-ROM

Question: 132

A family with children ranging from 2 to 14 years of age is embarking on a summer vacation in
rural Central or South America that includes an opportunity to engage in volunteer work with an
international aid agency. The specific country assignment is pending.

Of the following, you are MOST likely to counsel the family to

A. apply sulphur powder to the childrens ankles when they are outdoors to avoid tick bites

B. apply topical DEET in a 50% concentration to the children at night to avoid mosquito bites

C. arrange for malaria prophylaxis in the host country upon arrival

D. immunize their children with typhus, dengue, and cholera vaccines today

E. return to your office when they know which country they will be visiting for malaria
prophylaxis

Copyright 2010 by the American Academy of Pediatrics page 427


2011 PREP SA on CD-ROM

Critique: 132 Preferred Response: E

The family described in the vignette is planning travel to rural Central or South America but
does not yet know which countries they will be visiting. In areas endemic for malaria,
chemoprophylaxis should begin 1 month before travel, but chemoprophylaxis may not be
necessary if they are not traveling to an endemic area. Prophylactic regimens are determined by
the resistance patterns in the country where they will be traveling. Accordingly, the family
described in the vignette should return to the pediatricians office once they know the country
they will be visiting. Country- and region-specific information is available on the web sites for the
Centers for Disease Control and Prevention and the World Health Organization and should be
used to plan therapy. Commonly used medications for malaria chemoprophylaxis in children older
than 8 years include chloroquine, mefloquine, and atovaquone-proguanil. Because chloroquine
resistance is common, this agent should only be used when traveling to a country with known
chloroquine-susceptible malaria. Because children are at special risk for serious consequences
of both infection and inappropriate use and doses of medication, significant time should be spent
in ensuring appropriate medications for age and determining parental understanding of
medication use. Prophylaxis for infants who weigh less than 5 kg is so problematic that experts
recommend delaying travel until they are older.
Prevention of tick- and mosquito-borne infections during travel to developing countries also
is important. Use of protective clothing, such as long-sleeved garments and socks, is
inexpensive and effective. In tropical climates, the use of mosquito netting during sleep is a
mainstay of prevention. Permethrin-impregnated clothing and fabrics for sleeping also are used.
Insecticides and repellants should be applied carefully to clothing and skin. N,N-diethyl-meta-
toluamide (DEET) may be applied sparingly to children as young as 2 months of age, but
insecticides should not be applied to childrens hands because of potential ingestion. Previously
published concerns about the use of DEET in children are not evidence-based, but low
concentrations of DEET (eg, <30%) should be used for exposures of more than 1 to 2 hours in
endemic areas. Although less well studied, oil of lemon eucalyptus and picardin also may be
used in children.
Prevention of tick bites includes avoidance of tick-infested areas and attention to tick
removal if necessary. As for mosquitos, insecticides are widely available for children older than
infancy. Use of occlusive clothing to prevent tick bites is paramount. Use of sulphur powder is
largely ineffective and is a topical and respiratory irritant.
Although parents often express concern about immunization against tropical illnesses,
immunization against pathogens covered by the United States routine immunization schedule is
crucial. The need for other immunizations 1 to 2 months before travel, such as those for typhus,
dengue, and cholera, are determined by the prevalence of these diseases in the destination
country.

Suggested reading:

American Academy of Pediatrics. Ehrlichia and Anaplasma infections (human ehrlichiosis and
anaplasmosis). In: Pickering LK, Baker CJ, Kimberlin DW, Long SS, eds. Red Book: 2009 Report
of the Committee on Infectious Diseases. 28th ed. Elk Grove Village, Ill: American Academy of

Copyright 2010 by the American Academy of Pediatrics page 428


2011 PREP SA on CD-ROM

Pediatrics; 2009:284-287

American Academy of Pediatrics. Malaria. In: Pickering LK, Baker CJ, Kimberlin DW, Long SS,
eds. Red Book: 2009 Report of the Committee on Infectious Diseases. 28th ed. Elk Grove
Village, Ill: American Academy of Pediatrics; 2009:438-444

American Academy of Pediatrics. Prevention of mosquito borne infections. In: Pickering LK,
Baker CJ, Kimberlin DW, Long SS, eds. Red Book: 2009 Report of the Committee on Infectious
Diseases. 28th ed. Elk Grove Village, Ill: American Academy of Pediatrics; 2009:193-195

American Academy of Pediatrics. Prevention of tickborne infections. In: Pickering LK, Baker CJ,
Kimberlin DW, Long SS, eds. Red Book: 2009 Report of the Committee on Infectious Diseases.
28th ed. Elk Grove Village, Ill: American Academy of Pediatrics; 2009:191-193

Maples HD, Schutze GE. Consultation with the specialist: malaria prophylaxis in children. Pediatr
Rev. 2006;27:346-350. DOI: 10.1542/10.1542/pir.27-9-346. Available at:
http://pedsinreview.aappublications.org/cgi/content/extract/27/9/346

Razzaq S, Schutze GE. Rocky Mountain spotted fever: a physicians challenge. Pediatr Rev.
2005;26:125-130. DOI: 10.1542/10.1542/pir.26-4-125. Available at:
http://pedsinreview.aappublications.org/cgi/content/full/26/4/125

Copyright 2010 by the American Academy of Pediatrics page 429


2011 PREP SA on CD-ROM

Question: 133

You are monitoring a 4-week-old male infant who has a large ventricular septal defect. He is
developing tachypnea but is not yet receiving medications. The surgeon has informed the family
that surgery should occur at approximately 6 months of age. The pediatric resident asks you
about expected symptoms over the next several months until scheduled surgery.

Of the following, the MOST appropriate information to share with her is that

A. diuretic therapy may help to relieve tachypnea and diaphoresis but will not improve weight gain

B. even with successful surgery, symptoms of congestive heart failure are likely to persist until
18 to 24 months of age

C. feeding difficulties and problems with poor weight gain are rare

D. most affected children do not develop pulmonary overcirculation

E. symptoms of congestive heart failure likely will develop over the next several weeks

Copyright 2010 by the American Academy of Pediatrics page 430


2011 PREP SA on CD-ROM

Critique: 133 Preferred Response: E

The natural history of a ventricular septal defect depends on the magnitude of the left-to-
right shunt. Smaller defects are restrictive, resulting in a high-grade murmur due to the pressure
gradient between the normal systemic blood pressure of the left ventricle and the much lower
pulmonary artery pressure of the right ventricle. For these patients, the magnitude of the left-to-
right shunt is minimal and, therefore, pulmonary overcirculation and associated congestive heart
failure do not occur. However, moderate-to-large ventricular septal defects, as described for the
infant in the vignette, are associated with a lower-grade murmur due to the lack of a pressure
gradient between the two ventricles. In this case, there is systemic pressure in the right
ventricle, and assuming that there is no obstruction to pulmonary outflow (ie, pulmonary stenosis
or infundibular muscular obstruction of the right ventricular outflow tract, as is present in
tetralogy of Fallot), the pulmonary artery pressure will be at systemic levels. Therefore,
congestive heart failure evolves over the first 1 to 2 postnatal months, as the pulmonary
vascular resistance naturally falls.
Symptoms of congestive heart failure include tachypnea, poor feeding, diaphoresis, and
poor energy. Affected children grow poorly due to reduced caloric intake associated with their
lack of energy and resultant poor intake and unmet increases in metabolic requirements. A
combination of pharmacologic and nutritional interventions often is required. The standard
pharmacologic regimen includes diuretics and digoxin. Diuretics reduce the volume of shunting to
the lung bed, thereby reducing dilation of the left atrium and left ventricle. The left heart dilation
results from inefficient left-to-right shunting in which previously oxygenated blood from the left
heart is shunted again through the ventricular septal defect back out to the pulmonary
circulation. Many infants receive human milk or formula fortified to achieve a caloric density of 22
to 30 kcal/oz, depending on caloric needs. Medications and calories are titrated to achieve a
weight gain of between 10 and 30 g/day.
Occasionally, an infant who has a large, unrestricted ventricular septal defect will not
manifest symptoms of pulmonary overcirculation and congestive heart failure because the
normal decrease in pulmonary vascular resistance does not occur. This prevents the
development of a large left-to-right shunt. Such babies are at risk for the eventual development
of Eisenmenger syndrome and irreversible pulmonary vascular changes. If they undergo
surgery before such vascular changes develop, successful surgical intervention is possible and
can mitigate further trauma to the pulmonary vasculature. The vessel wall abnormalities
subsequently regress over time.
Surgical intervention typically is performed at 4 to 6 months of age. However, the child who
is in uncompensated congestive heart failure such that he or she is too tachypneic to feed, thus
requiring tube feeding; who is not growing adequately; or who is in respiratory distress may
require earlier surgical intervention. In prior eras, these children may have been candidates for
staged surgical intervention, with the initial placement of a pulmonary artery band (thereby
avoiding neonatal cardiopulmonary bypass) and subsequent pulmonary artery band removal and
ventricular septal defect patch repair. However, the current approach is to perform the complete
repair either at the "standard" timeframe of 4 to 6 months or when intractable symptoms are
present. The survival from surgical intervention for an isolated perimembranous ventricular
septal defect (the most common location for such a lesion requiring surgery) is more than 98%.

Copyright 2010 by the American Academy of Pediatrics page 431


2011 PREP SA on CD-ROM

Almost all patients experience brief, uncomplicated postoperative recoveries. Residual


symptoms of congestive heart failure are rare after successful repair, and most children are
weaned from all cardiac medications by 4 to 8 weeks after surgery. These children do well long
term, but they warrant routine follow-up evaluations. Rare complications include rhythm
disturbances, ventricular dysfunction, or valve dysfunction.

Suggested reading:

Macicek SM, Macias CG, Jefferies JL, Kim JJ, Price JF. Acute heart failure syndromes in the
pediatric emergency department. Pediatrics. 2009;124:e898-e904. DOI: 10.1542/peds.2008-
2198. Available at: http://pediatrics.aappublications.org/cgi/content/full/124/5/e898

Madriago E, Silberbach M. Heart failure in infants and children. Pediatr Rev. 2010;31:4-12. DOI:
10.1542/10.1542/pir.31-1-4. Available at:
http://pedsinreview.aappublications.org/cgi/content/full/31/1/4

Minette MS, Sahn DJ. Ventricular septal defects. Circulation. 2006;114:2190-2197. DOI:
10.1161/CIRCULATIONAHA.106.618124. Available at:
http://circ.ahajournals.org/cgi/content/full/114/20/2190

Copyright 2010 by the American Academy of Pediatrics page 432


2011 PREP SA on CD-ROM

Question: 134

A 12-year-old girl presents to the emergency department with back pain. She had been practicing
flips on her backyard trampoline with parental supervision and landed awkwardly. Within 1 hour
of presentation, she complains of an inability to move her legs and numbness in both legs. On
physical examination, she has normal upper limb strength but reduced strength of both legs. You
suspect a spinal cord injury.

Of the following, the additional clinical finding that is most suggestive of an acute spinal cord
injury in this girl is

A. ankle clonus

B. dermatomal band of sensory loss in the trunk

C. hyperalgesia of the soles of the feet

D. hyporeflexia at the knees

E. positive Romberg sign

Copyright 2010 by the American Academy of Pediatrics page 433


2011 PREP SA on CD-ROM

Critique: 134 Preferred Response: D

The girl described in the vignette has sustained a spinal cord contusion from the fall. The
back pain and combination of numbness and weakness in both legs but not arms localizes the
problem to the spinal cord. In the short term after the injury, reflexes typically are diminished,
even though this is an "upper motor neuron" lesion, above the anterior horn cell/motor neurons
exiting the cord. Therefore, hyporeflexia at the knees would be expected. Upper motor neuron
signs of a positive Babinski response (upgoing toes), ankle clonus, hyperreflexia, and spasticity
all may occur in days or weeks if there has been permanent damage to the spinal cord, but
these would not be present acutely.
Sensory symptoms and physical examination findings may be helpful in this setting to
localize the lesion along sensory pathways: skin, nerve, root, spinal cord, thalamus, and cortex.
Among the potential positive findings are tingling or shooting pains, and negative findings may
include loss of pain/temperature or vibration/proprioception in predictable body distributions. A
complete spinal cord lesion results in loss of sensation of all modalities, from the toes up to a
dermatomal level in the trunk 1 to 2 roots below the level of top of the cord lesion.
A dermatomal band of sensory loss would be expected for injury of a single nerve exiting
the cord, not a problem intrinsic to the cord. Hyperalgesia, or increased sensitivity to pain,
results from damage to peripheral nerves.
A Romberg test is performed by having a patient stand with feet together, touching each
other, and the examiner standing next to the patient. With eyes open, vision, proprioception, and
vestibular systems combine to allow the patient to maintain balance. The visual system allows a
patient to compensate for a mild proprioceptive or vestibular deficit. When the eyes close,
characteristic swaying and loss of balance may occur. This positive Romberg sign may occur in
spinal cord lesions when there is a loss of posterior column/proprioceptive pathways. This test
cannot be performed or interpreted accurately when there is leg weakness, as in the girl in the
vignette.

Suggested reading:

Blumenfeld H. Motor exam. In: Neuroanatomy Through Clinical Cases. Available at:
http://www.neuroexam.com

Fenichel GM. Paraplegia and quadriplegia. In: Clinical Pediatric Neurology: A Signs and
Symptoms Approach. 6th ed. Philadelphia, Pa: Saunders Elsevier; 2009:267-284

Haslam RHA. Spinal cord disorders. In: Kliegman RM, Behrman RE, Jenson HB, Stanton BF, eds.
Nelson Textbook of Pediatrics. 18th ed. Philadelphia, Pa: Saunders Elsevier; 2007:2526-2530

Copyright 2010 by the American Academy of Pediatrics page 434


2011 PREP SA on CD-ROM

Question: 135

You are called to the newborn nursery to evaluate an infant who has a heart murmur. He was
born at term and has normal weight, length, and head circumference. On physical examination,
you note multiple unusual features: epicanthal folds with ocular hypertelorism; a broad, low nasal
bridge; a short, upturned nose (Item Q135); a grade III/VI harsh holosystolic murmur at the left
sternal border; and hypoplastic finger- and toenails.

Of the following, this infants unusual features are MOST likely related to prenatal exposure to

A. cocaine

B. methamphetamine

C. phenytoin

D. valproic acid

E. warfarin

Copyright 2010 by the American Academy of Pediatrics page 435


2011 PREP SA on CD-ROM

Question: 135

(Courtesy of M Rimsza)
Upturned nose, as described for the infant in the vignette.

Copyright 2010 by the American Academy of Pediatrics page 436


2011 PREP SA on CD-ROM

Critique: 135 Preferred Response: C

There is good evidence that the prenatal use of some antiepileptic drugs (AEDs) for the
treatment of seizure disorders is associated with a two- to threefold increased risk for
congenital malformations in exposed infants compared with the general population. Even so, it is
generally accepted that the benefits of treatment outweigh the risks for congenital anomalies.
One of the challenges facing researchers in this area is to design sufficiently large studies to
draw meaningful conclusions because epilepsy occurs in 6 in 1,000 pregnant women, and
congenital malformations are present in 3 in 100 live births in the general population.
The pattern of unusual features described for children who have been prenatally exposed
to AEDs commonly is called "fetal anticonvulsant syndrome." This term refers to the overlap of
features seen in affected individuals, but it is important to recognize that each anticonvulsant
may be associated with its own characteristic abnormalities. For example, prenatal phenytoin
exposure is associated with a broad, low nasal bridge; epicanthal folds; wide-spaced eyes
(hypertelorism); cardiovascular abnormalities; and distal digital hypoplasia (fetal hydantoin
syndrome), as described for the infant in the vignette. Prenatal exposure to valproic acid is
associated with an increased risk for neural tube defects as well as cardiac, limb, and renal
anomalies, and a characteristic facies (fetal valproate syndrome). Phenobarbital and
carbamazepine also are associated with increased risks for malformations and dysmorphic
features following prenatal exposure. Of note, polydrug therapy to control seizures in the
pregnant woman is associated with an increased risk for anomalies over monotherapy. In
addition to malformations, prenatal AED exposures are associated with developmental delays
and cognitive impairment.
Prenatal cocaine exposure is not associated with a well-defined syndrome, and facial
features are typically normal. Cocaine is associated with placental abruption and fetal vascular
disruption.
Prenatal exposure to methamphetamine is not known to be associated with unusual facial
features or birth defects. Long-term outcome information is not available for exposed individuals,
but there is concern for potential neurodevelopmental problems.
Approximately one third of individuals exposed to warfarin between 6 and 9 weeks
gestation have facial anomalies and bone stippling referred to as "fetal warfarin syndrome."
Facial anomalies include nasal hypoplasia and a depressed nasal bridge that may contribute to
upper airway obstruction. Exposure to warfarin during the second and third trimesters is
associated with central nervous system abnormalities, most likely due to hemorrhage.

Suggested reading:

Cocaine, Methamphetamine. REPROTOX. 2009

Cocaine, Methamphetamine. TERIS. 2009

Dean JCS, Hailey H, Moore SJ, Lloyd DJ, Turnpenny PD, Little J. Long term health and
neurodevelopment in children exposed to antiepileptic drugs before birth. J Med Genet.
2002;39:251-259. DOI: 10.1136/jmg.39.4.251. Available at:

Copyright 2010 by the American Academy of Pediatrics page 437


2011 PREP SA on CD-ROM

http://www.ncbi.nlm.nih.gov/pmc/articles/PMC1735079/

Dolk H, McElhatton P. Assessing epidemiological evidence for the teratogenic effects of


anticonvulsant medications. J Med Genet. 2002;39:243-244. DOI: 10.1136/jmg.39.4.243.
Available at: http://www.ncbi.nlm.nih.gov/pmc/articles/PMC1735083/

Gallagher RC, Kingham K, Hoyme HE. Fetal anticonvulsant syndrome. In: Cassidy SB, Allanson
JE, eds. Management of Genetic Syndromes. 2nd ed. Hoboken, NJ: John Wiley & Sons;
2005:239-250

Holmes LB. The teratogenicity of anticonvulsant drugs: a progress report. J Med Genet.
2002;39:245-247. DOI: 10.1136/jmg.39.4.245. Available at:
http://www.ncbi.nlm.nih.gov/pmc/articles/PMC1735092/

Jones KL. Fetal warfarin syndrome. In: Smiths Recognizable Patterns of Human Malformation.
6th ed. Philadelphia, Pa: Elsevier Saunders; 2006:656-657

Copyright 2010 by the American Academy of Pediatrics page 438


2011 PREP SA on CD-ROM

Question: 136

A 17-year-old boy asks you for help with quitting cigarette use. He started smoking at age 14
years and now smokes between four and six cigarettes a day. He was able to quit for 2 months
in the past year but resumed smoking after an argument with his girlfriend.

Of the following, the medical literature indicates that MOST youth

A. can quit smoking on their own

B. do not wish to quit smoking

C. double their cessation rates with counseling

D. find scare tactics very effective

E. increase their cessation rates with medication use

Copyright 2010 by the American Academy of Pediatrics page 439


2011 PREP SA on CD-ROM

Critique: 136 Preferred Response: C

Most adolescents are light smokers who do not smoke daily and often smoke less than 10
cigarettes a day. However, they are at risk for developing smoking-related diseases and have
trouble quitting. Therefore, every adolescent should be asked about tobacco use, while
remembering to respect their privacy and maintain confidentiality.
Most smoking youth wish to quit and try to do so on their own, but those who enroll in a
cessation program are twice as likely to be successful. The new United States Public Health
Service clinical practice guidelines strongly recommend that clinicians use effective counseling
methods and medications to help their patients quit the use of tobacco products. However,
evidence of the effectiveness of these methods in youth is not as strong as in adults.
One approach in primary care settings is the 5As model for brief face-to-face intervention
(ask, advise, assess, assist, arrange). Patients who appear unwilling to quit may respond to
brief interventions that are based on principles of motivational interviewing, a form of directive,
patient-centered counseling. Studies indicate that, compared with usual care (brief advice, self-
help pamphlets, reading materials, or a referral), the use of counseling doubles long-term
abstinence rates, although absolute success rates still are low (abstinence rates increase from
6.7% to 11.6% [95% confidence interval, 7.5-17.5]). There is no clear evidence to recommend a
particular counseling technique. A recent meta-analysis of studies that employed cognitive
behavioral strategies (self-monitoring and coping skills), social influence strategies (addressing
social influences that serve to promote or maintain smoking), and motivational strategies
(techniques to clarify desire for change and reduce ambivalence toward change) did find
significant effects.
Two methods are deemed ineffective or inappropriate for youth. One is a sensory
deprivation environment method, which requires that youth be placed in an environment that
deprives them of sensory stimulation (eg, a dark room) to help them clarify any conflicting
feelings they have about tobacco use. The second relies solely on "scare tactics" (eg, showing
pictures of diseased lungs, presenting people who have been disfigured by a tobacco-related
disease) to change tobacco behavior by evoking fear of the possible consequences of tobacco
use.
Seven medications (five nicotine and two non-nicotine) reliably increase long-term smoking
abstinence rates in adults: nicotine gum, inhaler, lozenge, nasal spray, and patch; and bupropion
SR; and varenicline. Although studies indicate that these medications are safe for use in
adolescents, long-term cessation rates do not differ from placebo in available studies. Results of
research on the use of varenicline in adolescents are awaited.

Suggested reading:

Fiore MC, Jan CR, Baker TB, et al. Treating Tobacco Use and Dependence: 2008 Update.
Clinical Practice Guideline. Rockville, Md: U.S. Department of Health and Human Services.
Public Health Service. May 2008. Available at:
http://www.ncbi.nlm.nih.gov/bookshelf/br.fcgi?book=hsahcpr&part=A28163

Klein JD, Camenga DR. Tobacco prevention and cessation in pediatric patients. Pediatr Rev.

Copyright 2010 by the American Academy of Pediatrics page 440


2011 PREP SA on CD-ROM

2004;25:1726. DOI: 10.1542/10.1542/pir.25-1-17. Available at:


http://pedsinreview.aappublications.org/cgi/content/full/25/1/17

Miller WR, Rollnick S. Motivational Interviewing: Preparing People for Change. 2nd ed. New York,
NY: Guilford Press; 2002

Sussman S, Sun P. Youth tobacco use cessation: 2008 update. Tob Induc Dis. 2009;5:3. DOI:
10.1186/1617-9625-5-3. Available at: http://www.tobaccoinduceddiseases.com/content/5/1/3

Copyright 2010 by the American Academy of Pediatrics page 441


2011 PREP SA on CD-ROM

Question: 137

The final patient on your rounds in the pediatric intensive care unit is an 18-year-old boy who is
being treated for acute respiratory distress syndrome (ARDS) that developed after he was found
unconscious in his bedroom following a drug overdose. The medical student on the rotation asks
you what the most common cause of ARDS is.

Of the following, the MOST appropriate answer is

A. fat emboli

B. inhalational injury

C. near-drowning

D. pneumonia

E. pulmonary contusion

Copyright 2010 by the American Academy of Pediatrics page 442


2011 PREP SA on CD-ROM

Critique: 137 Preferred Response: D

Disease processes that are associated with the development of acute lung injury (ALI) and
acute respiratory distress syndrome (ARDS) can injure the lung directly or indirectly. Common
causes of direct lung injury are pneumonia and gastric aspiration; less common causes include
fat emboli, inhalational injury, near-drowning, and pulmonary contusion. The most common cause
of indirect lung injury is sepsis, and less common causes include blood transfusions, drug
overdoses, and pancreatitis.
A recent prospective, population-based, pediatric cohort study estimated the overall
incidence of ALI/ARDS to be 1.8 cases per 100,000 person-years, with a hospital mortality rate
of 18%, both significantly lower than found in adults. Severe sepsis (with pneumonia as the
infection focus) was the most common risk factor, which is consistent with adult studies.

Suggested reading:

Frankel LR. Respiratory distress and failure. In: Kliegman RM, Behrman RE, Jenson HB, Stanton
BF, eds. Nelson Textbook of Pediatrics. 18th ed. Philadelphia, Pa: Saunders Elsevier; 2007:421-
423

Vish M, Shanley TP. Acute lung injury and acute respiratory distress syndrome. In: Wheeler DS,
Wong HR, Shanley TP, eds. Pediatric Critical Care Medicine: Basic Science and Clinical
Evidence. New York, NY: Springer-Verlag; 2007:395-411

Ware LB, Matthay MA. The acute respiratory distress syndrome. N Engl J Med.
2000;342:13341349. Extract available at:
http://content.nejm.org/cgi/content/extract/342/18/1334

Zimmerman JJ, Akhtar SR, Caldwell E, Rubenfeld GB. Incidence and outcomes of pediatric lung
injury. Pediatrics. 2009;124:87-95. DOI: 10.1542/peds.2007-2462. Available at:
http://pediatrics.aappublications.org/cgi/content/full/124/1/87

Copyright 2010 by the American Academy of Pediatrics page 443


2011 PREP SA on CD-ROM

Question: 138

The mother of a 1-year-old girl is concerned that the child has breast development on the left
side. She wonders whether it could be related to her eating tofu five to six times a week while
breastfeeding the girl for the first 9 months after birth. She denies any use of estrogen or
phytoestrogen-containing topical preparations or medications. On physical examination, you
discern about 2 cm of compressible subareolar breast tissue on the left and barely any
subareolar breast tissue on the right side.

Of the following, the MOST appropriate statement to make to the mother is that

A. because the breast tissue is unilateral, a surgical consultation is warranted

B. breast development in little girls in the first and second postnatal years generally is benign and
may be unilateral

C. breast development is more common in young girls today because of exposure to


environmental estrogen

D. the breast tissue likely is benign, and replacement of the milk in the childs diet with organic
cow milk will hasten its disappearance

E. tofu contains sufficient phytoestrogen to have caused the breast development

Copyright 2010 by the American Academy of Pediatrics page 444


2011 PREP SA on CD-ROM

Critique: 138 Preferred Response: B

Breast tissue in the first 2 postnatal years, which may be unilateral, is premature thelarche.
This generally benign condition is related to the minipuberty in girls in whom sufficient release of
follicle-stimulating hormone and luteinizing hormone stimulates occasional ovarian estrogen
production. It usually begins to regress by 2 years of age. In the course of an evaluation of such
a child, it is important to look for potent estrogen sources in the child's environment. Rarely,
children may be exposed to exogenous sources of estrogen (maternal oral contraceptives,
strong phytoestrogens) that might mimic this condition.
Surgical removal of unilateral breast tissue will remove the breast anlage and prevent the
development of a breast on that side at puberty. There is no evidence that breast development in
girls younger than age 2 or 3 years is more common now than in the past. Tofu has some
phytoestrogen, but cultures that consume large quantities of tofu do not have an increase in
premature thelarche, and no good data suggest that premature thelarche is related to soy
ingestion. Although the use of organic cow milk may be reasonable because of lower
concentrations of some potential contaminants in milk, it has not been shown to decrease the
frequency or severity of premature thelarche or hasten its resolution.

Suggested reading:

de Vries L, Guz-Mark A, Lazar L, Reches A, Phillip M. Premature thelarche: age at presentation


affects clinical course but not clinical characteristics or risk to progress to precocious puberty. J
Pediatr. 2010;156:466-471. Abstract available at:
http://www.ncbi.nlm.nih.gov/pubmed/19914634

Diamantopoulos S, Bao Y. Gynecomastia and premature thelarche: a guide for practitioners.


Pediatr Rev. 2007;28:e57-e68. DOI: 10.1542/10.1542/pir.28-9-e57. Available at:
http://pedsinreview.aappublications.org/cgi/content/full/28/9/e57

Gilchrist JM, Moore MB, Andres A, Estroff JA, Badger TM. Ultrasonographic patterns of
reproductive organs in infants fed soy formula: comparison to infants fed breast milk and infant
formula. J Pediatr. 2010;156:215-220. DOI: 10.1016/j.jpeds.2009.08.043. Abstract available at:
http://www.ncbi.nlm.nih.gov/pubmed/19846109

Jacobson-Dickman E, Lee MM. The influence of endocrine disruptors on pubertal timing. Curr
Opin Endocrinol Diabetes Obes. 2009;16:25-30. DOI: 10.1097/MED.0b013e328320d560.
Abstract available at: http://www.ncbi.nlm.nih.gov/pubmed/19115521

Copyright 2010 by the American Academy of Pediatrics page 445


2011 PREP SA on CD-ROM

Question: 139

You are seeing a 3-year-old boy for a health supervision visit. He has a vocabulary of 50 words
that he says clearly, and he is just beginning to combine words. He follows simple two-step
commands. Past medical history reveals that he was born at term and had no perinatal problems.
He has no history of recurrent ear infections and achieved gross motor milestones at appropriate
ages. He resides in a bilingual household. According to the family history, his father also
experienced delayed speech development. His older brother received speech therapy in
elementary school, and his older sister had difficulty with reading comprehension but does well in
math.

Of the following, the MOST likely cause of this boys language delay is

A. a bilingual household

B. being third born

C. genetic predisposition

D. hearing loss

E. his sex

Copyright 2010 by the American Academy of Pediatrics page 446


2011 PREP SA on CD-ROM

Critique: 139 Preferred Response: C

The most likely cause for the delay in language development described for the boy in the
vignette is genetic, based on the strong family history of language delays and reading difficulty.
Children who have specific language impairment or a reading disorder often have family
histories that suggest a genetic basis for the problems. The rate of reading disorder in parents
of affected children ranges from 25% to 60%. A child who has a family history of both
protracted language problems and learning issues has greater risk for language-based learning
difficulties.
A language delay of greater than 25% (calculated as a language level of below 27 months in
a 36-month-old child) should be evaluated and not disregarded because the child is a boy,
second or third born in a family, or part of a household in which more than one language is
spoken. Language development in boys averages only a 1- to 2-month lag behind girls. Current
research suggests that monolingual and bilingual children meet major language developmental
milestones at similar times. A child raised in a bilingual home may use the two languages during a
conversation, which has been found to be a sign of mastery of both languages. The childs total
vocabulary size and length of utterance (number of words in a sentence) should be normal by
age 2 to 3 years.
Other causes of delayed language development include genetic and chromosomal disorders
as well as neurologic disorders such as cerebral palsy. An autism spectrum disorder should be
suspected when language is both deviant and delayed. Global developmental delay may be
suspected if delays in motor or cognitive development also are apparent. Universal screening of
newborns detects most infants who have moderate, severe, or profound sensorineural hearing
loss, although such screening may miss individuals who have mild-to-moderate, progressive, or
acquired hearing loss. Hearing loss is a particular concern if the child has difficulty saying or
hearing the sounds s, th, and f (high-frequency sounds).

Suggested reading:

Coplan J. Language delays. In: Parker S, Zuckerman B, Augustyn M, eds. Developmental and
Behavioral Pediatrics: A Handbook for Primary Care. 2nd ed. Philadelphia, Pa: Lippincott
Williams & Wilkins; 2005:222

Feldman HM. Evaluation and management of language and speech disorders in preschool
children. Pediatr Rev. 2005;26:131-142. DOI: 10.1542/10.1542/pir.26-4-131. Available at:
http://pedsinreview.aappublications.org/cgi/content/full/26/4/131

Grizzle KL, Simms MD. Early language development and language learning disabilities. Pediatr
Rev. 2005;26:274-283. DOI: 10.1542/10.1542/pir.26-8-274. Available at:
http://pedsinreview.aappublications.org/cgi/content/full/26/8/274

King K, Fogle L. Raising bilingual children: common parental concerns and current research.
Center for Applied Linguistics Digest. 2006. Available at:
http://www.cal.org/resources/digest/raisebilingchild.html

Copyright 2010 by the American Academy of Pediatrics page 447


2011 PREP SA on CD-ROM

Question: 140

A 4-year-old boy presents to your office for evaluation of a 3-day history of fever (temperature
to 38.5C), congestion, and sore throat. Physical examination of the well-appearing child shows
only rhinorrhea and pharyngeal erythema. His mother and 6-year-old sister have had colds over
the past week.

Of the following, the MOST appropriate treatment for this child, pending the results of the throat
culture, is

A. amoxicillin

B. azithromycin

C. nasal saline drops

D. prednisone

E. pseudoephedrine

Copyright 2010 by the American Academy of Pediatrics page 448


2011 PREP SA on CD-ROM

Critique: 140 Preferred Response: C

The congestion and sore throat described for the boy in the vignette, combined with the
history of upper respiratory tract infections in the family, strongly suggest that he has a viral
illness. Supportive therapy such as nasal saline drops to relieve congestion is appropriate.
Cough and cold remedies, including those containing the decongestant pseudoephedrine,
have not been demonstrated to be effective in treating viral upper respiratory tract infection
symptoms, and based on potential toxicities in young children, the American Academy of
Pediatrics and United States Food and Drug Administration have advised against their use in
children younger than 6 years of age. There is no indication for prednisone in this setting.
However, high-dose, short-term corticosteroid therapy may be beneficial in the treatment of the
patient who has marked pharyngitis and impending airway obstruction associated with acute
infectious mononucleosis. Antibiotics are not indicated to treat a viral illness and do not prevent
development of possible secondary bacterial infections (eg, otitis media, sinusitis). Increased
use of antibiotics has been associated with increased rates of carriage of resistant bacteria
(eg, penicillin-resistant Streptococcus pneumoniae, beta-lactamase-positive Haemophilus
influenzae, methicillin-resistant Staphylococcus aureus).
Other common illnesses that generally do not require antibiotic therapy in children include
bronchitis, middle ear effusion of short duration, mucopurulent rhinitis of less than 10 days
duration, and most cases of acute pharyngitis (unless group A streptococcal infection is
confirmed). Bronchitis in children is an acute cough illness that is generally self-limited and
caused by viruses.
If the child in the vignette has a positive diagnostic test result (rapid antigen detection or
throat culture), antibiotic treatment would be appropriate. Penicillin V is the drug of choice for
streptococcal pharyngitis, although amoxicillin often is used instead as first-line treatment. A first-
generation cephalosporin (eg, cephalexin or cefadroxil) also may be used. Broader-spectrum
agents (eg, amoxicillin-clavulanate, second- or third-generation cephalosporins) are not
indicated routinely for this infection. Azithromycin should be reserved for treating streptococcal
pharyngitis in the patient who is allergic to penicillins and cephalosporins. Streptococcal
serogroups C and G rarely have been associated with symptomatic pharyngitis. They have not
been associated with rheumatic fever, but antibiotic therapy (same agents as for group A
streptococcal infection) may be considered in the symptomatic patient who has a positive
culture and no other cause determined for the pharyngitis.

As a result of reviewing this information, do you intend to make a change in practice


to provide better patient care?
Yes No

Suggested reading:

American Academy of Pediatrics. Group A streptococcal infections. In: Pickering LK, Baker CJ,
Kimberlin DW, Long SS, eds. Red Book: 2009 Report of the Committee on Infectious Diseases.
28th ed. Elk Grove Village, Ill: American Academy of Pediatrics; 2009:616-628

Copyright 2010 by the American Academy of Pediatrics page 449


2011 PREP SA on CD-ROM

American Academy of Pediatrics. Non-group A streptococcal and enterococcal infections. In:


Pickering LK, Baker CJ, Kimberlin DW, Long SS, eds. Red Book: 2009 Report of the Committee
on Infectious Diseases. 28th ed. Elk Grove Village, Ill: American Academy of Pediatrics;
2009:634-636

Centers for Disease Control and Prevention. Infant deaths associated with cough and cold
medications--two states, 2005. MMWR Morb Mortal Wkly Rep. 2007;56:1-4. Available at:
http://www.cdc.gov/mmwr/preview/mmwrhtml/mm5601a1.htm

Copyright 2010 by the American Academy of Pediatrics page 450


2011 PREP SA on CD-ROM

Question: 141

A 15-year-old boy presents with a 5-day history of sore throat, fever, and progressive right-
sided neck pain and swelling. On physical examination, his temperature is 40.0C, he has trismus,
the right side of his neck is swollen and tender to palpation, and his chest is clear to auscultation.
His white blood cell count is 30.0x103/mcL (30.0x109/L), with 80% polymorphonuclear
leukocytes, 15% lymphocytes, and 5% monocytes. Computed tomography scan of the neck
reveals a deep parapharyngeal abscess (Item Q141).

Of the following, the MOST appropriate antimicrobial to include in his therapy is

A. ampicillin-sulbactam

B. azithromycin

C. clarithromycin

D. gentamicin

E. trimethoprim-sulfamethoxazole

Copyright 2010 by the American Academy of Pediatrics page 451


2011 PREP SA on CD-ROM

Question: 141

(Courtesy of D Palazzi)
Computed tomography scan with contrast demonstrates a right parapharyngeal abscess (upper
line) with extension to the parotid space (lower line). There is rim enhancement of the abscess
best seen posteriorly. The right internal jugular vein is compressed.

Copyright 2010 by the American Academy of Pediatrics page 452


2011 PREP SA on CD-ROM

Critique: 141 Preferred Response: A

The boy described in the vignette has an abscess in the deep tissues of the neck.
Streptococci, including S pyogenes, and Staphylococcus aureus are the most common
pathogens associated with infections of the parapharyngeal space. However, oral anaerobic
bacteria also are found frequently in these infections because the primary portals of entry for
organisms into the parapharyngeal space are the oropharynx, lower molars, nasopharynx,
paranasal sinuses, and mastoid. The most common anaerobic bacteria isolated from
parapharyngeal infections are Bacteroides, Peptostreptococcus, and Fusobacterium. Most of
these infections are polymicrobial. Because the parapharyngeal space is contiguous with the
retropharyngeal, submandibular, and peritonsillar spaces, infection may spread in any number of
directions and lead to a variety of clinical manifestations and complications.
Ampicillin-sulbactam is a beta-lactamase-resistant semisynthetic penicillin that has activity
against anaerobes, susceptible aerobic gram-positive organisms, and respiratory tract gram-
negative pathogens, making it an appropriate initial drug for the patient described in the vignette.
Because group A streptococci are becoming increasingly resistant to macrolide antibiotics such
as azithromycin and clarithromycin and to trimethoprim-sulfamethoxazole, these drugs are not
appropriate. In addition, macrolide antibiotics have less activity than ampicillin-sulbactam against
B fragilis and Fusobacterium. Gentamicin is not useful because aerobic enteric gram-negative
rods do not play a significant role in parapharyngeal infections.
Among the other manifestations of anaerobic infection of the head and neck are sinusitis,
cervical adenitis, chronic otitis media, Ludwig angina, dental abscess, and gingivitis. Anaerobic
pathogens also are associated with infections of the central nervous system (brain abscess,
subdural empyema, epidural empyema), pleura and lungs (aspiration pneumonia, pleural
empyema, lung abscess), gastrointestinal tract (peritonitis, appendicitis, intra-abdominal
abscess, pelvic inflammatory disease, ascending cholangitis), skin and soft tissues (necrotizing
fasciitis, infected bite wounds, cellulitis or abscesses), and bloodstream. Serious infections are
seen more often in immunocompromised hosts or neonates.

Suggested reading:

Buckingham SC. Other anaerobic infections. In: Feigin RD, Cherry JD, Demmler-Harrison GJ,
Kaplan SL, eds. Feigin & Cherrys Textbook of Pediatric Infectious Diseases. 6th ed.
Philadelphia, Pa: Saunders Elsevier; 2009:1885-1894

Goldstein NA, Hammerschlag MR. Peritonsillar, retropharyngeal, and parapharyngeal abscesses.


In: Feigin RD, Cherry JD, Demmler-Harrison GJ, Kaplan SL, eds. Feigin & Cherrys Textbook of
Pediatric Infectious Diseases. 6th ed. Philadelphia, Pa: Saunders Elsevier; 2009:177-184

Copyright 2010 by the American Academy of Pediatrics page 453


2011 PREP SA on CD-ROM

Question: 142

A 6-year-old girl presents with fever, abdominal pain, and diarrhea. Her oral intake of fluids
(water and fruit juice) has been fair, with occasional emesis. On physical examination, she has
mildly tacky mucous membranes, a capillary refill of 2 to 3 seconds, and no edema. Her
temperature is 38.6C, heart rate is 120 beats/min, respiratory rate is 18 breaths/min, and blood
pressure is 102/56 mm Hg. Laboratory tests reveal:

Sodium, 126 mEq/L (126 mmol/L)


Potassium, 3.7 mEq/L (3.7 mmol/L)
Chloride, 100 mEq/L (100 mmol/L)
Bicarbonate, 16 mEq/L (16 mmol/L)
Blood urea nitrogen, 25 mg/dL (8.9 mmol/L)
Creatinine, 0.6 mg/dL (53.0 mcmol/L)

Urinalysis documents a specific gravity of 1.025, pH of 6, and trace ketones.

Of the following, the MOST likely additional laboratory finding is

A. elevated brain natriuretic peptide

B. fractional excretion of sodium greater than 1%

C. urine osmolality less than 300 mOsm/kg

D. urine potassium less than 10 mEq/L

E. urine sodium less than 10 mEq/L

Copyright 2010 by the American Academy of Pediatrics page 454


2011 PREP SA on CD-ROM

Critique: 142 Preferred Response: E

The girl described in the vignette has had gastroenteritis of a few days duration. Her
physical examination shows features of mild-to-moderate dehydration, with tachycardia, tacky
mucous membranes, and delayed capillary refill. Her laboratory findings are most notable for
hyponatremia, a normal anion gap metabolic acidosis, and mild azotemia.
Hyponatremia is defined as an abnormally low serum sodium concentration, which can
result from solute (sodium) loss or water retention. In general, hyponatremia most commonly is
due to abnormalities in water balance, not sodium balance. Furthermore, gastrointestinal losses
of fluid from vomiting or diarrhea usually result in losses of isotonic/isosmotic fluid. The loss of
isosmotic fluid should not result in hyponatremia, unless it is "replaced" by hypotonic oral (water
or other soft drinks) or intravenous fluids.
To excrete free water, a person must have normal renal function, normal renal blood flow,
and inhibition of antidiuretic hormone (ADH) release by the posterior pituitary. For the patient
suffering from an illness that results in volume depletion, such as gastroenteritis, ADH release is
physiologically appropriate. The appropriate medical management is to replenish the patients
extracellular fluid depletion with isotonic fluids. The patient in the vignette was managing her
own volume depletion by oral repletion with hypotonic fluids, which yielded hyponatremia. Thus,
her gastroenteritis resulted in extracellular volume depletion with increased ADH release and
impaired water excretion. Her consumption of hypotonic fluids led to relative water loading and
hyponatremia. Therefore, this patient has what can be called the syndrome of appropriate ADH
secretion. In addition to high ADH concentrations, patients who have intravascular volume
depletion can be expected to have activation of the renin-angiotensin-aldosterone system
(RAAS) and renal sympathetic nerves, thereby resulting in avid sodium and water retention by
the kidneys.
Results of the laboratory evaluation for the patient in the vignette would include: a low
urinary sodium concentration (<10 mEq/L), a low fractional excretion of sodium (<1%), and a
high urine osmolality (>500 mOsm/kg). Because of the high activity of the RAAS, the high
aldosterone effect would lead to sodium retention in exchange for potassium in the urine,
thereby raising the urine potassium excretion. Brain natriuretic peptide (BNP), like atrial natriuretic
peptide, is released in the clinical setting of increased cardiac filling volume resulting in mild
natriuresis. The patient in the vignette is volume depleted and, thus, would not be expected to
have elevated BNP concentrations in the blood.

Suggested reading:

Holliday MA, Ray PE, Friedman AL. Fluid therapy for children: facts, fashions and questions.
Arch Dis Child. 2007;92:546550. DOI: 10.1136/adc.2006.106377. Abstract available at:
http://www.ncbi.nlm.nih.gov/pubmed/17175577

Rivkees SA. Differentiating appropriate antidiuretic hormone secretion, inappropriate antidiuretic


hormone secretion and cerebral salt wasting: the common, uncommon, and misnamed. Curr
Opin Pediatr. 2008;20:448452. DOI: 10.1097/MOP.0b013e328305e403. Abstract available at:
http://www.ncbi.nlm.nih.gov/pubmed/18622203

Copyright 2010 by the American Academy of Pediatrics page 455


2011 PREP SA on CD-ROM

Rose BD, Post TW. Hypoosmolal states-hyponatremia. In: Clinical Physiology of Acid-base and
Electrolyte Disorders. 5th ed. New York, NY: McGraw-Hill Medical Publishing Division; 2001:696-
745

Copyright 2010 by the American Academy of Pediatrics page 456


2011 PREP SA on CD-ROM

Question: 143

The parents of a 6-week-old male infant bring in their son for evaluation of noisy breathing of 2
weeks duration. They state that the vaginal delivery was uncomplicated and the infant has been
bottle-feeding with appropriate weight gain on a cow milk formula. The noise occurs during
inspiration and worsens when the infant is placed supine or cries. The parents have not noticed
any rhinorrhea, fever, or other symptoms consistent with an upper respiratory tract infection.

Of the following, the MOST likely explanation for the noisy breathing is

A. airway foreign body

B. gastroesophageal reflux

C. laryngomalacia

D. milk protein allergy

E. vascular ring

Copyright 2010 by the American Academy of Pediatrics page 457


2011 PREP SA on CD-ROM

Critique: 143 Preferred Response: C

Laryngomalacia is the most common congenital laryngeal abnormality resulting in stridor.


Symptoms may begin shortly after birth, although typically they occur between 1 and 2 months
of age. As described for the boy in the vignette, infants are happy, thriving, and not having
difficulty during feedings, but stridor usually worsens during supine positioning, increased
crying or agitation, or a viral illness. Direct viewing of an omega-shaped epiglottis that prolapses
during inspiration is a hallmark of the condition (Item C143). Severe cases may require surgical
correction, although most children spontaneously improve by the second postnatal year.
Extrinsic compression of the trachea by vascular anomalies such as a vascular ring can
result in recurrent wheezing that is worsened with crying, feeding, or neck flexion. The right-
sided aortic arch with left ligamentum arteriosum and the double aortic arch account for most
cases. The presence of inspiratory stridor for this infant, rather than expiratory wheezing,
makes a vascular ring unlikely.
Milk protein allergy is an immunoglobulin E-mediated food allergy that is the most common
food allergy in the first year after birth. Approximately 80% to 90% of affected children present
with cutaneous symptoms (eg, eczema, urticaria, flushing, perioral rash), and the absence of
skin symptoms for this infant makes milk allergy unlikely. The more common adverse reaction to
cow milk occurring within the first 3 postnatal months is milk protein enterocolitis, which typically
results in hematochezia.
Gastroesophageal reflux (GER) is extremely common in infants younger than 6 months of
age. Typical symptoms can include choking, gagging, irritability, arching, and vomiting. In addition,
supine and side positions result in increased GER more than prone positioning.
A foreign body lodged in the upper airway proximal to the glottis opening could result in
inspiratory stridor. However, a foreign body typically is reported in older children who
inadvertently swallow the object. Although not present in all cases, a history of coughing,
choking, or airway obstruction frequently is reported at the time of aspiration. Radiography or
bronchoscopy often is required for definitive diagnosis.
A key approach to evaluating stridor or wheezing is to determine if the symptoms are
localized or diffuse. Localized wheezing may be due to a foreign body, vascular ring, or
tracheomalacia; diffuse or migratory wheezing can occur during a viral infection, GER, or an
asthma exacerbation.

Suggested reading:

Benjamin JR, Goldberg RN, Malcolm WF. Neonatal vocal cord paralysis. NeoReviews.
2009;10:e494-e501. Available at:
http://neoreviews.aappublications.org/cgi/content/full/10/10/e494

Vicencio AG, Parikh S, Adam HM. In brief: laryngomalacia and tracheomalacia: common dynamic
airway lesions. Pediatr Rev. 2006;27:e33-e35. DOI: 10.1542/10.1542/pir.27-4-e33. Available at:
http://pedsinreview.aappublications.org/cgi/content/full/27/4/e33

Copyright 2010 by the American Academy of Pediatrics page 458


2011 PREP SA on CD-ROM

Critique: 143

(Reprinted with permission from Vicencio AG, Parikh S. In brief: laryngomalacia and
tracheomalacia: common dynamic airway lesions. Pediatr Rev. 2006;27:e33-e35)
Laryngoscopic findings in laryngomalacia. During inspiration, the epiglottitis (arrows) collapses
onto the glottis and obstructs the airway. The obstruction is relieved with expiration.

Copyright 2010 by the American Academy of Pediatrics page 459


2011 PREP SA on CD-ROM

Question: 144

A 6-year-old boy comes to the emergency department after falling off of the monkey bars onto his
left arm. His left elbow is swollen, ecchymotic, and painful. You splint his arm, administer
parenteral morphine sulfate for pain, and obtain a radiograph (Item Q144). On return from
radiology, his left hand is pale and he complains of worsening pain. He can move all of his
fingers. His symptoms do not improve after you loosen the splint.

Of the following, the MOST likely cause of this boys findings is

A. anterior interosseous nerve injury

B. brachial artery injury

C. compartment syndrome

D. cubitus varus

E. Volkmann contracture

Copyright 2010 by the American Academy of Pediatrics page 460


2011 PREP SA on CD-ROM

Question: 144

(Courtesy of B Wood)
Radiographic findings, as described for the boy in the vignette.

Copyright 2010 by the American Academy of Pediatrics page 461


2011 PREP SA on CD-ROM

Critique: 144 Preferred Response: B

The acute onset of hand pallor following a supracondylar fracture of the humerus, as
described for the boy in the vignette, should alert the clinician to a brachial artery injury, an
uncommon but serious complication of this type of fracture. Approximately 5% to 20% of
children who have distal humeral fractures experience vascular injury, especially with fractures
that involve posterolateral displacement of the distal fragment. In most cases, the artery is
compressed by the fracture fragment, and prompt reduction of the fracture restores distal
perfusion. The brachial artery is particularly vulnerable because it travels along the anterior
humerus before dividing into the radial and ulnar branches below the antecubital fossa.
Anterior interosseous nerve injury can occur with this fracture but causes weakness of the
index and middle fingers. Compartment syndrome occurs in approximately 1% of supracondylar
fractures, but pallor is a late finding. Cubitus varus is the most common late complication of a
supracondylar fracture and results in a cosmetic deformity in which the forearm deviates
toward the torso when the arm is extended with the palm facing forward. Volkmann contracture
is a late sequela of forearm muscle ischemia related to upper extremity compartment syndrome
and results in a flexion contracture of the fingers, hand, and wrist.
Other pediatric fractures that may be associated with vascular complications include tibial
fractures, which may precipitate compartment syndrome in the distal lower extremity; scaphoid
fractures of the wrist, which are at risk for ischemic necrosis and malunion if the arterial blood
supply to the bone is disrupted; and posterior sternoclavicular dislocations, in which the
dislocated proximal clavicle may compress both the upper airway and the subclavian vessels.

Suggested reading:

Gholve PA, Hosalkar HS, Wells L. Common fractures. In: Kleigman RM, Behrman RE, Jenson HB,
Stanton BF, eds. Nelson Textbook of Pediatrics. 18th ed. Philadelphia, Pa: Saunders Elsevier;
2007:2834-2841

Manning Ryan L. Evaluation and management of supracondylar fractures in children. UpToDate


Online 17.3. 2009. Available at:
http://www.utdol.com/online/content/topic.do?topicKey=ped_trau/18971&selectedTitle=15~150&
source=search_result

Mathison DJ, Agrawal D. General principles of fracture management: fracture patterns and
description in children. UpToDate Online 17.3. 2009. Available at:
http://www.utdol.com/online/content/topic.do?topicKey=ped_trau/18027&selectedTitle=7~150&s
ource=search_result

Wheeless CR III. Pediatric supracondylar fracture of the humerus. In: Wheeless Textbook of
Orthopaedics. 2008. Available at:
http://www.wheelessonline.com/ortho/pediatric_supracondylar_fractures_of_the_humerus

Copyright 2010 by the American Academy of Pediatrics page 462


2011 PREP SA on CD-ROM

Question: 145

You are caring for an 18-year-old girl who was diagnosed 6 months ago with Crohn disease
affecting the terminal ileum and colon. Her medications include prednisone 10 mg/day,
sulfasalazine 1 g three times per day, and 6-mercaptopurine 75 mg/day. She has experienced an
excellent symptomatic response to treatment and presents today complaining only of occasional
streaks of blood in an otherwise normal daily bowel movement. Since diagnosis, she has gained
2.5 kg on a regular diet with only iron supplementation.

Of the following, the girl is MOST at risk of developing a deficiency of

A. copper

B. folic acid

C. selenium

D. vitamin D

E. zinc

Copyright 2010 by the American Academy of Pediatrics page 463


2011 PREP SA on CD-ROM

Critique: 145 Preferred Response: B

Malnutrition is a common finding in inflammatory bowel disease (IBD), particularly Crohn


disease. Although the cause is multifactorial, decreased energy intake is the most frequent
contributing factor. Other causes include increased energy requirements, enteric losses,
malabsorption, and drug-nutrient interactions. As a result, up to 85% of pediatric patients who
have Crohn disease exhibit documented weight loss at the time of diagnosis, and growth failure
has been described in 15% to 40% of affected children. Although nutrient deficits are more
prevalent in patients who have extensive small bowel involvement and in those who exhibit
increased disease activity, nutritional problems are not limited to these subgroups.
In most cases, nutritional status may be improved and catch-up growth realized by enteral
alimentation (which also may serve as primary disease therapy), nutritional supplements, and
controlling symptoms, which increases appetite.
Treatment for the girl described in the vignette included the use of sulfasalazine, a known
competitive inhibitor of folate absorption. Accordingly, she is at risk of developing folic acid
deficiency. All patients receiving this drug, many of whom already may manifest reduced folate
stores, should receive dietary supplementation with folic acid to prevent a deficiency state.
Because, in part, of this property of sulfasalazine, its use in IBD largely has been supplanted by
the 5-aminosalicylate analogs, which do not interfere with folate metabolism.
Although one recent study in pediatric patients demonstrated normal folate stores at the time
of Crohn disease diagnosis, previous work in adults found both reduced folate concentrations
and impaired folate absorption. Low folate stores may result from several factors, including
reduced intake of green, leafy vegetables; increased disease activity; and drug-nutrient
interactions. The clinical consequences of this micronutrient deficit in IBD have not been fully
elucidated, but reduced folate has been associated with elevated homocysteine concentrations
in children who have both Crohn disease and ulcerative colitis, suggesting a possible link
between folate deficiency and the thrombotic complications of these disorders. Deficiencies of
vitamins B6 and B12, the latter a particular concern in patients who have extensive ileal disease,
also have been linked to elevated concentrations of homocysteine.
Iron deficiency is the most common mineral deficiency associated with Crohn disease and is
usually due to enteric blood loss. Children who have Crohn disease may exhibit reduced serum
concentrations of copper, selenium, and zinc. These mineral deficiencies appear to be related
directly to disease activity and develop as a result of both enteric loss and oxidative stress. Zinc
deficiency also has been associated with impaired metabolism of retinol-binding protein, leading
to vitamin A deficiency. However, such nutrient deficits are unlikely to develop in patients who
exhibit low disease activity indices.
Oxidative stress may lead directly to impaired vitamin A and vitamin E nutriture. In such
cases, concentrations normalize in response to control of disease activity. Vitamins A and E, as
well as vitamins D and K, are fat-soluble and require micellar solubilization for absorption.
Accordingly, nutritional adequacy may be a concern in the setting of extensive ileal disease
leading to bile salt loss and fat malabsorption.
Maintenance of skeletal health is an important objective in the care of patients who have
Crohn disease. This is especially critical in children, for whom dysregulation of calcium and
vitamin D metabolism may lead to osteopenia and inhibit skeletal growth. Problems related to

Copyright 2010 by the American Academy of Pediatrics page 464


2011 PREP SA on CD-ROM

calcium homeostasis have been demonstrated in those who have Crohn disease, independent
of glucocorticoid use. Studies have shown that a significant percentage of patients exhibit
increased 1,25-dihydroxyvitamin D and reduced 25-hydroxyvitamin D concentrations, probably
because of the overexpression of 1-alpha-hydroxylase in inflamed small bowel mucosa. Low
concentrations of 25-hydroxyvitamin D result in reductions in serum calcium that, in turn, lead to
increased parathormone secretion. As a consequence, 1,25-dihydroxyvitamin D concentrations
rise, stimulating bone resorption and calcium release.

Suggested reading:

Chan EP, Lichtenstein GR. Chemoprevention: risk reduction with medical therapy of inflammatory
bowel disease. Gastroenterol Clin North Am. 2006;35:675-712. DOI: 10.1016/j.gtc.2006.07.003.
Abstract available at: http://www.ncbi.nlm.nih.gov/pubmed/16952746

Heyman MB, Garnett EA, Shaikh N, et al. Folate concentrations in pediatric patients with newly
diagnosed inflammatory bowel disease. Am J Clin Nutr. 2009;89:545-550. Abstract available at:
http://www.ncbi.nlm.nih.gov/pubmed/19116333

Nakano E, Taylor CJ, Chada L, McGaw J, Powers HJ. Hyperhomocystinemia in children with
inflammatory bowel disease. J Pediatr Gastroenterol Nutr. 2003;37:586-590. Available at:
http://journals.lww.com/jpgn/Fulltext/2003/11000/Hyperhomocystinemia_in_Children_With_Inflam
matory.16.aspx

Copyright 2010 by the American Academy of Pediatrics page 465


2011 PREP SA on CD-ROM

Question: 146

You are evaluating a 2-month-old infant in your office for "wheezing" with crying. He had severe
group B streptococcal sepsis, meconium aspiration syndrome, and persistent pulmonary
hypertension at birth that required mechanical ventilation in the neonatal intensive care unit for the
first 3 postnatal weeks. He was discharged 1 week ago breathing room air and receiving inhaled
corticosteroids and diuretic therapy. He is receiving human milk through a percutaneous gastric
tube with no evidence of gastroesophageal reflux. Physical examination reveals a pink, well-
perfused infant who has a respiratory rate of 40 breaths/min. Auscultation demonstrates
expiratory low-pitched, homophonous wheezing transmitting equally to all lung fields that appears
to worsen when the infant is supine and crying. Inspiratory sounds are normal.

Of the following, the MOST likely cause of this infants signs and symptoms is

A. chronic lung disease

B. reactive airway disease

C. subglottic stenosis

D. tracheomalacia

E. vocal cord paralysis

Copyright 2010 by the American Academy of Pediatrics page 466


2011 PREP SA on CD-ROM

Critique: 146 Preferred Response: D

The symptoms of expiratory wheezing while crying and lying supine described for the infant
in the vignette most likely represent tracheomalacia. Tracheomalacia results from weakness in
the airway cartilage that leads to floppiness, causing the airway to collapse on itself during
expiration (Item C146). Tracheomalacia may be congenital, but it also is associated with
mechanical ventilation due to weakening of the tracheal cartilage because of high ventilatory
pressures. It may present with expiratory wheezing, coughing, or life-threatening "death spells."
Such spells may be triggered by feeding, crying, or coughing and can result in cyanosis,
hypoxia, apnea, and bradycardia. Supine positioning or crying may worsen the clinical findings;
prone positioning results in improvement.
Chronic lung disease and reactive airway disease also may be seen in association with
mechanical ventilation. Pulmonary oxygen toxicity, barotrauma, and inflammatory mediators all
are believed to play roles in the development of these chronic changes. The wheezing that can
be associated with chronic lung disease and reactive airway disease also is expiratory but
does not improve when the infant is placed prone. Additional clinical findings in infants who have
chronic lung disease may include grunting, flaring, and retracting.
Intubation can lead to subglottic stenosis in the developing airway, when soft-tissue
stenosis develops from granulation tissue, submucosal gland hyperplasia, or submucosal
fibrosis. Severe stenosis presents with inspiratory or biphasic stridor and respiratory distress;
less severe cases present with recurrent croup with viral infections.
Vocal cord paralysis in the infant may be congenital or acquired. The most common cause of
acquired vocal cord paralysis is believed to be stretching of the laryngeal nerves during delivery.
Whether vocal cord paralysis is associated with intubation is controversial. The clinical findings
include inspiratory stridor and weak cry.

Suggested reading:

Cullen AB, Wolfson MR, Shaffer TH. Impact of mechanical ventilation on the developing airway.
NeoReviews. 2002;3:e137-e142. Available at:
http://neoreviews.aappublications.org/cgi/content/full/3/7/e137

Holinger LD. Congenital anomalies of the larynx, trachea, and bronchi. In: Kliegman RM, Behrman
RE, Jenson HB, Stanton BF, eds. Nelson Textbook of Pediatrics. 18th ed. Philadelphia, Pa:
Saunders Elsevier; 2007:1767-1768

Vicencio AG, Parikh S, Adam HM. In brief: laryngomalacia and tracheomalacia: common dynamic
airway lesions. Pediatr Rev. 200627:e33-e35. DOI: 10.1542/10.1542/pir.27-4-e33. Available at:
http://pedsinreview.aappublications.org/cgi/content/full/27/4/e33

Copyright 2010 by the American Academy of Pediatrics page 467


2011 PREP SA on CD-ROM

Critique: 146

(Reprinted with permission from Vicencio AG, Parikh S. In brief: laryngomalacia and
tracheomalacia: common dynamic airway lesions. Pediatr Rev. 2006;27:e33-e35)
Bronchoscopic findings in tracheomalacia. During inspiration, the trachea remains patent and the
carina is easily visible. During expiration, the trachea collapses on itself, obstructing the carina
and distal airways.

Copyright 2010 by the American Academy of Pediatrics page 468


2011 PREP SA on CD-ROM

Question: 147

You are evaluating a 15-month-old child for mild developmental delay. He sits well alone, crawls,
claps his hands, and waves, but he does not pull to stand or say any words. His father and
paternal uncle have epilepsy and attended special education classes when they were in school.
You note three hypopigmented macules scattered over his trunk (Item Q147) and arms.

Of the following, the MOST appropriate next step in this boys evaluation is

A. auditory brainstem evoked response testing

B. biopsy of a hypopigmented macule

C. electroencephalography

D. magnetic resonance imaging of the brain

E. serum amino acid studies

Copyright 2010 by the American Academy of Pediatrics page 469


2011 PREP SA on CD-ROM

Question: 147

(Courtesy of D Krowchuk)
Hypopigmented macules, as described for the child in the vignette.

Copyright 2010 by the American Academy of Pediatrics page 470


2011 PREP SA on CD-ROM

Critique: 147 Preferred Response: D

The combination of skin findings and developmental delays, along with the family history of
seizures, described for the child in the vignette should raise concern for a neurocutaneous
syndrome. In particular, hypomelanotic macules are one of the characteristic, although not
pathognomonic, findings of tuberous sclerosis complex (TSC).
TSC is an autosomal dominant condition with a high rate (2/3) of sporadic mutations. Two
genes that have numerous allelic forms have been identified as causative in most cases: TSC1
on chromosome 9 and TSC2 on chromosome 16. In 10% to 15% of cases, a mutation cannot be
located. In general, mutations to TSC2 cause more severe disease.
Classically, TSC was diagnosed based on the triad of seizures, intellectual disability, and
facial angiofibromas (previously known as adenoma sebaceum) (Item C147A). Revised
diagnostic criteria for TSC (Item C147B) require two major criteria or one major and two minor
criteria for definite diagnosis, one major and one minor for probable TSC, and one major or two
or more minor criteria for possible TSC. Signs and symptoms result from tumors involving a
number of organ systems.
Several characteristic skin lesions noted in the diagnostic criteria for TSC present at various
ages and have various configurations (Item C147C). The earliest lesion is the hypomelanotic
macule, sometimes called ash leaf spot or Fitzpatrick patch. These may be visible at birth and are
nearly always noted by 2 years of age. They become even more visible when examined under
ultraviolet light. They are present in nearly all patients who have TSC, but isolated hypomelanotic
macules also may be seen in healthy individuals. However, multiple lesions are rare in the
general population, so the presence of three or more hypomelanotic macules is a major criterion
for the diagnosis of TSC.
The greatest morbidity in TSC is related to the presence of central nervous system (brain)
hamartomas. Characteristic lesions include cortical tubers, subependymal nodules, and
subependymal giant cell astrocytomas. Seizures (often infantile spasms) are frequent; 85% of
patients have their first seizure before age 2 years. Cognitive disability, including intellectual
disability and autism, is frequent, affecting up to 50% of patients.
Renal, eye, pulmonary, and cardiac systems also may be involved. Cardiac rhabdomyomas
affect 47% to 60% of children who have TSC and may be diagnosed prenatally by
ultrasonography or magnetic resonance imaging. Many of these tumors regress as the patient
grows older. The most common renal findings are angiomyolipomas, which are present in up to
80% of patients by age 10.5 years. Interestingly, the TSC2 gene is adjacent to PDK1, the
polycystic kidney disease 1 gene, on the chromosome. Retinal hamartomas also increase in
frequency with age and can be found in 40% to 50% of patients. Lung involvement is rare,
affecting 1% to 6% of patients, usually adult women. The most common lung finding is
lymphangiomyomatosis, manifesting as progressive, cystic lung lesions with pneumothoraces.
The prognosis for this lesion is poor.
The skin findings reported for the patient in the vignette suggest possible TSC. Cranial
magnetic resonance imaging is necessary to determine the presence of brain hamartomas.
Additional diagnostic testing should include ophthalmologic evaluation, neurodevelopmental
testing, echocardiography, and renal ultrasonography. Although the boys delayed speech
suggests the need for audiology evaluation, behavioral audiologic testing is the preferred initial

Copyright 2010 by the American Academy of Pediatrics page 471


2011 PREP SA on CD-ROM

method for a child of this age. Biopsy of a characteristic lesion is not needed to make the
diagnosis. Because the child has not exhibited evidence of a seizure disorder,
electroencephalography is not needed at this time. Metabolic studies, including serum amino acid
studies, are not indicated.

Suggested reading:

Curatola P, Bombardieri R, Jwiak S. Tuberous sclerosis. Lancet. 2008;372:657-668. DOI:


10.1016/S0140-6736(08)61279-9. Abstract available at:
http://www.ncbi.nlm.nih.gov/pubmed/18722871

Ewalt DH, Sheffield E, Sparagana SP, Delgado MR, Roach ES. Renal lesion growth in children
with tuberous sclerosis complex. J Urol. 1998;160:141-145. Abstract available at:
http://www.ncbi.nlm.nih.gov/pubmed/9628635

Jwiak J, Galus R. Molecular implications of skin lesions in tuberous sclerosis. Am J


Dermatopathol. 2008;30:256-261. DOI: 10.1097/DAD.0b013e31816e22a5. Abstract available at:
http://www.ncbi.nlm.nih.gov/pubmed/18496427

Schwartz RA, Fernndez G, Kotulska K, Jwiak S. Tuberous sclerosis complex: advances in


diagnosis, genetics and management. J Am Acad Dermatol. 2007;57:189-202. DOI:
10.1016/j.jaad.2007.05.004. Abstract available at:
http://www.ncbi.nlm.nih.gov/pubmed/17637444

Siegel DH, Howard R. Molecular advances in genetic skin diseases. Curr Opin Pediatr.
2002;14:419-425. Abstract available at: http://www.ncbi.nlm.nih.gov/pubmed/12130905

Copyright 2010 by the American Academy of Pediatrics page 472


2011 PREP SA on CD-ROM

Critique: 147

(Reprinted with permission from Krowchuk DP, Mancini AJ, eds. Pediatric Dermatology. A Quick
Reference Guide. Elk Grove Village, Ill: American Academy of Pediatrics; 2007)
Facial angiofibromas (adenoma sebaceum) are observed in children who have tuberous
sclerosis complex.

Copyright 2010 by the American Academy of Pediatrics page 473


2011 PREP SA on CD-ROM

Critique: 147

Copyright 2010 by the American Academy of Pediatrics page 474


2011 PREP SA on CD-ROM

Critique: 147

Copyright 2010 by the American Academy of Pediatrics page 475


2011 PREP SA on CD-ROM

Critique: 147

Copyright 2010 by the American Academy of Pediatrics page 476


2011 PREP SA on CD-ROM

Critique: 147

Copyright 2010 by the American Academy of Pediatrics page 477


2011 PREP SA on CD-ROM

Question: 148

A family in your practice has struggled for years with recurrent head lice. Despite careful
application of permethrin products, the children are once again in your office with resistant
infestation. Examination of the two children, ages 4 and 6 years, reveals both live lice and nits in
their hair. The family states that they have complied with all instructions regarding use of
permethrin, nit combs, and environmental eradication.

Of the following, the MOST appropriate next step is

A. oral trimethoprim-sulfamethoxazole for 1 week

B. topical application of tea tree oil for 1 week

C. topical lindane application once

D. topical malathion application once

E. topical permethrin reapplication weekly for 6 weeks

Copyright 2010 by the American Academy of Pediatrics page 478


2011 PREP SA on CD-ROM

Critique: 148 Preferred Response: D

Head lice are a common and frustrating problem for parents and practitioners. Most
remedies have involved killing of the mature louse; ovicidal agents to kill the nits have been
slower to develop. Use of pediculocides without ovicidal activity has necessitated more than
one application of the insecticide over several days to weeks. The family described in the
vignette has experienced a problem of recurrent infestation of head lice, most likely due to
treatment resistance. In this situation, the most effective therapeutic option is a single topical
application of malathion.
Malathion has become more of a mainstay of treatment for head lice in children. Although it is
an organophosphate insecticide with theoretical risks for toxicity, no evidence of high risk for
children has emerged from widespread use of a dilute solution in isopropyl alcohol in Europe.
Malathion has the added benefit of being ovicidal.
The use of lindane has waned for three reasons: resistance to its effect, toxicity due to
gamma aminobutyric acid (GABA) receptor inhibition, and lack of ovicidal activity. The efficacy of
permethrin and pyrethrins is even more restricted because of significant resistance throughout
the United States, most likely due to over-the-counter use. Presumably, the children described in
the vignette are experiencing the results of such resistance, making reapplication unhelpful.
Trimethoprim-sulfamethoxazole presumably kills gut bacteria in the louse, but the efficacy of
this method is not proven in large trials. Tea tree oil (terpineol) is a plant derivative that has both
pediculocidal and ovicidal activity. However, topical application may be irritating to the eyes and
may limit its usefulness. It is found in malathion products as an adjunct.

As a result of reviewing this information, do you intend to make a change in practice


to provide better patient care?
Yes No

Suggested reading:

American Academy of Pediatrics. Pediculosis capitis (head lice). In: Pickering LK, Baker CJ,
Kimberlin DW, Long SS, eds. Red Book: 2009 Report of the Committee on Infectious Diseases.
28th ed. Elk Grove Village, Ill: American Academy of Pediatrics; 2009:495-497

Lebwohl M, Clark L, Levitt J. Therapy for head lice based on life cycle, resistance, and safety
considerations. Pediatrics. 2007;119:965-974. DOI: 10.1542/peds.2006-3087. Available at:
http://pediatrics.aappublications.org/cgi/content/full/119/5/965

Pearlman DL. A simple treatment for head lice: dry-on, suffocation-based pediculicide.
Pediatrics. 2004;114:e275-e279. DOI: 10.1542/peds.2003-0666-F. Available at:
http://pediatrics.aappublications.org/cgi/content/full/114/3/e275

Copyright 2010 by the American Academy of Pediatrics page 479


2011 PREP SA on CD-ROM

Question: 149

The pediatric cardiologist to whom you referred a 4-year-old girl who has a heart murmur
identified a bicuspid aortic valve. He reports moderate aortic stenosis without aortic insufficiency.
Although no intervention is required at present, he suggests that progressive valve disease is
possible. The parents ask you about their childs prognosis.

Of the following, you are MOST likely to advise the parents that

A. their daughter eventually will require cardiovascular surgery or catheter-based intervention


due to increasing stenosis

B. their daughter is more likely to develop atrial rather than ventricular arrhythmias as the severity
of aortic stenosis worsens

C. their daughter is unlikely to develop aortic insufficiency

D. their daughter will require antibiotic prophylaxis to prevent subacute bacterial endocarditis

E. their daughters children will have the same risk for inheritance of a cardiac defect as the
general population

Copyright 2010 by the American Academy of Pediatrics page 480


2011 PREP SA on CD-ROM

Critique: 149 Preferred Response: A

A bicuspid (two-leaflet) aortic valve (Item C149) is the most common congenital heart defect,
present in approximately 1% of the general population. Most of these valves function well with
minimal, if any, aortic valve stenosis or insufficiency. This subset of patients is asymptomatic,
with their sole clinical manifestation being a soft murmur or early systolic valve click. Patients
who have bicuspid aortic valves and, at most, mild aortic stenosis or insufficiency are evaluated
every 2 to 3 years because as many as 70% develop some degree of stenosis or insufficiency
by the age of 30 years.
A small percentage of patients have more severe valve disease and may require catheter-
based or surgical intervention either in infancy or childhood. Maximal rates of change are most
likely to occur during the first months after birth and in adolescence, although progressive valve
dysfunction is of variable onset. Calcification of the bicuspid aortic valve occurs commonly after
the third and fourth decades of life. Children who develop progressive aortic stenosis may
develop ventricular arrhythmias and exercise intolerance. As symptoms progress, the left
ventricle often becomes increasingly hypertrophied. With an increasingly stenotic valve, the
development of progressive chronic aortic insufficiency also is common. Insufficiency results in
dilation of the left ventricle and eventually left ventricular systolic dysfunction.
The child who has isolated aortic stenosis can undergo balloon aortic valvuloplasty in the
cardiac catheterization laboratory. This procedure is not advised for the child who has more
than mild aortic insufficiency or the young adult who has isolated calcific aortic stenosis
because in these individuals, it may produce marked aortic insufficiency. Although mild
insufficiency is well tolerated, acute severe aortic insufficiency can result in left ventricular
dysfunction and congestive heart failure. For this reason, the procedure of choice in the
adolescent and adult is surgical intervention upon the valve. Occasionally, the stenotic valve can
be palliated surgically, but the repair is rarely durable. In addition, many individuals have not only
stenosis but also insufficiency, necessitating valve replacement.
The most common valves on the market today are mechanical and bioprosthetic valves. The
advantage of the mechanical valve resides in its durability, but its placement necessitates lifelong
anticoagulation with warfarin. The advantage of the bioprosthetic valve is that no anticoagulation
regimen is necessary, but the valve may deteriorate and require replacement after 5 to 10 years.
Therefore, the conversation regarding valve type is complex, and lifestyle choices may play an
important role in decision-making.
Previously, children who had aortic valve disease required antibiotic prophylaxis to reduce
the risk for bacterial endocarditis. However, the most recent consensus guidelines suggest that
prophylaxis does not reduce risk in this population, and appropriate counseling regarding the
benefits of good dental hygiene is the most effective strategy.
Data show that first-degree relatives of those who have congenital heart disease are at
increased risk for the identification of a cardiac anomaly. The overall risk is between 2% and 3%
for all forms of congenital cardiac disease. The inheritance risk of aortic valve disease is well
described. In most reports, up to 10% of first-degree relatives of an individual who has a
bicuspid aortic valve also show evidence of a congenital anomaly of the aortic valve. The
severity of disease in one individual does not predict the severity in others found to have valve
pathology. Therefore, patients who have aortic valve disease and are in their childbearing years

Copyright 2010 by the American Academy of Pediatrics page 481


2011 PREP SA on CD-ROM

should be offered genetic counseling to provide the most accurate information regarding
inheritance risk and fetal echocardiography to determine if significant valve disease is detected.

Suggested reading:

Gerber MA, Baltimore RS, Eaton CB, et al. Prevention of rheumatic fever and diagnosis and
treatment of acute streptococcal pharyngitis: a scientific statement from the American Heart
Association Rheumatic Fever, Endocarditis, and Kawasaki Disease Committee of the Council on
Cardiovascular Disease in the Young, the Interdisciplinary Council on Functional Genomics and
Translational Biology, and the Interdisciplinary Council on Quality of Care and Outcomes
Research: endorsed by the American Academy of Pediatrics. Circulation. 2009;119:1541-1551.
DOI: 10.1161/CIRCULATIONAHA.109.191959. Available at:
http://circ.ahajournals.org/cgi/content/full/119/11/1541

Pham P, Silberbach M. What's new in pediatric cardiology. Pediatr Rev. 2004;25:381-387. DOI:
10.1542/10.1542/pir.25-11-381. Available at:
http://pedsinreview.aappublications.org/cgi/content/full/25/11/381

Copyright 2010 by the American Academy of Pediatrics page 482


2011 PREP SA on CD-ROM

Critique: 149

(Courtesy of P Lynch)
In a bicuspid aortic valve, one of the commissures fails to form (arrow), resulting in two rather
than three leaflets.

Copyright 2010 by the American Academy of Pediatrics page 483


2011 PREP SA on CD-ROM

Question: 150

A 16-year-old boy presents with diffuse muscle pain and dark urine after a summer football
practice. On physical examination, he appears uncomfortable, and his arm and leg muscles are
painful to palpation.

Of the following, the MOST likely cause of this boys findings is

A. bacterial abscess of the quadriceps

B. genetic mitochondrial disorder

C. inflammatory dermatomyositis

D. limb girdle muscular dystrophy

E. myositis due to influenza

Copyright 2010 by the American Academy of Pediatrics page 484


2011 PREP SA on CD-ROM

Critique: 150 Preferred Response: B

Most children presenting with diffuse acute muscle pain that is painful to palpation have
myositis, with inflammation of the muscles, which usually is related to a viral illness such as
influenza or upper respiratory tract infections and is not life-threatening. Blood testing may
reveal elevated creatine phosphokinase, but such testing usually is not necessary. Other
causes of myositis include inflammatory diseases such as dermatomyositis, infections of the
muscle, direct muscle trauma, and adverse effects of certain medications such as cholesterol-
lowering statins and zidovudine.
However, two features described for the adolescent in the vignette warrant additional
concern. First, the event occurred after a football practice, where exercise often is extreme.
Some rare mitochondrial conditions affecting energy metabolism tend to present in precisely this
setting. Second, there is the darkening of the urine. Myoglobinuria indicates rhabdomyolysis or
breakdown of muscle fibers. Accordingly, the adolescent in the vignette most likely has a genetic
disorder affecting mitochondrial function (carnitine palmitoyl transferase II deficiency).
A bacterial abscess of the quadriceps would not produce diffuse pain of the arms and legs.
Inflammatory dermatomyositis presents more indolently, with proximal weakness, pain, and
characteristic skin findings. Neither dermatomyositis nor limb girdle muscular dystrophy presents
acutely with rhabdomyolysis. Influenza can present with myositis, but this adolescent has been
at summer football practice and had no signs of systemic illness prior to this dramatic
presentation.

Suggested reading:

Bhagtani H, Love E, Baci G, et al. Index of suspicion: case 2. Pediatr Rev. 2007;28:225-229. DOI:
10.1542/10.1542/pir.28-6-225. Available at:
http://pedsinreview.aappublications.org/cgi/content/full/28/6/225

Compeyrot-Lacassagne S, Feldman BM. Inflammatory myopathies in children. Pediatr Clin North


Am. 2005;52:493-520. DOI: 10.1016/j.pcl.2005.01.004. Abstract available at:
http://www.ncbi.nlm.nih.gov/pubmed/15820377

Khan MA, Das BB. Index of suspicion: case 2. Pediatr Rev. 2002;23:249-253. DOI:
10.1542/10.1542/pir.23-7-249. Available at:
http://pedsinreview.aappublications.org/cgi/content/full/23/7/249

Koenig MK. Presentation and diagnosis of mitochondrial disorders in children. Pediatr Neurol.
2008;38:305-313. DOI: 10.1016/j.pediatrneurol.2007.12.001. Abstract available at:
http://www.ncbi.nlm.nih.gov/pubmed/18410845

Longo N, Amat di San Filippo C, Pasquali M. Disorders of carnitine transport and the carnitine
cycle. Am J Med Genet C Semin Med Genet. 2006;142C:77-85. DOI: 10.1002/ajmg.c.30087.
Available at: http://www.ncbi.nlm.nih.gov/pmc/articles/PMC2557099/?tool=pubmed

Copyright 2010 by the American Academy of Pediatrics page 485


2011 PREP SA on CD-ROM

Mannix R, Tan ML, Wright R, Baskin M. Acute pediatric rhabdomyolysis: causes and rates of
renal failure. Pediatrics. 2006;118:2119-2125. DOI: 10.1542/peds.2006-1352. Available at:
http://pediatrics.aappublications.org/cgi/content/full/118/5/2119

Copyright 2010 by the American Academy of Pediatrics page 486


2011 PREP SA on CD-ROM

Question: 151

You are examining a baby girl who was brought to the clinic by her foster mother for her 2-week
health supervision visit. You do not have access to her prenatal or newborn records, but the
foster parent tells you that the infants mother is a polydrug abuser who was incarcerated shortly
before delivery. No information is available on the babys father. On physical examination, the
babys weight, length, and head circumference are less than the 5th percentiles. She is irritable
and tremulous, and she has short palpebral fissures, midface hypoplasia, a narrow upper
vermilion border, and hirsutism (Item Q151).

Of the following, this babys features are MOST likely due to prenatal exposure to

A. alcohol

B. barbiturates

C. cocaine

D. methamphetamine

E. phencyclidine (PCP)

Copyright 2010 by the American Academy of Pediatrics page 487


2011 PREP SA on CD-ROM

Question: 151

(Courtesy of M Rimsza)
Narrow vermilion border and hirsutism of the forehead, as described for the infant in the vignette.

Copyright 2010 by the American Academy of Pediatrics page 488


2011 PREP SA on CD-ROM

Critique: 151 Preferred Response: A

The infant described in the vignette has physical features that are consistent with fetal
alcohol syndrome (FAS); that is, evidence for poor somatic growth, deficient brain growth
(microcephaly), and characteristic facial dysmorphisms (short palpebral fissures and narrow
upper vermilion). In early infancy, hirsutism and tremulousness are common, but these conditions
typically are self-limiting and may no longer be evident by 6 months of age.
The offspring of women who have epilepsy and take barbiturates such as phenobarbital
during pregnancy may have a two- to threefold risk for birth defects that include cardiovascular
defects, oral clefts, and genital malformations. This risk may be greater when phenobarbital is
taken with another anticonvulsant, such as phenytoin. Although the unusual features associated
with prenatal exposure to antiepileptic drugs vary with the drug ingested, they do overlap to
some degree, and the term "fetal anticonvulsant syndrome" sometimes is used to describe them.
Prenatal cocaine exposure has received a great deal of attention in the medical literature,
but there may have been a bias toward reporting unfavorable outcomes. A number of early
claims regarding resultant anomalies have dissipated. Maternal use of cocaine during pregnancy
is associated with an increased risk for placental abruption and vascular disruption. The extent
to which prenatal cocaine exposure influences cognitive function remains unclear. One of the
confounding variables in studying pregnancy outcomes associated with cocaine is that the drug
frequently is used concomitantly with other substances of abuse.
There are not sufficient data on the effects of prenatal exposures to methamphetamine and
phencyclidine at this time to draw conclusions about their impact on physical and neurological
development. Some early evidence suggests that both drugs may cause neurodevelopmental
abnormalities.
As with all potential teratogens, it is important to inquire about the amount of the substance
to which the mother has been exposed, the manner in which it was delivered, the timing of
exposure(s) during the pregnancy, and associated adverse outcomes for the mother.
Unfortunately, these questions are not asked regularly. This is especially problematic when the
exposed baby is raised outside of the biological home and no one is available to answer such
questions.

Suggested reading:

Ackerman JP, Riggins T, Black MM. A review of the effects of prenatal cocaine exposure among
school-aged children. Pediatrics. 2010;125:554-565. DOI: 10.1542/peds.2009-0637. Available at:
http://pediatrics.aappublications.org/cgi/content/full/125/3/554?nfstatus=200&nfstatusdescription
=SUCCESS%3A+Login+worked

Gallagher RC, Kingham K, Hoyme HE. Fetal anticonveulsant syndrome. In: Cassidy SB, Allanson
JE, eds. Management of Genetic Syndromes. 2nd ed. Hoboken, NJ: John Wiley & Sons;
2005:239-250

Hoyme HE, May PA, Kalberg WO, et al. A practical approach to diagnosis of fetal alcohol
spectrum disorders: clarification of the 1996 Institute of Medicine criteria. Pediatrics.

Copyright 2010 by the American Academy of Pediatrics page 489


2011 PREP SA on CD-ROM

2005;115:39-47. DOI: 10.1542/peds.2004-0259. Available at:


http://pediatrics.aappublications.org/cgi/content/full/115/1/39

Phenobarbital, Cocaine, Methamphetamine, Phencyclidine. REPROTOX. 2009.

Phenobarbital, Cocaine, Methamphetamine, Phencyclidine. TERIS. 2009.

Copyright 2010 by the American Academy of Pediatrics page 490


2011 PREP SA on CD-ROM

Question: 152

An 18-year-old boy, who has a past medical history of poor school performance, behavior
problems, and one episode of visual hallucinations, is brought to the emergency department
because of incoherent speech and agitation. On physical examination, you note that the
adolescent is staring into space and has occasional garbled speech. His heart rate is 125
beats/min, temperature is 37.0C, and blood pressure is 125/82 mm Hg. His pupils are 5 mm
bilaterally. His skin is flushed and sweaty, he has no needle track marks, and his abdomen is
slightly distended. His reflexes are hyperactive, but there are no focal neurologic findings. The
rest of his examination findings are unremarkable.

Of the following, the MOST likely cause of this boys findings is

A. anticholinergic intoxication

B. depression with psychotic features

C. early-onset schizophrenia

D. marijuana use

E. phencyclidine (PCP) use

Copyright 2010 by the American Academy of Pediatrics page 491


2011 PREP SA on CD-ROM

Critique: 152 Preferred Response: E

The boy described in the vignette is exhibiting signs of the use of the dissociative drug
phencyclidine (PCP), including blank staring, incoherent speech, tachycardia, sweating, and
muscle rigidity. Characteristic symptoms at higher doses are nystagmus, which may be vertical,
rotary, or horizontal; hallucinations; seizures; coma; and death. The effects of high doses of PCP
may mimic schizophrenia, with disordered speech, delusions, hallucinations, disordered thinking,
and catatonia. PCP sometimes is considered a hallucinogen because it has some of the same
effects, altering a person's perceptions, sensations, thinking, self-awareness, and emotions.
However, PCP does not fit easily into any one drug category because it also can relieve pain or
act as a stimulant.
Anticholinergic drugs may cause hallucination and tachycardia but typically are associated
with elevations in body temperature, dryness of the skin, and mydriasis, features not exhibited
by the boy in the vignette. Marijuana may cause tachycardia, redness of the eyes, and acute
anxiety but not the other features noted for the boy in the vignette.
Street drugs may unmask a latent mental condition. However, underlying depression with
psychosis or schizophrenia should not result in the physical symptoms noted for this boy. To
make a diagnosis of depression or schizophrenia, it is important to determine that the symptoms
preceded the onset of the substance or medication use, that they persist for 1 month after
cessation of acute withdrawal or severe intoxication, that they are substantially in excess of
what would be expected given the type or amount of substance used or the duration of use, or
that other evidence suggests the existence of an independent non-substance-induced
psychotic disorder (eg, a history of recurrent non-substance-related episodes). Of note, the
hallucinations seen with these conditions, compared with those seen with substance use, are
more likely auditory than visual. Finally, a positive family history of either disorder is helpful in
making the diagnosis.

Suggested reading:

American Psychiatric Association. Diagnostic and Statistical Manual of Mental Disorders,


Fourth Edition, Text Revision. Washington, DC: American Psychiatric Association; 2000:163-165

Eldridge DL, Hillenbrand K,Serwint JR. In brief: hallucinogens. Pediatr Rev. 2006;27:314-315.
DOI: 10.1542/10.1542/pir.27-8-314. Available at:
http://pedsinreview.aappublications.org/cgi/content/full/27/8/314

Groisman AE, Seminatore ML, Cheng TL. In brief: children and adolescents who have
schizophrenia. Pediatr Rev. 2003;24:356-357. DOI: 10.1542/10.1542/pir.24-10-356. Available at:
http://pedsinreview.aappublications.org/cgi/content/full/24/10/356

National Institute of Mental Health. Schizophrenia. 2010. Available at:


http://www.nimh.nih.gov/health/topics/schizophrenia/index.shtml

National Institute on Drug Abuse. PCP/phencyclidine. Available at:

Copyright 2010 by the American Academy of Pediatrics page 492


2011 PREP SA on CD-ROM

http://www.nida.nih.gov/DrugPages/PCP.html

Sanchez-Samper X, Knight JR. Drug abuse by adolescents: general considerations. Pediatr Rev.
2009;30:83-93. DOI: 10.1542/10.1542/pir.30-3-83. Available at:
http://pedsinreview.aappublications.org/cgi/content/full/30/3/83

Copyright 2010 by the American Academy of Pediatrics page 493


2011 PREP SA on CD-ROM

Question: 153

You are evaluating a 14-year-old boy who has a 1-month history of fatigue. For the past 2
weeks, he has had distended neck veins, cough, orthopnea, and dyspnea on exertion. His
temperature is 37.0C, heart rate is 120 beats/min, respiratory rate is 35 breaths/min, and blood
pressure is 120/80 mm Hg. On physical examination, he has obvious facial swelling and jugular
venous distention. His trachea is not deviated. His lungs demonstrate an expiratory wheeze on
the right side with mildly decreased aeration. His liver is 1 cm below the right costal margin.
Based on his chest radiograph (Item Q153), you order computed tomography scan of his chest.
He states that he feels anxious, and his parents ask if he can be sedated for the study.

Of the following, the MOST immediate concern regarding the administration of sedation to the
patient is the possibility of

A. airway compromise

B. allergic reaction

C. cardiovascular collapse

D. dysrhythmias

E. pulmonary edema

Copyright 2010 by the American Academy of Pediatrics page 494


2011 PREP SA on CD-ROM

Question: 153

(Courtesy of D Krowchuk)
Chest radiograph, as described for the patient described in the vignette.

Copyright 2010 by the American Academy of Pediatrics page 495


2011 PREP SA on CD-ROM

Critique: 153 Preferred Response: A

An estimated 40% of pediatric cancer patients need intensive care at some point during their
illness and represent 3% of all pediatric intensive care unit admissions. Acute respiratory failure
due to a new-onset chest mass is uncommon but must be recognized quickly and managed
appropriately to prevent fatal consequences. Clinical presentation can vary, depending on the
degree of vascular and airway compression by the mass, but 60% of children present with
respiratory symptoms that include dyspnea, stridor, cough, and orthopnea. As the mass
enlarges, frank respiratory failure can develop. Superior vena cava compression results in
head, neck, and upper extremity edema. Rarely, patients can present with cardiac compression
resulting in syncope, dysrhythmias, and death. The boy described in the vignette demonstrates
signs of both airway and superior vena cava compression, which were demonstrated on
computed tomography scan of the chest (Item C153).
Hodgkin disease and neuroblastoma are the most common chest masses seen in the
pediatric population, but a variety of benign or malignant masses may occur. Initial management is
independent of the cause of the mass and consists of prompt attention to the potential for
airway compromise, which can occur in up to 20% of patients. Sedation (such as would be
given for imaging procedures) increases the risk for respiratory failure due to a combination of
mechanisms, including decreased upper airway muscle tone, loss of protective airway reflexes,
decreased lung volumes, and relaxation of bronchial smooth muscle with increased external
compression. For patients who present with signs of respiratory compromise, such as the boy
in the vignette, sedation should be undertaken by experienced personnel, with careful
consideration of intubation.
Cardiovascular compromise such as hypotension, dysrhythmias, and sudden death can
occur, but airway compromise is the most common emergency. Allergic reactions can be seen
with sedative administration, and a history of prior adverse events with sedative administration
should be obtained. However, true allergic reactions are relatively rare. Pulmonary edema can
result from forced inspiration against an obstructed airway, resulting in transudation of fluid from
pulmonary capillaries to the interstitium, but it is a relatively rare event.

Suggested reading:

Gangadharan SP. Evaluation of mediastinal masses. UpToDate Online 17.3. 2009. Available at:
http://www.utdol.com/online/content/topic.do?topicKey=pulm_dxs/4387

Tamburro R, Barfield RC, Metzger ML, Geiger TL, Call S, Gow KW. Care of the oncology patient
in the pediatric intensive care unit. In: Wheeler DS, Wong HR, Shanley TP, eds. Pediatric Critical
Care Medicine: Basic Science and Clinical Evidence. New York, NY: Springer-Verlag;
2007:1332-1354

Copyright 2010 by the American Academy of Pediatrics page 496


2011 PREP SA on CD-ROM

Critique: 153

(Courtesy of D Krowchuk)
Computed tomography scan of the chest for the patient described in the vignette. There is a large
anterior mediastinal mass (arrows) compressing the right main bronchus and superior vena cava
(SVC). There also is a right pleural effusion. The boy was diagnosed as having a lymphoma.

Copyright 2010 by the American Academy of Pediatrics page 497


2011 PREP SA on CD-ROM

Question: 154

A mother brings her 3-year-old daughter to your office for evaluation of a lump in the child's neck.
On physical examination, you note a 1.0x1.5-cm ovoid mass that seems to move with swallowing
and is centrally located just above the thyroid (Item Q154). It is not red, inflamed, or painful, but it
is firm.

Of the following, the MOST appropriate next step is

A. blood test for free thyroxine and thyroid-stimulating hormone

B. computed tomography scan of the neck region

C. magnetic resonance imaging of the neck region

D. oncology consultation

E. surgery consultation

Copyright 2010 by the American Academy of Pediatrics page 498


2011 PREP SA on CD-ROM

Question: 154

(Courtesy of M Rimsza)
Mass, as described for the girl in the vignette.

Copyright 2010 by the American Academy of Pediatrics page 499


2011 PREP SA on CD-ROM

Critique: 154 Preferred Response: E

An ovoid mass that moves with swallowing and is centrally located just above the thyroid
almost always is a thyroglossal cyst. Such cysts result when a tract is left during embryologic
descent of the thyroid anlage from the base of the tongue. They generally are not associated
with hypothyroidism, and, therefore, assessment of free thyroxine and thyroid-stimulating
hormone is not necessary. Occasionally, the thyroid itself is located within a cyst, and extirpative
surgery renders an individual hypothyroid. Ultrasonography or a radioactive scan using
technetium often is used to confirm the location of the thyroid before surgery, but computed
tomography scan or magnetic resonance imaging is unnecessary. Surgical removal of the mass
usually is indicated, particularly if there have been episodes of infection. Oncology consultation
is unnecessary, although occasional papillary carcinomas have been reported in these cysts.

Suggested reading:

Chang YW, Hong HS, Choi DL. Sonography of the pediatric thyroid: a pictorial essay. J Clin
Ultrasound. 2009;37:149-157. DOI: 10.1002/jcu.20555. Abstract available at:
http://www.ncbi.nlm.nih.gov/pubmed/19184992

Emerick K, Lin D. Differential diagnosis of a neck mass. UpToDate Online 17.3. 2009. Available
at:
http://www.uptodateonline.com/online/content/topic.do?topicKey=prim_ent/8791&selectedTitle=2
%7E15&source=search_result

Hirshoren N, Neuman T, Udassin R, Elidan J, Weinberger JM. The imperative of the Sistrunk
operation: review of 160 thyroglossal tract remnant operations. Otolaryngol Head Neck Surg.
2009;140:338-342. Abstract available at: http://www.ncbi.nlm.nih.gov/pubmed/19248939

Copyright 2010 by the American Academy of Pediatrics page 500


2011 PREP SA on CD-ROM

Question: 155

An 8-year-old boy has an above-average intelligence quotient, but he is struggling in school and
consistently brings home failing grades. He is generally well behaved, but he gets angry with the
poor grades. He enjoys being with his friends and is active in after-school activities.

Of the following, the MOST appropriate intervention is to

A. have the parents set up a behavioral chart to encourage him to improve his grades

B. reassure the parents that he is smart and schedule a follow-up appointment in 6 months

C. refer him for psychoeducational evaluation

D. refer him to a psychiatrist

E. tell the parents to punish him if he continues to fail

Copyright 2010 by the American Academy of Pediatrics page 501


2011 PREP SA on CD-ROM

Critique: 155 Preferred Response: C

The boy described in the vignette has above-average intelligence but is having academic
difficulties, warranting psychoeducational evaluation for a learning disability. The reported
prevalence of dyslexia (specific reading disability) ranges from 5% to 17% in the general
population. The rate of dyscalculia (specific learning disability in mathematics) is 4% to 6%.
Individuals may have learning difficulties that occur independent of intelligence. Frequently,
individuals who have high intelligence are not identified as having a learning issue or are
considered to lack motivation. Such children who have above-average intelligence may have
uneven profiles when evaluating their cognitive, social, and emotional development.
The boy in the vignette should not be expected to improve spontaneously. The academic
difficulty is not within his control and would not be remediated via discipline or use of a behavior
chart. He is not showing evidence of a primary mood disorder for which a referral to a
psychiatrist would be indicated. Additional causes of school failure are attention-
deficit/hyperactivity disorder, sensory deficits (eg, diminished visual or auditory acuity),
disordered sleep, or epilepsy. An older child should be evaluated for use of illicit drugs or
alcohol.

Suggested reading:

Committee on Children with Disabilities. The pediatricians role in development and implementation
of an Individual Education Plan (IEP) and Individual Family Service Plan (IFSP). Pediatrics.
1999;104:124-127. Available at: http://pediatrics.aappublications.org/cgi/content/full/104/1/124

Lambros KM, Leslie LK. Management of the child with a learning disorder. Pediatr Ann.
2005;34:275-287. Abstract available at: http://www.ncbi.nlm.nih.gov/pubmed/15871432

Shaywitz SE, Shaywitz BA. Dyslexia (specific reading disability). Pediatr Rev. 2003;24:147-153.
DOI: 10.1542/10.1542/pir.24-5-147. Available at:
http://pedsinreview.aappublications.org/cgi/content/full/24/5/147

Copyright 2010 by the American Academy of Pediatrics page 502


2011 PREP SA on CD-ROM

Question: 156

A 6-year-old boy who has acute lymphocytic leukemia and is receiving maintenance
chemotherapy presents with a 2-day history of a temperature to 39.4C. He has a Broviac
catheter in place for venous access. On physical examination, he is alert, his vital signs are
stable, and he has no apparent focus of infection. His white blood cell count is 0.1x103/mL
(0.1x109/L) with 0% neutrophils.

Of the following, the MOST appropriate empiric antibiotic regimen, pending culture results, is

A. amphotericin B

B. cefazolin and gentamicin

C. ciprofloxacin and nafcillin

D. meropenem

E. vancomycin and gentamicin

Copyright 2010 by the American Academy of Pediatrics page 503


2011 PREP SA on CD-ROM

Critique: 156 Preferred Response: D

Children who have cancer, are receiving chemotherapy, and become febrile while
neutropenic with counts of 0.5x103/mcL (0.5x109/L) or less are at markedly increased risk of
bacteremia. Vascular access devices (catheters) and the alimentary canal may provide portals
of entry for infectious organisms, even in the absence of an apparent focus of infection on
examination, such as reported for the boy in the vignette. Such infections may be caused by a
wide variety of bacteria, including staphylococci and enteric gram-negative organisms. A
number of broad-spectrum antibiotic regimens may be considered for the well-appearing, stable
child who has a short duration of fever and neutropenia, pending the results of cultures of blood
and urine. Monotherapy with meropenem, cefepime, or ceftazidime has been shown to be
effective empiric therapy. Although such regimens do not afford coverage for methicillin-
resistant Staphylococcus aureus and coagulase-negative staphylococci, vancomycin should be
withheld in the well-appearing, low-risk patient pending cultures in an effort to prevent
emergence of vancomycin-resistant organisms such as vancomycin-resistant enterococci. If the
child appears ill or unstable at the onset of therapy or cultures so dictate, vancomycin therapy
can be included in the treatment regimen.
Two-drug regimens also have been used in initial treatment of fever and neutropenia, but
cefazolin and gentamicin or vancomycin and gentamicin do not provide adequate coverage of
gram-negative pathogens. Examples of appropriate combination therapy without vancomycin for
fever and neutropenia include piperacillin-tazobactam and gentamicin or the addition of an
aminoglycoside to the previously cited monotherapy agents. Ciprofloxacin provides broad-
spectrum gram-negative pathogen coverage, but fluoroquinolones are not indicated routinely for
children younger than 18 years of age.
Fungal infections may cause fever in the presence of neutropenia, but these generally
occur in patients who have received prior broad-spectrum antibiotics. Persistent fever with
neutropenia when the patient is receiving antibiotics and has negative cultures may warrant the
addition of empiric antifungal therapy such as amphotericin B or fluconazole.

Suggested reading:

Fishman JA. Infection in solid-organ transplant recipients. N Engl J Med. 2007;357:2601-2614.


Extract available at: http://content.nejm.org/cgi/content/extract/357/25/2601

Hughes WT, Armstrong D, Bodey GP, et al. 2002 guidelines for the use of antimicrobial agents in
neutropenic patients with cancer. Clin Infect Dis. 2002;34:730-751. DOI: 10.1086/339215.
Available at: http://www.journals.uchicago.edu/doi/full/10.1086/339215

Copyright 2010 by the American Academy of Pediatrics page 504


2011 PREP SA on CD-ROM

Question: 157

A 13-year-old girl presents with a 2-day history of fever, sore throat, and a rash that began on
her arms and legs and spread to her chest and back. Physical examination reveals pharyngeal
exudate; bilateral cervical adenopathy; and a "sandpapery" rash over her arms (Item Q157A),
legs, and trunk. A rapid diagnostic test for group A Streptococcus yields negative results. At 48
hours, a throat culture is growing small colonies with narrow bands of hemolysis on sheep blood
agar.

Of the following, the MOST appropriate antibiotic for treating this patient is

A. ceftazidime

B. erythromycin

C. penicillin

D. tetracycline

E. trimethoprim-sulfamethoxazole

Copyright 2010 by the American Academy of Pediatrics page 505


2011 PREP SA on CD-ROM

Question: 157

(Courtesy of D Krowchuk)
Rash, as described for the girl in the vignette.

Copyright 2010 by the American Academy of Pediatrics page 506


2011 PREP SA on CD-ROM

Critique: 157 Preferred Response: B

The girl described in the vignette has acute pharyngitis caused by Arcanobacterium
(formerly Corynebacterium) haemolyticum, a gram-positive bacillus that grows slowly as small
colonies with narrow bands of hemolysis on blood-enriched agar. Colony detection often
requires incubation for at least 48 to 72 hours and is enhanced by culture on rabbit or human
blood agar rather than on the more commonly used sheep blood agar. Incubation in 5% carbon
dioxide also assists in colony growth.
Erythromycin is the drug of choice for treating pharyngitis caused by A haemolyticum,
although no controlled trials have been conducted of antimicrobial therapy for pharyngitis
caused by this pathogen. In vitro, the organism also is susceptible to azithromycin, clindamycin,
chloramphenicol, and gentamicin. Tetracycline resistance has been observed in up to 30% of
isolates tested. Susceptibility to penicillin is variable, and resistance to trimethoprim-
sulfamethoxazole is common. Ceftazidime is appropriate for the treatment of invasive infections
caused by Pseudomonas.
Except for the absence of palatal petechiae and strawberry tongue, the clinical illness
caused by A haemolyticum is indistinguishable from that caused by group A Streptococcus and
is manifested by fever, pharyngeal exudate, and cervical lymphadenopathy. A scarlatiniform or
maculopapular and often pruritic rash is present in about 50% of cases and usually begins on
the extensor surfaces of the extremities, spreading centripetally to the trunk but sparing the
palms, soles, and face. The illness is most common in teenagers and young adults and accounts
for 0.5% to 3% of all cases of acute pharyngitis. Respiratory tract infections that can mimic
diphtheria and an array of skin and soft-tissue infections also have been reported with A
haemolyticum. Severe infections such as septicemia, pneumonia, brain abscess, meningitis,
endocarditis, and osteomyelitis occur almost exclusively in immunocompromised persons.

As a result of reviewing this information, do you intend to make a change in practice


to provide better patient care?
Yes No

Suggested reading:

American Academy of Pediatrics. Arcanobacterium haemolyticum Infections. In: Pickering LK,


Baker CJ, Kimberlin DW, Long SS, eds. Red Book: 2009 Report of the Committee on Infectious
Diseases. 28th ed. Elk Grove Village, Ill: American Academy of Pediatrics; 2009:220-221

Ching N. Arcanobacterium haemolyticum. In: Feigin RD, Cherry JD, Demmler-Harrison GJ, Kaplan
SL, eds. Feigin & Cherrys Textbook of Pediatric Infectious Diseases. 6th ed. Philadelphia, Pa:
Saunders Elsevier; 2009:1413-1417

Copyright 2010 by the American Academy of Pediatrics page 507


2011 PREP SA on CD-ROM

Question: 158

A 5-year-old boy has had 2 days of fever, crampy abdominal pain, and diarrhea. In the past 12
hours, his diarrhea has progressed from nonbloody to bloody. You are concerned about the
possibility that he is at risk for developing hemolytic-uremic syndrome (HUS) and send blood for a
complete blood count and a stool sample for culture.

Of the following, this boys potential risk for the development of HUS would be decreased if

A. antibiotics are prescribed

B. antimotility agents are prescribed

C. he has been vomiting

D. he is febrile

E. his white blood cell count is less than 13.0x103/mcL (13.0x109/L)

Copyright 2010 by the American Academy of Pediatrics page 508


2011 PREP SA on CD-ROM

Critique: 158 Preferred Response: E

Hemolytic-uremic syndrome (HUS) is characterized by the triad of hemolytic anemia,


thrombocytopenia, and acute renal failure. Most pediatric cases (90%) are preceded by a
diarrheal illness and are designated D+. Shiga toxin-producing Escherichia coli (STEC) is
responsible for more than two thirds of cases of D+ HUS. Furthermore, the strain of STEC most
often responsible for HUS is O157:H7. Up to 9% of children infected with E coli O157:H7
develop clinical HUS; most affected children are younger than 5 years of age. Children infected
with this strain of E coli often present clinically with watery diarrhea for the first few days,
followed later by bloody diarrhea, as described for the boy in the vignette. Shiga toxin produced
by the bacterium is believed to cross into the bloodstream, resulting in endothelial injury, which
initiates microangiopathic hemolytic anemia with thrombocytopenia and subsequent renal injury
from the thrombotic microangiopathy.
Whether to administer antibiotics to children who have E coli O157:H7-associated diarrhea
remains controversial, but most experts do not initiate antimicrobial therapy because no benefit
has been proven. Studies have failed to confirm that children who have E coli O157:H7-
associated diarrhea and are treated with antibiotics have a greater risk of developing HUS.
Antimotility agents delay gastrointestinal transit time, thereby prolonging the exposure to
STEC, which prolongs bloody diarrhea and increases the patient risk for development of HUS.
Undercooked hamburger is a common source of STEC. This organism is part of the intestinal
tracts of cattle, and the ground beef used in hamburgers harbors STEC that can be pathogenic if
the meat is not cooked sufficiently to eradicate the viable bacteria at the center of the
hamburger, which is the last to cook. Other sources include unpasteurized apple cider and cow
and goat milk.
Prognostic features for the development of HUS have been gathered by researchers in
Washington state, who studied more than 500 cases of STEC during a 1993 outbreak from
undercooked hamburger. In their cohort, HUS developed in 14% and was associated in a
multivariate analysis with: 1) the use of antimotility agents in the first 3 days of illness, 2)
vomiting in children younger than 5.5 years of age, and 3) a white blood cell count greater than
13.0x103/mcL (13.0x109/L) in the first 3 days of illness. The risk of developing HUS was
increased sevenfold when the white blood cell count was greater than 13.0x103/mcL
(13.0x109/L). Fever was not a statistically significant factor.

Suggested reading:

Bell BP, Griffin PM, Lozano P, Christie DL, Kobayashi JM, Tarr PI. Predictors of hemolytic uremic
syndrome in children during a large outbreak of Escherichia coli O157:H7 infections. Pediatrics.
1997;100:e12. Available at: http://pediatrics.aappublications.org/cgi/content/full/100/1/e12

Corrigan JJ Jr, Boineau FG. Hemolytic-uremic syndrome. Pediatr Rev. 2001;22:365-369. DOI:
10.1542/10.1542/pir.22-11-365. Available at:
http://pedsinreview.aappublications.org/cgi/content/full/22/11/365

Safdar N, Said A, Gangnon RE, Maki DG. Risk of hemolytic uremic syndrome after antibiotic

Copyright 2010 by the American Academy of Pediatrics page 509


2011 PREP SA on CD-ROM

treatment of Escherichia coli O157:H7 enteritis: a meta-analysis. JAMA. 2002;288:996-1001


Available at: http://jama.ama-assn.org/cgi/content/full/288/8/996

Tarr PI, Neill MA. Escherichia coli O157:H7. Gastroenterol Clin North Am. 2001;30:735-751.
Abstract available at: http://www.ncbi.nlm.nih.gov/pubmed/11586555

Copyright 2010 by the American Academy of Pediatrics page 510


2011 PREP SA on CD-ROM

Question: 159

You are meeting with a group of medical students to discuss the significance of allergic rhinitis
and asthma morbidity. You review how allergens not only can result in allergic rhinitis but can lead
to asthma exacerbations, missed school, and hospitalization in sensitized individuals, particularly
inner-city children.

Of the following, the MOST likely allergen responsible for asthma development and exacerbations
for inner-city children is

A. cat dander

B. cockroaches

C. dog dander

D. dust mites

E. mouse urine

Copyright 2010 by the American Academy of Pediatrics page 511


2011 PREP SA on CD-ROM

Critique: 159 Preferred Response: B

All children who have asthma should be evaluated for allergic rhinitis because 60% to 80%
of children who have persistent asthma have coexisting allergic rhinitis. Although allergic rhinitis
may be limited to upper airway symptoms (eg, nasal congestion, rhinorrhea, sneezing, ocular
pruritus), allergic rhinitis is a common trigger for asthma. Cockroach is probably the most
important allergen for inner-city children. One study of a cohort of almost 500 children ages 4 to
11 years who had asthma demonstrated that children sensitized (ie, positive immunoglobulin E
skin test) and exposed to high concentrations of cockroach allergen were 3.3 times more likely
to be hospitalized for asthma, miss more school due to asthma, and have more unscheduled
asthma visits.
Cat exposure and sensitization can result in the development in allergic rhinitis and asthma,
although such development is less common than with cockroach exposure in inner-city children.
Dog dander shares homology with cat dander, but it triggers symptoms less often. Dust mite
allergen is common, particularly in cities in which humidity is consistently greater than 50%.
When compared with cockroach, dust mite allergen is less likely to trigger asthma in inner-city
children. Recent studies have suggested that mouse allergen may be one of the most common
allergens in the inner city, but the association of mouse exposure and subsequent sensitization
and asthma exacerbations is less well documented than are exacerbations associated with
cockroach exposure.

Suggested reading:

Matsui EC, Eggleston PA, Buckley TJ, et al. Household mouse allergen exposure and asthma
morbidity in inner-city preschool children. Ann Allergy Asthma Immunol. 2006;97:514-520.
Abstract available at: http://www.ncbi.nlm.nih.gov/pubmed/17069107

Rosenstreich DL, Eggleston P, Kattan M, et al. The role of cockroach allergy and exposure to
cockroach allergen in causing morbidity among inner-city children with asthma. N Engl J Med.
1997;336:1356-1363. Available at: http://content.nejm.org/cgi/content/full/336/19/1356

Wilson J, Dixon SL, Breysse P, et al. Housing and allergens: a pooled analysis of nine US
studies. Environ Res. 2010;110:189-198. DOI: 10.1016/j.envres.2009.10.007. Abstract available
at: http://www.ncbi.nlm.nih.gov/pubmed/19939359

Copyright 2010 by the American Academy of Pediatrics page 512


2011 PREP SA on CD-ROM

Question: 160

You are seeing a 5-year-old boy who complains of right arm pain after a fall while jumping on the
bed. He is holding his right arm against his body and is unwilling to move it. He has no deformity or
swelling of his right arm, but he does have a tender swelling in his mid-clavicle. You obtain a
radiograph (Item Q160).

Of the following, you are MOST likely to advise the parents that

A. complications include ulnar nerve palsy

B. he should be tested for osteogenesis imperfecta

C. his right arm should be placed in a sling for comfort

D. surgical reduction will be needed

E. the injury typically heals in 8 to 10 weeks

Copyright 2010 by the American Academy of Pediatrics page 513


2011 PREP SA on CD-ROM

Question: 160

(Courtesy of M Wright)
Radiographic findings, as described for the boy in the vignette.

Copyright 2010 by the American Academy of Pediatrics page 514


2011 PREP SA on CD-ROM

Critique: 160 Preferred Response: C

The patient described in the vignette has sustained a midshaft clavicle fracture, as
evidenced by the tender swelling over the clavicle and pain with arm movement. Most of these
injuries can be diagnosed clinically, but radiographs may be obtained if the diagnosis is in
question. A relatively common injury in childhood, clavicle fractures typically occur following
blunt trauma to the clavicle or from a fall onto the shoulder with the arm adducted. Eighty percent
occur in the midshaft.
Treatment involves immobilization of the arm on the affected side for comfort. Evidence
shows that the outcome is similar whether a sling or figure of eight bandage is used, and most
children find a simple sling more comfortable. Most of these fractures heal in 4 to 6 weeks
without incident. Rare complications include brachial plexus injury and nonunion, which is
associated most commonly with distal clavicle fractures. Because a simple clavicle fracture is
not uncommon in childhood, this finding in isolation should not prompt investigation for
osteogenesis imperfecta.

Suggested reading:

Andersen K, Jensen PO, Lauritzen J. Treatment of simple clavicular fractures: figure-of-eight


bandage versus simple sling. Acta Orthop Scand. 1987;58:71-74. Abstract available at:
http://www.ncbi.nlm.nih.gov/pubmed/3554886

Andrews S, Davies M, Donatelli R, Whiteside JA. Fracture of the clavicle. In: Griffin LY, ed.
Essentials of Musculoskeletal Care. 3rd ed. Rosemont, Ill: American Academy of Orthopaedic
Surgeons; 2005:171-173

Gholve PA, Hosalkar HS, Wells L. Common fractures. In: Kleigman RM, Behrman RE, Jenson HB,
Stanton BF, eds. Nelson Textbook of Pediatrics. 18th ed. Philadelphia, Pa: Saunders Elsevier;
2007:2834-2841

Wheeless CR III. Clavicle fractures. In: Wheeless Textbook of Orthopaedics. 2008. Available at:
http://www.wheelessonline.com/ortho/clavicle_fractures

Copyright 2010 by the American Academy of Pediatrics page 515


2011 PREP SA on CD-ROM

Question: 161

You receive a call from the emergency department at 11:00 pm on New Years Eve. Five patients
have presented with foreign body ingestions.

Of the following, the foreign body that is MOST appropriate to be removed first is a

A. 1999 penny in the stomach of an 18-month-old

B. AAA battery in the stomach of a 4-year-old

C. button battery in the mid-esophagus of a 2-year-old

D. quarter in the esophagus of a 3-year-old who vomited twice and is refusing solids

E. toothbrush in the stomach of an asymptomatic 16-year-old, which she swallowed 2 hours ago

Copyright 2010 by the American Academy of Pediatrics page 516


2011 PREP SA on CD-ROM

Critique: 161 Preferred Response: C

Management of ingested foreign bodies depends on the item ingested, its anatomic location,
and the presence of symptoms. In most cases, gastric foreign bodies may be managed
conservatively, but esophageal impactions require urgent removal. Button batteries present a
unique problem because they contain toxic heavy metals as well as alkaline compounds (eg,
sodium and potassium hydroxide) that are caustic to esophageal mucosa. Significant
esophageal injury (including perforation) has been reported from button batteries lodged in the
esophagus for as few as 6 hours. Most complications are caused by larger batteries (20 to 23
mm diameter), although significant esophageal injury has been reported with batteries as small
as 8 mm in diameter. Symptoms of dysphagia (including feeding refusal, excessive drooling,
difficulty swallowing) or emesis suggest esophageal impaction.
Regardless of the presence or absence of symptoms, a radiograph of the neck, chest, and
abdomen should be obtained in all patients who present with a history of possible battery
ingestion (Item C161A). Button batteries lodged in the esophagus must be removed
endoscopically as soon as possible after detection. Batteries (both button and cylindrical)
detected in the stomach usually traverse the gastrointestinal tract without incident, with more
than 80% passing within 48 hours. In asymptomatic patients, battery location should be
assessed radiographically at that time, and if the object has not passed the pylorus, removal is
recommended. In symptomatic patients, urgent endoscopic removal is indicated.
In the United States, foreign body ingestions are primarily a pediatric problem, with more
than 80% of reported cases occurring in children. The exception is esophageal meat impaction,
which is the most common foreign body ingestion-related problem in adults who do not have
underlying psychiatric disturbances. Although the precise incidence is unknown, more than
100,000 cases of foreign body ingestion in children were reported to the American Association
of Poison Control Centers toxic exposure surveillance system in 2000. Of these, 98% were
unintentional. Coins are the most commonly reported foreign bodies, and most traverse the
gastrointestinal tract without difficulty. Toy parts, sharp objects (needles, pins), batteries,
chicken or fish bones, and food impactions are other frequently reported items.
A coin lodged in the esophagus must be removed emergently if the patient is unable to
handle secretions (Item C161B). Otherwise, endoscopy may be carried out within 12 to 24
hours, by which time up to 30% of coins (mostly those in the distal third of the esophagus) will
have passed into the stomach. In all instances of esophageal and gastric foreign bodies, a chest
radiograph should be performed immediately prior to endoscopy to confirm location and
determine whether the object has passed the lower esophageal sphincter or the pyloric outlet.
Coins in the stomach require no immediate therapy in asymptomatic patients. Parents should be
instructed to examine stools for coin passage, and if the coin is not retrieved, a follow-up
radiograph may be obtained in 2 to 3 weeks. Only then, if the coin is retained in the stomach,
should removal be considered. Of note, the composition of pennies changed from copper to
predominantly zinc in 1982. Although corrosion by gastric acid may release absorbable zinc
chloride, no difference in the management of ingested pennies is recommended in asymptomatic
patients. Toothbrushes and other long objects (eg, tongue depressors) have been reported as
unintentional ingestions in adolescents who have bulimia. The objects are swallowed while
using them to induce vomiting and require endoscopic removal.

Copyright 2010 by the American Academy of Pediatrics page 517


2011 PREP SA on CD-ROM

Suggested reading:

Arana A, Hauser B, Hachimi-Idrissi S, Vandenplas Y. Management of ingested foreign bodies in


childhood and review of the literature. Eur J Pediatr. 2001;160:468-472. DOI:
10.1007/s004310100788. Abstract available at: http://www.ncbi.nlm.nih.gov/pubmed/11548183

Litovitz T, Schmitz BF. Ingestion of cylindrical and button batteries: an analysis of 2382 cases.
Pediatrics. 1992;89: 747-757. Abstract available at:
http://pediatrics.aappublications.org/cgi/content/abstract/89/4/747

Mas E, Olives J-P. Toxic and traumatic injury of the esophagus. In: Kleinman RE, Goulet O, Mieli-
Vergani G, Sanderson I, Sherman P, Shneider B, eds. Walkers Pediatric Gastrointestinal
Disease. 5th ed. Hamilton, Ontario, Canada: BC Decker; 2008:105-116

Rebhandl W, Steffan I, Schramel P, et al. Release of toxic metals from button batteries retained in
the stomach: an in vitro study. J Pediatr Surg. 2002;37:87-92. DOI: 10.1053/jpsu.2002.29435.
Abstract available at: http://www.ncbi.nlm.nih.gov/pubmed/11781994

Yardeni D, Yardeni H, Coran AG, Golladay ES. Severe esophageal damage due to button battery
ingestion: can it be prevented? Pediatr Surg Int. 2004;20:496-501. DOI: 10.1007/s00383-004-
1223-6. Available at: http://springerlink.metapress.com/content/rdrxu127rlw6twgb/fulltext.html

Copyright 2010 by the American Academy of Pediatrics page 518


2011 PREP SA on CD-ROM

Critique: 161

(Courtesy of D Mulvihill)
A 2-year-old child who ingested a watch battery that is now located in mid esophagus. Although
the foreign body resembles a coin, the more radio-opaque peripheral rim is not a feature of coins.

Copyright 2010 by the American Academy of Pediatrics page 519


2011 PREP SA on CD-ROM

Critique: 161

(Courtesy of D Mulvihill)
Coin in the upper esophagus of a child who had been coughing for weeks. There is widening of
the esophageal wall.

Copyright 2010 by the American Academy of Pediatrics page 520


2011 PREP SA on CD-ROM

Question: 162

A mother brings in her 3-month-old daughter for evaluation of a brassy, barking cough of 1
weeks duration. The infant had a tracheoesophageal fistula diagnosed at birth, but there were no
associated congenital anomalies. She underwent a primary repair within 24 hours of birth and
was discharged at 2 weeks of age after an uneventful hospital course. She has been
breastfeeding well, with good weight gain. Her mother reports that she has some small spit-ups
after feeding, but she does not arch her back. Her 3-year-old brother had a respiratory infection
2 weeks ago. On physical examination, the infant appears well and has biphasic stridor at rest.

Of the following, the MOST likely explanation of this infants cough is

A. an anastomotic leak

B. gastroesophageal reflux disease

C. recurrent tracheoesophageal fistula

D. respiratory infection

E. tracheomalacia

Copyright 2010 by the American Academy of Pediatrics page 521


2011 PREP SA on CD-ROM

Critique: 162 Preferred Response: E

A brassy, barking cough and biphasic stridor in an infant who has had tracheoesophageal
fistula (TEF) repair, such as the infant described in the vignette, are explained best by
tracheomalacia. Tracheomalacia can be found in pathology specimens in 75% of patients who
have esophageal atresia or a TEF (EA/TEF) and typically is found at or above the level of the
original site of the lesion. Most infants only have the brassy, barking cough, but severe
tracheomalacia may manifest with stridor at rest, biphasic stridor, dyspnea with feeding, or
expiratory wheezing with respiratory infections. Some infants who have EA/TEF with severe
tracheomalacia develop "death spells," which begin after 2 to 3 months of age. Such spells are
associated with feeding, crying, or coughing and are characterized by the infant becoming
cyanotic, apneic, and bradycardic.
Gastroesophageal reflux disease (GERD) is seen in up to 60% of infants who have EA/TEF
and is believed to be related to motor dysfunction of the esophagus. Daily cough, but not stridor,
is seen with GERD, and some patients demonstrate failure to thrive. Ongoing reflux may lead to
aspiration pneumonia, with the associated clinical findings of tachypnea, wheezing, and fever.
The lack of fever or tachypnea also makes the diagnosis of respiratory infection less likely for
this infant.
Leak at the anastomatic site is seen in fewer than 17% of patients. Many tiny leaks revealed
on imaging obtained prior to feeding seal spontaneously. Large leaks at the anastomatic site may
present with respiratory distress due to pneumothorax or empyema. The leaks often are
associated with the development of a recurrent TEF. Recurrent TEF can be seen in 3% to 15%
of patients and occurs at the site of the esophageal anastomosis where anastomatic leak,
GERD, or trauma causes tissue damage. Patients who have recurrent TEF commonly present
with coughing and choking with feedings, although excessive salivation and dysphagia also may
be seen. Although tracheomalacia and GERD are the more common causes of respiratory
symptoms in infants who have EA/TEF repair, 13% of infants who have respiratory symptoms
may have recurrent TEF. Further investigation should include an upper gastrointestinal
radiographic series to evaluate esophageal motility and potential recurrence.

Suggested reading:

Kovesi T, Rubin S. Long-term complications of congenital esophageal atresia and/or


tracheoesophageal fistula. Chest. 2004;126:915-925. Available at:
http://chestjournal.chestpubs.org/content/126/3/915.long

Meier JD, Sulman CG, Almond PS, Holinger LD. Endoscopic management of recurrent congenital
tracheoesophageal fistulas: a review of the techniques and results. Int J Pediatr
Otorhinolaryngol. 2007;71:691-697. DOI: 10.1016/j.ijporl.2007.02.022. Abstract available at:
http://www.ncbi.nlm.nih.gov/pubmed/17408757

Vicencio AG, Parikh S, Adam HM. In brief: laryngomalacia and tracheomalacia: common dynamic
airway lesions. Pediatr Rev. 2006;27:e33-e35. DOI: 10.1542/10.1542/pir.27-4-e33. Available at:
http://pedsinreview.aappublications.org/cgi/content/full/27/4/e33

Copyright 2010 by the American Academy of Pediatrics page 522


2011 PREP SA on CD-ROM

Question: 163

A 14-year-old boy has been treated with daily prednisolone and weekly methotrexate for a
rheumatologic condition. He has been responding to treatment well, but he recently developed a
rash involving his neck, back, and chest. Physical examination reveals a widespread symmetric
eruption of uniform-appearing 1- to 3-mm erythematous papules and pustules (Item Q163), see
page 37.

Of the following, the MOST likely cause for the rash is

A. acne vulgaris

B. herpes simplex virus infection

C. steroid acne

D. tinea versicolor

E. viral exanthema

Copyright 2010 by the American Academy of Pediatrics page 523


2011 PREP SA on CD-ROM

Question: 163

(Courtesy of Sugathan Paramoo, MD, and DermAtlas; www. DermAtlas.org)


Eruption, as described for the boy in the vignette.

Copyright 2010 by the American Academy of Pediatrics page 524


2011 PREP SA on CD-ROM

Critique: 163 Preferred Response: C

The rash described for the boy in the vignette, who has been receiving corticosteroid
therapy, most likely represents steroid acne. Steroid acne occurs in many pubertal and
postpubertal patients 2 to 5 weeks after exposure to systemic corticosteroids. In a study of
patients who underwent solid organ transplants and received subsequent immunosuppressive
medication, nearly 40% developed acne within 1 year after the transplant. The outbreak typically
appears as uniform small red papules or pustules, primarily on the face, neck, chest, upper
back, shoulders, and occasionally scalp. The appearance is more like folliculitis than acne, and
although comedones can appear late, nodulocystic lesions and scarring are rare. It can be
pruritic. The lesions are not vesicular, as would be expected with herpes simplex, and they are
not accompanied by systemic symptoms, as often occurs with a viral exanthem. Tinea
versicolor has a typical appearance of macules that are hypopigmented, salmon-colored, or light
brown and often covered with fine scale. Also, they generally are located on the upper trunk
and extremities and less commonly on the face.
Steroid acne resolves once the steroids are discontinued, and its occurrence is not a
contraindication to future use of the medication. If the steroids must be continued because of the
underlying condition, treatment of the rash is more difficult. However, standard acne therapies,
including tretinoin, benzoyl peroxide, or sulfacetamide/sulfur preparations, may be effective.

Suggested reading:

Fortina AB, Piaserico S, Alaibac M, et al. Skin disorders in patients transplanted in childhood.
Transplant Int. 2005;18:360-365. DOI: 10.1111/j.1432-2277.2004.00069.x. Abstract available at:
http://www.ncbi.nlm.nih.gov/pubmed/15730499

Habif TB. Acne. In: Clinical Dermatology: A Color Guide to Diagnosis and Therapy. 5th ed.
Philadelphia, Pa: Mosby Elsevier; 2010:217-251

Morelli JG. Acne. In: Kliegman RM, Behrman RE, Jenson HB, Stanton BF, eds. Nelson Textbook of
Pediatrics. 18th ed. Philadelphia, Pa: Saunders Elsevier; 2007:2759-2763

Copyright 2010 by the American Academy of Pediatrics page 525


2011 PREP SA on CD-ROM

Question: 164

An 18-year-old girl returns from her first semester in college with a 3-week history of malaise
she has attributed to fatigue from final exams. One week ago, she visited the university health
service for symptoms of a cough and cold and was prescribed amoxicillin for bronchitis and
pharyngitis, according to the documentation provided by the clinic. She and her parents are
concerned because in addition to the persistent cough, nausea, and sore throat, she has
developed a rash. On physical examination, the tired-appearing girl has a temperature of 38.5C,
an erythematous pharynx without exudates, and rhonchi bilaterally on auscultation of the lungs.
There is a faint maculopapular rash over her extremities and trunk. Her white blood cell count is
10.3x103/mcL (10.3x109/L), with 60% polymorphonuclear leukocytes, 15% leukocytes, 13%
monocytes, 10% eosinophils, and 2% basophils; hematocrit is 29% (0.29); platelet count is
296x103/mcL (296x109/L); and erythrocyte sedimentation rate is 42 mm/hr. You order a chest
radiograph (Item Q164).

Of the following, the MOST appropriate approach to this girls treatment is to prescribe

A. amoxicillin-clavulanic acid

B. azithromycin

C. cefuroxime sodium

D. ciprofloxacin

E. erythromycin estolate

Copyright 2010 by the American Academy of Pediatrics page 526


2011 PREP SA on CD-ROM

Question: 164

(Reprinted with permission from Durbin WJ, Stille C. Pediatr Rev. 2008;29:147-160)
Chest radiograph, as described for the girl in the vignette.

Copyright 2010 by the American Academy of Pediatrics page 527


2011 PREP SA on CD-ROM

Critique: 164 Preferred Response: B

The girl described in the vignette demonstrates findings consistent with Mycoplasma
pneumoniae infection. This organism is widespread in nature but is pathogenic solely to humans.
The most common syndrome is an acute upper or lower respiratory tract infection. M
pneumoniae is a leading cause of pneumonia in school-age children and young adults, and
infection is especially prevalent in persons living in group settings such as dormitories, military
institutions, and other residential facilities.
As described for the girl in the vignette, a patient may have a prodromal phase, an acute
phase where pharyngitis is prominent (leading to confusion with streptococcal, adenoviral, and
other infections), and finally a prolonged phase characterized by bronchial inflammation. Rash
occurs in approximately 10% of children.
Central nervous system symptoms are not as common but may have protean
manifestations, including encephalitis, neuropathy, and transverse myelitis. In addition, hemolytic
anemia, Stevens-Johnson syndrome, carditis, arthritis, and pleural effusion may occur.
Although illness caused by M pneumoniae generally is self-limited, evidence indicates that
its clinical course may be shortened by treatment with an appropriate antibiotic. Macrolides are
the drugs of choice. Azithromycin is preferred due to a once-daily dosing regimen and fewer
gastrointestinal adverse effects; erythromycin estolate is poorly tolerated and hepatotoxic.
Mycoplasma lacks a cell wall and does not respond to treatment with beta-lactam agents (eg,
amoxicillin-clavulanic acid, cefuroxime sodium). Fluoroquinolones (eg, ciprofloxacin) may be
effective but are broad-spectrum antibiotics and not considered first-line drugs.

Suggested reading:

American Academy of Pediatrics. Mycoplasma pneumoniae and other Mycoplasma species


infections. In: Pickering LK, Baker CJ, Kimberlin DW, Long SS, eds. Red Book: 2009 Report of the
Committee on Infectious Diseases. 28th ed. Elk Grove Village, Ill: American Academy of
Pediatrics; 2009:473-475

Christie LJ, Honarmand S, Talkington DF, et al. Pediatric encephalitis: what is the role of
Mycoplasma pneumoniae? Pediatrics. 2007;120:305-313. DOI: 10.1542/peds.2007-0240.
Available at: http://pediatrics.aappublications.org/cgi/content/abstract/120/2/305

Copyright 2010 by the American Academy of Pediatrics page 528


2011 PREP SA on CD-ROM

Question: 165

You note severe cyanosis and an oxygen saturation of 73% in a 3-hour-old infant in the newborn
nursery. Urgent echocardiography identifies severe pulmonary stenosis. The child is
hemodynamically stable.

Of the following, the BEST next step in management is

A. administration of oxygen

B. initiation of intravenous inotropic support to augment cardiac output

C. initiation of prostaglandin therapy

D. referral to a pediatric cardiac surgeon for surgical valvotomy

E. referral to a pediatric cardiologist for balloon pulmonary valvuloplasty

Copyright 2010 by the American Academy of Pediatrics page 529


2011 PREP SA on CD-ROM

Critique: 165 Preferred Response: C

Isolated pulmonary valve stenosis is seen in approximately 10% of patients who have
congenital heart disease. Severe (critical) pulmonary valve stenosis is life-threatening to the
neonate due to the onset of profound cyanosis upon closure of the ductus arteriosus. Some
cases are expected prenatally because fetal echocardiography establishes the diagnosis. This
allows for the development of a postnatal management strategy that involves central venous
access and initiation of prostaglandin E1. However, most patients who have critical pulmonary
valve stenosis come to medical attention postnatally, when marked cyanosis and a heart murmur
are detected, as described for the infant in the vignette. Generally, the infants are
hemodynamically stable as long as there is no restriction to right-to-left shunting across the
patent foramen ovale and the ductus remains patent. Critical pulmonary valve stenosis typically
is an isolated condition. Most affected individuals have markedly hypertrophied right ventricles,
which develop as a compensatory phenomenon because of the downstream pressure.
Occasionally, concomitant tricuspid valve disease and coronary artery anomalies may be seen.
Once prostaglandin E1 is administered, the neonate typically stabilizes. The current best
practice is interventional cardiac catheterization with balloon pulmonary valvuloplasty, which
generally is highly effective and durable. Prostaglandin usually can be discontinued after
valvuloplasty, although some patients require transient maintenance of prostaglandin infusion
while the pulmonary vascular resistance is allowed to drop. Once this occurs, oxygen
saturation stabilizes and prostaglandin can be weaned. Many children leave the hospital without
the oxygen saturation normalizing, remaining in the range of 80% to 90%; slow recovery occurs
over subsequent days to weeks. Persistent desaturation may be due, at least in part, to the
poorly compliant and hypertrophied right ventricle, which accepts tricuspid inflow poorly,
resulting in an increase in right atrial pressure and an associated right-to-left shunting across
the patent foramen ovale.
Rarely, balloon pulmonary valvuloplasty is unsuccessful. This is more common when the
valve disorder is associated with specific genetic conditions (eg, Noonan syndrome) in which
the valve leaflets are very thickened, "rubbery," and have a tendency to recoil to their native
state, even after aggressive attempts at valvuloplasty. Other patients may have not only valvular
but also subvalvular or supravalvular pulmonary stenosis (the latter commonly seen in Williams
syndrome), which are not amenable to balloon valvuloplasty. In these cases, surgical valvotomy
(at times accompanied by infundibular muscle resection or patch augmentation of the right
ventricular outflow tract and main pulmonary artery) is the therapeutic intervention of choice.
Although administration of oxygen and inotropic support may be integral to stabilization of
the patient, prostaglandin E1 is the only pharmacologic intervention that is directly therapeutic
and is the mainstay of therapy. Oxygen has pulmonary vasodilatory properties and may be of
benefit in hastening the normal decrease in pulmonary vascular resistance. This may allow for
more robust pulmonary blood flow via the ductus arteriosus, thereby improving oxygen
saturation. On the other hand, in the absence of ventricular dysfunction, inotropic agents have
the potential for worsening cyanosis because the resultant hyperdynamic state may result in
reduced filling of the hypertrophied right ventricle and inhibition of cardiac output.

Suggested reading:

Copyright 2010 by the American Academy of Pediatrics page 530


2011 PREP SA on CD-ROM

Brinkman WT, Mack MJ. Transcatheter cardiac valve interventions. Surg Clin North Am.
2009;89:951-966. DOI: 10.1016/j.suc.2009.06.004. Abstract available at:
http://www.ncbi.nlm.nih.gov/pubmed/19782846

Nembhard WN, Wang T, Loscalzo ML, Salemi JL. Variation in the prevalence of congenital heart
defects by maternal race/ethnicity and infant sex. J Pediatr. 2009;156:259-264. DOI:
10.1016/j.jpeds.2009.07.058. Abstract available at:
http://www.ncbi.nlm.nih.gov/pubmed/19818453

Copyright 2010 by the American Academy of Pediatrics page 531


2011 PREP SA on CD-ROM

Question: 166

A 14-year-old girl has had 3 days of new, unremitting headache associated with vomiting and
awakening from sleep and 1 day of double vision. Physical examination reveals normal visual
acuity in each eye, normal pupillary responses in each eye, subjective diplopia when looking to
the left, and incomplete abduction of the left eye. Funduscopic examination reveals bilateral
papilledema (Item Q166). Noncontrast head computed tomography scan yields normal results.

Of the following, the MOST likely diagnosis is

A. brain tumor in the posterior fossa

B. communicating hydrocephalus

C. complicated migraine

D. optic neuritis

E. pseudotumor cerebri

Copyright 2010 by the American Academy of Pediatrics page 532


2011 PREP SA on CD-ROM

Question: 166

(Courtesy of RG Weaver, Jr)

Copyright 2010 by the American Academy of Pediatrics page 533


2011 PREP SA on CD-ROM

Critique: 166 Preferred Response: E

The girl described in the vignette has a headache due to elevated intracranial pressure. The
most likely diagnoses when the history strongly supports elevated intracranial pressure and the
neurologic examination shows bilateral papilledema and a 6th nerve palsy are either a space-
occupying process such as a brain tumor or hydrocephalus or pseudotumor cerebri. Normal
findings on head computed tomography scan generally exclude space-occupying processes
sufficient to cause such signs and symptoms, including hydrocephalus and a posterior fossa
(cerebellum, brainstem) tumor. Accordingly, the most likely diagnosis is pseudotumor cerebri.
Complicated migraines are diagnoses of exclusion. Essentially, these are migraines
accompanied by strokelike symptoms that may present with visual complaints, but more
commonly they present with loss of strength on one side of the body or even aphasia. They do
not present with papilledema.
Optic neuritis, or inflammation of the optic nerve, typically is painful, especially with eye
movement; is unilateral; and causes loss of central visual acuity. The girl in the vignette has
normal visual acuity, which is often the case in pseudotumor. In optic neuritis, papillitis, which
can look like papilledema, may be seen (Item C166).
The first goal in assessing a new complaint of headache is to determine whether the
headache is due to an intracranial disease process, also known as a secondary headache, or is
a paroxysmal event related to an idiopathic headache disorder such as migraine or tension
headache, also known as a primary headache.
Because many secondary headaches produce pain related to elevations in intracranial
pressure, symptoms in the history that are consistent with this should be elicited, including:
1.Pain worse when recumbent
2.Pain awakening from sleep or noted prominently on morning awakening
3.Pain resulting in vomiting, with subsequent temporary relief of pain
4.Pain worse with Valsalva maneuvers such as bowel movements or coughing
5.Pain unlike prior headache pains
6.Pain that has been unremitting/gradually increasing as opposed to typical waxing and
waning in chronic, nonprogressive primary headache disorders
The constellation of symptoms described for this girl raises these concerns and should
prompt a careful general and neurologic examination.
Two eye signs also should be sought carefully in the presence of a new headache. The
first is oculomotor paralysis. The subjective double vision looking laterally indicates a new
acquired ocular malalignment. This can localize to the brainstem, but this patient has no other
neighboring brainstem findings, such as facial weakness, suggesting that the brainstem is not
involved. In the setting of high intracranial pressure headache, this can be a "false localizing
sign"; that is, the brainstem is not involved, although the finding does indicate brain pathology:
raised intracranial pressure. The second sign is the funduscopic examination finding of
papilledema. The funduscopic examination finding of blurred margins of the optic disc (ie,
papilledema) provides important confirmation of this clinical impression and warrants urgent
neuroimaging.
For this girl, head computed tomography (CT) scan should be performed in the emergency
department. Although head CT scan is less sensitive than magnetic resonance imaging for some

Copyright 2010 by the American Academy of Pediatrics page 534


2011 PREP SA on CD-ROM

important intracranial lesions, including posterior fossa tumors, it can exclude hydrocephalus or
any posterior fossa lesions sufficiently large to produce headache by inducing hydrocephalus.
Head CT scan in pseudotumor cerebri or idiopathic intracranial hypertension yields normal
results. The diagnosis is confirmed by lumbar puncture showing elevated pressure, greater than
20 cm, on manometry.
When pseudotumor has been diagnosed, a thorough search should be undertaken for the
cause. Two common causes are obesity and use of oral acne medications such as minocycline.
Neurology referral is recommended. The major long-term risk is permanent visual loss. At the time
of presentation, subjective visual acuity may be normal, as might be results of bedside testing.
However, careful perimetry testing by the ophthalmologist may show an enlarged blind spot. Left
untreated, these may enlarge further and become permanent.

Suggested reading:

Evans RW. Diagnostic testing for migraine and other primary headaches. Neurol Clin.
2009;27:393-415. DOI: 10.1016/j.ncl.2008.11.009. Abstract available at:
http://www.ncbi.nlm.nih.gov/pubmed/19289222

Genizi J, Lahat E, Zelnik N, Mahajnah M, Ravid S, Shahar E. Childhood-onset idiopathic


intracranial hypertension: relation of sex and obesity. Pediatr Neurol. 2007;36:247-249. DOI:
10.1016/j.pediatrneurol.2007.01.002. Abstract available at:
http://www.ncbi.nlm.nih.gov/pubmed/17437908

Kan L, Nagelberg J, Maytal J. Headaches in a pediatric emergency department: etiology, imaging,


and treatment. Headache. 2000;40:25-29. DOI: 10.1046/j.1526-4610.2000.00004.x. Abstract
available at: http://www.ncbi.nlm.nih.gov/pubmed/10759899

Copyright 2010 by the American Academy of Pediatrics page 535


2011 PREP SA on CD-ROM

Critique: 166

(Courtesy of the Wake Forest University Eye Center)


Optic neuritis causes swelling of the optic disc that may mimic papilledema.

Copyright 2010 by the American Academy of Pediatrics page 536


2011 PREP SA on CD-ROM

Question: 167

You are examining male and female twin siblings who were delivered by cesarean section
several hours ago to a 24-year-old gravida 1 woman. Both infants are normally grown and
vigorous. The female twin has torticollis, and the male has varus positioning of his feet that can
be corrected passively.

Of the following, the MOST plausible explanation for the twins musculoskeletal findings is a
history of fetal

A. akinesia

B. deformation

C. disruption

D. dysplasia

E. malformation

Copyright 2010 by the American Academy of Pediatrics page 537


2011 PREP SA on CD-ROM

Critique: 167 Preferred Response: B

When evaluating the newborn who has dysmorphisms, it is critical not only to identify the
abnormalities that are present but to recognize the mechanisms by which they have occurred.
Understanding mechanisms allows appropriate management, discussion of natural history, and
provision of information on recurrence risks.
The four primary mechanisms of unusual embryonic/fetal formation are malformation,
deformation, disruption, and dysplasia. Malformations, such as cardiac defects, are due to
intrinsic problems in the developing tissue. Deformations, such as pugilistic facies, are caused
by extrinsic forces acting upon an otherwise normally developing structure. Disruptions, such
as amniotic bands, are due to the destruction of previously normally developed tissue.
Dysplasias, such as hemangiomas, occur when organization of cells into tissue is abnormal.
The twin newborns described in the vignette each have deformations related to in utero
constraint. Deformations are more common in the context of multiple births and first pregnancies
or in any situation where the fetus is crowded (eg, uterine leiomyoma, abnormal uterine
structure, large fetus). In twin pregnancies, the more inferiorly placed twin is at increased risk
for torticollis related to head and neck position. Unusual foot position also is associated with in
utero crowding. Both of these conditions resolve gradually after birth because the constraining
forces no longer exist. Of note, not all torticollis and unusual foot positioning are due to in utero
constraint; both can be caused by structural defects that do not self-correct.
Fetal akinesia, the most extreme form of decreased fetal movement, has myriad causes,
including central nervous system, spinal cord, peripheral nerve, neuromuscular, and muscular
abnormalities. Affected infants may have a variety of mechanisms of abnormal formation,
depending on the primary cause of absent in utero movement.

Suggested reading:

Graham JM Jr. Clinical approach to deformation problems. In: Smiths Recognizable Patterns of
Human Deformation. 3rd ed. Philadelphia, Pa: Saunders Elsevier; 2007:3-24

Jones KL. Morphogenesis and dysmorphogenesis. In: Smiths Recognizable Patterns of Human
Malformation. 6th ed. Philadelphia, Pa: Elsevier Saunders; 2006:783-795

Jones KL. Pena-Shokeir phenotype. In: Smiths Recognizable Patterns of Human Malformation.
6th ed. Philadelphia, Pa: Elsevier Saunders; 2006:188-189

Copyright 2010 by the American Academy of Pediatrics page 538


2011 PREP SA on CD-ROM

Question: 168

You are seeing a 17-year-old boy for his annual physical examination. He says he is fine and has
no concerns today. His mother states that he has had a few nosebleeds and a chronically runny
nose over the past year, his appetite appears to be decreased, and he has had some weight
loss. He had vague complaints of chest pain and some shortness of breath a few months ago but
not lately. She also notes that he has not been doing as well at school. His physical examination
yields no findings of note. With permission, you obtain a urine drug screen, which yields negative
results, but you still suspect substance abuse.

Of the following, the findings for this boy MOST likely are due to repeated use of

A. cocaine

B. dextromethorphan

C. heroin

D. marijuana

E. tobacco

Copyright 2010 by the American Academy of Pediatrics page 539


2011 PREP SA on CD-ROM

Critique: 168 Preferred Response: A

The adolescent described in the vignette is exhibiting the effects of snorting cocaine.
Cocaine is an illegal, highly addictive drug manufactured from the leaves of the coca plant and
used for its central nervous system stimulant properties and as an appetite suppressant. It is
available in various forms that can be ingested, snorted/sniffed, injected, or smoked, either alone
or along with tobacco and other drugs. Each of these methods can produce different adverse
effects. Regularly snorting cocaine may lead to a chronic runny nose, loss of the sense of smell,
nosebleeds, and nasal septal perforation resulting from vascular constriction. Rubbing the drug
along the gums can result in numbness of these tissues and dry mouth. Smoking cocaine may
cause numbness of the tongue, dry mouth, problems with swallowing, hoarseness, and
potential damage to the lungs. Systemic symptoms include rapid breathing, sweating,
palpitations, high blood pressure, severe headache, pallor, and sometimes fatal arrhythmias,
seizures, strokes, and death. Chest pain may result from a spontaneous pneumothorax, asthma
exacerbation, or myocardial infarction. When ingested, cocaine can cause severe bowel
gangrene as a result of reduced blood flow. Injecting cocaine may be associated with severe
allergic reactions, local abscesses and scars, sepsis, endocarditis, and an increased risk for
contracting human immunodeficiency virus infection and other bloodborne diseases.
The duration and intensity of cocaines effects depend on the route of administration.
Smoking or injecting cocaine results in a more rapid and intense high than snorting. This faster
absorption, however, is associated with a shorter duration of action and, thus, the urge to use
again. A binge pattern of use may lead to irritability, restlessness, anxiety, and paranoia. Cocaine
abusers can suffer a temporary state of paranoid psychosis, in which they lose touch with
reality and experience hallucinations.
None of the other drug choices result in nose bleeds or a chronic runny nose.
Dextromethorphan is the dextrorotatory isomer of the codeine analog levorphanol and is the
major component of many cough and cold preparations. It is ingested commonly to induce
euphoria, and the effects vary with the dose. At low doses, it has a mild stimulant effect, with
distorted visual perception, but at high doses, it acts as a dissociative anesthetic and
hallucinogen, similar to phencyclidine (PCP). Dextromethorphan-containing cough medicines
often also contain antihistaminics and pain medications that can add to the commonly seen
effects. Dextromethorphan may cause a false-positive test result for PCP in urine drug screen
analysis.
Heroin may be snorted/sniffed, smoked, or injected. Acutely, it causes an altered mental
status and depression of the central nervous system in contrast to the stimulant effects of
cocaine. It does not cause vasoconstriction and, therefore, the nasal symptoms experienced by
the adolescent in the vignette. Tobacco use may result in decreased appetite and weight loss;
marijuana may cause weight gain from increased appetite. Urine drug tests yield negative results
within 3 to 5 days of use for all of these substances, except marijuana, which may remain in the
system for 1 month after chronic use.

Suggested reading:

Greene JP, Ahrendt D, Stafford EM. Adolescent abuse of other drugs. Adolesc Med Clin.

Copyright 2010 by the American Academy of Pediatrics page 540


2011 PREP SA on CD-ROM

2006;17:283-318. Abstract available at: http://www.adolescent.theclinics.com/article/S1547-


3368(06)00020-9/abstract

National Institute on Drug Abuse. Drugs of Abuse Information. Available at:


http://www.nida.nih.gov/drugpages/

Rizkalla C, Sue Y-J, Adam HM. In brief: cocaine. Pediatr Rev. 2006;27:436-438. DOI:
10.1542/10.1542/pir.27-11-436. Available at:
http://pedsinreview.aappublications.org/cgi/content/full/27/11/436

Copyright 2010 by the American Academy of Pediatrics page 541


2011 PREP SA on CD-ROM

Question: 169

You are caring for an 18-month-old girl who underwent uncomplicated repair of a large
ventricular septal defect yesterday. She has been doing well and has been receiving 1 L of
oxygen via nasal cannula. She was given a dose of morphine and midazolam approximately 30
minutes ago for removal of her chest tube, and her nurse reports she needed additional doses
because of discomfort during the procedure. An arterial blood gas (ABG) obtained several hours
prior to the procedure showed a pH of 7.40, PacO2 of 35 mm Hg, and PaO2 of 100 mm Hg. The
nurse has just performed another ABG, which shows a pH of 7.25, PacO2 of 48 mm Hg, and
PaO2 of 90 mm Hg.

Of the following, the MOST accurate interpretation of the current ABG is

A. compensated metabolic acidosis

B. compensated respiratory acidosis

C. uncompensated metabolic acidosis

D. uncompensated respiratory acidosis

E. uncompensated respiratory alkalosis

Copyright 2010 by the American Academy of Pediatrics page 542


2011 PREP SA on CD-ROM

Critique: 169 Preferred Response: D

Regulation of the bodys acid-base balance occurs primarily in the kidney and lung and is
vital for cellular homeostasis. The body normally regulates the pH tightly between 7.35 and 7.45.
Acidemia is defined as a pH less than 7.35; alkalemia is defined as a pH greater than 7.45.
Disturbances in the acid-base balance are termed alkalosis or acidosis and are either primarily
respiratory or metabolic. Compensatory mechanisms attempt to restore the pH to normal values
(such as increasing minute ventilation, with a resultant lowering of Paco2, in response to
metabolic acidosis). Many of the acid-base disorders seen in children are simple disorders (one
primary disturbance with one secondary compensatory mechanism), but mixed-acid base
disorders can be seen in critically ill children.
Primary respiratory disturbances include respiratory acidosis (pH <7.35 due to an elevated
Paco2) and respiratory alkalosis (pH >7.45 due to a decreased Paco2). Paco2 alterations can
occur via either changes in elimination (hyper- or hypoventilation) or carbon dioxide production.
The child in the vignette has an uncompensated respiratory acidosis, as evidenced by his
decreased pH of 7.25 and elevated Paco2 of 48 mm Hg. Compensated respiratory acidosis is
indicated by a normal pH due to compensatory decreased renal excretion of bicarbonate as a
result of the elevated Paco2.

Suggested reading:

Greenbaum LA. Electrolyte and acid-base disorders. In: Kliegman RM, Behrman RE, Jenson HB,
Stanton BF, eds. Nelson Textbook of Pediatrics. 18th ed. Philadelphia, Pa: Saunders Elsevier;
2007:267-308

Rose BD. Simple and mixed acid-base disorders. UpToDate Online 17.3. 2008. Available at:
http://www.utdol.com/online/content/topic.do?topicKey=fldlytes/30198&selectedTitle=1~150&so
urce=search_result

Copyright 2010 by the American Academy of Pediatrics page 543


2011 PREP SA on CD-ROM

Question: 170

You are seeing a 13-year-old girl in whom you diagnosed diabetes 8 months ago. She has done
very well to date. Her family helps her maintain careful blood glucose records, which have
revealed values between 70 and 150 mg/dL (3.9 and 8.3 mmol/L). Her hemoglobin A1c (HbA1c)
decreased from 11.0% at diagnosis to 7.0% after 4 months. At the last visit, 6 months after
diagnosis, the HbA1c was 7.3%. Today, the mother is visibly angry with her daughter and tells
you that her daughter has become secretive and uncooperative about sharing her blood glucose
records and does not want to check her blood sugars. She has forgotten her blood glucose
meter but brought her record book with her. In the record book, blood glucoses recorded three
times a day range from 83 to 160 mg/dL (4.6 to 8.9 mmol/L). An HbA1c obtained at this visit is
8.6%. The girls weight is stable and she is otherwise well.

Of the following, the MOST likely explanation for the rise in HbA1c that does not match the written
blood glucose records is

A. development of an eating disorder

B. failure to follow the prescribed meal plan

C. improperly calibrated blood glucose meter

D. omission of some insulin doses

E. waning of the partial remission of early diabetes

Copyright 2010 by the American Academy of Pediatrics page 544


2011 PREP SA on CD-ROM

Critique: 170 Preferred Response: E

The HbA1c is a powerful tool to analyze progression of diabetes and presently is


recommended as appropriate to use for diagnosing diabetes. There is a reasonable correlation
between HbA1c and blood glucose (Item C170). At the time of diagnosis of type 1 diabetes, the
pancreatic beta cells are incapable of making much insulin because of the autoimmune
inflammatory attack and persistent hyperglycemia that is toxic to the beta cell. After initial
treatment and stabilization, the beta cells recover some insulin secretory ability, and during that
time, diabetes is relatively easily controlled. The HbA1c may be close to the normal range of less
than 6%. However, over the first 1 to 2 years of diabetes, the initial recovery, which has been
termed the remission or honeymoon phase, wanes, leading to increasing insulin deficiency and
rising blood glucose and HbA1c values. Often, children who are striving for blood glucose
values to please their parents and their doctors become frustrated by their rising readings and
may try to conceal them from their parents and medical caregivers because they perceive
themselves to blame for the poorer diabetes control. The findings reported for the girl in the
vignette most likely reflect the waning of the partial remission of early diabetes.
Eating disorders commonly are associated with diabetes. However, weight loss or gain,
very poor diabetes control, and psychiatric findings (eg, depression) would be expected. Failure
to follow a prescribed meal plan may lead to poor diabetes control if the patients regimen
matches food to insulin. However, most people who have diabetes today match insulin to food
and, within limits, can eat what they wish as long as they take insulin to match. Omission of
insulin doses is a common cause of hyperglycemia and high HbA1c values, but the most
common cause is waning of the remission phase of diabetes. Glucose meters require calibration
from time to time and can confuse families, but this is not as common an explanation for this girl's
increasing HbA1c as the natural progression of her type 1 diabetes.

Suggested reading:

International Expert Committee. International Expert Committee report on the role of the A1C
assay in the diagnosis of diabetes. Diabetes Care. 2009;32:1327-1334. Available at:
http://care.diabetesjournals.org/content/32/7/1327.long

Levitsky LL, Misra M. Management of type 1 diabetes mellitus in children and adolescents.
UpToDate Online 17.3. 2009. Available at:
http://www.uptodateonline.com/online/content/topic.do?topicKey=pediendo/16910&selectedTitle
=2%7E150&source=search_result

Nathan DM, Kuenen J, Borg R, Zheng H, Schoenfeld D, Heine RJ; A1c-Derived Average Glucose
Study Group. Translating the A1C assay into estimated average glucose values. Diabetes Care.
2008;31:1473-1478. Available at: http://care.diabetesjournals.org/content/31/8/1473.long

Sherry NA, Tsai EB, Herold KC. Natural history of beta-cell function in type 1 diabetes. Diabetes.
2005;54(suppl 2):S32-S39. Available at:
http://diabetes.diabetesjournals.org/content/54/suppl_2/S32.long

Copyright 2010 by the American Academy of Pediatrics page 545


2011 PREP SA on CD-ROM

Critique: 170

Copyright 2010 by the American Academy of Pediatrics page 546


2011 PREP SA on CD-ROM

Question: 171

A mother is concerned that her 5 1/2-year-old boy has been struggling to learn his alphabet and
numbers in kindergarten. He is in good health, and results of his vision and hearing screens are
normal. His preschool teacher commented last year that he was a bit shy but enjoyed being with
the other children.

Of the following, the MOST appropriate next step is to

A. evaluate the boy for attention-deficit/hyperactivity disorder

B. reassure the parents and schedule a 6-month follow-up appointment

C. refer the boy for behavioral counseling

D. refer the boy for psychoeducational evaluation

E. refer the boy for vision therapy

Copyright 2010 by the American Academy of Pediatrics page 547


2011 PREP SA on CD-ROM

Critique: 171 Preferred Response: D

Approximately 5% to 17.5% of individuals in the United States have learning disabilities


(depending on the definition used). Dyslexia (reading disability) is seen in approximately 80% of
individuals who have learning disabilities. Children in preschool may present with delays in
speech and language and may have difficulty following directions and classroom routine. The
boy described in the vignette may have a learning disability, suggested by the difficulty he is
having in learning the alphabet or numbers in kindergarten. Other indicators of learning
disabilities are difficulty paying attention, learning new skills, and connecting letters to sounds.
There also may be poor coordination, including difficulty pasting, coloring, and writing.
Therefore, reassurance of the parents is not appropriate. The boy requires a formal
psychoeducational evaluation that includes cognitive and academic assessment. The evaluation
should be performed in a timely manner so that his learning difficulty does not lead to frustration
and poor self-image. If the evaluation identifies issues with attention or self-esteem, behavioral
counseling and further assessment for attention-deficit/hyperactivity disorder should be
pursued.
Currently, no scientific evidence supports the position that subtle eye or visual problems
cause learning disabilities or that vision therapy improves academic achievement. Other than
convergence-insufficiency treatment, therapy to improve visual function is not supported and is
poorly validated. Convergence insufficiency occurs when an individual is focusing at a near
object and the eyes do not turn in properly. Treating this disorder with eye-focusing exercise
can make reading more comfortable, but it does not improve decoding or comprehension of
reading.

Suggested reading:

American Academy of Pediatrics, Section on Ophthalmology, American Academy of


Ophthalmology, American Association for Pediatric Ophthalmology and Strabismus, American
Association of Certified Orthoptists. Learning disabilities, dyslexia, and vision. Pediatrics.
2009;124:837-844. DOI: 10.1542/peds.2009-1445. Available at:
http://pediatrics.aappublications.org/cgi/content/full/124/2/837

Committee on Children with Disabilities. The pediatricians role in development and implementation
of an Individual Education Plan (IEP) and Individual Family Service Plan (IFSP). Pediatrics.
1999;104:124-127. Available at: http://pediatrics.aappublications.org/cgi/content/full/104/1/124

Learning Disabilities Online web site. Available at: http://www.ldonline.org/index.php

Copyright 2010 by the American Academy of Pediatrics page 548


2011 PREP SA on CD-ROM

Question: 172

You have been reviewing the mechanisms of action of various antibiotic classes with a group of
medical students. Today you discuss penicillin and other beta-lactam antibiotics.

Of the following, the MOST likely mechanism by which these antibiotics inhibit bacterial replication
is by interfering with bacterial

A. cell wall synthesis

B. DNA gyrase

C. folic acid synthesis

D. protein synthesis at the 30S ribosome

E. protein synthesis at the 50S ribosome

Copyright 2010 by the American Academy of Pediatrics page 549


2011 PREP SA on CD-ROM

Critique: 172 Preferred Response: A

Antibiotics inhibit bacteria by inhibiting specific steps in the bacterial reproduction process.
Penicillin and other beta-lactam antibiotics act by binding to and inhibiting proteins in the bacterial
cell wall called penicillin-binding proteins (PBPs). Such actions interfere with bacterial cell wall
synthesis. A number of different PBPs have been described, and different beta-lactam drugs
have varying affinities for these PBPs, accounting for potential differences in activity among the
beta-lactam drugs against particular bacteria. Bacterial resistance against these agents may
develop by selection of strains whose PBPs have decreased affinity for the drugs; production
of the enzyme beta-lactamase, which inactivates the drug by cleaving the beta-lactam bond of
the drug; or changes in the cell membrane that decrease drug permeability or actively pump the
drug out of the cell.
Fluoroquinolone antibiotics are derivatives of the urinary tract agent nalidixic acid. They bind
to bacterial DNA gyrase and topoisomerase IV, interfering with bacterial DNA synthesis.
Aminoglycosides and tetracyclines act by binding to the bacterial 30S ribosome, inhibiting
bacterial protein synthesis. Macrolide antibiotics inhibit bacterial protein synthesis by reversibly
binding to the 50S ribosomal subunit. Sulfonamide antibiotics inhibit steps in bacterial folic acid
synthesis.

Suggested reading:

Kaye KS, Engemann JJ, Fraimow HS, Abrutyn E. Pathogens resistant to antimicrobial agents:
epidemiology, molecular mechanisms, and clinical management. Infect Dis Clin North Am.
2004;18:467-511. DOI: 10.1016/j.idc.2004.04.003. Abstract available at:
http://www.ncbi.nlm.nih.gov/pubmed/15308273

Michelow IC, McCracken GH Jr. Antibacterial therapeutic agents. In: Feigin RD, Cherry JD,
Demmler-Harrison GJ, Kaplan SL, eds. Feigin & Cherrys Textbook of Pediatric Infectious
Diseases. 6th ed. Philadelphia, Pa: Saunders Elsevier; 2009:3178-3226

Copyright 2010 by the American Academy of Pediatrics page 550


2011 PREP SA on CD-ROM

Question: 173

A 14-year-old boy who has been working on a cattle ranch during the summer vacation has had
a temperature of up to 40.0C, diffuse myalgias, and back pain for 2 weeks. Physical examination
reveals a febrile adolescent who has no pharyngeal erythema or exudate. He has mild
hepatosplenomegaly. The remainder of his examination findings are normal.

Of the following, the laboratory test MOST likely to establish the diagnosis is a

A. blood culture

B. complete blood count

C. peripheral blood smear

D. throat culture

E. urine culture

Copyright 2010 by the American Academy of Pediatrics page 551


2011 PREP SA on CD-ROM

Critique: 173 Preferred Response: A

The boy described in the vignette has prolonged fever, and a source is not evident, despite
a careful history and initial physical examination. His exposure to cattle raises the possibility that
brucellosis is causing his symptoms.
Brucellosis is diagnosed by growing Brucella in blood or tissue culture, but the organism
can be difficult to cultivate because it grows slowly. A peripheral blood smear is useful for
diagnosing malaria but would not reveal the organism in patients who have brucellosis. Serology
also can confirm the disease if blood samples taken at least 2 weeks apart demonstrate at least
a fourfold increase in antibody titer. A single antibody titer of greater than 1:160, although not
diagnostic, is suggestive of disease in a patient outside of an endemic area. Patients who have
brucellosis may have a complete blood count that reveals a number of abnormalities, including
anemia, leukopenia, thrombocytopenia, or pancytopenia, but it also can yield normal results. For
patients who have epididymo-orchitis caused by Brucella, epididymal aspiration and culture of
the fluid may recover the organism, but urine culture is not useful. A routine throat culture would
be helpful in diagnosing acute pharyngitis caused by Streptococcus pyogenes.
Brucella are gram-negative coccobacilli that usually infect wild and domestic animals.
Humans contract brucellosis by consuming unpasteurized milk or cheese or by direct contact
with infected animals such as cattle, sheep, and goats. Most cases in the United States occur in
immigrants or travelers returning from endemic areas.
In children, infection due to Brucella often is mild and self-limited, but occasionally it can be
more chronic, as usually occurs in adults. Brucellosis is a well-known cause of fever of
unknown origin and can present with many and varied clinical manifestations, including fever,
sweats, myalgias, arthralgias, fatigue, abdominal pain, headache, anorexia, and weight loss.
Mild hepatosplenomegaly and lymphadenopathy are common. Osteoarticular infection in children
usually involves the sacroiliac joint. Serious complications such as endocarditis, pneumonitis,
and meningitis are rare.
Several regimens have been used for the treatment of brucellosis, and none are 100%
effective in preventing relapse. Oral doxycycline or tetracycline can be used in children 8 years
of age or older. Oral trimethoprim-sulfamethoxazole is safe and effective in younger children.
Because monotherapy is associated with a higher rate of relapse, combination therapy with oral
rifampin is recommended. The duration of treatment is at least 6 weeks. For patients who have
severe complications of brucellosis, the addition of gentamicin or streptomycin to the
antimicrobial regimen is recommended for the first 2 weeks of treatment. Relapses are
associated with premature discontinuation of therapy and usually occur within 3 months of
stopping antibiotics.

Suggested reading:

American Academy of Pediatrics. Brucellosis. In: Pickering LK, Baker CJ, Kimberlin DW, Long SS,
eds. Red Book: 2009 Report of the Committee on Infectious Diseases. 28th ed. Elk Grove
Village, Ill: American Academy of Pediatrics; 2009:237-239

Shen MW. Diagnostic and therapeutic challenges of childhood brucellosis in a nonendemic

Copyright 2010 by the American Academy of Pediatrics page 552


2011 PREP SA on CD-ROM

country. Pediatrics. 2008;121:e1178-e1183. DOI: 10.1542/peds.2007-1874. Available at:


http://pediatrics.aappublications.org/cgi/content/full/121/5/e1178

Copyright 2010 by the American Academy of Pediatrics page 553


2011 PREP SA on CD-ROM

Question: 174

A 4-month-old male infant presents with abdominal distention, vomiting, and poor weight gain. His
temperature is 37.3C, heart rate is 110 beats/min, respiratory rate is 32 breaths/min, and blood
pressure is 96/56 mm Hg. On physical examination, you note abdominal distention, with a palpable
mass above the pubic symphysis. Results of laboratory studies include:

Sodium, 136 mEq/L (136 mmol/L)


Potassium, 7.2 mEq/L (7.2 mmol/L)
Chloride, 110 mEq/L (110 mmol/L)
Bicarbonate, 16 mEq/L (16 mmol/L)
Blood urea nitrogen, 25 mg/dL (8.9 mmol/L)
Creatinine, 1.3 mg/dL (114.9 mcmol/L)
Calcium, 9.5 mg/dL (2.38 mmol/L)
Magnesium, 1.8 mg/dL (0.74 mmol/L)
Phosphorus, 5.5 mg/dL (1.8 mmol/L)

Of the following, the next BEST step in the management of this patients electrolyte abnormality is
administration of

A. intravenous calcium gluconate

B. intravenous dextrose and insulin

C. nebulized albuterol

D. oral furosemide

E. oral sodium polystyrene sulfonate

Copyright 2010 by the American Academy of Pediatrics page 554


2011 PREP SA on CD-ROM

Critique: 174 Preferred Response: A

The infant described in the vignette has abdominal distention, failure to thrive, and a palpable
suprapubic mass as well as hyperkalemia, azotemia, and a normal anion gap metabolic acidosis.
The most likely diagnosis is obstructive uropathy (probably from posterior urethral valves) with
resultant type IV renal tubular acidosis and hyperkalemia. The most urgent condition to address
is the hyperkalemia.
Potassium is contained primarily extracellularly, and its extracellular concentration (relative to
its intracellular concentration) helps set the resting membrane potential for cells. A true elevation
of extracellular potassium raises the resting potential closer to that of an action potential. True
hyperkalemia (>6 mEq/L [6 mmol/L]) can result in electrocardiographic changes such as peaked,
narrow T waves initially (Item C174), with a short QT interval reflecting rapid repolarization. A
potassium concentration greater than 7 to 8 mEq/L (7 to 8 mmol/L) results in prolongation of the P-
R interval, followed by widening of the QRS complex. This is attributed to delayed depolarization
(from inactivation of sodium channels as a consequence of an alteration of the resting
membrane potential from hyperkalemia). Further deterioration of the cardiac rhythm ultimately can
lead to ventricular fibrillation and asystole when the potassium concentration is greater than 10
mEq/L (10 mmol/L). Skeletal muscle weakness also occurs with hyperkalemia due to alteration of
neuromuscular conduction. Thus, although the cellular effects of hyperkalemia are not unique to
the cardiac myocyte, these are the most critical clinically.
The extracellular potassium that is important to the cell is the plasma value. Therefore, blood
samples should be drawn into heparinized (green top) tubes to ensure an accurate analysis
when hyperkalemia is suspected. Falsely elevated potassium values can occur when blood
samples are drawn into unheparinized (red top) tubes because the serum rather than the
plasma concentration will be measured. Potassium values also can be falsely elevated in the
setting of marked leukocytosis or thrombocytosis due to clot retraction, an entity known as
pseudohyperkalemia. The most common cause for pseudohyperkalemia results from hemolysis
of a difficult sample collection, which is common in newborns and young children.
Pseudohyperkalemia should be considered in well children but remains a diagnosis of
exclusion. However, because of the high morbidity and potential mortality associated with true
hyperkalemia, the clinician should approach elevated potassium values as "real until proven
otherwise," with an expeditious approach to repeating the measurement and potentially treating
as if true hyperkalemia exists while awaiting the results of the repeat measurement.
The treatment approach for a child who has true hyperkalemia is targeted at quickly
reducing the decreased membrane excitability (from inactivated membrane sodium channels)
that results in delayed cardiac conduction. This can be accomplished by intravenous calcium,
which counteracts the effects of hyperkalemia on the altered membrane excitability. The effect
is short-lived but very effective. It is considered the first-line approach to severe hyperkalemia.
The second-line approach to hyperkalemia treatment is to shift potassium from the
extracellular to the intracellular compartment. This can be accomplished with intravenous
glucose and insulin, intravenous sodium bicarbonate, and beta-adrenergic agents (eg,
aerosolized albuterol). Because neither intravenous calcium nor agents that shift potassium
intracellularly actually remove potassium from the body, their effects are short-lived.
Three options exist for removing potassium from the body: removal from the gastrointestinal

Copyright 2010 by the American Academy of Pediatrics page 555


2011 PREP SA on CD-ROM

tract with a cation-exchange resin (sodium polystyrene sulfonate administered orally), removal
into the urine with diuretics, or bloodstream removal for patients who have limited urine output or
renal function with dialysis.

Suggested reading:

Quan A, Quigley R, Satlin LM, Baum M. Water and electrolyte handling by the kidney. In: Kher KK,
Schnaper HW, Makker SP, eds. Clinical Pediatric Nephrology. 2nd ed. London, United Kingdom:
Informa Healthcare; 2007:15-35

Rose BD, Post TW. Hyperkalemia. In: Clinical Physiology of Acid-base and Electrolyte
Disorders. 5th ed. New York, NY: McGraw-Hill Medical Publishing Division; 2001:888-930

Copyright 2010 by the American Academy of Pediatrics page 556


2011 PREP SA on CD-ROM

Critique: 174

(Courtesy of A Friedman)
Peaked T waves may be seen in hyperkalemia.

Copyright 2010 by the American Academy of Pediatrics page 557


2011 PREP SA on CD-ROM

Question: 175

A 12-year-old girl presents with recurrent rhinorrhea, sneezing, ocular pruritus, and nasal
congestion each spring and fall. On physical examination, the patient has bilateral conjunctival
erythema, enlarged and pale nasal turbinates, and a transverse nasal crease.

Of the following, the medication that will provide the BEST symptom improvement is an

A. intranasal corticosteroid

B. intranasal decongestant

C. ocular antihistamine

D. oral decongestant

E. oral leukotriene antagonist

Copyright 2010 by the American Academy of Pediatrics page 558


2011 PREP SA on CD-ROM

Critique: 175 Preferred Response: A

The symptoms described for the girl in the vignette suggest that she has seasonal allergic
rhinitis. Intranasal corticosteroids or decongestants, ocular antihistamines, oral decongestants,
or leukotriene antagonists all can be used for treatment, but a nasal corticosteroid or oral
antihistamine is considered first-line therapy. Nasal corticosteroids decrease nasal pruritus,
sneezing, nasal obstruction, and rhinorrhea. Further, they are generally well tolerated and have
minimal adverse effects. Nasal decongestants can be purchased over-the-counter and reduce
nasal congestion, but recurrent or prolonged use can result in rhinitis medicamentosa.
Similar to the first-generation antihistamines (eg, diphenhydramine, hydroxyzine), the
second-generation (eg, fexofenadine, loratadine, cetirizine) and third-generation (eg,
levocetirizine, desloratadine) antihistamines can decrease symptoms of allergic rhinitis. In 2007,
the United States Food and Drug Administration approved two second-generation antihistamines
as over-the-counter medications. In addition, combination tablets of a second-generation
antihistamine and decongestant are available for the treatment of allergic rhinitis in children 12
years of age and older. These combination medications provide the benefit of once- or twice-
daily dosing for older children.
Oral decongestants improve rhinorrhea and nasal congestion, but they have limited effect
on ocular symptoms compared with intranasal corticosteroids or oral antihistamines. An
ophthalmic antihistamine or vasoconstricting eyedrop can be added when ocular symptoms do
not improve with oral antihistamines or intranasal corticosteroids alone, but it would not improve
the significant nasal symptoms observed in this girl.
Montelukast, an oral leukotriene, is approved for perennial allergic rhinitis in children ages 6
months of age and older and for seasonal allergic rhinitis in children 2 years and older. Although
it can be used in combination with other allergic rhinitis medications, monotherapy with an oral
leukotriene is inferior to an oral antihistamine or intranasal corticosteroid.
Other treatments used for allergic rhinitis include mast cell stabilizers, nasal saline rinses,
nasal antihistamines, allergen avoidance measures, and allergen immunotherapy. Nasal cromolyn
is approved for allergic rhinitis, although it must be taken multiple times a day. Nasal saline should
be considered as a routine adjunct for nasal toilet, but it is inferior to nasal corticosteroids for the
treatment of allergic rhinitis. Nasal antihistamines are approved for allergic rhinitis, although they
generally are reserved for patients who do not tolerate intranasal corticosteroids or are used in
combination with other medications for patients who have severe allergic rhinitis. Allergen
avoidance always should be considered, although specific allergen avoidance can be difficult.
Avoiding pets and outdoor exposure on peak pollen days as well as encasing bedding to avoid
dust mite allergens are some implementations to consider. Allergen immunotherapy can be
considered for patients who fail allergen avoidance or in whom allergy medications are either
ineffective or not tolerated.

Suggested reading:

Al Sayyad JJ, Fedorowicz Z, Alhashimi D, Jamal A. Topical nasal steroids for intermittent and
persistent allergic rhinitis in children. Cochrane Database Syst Rev. 2007;1:CD003163. DOI:
10.1002/14651858.CD003163.pub4. Available at:

Copyright 2010 by the American Academy of Pediatrics page 559


2011 PREP SA on CD-ROM

http://www.mrw.interscience.wiley.com/cochrane/clsysrev/articles/CD003163/frame.html

Calderon MA, Alves B, Jacobson M, Hurwitz B, Sheikh A, Durham S. Allergen injection


immunotherapy for seasonal allergic rhinitis. Cochrane Database Syst Rev. 2007;1:CD001936.
DOI: 10.1002/14651858.CD001936.pub2. Available at:
http://www.mrw.interscience.wiley.com/cochrane/clsysrev/articles/CD001936/frame.html

Copyright 2010 by the American Academy of Pediatrics page 560


2011 PREP SA on CD-ROM

Question: 176

A 15-year-old hockey player comes to you because of right shoulder pain after being checked
into the boards during a game 3 days ago. After the game, he noticed swelling and pain in his
shoulder. He has been unable to raise his arm without pain since the injury. On physical
examination, you note prominence and pain with palpation of the distal clavicle.

Of the following, the MOST likely cause of this athletes findings is

A. acromioclavicular separation

B. clavicle fracture

C. proximal humeral fracture

D. rotator cuff tear

E. shoulder dislocation

Copyright 2010 by the American Academy of Pediatrics page 561


2011 PREP SA on CD-ROM

Critique: 176 Preferred Response: A

The patient described in the vignette is suffering from an acute shoulder injury. Although a
clavicle fracture is possible, fracture of the distal clavicle is extremely rare; the more common
injury in this location is an acromioclavicular (AC) separation. AC separation is the term used to
describe injury to the ligaments that connect the acromion to the distal clavicle. These injuries
typically occur after direct trauma to the area or a fall onto the shoulder with the arm adducted.
Most commonly seen in adolescent and young adult males, AC separations usually are sports-
related. AC ligament injuries often are graded as I (ligamentous overstretching), II (partial
ligamentous tears without complete disruption), and III (complete disruption).
A patient who has an AC ligamentous injury presents with a suggestive injury mechanism,
swelling and tenderness over the AC joint, and pain with arm elevation. The AC crossover test,
in which the patients arm is brought across the chest and compresses the AC joint, produces
pain at the AC joint in affected patients.
Shoulder radiographs may be helpful, and anteroposterior views of the affected and
unaffected sides should be obtained for comparison. Widening of the AC joint is suggestive of
AC joint injury (Item C176A) and (Item C176B). Weighted, or stress, views in which the shoulder
is imaged while the patient is holding weights no longer are recommended.
A proximal humeral fracture, rotator cuff tear, or shoulder dislocation do not present with
swelling and tenderness over the distal clavicle.

Suggested reading:

Anderson BC, Anderson RJ. Evaluation of the patient with shoulder complaints. UpToDate
Online 17.3. 2008. Available at:
http://www.utdol.com/online/content/topic.do?topicKey=ad_orth/2929&selectedTitle=2~5&sourc
e=search_result

Andrews S, Davies M, Donatelli R, Whiteside JA. Acromioclavicular injuries. In: Griffin LY, ed.
Essentials of Musculoskeletal Care. 3rd ed. Rosemont, Ill: American Academy of Orthopaedic
Surgeons; 2005:157-160

Koehler SM. Acromioclavicular injuries. UpToDate Online 17.3. 2008. Available at:
http://www.utdol.com/online/content/topic.do?topicKey=ad_orth/8589&selectedTitle=1~5&sourc
e=search_result

Wheeless CR III. AC joint separation. In: Wheeless Textbook of Orthopaedics. 2008. Available
at: http://www.wheelessonline.com/ortho/ac_joint_separation

Copyright 2010 by the American Academy of Pediatrics page 562


2011 PREP SA on CD-ROM

Critique: 176

(Courtesy of D Mulvihill)
Anteroposterior radiograph of the shoulder in acromioclavicular separation: The distal end of the
clavicle is not in contact with the acromion. There is an incidental finding of disconnection of the
ventriculoperitoneal shunt tubing in the neck.

Copyright 2010 by the American Academy of Pediatrics page 563


2011 PREP SA on CD-ROM

Critique: 176

(Courtesy of D Mulvihill)
In the normal shoulder, the acromion and the clavicle are closely approximated.

Copyright 2010 by the American Academy of Pediatrics page 564


2011 PREP SA on CD-ROM

Question: 177

The father of a 4-year-old boy calls you from the emergency department near the familys
vacation home in another state. His son just swallowed a quarter, and an abdominal film shows it
to be in the gastric fundus.

Of the following, you are MOST likely to recommend

A. a follow-up abdominal radiograph in 2 weeks

B. a short course of metoclopramide

C. endoscopic removal of the coin

D. no further follow-up because the coin will pass unaided

E. polyethylene glycol 3550 as an osmotic cathartic

Copyright 2010 by the American Academy of Pediatrics page 565


2011 PREP SA on CD-ROM

Critique: 177 Preferred Response: A

Coins are the most common foreign bodies ingested by children in the United States and are
responsible for approximately 25,000 emergency department visits annually. However, because
many coin ingestions are not witnessed and most are asymptomatic, a significant number likely
are unreported, and others may be managed at home. Most swallowed coins traverse the
gastrointestinal tract unaided and without incident. Despite the fact that most patients do not
present with symptoms, a radiograph should be taken of the neck, chest, and abdomen to
confirm the coin ingestion and estimate its size and location, as described for the boy in the
vignette. For an asymptomatic child who has an ingested coin located in the stomach (or small
bowel), no further immediate management is required. Parents should be instructed to examine
the childs stools because most coins pass within 4 to 6 days. However, if nothing is recovered,
a follow-up radiograph at 2 weeks should confirm coin passage versus retention and guide
further therapy. Interestingly, up to 40% of excreted coins are missed by parents, and follow-up
radiographs are negative.
Because most ingested coins pass spontaneously and are not associated with symptoms,
the necessity of an initial radiograph has been questioned. However, some coins may lodge in
the esophagus (Item C177). Most esophageal foreign bodies cause immediate problems with
handling of secretions, feeding refusal, dysphagia, or a sensation of something in the chest,
indicating the need for urgent endoscopic removal. Coins located in the distal third of the
esophagus, however, may not precipitate acute symptoms. Nevertheless, retained esophageal
foreign bodies are responsible for significant morbidity, including esophageal ulceration and
perforation. A coin also is more likely to lodge in the esophagus of a patient who has underlying
esophageal pathology (eg, previous surgery, esophageal stricture, fundoplication, eosinophilic
esophagitis). Accordingly, a radiograph is recommended in all cases.
Because of its size, a quarter may be retained in the stomach, especially in toddlers. In the
absence of symptoms, it can be removed safely via endoscopy after a few weeks if a follow-
up radiograph indicates gastric retention. Smaller coins also may remain in the stomach,
particularly when multiple coins are ingested and among patients who have underlying motility
disturbances or a history of prior surgery for pyloric stenosis. Patients who present with or
develop symptoms should undergo endoscopy within 12 to 24 hours.
Because the composition of pennies changed from copper to predominantly zinc in 1982,
concerns have been raised about the potential toxicity of zinc chloride released from the
corrosive action of gastric acid following penny ingestion. However, no difference in the
management of ingested pennies from that of other coins currently is recommended in
asymptomatic patients.
Alternate or adjunct therapeutic options following coin ingestion are not recommended.
Drinking carbonated beverages to dilate the esophagus and facilitate passage of esophageal
coins (previously studied in adult subjects), parenteral administration of glucagon, treatment with
prokinetic agents to enhance gastrointestinal motility, and the use of cathartics have not proved
to be effective.

Suggested reading:

Copyright 2010 by the American Academy of Pediatrics page 566


2011 PREP SA on CD-ROM

Arana A, Hauser B, Hachimi-Idrissi S, Vandenplas Y. Management of ingested foreign bodies in


childhood and review of the literature. Eur J Pediatr. 2001;160:468-472. DOI:
10.1007/s004310100788. Abstract available at: http://www.ncbi.nlm.nih.gov/pubmed/11548183

Conners GP. Management of asymptomatic coin ingestion. Pediatrics. 2005;116:752-753. DOI:


10.1542/peds.2005-0062. Available at:
http://pediatrics.aappublications.org/cgi/content/full/116/3/752

Wahbeh G, Wyllie R, Kay M. Foreign body ingestion in infants and children: location, location,
location. Clin Pediatr (Phila). 2002;41:633-640

Copyright 2010 by the American Academy of Pediatrics page 567


2011 PREP SA on CD-ROM

Critique: 177

(Courtesy of M Wright)
Coin in the upper esophagus of a child who had been coughing for weeks.

Copyright 2010 by the American Academy of Pediatrics page 568


2011 PREP SA on CD-ROM

Question: 178

You are evaluating a 2-month-old infant for noisy breathing. She was born at term via an
uncomplicated vaginal delivery and discharged at 2 days of age. She has been breastfeeding
well, with good weight gain. Her mother describes the onset of high-pitched, harsh breathing at 6
weeks of age that seems to come from her upper airway rather than her chest. It occurs only
when the infant is lying on her back or crying. The mother denies hearing any wheezing or
coughing.

Of the following, the MOST likely finding on physical examination of this infant is

A. biphasic stridor

B. cyanosis

C. expiratory stridor

D. hoarse crying

E. inspiratory stridor

Copyright 2010 by the American Academy of Pediatrics page 569


2011 PREP SA on CD-ROM

Critique: 178 Preferred Response: E

Noisy inspiratory breathing in an otherwise well infant, such as the child described in the
vignette, suggests the diagnosis of laryngomalacia, and the physical examination is most likely to
reveal inspiratory stridor. Laryngomalacia is caused by the collapse of the supraglottic
structures during inspiration due to abnormalities in the laryngeal cartilage affecting the epiglottis
or arytenoids. The floppy cartilage allows prolapse of the affected structure over the larynx
during inspiration because of the negative relative air pressure and inward air movement. The
resultant inspiratory noise may be high-pitched, coarse, or low-pitched. Laryngomalacia is the
most common cause of inspiratory noise in infants and classically is associated with inspiratory
stridor. The infant is generally well, with a normal cry and no cyanosis. The inspiratory noise is
worse with supine positioning, as the epiglottis flops down to cause further obstruction. The
inspiratory noise is also worse with crying. Cyanosis and hoarseness are not symptoms of
laryngomalacia.
Tracheomalacia, which is due to a weakness in airway cartilage of the trachea, may be
difficult to distinguish from laryngomalacia. In contrast to laryngomalacia, compression of the
thorax during expiration creates positive pressure in the thoracic cavity that collapses the
weakened trachea (Item C178). A brassy, barking cough; expiratory wheezing; and expiratory
stridor are classic findings in tracheomalacia. If the extrathoracic trachea is involved, the stridor
becomes biphasic. With tracheomalacia, the infant also has a normal cry and generally appears
well. Some infants who have tracheomalacia can have feeding difficulties, which are attributed
to anatomic and reflex mechanisms. If the esophagus compresses the trachea, intermittent
airway obstruction and desaturations occur. In the extreme scenario, the infant may experience
a "death spell," in which the trachea is stimulated by a bolus of food or secretions in the
esophagus that leads to apnea and bradycardia.
A thorough history and clinical examination of the infant who presents with noisy breathing
can lead to an accurate diagnosis. Because extrinsic compression from vascular rings and
masses can mimic tracheomalacia, further studies may be needed, including chest radiography,
esophagraphy, and fiberoptic bronchoscopy.

Suggested reading:

Boogaard R, Huijsmans SH, Pijnenburg MWH, Tiddens HAWM, de Jonste JC, Merkus PJFM.
Tracheomalacia and bronchomalacia in children: incidence and patient characteristics. Chest.
2005;128:3391-3397. Available at: http://chestjournal.chestpubs.org/content/128/5/3391.long

Carden KA, Boiselle PM, Waltz DA, Ernst A. Tracheomalacia and tracheobronchomalacia in
children and adults: an in-depth review. Chest. 2005;127:984-1005. Available at:
http://chestjournal.chestpubs.org/content/127/3/984.long

Vicencio AG, Parikh S, Adam HM. In brief: laryngomalacia and tracheomalacia: common dynamic
airway lesions. Pediatr Rev. 2006;27:e33-e35. DOI: 10.1542/10.1542/pir.27-4-e33. Available at:
http://pedsinreview.aappublications.org/cgi/content/full/27/4/e33

Copyright 2010 by the American Academy of Pediatrics page 570


2011 PREP SA on CD-ROM

Critique: 178

(Reprinted with permission from Vicencio AG, Parikh S. In brief: laryngomalacia and
tracheomalacia: common dynamic airway lesions. Pediatr Rev. 2006;27:e33-e35)
Airway pressures during the respiratory cycle. During inspiration, expansion of the thorax
creates negative pressure within the thoracic cavity and airways (black arrows), allowing inflow
of air (blue arrow). Thus, a weakness in the extrathoracic airway (red arrow) is likely to be
symptomatic during inspiration. During expiration, compression of the thorax creates positive
pressures within the thoracic cavity and airways (black arrows), allowing outflow of air (blue
arrows). Thus, a weakness in an intrathoracic airway (red arrow) is likely to be symptomatic
during expiration.

Copyright 2010 by the American Academy of Pediatrics page 571


2011 PREP SA on CD-ROM

Question: 179

At the 1-month health supervision visit, you noted a 1-cm hemangioma involving the infants upper
eyelid. The parents return 3 weeks later because the lesion has grown rapidly. The hemangioma
has thickened and involves the entire upper eyelid and supraorbital region. There is no bleeding or
oozing from the lesion (Item Q179). The baby is growing and developing normally, and the
remainder of the physical examination findings are normal.

Of the following, the MOST appropriate management plan for this infant is to

A. consult with dermatology and ophthalmology about initiating treatment

B. continue observation, expecting the lesion to involute

C. measure platelet count, prothrombin time, partial thromboplastin time, and fibrinogen

D. prescribe a course of topical mupirocin

E. refer the infant for ultrasonography of the liver

Copyright 2010 by the American Academy of Pediatrics page 572


2011 PREP SA on CD-ROM

Question: 179

(Courtesy of D Krowchuk)
Hemangioma, as described for the infant in the vignette.

Copyright 2010 by the American Academy of Pediatrics page 573


2011 PREP SA on CD-ROM

Critique: 179 Preferred Response: A

Hemangiomas are benign tumors of endothelial origin characterized by a rapid growth phase
and slower involution phase. They affect between 4% and 10% of all infants, and they are more
common among girls, white non-Hispanic infants, and low-birthweight infants. Infantile
hemangiomas are distinguished from vascular malformations by the absence of proliferation and
lack of regression of vascular malformations.
Because of their benign nature and natural history of involution, most infantile hemangiomas
simply are monitored without intervention. Approximately 50% involute by 5 years of age, 70%
by 7 years, and 90% by 9 years. However, in up to 50% of children whose hemangiomas
involute spontaneously, there is some residual change to the skin, including scarring,
redundancy, atrophy, color change, or telangiectasia.
In 10% to 20% of cases, treatment for hemangiomas is recommended because of potential
complications. These complications generally fall into the categories of ulceration, functional
impairment, disfigurement, and association with visceral hemangiomas. Ulceration may be
painful, leads to bleeding, increases the potential for infection, and often causes scarring (Item
C179A). Ulceration may occur in as many as 15% to 20% of affected children. Topical antibiotics
may be used briefly for ulcerated lesions if there is concern for infection, but this treatment has
no effect on the underlying hemangioma. As described for the child in the vignette, one of the
most common (5.6%) functional impairments associated with infantile hemangiomas is visual
disruption because the lesion overgrows and obstructs the eye, leading to amblyopia.
Accordingly, this child should receive treatment after consultation with a dermatologist.
Involvement of ophthalmology for monitoring and addressing visual impairments is also crucial.
Other functional impairments can occur if the hemangioma occludes the auditory canal or when
hemangiomas are in the "beard" distribution (cheeks, chin, lower lip, and anterior neck) (Item
C179B). The latter may be associated with symptomatic lesions of the airway, a life-threatening
complication. Although visceral, primarily liver, hemangiomas are rare (0.4% to 1.4% of patients
in one study), they have the potential for causing high-output cardiac failure. When there is a
palpable abdominal mass, multiple cutaneous hemangiomas, or signs of heart failure, imaging the
liver is indicated. However, in the setting of a single cutaneous hemangioma with otherwise
normal physical examination findings, no imaging is required. Because the face is the most
common location for hemangiomas (41%), potential disfigurement is important when considering
indications for treatment. Areas particularly vulnerable to disfigurement include the tip of the
nose, ear, and chin.
Kasabach-Merritt phenomenon (hemangioma, thrombocytopenia, and coagulopathy) has
been a concern for many practitioners when confronted with what appear to be large
hemangiomas. However, the lesions represent a tufted angioma or kaposiform
hemangioendiothelioma rather than a benign infantile hemangioma. The vascular tumor can be
located anywhere but most often affects the extremities. Because of the associated
coagulopathy, this phenomenon can present with life-threatening bleeding. Monitoring
coagulation studies is appropriate for affected patients.

Suggested reading:

Copyright 2010 by the American Academy of Pediatrics page 574


2011 PREP SA on CD-ROM

Haggestrom AN, Drolet BA, Baselga E, et al. Prospective study of infantile hemangiomas: clinical
characteristics predicting complications and treatment. Pediatrics. 2006;118:882-887. DOI:
10.1542/peds.2006-0413. Available at:
http://pediatrics.aappublications.org/cgi/content/full/118/3/882

Isaacs H Jr. Fetal and neonatal hepatic tumors. J Pediatr Surg. 2007;42:1797-1803. DOI:
10.1016/j.jpedsurg.2007.07.047. Abstract available at:
http://www.ncbi.nlm.nih.gov/pubmed/18022426

Pope E, Krafchik BR, Macarthur C, et al. Oral versus high-dose pulse corticosteroids for
problematic infantile hemangiomas: a randomized, controlled trial. Pediatrics. 2007;119:e1239-
e1247. DOI: 10.1542/peds.2006-2962. Available at:
http://pediatrics.aappublications.org/cgi/content/full/119/6/e1239

Van Aalst JA, Bhuller A, Sadove AM. Pediatric vascular lesions. J Craniofac Surg. 2003;14:566-
583. Abstract available at: http://www.ncbi.nlm.nih.gov/pubmed/12867875

Copyright 2010 by the American Academy of Pediatrics page 575


2011 PREP SA on CD-ROM

Critique: 179

(Courtesy of D Krowchuk)
Hemangioma in the crural fold that has begun to ulcerate.

Copyright 2010 by the American Academy of Pediatrics page 576


2011 PREP SA on CD-ROM

Critique: 179

(Courtesy of D Krowchuk)
Hemangiomas in a "beard-like" distribution may be associated with airway hemangiomas.

Copyright 2010 by the American Academy of Pediatrics page 577


2011 PREP SA on CD-ROM

Question: 180

A family in your practice has adopted a 1-year-old child from Africa. On the initial health
supervision visit, you note that the pale-appearing girl is short but has normal weight for age.
Physical examination is normal except for mild edema of the hands and feet and upper eyelids.
Her gross motor and personal social skills are at age level, with slight diminution in language and
fine motor skills. Laboratory test results include a lead concentration of 5 mcg/dL (0.24 mcmol/L),
hematocrit of 29% (0.29), mean corpuscular volume of 69 fL, creatinine of 0.4 mg/dL (35.4
mcmol/L), and 1+ protein on urine dipstick analysis. Human immunodeficiency virus rapid test
results are negative.

Of the following, the MOST likely cause of this childs signs and symptoms is

A. hookworm infection

B. kwashiorkor

C. marasmus

D. nephrotic syndrome

E. tuberculosis

Copyright 2010 by the American Academy of Pediatrics page 578


2011 PREP SA on CD-ROM

Critique: 180 Preferred Response: A

The child described in the vignette has findings consistent with hookworm infection.
Although the hookworms Ancylostoma duodenale and Necator americanus are ubiquitous
organisms in rural, tropical, and subtropical locales, humans are the only reservoir of infection.
Hookworms flourish where soil is contaminated with human feces. Eggs hatch in soil in 1 to 2
days and become larvae. They may infect humans percutaneously, causing itching and burning
of the skin, or may be ingested, causing pharyngitis and gastroenteritis (Item C180). Patients may
be asymptomatic for years, and gradual burgeoning of immunity causes the worm burden to
decrease by 70% within 1 to 2 years.
Diagnosis is made by finding eggs in stool, usually within 5 to 10 weeks after infection.
Children who are infested with large worm burdens may exhibit failure to thrive, short stature,
and anemia due to chronic blood loss in the intestine and inability to absorb iron. Such symptoms
are especially difficult to elucidate in children who are adoptees or immigrants from developing
countries because they may overlap with symptoms from malnutrition, poverty, and other
infections.
Although the child described in the vignette may be at risk for marasmus or kwashiorkor, her
normal weight for age makes these diagnoses less likely. However, hypoproteinemia
undoubtedly is the cause of the childs swollen hands and feet. Lack of any other abnormal
findings on examination and the prevalence of hookworm infection in developing countries make
this diagnosis most likely. Lack of fever, lymphadenopathy, or other systemic signs makes
tuberculosis disease less likely. Nephrotic syndrome is associated with 3+ to 4+ protein on urine
dipstick analysis.

Suggested reading:

American Academy of Pediatrics. Hookworm infections (Ancylostoma duodenale and Necator


americanus). In: Pickering LK, Baker CJ, Kimberlin DW, Long SS, eds. Red Book: 2009 Report of
the Committee on Infectious Diseases. 28th ed. Elk Grove Village, Ill: American Academy of
Pediatrics; 2009:375-376

Boyle JT. Gastrointestinal bleeding in infants and children. Pediatr Rev. 2008;29 39-52. DOI:
10.1542/10.1542/pir.29-2-39. Available at:
http://pedsinreview.aappublications.org/cgi/content/full/29/2/39

Copyright 2010 by the American Academy of Pediatrics page 579


2011 PREP SA on CD-ROM

Critique: 180

(Courtesy of the Centers for Disease Control and Prevention Public Health Image Library)
Hookworms attached to the intestinal mucosa.

Copyright 2010 by the American Academy of Pediatrics page 580


2011 PREP SA on CD-ROM

Question: 181

You have followed an 18-year-old boy who has trisomy 21 in your practice since early
childhood. He was born and raised for the first several years of his life in a developing country
without access to specialty pediatric care. You saw him initially at 3 years of age, at which time
you referred him for cardiac assessment. He was diagnosed with a complete atrioventricular
canal defect that could not be repaired due to the development of pulmonary vascular obstructive
disease. He had been doing well until a syncopal episode last month. Evaluation today reveals an
oxygen saturation of 75% and hemoglobin of 22 g/dL (220 g/L). The parents ask you about his
prognosis.

Of the following, you are MOST likely to advise his parents that

A. the boys congestive heart failure will worsen over time

B. the boys symptoms will decrease following furosemide therapy to treat the left-to-right shunt

C. their son is likely to experience progressive desaturation, fatigue, and polycythemia due to
Eisenmenger syndrome

D. their sons oxygenation can be reversed with anticoagulant therapy

E. there is little likelihood of clinical deterioration until their son is in his 50s

Copyright 2010 by the American Academy of Pediatrics page 581


2011 PREP SA on CD-ROM

Critique: 181 Preferred Response: C

The teenager described in the vignette has findings consistent with Eisenmenger syndrome.
In this clinical process, a left-to-right shunt commonly caused by a ventricular septal defect,
atrial septal defect, atrioventricular canal defect, or patent ductus arteriosus results in
pulmonary hypertension, gradual muscularization of the pulmonary arterial tree, and eventual
shunt reversal that yields right-to-left circulation. The universal finding in patients who have
Eisenmenger syndrome is the development of cyanosis due to the right-to-left shunt. The
development of Eisenmenger syndrome is becoming increasingly uncommon in this era in which
most children who have congenital heart disease obtain timely diagnosis and surgical repair, but
the findings are well described because prior to the 1950s, most patients who had congenital
heart disease did not have the defects repaired.
Patients who have Eisenmenger syndrome paradoxically are subject to the possibility of
both uncontrolled bleeding due to damaged capillaries and thromboembolic events due to
hyperviscosity. These thromboembolic events can manifest as cerebrovascular accidents
(strokes) as well as other end-organ damage. The chronic hypoxemia results in endogenous
stimulation to produce more red blood cells and increase the oxygen-carrying capacity of the
blood. In addition to increased blood viscosity, patients also are at increased risk for stroke due
to the less deformable nature of the red blood cells.
Other findings associated with Eisenmenger syndrome include right heart failure, valve
dysfunction, and arrhythmias. Symptomatically, patients become increasingly fatigued and
frequently experience syncopal events due to rhythm disturbances, microemboli, and orthostatic
hypotension. The presence of chronic pulmonary hypertension places stress upon the right
ventricle, causing dilation, hypertrophy, and dysfunction. As right ventricular hypertension
persists, excessive back-pressure is placed upon the tricuspid valve, resulting in tricuspid
regurgitation, an increase in right atrial pressure, and hepatic congestion. Peripheral edema also
can occur. The stress upon the right atrium and right ventricle explains the occurrence of
supraventricular tachycardia, atrial flutter and fibrillation, ventricular tachycardia, and conduction
disorders.
Although right heart failure is common in Eisenmenger syndrome, left ventricular function
typically is preserved until end-stage disease is present. Congestive heart failure, therefore, is
less common because the intrinsic irreversible pulmonary vascular resistance reduces
pulmonary edema. Furosemide can be used to effectively treat the right heart failure that
develops, but by definition, patients who have Eisenmenger syndrome no longer have left-to-
right shunts. Anticoagulation is used frequently, especially after the development of
thromboembolic events, to provide prophylaxis against additional episodes. In addition,
anticoagulant therapy commonly is administered to patients who develop incessant atrial
arrhythmias and severe right heart dilation and dysfunction. However, anticoagulation has little-
to-no effect on improving oxygenation because the pulmonary vascular wall changes are
irreversible. A patient experiencing symptoms from Eisenmenger syndrome in late adolescence is
likely to have worsening findings well before he reaches the age of 50 years. The patient who
has trisomy 21, Eisenmenger syndrome, and worsening symptoms of this condition, such as the
boy in the vignette, is unlikely to survive into his fourth decade of life.

Copyright 2010 by the American Academy of Pediatrics page 582


2011 PREP SA on CD-ROM

Suggested reading:

Landzberg MJ, Ungerleider R. Pediatric cardiology and adult congenital heart disease. J Am Coll
Cardiol. 2006;47(11 suppl):D33-D36. DOI: 10.1016/j.jacc.2006.05.010. Available at:
http://www.sciencedirect.com/science?_ob=ArticleURL&_udi=B6T18-4K3K8M5-
9&_user=10&_coverDate=06%2F06%2F2006&_rdoc=1&_fmt=high&_orig=search&_sort=d&_do
canchor=&view=c&_acct=C000050221&_version=1&_urlVersion=0&_userid=10&md5=824c0ca
238296c465873b99c84a30581

Rosenzweig EB, Barst RJ. Pulmonary arterial hypertension in children: a medical update. Curr
Opin Pediatr. 2008;20:288-293. DOI: 10.1097/MOP.0b013e3282ff5fdc. Abstract available at:
http://www.ncbi.nlm.nih.gov/pubmed/18475097

Copyright 2010 by the American Academy of Pediatrics page 583


2011 PREP SA on CD-ROM

Question: 182

A 22-month-old boy presents with irritability and broad-based gait. In addition, he has mild tremor
of the trunk and jerky movements of his limbs, particularly when excited or upset. His mother
noted some "funny" movements of his eyes yesterday that she described as "bobbing." Notable
findings on neurologic examination include irritability, poor cooperation, full extraocular movements
with no nystagmus, normal face and tongue movements, general tremulousness, jerky limb and
trunk movements, and broad-based, lurching gait. Brain magnetic resonance imaging and
cerebrospinal fluid studies yield normal results.

Of the following, the test that is MOST likely to be useful in establishing the diagnosis is

A. alpha-fetoprotein measurement

B. electroencephalography

C. frataxin gene testing

D. serum ceruloplasmin and urinary copper measurement

E. urinary vanillylmandelic acid and homovanillic acid measurement

Copyright 2010 by the American Academy of Pediatrics page 584


2011 PREP SA on CD-ROM

Critique: 182 Preferred Response: E

The diagnostic key for the boy described in the vignette is clinical recognition that he has
opsoclonus myoclonus ataxia syndrome. One possible cause is neuroblastoma, a tumor that can
trigger an autoimmune response, leading to the paraneoplastic opsoclonus myoclonus
syndrome. There is no characteristic antibody test for this diagnosis, but the tumor may produce
high quantities of catecholamines, which can be identified in urine. Therefore, measurement of
vanillylmandelic acid and homovanillic acid in the urine is the most useful test for helping to
establish the diagnosis.
The symptoms of encephalopathy, with irritability and changes in motor control affecting
hands (tremor), gait, and balance, are ominous, and the differential diagnosis is relatively broad.
Anatomic localization of the gait problems and tremor with behavioral change could be localized
to the central nervous system above the spinal cord, possibly the cerebellum and brainstem. The
appropriate initial test, brain magnetic resonance imaging, was obtained, and findings were
normal/nondiagnostic. Therefore, cerebrospinal fluid studies for infection/inflammation were
appropriately obtained.
The differential diagnosis of this clinical presentation includes two inflammatory conditions:
opsoclonus myoclonus ataxia syndrome and acute cerebellar ataxia. These can be very difficult
diagnoses to distinguish, particularly because opsoclonus can be transient and clinicians may
not recognize opsoclonus versus nystagmus. Moreover, the multifocal and truncal myoclonus of
opsoclonus myoclonus ataxia syndrome visually resembles truncal ataxia of acute cerebellar
ataxia. Urgent neurology consultation is advisable.
Alpha-fetoprotein (AFP) is an important blood test to consider for a young child who has
chronic progressive choreoathetosis or ataxia. Elevated AFP values are present in toddlers
presenting with ataxia due to ataxia telangiectasia (AT), a condition in which neurologic
symptoms are present several years before telangiectasias emerge. The extreme irritability
exhibited by the child in the vignette is not characteristic of AT. Moreover, the time course of his
symptoms is subacute, over days, whereas AT and other degenerative diseases typically
progress over months or years.
Electroencephalography (EEG) provides information about the physiology of the cerebral
cortex that usually aids in epilepsy syndrome diagnosis and occasionally to diagnose the cause
of an encephalopathy or to help determine if myoclonus has a cerebral cortical origin. It is
unlikely to be helpful in this case because there are no paroxysmal events under consideration,
and EEG is not useful for diagnosing causes of ataxic gait.
Frataxin gene testing is used to confirm the diagnosis of Friedreich ataxia, a degenerative
disease that presents after age 5 years with progressive gait ataxia, distal sensory loss, and
muscle wasting. Serum ceruloplasmin and urinary copper testing are used in diagnosing Wilson
disease. Neurologic manifestations of Wilson disease, including dysarthria, dystonia, tremor, and
mood disorder, do not present until after the age of 5 years.

Suggested reading:

Cooper R, Khakoo Y, Matthay KK, et al. Opsoclonus-myoclonus-ataxia syndrome in


neuroblastoma: histopathologic features-a report from the Children's Cancer Group. Med Pediatr

Copyright 2010 by the American Academy of Pediatrics page 585


2011 PREP SA on CD-ROM

Oncol. 2001;36:623-629. Abstract available at: http://www.ncbi.nlm.nih.gov/pubmed/11344493

Keller S, Dure LS. Tremor in childhood. Semin Pediatr Neurol. 2009;16:60-70. DOI:
10.1016/j.spen.2009.03.007. Abstract available at:
http://www.ncbi.nlm.nih.gov/pubmed/19501333

Pranzatelli MR, Tate ED, Travelstead AL, Longee D. Immunologic and clinical responses to
rituximab in a child with opsoclonus-myoclonus syndrome. Pediatrics. 2005;115:e115-e119. DOI:
10.1542/peds.2004-0845. Available at:
http://pediatrics.aappublications.org/cgi/content/full/115/1/e115

Pranzatelli MR, Tate ED, Wheeler A, et al. Screening for autoantibodies in children with
opsoclonus-myoclonus-ataxia. Pediatr Neurol. 2002;27:384-387. Abstract available at:
http://www.ncbi.nlm.nih.gov/pubmed/12504207

Wheeler DS, Starr SR. Index of suspicion: case 2. Pediatr Rev. 1998;19:281-284. DOI:
10.1542/10.1542/pir.19-8-281. Available at:
http://pedsinreview.aappublications.org/cgi/content/full/19/8/281

Copyright 2010 by the American Academy of Pediatrics page 586


2011 PREP SA on CD-ROM

Question: 183

The attending neonatologist notifies you that she is caring for a newborn boy in your practice
who has multiple congenital anomalies. The baby was transferred from the newborn nursery to
the intensive care unit due to cyanosis. Echocardiography reveals findings consistent with
tetralogy of Fallot. The infant has a cleft palate. Chest radiographs show multiple thoracic
hemivertebrae (Item Q183). Renal ultrasonography documents unilateral renal agenesis. On
physical examination, the infant has hypoplastic thumbs. The baby is normally grown and
vigorous.

Of the following, the MOST likely diagnosis for this infant is

A. CHARGE syndrome

B. Fanconi anemia

C. trisomy 18

D. 22q11 deletion syndrome

E. VACTERL association

Copyright 2010 by the American Academy of Pediatrics page 587


2011 PREP SA on CD-ROM

Question: 183

(Courtesy of B Wood)
Hemivertebrae, as described for the infant in the vignette.

Copyright 2010 by the American Academy of Pediatrics page 588


2011 PREP SA on CD-ROM

Critique: 183 Preferred Response: D

The combination of tetralogy of Fallot, hypoplastic thumbs, cleft palate, and a renal
abnormality described for the infant in the vignette is highly suspicious for 22q11 deletion
syndrome. This syndrome can be diagnosed by fluorescence in situ hybridization or comparative
genomic hybridization microarray. The incidence of 22q11 deletion syndrome is believed to be
approximately 1 in 4,000, making it one of the most common multiple congenital anomaly
syndromes.
Fanconi anemia is an autosomal recessive condition that is extremely rare (1 in 100,000 to 1
in 350,000) and is associated with physical abnormalities (thumb hypoplasia, heart defect, renal
anomalies, and more) in 60% of affected individuals as well as bone marrow failure and
significant cancer risk. It is more common among persons of Ashkenazi Jewish heritage.
Trisomy 18 occurs in approximately 1 in 3,000 births. Affected infants may share a number
of the features with the infant in the vignette, but they typically are small for gestational age and
usually are feeble.
In medical genetics, the term "association" is used to describe abnormalities that occur
together more than would be expected by chance but that do not have a common cause.
Probably the most well known of these is the VATER association, where V=vertebral anomalies,
A=anal atresia, TE=tracheoesophageal fistula, and R=renal anomalies (Item C183). The VATER
association has been expanded since its original description to "VACTERL" association, wherein
C=cardiac defects, and L=(radial) limb anomalies. To consider VACTERL association as a
possibility, at least one anomaly in each of three geographic regions of involvement (pelvis/lower
abdomen, limb, and thorax) is deemed necessary, but no official minimum criteria are established.
It is important to recognize that associations are better classified as descriptions than as
diagnoses. For an infant to receive the label of VACTERL association, all other reasonable
possibilities should be ruled out, including chromosomal and microarray abnormalities,
teratogenic exposures, and syndromic diagnoses.
CHARGE (C=coloboma defect, H=heart, A=atresia choanae, R=retardation of growth and/or
development, G=genital anomalies, E=ear anomalies) was considered an association until it was
described in more detail and ultimately discovered to be caused by alterations in the CHD7 gene
in 2006. It is now appropriately called "CHARGE syndrome," and the features that are most
consistent with this diagnosis have been delineated even further. For example, cranial nerve
dysfunction and internal ear anomalies are more prevalent than once thought, and intelligence
often is normal.

Suggested reading:

Hall BD. VATER association. In: Cassidy SB, Allanson JE, eds. Management of Genetic
Syndromes. 2nd ed. Hoboken, NJ: John Wiley & Sons; 2005:607-632

Jones KL. VATERR association. In: Smiths Recognizable Patterns of Human Malformation. 6th
ed. Philadelphia, Pa: Elsevier Saunders; 2006:756-759

Lalani SR, Hefner MA, Belmont JW, Davenport SLH. CHARGE syndrome. 2009. GeneReviews.

Copyright 2010 by the American Academy of Pediatrics page 589


2011 PREP SA on CD-ROM

Available at: http://www.ncbi.nlm.nih.gov/bookshelf/br.fcgi?book=gene&part=charge

Oley CA. CHARGE association. In: Cassidy SB, Allanson JE, eds. Management of Genetic
Syndromes. 2nd ed. Hoboken, NJ: John Wiley & Sons; 2005:117-126

Taniguchi T. Fanconi anemia. 2008. GeneReviews. Available at:


http://www.ncbi.nlm.nih.gov/bookshelf/br.fcgi?book=gene&part=fa

Copyright 2010 by the American Academy of Pediatrics page 590


2011 PREP SA on CD-ROM

Critique: 183

(Courtesy of B Wood)
Radiograph of the chest and abdomen in an infant who has VATER syndrome: There is a
hemivertebra (arrow) and esophageal atresia (note that the nasogastric tube ends in the
esophageal pouch).

Copyright 2010 by the American Academy of Pediatrics page 591


2011 PREP SA on CD-ROM

Question: 184

The parents of a 15-year-old boy bring him to see you because he has been more irritable and
aggressive lately. He has no prior history of depression and has maintained a good academic
record. You saw him recently for concerns about slow growth, acne, and gynecomastia and
noted that he was being bullied at school. He is at Sexual Maturity Rating 5 of pubertal
development. His urine drug screen yields negative results.

Of the following, the MOST likely cause of this adolescents findings is

A. anabolic-androgenic steroid use

B. attention-deficit/hyperactivity disorder

C. intermittent marijuana use

D. Klinefelter syndrome

E. new-onset major depression

Copyright 2010 by the American Academy of Pediatrics page 592


2011 PREP SA on CD-ROM

Critique: 184 Preferred Response: A

The boy described in the vignette is at Sexual Maturity Rating (SMR) 5. Gynecomastia is a
common physiologic finding at SMR 2 and resolves thereafter. Breast tissue greater than 2 cm in
diameter at SMR 5, as described for this boy, should prompt consideration of nonphysiologic
causes. Among the numerous medications that are possible causes of gynecomastia are
marijuana and anabolic-androgenic steroids (AAS). The composite picture of slow growth and
bullying combined with the physical and psychological findings reported for this boy suggest
AAS use. AAS abuse is not confined to athletes; these agents may be used by adolescents of
both sexes to increase muscle size or reduce body fat, often as a response to being physically
or sexually abused. In addition to acne and gynecomastia, AAS use may result in reduced sperm
count, testicular atrophy, and male pattern baldness. In females, these agents may cause
masculinization, with oligo- or amenorrhea, deepening of the voice, decreased body fat, loss of
scalp hair, increased body hair, decreased breast size, and clitoromegaly. Although growth may
accelerate with initial use, continued use reduces final height. Changes in lipid concentrations
and platelet function increase the risk of atherosclerosis, myocardial infarction, stroke, and
sudden death. Long-term use is associated with peliosis hepatis and liver and kidney tumors.
Intravenous use may be associated with infections, needle track marks, and keloid formation.
Psychological changes include irritability, aggression (roid rage), euphoria, sexual arousal, mood
swings, forgetfulness, and confusion.
Most children and adolescents who have attention-deficit/hyperactivity disorder (ADHD)
exhibit academic achievement issues, which this teenager does not have. Depression in
adolescents from any cause, including that associated with ADHD, may present as irritability and
aggression rather than with sadness or a depressed mood, but depression is not associated
with the other findings reported for this patient. Gynecomastia may occur with marijuana use
and Klinefelter syndrome, but marijuana slows reflex time and has more of a sedative effect
rather than inducing aggression. Academic performance may deteriorate with chronic use.
Klinefelter syndrome is the most common cause of testicular failure, with small testes and low
testosterone concentrations, and commonly presents with learning disabilities and rapid growth
in mid-childhood.

Suggested reading:

Casavant MJ, Blake K, Griffith J, Yates A, Copley LM. Consequences of use of anabolic
androgenic steroids. Pediatr Clin North Am. 2007;54:677-690. DOI: 10.1016/j.pcl.2007.04.001.
Abstract available at: http://www.ncbi.nlm.nih.gov/pubmed/17723870

Diamantopoulos S, Bao Y. Gynecomastia and premature thelarche: a guide for practitioners.


Pediatr Rev. 2007;28:e57-e68. DOI: 10.1542/10.1542/pir.28-9-e57. Available at:
http://pedsinreview.aappublications.org/cgi/content/full/28/9/e57

Kanayama G, Hudson JI, Pope HG Jr. Long-term psychiatric and medical consequences of
anabolic-androgenic steroid abuse: a looming public health concern? Drug Alcohol Depend.
2008;98:1-12. DOI: 10.1016/j.drugal. Abstract available at:

Copyright 2010 by the American Academy of Pediatrics page 593


2011 PREP SA on CD-ROM

http://www.ncbi.nlm.nih.gov/pubmed/18599224

Kerr JM, Congeni JA. Anabolic-androgenic steroids: use and abuse in pediatric patients. Pediatr
Clin North Am. 2007;54:771-785. DOI: 10.1016/j.pcl.2007.04.010. Abstract available at:
http://www.ncbi.nlm.nih.gov/pubmed/17723876

Trenton AJ, Currier GW. Behavioral manifestations of anabolic steroid use. CNS Drugs.
2005;19:571-595. Abstract available at: http://www.ncbi.nlm.nih.gov/pubmed/15984895

Copyright 2010 by the American Academy of Pediatrics page 594


2011 PREP SA on CD-ROM

Question: 185

You are evaluating a 12-year-old boy who has asthma and has had increased coughing and
difficulty catching his breath over the past 2 days despite using his albuterol inhaler "12 to 15
times." On physical examination, his respiratory rate is 30 breaths/min and heart rate is 120
beats/min. He has labored breathing, with nasal flaring and sternal retractions. He has a markedly
prolonged expiratory phase on lung auscultation, with decreased breath sounds bilaterally at the
lung bases. His oxygen saturation on room air is 83%, but it increases to 90% on 3 L/min of
oxygen administered by nasal cannula. An arterial blood gas shows a pH of 7.24, a PacO2 of 55
mm Hg, and a PaO2 of 75 mm Hg.

Of the following, the finding that is MOST consistent with a diagnosis of respiratory failure in this
boy is

A. accessory muscle use

B. PacO2 greater than 50 mm Hg

C. PaO2 less than 80 mm Hg

D. tachycardia

E. tachypnea

Copyright 2010 by the American Academy of Pediatrics page 595


2011 PREP SA on CD-ROM

Critique: 185 Preferred Response: B

Respiratory failure is defined as the inability to maintain adequate oxygenation (hypoxemia,


as defined by a Pao2 <60 mm Hg) and ventilation (hypercarbia, as defined by a Paco2 >50 mm
Hg). The patient described in the vignette has significant hypercarbia, with a Paco2 of 55 mm Hg
and, therefore, meets the definition of respiratory failure. Tachypnea, increased use of
accessory muscles, or retractions may be signs of impending respiratory failure in pediatric
patients, but they are not diagnostic. Tachycardia often is seen in patients who have respiratory
failure, but it is not specific for respiratory illness.

Suggested reading:

Frankel LR. Respiratory distress and failure. In: Kliegman RM, Behrman RE, Jenson HB, Stanton
BF, eds. Nelson Textbook of Pediatrics. 18th ed. Philadelphia, Pa: Saunders Elsevier; 2007:421-
423

Pope J, McBride J. Consultation with the specialist: respiratory failure in children. Pediatr Rev.
2004;25:160-167. DOI: 10.1542/10.1542/pir.25-5-160. Available at:
http://pedsinreview.aappublications.org/cgi/content/full/25/5/160

Copyright 2010 by the American Academy of Pediatrics page 596


2011 PREP SA on CD-ROM

Question: 186

The family of a 4-year-old boy with diabetes calls at 11 am to tell you that the boy has an acute
vomiting illness just like his brother. It lasted for 12 hours in his brother. He has not been sick
since the onset of diabetes 6 months ago, and the family wants to verify how to handle sick
days. He normally receives glargine insulin every evening and a small amount of an ultrashort-
acting insulin based on his blood glucose measurement and planned carbohydrate intake before
every meal. He received his glargine insulin dose last night but has not been able to keep down
any food since 8 am. The family is planning to check his blood glucose every 2 hours and urine or
blood ketones every 2 to 4 hours and will try to get him to eat and drink. They understand that if
he continues to vomit, he will require evaluation.

Of the following, the MOST appropriate action for them is to

A. administer an additional dose of glargine insulin immediately and follow this with frequent sips
of glucose-containing fluids

B. administer sugar-containing fluids but no ultrashort-acting insulin until he stops vomiting

C. offer one glass of water every 2 hours even if he is vomiting

D. provide only regular insulin in a small dose every 4 to 6 hours as long as his blood glucose
remains greater than 200 mg/dL (11.1 mmol/L) and he can tolerate solid foods

E. provide small amounts of his normal short-acting insulin every 2 to 3 hours as long as his blood
glucose is greater than 100 mg/dL (5.6 mmol/L) and he can consume clear sugar-containing
liquids

Copyright 2010 by the American Academy of Pediatrics page 597


2011 PREP SA on CD-ROM

Critique: 186 Preferred Response: E

Children who have diabetes and receive a long-acting insulin regimen such as glargine
insulin to cover basal insulin needs and an ultrashort-acting insulin such as insulin lispro or
aspart usually are protected from ketoacidosis during periods when they cannot eat because
they have sufficient basal insulin administered once daily. If the dose is correct, it will not cause
hypoglycemia during periods of food deprivation. However, illness itself may induce insulin
resistance, necessitating insulin supplementation. Because the action of ultrashort-acting
insulins generally begins about 10 to 15 minutes after administration, peaks at 1 hour, and lasts
no longer than 3 to 4 hours, supplementing with small amounts of this insulin every 2 to 3 hours
if glucose is greater than 100 mg/dL (5.6 mmol/L) for the boy described in the vignette is
reasonable. If he can take sips of carbohydrate- and electrolyte-containing fluids, he should
remain free from significant ketosis and can be managed with oral rehydration and ultrashort-
acting insulin every 2 to 3 hours.
Glargine insulin should not be used as an acute supplement because its action is spread
over 24 hours. Insulin can be administered, even to a child who is vomiting, as long as blood
glucose concentrations are elevated. The dosing should be based on whether the child can take
carbohydrates orally. Providing plain water alone to a child who has diabetes and is vomiting will
result in electrolyte depletion. Regular insulin can be provided every 4 to 6 hours to treat
hyperglycemia, but the time course of action is slower and less predictable than that of the
shorter-acting insulins. Also, use of regular insulin is more likely to lead to prolonged
hypoglycemia in a child who no longer can consume carbohydrates orally. Because this child is
vomiting, oral rehydration rather than supplying solid food is a reasonable response.

Suggested reading:

Brink S, Laffel L, Likitmaskul S, et al; International Society for Pediatric and Adolescent Diabetes
(ISPAD). Sick day management in children and adolescents with diabetes. Pediatr Diabetes.
2007;8:401-407. DOI: 10.1111/j.1399-5448.2007.00351.x

Silverstein J, Klingensmith G, Copeland K, et al; American Diabetes Association. Care of children


and adolescents with type 1 diabetes: a statement of the American Diabetes Association.
Diabetes Care. 2005;28:186-212. Available at:
http://care.diabetesjournals.org/content/28/1/186.long

Tanyolac S, Goldfine ID, Kroon L. Insulin pharmacology, types of regimens and adjustments.
Endotext.org. 2008. Available at: http://www.endotext.org/diabetes/diabetes17/diabetes17.htm

Copyright 2010 by the American Academy of Pediatrics page 598


2011 PREP SA on CD-ROM

Question: 187

A 10-year-old boy has been receiving specialized educational services in school due to a learning
disability. His parents encourage after-school or sports activities to promote positive self-esteem.
They realize that academics are a challenge for him, and they express concerns about his future
as an adult.

Of the following, the factor that has been shown to have the MOST positive effect on prognosis
for such a child is

A. early intervention therapy

B. fathers level of education

C. having two or more siblings

D. high intelligence quotient

E. strong family support

Copyright 2010 by the American Academy of Pediatrics page 599


2011 PREP SA on CD-ROM

Critique: 187 Preferred Response: E

The presence of strong family support has been shown to have a positive effect on the
prognoses of children who have learning disabilities, such as the boy described in the vignette.
This effect is greater than that associated with the parents education, number of siblings in the
family, intelligence quotient score, or early educational intervention. Parent involvement and
collaboration with physicians and schools is critical to ensure appropriate services.
Support in the home via interactive learning activities (use of readers, audiobooks) is
especially important for children who have learning disabilities because they often are
overwhelmed, disorganized, and frustrated in learning situations. Specific academic support
such as one-on-one tutoring in the home frequently is sought to help focus on homework. Family
support can augment specialized educational intervention. Educational strategies such as
remediation work targeting the underlying cognitive function that is impaired and phonologic
programs (those that improve word decoding) have shown efficacy up to the sixth grade. If the
area of weakness is unlikely to be corrected, circumvention strategies are useful in improving
the outcome. Such strategies include using a keyboard or verbal exams for a child who has
motor dysgraphia, using audiobooks for a child who has dyslexia, and using a calculator for the
child who has dyscalculia (math disorder).

Suggested reading:

Committee on Children with Disabilities. The pediatricians role in development and implementation
of an Individual Education Plan (IEP) and Individual Family Service Plan (IFSP). Pediatrics.
1999;104:124-127. Available at: http://pediatrics.aappublications.org/cgi/content/full/104/1/124

Lambros KM, Leslie LK. Management of the child with a learning disorder. Pediatr Ann.
2005;34:275-287. Abstract available at: http://www.ncbi.nlm.nih.gov/pubmed/15871432

Copyright 2010 by the American Academy of Pediatrics page 600


2011 PREP SA on CD-ROM

Question: 188

A 3-year-old boy is referred to you for evaluation of his third episode of soft-tissue infection. You
note that he was hospitalized at 1 year of age for a perirectal infection requiring incision and
drainage, and his umbilical cord detached at 8 weeks of age. On physical examination, his
temperature is 39.3C, he has an approximately 5-cm area of erythema with central necrosis over
his left lateral thigh, and his gums are markedly inflamed. His white blood cell count is
42.0x103/mcL (42.0x109/L), with 80% neutrophils, 3% bands, and 17% lymphocytes. You
suspect a primary immunodeficiency disorder.

Of the following, the immunodeficiency is MOST likely related to a defect in function or number of

A. complement proteins

B. immunoglobulins

C. interleukin-12

D. neutrophils

E. T-cell subsets

Copyright 2010 by the American Academy of Pediatrics page 601


2011 PREP SA on CD-ROM

Critique: 188 Preferred Response: D

The combination of delayed umbilical cord separation, recurrent bacterial skin and soft-
tissue infections, and gingival or periodontal disease described for the boy in the vignette are
suggestive of a defect in neutrophil number or function. The child has an elevated white blood
cell count with increased neutrophils in the differential count, but the description of a necrotic
area without pus suggests a deficit in neutrophil migration or function. The disorder is most
consistent with an autosomal recessive disorder of neutrophils called leukocyte adhesion
deficiency.
Congenital deficiencies of primary complement components may present with severe or
recurrent encapsulated bacterial (Neisseria meningitidis, Streptococcus pneumoniae,
Haemophilus influenzae type b, disseminated Neisseria gonorrhoeae) infections or autoimmune
disease. Children who have immunoglobulin deficiencies typically present with recurrent or
severe respiratory tract bacterial infections, sepsis, or meningitis. Interleukin (IL)-12 is released
by macrophages in response to intracellular infections caused by mycobacteria or salmonellae.
IL-12 receptor deficiencies are very rare and have been associated with disseminated
nontuberculous mycobacterial infections, tuberculosis, and Salmonella infection. T-cell subset
deficiencies are associated with severe viral, fungal, or Pneumocystis infections.

Suggested reading:

Etzoni A. Leukocyte adhesion deficiency. UpToDate Online 17.3. 2009. Available at:
http://www.utdol.com/online/content/topic.do?topicKey=immunnon/14958&selectedTitle=1%7E17
&source=search_result819

Lekstrom-Himes JA, Gallin JI. Immunodeficiency diseases caused by defects in phagocytes. N


Engl J Med. 2000;343:1703-1714. Extract available at:
http://content.nejm.org/cgi/content/extract/343/23/1703

Copyright 2010 by the American Academy of Pediatrics page 602


2011 PREP SA on CD-ROM

Question: 189

A 2-month-old previously well term infant presents with constipation, poor feeding, and
weakness. She was breastfed until 1 week ago, when she was started on infant formula.
Physical examination reveals an alert, afebrile, and floppy infant who has a weak cry and
diminished gag reflex.

Of the following, the test MOST likely to establish the diagnosis is

A. blood culture

B. electroencephalography

C. rectal biopsy

D. stool toxin assay

E. urine drug screen

Copyright 2010 by the American Academy of Pediatrics page 603


2011 PREP SA on CD-ROM

Critique: 189 Preferred Response: D

The infant described in the vignette has botulism caused by Clostridium botulinum. The
disease classically presents with constipation, progressive descending weakness and
hypotonia, diminished gag reflex, ocular palsies, poor feeding, and weak cry.
C botulinum causes human disease by producing neurotoxins A, B, E, and F. Infant botulism
usually is caused by toxins A or B. The diagnosis requires toxin assay and culture of stool. The
state health department should be contacted to perform the mouse toxin neutralization bioassay
because it is not generally available. If stool is difficult to obtain because of constipation,
administration of a small nonbacteriostatic water enema is recommended. Toxin is present in the
serum in only 1% of affected infants. A routine blood culture does not grow the organism.
Selective and enriched media is required to isolate the organism from stool.
Electroencephalography may be indicated in infants who have seizures or altered mental status,
but it is not helpful in the diagnosis of infant botulism. A rectal biopsy can establish the diagnosis
of Hirschsprung disease. An alert infant who has neurologic deficits such as those described in
the vignette would be expected to have negative findings on a urine drug screen.
Infant botulism generally occurs in children younger than 6 months of age who are receiving
nonhuman milk formula contaminated by Clostridium spores. Honey also is a known source of
botulism. Clostridium spores are ingested, germinate, and produce botulinum toxin in the
intestine. This differs from foodborne botulism in which illness follows ingestion of preformed
botulinum toxin in food that was stored improperly under anaerobic conditions that allowed
spores to germinate, multiply, and produce toxin.
Treatment with botulism antitoxin should be initiated immediately and should not await
laboratory confirmation of disease because results may be delayed. Human-derived botulism
immune globulin for intravenous use is used for the treatment of infant botulism caused by C
botulinum toxin A or B. Antibiotic therapy is not recommended for the treatment of infant
botulism.

Suggested reading:

American Academy of Pediatrics. Botulism and infant botulism (Clostridium botulinum). In:
Pickering LK, Baker CJ, Kimberlin DW, Long SS, eds. Red Book: 2009 Report of the Committee
on Infectious Diseases. 28th ed. Elk Grove Village, Ill: American Academy of Pediatrics;
2009:259-262

Arnon SS. Infant botulism. In: Feigin RD, Cherry JD, Demmler-Harrison GJ, Kaplan SL, eds. Feigin
& Cherrys Textbook of Pediatric Infectious Diseases. 6th ed. Philadelphia, Pa: Saunders
Elsevier; 2009:1862-1869

Risko W. In brief: infant botulism. Pediatr Rev. 2006;27:36-37. DOI: 10.1542/10.1542/pir.27-1-36.


Available at: http://pedsinreview.aappublications.org/cgi/content/full/27/1/36

Copyright 2010 by the American Academy of Pediatrics page 604


2011 PREP SA on CD-ROM

Question: 190

The mother of a 7-year-old girl reports that her daughters urine has a "cola" color but contains no
clots. She was well until she developed a sore throat yesterday. She has no dysuria, frequency,
back pain, or trauma. On physical examination, the girl has trace edema but no suprapubic or
flank tenderness. Her temperature is 38.1C, heart rate is 80 beats/min, respiratory rate is 16
breaths/min, and blood pressure is 134/84 mm Hg. Urinalysis reveals:

Specific gravity, 1.020


pH, 6
3+ blood
3+ protein
Negative nitrite
1+ leukocyte esterase
More than 100 red blood cells/high-power field (hpf)
5 to 10 white blood cells/hpf

Of the following, the MOST likely associated laboratory abnormality is

A. bilateral cysts on renal ultrasonography

B. depressed complement component 3

C. elevated serum creatinine

D. elevated urinary calcium-to-creatinine ratio

E. positive urine culture

Copyright 2010 by the American Academy of Pediatrics page 605


2011 PREP SA on CD-ROM

Critique: 190 Preferred Response: C

The child described in the vignette has gross hematuria (cola-colored urine) without clots.
Her history is notable for recent symptoms of sore throat and negative for lower urinary tract
symptoms (dysuria, frequency, or urgency), flank pain, and trauma. Physical examination
reveals trace edema and mild hypertension (the 95th percentile for a 7-year-old child of average
height is 113/75 mm Hg). Her urinalysis documents the presence of 3+ blood and 3+ protein, with
more than 100 red blood cells/high-power field (hpf) and mild pyuria (5 to 10 white blood
cells/hpf). The hematuria/proteinuria, mild hypertension, and trace edema, especially in light of
the cola-colored urine, is most compatible with acute glomerulonephritis. The "nephritic"
syndrome usually is accompanied by azotemia, as demonstrated by an elevated serum
creatinine concentration.
An additional consideration in this clinical setting is hypocomplementemic nephritis, which
necessitates measurement of complement component 3 (C3). Postinfectious glomerulonephritis
classically features cola-colored urine, develops 7 to 21 days after an episode of pharyngitis,
and is characterized by hypocomplementemia (low C3). The girl in the vignette reports a viral
illness (sore throat) only 1 to 2 days before the onset of the cola-colored urine, a history that is
most compatible with synpharyngitic hematuria and most often is due to immunoglobulin A (IgA)
nephritis. IgA nephritis is a normocomplementemic nephritis, in which depressed C3 values are
unlikely. A positive urine culture would be unlikely for this girl because she has no symptoms
consistent with a urinary tract infection, and cola-colored urine is not seen with such an
infection. Hypercalciuria (determined by an elevated urine calcium-to-creatinine ratio) can be
associated with gross hematuria, but the urine color usually is bright red and may or may not
have clots present. Bilateral renal cysts (as can be seen with polycystic kidney disease) can be
associated with gross hematuria following trauma, but this event typically leads to bright red
urine with clots.
When establishing a differential diagnosis for the child who has gross hematuria, a good
starting point is to characterize the urine color in an effort to help localize the lesion to a
particular anatomic region. Patients who have cola- or tea-colored urine often have painless
hematuria without clots, the classic finding with glomerulonephritis. Those who have bright red
urine (often with clots) may have an underlying nonglomerular renal cause, a lower urinary tract
condition (ureter or bladder), or rarely a hematologic disorder.
Patients who have cola-colored urine require close monitoring of blood pressure and renal
function during the evaluation for the underlying cause of the suspected glomerulonephritis.
Measuring the C3 is the best single diagnostic test in the evaluation of acute glomerulonephritis.
Additional adjunctive diagnostic testing typically undertaken at the outset includes assessment of
C4, antinuclear antibody, anti-double-stranded DNA antibody, and serum albumin. These tests
help the clinician categorize the glomerulonephritis as hypocomplementemic or
normocomplementemic, which can narrow the differential diagnosis further. A renal biopsy
usually is considered in patients who have acute glomerulonephritis in the setting of a rising
serum creatinine or in cases of normocomplementemic glomerulonephritis or suspected lupus
nephritis. The patient in the vignette, who has suspected normocomplementemic
glomerulonephritis, may require a renal biopsy early in the course of the illness, but this decision
should be left to a pediatric nephrologist with whom consultation should be sought early in the

Copyright 2010 by the American Academy of Pediatrics page 606


2011 PREP SA on CD-ROM

disease course.
Patients who have bright red urine with clots should be assessed for possible structural
causes for hematuria. These nonglomerular renal causes include a ruptured cyst in cases of
cystic kidney disease (as in polycystic kidney disease, both autosomal dominant and recessive
forms) or related to a renal mass (Wilms tumor) or a renal stone (nephrolithiasis or urolithiasis).
Among the additional causes are renal vein thrombosis, papillary necrosis, or hypercalciuria.
Hematologic disorders such as sickle trait/disease or a bleeding disorder (von Willebrand
disease) can result in a similar clinical picture and requires investigation. Among the lower
urinary tract causes are hemorrhagic cystitis, bladder calculi, or rarely a tumor of the lower
urinary tract (hemangioma of the bladder or rhabdomyosarcoma of the bladder). In an effort to
evaluate for possible cysts, stones, or tumors, all children who have gross hematuria should
undergo renal/bladder ultrasonography to look for a structural abnormality. In rare cases,
additional testing is required, such as abdominal computed tomography scan to look for renal
calculi if the clinical suspicion is high and ultrasonography yields negative results. Routine use of
this test is strongly discouraged because of long-term concerns regarding ionizing radiation
exposure (estimated to be equivalent to the radiation dose in 250 two-view chest radiographs).

Suggested reading:

Brody AS, Frush DP, Huda W, Brent RL; Section on Radiology. Radiation risk to children from
computed tomography. Pediatrics. 2007;120:677-682. DOI: 10.1542/peds.2007-1910. Available
at: http://pediatrics.aappublications.org/cgi/content/full/120/3/677?

Massengill SF. Hematuria. Pediatr Rev. 2008;29:342-348. DOI: 10.1542/10.1542/pir.29-10-342.


Available at: http://pedsinreview.aappublications.org/cgi/content/full/29/10/342

Moxey-Mims M. Hematuria and proteinuria. In: Kher KK, Schnaper HW, Makker SP, eds. Clinical
Pediatric Nephrology. 2nd ed. London, United Kingdom: Informa Healthcare; 2007:129-141

National High Blood Pressure Education Program Working Group on High Blood Pressure in
Children and Adolescents. The fourth report on the diagnosis, evaluation, and treatment of high
blood pressure in children and adolescents. Pediatrics. 2004;114:555576. Available at:
http://pediatrics.aappublications.org/cgi/content/full/114/2/S2/555

Pan CG. Evaluation of gross hematuria. Pediatr Clin North Am. 2006;53:401-412. DOI:
10.1016/j.pcl.2006.03.002. Abstract available at: http://www.ncbi.nlm.nih.gov/pubmed/16716787

Youn T, Trachtman H, Gauthier B. Clinical spectrum of gross hematuria in pediatric patients. Clin
Pediatr (Phila). 2006;45:135-141. Abstract available at:
http://www.ncbi.nlm.nih.gov/pubmed/16528433

Copyright 2010 by the American Academy of Pediatrics page 607


2011 PREP SA on CD-ROM

Question: 191

You are evaluating a 2-year-old boy who was admitted for a suspected asthma exacerbation.
Initially, he required frequent nebulization treatments and supplemental oxygen, but he has
improved significantly over the past 3 days after initiation of oral corticosteroids. While you are
reviewing his discharge medications with his parents, they ask you about possible asthma
triggers.

Of the following, the MOST likely asthma trigger for a child of this age is

A. allergic rhinitis

B. chronic sinusitis

C. exercise

D. gastroesophageal reflux

E. upper respiratory tract infection

Copyright 2010 by the American Academy of Pediatrics page 608


2011 PREP SA on CD-ROM

Critique: 191 Preferred Response: E

Upper respiratory tract infections are the most common triggers for asthma in infants and
young children. Previously believed to be a virus that only affects the upper airway, rhinovirus
can infect the lower airway directly and is the most common virus implicated in childhood asthma
exacerbations.
Gastroesophageal reflux (GER) can cause cough and trigger asthma, but the boy in the
vignette has no evidence of GER symptoms. Often, GER worsens during exercise, eating, or
when supine.
Postnasal drip syndrome, now termed upper airway cough syndrome, can result in
coughing due to either allergic (allergic rhinitis) or nonallergic (nonallergic rhinitis, chronic
sinusitis) triggers. Allergic rhinitis can develop as early as the first birthday but generally does
not present until 5 to 10 years of age. Chronic sinusitis can complicate a viral upper respiratory
tract infection, but it is characterized by sinus symptoms lasting for more than 12 weeks.
Exercise is a common trigger for patients who have asthma, resulting in symptoms in 80% to
90% of affected children. However, exercise-induced asthma generally occurs in older, school-
age children.
When discharging children who have asthma from the hospital, the medical team should
consider a number of recommendations offered in the 2007 Expert Panel Report-3 asthma
management guidelines (Item C191). Although time-consuming, implementation of these
recommendations improves quality of life, reduces future hospitalizations, decreases
unscheduled acute visits, and results in fewer missed school or work days.

Suggested reading:

Adams NP, Bestall JC, Lasserson TJ, Jones P, Cates CJ. Fluticasone versus placebo for chronic
asthma in adults and children. Cochrane Database Syst Rev. 2008;4:CD003135. DOI:
10.1002/14651858.CD003135.pub4. Available at:
http://www.mrw.interscience.wiley.com/cochrane/clsysrev/articles/CD003135/frame.html

National Asthma Education and Prevention Program. Expert Panel Report 3 (EPRS): Guidelines
for the Diagnosis and Management of Asthma. Full Report 2007. NIH Publication 07-4051.
Bethesda, Md: National Heart, Lung, and Blood Institute; 2007. Available at:
http://www.nhlbi.nih.gov/guidelines/asthma/asthgdln.htm

Rowe BH, Spooner CH, Ducharme FM, Bretzlaff JA, Bota GW. Corticosteroids for preventing
relapse following acute exacerbations of asthma. Cochrane Database Syst Rev.
2007;3:CD000195. DOI: 10.1002/14651858.CD000195.pub2. Available at:
http://www.mrw.interscience.wiley.com/cochrane/clsysrev/articles/CD000195/frame.html

Copyright 2010 by the American Academy of Pediatrics page 609


2011 PREP SA on CD-ROM

Critique: 191

Copyright 2010 by the American Academy of Pediatrics page 610


2011 PREP SA on CD-ROM

Question: 192

You are giving a talk to a community group about water safety. One of the attendees asks about
ways to prevent children from drowning in home pools.

Of the following, the MOST effective drowning prevention measure is to

A. have young children use inflatable arm floats ("water wings") when they are in the pool

B. install four-sided fences around backyard pools

C. provide swimming lessons to all children 2 years of age and older

D. recommend adult supervision

E. use a water motion detection alarm in the pool

Copyright 2010 by the American Academy of Pediatrics page 611


2011 PREP SA on CD-ROM

Critique: 192 Preferred Response: B

Drowning is the second leading cause of unintentional injury-related death for children
younger than 14 years of age. Males are four times more likely to drown than females. In
children younger than 1 year, bathtubs, toilets, and buckets pose the greatest threat; toddlers
drown most commonly in home pools and adolescents in natural bodies of water. Minorities are
at particular risk, with Native American, African American, and Native Alaskan children
demonstrating a 1.2 to 1.8 times higher incidence than white children of the same age. This
disparity is presumably due to differential access to pools and sociocultural issues such as
value placed on swimming skills.
The most common location for drowning events involving toddlers is backyard swimming
pools. This is especially true in states such as California and Florida, where home pools are
common and used throughout the year. Most common in male toddlers, 75% of backyard
swimming pool deaths occur in children between the ages of 1 and 3 years and most deaths are
seen in pools owned by the family or friends.
Physical barriers between the child and the pool are the most effective means for
preventing swimming pool drownings. Experts recommend that the pool be surrounded by a four-
sided fence that is at least 4 feet high. The gate should open inward and be secured with a self-
closing latch or lock.
Although adult supervision is critical for children who are swimming, it is not adequate to
prevent drowning. In most cases of fatal drowning or near-drowning, the child was reported to
be under the supervision of an adult at the time of the event and out of sight for less than 5
minutes before being found.
Some new but limited data suggest that swimming lessons and water survival skills training
may lower drowning rates in 1- to 4-year-old children. However, there is insufficient evidence to
support a recommendation that all 1- to 4-year-old children receive swimming lessons.
Inflatable arm floats (or water wings) and other foam or air-filled floating devices are toys
and not intended to prevent drowning. Because most drowning events in toddlers are "silent"
(involve minimal splashing or water motion), pool motion detectors do not reliably alert adults to
an impending drowning event.

Suggested reading:

Brenner RA, Taneja GS, Haynie DL, et al. Association between swimming lessons and drowning
in childhood: a case-control study. Arch Pediatr Adolesc Med. 2009;163:203-210. Abstract
available at: http://www.ncbi.nlm.nih.gov/pubmed/19255386

Centers for Disease Control and Prevention. Unintentional Drowning: Fact Sheet. 2010. Available
at: http://www.cdc.gov/HomeandRecreationalSafety/Water-Safety/waterinjuries-factsheet.html

Committee on Injury, Violence and Poison Prevention. Policy Statement Prevention of


Drowning. Pediatrics. 2010; 126: e253 - e262. DOI: 10.1542/peds.2010-1265. Available at:
http://pediatrics.aappublications.org/cgi/reprint/126/1/e253

Copyright 2010 by the American Academy of Pediatrics page 612


2011 PREP SA on CD-ROM

United States Consumer Product Safety Commission. Safety Barrier Guidelines for Home Pools.
Washington, DC: United States Consumer Product Safety Commission; 2004. Available at:
http://www.cpsc.gov/cpscpub/pubs/pool.pdf

Copyright 2010 by the American Academy of Pediatrics page 613


2011 PREP SA on CD-ROM

Question: 193

You are called to the emergency department to see a 2 1/2-year-old boy at 9 pm on New Year's
Eve. His parents found him crying on the kitchen floor with an open, spilled container of granular
drain cleaner beside him. On physical examination, the boy is alert and screaming. His lips are
erythematous and edematous, but he can handle his secretions without difficulty.

Of the following, the MOST appropriate next step is

A. administration of broad-spectrum antibiotics

B. administration of intravenous corticosteroids

C. barium swallow study

D. observation in the hospital for 24 hours

E. upper gastrointestinal endoscopy

Copyright 2010 by the American Academy of Pediatrics page 614


2011 PREP SA on CD-ROM

Critique: 193 Preferred Response: E

More than 200,000 known exposures to household cleaning agents are reported annually in
the United States. Caustic ingestions represent an unknown fraction of these exposures,
although ingestions occur most commonly in children younger than 6 years of age, with most
affected children being 1 to 4 years old. The granular drain cleaner to which the toddler
described in the vignette has been exposed typically is formulated as a strong base (often
sodium hydroxide) with a pH of greater than 11.5. Despite evidence of oral burns (erythematous,
swollen lips), the boy apparently can handle secretions without difficulty. Nevertheless, any
patient who has signs of oral injury or symptoms of odynophagia/dysphagia (drooling, choking,
solid food refusal) following a known or suspected caustic ingestion should undergo upper
gastrointestinal endoscopy within 12 to 24 hours.
Acids and alkalis, including oven, toilet bowl, tile, and drain cleaners, comprise 50% of
ingested caustic agents; bleaches (30% to 40%) and laundry detergents (10% to 20%)
comprise the remainder. Fortunately, because of the use of childproof containers and the
enactment of laws to limit the concentration of household cleaning products, the overall
incidence of caustic ingestions in children appears to be declining. Nevertheless, these agents
are responsible for significant morbidity in young children and are associated with a high
mortality rate in adolescents and adults, in whom the ingestion most often represents a suicide
attempt.
Among pediatric patients, alkalis are ingested more commonly than acids and account for
most postingestion complications. Strong acids, which account for approximately 15% of
ingestions in children, cause immediate oral pain and have a bitter taste, thus limiting the quantity
of ingestion. These agents have a low viscosity and, therefore, undergo rapid transit to the
stomach, where the caustic injury is most severe. In fact, the esophagus is spared in 80% of
cases. Alkalis usually are odorless and tasteless, even after several swallows. As a
consequence, alkaline products are likely to be ingested in greater amounts. Alkali causes a
liquefaction necrosis rather than the coagulative necrosis from strong acids. The esophagus is
the site of greatest mucosal damage following alkali consumption, although gastric injury also
may occur, and the degree of injury is related to both the amount and concentration of the
material ingested. Products that have a pH greater than 11 (drain cleaner typically has a pH of
greater than 11.5) are associated with a high risk of caustic burns, and granular products cause
a higher rate of injury compared with liquids.
Postingestion evaluation and management depend on the nature of the ingestion and the
presence or absence of symptoms. Standard laundry bleach probably is the single most
commonly ingested household cleaning product. This agent contains 5% sodium hypochlorite
buffered to a pH of less than 11, and patients rarely require a diagnostic evaluation or specific
therapy following exposure. In contrast, endoscopic evaluation is indicated following the
ingestion of known caustic agents irrespective of the degree of symptoms. In some cases, the
ingested product may be swallowed rapidly (particularly with an intentional ingestion), and early
symptoms may be absent. Endoscopy undertaken before 12 hours after the incident may fail to
identify the severity and extent of injury, and delay of endoscopy more than 24 hours after the
incident can increase the risk of esophageal perforation.
Asymptomatic patients who present with a questionable history of corrosive ingestion may

Copyright 2010 by the American Academy of Pediatrics page 615


2011 PREP SA on CD-ROM

be observed and allowed to consume liquids. If symptoms of dysphagia develop, a barium


swallow study should be performed to assess for esophageal stricture. However, a barium
swallow study is not helpful early postingestion in symptomatic patients, such as the child
described in the vignette, because it cannot be used to determine the nature or extent of
esophageal injury.
Endoscopy permits grading of the injury and guides further management. Caustic injury to
the esophagus is graded on a scale of 0 to 3. Patients who have grades 0 and 1 injuries (grade
1 is seen in 80% of patients who have an endoscopically confirmed injury) may be sent home
after the procedure if they are tolerating feedings. Patients who have grades 2 to 3 lesions are
at greater risk for both short- and long-term complications, including bleeding and esophageal
strictures. Those who have grade 2 injuries require intravenous support until the extent of
damage becomes evident. They may be fed postendoscopy, as long as no signs of perforation
are noted. Grade 3 injuries, characterized by circumferential mucosal necrosis, pose the
greatest threat of early perforation and fistula formation and require long-term parenteral support
for administration of fluids and nutrition. Grade 2 lesions are associated with a greater than 10%
likelihood of subsequent esophageal stricture; stricture formation is likely with grade 3 lesions.
Pharmacologic management approaches to caustic injuries are either controversial
(corticosteroids) or have failed to undergo controlled clinical trials (antibiotics). Corticosteroids
may be used in patients who have airway symptoms related to perilaryngeal edema, although
their efficacy has not been fully established. Antibiotics, when prescribed, should be reserved
for patients who have grades 2 or 3 injuries, in cases where steroids are administered, and in
the setting of suspected or documented perforation.

Suggested reading:

Betalli P, Falchetti D, Giuliani S, et al. Caustic ingestion in children: is endoscopy always


indicated? The results of an Italian multicenter observational study. Gastrointest Endosc.
2008;68:434-439. DOI: 10.1016/j.gie.2008.02.016. Abstract available at:
http://www.ncbi.nlm.nih.gov/pubmed/18448103

Kay M, Wyllie R. Caustic ingestions in children. Curr Opin Pediatr. 2009;21:651-654. Abstract
available at: http://www.ncbi.nlm.nih.gov/pubmed/19543088

Kay M, Wyllie R. Symptoms may not adequately predict extent of injury in pediatric patients after
a caustic ingestion [letter]. Gastrointest Endosc. 2009;69:1407. DOI: 10.1016/j.gie.2008.10.003.
Available at: http://www.giejournal.org/article/PIIS0016510708027041/fulltext

Lamireau T, Rebouissoux L, Denis D, Lancelin F, Vergnes P, Fayon M. Accidental caustic


ingestion in children: is endoscopy always mandatory? J Pediatr Gastroenterol Nutr.
2001;33:81-84. Available at:
http://journals.lww.com/jpgn/Fulltext/2001/07000/Accidental_Caustic_Ingestion_in_Children__Is.1
4.aspx

Copyright 2010 by the American Academy of Pediatrics page 616


2011 PREP SA on CD-ROM

Turner A, Robinson P. Respiratory and gastrointestinal complications of caustic ingestion in


children. Emerg Med J. 2005;22:359361.

Wilsey MJ Jr, Scheimann AO, Gilger MA. The role of upper gastrointestinal endoscopy in the
diagnosis and treatment of caustic ingestion, esophageal strictures, and achalasia in children.
Gastrointest Endosc Clin N Am. 2001;11:767-787. Abstract available at:
http://www.ncbi.nlm.nih.gov/pubmed/11689365

Copyright 2010 by the American Academy of Pediatrics page 617


2011 PREP SA on CD-ROM

Question: 194

You are present at the birth of a term female infant who is delivered by elective repeat cesarean
section. The membranes are ruptured at delivery and the fluid is clear. She emerges with a good
cry but develops respiratory distress within minutes. Prompt administration of blow-by oxygen
fails to improve her grunting and cyanosis significantly. You transport her to the nursery for
evaluation. Physical examination reveals decreased breath sounds on the left and oxygen
saturation in her right hand of 92% while receiving 100% hood oxygen. You order a chest
radiograph (Item Q194).

Of the following, the MOST appropriate next step is the insertion of a(n)

A. chest tube

B. Foley catheter

C. orogastric catheter

D. percutaneous intravenous central catheter

E. umbilical arterial catheter

Copyright 2010 by the American Academy of Pediatrics page 618


2011 PREP SA on CD-ROM

Question: 194

(Courtesy of S Izatt)
Radiographic findings, as exhibited by the infant in the vignette.

Copyright 2010 by the American Academy of Pediatrics page 619


2011 PREP SA on CD-ROM

Critique: 194 Preferred Response: C

The infant described in the vignette has a left-sided congenital diaphragmatic hernia (CDH)
and requires the prompt insertion of an orogastric catheter for bowel decompression. CDH is the
incomplete closure of the diaphragm leading to herniation of bowel contents into the chest during
fetal development. It is seen in 1 in 2,500 to 3,000 live births. The three basic types of CDH are:
posterolateral herniation through the foramen of Bochdalek, anterior herniation through the
foramen of Morgagni, and hiatus hernia. The left-sided Bochdalek hernias constitute 85% of all
cases of CDH. Infants who have a CDH must be examined closely for other anomalies, which
are seen in 40% of cases. Up to 20% have an identifiable genetic cause. Importantly, 11% of
affected infants have serious cardiac problems.
Herniation of the bowel contents into the pulmonary cavity leads to variable degrees of
pulmonary hypoplasia. The pulmonary vascular bed has a decreased cross-sectional area as
well as muscularization of the intra-acinar pulmonary arteries. Such pathophysiology leads to
the two primary medical issues faced by the clinician: pulmonary hypoplasia and pulmonary
hypertension.
The infant who has CDH classically appears in the delivery room with a scaphoid abdomen
and barrel chest due to the displaced bowel contents (Item C194), although some infants may
not have this finding. Respiratory distress and cyanosis develop immediately in most cases, with
the clinical examination revealing decreased aeration and occasionally bowel sounds on the
affected side of the chest. If not diagnosed prenatally, the presentation may be confused with a
pneumothorax. Delivery room management includes intubation and prompt insertion of an
orogastric tube (or replogle). Placement of the orogastric tube and subsequent initiation of
suction minimizes air entry into the bowel, which prevents further respiratory compromise due to
dilation of the bowel loops in the chest with air. If the condition is identified subsequent to
delivery, the same initial steps in management are needed for stabilization.
After initial stabilization, a chest radiograph is needed for determination of endotracheal tube
and orogastric tube placement and assessment of the CDH. Gentle ventilation with permissive
hypercarbia decreases the incidence of pneumothorax initially and the subsequent incidence of
chronic lung disease. Central access that includes an umbilical arterial catheter for blood
pressure monitoring and blood drawing and an umbilical venous catheter for administration of
parenteral fluids and vasopressor agents is essential. Blood pressure should be maintained at
normal or higher levels due to underlying pulmonary hypertension. If central access for
parenteral fluids and vasopressors is not obtained, a peripherally inserted central catheter must
be considered. The infant should be transferred promptly to a tertiary center where inhaled nitric
oxide (treatment for pulmonary hypertension), extracorporeal membrane oxygenation, and
pediatric surgery consultation are available.
The survival rate for infants who have CDH is almost 70%, but declines to 40% if
associated congenital heart disease is present. Infants who survive are at risk for
neurodevelopmental issues, hearing loss, poor growth with feeding difficulties and
gastroesophageal reflux (GER), pectus excavatum, scoliosis, and chronic lung disease.
Neurologic outcome is good for most infants, with the greatest predictor of neurodevelopmental
issues being the need for oxygen at the time of discharge. Sensorineural hearing loss is seen in
approximately 50% of affected infants. Even if the hearing screening yields normal results at

Copyright 2010 by the American Academy of Pediatrics page 620


2011 PREP SA on CD-ROM

discharge, follow-up testing is recommended at 6 to 12 months of age. GER and growth failure
are seen in more than 50% of the infants, requiring close monitoring after discharge. Chronic
lung disease may be seen in more than 50% of the infants, with 25% of infants demonstrating
obstructive airway disease at 5 years of age.
The infant in the vignette does not require a chest tube because her chest radiograph does
not demonstrate the presence of a pneumothorax. A Foley catheter is not needed for
stabilization of the infant, although use of analgesic agents such as morphine may lead to
urinary retention and the subsequent need for catheter placement.

Suggested reading:

Cortes RA, Keller RL, Townsend T, et al. Survival of severe congenital diaphragmatic hernia has
morbid consequences. J Pediatr Surg. 2005;40:36-46. DOI: 10.1016/j.jpedsurg.2004.09.037.
Abstract available at: http://www.ncbi.nlm.nih.gov/pubmed/15868556

Dietz HC. Marfan syndrome. GeneReviews. 2009. Available at:


http://www.ncbi.nlm.nih.gov/bookshelf/br.fcgi?book=gene&part=marfan

Kays DW. Congenital diaphragmatic hernia: real improvements in survival. NeoReviews.


2006;7:e428-e439. Available at: http://neoreviews.aappublications.org/cgi/content/full/7/8/e428

Pober BR. Congenital diaphragmatic hernia overview. GeneReviews. 2010. Available at:
http://www.ncbi.nlm.nih.gov/bookshelf/br.fcgi?book=gene&part=cdh-ov

Copyright 2010 by the American Academy of Pediatrics page 621


2011 PREP SA on CD-ROM

Critique: 194

(Courtesy of the Media Lab at Doernbecher)


Herniation of bowel into the chest in patients who have congenital diaphragmatic hernia may
cause a scaphoid abdomen.

Copyright 2010 by the American Academy of Pediatrics page 622


2011 PREP SA on CD-ROM

Question: 195

A 14-year-old otherwise healthy boy developed an oval pink, scaly lesion on his back about 1
week ago. He presents today with widespread, moderately pruritic, scaling macules involving his
trunk (Item Q195) and upper extremities but sparing the scalp, face, palms, and soles. He admits
to being sexually active but has no other known exposures and takes no medications. Aside from
the rash, physical examination findings are normal.

Of the following, the MOST appropriate next step in management is to

A. obtain serology for Mycoplasma and Legionella

B. obtain serology for syphilis

C. perform a skin biopsy

D. prescribe a course of azithromycin

E. prescribe a topical antifungal cream

Copyright 2010 by the American Academy of Pediatrics page 623


2011 PREP SA on CD-ROM

Question: 195

(Courtesy of D Krowchuk)

Copyright 2010 by the American Academy of Pediatrics page 624


2011 PREP SA on CD-ROM

Critique: 195 Preferred Response: B

The patient described in the vignette has the typical history and physical findings of
pityriasis rosea. This inflammatory skin condition primarily affects children, adolescents, and
young adults in their 20s. The cause of the rash is unknown, but it has characteristics
suggesting an infectious source. It frequently follows a prodromal upper respiratory tract illness;
occasionally occurs in clusters; has a predictable course with resolution; and rarely recurs,
implying immunity. However, studies for known viruses generally have been unrevealing. Some
studies have found evidence for human herpesvirus 7, but subsequent evaluation could not
confirm this. Other infectious agents, including Legionella pneumophila and Mycoplasma, have
been considered but have not been identified consistently. Drug reactions also have been
described for a wide variety of medications, but these primarily have been single case reports.
The diagnosis of pityriasis rosea is based on clinical findings and course. Patients often
have a history of a mild respiratory illness preceding the first sign of the rash. The initial skin
finding, called a herald patch, is a 2- to 10-cm diameter oval, red, flat or minimally raised lesion
that has scale at the periphery (Item C195A). The herald patch can be mistaken for atopic
dermatitis or tinea corporis, and the diagnosis may not become clear until subsequent lesions
occur. Approximately 2 to 21 days after the appearance of the herald patch, crops of 5- to 10-
mm oval, salmon-colored thin plaques appear. These range in number from less than 50 to more
than 200 on the body. They are classically distributed along Langers lines, producing a
Christmas (fir) tree pattern on the back (Item C195B). They also may appear transversely across
the lower abdomen (Item C195C) and back, circumferentially around the shoulders, or in a V-
shaped pattern on the upper chest. Atypical forms of pityriasis rosea can appear in an inverse
pattern; not have a herald patch; or be pustular, vesicular, or giant lesions. African-American
children may have some variations not typical for white children, including more frequent scalp
and face involvement and more residual hyper- or hypopigmentation. Regardless of appearance,
the rash resolves spontaneously in 2 to 12 weeks.
Among the most important differential diagnoses for pityriasis rosea is secondary syphilis,
particularly for patients who have rash on the palms (Item C195D) and soles. Therefore,
screening for syphilis should be obtained for any sexually active patient, including the boy
described in the vignette. Other diagnostic considerations include atopic dermatitis, tinea
corporis, drug reactions, guttate psoriasis, lichen planus, and other rare dermatologic conditions.
The course and appearance of pityriasis rosea is typical and does not require any further
evaluation beyond physical examination. In rare cases that do not resolve in the expected time
frame or that have particularly unusual appearances, a skin biopsy might be indicated. Biopsy
findings for pityriasis rosea show nonspecific acute and chronic inflammatory changes and are
most useful to exclude other conditions in the differential diagnosis.
A substantial portion of patients who have pityriasis rosea (25% to 90%) experience
pruritus, and some children may have significant residual pigmentary changes. To control these
symptoms and shorten the course of the condition, many possible treatments have been
proposed. Practitioners have recommended emollients, topical and oral steroids, and topical and
oral antihistamines, but few of these treatments are consistently helpful. Topical antifungal
agents are effective for tinea corporis but have no effect on pityriasis rosea. Initial evidence
suggested that ultraviolet light may decrease rash duration and intensity of itch for pityriasis

Copyright 2010 by the American Academy of Pediatrics page 625


2011 PREP SA on CD-ROM

rosea, but follow-up studies did not confirm these effects. In another study, a 2-week course of
erythromycin resulted in a 73% resolution of the rash compared with no resolution for controls.
The resolution was presumed to be due to the anti-inflammatory effects of erythromycin.
However, a recent study in 50 children using azithromycin failed to show any benefit to this
treatment compared with placebo.

Suggested reading:

Amer A, Fischer H. Azithromycin does not cure pityriasis rosea. Pediatrics. 2006;117:1702-
1705. DOI: 10.1542/peds.2005-2450. Available at:
http://pediatrics.aappublications.org/cgi/content/full/117/5/1702

Amer A, Fischer H, Li X. The natural history of pityriasis rosea in black American children: how
correct is the "classic" description? Arch Pediatr Adolesc Med. 2007;161:503-506. Available at:
http://archpedi.ama-assn.org/cgi/content/full/161/5/503

Chuh AAT, Dofitas BL, Comisel G, et al. Interventions for pityriasis rosea. Cochrane Database
Syst Rev. 2009;2:CD005068. DOI: 10.1002/14651858.CD005068.pub2. Available at:
http://www.mrw.interscience.wiley.com/cochrane/clsysrev/articles/CD005068/frame.html

Hartley AH. Pityriasis rosea. Pediatr Rev. 1999;20:266-269. doi:10.1542/10.1542/pir.20-8-266.


Available at: http://pedsinreview.aappublications.org/cgi/content/full/20/8/266

Stulberg DL, Wolfrey J. Pityriasis rosea. Am Fam Physician. 2004;69:87-91. Abstract available
at: http://www.ncbi.nlm.nih.gov/pubmed/14727822

Copyright 2010 by the American Academy of Pediatrics page 626


2011 PREP SA on CD-ROM

Critique: 195

(Courtesy of D Krowchuk)
The herald patch of pityriasis rosea is erythematous and flat or minimally raised. Often there is
scale at the periphery of the lesion.

Copyright 2010 by the American Academy of Pediatrics page 627


2011 PREP SA on CD-ROM

Critique: 195

(Courtesy of D Krowchuk)
On the back, the lesions of pityriasis rosea are aligned with their long axes parallel to Langers
lines of skin cleavage. This distribution mimics the appearance of the branches of a fir tree.

Copyright 2010 by the American Academy of Pediatrics page 628


2011 PREP SA on CD-ROM

Critique: 195

(Courtesy of D Krowchuk)
On the abdomen, the lesions of pityriasis rosea may be distributed transversely, with long axes
parallel to the lines of skin cleavage. Note that in this patient, as in many who are more deeply
pigmented, some lesions appear as papules, not plaques.

Copyright 2010 by the American Academy of Pediatrics page 629


2011 PREP SA on CD-ROM

Critique: 195

(Reprinted with permission from Hartley AH. Pityriasis rosea. Pediatr Rev. 1999;20:266-270)
Copper-colored macules involving the palms and soles may be observed in secondary syphilis.

Copyright 2010 by the American Academy of Pediatrics page 630


2011 PREP SA on CD-ROM

Question: 196

A 14-year-old girl presents with her mother for concerns about a mass in her left breast of 6
months duration. The girl had thelarche at age 10 and menarche at age 12. There is a strong
family history of both breast cancer and fibrocystic breast disease in young women. Physical
examination reveals symmetric Sexual Maturity Rating 5 breast development, with a 1 x 1-cm
mass palpable in the left breast just adjacent to the areola that is discrete from the rest of the
breast. There are no other lumps or lesions and no overlying skin changes. There is no drainage
or secretion from the breast upon gentle squeezing of the nipple.

Of the following, the BEST approach to evaluation of this teens breast mass is

A. breast ultrasonography

B. digital mammography

C. excisional biopsy

D. fine-needle biopsy

E. reassurance

Copyright 2010 by the American Academy of Pediatrics page 631


2011 PREP SA on CD-ROM

Critique: 196 Preferred Response: A

Many young girls present to the pediatricians office with concern about a breast mass.
Often, this is the unintended consequence of widespread media efforts to raise awareness for
breast cancer in the adult female population. In young girls, a breast mass usually represents a
benign structure such as the breast tissue itself (often asymmetric in development), fibrocystic
breast disease, or normally sized axillary lymph nodes that are palpable. Rarely, breast cancer
does occur in adolescents, most commonly in girls who have first-degree relatives who also
had breast cancer at a young age. Although self-examination remains controversial in terms of
validity and reliability for early cancer detection, supporting the awareness of breast cancer
screening in the young woman seems prudent.
Because of the growth of the lesion and the family history of breast cancer for the girl
described in the vignette, reassurance is not appropriate. The lesion should be evaluated with
ultrasonography because this modality is noninvasive and does not involve the relative high
dose of radiation related to mammography. Screening mammography is not recommended for
women younger than 30 years of age and is not helpful in the diagnosis of a breast mass in this
age group because the density of the breast tissue makes it difficult to distinguish between
benign and malignant conditions. If ultrasonography is not informative, the girl should be referred
to a surgeon skilled in management of adolescent breast lesions who can perform a fine-needle
biopsy if indicated. The evaluations should not include overly invasive procedures that might
involve excision of breast tissue and result in an inability to breastfeed in the future or significant
and unnecessary cosmetic asymmetry.

Suggested reading:

Aksglaede L, Srensen K, Petersen JH, Skakkebaek NE, Juul A. recent decline in age at breast
development: The Copenhagen Puberty Study. Pediatrics. 2009;123:e932-e939. DOI:
10.1542/peds.2008-2. Available at:
http://pediatrics.aappublications.org/cgi/content/full/123/5/e932

Arca MJ, Caniano DA. Breast disorders in the adolescent patient. Adolesc Med Clin.
2004;15:473-485. Abstract available at: http://www.ncbi.nlm.nih.gov/pubmed/15625988

Bock K, Duda VF, Hadji P, et al. Pathologic breast conditions in childhood and adolescence:
evaluation by sonographic diagnosis. J Ultrasound Med. 2005;24:1347-1354. Abstract available
at: http://www.ncbi.nlm.nih.gov/pubmed/16179617

Cavanaugh RM Jr. Screening adolescent gynecology in the pediatrician's office: have a listen,
take a look. Pediatr Rev. 2007;28:332-342. DOI: 10.1542/10.1542/pir.28-9-332. Available at:
http://pedsinreview.aappublications.org/cgi/content/full/28/9/332

Copyright 2010 by the American Academy of Pediatrics page 632


2011 PREP SA on CD-ROM

Question: 197

A 14-month-old boy presents to the emergency department for evaluation of respiratory distress,
recent fever, cough, and congestion. He has received all of his vaccinations. Physical
examination shows tachycardia, poor perfusion, and hepatomegaly. Chest radiography
documents an enlarged heart with hazy lung markings. Electrocardiography reveals sinus
tachycardia. Echocardiography identifies severe left ventricular dilation and systolic dysfunction,
with mitral regurgitation. You suspect dilated cardiomyopathy.

Of the following, the MOST likely cause is

A. bacterial pneumonia leading to bacterial myocarditis

B. congenital coronary artery anomaly

C. Duchenne muscular dystrophy

D. previously undetected supraventricular tachycardia

E. viral myocarditis due to enterovirus

Copyright 2010 by the American Academy of Pediatrics page 633


2011 PREP SA on CD-ROM

Critique: 197 Preferred Response: E

Myocarditis, defined as inflammation of the myocardium, most often is due to infection from
common viruses. The central feature is the development of a myocardial inflammatory infiltrate,
and myocyte necrosis might develop, but the condition typically results in a dilated
cardiomyopathy, as described for the boy in the vignette. The consequences of myocarditis vary
widely. Disease may be mild and asymptomatic, resolving without treatment, or it may result in
congestive heart failure, poor cardiac output, arrhythmias, and death. Signs and symptoms of
myocarditis include fever, chest pain, abdominal distress, peripheral edema, respiratory
compromise, hepatic congestion, and palpitations. Symptoms in infants and toddlers tend to be
more nonspecific and include generalized malaise, poor appetite, abdominal pain, and cough.
Later stages of the illness present with respiratory symptoms and increased work of breathing
that often is mistaken for asthma. Because myocarditis frequently is due to a viral illness, many
patients have a history of symptoms consistent with a recent viral infection, including fever,
rash, diarrhea, joint pain, and easy fatiguing. Myocarditis often is associated with pericarditis.
Although other common noninflammatory causes of cardiomyopathy must be excluded,
myocardial inflammation can be suspected on the basis of elevated C-reactive protein or
erythrocyte sedimentation rate and echocardiographic criteria of left ventricular dilation,
myocardial edema, and ventricular dysfunction. Markers of myocardial damage (troponin or
creatine kinase cardiac-specific isoenzymes) are elevated. Viral polymerase chain reaction
(PCR) testing of stool (in the case of a gastrointestinal symptom complex) or the respiratory tract
(in the case of respiratory infectious symptomatology) can be used to make a presumptive
diagnosis. The gold standard remains biopsy of the myocardium, assessed via light microscopy,
immunochemical methods, and PCR. Histopathologic features are myocardial edema and an
inflammatory infiltrate rich in lymphocytes and macrophages. Focal destruction of myocytes
explains the myocardial pump failure.
Among the causative viral pathogens are enterovirus, coxsackievirus, adenovirus,
parvovirus B19, human immunodeficiency virus, and cytomegalovirus. Bacterial causes are rare,
except for patients who have immunodeficiency. Pathogens include Brucella, Corynebacterium
diphtheriae, Haemophilus influenzae, and Borrelia burgdorferi (Lyme disease). Fungal
pathogens are rare, except in the immunocompromised population.
Toxins causing myocarditis include ethanol, carbon monoxide, anthracyclines and other
forms of chemotherapy, some antipsychotics (eg, clozapine), and immunologic reactions to
acetazolamide and amitriptyline. Among other causes are autoimmune reactions, such as that
seen in scleroderma and systemic lupus erythematosus.
Because most viral infections cannot be treated with directed therapy, symptomatic
treatment is the primary intervention. In the acute phase, this includes inotropic support, afterload
reduction, and diuretics for symptomatic patients. All patients who have evidence of elevated
acute-phase reactants should receive intravenous immune globulin, and corticosteroids should
be considered. Chronic therapy is directed at the treatment of dilated cardiomyopathy, including
treatment with angiotensin-converting enzyme inhibitors (captopril, lisinopril). Patients who do
not respond to conventional therapy are candidates for bridge therapy with left ventricular
assist devices. Heart transplantation is reserved for patients who fail to improve with
conventional therapy.

Copyright 2010 by the American Academy of Pediatrics page 634


2011 PREP SA on CD-ROM

Congenital coronary anomalies such as coronary fistula can cause myocardial steal and
associated myocardial ischemia. Anomalous coronary artery arising from the pulmonary artery
can result in similar findings to myocarditis. Affected children develop a dilated cardiomyopathy
and demonstrate symptoms of heart failure as well as echocardiographic findings of left
ventricular dilation and dysfunction. However, the infectious signs and symptoms described for
this child do not correlate with such a cause. Duchenne muscular dystrophy results in a dilated
cardiomyopathy, but cardiomyopathic changes are atypical until late childhood or early
adolescence. Supraventricular tachycardia (in addition to all forms of atrial tachyarrhythmia) can
cause a dilated cardiomyopathy if the tachyarrhythmia is present for sufficient duration to cause
myocardial fatigue. For this boy, the presence of fever and the absence on electrocardiography
of supraventricular tachycardia argue against this diagnosis.

Suggested reading:

Bohn D, Benson L. Diagnosis and management of pediatric myocarditis. Paediatr Drugs.


2002;4:171-181. Abstract available at: http://www.ncbi.nlm.nih.gov/pubmed/11909009

Freedman SB, Haladyn JK, Floh A, Kirsh JA, Taylor G, Thull-Freedman J. Pediatric myocarditis:
emergency department clinical findings and diagnostic evaluation. Pediatrics. 2007;120:1278-
1285. DOI: 10.1542/peds.2007-1073. Available at:
http://pediatrics.aappublications.org/cgi/content/full/120/6/1278

Klugman D, Berger JT, Sable CA, He J, Khandelwal SG, Slonim AD. Pediatric patients hospitalized
with myocarditis: a multi-institutional analysis. Pediatr Cardiol. 2010;31:222-228. DOI:
10.1007/s00246-009-9589-9. Abstract available at:
http://www.ncbi.nlm.nih.gov/pubmed/19936586

Leonard EG. Viral myocarditis. Pediatr Infect Dis J. 2004;23:665-666

Levi D, Alejos J. An approach to the treatment of pediatric myocarditis. Paediatr Drugs.


2002;4:637-647. Abstract available at: http://www.ncbi.nlm.nih.gov/pubmed/12269840

Copyright 2010 by the American Academy of Pediatrics page 635


2011 PREP SA on CD-ROM

Question: 198

A 17-year-old boy who has spina bifida, hydrocephalus, multiple prior tethered cord releases,
and neurogenic bladder presents to the emergency department 2 weeks after hospital discharge
following scoliosis surgery. He has new complaints of bilateral hand tremor, upper back and
shoulder pain, and hand numbness. He has been unable to walk since age 8 years. He has had
bowel and bladder dysfunction for many years, which is stable. General physical examination
reveals no new findings. Neurologic examination documents normal mental status, cranial nerves,
upper limb strength, and reflexes. He has slightly decreased sensation to pain and temperature,
but proprioception and vibratory sense in his hands are normal. A regular tremor is evident when
his hands are outstretched and on finger-to-nose testing, with the tremor greater on the left than
the right. You decide to obtain magnetic resonance imaging of his cervical and thoracic spinal
cord.

Of the following, the MOST likely finding of such a study is a(n)

A. extramedullary spinal tumor posterior to the spinal cord

B. herniation of multiple cervical vertebral discs

C. intramedullary spinal tumor

D. stroke in the distribution of the anterior spinal artery

E. syrinx at multiple cervical levels

Copyright 2010 by the American Academy of Pediatrics page 636


2011 PREP SA on CD-ROM

Critique: 198 Preferred Response: E

The adolescent described in the vignette has classic symptoms of spinal cord dysfunction
from spina bifida, with loss of both bowel and bladder control, inability to walk, and lower body
sensory loss. Such dysfunction can result from congenital spinal cord anomalies, as in this
case, or from a variety of disease processes affecting the spinal cord.
Within the setting of congenital spine abnormalities, the new complex of symptoms he has in
the hands is most consistent with a new problem in the upper spinal cord (ie, the cervical spine),
where the brachial plexus nerves emerge to innervate the hands. The cause of the new
problems is most likely the development of a symptomatic syrinx. A syrinx is a fluid-filled cavity in
the central spinal cord, commonly in the cervical region (Item C198). As it grows, it compresses
the contents of the cord from within, initially affecting fibers carrying pain and temperature
information. Localized pain in a capelike distribution over the back, hand tremors, and eventually
distal hand wasting may occur. Symptoms often emerge in adolescence or adulthood. Spinal
cord tumors may present with localized pain and tenderness to percussion as well as motor
loss, bowel and bladder loss, and sensory loss below a dermatome. Tumors may be
intramedullary (within the spinal cord) or extramedullary (in the meninges surrounding the cord).
The pattern of sensory loss, with preservation of vibration and proprioception, described for this
boy is not typical of a spinal cord tumor. In addition, a tumor probably would have been identified
during prior neuroimaging, particularly at the time of his surgery.
Herniation of multiple cervical discs may be caused by trauma and other chronic factors but
would not be typical in an adolescent who has spina bifida. The clinical presentation may include
numbness, tingling, or pain due to compression by disc material of nerves exiting the spinal cord.
A stroke in the spinal artery usually results from atherosclerosis and, therefore, is less likely
for this boy. The presentation involves a sudden-onset anterior cord syndrome, with bilateral
weakness as well as loss of pain and temperature sensation. Although the sensory symptom
complex is similar to that of a syrinx, the time course of spinal artery stroke is acute, and there is
prominent initial weakness.

Suggested reading:

Fenichel GM. Paraplegia and quadriplegia. In: Clinical Pediatric Neurology: A Signs and
Symptoms Approach. 6th ed. Philadelphia, Pa: Saunders Elsevier; 2009:267-284

Hakimi KN, Massagli TL. Anterior spinal artery syndrome in two children with genetic thrombotic
disorders. J Spinal Cord Med. 2005;28:69-73. Abstract available at:
http://www.ncbi.nlm.nih.gov/pubmed/15832907

Haslam RHA. Spinal cord disorders. In: Kliegman RM, Behrman RE, Jenson HB, Stanron BF, eds.
Nelson Textbook of Pediatrics. 18th ed. Philadelphia, P1: Saunders Elsevier; 2007:2526-2530

McGirt MJ, Chaichana KL, Atiba A, Attenello F, Yao KC, Jallo GI. Resection of intramedullary spinal
cord tumors in children: assessment of long-term motor and sensory deficits. J Neurosurg
Pediatr. 2008;1:63-67. DOI: 10.3171/PED-08/01/063. Abstract available at:

Copyright 2010 by the American Academy of Pediatrics page 637


2011 PREP SA on CD-ROM

http://www.ncbi.nlm.nih.gov/pubmed/18352805

Menkes JH, Ellenbogen RC. Traumatic brain and spinal cord injuries in children. In: Maria BL, ed.
Current Management in Child Neurology. 3rd ed. Hamilton, Ontario, Canada: BC Decker;
2005:515-527

Nejat F, Radmanesh F, Ansari S, Tajik P, Kajbafzadeh A, El Khashab M. Spina bifida occulta: is it a


predictor of underlying spinal cord abnormality in patients with lower urinary tract dysfunction?
J Neurosurg Pediatr. 2008;1:114-117. DOI: 10.3171/PED/2008/1/2/114. Abstract available at:
http://www.ncbi.nlm.nih.gov/pubmed/18352778

Copyright 2010 by the American Academy of Pediatrics page 638


2011 PREP SA on CD-ROM

Critique: 198

(Courtesy of D Gilbert)
A syrinx is a fluid-filled cavity in the central spinal cord (arrows).

Copyright 2010 by the American Academy of Pediatrics page 639


2011 PREP SA on CD-ROM

Question: 199

You are called to the newborn nursery to evaluate a term female infant who has dysmorphic
facial features. She was born to a woman who received no prenatal care. Labor and delivery
were uncomplicated. The infant has normal weight, length, and head circumference. On physical
examination, you document notched lower eyelids, hypoplastic zygomatic arches, malar
hypoplasia, a hypoplastic ear on one side with two preauricular tags on the same side, and
retrognathia. The remainder of the findings are normal.

Of the following, the condition with which this infants features are MOST consistent is

A. diabetic embryopathy

B. Pierre Robin sequence

C. Stickler syndrome

D. Treacher Collins syndrome

E. 22q11 deletion syndrome

Copyright 2010 by the American Academy of Pediatrics page 640


2011 PREP SA on CD-ROM

Critique: 199 Preferred Response: D

The infant described in the vignette has features of mandibulofacial dysostosis, a


description that applies to a group of disorders in which there is underdevelopment of the
mandible and variable facial and ear anomalies. The best characterized of the mandibulofacial
dysostoses is the Treacher Collins syndrome (TCS).
TCS is an autosomal dominant condition that is highly variable and is caused by
spontaneous gene mutations in 60% of affected individuals. Unusual features associated with
TCS include downslanting eyes with lower lid coloboma (notching) and absent eyelashes medial
to the notch, malar hypoplasia, hypoplastic/malformed external ears with conductive and/or
sensorineural hearing loss, and microretrognathia that has the potential to compromise the
airway. Features may be highly variable, even within the same family.
TCS is known to be associated with only one gene, TCOF1, and most individuals who meet
clinical diagnostic criteria for TCS have detectable mutations in this gene. Clinical molecular
testing is available and may serve to establish recurrence risk for very mildly affected or
apparently unaffected individuals.
The management of TCS is supportive. Special attention should be given to the airway
because of the potential for choanal stenosis/atresia or glossoptosis (posterior displacement of
the tongue). The newborn should be examined carefully for cleft palate and swallowing
difficulties. Formal audiologic evaluation is important to establish a baseline and to detect
possible hearing loss. Ophthalmology referral is indicated in early infancy due to increased risk
for corneal damage, strabismus, and refractive errors. Craniofacial computed tomography scan
with three-dimensional reconstruction may be indicated in the first 6 months to define anatomy
better in anticipation of future treatments such as plastic surgery.
Diabetic embryopathy can include malformation/hypoplasia of the ears. However, affected
individuals do not have notched lower eyelids or malar hypoplasia, as seen in TCS, and they
frequently have heart defects and other anomalies.
Pierre Robin sequence (retrognathia, retroglossia, and cleft palate) may be isolated or may
be part of a bigger picture, such as Stickler (Item C199) and 22q11 deletion syndromes. In each
case, there is marked retrusion of the mandible, but the eye findings associated with TCS are
not present, and although the ears may be unusual in 22q11 deletion, typically they are not
severely malformed.

Suggested reading:

Jones MC. Treacher Collins syndrome. In: Cassidy SB, Allanson JE, eds. Management of
Genetic Syndromes. 2nd ed. Hoboken, NJ: John Wiley & Sons; 2005:547-553

Katsanis SH, Cutting GR. Treacher Collins syndrome. GeneReviews. 2006. Available at:
http://www.ncbi.nlm.nih.gov/bookshelf/br.fcgi?book=gene&part=tcs

Copyright 2010 by the American Academy of Pediatrics page 641


2011 PREP SA on CD-ROM

Critique: 199

(Courtesy of T Jewett)
Retrognathia, rounded facies, and myopia are features of Stickler syndrome.

Copyright 2010 by the American Academy of Pediatrics page 642


2011 PREP SA on CD-ROM

Question: 200

You are the preceptor in a pediatric clinic and a resident has just finished seeing an adolescent
male for a routine physical examination. This is the young mans first visit in the clinic, and as part
of the routine screening, the resident takes a comprehensive psychosocial history. The patient
reveals that he is more attracted to males than females. You guide the resident on the range of
issues she should address.

Of the following, the MOST immediate attention is required for

A. contraception

B. depression

C. harassment

D. social isolation

E. substance use

Copyright 2010 by the American Academy of Pediatrics page 643


2011 PREP SA on CD-ROM

Critique: 200 Preferred Response: B

Two thirds of physicians reportedly never ask their patients about sexual orientation. The
importance of exploring this area lies in knowing that gay, lesbian, bisexual, and transgendered
(GLBT) youth are at significantly increased risk for engaging in self-destructive behaviors,
primarily as a response to feeling marginalized and discriminated against.
Suicide is the third leading cause of death among adolescents, but it is the leading cause of
death among GLBT youth. Therefore, although these youth are at risk for harassment, social
isolation, and substance use, it is most important to screen them carefully for depression and
suicidal ideation because they are at higher risk for more frequent and serious suicide attempts.
Fear of discrimination at home, school, or work often leads to social isolation among these youth,
and verbal harassment and threats of physical violence are common. In addition, these youth are
at risk for homelessness, prostitution, sexual abuse, and substance use and misuse. Their risk
for pregnancy or acquisition of sexually transmitted infections depends on the behaviors in
which they engage. The need for contraception should be discussed with all youth, regardless
of their sexual orientation.

Suggested reading:

Cavanaugh RM Jr. Screening adolescent gynecology in the pediatrician's office: have a listen,
take a look. Pediatr Rev. 2007;28:332-342. DOI: 10.1542/10.1542/pir.28-9-332. Available at:
http://pedsinreview.aappublications.org/cgi/content/full/28/9/332

Garofalo R, Katz E. Health care issues of gay and lesbian youth Curr Opin Pediatr. 2001;13:298-
302. Abstract available at: http://www.ncbi.nlm.nih.gov/pubmed/11717552

Gay, lesbian and transgender health. Medline Plus. 2010. Available at:
http://www.nlm.nih.gov/medlineplus/gaylesbianandtransgenderhealth.html

Perrin EC, Cohen KM, Gold M, Ryan C, Savin-Williams RC, Schorzman CM. Gay and lesbian
issues in pediatric health care. Curr Probl Pediatr Adolesc Health Care. 2004;34:355-398

US Preventive Services Task Force. Screening and treatment for major depressive disorder in
children and adolescents: US Preventive Services Task Force recommendation statement.
Pediatrics. 2009;123:1223-1228. DOI: 10.1542/peds.2008-2381. Available at:
http://pediatrics.aappublications.org/cgi/content/full/123/4/1223

Williams SB, O'Connor EA, Eder M, Whitlock EP. Screening for child and adolescent depression in
primary care settings: a systematic evidence review for the US Preventive Services Task Force.
Pediatrics. 2009;;123:e716-e735. DOI: 10.1542/peds.2008-2415. Available at:
http://pediatrics.aappublications.org/cgi/content/full/123/4/e716

Copyright 2010 by the American Academy of Pediatrics page 644


2011 PREP SA on CD-ROM

Question: 201

You are seeing a 14-year-old girl who has mild persistent asthma and idiopathic scoliosis. She is
scheduled for an anteroposterior spinal fusion next week. Her mother asks you if her daughters
asthma is a significant risk factor for this surgery.

Of the following, the postoperative complication for which patients who have asthma are MOST
at risk is

A. atelectasis

B. laryngospasm

C. pneumonia

D. pneumothorax

E. respiratory failure

Copyright 2010 by the American Academy of Pediatrics page 645


2011 PREP SA on CD-ROM

Critique: 201 Preferred Response: B

Asthma remains the most common pediatric respiratory disorder in the United States and is a
significant health and economic burden. The classic asthma physiologic response consists of
airway inflammation, hyperresponsiveness, remodeling, and obstruction. Asthma can produce a
variety of acute pulmonary complications, including atelectasis, bronchospasm, laryngospasm,
pneumothorax, pneumomediastinum, and respiratory failure.
Although the risk of perioperative complications among children who have asthma has been
shown to be increased over the general population, the overall risk remains low (2%) among
those whose asthma is well-controlled. Bronchospasm and laryngospasm represent most of the
complications seen in the perioperative setting. Although air leak syndromes such as
pneumothorax or pneumomediastinum historically have been associated with asthma, newer
ventilation techniques, such as low tidal volumes and permissive hypercapnia, have resulted in
significant reductions in mortality and morbidity in the hospitalized patient who has asthma. The
exact incidence of air leak syndromes in patients who have asthma is difficult to define but has
been estimated at less than 0.5% in outpatients and up to 5% for patients hospitalized with
severe asthmatic attacks in the past. Although patients who have asthma do have an increased
risk of invasive bacterial pneumonia compared with those not affected with asthma, their
perioperative risk of pneumonia is considerably less than the risks of bronchospasm and
laryngospasm.

Suggested reading:

Bohn D, Kissoon N. Acute Asthma. Pediatr Crit Care Med. 2001;2:151-163. Abstract available
at: http://www.ncbi.nlm.nih.gov/pubmed/12797875

Liu AH, Covar RA, Spahn JD, Leung DYM. Childhood asthma. In: Kliegman RM, Behrman RE,
Jenson HB, Stanton BF, eds. Nelson Textbook of Pediatrics. 18th ed. Philadelphia, Pa: Saunders
Elsevier; 2007:953-969

Copyright 2010 by the American Academy of Pediatrics page 646


2011 PREP SA on CD-ROM

Question: 202

The father of a 3-year-old girl carries her into your office. The child looks dehydrated and is
breathing rapidly. Point-of-care blood glucose measures 563 mg/dL (31.2 mmol/L). You arrange
for immediate transfer to the emergency department to initiate evaluation and treatment for
diabetic ketoacidosis.

Of the following, the factor that increases this childs risk of severe complications of diabetic
ketoacidosis during treatment is a

A. father who developed type 1 diabetes at age 12 years

B. history of 5 days of increasing illness and obtundation

C. 3-hour delay in starting an insulin infusion

D. venous serum pH of 7.25

E. white blood cell count of 12.0x103/mcL (12.0x109/L) with 75% polymorphonuclear leukocytes

Copyright 2010 by the American Academy of Pediatrics page 647


2011 PREP SA on CD-ROM

Critique: 202 Preferred Response: B

Cerebral edema is a known complication of the treatment of diabetic ketoacidosis (DKA) and
diabetic coma and complicates the management of up to 1% of children who have diabetic
ketoacidosis. The more severe and prolonged the acidosis, the greater the likelihood of this
complication. Risk factors for cerebral edema include young age and prolonged length of time
before diagnosis and treatment. Accordingly, a history of 5 days of increasing illness and
obtundation would increase the risk for the girl described in the vignette.
The cause of cerebral edema is unclear, but acetoacetate and beta hydroxybutyrate have
been implicated in an animal model as activators of the sodium-potassium chloride (Na-K-Cl)
exchange pump and, therefore, may be etiologic in the development of cerebral edema.
Pretreatment with bumetanide, which blocks the Na-K-Cl cotransporter, has prevented the
development of cerebral edema in this model.
Cerebral edema may be a form of reperfusion edema following cerebral ischemia. Of
interest, in one case-control study, early treatment with insulin in the first 2 hours after
diagnosing DKA worsened the risk for cerebral edema.
The risk to a child of developing type 1 diabetes is about 6% if the father has type 1
diabetes and 4% if the mother has type 1 diabetes. However, having a parent who has diabetes
does not increase a child's risk of developing severe DKA. A venous serum pH of 7.25 does not
indicate severe acidosis and suggests an arterial pH of about 7.3. The severity of acidosis
seems to correlate in some studies with the risk of development of cerebral edema, making
relatively mild acidosis a good prognostic sign. Stress-related release of catecholamines usually
leads to a leukocytosis with a left shift to more polymorphonuclear leukocytes in children who
have DKA. This finding rarely is associated with bacterial infection and in itself is not a risk
factor for the development of complications of DKA.

Suggested reading:

Edge JA, Jakes RW, Roy Y, et al. The UK case-control study of cerebral oedema complicating
diabetic ketoacidosis in children. Diabetologia. 2006;49:2002-2009. DOI: 10.1007/s00125-006-
0363-8. Available at: http://www.springerlink.com/content/j9042v55n8556267/fulltext.html

Glaser N. Cerebral injury and cerebral edema in children with diabetic ketoacidosis: could
cerebral ischemia and reperfusion injury be involved? Pediatr Diabetes. 2009;10:534-541. DOI:
10.1111/j.1399-5448.2009.00511.x

Wolfsdorf J, Craig ME, Daneman D, et al. Diabetic ketoacidosis in children and adolescents with
diabetes. Pediatr Diabetes. 2009(suppl 12):10:118-133

Copyright 2010 by the American Academy of Pediatrics page 648


2011 PREP SA on CD-ROM

Question: 203

A 9-year-old boy has failing grades in his math and reading classes. He achieved his motor and
language milestones at appropriate ages. His teacher reports that he is a compliant and likeable
boy who tries hard but continually has difficulty on tests. Although he brings home the necessary
books, his parents must help him complete his daily homework assignments. He loves to be
outdoors and engages in competitive sports. His health has been good, and he has had no
change in his activity level.

Of the following, the MOST likely diagnosis is

A. attention-deficit/hyperactivity disorder

B. depressive disorder

C. intellectual disability

D. obsessive-compulsive disorder

E. specific learning disability

Copyright 2010 by the American Academy of Pediatrics page 649


2011 PREP SA on CD-ROM

Critique: 203 Preferred Response: E

The child described in the vignette is having academic problems only in math and reading
that likely are due to learning disabilities. A number of other conditions may result in academic
difficulty. A child who has intellectual disability typically presents with delays in developmental
milestones and struggles in all academic subjects. Children who have attention problems
generally exhibit distractibility and disorganization. A child who has shown a recent lack of
interest for activities or friends might raise concern about depression. Of note, a child who has
a learning disability is at risk for problems with self-esteem. Obsessive-compulsive disorder may
cause academic difficulties, but the boy in the vignette does not exhibit any of the behaviors
characteristic of this condition.
A child who has academic issues should undergo a comprehensive physical examination to
rule out visual and auditory problems, disordered sleep, nutritional deficiencies, thyroid disease,
history of an acquired neurologic insult, or epilepsy. An older child should be evaluated for use
of illicit drugs or alcohol.

Suggested reading:

Committee on Children with Disabilities. The pediatricians role in development and implementation
of an Individual Education Plan (IEP) and Individual Family Service Plan (IFSP). Pediatrics.
1999;104:124-127. Available at: http://pediatrics.aappublications.org/cgi/content/full/104/1/124

Lambros KM, Leslie LK. Management of the child with a learning disorder. Pediatr Ann.
2005;34;:275-287. Abstract available at: http://www.ncbi.nlm.nih.gov/pubmed/15871432

Stein MT, Perrin JM. Diagnosis and treatment of ADHD in school-age children in primary care
settings: a synopsis of the AAP practice guidelines. Pediatr Rev. 2003;24:92-98. DOI:
10.1542/10.1542/pir.24-3-92. Available at:
http://pedsinreview.aappublications.org/cgi/content/full/24/3/92

Copyright 2010 by the American Academy of Pediatrics page 650


2011 PREP SA on CD-ROM

Question: 204

A 6-day-old infant is brought to the emergency department in August with a 1-day history of
decreased feeding, decreased activity, tactile fever, and rapid breathing. He was born at term by
normal spontaneous vaginal delivery and weighed 3,742 g. His mother reports that she had a
nonspecific febrile illness 1 week before delivery for which she received no treatment. Her group
B Streptococcus screen was positive at 36 weeks gestation, and she received two doses of
ampicillin (>4 hours apart) during labor. The baby received no antibiotics and was discharged at
48 hours of age. Physical examination today reveals a toxic, lethargic infant who is grunting and
has a temperature of 39.4C, heart rate of 180 beats/min, and respiratory rate of 60 breaths/min.
His lungs are clear, with subcostal retractions. He has a regular heart rhythm with gallop, his
pulses are thready, his capillary refill is 4 seconds, and his extremities are cool. His liver edge is
decreased 3 cm.

Of the following, the MOST likely cause of this babys illness is

A. early-onset group B Streptococcus infection

B. echovirus 11 infection

C. herpes simplex virus infection

D. hypoplastic left heart syndrome

E. respiratory syncytial virus infection

Copyright 2010 by the American Academy of Pediatrics page 651


2011 PREP SA on CD-ROM

Critique: 204 Preferred Response: B

The ubiquitous human nonpolio enteroviruses (coxsackievirus A and B, echoviruses, and


enteroviruses) are responsible for a wide range of diseases in persons of all ages. In older
children and adults, they are associated most with mild febrile illness, viral exanthems (eg, hand-
foot-and-mouth disease), herpangina, and aseptic meningitis. More serious manifestations of
infections with these viruses may include encephalitis and myocarditis. Enterovirus infections in
neonates may present with a spectrum of illness, ranging from an unapparent infection or mild,
nonspecific febrile illness to encephalitis or a full blown sepsislike illness. Differentiating
enteroviral illness from bacterial infection in the neonate may be difficult. The history of recent
maternal illness, benign birth history without prolonged rupture of membranes, benign birth
course, and presence of fever in the infant described in the vignette are seen more often with
enteroviral disease in the neonate than bacterial disease.
Although the mother had a positive result for group B streptococcal (GBS) infection on
prenatal screening, she received appropriate two-dose antibiotic therapy at time of delivery,
which greatly diminishes the likelihood of the baby developing early-onset GBS infection. The
baby also had a negative blood culture initially. However, enteric gram-negative sepsis or other
bacterial infection still must be considered, and antibiotic coverage pending results of cultures is
appropriate.
Neonatal herpes simplex virus (HSV) infection may present as a sepsislike illness. Although
no genital lesions were described for this mother, up to 75% of neonates who have HSV
disease are born to mothers who have no clinical findings of active HSV infection. The season
of the year and nonspecific illness described in the mother, however, argue for enteroviral
infection as the more likely diagnosis. It still is reasonable, though, to order surface HSV cultures
and obtain cerebrospinal fluid for HSV polymerase chain reaction testing on the baby and
administer acyclovir pending these results.
Hypoplastic left heart syndrome may present with shock as the patent ductus arteriosus
closes, but the fever in this infant argues for infection as the cause of shock.
Respiratory syncytial virus (RSV) infection in neonates may present with apnea out of
proportion to respiratory signs and symptoms or with a sepsislike picture. However, high fever,
evidence of shock, and the presence of maternal illness 1 week prior to delivery are not typical
with neonatal RSV infection.

Suggested reading:

American Academy of Pediatrics. Enterovirus (nonpoliovirus) infections. In: Pickering LK, Baker
CJ, Kimberlin DW, Long SS, eds. Red Book: 2009 Report of the Committee on Infectious
Diseases. 28th ed. Elk Grove Village, Ill: American Academy of Pediatrics; 2009:287-288

Lin TY, Kao HT, Hsieh SH, et al. Neonatal enterovirus infections: emphasis on risk factors of
severe and fatal infections. Pediatr Infect Dis J. 2003;22:889-894. Abstract available at:
http://www.ncbi.nlm.nih.gov/pubmed/14551490

Verboon-Maciolek MA, Nijhuis M, Van Loon AM, et al. Diagnosis of enterovirus infection in the

Copyright 2010 by the American Academy of Pediatrics page 652


2011 PREP SA on CD-ROM

first 2 months of life by real-time polymerase chain reaction. Clin Infect Dis. 2003;37:1-6. DOI:
10.1086/375222. Available at: http://www.journals.uchicago.edu/doi/full/10.1086/375222

Copyright 2010 by the American Academy of Pediatrics page 653


2011 PREP SA on CD-ROM

Question: 205

A 17-year-old girl has been hospitalized for 3 weeks due to complicated peritonitis following a
ruptured appendix. She has undergone laparotomy and drainage of intra-abdominal abscesses
and improved while receiving intravenous ampicillin, gentamicin, and clindamycin. Her nasogastric
tube was removed last week, and she has been tolerating a bland diet. Today, however, she
complains of abdominal pain and distention and has had three episodes of diarrhea. Physical
examination reveals a febrile adolescent who has a diffusely tender abdomen.

Of the following, the stool test MOST likely to establish the diagnosis is

A. bacterial culture

B. evaluation for ova and parasites

C. fecal occult blood

D. toxin assay

E. viral culture

Copyright 2010 by the American Academy of Pediatrics page 654


2011 PREP SA on CD-ROM

Critique: 205 Preferred Response: D

The teenage girl in the vignette has been hospitalized and receiving intravenous antibiotic
therapy for a prolonged period of time following gastrointestinal tract surgery. She has
developed colitis, probably due to Clostridium difficile.
C difficile colitis usually is diagnosed by testing stool for the presence of C difficile toxins.
Commercially available tests can detect toxins A and B by enzyme immunoassay. Because the
toxin degrades rapidly at room temperature, stool specimens should be tested promptly or stored
at 4oC until testing can be performed. Pseudomembranes and friable colorectal mucosa on
endoscopy also suggest the diagnosis. The colon can be colonized in the absence of disease,
making bacterial culture of stool unhelpful. Evaluation of the stool for occult blood would not be
diagnostic because of the many causes of blood in the stool and bloody diarrhea. A stool study
for ova and parasites would not be useful because a patient who has been hospitalized for a
prolonged period of time is unlikely to have developed a gastrointestinal tract infection due to a
parasite. Nosocomial viral infections of the gastrointestinal tract do occur, and a stool viral
culture may be appropriate for a patient in whom this is suspected. However, severe abdominal
distention and pain in a patient who has undergone recent abdominal surgery and received
prolonged antibiotics is much more suggestive of C difficile colitis than a viral infection.
The spectrum of illness caused by C difficile ranges from asymptomatic carriage to watery
diarrhea to pseudomembranous colitis. Asymptomatic carriage of toxin-producing strains can
occur at any age but is common in newborns and infants younger than 1 year of age. Patients
who have colitis generally present with fever, abdominal pain and cramps, and diarrhea and
appear systemically ill. Toxic megacolon and intestinal perforation are complications of
pseudomembranous colitis and are more likely to be fatal in infants who have underlying
gastrointestinal disease (Hirschsprung disease or inflammatory bowel disease) or are
immunocompromised.
Risk factors for developing C difficile disease include antimicrobial therapy, underlying
bowel disease, gastrointestinal tract surgery, prolonged nasogastric tube insertion, repeated
enemas, and renal insufficiency. C difficile colitis has been associated with many antibiotics,
especially beta-lactam drugs, clindamycin, fluoroquinolones, and macrolides, but it can occur
following therapy with any antimicrobial agent. Some patients present with no history of recent
antibiotic exposure. Prolonged hospitalization, rooming with an infected patient, and being on the
same hospital ward as a symptomatically infected patient increase the risk for acquiring C
difficile. Transmission of the organism can be prevented by exercising hand hygiene, limiting
antibiotic use, properly disposing of contaminated materials, and adequately cleaning
contaminated surfaces. Alcohol-based hand hygiene products and many common hospital
disinfectants do not eradicate C difficile spores; hand hygiene using soap and water and
adequate friction is preferred. Diluted bleach solutions are best for cleaning and decontaminating
surfaces. Children who have C difficile diarrhea should be excluded from group activities,
including child care, for the duration of the diarrhea.
Metronidazole (30 mg/kg per day in four divided doses; maximum, 2 g/day) administered
orally or intravenously is effective and the drug of choice for the initial treatment of C difficile
colitis in children and adolescents. Oral vancomycin (40 mg/kg per day in four divided doses;
maximum, 125 mg 4 times/day) can be used with or without metronidazole as the initial therapy

Copyright 2010 by the American Academy of Pediatrics page 655


2011 PREP SA on CD-ROM

for patients who have severe disease or can be used alone for those who do not respond to
metronidazole. Therapy should be administered for at least 10 days.

Suggested reading:

American Academy of Pediatrics. Clostridium difficile. In: Pickering LK, Baker CJ, Kimberlin DW,
Long SS, eds. Red Book: 2009 Report of the Committee on Infectious Diseases. 28th ed. Elk
Grove Village, Ill: American Academy of Pediatrics; 2009:263-265

Ferry GD, Versalovic J. Antibiotic-associated colitis. In: Feigin RD, Cherry JD, Demmler-Harrison
GJ, Kaplan SL, eds. Feigin & Cherrys Textbook of Pediatric Infectious Diseases. 6th ed.
Philadelphia, Pa: Saunders Elsevier; 2009:653-659

Copyright 2010 by the American Academy of Pediatrics page 656


2011 PREP SA on CD-ROM

Question: 206

A 3-year-old boy presents with abdominal discomfort and occasional vomiting. The only finding of
note on physical examination is a blood pressure of 104/58 mm Hg. Urinalysis reveals:

Specific gravity, 1.020


pH, 6
3+ blood
Negative for protein, nitrite, and leukocyte esterase
10 to 20 red blood cells/high-power field (hpf)
Fewer than 5 white blood cells/hpf

Upon further questioning, the mother states that she has a history of "blood in her urine" and two
maternal uncles required dialysis in their teenage years.

Of the following, the MOST likely cause for this patients urinary findings is

A. Alport nephritis

B. autosomal dominant polycystic kidney disease

C. juvenile nephronophthisis

D. Lowe syndrome

E. thin glomerular basement membrane disease

Copyright 2010 by the American Academy of Pediatrics page 657


2011 PREP SA on CD-ROM

Critique: 206 Preferred Response: A

The young boy described in the vignette has undergone a urinalysis as part of the
evaluation for his abdominal pain, and it shows microscopic hematuria. More importantly, the
family history obtained subsequent to the detection of hematuria is positive for microscopic
hematuria in the mother and early-onset end-stage renal disease (ESRD) in two maternal uncles.
Two features of this family history that should be helpful to the clinician in evaluating a possible
genetic disease are the presence of disease in successive generations and males appearing to
be affected more severely than females. A genetic disease that has a milder presentation in
females raises the possibility of an X-linked disorder. One such disorder that presents as
microscopic hematuria in young boys, progressing to hematuria with proteinuria by 10 years of
age and ESRD in the adolescent and young adult years, is Alport syndrome (AS). ESRD has
been reported in 50% to 90% of affected males by age 30 years, depending on the severity of
the genetic defect. High-frequency sensorineural hearing loss occurs in approximately 50% of
patients. Ophthalmologic findings have been reported as well; anterior lenticonus, a defect of the
lens bowing into the anterior chamber, is the most common abnormality.
AS is inherited as an X-linked disease in 85% of cases. The genetic defect involves the
COL4A5 gene on the X chromosome; a mutation at this locus encoding for alpha-5 chain of type
IV collagen leads to AS. As noted, the X-linked inheritance pattern means that males often
present earlier than females and generally have more severe disease. Therefore, males are
more likely than females to develop chronic renal failure. Similar to this case, the evaluation of a
male child who has persistent asymptomatic hematuria should include a dipstick urinalysis of the
mother to screen for hematuria. Based on the genetics of this disorder, 50% of the male children
from female carriers are affected.
The remaining 15% of cases of AS are autosomally inherited (recessive or dominant). These
forms are caused by genetic defects of the alpha-3 or alpha-4 chain of type IV collagen.
At present, there is no specific treatment for AS. Nephrologists focus on blood pressure
control and reduction of proteinuria, often with the use of angiotensin-converting enzyme
inhibitors. Patients reaching ESRD are candidates for renal transplantation.
The disease most often confused with AS is thin glomerular basement membrane (GBM)
disease. Like AS, thin GBM disease is familial and presents with asymptomatic microscopic
hematuria. It is attributed to a defect of the GBM causing a disruption of the glomerular capillary
barrier within the nephron. The disruption to the barrier explains the microscopic hematuria
characteristic of both disorders. Unlike AS, thin GBM disease tends to affect males and females
equally and, therefore, is not an X-linked disorder. Recently, thin GBM disease has been traced
to a heterozygous state for the gene defect of alpha-3 or alpha-4 of type IV collagen, which
explains why it has a much more favorable prognosis than either X-linked or autosomal forms of
AS.
Other genetic diseases that have renal manifestations should be considered but are less
likely for this patient. Autosomal dominant polycystic kidney disease (ADPKD) can present with
microscopic hematuria, is familial, and can lead to ESRD. However, the autosomal inheritance of
ADPKD results in males and females being affected equally. Juvenile nephronophthisis (NPH) is
an autosomal recessive disorder characterized by chronic tubulointerstitial disease and
manifesting clinically with polyuria and polydipsia and laboratory features of anemia and

Copyright 2010 by the American Academy of Pediatrics page 658


2011 PREP SA on CD-ROM

azotemia; hematuria or proteinuria often is absent. Lowe syndrome is an X-linked disorder that
presents with congenital cataracts, Fanconi syndrome (metabolic acidosis, glycosuria,
hypophosphatemia, hypokalemia, and amino aciduria), and intellectual disability.

Suggested reading:

Jais JP, Knebelmann B, Giatras I, et al. X-linked Alport syndrome: natural history in 195 families
and genotype-phenotype correlations in males. J Am Soc Nephrol. 2000;11:649-657. Available
at: http://jasn.asnjournals.org/cgi/content/full/11/4/649

Kashtan CE. Familial hematurias: what we know and what we don't. Pediatr Nephrol.
2005;20:1027-1035. DOI: 10.1007/s00467-005-1859-z. Abstract available at:
http://www.ncbi.nlm.nih.gov/pubmed/15856317

Massengill SF. Hematuria. Pediatr Rev. 2008;29:342-348. DOI: 10.1542/10.1542/pir.29-10-342.


Available at: http://pedsinreview.aappublications.org/cgi/content/full/29/10/342

Roth KS, Amaker BH, Chan JC. Pediatric hematuria and thin basement membrane nephropathy:
what is it and what does it mean? Clin Pediatr (Phila). 2001;40:607-613. Abstract available at:
http://www.ncbi.nlm.nih.gov/pubmed/11758961

Savige J, Rana K, Tonna S, Buzza M, Dagher H, Wang YY. Thin basement membrane
nephropathy. Kidney Int. 2003;64:1169-1178. DOI: 10.1046/j.1523-1755.2003.00234.x. Available
at: http://www.nature.com/ki/journal/v64/n4/full/4494008a.html

Copyright 2010 by the American Academy of Pediatrics page 659


2011 PREP SA on CD-ROM

Question: 207

A 5-year-old girl presents to the emergency department with acute wheezing and coughing. She
appears in mild respiratory distress, with tachypnea and nasal flaring. Her room air pulse
oximetry reading is 92%. Her parents state that "this always happens with a cold" and they have
been administering an over-the-counter oral decongestant over the past 2 days for coughing and
rhinorrhea. On physical examination, you note decreased air movement bilaterally and expiratory
wheezing.

Of the following, the MOST appropriate initial treatment is

A. administration of an inhaled beta2 agonist

B. administration of an inhaled corticosteroid

C. administration of heliox therapy

D. bilateral needle thoracentesis

E. rapid-sequence intubation

Copyright 2010 by the American Academy of Pediatrics page 660


2011 PREP SA on CD-ROM

Critique: 207 Preferred Response: A

One of the primary concerns during an acute asthma exacerbation is whether the patient is
having respiratory failure or impending fatal asthma. A patient whose dyspnea interferes with
speaking, who is perspiring, or who has a peak expiratory flow of less than 25% of expected
may require admission to the intensive care unit and intubation. The patient in the vignette, who
has signs and symptoms of respiratory distress but does not have signs of respiratory failure,
should receive nebulization treatments, supplemental oxygen, and initiation of oral or intravenous
corticosteroids. Her initial management should be aimed at decreasing airway obstruction,
correcting hypoxemia, and preventing further progression of symptoms. Immediate goals include
close patient monitoring, oxygen supplementation to maintain oxygen saturation at 90% or
greater, and administration of bronchodilators. Inhaled beta2 agonist via nebulization or a
metered-dose inhaler administered every 20 minutes for 1 hour should be the initial therapy.
Inhaled anticholinergics, which are less potent bronchodilators than beta agonists, may provide
additional bronchodilation and have been shown to improve lung function and reduce
hospitalization rates.
Administration of intravenous or oral corticosteroids is recommended during an asthma
exacerbation; both delivery methods result in similar outcomes in children hospitalized for
asthma. Patients who are taking daily inhaled corticosteroids may benefit from doubling or
quadrupling the dose at the onset of the exacerbation. However, this dose increase is not well
studied in children, and oral or intravenous corticosteroids still are recommended.
Needle thoracentesis at the second intercostal space along the mid-clavicular line is the
treatment of choice for a tension pneumothorax. Symptoms of a tension pneumothorax include
reduced or absent breath sounds, pleuritic chest pain, cyanosis, and shifting of mediastinal
structures. The bilateral wheezing heard on auscultation of the girl in the vignette is consistent
with airway obstruction due to asthma rather than a bilateral pneumothorax.
The indications for intubation for asthma include apnea, coma, or semielectively before an
impending respiratory crisis. Patients who have asthma and require intubation should be treated
differently from patients who are intubated but do not have asthma. Because of the prolonged
expiratory phase with asthma, setting the ventilatory rate to match the patients tachypnea does
not allow sufficient time to expel carbon dioxide. An inappropriately high respiratory rate can
lead to increased thoracic pressure, increased positive end-expiratory pressure, and iatrogenic
pneumothorax.
Two additional therapies, intravenous magnesium sulfate and heliox, should be considered
for patients who have life-threatening asthma exacerbations or whose severe symptoms
persist despite initial inhaled bronchodilator therapy. Heliox, a mixture of helium and oxygen, may
improve oxygen delivery in those who have severe obstruction, but it does not have a role in the
initial management of asthma. In addition, a Cochrane review did not support its use. Magnesium
sulfate relaxes smooth muscle and has demonstrated benefit as an adjunctive therapy during a
severe asthma exacerbation.
Methylxanthines, antibiotics, aggressive hydration, chest physical therapy, mucolytics, and
sedation currently are not recommended treatment modalities for an asthma exacerbation.

Suggested reading:

Copyright 2010 by the American Academy of Pediatrics page 661


2011 PREP SA on CD-ROM

Boyd M, Lasserson TJ, McKean MC, Gibson PG, Ducharme FM, Haby M. Interventions for
educating children who are at risk of asthma-related emergency department attendance.
Cochrane Database Syst Rev. 2009;2:CD001290. DOI: 10.1002/14651858.CD001290.pub2.
Available at:
http://www.mrw.interscience.wiley.com/cochrane/clsysrev/articles/CD001290/frame.html

Rodrigo GJ, Pollack CV, Rodrigo C, Rowe BH. Heliox for nonintubated acute asthma patients.
Cochrane Database Syst Rev. 2006;4:CD002884. DOI: 10.1002/14651858.CD002884.pub2.
Available at:
http://www.mrw.interscience.wiley.com/cochrane/clsysrev/articles/CD002884/frame.html

Rowe BH, Bretzlaff J, Bourdon C, Bota G, Blitz S, Camargo CA. Magnesium sulfate for treating
exacerbations of acute asthma in the emergency department. Cochrane Database Syst Rev.
2000;1:CD001490. DOI: 10.1002/14651858.CD001490. Available at:
http://www.mrw.interscience.wiley.com/cochrane/clsysrev/articles/CD001490/frame.html

Copyright 2010 by the American Academy of Pediatrics page 662


2011 PREP SA on CD-ROM

Question: 208

A 2-year-old boy is brought to the emergency department after his father found him with the leaf
from a foxglove plant in his mouth (Item Q208). He has had one episode of emesis and is
complaining of abdominal pain. On physical examination, his heart rate is 140 beats/min,
respiratory rate is 24 breaths/min, blood pressure is 100/60 mm Hg, and oxygen saturation is
100%. His pupils are 4 mm and briskly reactive to 2 mm. The remainder of his examination findings
are normal.

After administering activated charcoal, the MOST appropriate next step is

A. abdominal radiography

B. electrocardiography

C. serum creatine phosphokinase assessment

D. serum sodium assessment

E. urine toxicology screening

Copyright 2010 by the American Academy of Pediatrics page 663


2011 PREP SA on CD-ROM

Question: 208

(Courtesy of M Wright)
Foxglove

Copyright 2010 by the American Academy of Pediatrics page 664


2011 PREP SA on CD-ROM

Critique: 208 Preferred Response: B

More than 100,000 plant exposures are reported to the American Association of Poison
Control Centers annually, most of which are not serious. Although there are many potentially
toxic plants in the environment, only a few account for most of the fatalities and emergency
department visits. In addition, most of these occur in adults who intentionally ingest the plant, use
it for perceived medicinal purposes, or mistake the plant for a nontoxic look-alike.
In general, the toxicities of plants affect one or more of three organ systems: cardiac,
neurologic, or gastrointestinal (Item C208). As with most poisonings, evaluation and treatment
are directed at decontamination and assessment/support of vital functions. Plant identification is
helpful in determining a treatment plan (and can be facilitated by the local Poison Control Center),
but it should not delay the initial evaluation and decontamination.
Because most serious plant ingestions have cardiac effects, electrocardiography should be
considered in affected patients. This is especially true for the patient described in the vignette,
who has ingested foxglove, a source of potent cardiac glycosides. Abdominal radiography,
serum sodium determination, and urine toxicology screening are of no utility. Rhabdomyolysis
may be a late finding in foxglove ingestion. A serum creatine phosphokinase determination may
be considered if the patient develops muscle tenderness or myoglobinuria.

Suggested reading:

Franklin RL, Rodgers GB. Unintentional child poisonings treated in United States hospital
emergency departments: national estimates of incident cases, population-based poisoning rates,
and product involvement. Pediatrics. 2008;122:1244-1251. DOI: 10.1542/peds.2007-3551.
Available at: http://pediatrics.aappublications.org/cgi/content/full/122/6/1244

Froberg B, Ibrahim D, Furbee RB. Plant poisoning. Emerg Med Clin North Am. 2007;25:375-433.
DOI: 10.1016/j.emc.2007.02.013. Abstract available at:
http://www.ncbi.nlm.nih.gov/pubmed/17482026

Copyright 2010 by the American Academy of Pediatrics page 665


2011 PREP SA on CD-ROM

Critique: 208

Copyright 2010 by the American Academy of Pediatrics page 666


2011 PREP SA on CD-ROM

Question: 209

You are caring for a 16-year-old girl who has juvenile idiopathic arthritis. Her musculoskeletal
symptoms have come under good control using naproxen sodium. Recently, however, she has
been complaining of vague abdominal pain and occasional loose bowel movements. Physical
examination demonstrates an alert, cooperative adolescent in no distress whose vital signs are
normal for her age. She has mild, direct tenderness in the epigastric region, and rectal
examination produces a scant amount of brown stool that is positive on fecal occult blood test.
Laboratory data include:

Hemoglobin, 10.5 g/dL (105 g/L)


White blood cell count, 4.5x10 3/mcL (4.5x109/L)
Erythrocyte sedimentation rate, 25 mm/hr
Albumin, 3.4 g/dL (34 g/L)

Of the following, the MOST appropriate additional medication for this patient is

A. celecoxib

B. ibuprofen

C. methotrexate

D. misoprostol

E. sucralfate

Copyright 2010 by the American Academy of Pediatrics page 667


2011 PREP SA on CD-ROM

Critique: 209 Preferred Response: D

Nonsteroidal anti-inflammatory drugs (NSAIDs) remain first-line therapy for the management
of many rheumatic diseases because of their anti-inflammatory and analgesic properties.
However, chronic use is associated with an increased frequency of gastrointestinal complaints,
peptic ulceration, and complications that include bleeding and perforation. NSAID-related small
bowel enteropathy may result in enteric losses of both blood and protein. Although the arthritis
symptoms of the 16-year-old girl described in the vignette are under control with naproxen, she
has developed symptoms that are consistent with an NSAID-induced gastroenteropathy. To
prevent NSAID-related complications, several treatment strategies have been recommended as
either therapeutic adjuncts or alternatives to standard, cyclooxygenase-1 (COX-1) inhibitory
NSAID therapy, including switching to a COX-2 selective inhibitor or adding a gastric
cytoprotective agent. The synthetic prostaglandin-2 analog misoprostol, a cytoprotective agent,
has demonstrated efficacy in reducing the frequency of gastric and duodenal ulcers among
pediatric patients who require long-term NSAID therapy.
Celecoxib is a selective COX-2 inhibitor. Both as monotherapy and coupled with proton pump
inhibitors (PPIs), COX-2 inhibitors have demonstrated reduced gastrointestinal adverse effects
during management of rheumatologic diseases. However, several COX-2 inhibitors have been
removed from the market because of potential cardiovascular complications, so that their use
must be balanced against potential risks. Furthermore, the safety and effectiveness of COX-2
inhibitors in long-term use have not been confirmed in children. Ibuprofen is a COX-1 inhibitor that
has a similar risk profile to that of naproxen. Methotrexate, a tetrahydrofolate reductase inhibitor,
is a potent immunomodulator that may be used as second-line therapy in patients who have
rheumatic diseases that cannot be controlled by alternative medications. Finally, sucralfate is a
locally acting agent that forms a protective gel over inflamed and ulcerated gastroduodenal
mucosa, thus accelerating healing. It often is employed as adjunctive therapy in the treatment of
peptic ulcer disease, and its potential use for the girl in the vignette would be solely during acute
management.
Adult studies have documented endoscopically confirmed ulcer prevalence rates as high as
15% to 30% among patients receiving NSAIDs, with the highest rate of ulceration occurring
during the first 6 months of therapy. Overall, the incidence of NSAID-related gastroduodenal
complications has been estimated at 2% per year. Dyspepsia appears to be the most common
symptom accompanying long-term treatment, affecting up to 25% of patients. In a retrospective
review of 700 children who had juvenile idiopathic arthritis and were treated with a variety of
anti-inflammatory drugs, 0.8% developed symptoms of a secondary NSAID gastropathy.
Although frank peptic ulceration and associated complications of bleeding and perforation are
not frequent events in pediatric patients, they are of particular concern because NSAID-induced
ulcers often are not preceded by typical symptoms of ulcer disease.
The mechanism responsible for gastric and small intestinal mucosal damage from NSAIDs is
likely multifactorial and includes reduction in local blood flow, direct irritant effects, and inhibition
of mucosal healing. As stated previously, NSAIDs are inhibitors of COX-1, a constitutive enzyme
of the gastric mucosa that mediates synthesis of prostaglandins such as PGE-2. Such inhibition
leads to reductions in gastric blood flow, mucus production, and bicarbonate secretion and
results in impaired gastroduodenal mucosal barrier function. Misoprostol, a PGE-2 analog, exerts

Copyright 2010 by the American Academy of Pediatrics page 668


2011 PREP SA on CD-ROM

its gastrointestinal cytoprotective effects by directly countering a primary mechanism


responsible for the damaging effects of naproxen and other COX-1 inhibitors. Among pediatric
patients receiving NSAIDs, coadministation of misoprostol has been shown to be an effective
strategy to reduce gastrointestinal symptoms and increase hemoglobin concentrations.
Because the girl in the vignette displays signs and symptoms consistent with peptic ulcer
disease, immediate management may include diagnostic endoscopy. Confirmation of
gastroduodenal damage should prompt therapy with a PPI. In fact, although not offered as an
option, the addition of a PPI has been shown to be effective in reducing gastrointestinal
complications among patients requiring anti-inflammatory medications.

Suggested reading:

Inoue M, Ito S. Drug-induced bowel injury. In: Kleinman RE, Goulet O, Mieli-Vergani G, Sanderson
I, Sherman P, Shneider B, eds. Walkers Pediatric Gastrointestinal Disease. 5th ed. Hamilton,
Ontario, Canada: BC Decker; 2008:739-744

Lanas A. Prevention and treatment of NSAID-induced gastroduodenal injury. Curr Treat Options
Gastroenterol. 2006;9:147-156. Abstract available at:
http://www.ncbi.nlm.nih.gov/pubmed/16539875

Regula J, Butruk E, Dekkers CP, et al. Prevention of NSAID-associated gastrointestinal lesions: a


comparison study pantoprazole versus omeprazole. Am J Gastroenterol. 2006;101:1747-1755.
DOI: 10.1111/j.1572-0241.2006.00686.x. Abstract available at:
http://www.ncbi.nlm.nih.gov/pubmed/16817839

Scheiman JM. Prevention of NSAID-induced ulcers. Curr Treat Options Gastroenterol.


2008;11:125-134. Abstract available at: http://www.ncbi.nlm.nih.gov/pubmed/18321440

Targownik LE, Metge CJ, Leung S, Chateau DG. The relative efficacies of gastroprotective
strategies in chronic users of nonsteroidal anti-inflammatory drugs. Gastroenterology.
2008;134:937-944. DOI: 10.1053/j.gastro.2008.01.010. Abstract available at:
http://www.ncbi.nlm.nih.gov/pubmed/18294634

Copyright 2010 by the American Academy of Pediatrics page 669


2011 PREP SA on CD-ROM

Question: 210

A woman who has chronic hypertension presents for a prenatal pediatric visit at 32 weeks
gestation. She is coming in early because her obstetrician has said that she is likely to deliver
prematurely. She has a previous history of an intrauterine demise at 34 weeks gestation that
was attributed to "a small placenta." During your discussion, she asks how her baby can be
monitored prenatally.

Of the following, the MOST appropriate statement regarding evaluation of her fetus is that

A. amniotic fluid volume assessment can be used to predict perinatal outcome

B. biophysical profile testing can be used to assess fetal growth

C. contraction stress testing can be used to look for evidence of uteroplacental insufficiency

D. home uterine activity monitoring can be used to decrease preterm delivery and neonatal
complications

E. nonstress testing can be used to document flow through the umbilical vessels

Copyright 2010 by the American Academy of Pediatrics page 670


2011 PREP SA on CD-ROM

Critique: 210 Preferred Response: C

The mother described in the vignette has chronic hypertension is at risk for uteroplacental
insufficiency, and may require a contraction stress test (CST) for assessment of fetal well-
being. Medical conditions associated with uterine ischemia such as chronic hypertension are
associated with uteroplacental insufficiency and intrauterine growth restriction. It has been
estimated that more than 50% of unexplained stillbirths near term may be related to restricted
growth and poor placental function. Mothers at risk should be followed closely with antenatal
fetal surveillance that includes measurement of fetal growth as well as monitoring of placental
function.
Growth of the at-risk fetus is monitored by serial ultrasonography that is used to measure
biparietal diameter, head circumference, abdominal circumference, and femur length for
estimation of fetal weight and correlation to existing standards. Further assessment of fetal well-
being is performed using the nonstress test (NST). During the NST, the fetal heart rate is
monitored to observe accelerations that occur with fetal movement. The heart rate accelerates
unless the fetus is acidotic or neurologically depressed. A reactive NST includes two or more
fetal heart rate accelerations within a 20-minute period. Results of this study should be
interpreted cautiously at less than 28 weeks gestation because up to 50% of the NSTs may not
be reactive. The NST does not assess flow through the umbilical vessels.
If the NST is nonreactive, further evaluation is indicated. One study that may be performed is
the CST, which examines the response of the fetal heart rate to uterine contractions and can
suggest evidence of uteroplacental insufficiency. Nipple stimulation or intravenous oxytocin is
used to initiate uterine contractions if less than three spontaneous contractions occur in a 10-
minute period. The test is considered negative if there are no late or significant variable
decelerations with at least three moderate uterine contractions and positive if there are late
decelerations with at least 50% of the contractions. A positive CST suggests that the worsened
oxygenation during a contraction negatively affects an already compromised fetus and supports
consideration of early delivery.
Biophysical profile testing is an additional tool that may be used to assess fetal well-being
but does not assess fetal growth. It is a score composed of the results of the NST, fetal
breathing movements, fetal body movements, fetal reflex movements, and amniotic fluid volume.
Doppler assessment of umbilical artery flow also may be used to assess placental function in
uteroplacental insufficiency. These two fetal assessment tools frequently are used in
combination with the NST. Amniotic fluid volume assessment can reflect the uterine environment,
but it is not predictive of outcome. Unfortunately, home uterine activity monitoring has not been
found to decrease preterm delivery in at-risk mothers.

Suggested reading:

ACOG practice bulletin. Antepartum fetal surveillance. Number 9, October 1999. Clinical
management guidelines for obstetrician-gynecologists. Int J Gynecol Obstet. 2000;68:175-185.
Abstract available at: http://www.ncbi.nlm.nih.gov/pubmed/10717828

Devoe LD. Antenatal fetal assessment: contraction stress test, nonstress test, vibroacoustic

Copyright 2010 by the American Academy of Pediatrics page 671


2011 PREP SA on CD-ROM

stimulation, amniotic fluid volume, biophysical profile, and modified biophysical profile-an
overview. Semin Perinatol. 2008;32:247-252. DOI:10.1053/j.semperi.2008.04.005. Abstract
available at: http://www.ncbi.nlm.nih.gov/pubmed/18652922

Miller J, Turan S, Baschat AA. Fetal growth restriction. Semin Perinatol. 2008;32:274-280. DOI:
10.1053/j.semperi.2008.04.010. Abstract available at:
http://www.ncbi.nlm.nih.gov/pubmed/18652928

Turan S, Miller J, Baschat AA. Integrated testing and management in fetal growth restriction.
Semin Perinatol. 2008;32:194-200. DOI: 10.1053/j.semperi.2008.02.008. Abstract available at:
http://www.ncbi.nlm.nih.gov/pubmed/18482621

Copyright 2010 by the American Academy of Pediatrics page 672


2011 PREP SA on CD-ROM

Question: 211

A 12-year-old boy has been his baseball teams star pitcher for 2 years. Over the past month, he
has noticed the gradual onset of elbow pain with throwing as well as a decrease in control and
pitch velocity. On physical examination, he has tenderness over the medial epicondyle and a slight
flexion contracture at the elbow.

Of the following, the MOST likely cause for this condition is

A. avascular necrosis of the capitellum

B. avulsion fracture of the medial epicondyle

C. fracture of the radial head

D. medial epicondyle apophysitis

E. olecranon apophyseal injury

Copyright 2010 by the American Academy of Pediatrics page 673


2011 PREP SA on CD-ROM

Critique: 211 Preferred Response: D

"Little League elbow" is a general term that has come to mean any elbow pain in a
skeletally immature athlete who participates in an "overhead" sport such as baseball, softball,
swimming, or gymnastics. However, as for the patient described in the vignette, classically and
most commonly this condition refers to apophysitis of the medial epicondyle in 9- to 12-year-old
athletes. It is estimated that 20% to 40% of baseball players in this age range experience elbow
pain while playing.
Growth of the elbow is complex and involves six auxiliary ossification centers developing
over a period of years. The apophyses can be recalled using the mnemonic "CRITOE":
capitellum, radial head, internal (medial) epicondyle, trochlea, olecranon, and external (lateral)
epicondyle (Item C211A). The capitellar apophysis begins to ossify at age 1 to 2 years, and the
subsequent centers appear approximately every 2 years later. Fusion of these centers is
complete by approximately 14 to 16 years of age, with girls completing growth 1 to 2 years
earlier than boys.
Because these physes have not fused in the young athlete, they are particularly susceptible
to the forces involved in overhead sports, especially throwing. The throw goes through multiple
steps: wind-up, cocking (early and late), acceleration (early and late), deceleration, and follow-
through. During cocking and acceleration phases, the valgus overload on the elbow is
substantial, causing tensile stress medially, compression stress laterally, and shear stress
posteriorly on the elbow. The ulnar collateral ligament (UCL) and the common flexor tendon
provide the greatest support for the elbow during these movements, and in adults, the UCL is
susceptible to injury (Item C211B). However, in children, the apophyses are damaged more
frequently by both acute events and more often by repetitive injury. Although improper technique
and inadequate muscle strength and balance contribute to injury, overuse probably has the
greatest impact. Indeed, one of the best predictors of elbow pain in young athletes is the number
of pitches they have thrown in a season.
Evaluation of the young athlete who has elbow pain begins with a history that includes
questions about hand dominance, position played, number of pitches per week, types of pitches
(ie, does he or she throw breaking pitches), number of teams played on, and amount of rest
between outings. The athlete may complain of decreased throwing velocity, distance, or
accuracy concurrent with the elbow pain. Physical examination may reveal swelling and
tenderness over the distal upper arm, medial epicondyle, and proximal forearm. Radiographs
should include anteroposterior, oblique, and lateral views, often with comparison views of the
other elbow. Such films appear normal in about 85% of cases but can show hypertrophy or
fragmentation of the medial epicondyle or subtle apophyseal widening. Rarely, an avulsion
fracture of the medial epicondyle or intra-articular loose body may be documented, suggesting a
worse injury. Magnetic resonance imaging is reserved for severe injuries, those not responding
to conservative treatment, when there is concern for intra-articular bodies, or for more skeletally
mature patients. In these patients, the clinician should evaluate the patient for UCL injury,
excessive physeal widening, or intra-articular bodies that may necessitate surgical intervention
(Item C211C).
The primary treatment for medial apophysitis is rest for 4 to 6 weeks or until the patient is
asymptomatic; application of ice for swelling and oral analgesics for pain are adjuvant

Copyright 2010 by the American Academy of Pediatrics page 674


2011 PREP SA on CD-ROM

treatments. If there is a flexion contracture, the patient may wear an elbow extension brace.
During the period of rest, the athlete can continue with general conditioning, with special
emphasis on core and shoulder strengthening. After the pain is relieved, the child can increase
throwing activities incrementally under supervision. Most athletes can return to pitching by 12
weeks after treatment begins.
The primary approach to these injuries should be prevention. The USA Baseball Medical and
Safety Advisory Committee specifies throwing limits that vary according to age. In general, they
recommend limits of 75 to 125 pitches/week or 50 to 75 pitches/outing, depending on age. They
also recommend a season limit of 600 pitches and that athletes compete no more than 9 months
per year, with 3 months of rest from overhead activity.
Although less common, other elbow injuries are possible in the young athlete. Fractures,
most commonly medial epicondyle avulsion, occur, but the onset of pain tends to be much more
acute, with a specific inciting event. UCL injury occurs in adult athletes but is rare in children.
Avascular necrosis or osteochondrosis of the capitellum (Panner disease) is analogous to Legg-
Calv-Perthes disease. It appears as a focal lesion of capitellar subchondral bone and the
overlying cartilage and generally affects children younger than 10 years of age. Pain is located
laterally. Olecranon apophyseal injury occurs with forceful triceps contractions during overhead
throwing, similar to what occurs with Osgood-Schlatter disease. Affected children often have
acute pain and swelling over the posterior elbow and decreased range of motion.

Suggested reading:

Benjamin HJ, Briner WW Jr. Little league elbow. Clin J Sport Med. 2005;15:37-40

Cassas KJ, Cassettari-Wayhs A. Childhood and adolescent sports-related overuse injuries. Am


Fam Physician. 2006;73:1014-1022. Abstract available at:
http://www.ncbi.nlm.nih.gov/pubmed/16570735

Emery KH. Imaging of sports injuries of the upper extremity in children. Clin Sports Med.
2006;25:543-568. DOI: 10.1016/j.csm.2006.02.003. Abstract available at:
http://www.ncbi.nlm.nih.gov/pubmed/16798142

Green NE, Van Zeeland ML. Fractures and dislocations about the elbow. In: Green NE,
Swiontkowski MF, eds. Skeletal Trauma in Children. 4th ed. Philadelphia, Pa: Elsevier; 2008:
Chapter 9

Rudzki JR, Paletta GA Jr. Juvenile and adolescent elbow injuries in sports. Clin Sports Med.
2004;23:581-608. DOI: 10.1016/j.csm.2004.05.001. Abstract available at:
http://www.ncbi.nlm.nih.gov/pubmed/15474224

Savoie FH, Field LD. Elbow and forearm: Section D: Osteochondritis dissecans of the elbow. In:
DeLee JC, Drez D Jr, Miller MD, eds. DeLee and Drezs Orthopaedic sports Medicine: Principles
and Practice. 3rd ed. Philadelphia, Pa: Saunders Elsevier; 2009: Chapter 19

Copyright 2010 by the American Academy of Pediatrics page 675


2011 PREP SA on CD-ROM

USA Baseball Medical and Safety Advisory Committee. Youth Baseball Pitching Injuries.2008.
Available at:
http://web.usabaseball.com/news/article.jsp?ymd=20090813&content_id=6409508&vkey=news
_usab&gid=

Copyright 2010 by the American Academy of Pediatrics page 676


2011 PREP SA on CD-ROM

Critique: 211

(Reprinted with permission from Harris SS, Anderson SJ, eds. Care of the Young Athlete. 2nd
ed. Elk Grove Village, Ill: American Academy of Pediatrics; 2010)
The ossification centers of the elbow in order of appearance (1=first).

Copyright 2010 by the American Academy of Pediatrics page 677


2011 PREP SA on CD-ROM

Critique: 211

(Reprinted with permission from Harris SS, Anderson SJ, eds. Care of the Young Athlete. 2nd
ed. Elk Grove Village, Ill: American Academy of Pediatrics; 2010)
Ulnar collateral ligament anatomy and the mechanism of injury.

Copyright 2010 by the American Academy of Pediatrics page 678


2011 PREP SA on CD-ROM

Critique: 211

(Reprinted with permission from Harris SS, Anderson SJ, eds. Care of the Young Athlete. 2nd ed.
Elk Grove Village, Ill: American Academy of Pediatrics; 2010)
The spectrum of anatomic injuries associated with Little League elbow.

Copyright 2010 by the American Academy of Pediatrics page 679


2011 PREP SA on CD-ROM

Question: 212

A 10-year-old child who has cystic fibrosis was the restrained back seat passenger in a serious
motor vehicle crash. He has been hospitalized for several days with a tibia-fibula fracture, a
splenic contusion, and multiple abrasions, but he is improving. His mother has been attempting to
help him comply with his usual cystic fibrosis regimen, including chest physiotherapy, with the
assistance of the hospital staff. She notes diminished cough after his regimen compared with
what he experienced prior to the accident.

Of the following, the diminished cough in this boy is MOST likely a result of

A. head injury

B. occult spinal cord injury

C. pain

D. pulmonary contusion

E. worsening cystic fibrosis

Copyright 2010 by the American Academy of Pediatrics page 680


2011 PREP SA on CD-ROM

Critique: 212 Preferred Response: C

Cough is a complex response via neurologic reflexes to irritants of the mucosa in the
tracheobronchial tree. Any influence that decreases neurologic signals to the cough center in
the brain or afferent neurologic stimuli to chest wall muscles and diaphragm or that causes
changes in airway protection may impede cough. Such impedance may cause further airway
obstruction or worsening of infection or chemical or mechanical irritants that initially led to the
cough. The child described in the vignette has experienced chest and abdominal trauma. This
may result in increased pain during cough, thus impeding this critical airway clearing mechanism.
Patients who have muscle weakness, such as seen with cystic fibrosis, may have a cough
that is ineffective in clearing the airway and bronchial tree. Ineffective cough may lead to fatigue
and subsequent respiratory compromise. Medications such as opiates may decrease cough by
decreasing the cough reflex via an inability to detect irritants or lack of neurologic integrity to
initiate cough.
In addition to pain itself, the use of analgesic medications, especially opiates, may suppress
cough. A newly diagnosed central nervous system injury (eg, occult spinal cord or head injury)
is unlikely based on the improvement of his other injuries. Pulmonary contusion probably would
have been detected earlier in this patients course and would have presented as hypoxia,
respiratory distress, and worsened underlying disease. Although an exacerbation of his cystic
fibrosis should be considered, his diminished cough response more likely is related to the pain
and his medications. Other conditions that impair cough include cerebral palsy, muscle
weakness, vocal cord dysfunction, and thoracic deformities.

Suggested reading:

Boitano LJ. Equipment options for cough augmentation, ventilation, and noninvasive interfaces in
neuromuscular respiratory management. Pediatrics. 2009;123(suppl):S226-S230. DOI:
10.1542/peds.2008-2952F. Available at:
http://pediatrics.aappublications.org/cgi/content/full/123/Supplement_4/S226

Vernacchio L, Kelly JP, Kaufman DW, Mitchell AA. Cough and cold medication use by US children,
19992006: results from the Slone Survey. Pediatrics. 2008;122:e323-e329. DOI:
10.1542/peds.2008-0498. Available at:
http://pediatrics.aappublications.org/cgi/content/full/122/2/e323

Copyright 2010 by the American Academy of Pediatrics page 681


2011 PREP SA on CD-ROM

Question: 213

You are evaluating an 8-year-old girl who has had several episodes of palpitations. The
episodes last several minutes and are marked by the abrupt onset of a rapid heart rate followed
by an equally abrupt termination of symptoms. She experiences only slight dizziness, and her
parents report that they can terminate the episodes by having her hold her breath and bearing
down. Baseline electrocardiography shows normal results (without pre-excitation). During 24-
hour ambulatory electrocardiography (Holter monitoring), she has one 30-second episode of
supraventricular tachycardia.

Of the following, you are MOST likely to advise the parents that

A. activity restriction is recommended

B. caffeinated beverages do not exacerbate this arrhythmia

C. echocardiography is needed because of the high risk of an associated congenital cardiac


anatomic defect

D. electrophysiology study with radiofrequency ablation is often effective

E. propranolol is unlikely to be of benefit

Copyright 2010 by the American Academy of Pediatrics page 682


2011 PREP SA on CD-ROM

Critique: 213 Preferred Response: D

The girl described in the vignette is experiencing typical supraventricular tachycardia (SVT),
the result of re-entry of electrical activity passing from the atria to the ventricles over the
atrioventricular (AV) node and back again to the atria via a retrograde conducting pathway. In
infants, the retrograde conducting pathway usually is an accessory pathway via the AV node.
This type of accessory pathway is common in older children, but the retrograde tract by which
the electrical wave re-enters the atria is a so-called retrograde AV nodal pathway. In this
situation, when there are dual AV nodal pathways, the technical name for the SVT is AV node re-
entry tachycardia.
During AV re-entry tachycardia, careful inspection of the tracing from electrocardiography
(ECG) often reveals the presence of retrograde P waves after the normal QRS complex. The
QRS complex is of normal morphology (unchanged from baseline sinus rhythm). Resting ECG
may reveal the pattern of Wolff-Parkinson-White syndrome, with a short PR interval and a
slurred upstroke of the QRS complex, termed a delta wave (Item C213). The diagnosis of SVT is
based upon either capture of the acute event by 12-lead ECG or additional testing after
suspicious symptoms of tachycardia, palpitations, and lightheadedness are reported. Patients
often describe a rapid onset and cessation of the tachycardia rather than the gradual
acceleration and deceleration of heart rate more typical of sinus tachycardia. Diagnostic testing
can include 24-hour ambulatory (Holter) monitoring, although this only occasionally is effective
because episodes usually are sporadic. More efficacious testing includes longer-term
ambulatory (event) monitoring in which the patient wears or carries a monitor that can be used
to capture events and transmit via a transtelephonic device.
The goal of pharmacologic therapy is to interrupt the re-entry tachycardia loop. The AV node
is highly innervated by vagal parasympathetic efferents, which is why vagal maneuvers, such
as described for the girl in the vignette, are effective. The AV node also is sensitive to
intravenous administration of adenosine, which causes transient complete AV block by blocking
AV node conduction and terminates the re-entry SVT. Direct current cardioversion also is
effective in converting SVT to sinus rhythm.
After the acute episode of SVT is terminated, decisions must be made about chronic
pharmacologic management. The most common treatment in the pediatric population is beta
blocker therapy with propranolol or atenolol. Both drugs are nonselective beta blockers, blocking
the action of epinephrine and norepinephrine on both beta1- and beta2-adrenergic receptors.
Beta-adrenergic antagonists should be used with caution in patients who have reactive airway
disease because they can exacerbate a bronchospastic event. Digoxin also is employed in the
chronic treatment of SVT, although the serum concentrations required to obtain efficacious
therapy are close to the toxic range, which has prompted most clinicians to use safer
medications. A variety of other medications can be used, including flecanide, sotalol, and
amiodarone. Flecainide should not be used in children who have SVT and congenital heart
disease because of reports of sudden death in this population. Catheter-based intervention can
be used for children who are either intolerant of medications or in whom medication is
ineffective. Radiofrequency catheter ablation applies energy to the site of retrograde
conduction, either burning or freezing the tissue, resulting in cell necrosis and interruption of the
re-entrant loop. This is a highly effective strategy, with greater than 90% success in routine re-

Copyright 2010 by the American Academy of Pediatrics page 683


2011 PREP SA on CD-ROM

entry SVT.
Activity restriction is unnecessary for the typical forms of SVT because an increased heart
rate would not be expected to promote an episode of SVT. Caffeine intake should be avoided
because the stimulatory effects of these agents can reduce the threshold of event induction.
Similarly, certain beta-adrenergic receptor agonists (eg, albuterol) and sympathomimetic agents
(catecholamines such as epinephrine) can stimulate an episode of SVT and should be used
sparingly or alternatives should be employed. Although congenital heart defects occasionally are
found in patients who have SVT, screening echocardiography is not necessary unless there is
a clinical suspicion based upon findings from physical examination or diagnostic testing. The
most common cardiac lesions in patients who have SVT are atrial septal defects and Ebstein
anomaly of the tricuspid valve.

Suggested reading:

Saarel EV, Stefanelli CB, Fischbach PS, Serwer GA, Rosenthal A, Dick M II. Transtelephonic
electrocardiographic monitors for evaluation of children and adolescents with suspected
arrhythmias. Pediatrics. 2004;113:248-251. Available at:
http://pediatrics.aappublications.org/cgi/content/full/113/2/248

Salerno JC, Seslar SP. Supraventricular tachycardia. Arch Pediatr Adolesc Med. 2009;163:268-
274. Abstract available at: http://www.ncbi.nlm.nih.gov/pubmed/19255396

Tingelstad J. Consultation with the specialist: cardiac dysrhythmias. Pediatr Rev. 2001;22:91-94.
DOI: 10.1542/10.1542/pir.22-3-91. Available at:
http://pedsinreview.aappublications.org/cgi/content/full/22/3/91

Copyright 2010 by the American Academy of Pediatrics page 684


2011 PREP SA on CD-ROM

Critique: 213

(Courtesy of A Friedman)
In Wolff-Parkinson-White syndrome, delta waves (arrows) are present that represent pre-
excitation (early depolarization of the QRS complex).

Copyright 2010 by the American Academy of Pediatrics page 685


2011 PREP SA on CD-ROM

Question: 214

A 13-year-old girl has had 5 days of unremitting headache that is relieved by vomiting but not
sleep and 1 day of double vision. She was previously healthy and has no history of migraine
headaches, but she was treated for otitis media 6 weeks ago. Findings on physical examination
are normal except for some tenderness over her left mastoid. On neurologic examination, you
note normal mental status and normally reactive pupils and vision but florid papilledema. She
cannot abduct her right eye fully and has subjective double vision with both eyes open looking to
the right but not looking to the left or with either eye covered. Facial sensation and movements
are normally symmetric, and the rest of the findings are normal. Head computed tomography scan
yields normal results.

Of the following, the diagnostic test or procedure that is MOST likely to be helpful is

A. brain magnetic resonance imaging

B. cerebral angiography

C. lumbar puncture with manometry

D. ocular nerve sheath fenestration

E. serum vitamin A measurement

Copyright 2010 by the American Academy of Pediatrics page 686


2011 PREP SA on CD-ROM

Critique: 214 Preferred Response: C

The girl described in the vignette has unremitting head pain that improves transiently after
vomiting but not sleep. In addition, she has a new symptom of double vision when looking to the
right, not left, an acquired ocular malalignment consistent with a right 6th nerve palsy. The lack of
facial weakness or facial sensory loss suggests that this symptom comes from outside of the
brainstem. Funduscopic examination reveals bilateral papilledema. This complex of symptoms is
characteristic of raised intracranial pressure due to pseudotumor cerebri. Diagnosis and
treatment are needed to reduce pain and prevent visual loss.
An urgent head computed tomography (CT) scan in the emergency department showed
normal findings. This rules out hydrocephalus or a mass lesion but does not rule out elevated
intracranial pressure. The most helpful procedure to determine this finding is a lumbar puncture
with manometry. Manometry pressure measurement should be obtained with the child in the
lateral decubitus position, with legs relaxed. A pressure greater than 20 cm H2O confirms the
clinical impression of elevated intracranial pressure and relieves the pressure. The pressure
often remains low for several days because cerebrospinal fluid drains into the soft tissues
around the lumbar puncture site. Cerebrospinal fluid studies should include assessment of
protein, glucose, and cell count. Bacterial, fungal, and Mycobacterium tuberculosis studies as
well as cytology for malignancy may be indicated in some cases.
The differential diagnosis of pseudotumor cerebri includes complications of obesity; adverse
effects of medications such as minocycline, isotretinoin, or steroids; hypervitaminosis A; venous
sinus thrombosis; anemia; renal failure; and hypercalcemia.
The recent history of otitis media and current pain over the mastoid described for the girl are
consistent with mastoiditis. The cause for her pseudotumor was venous sinus thrombosis. This
may be apparent on head CT scan, but magnetic resonance imaging (MRI) with contrast and
with venography (Item C214) is more sensitive for detection than head CT or routine MRI.
Cerebral angiography typically is used to characterize arterial vascular pathology. However,
this invasive procedure is undertaken infrequently because of improvements in magnetic
resonance angiography. Ocular nerve sheath fenestration is performed occasionally as a longer
term treatment for refractory pseudotumor cerebri to reduce pressure on the eyes and preserve
vision. Serum vitamin A measurement is reasonable after lumbar puncture as part of the
evaluation for causes of intracranial hypertension.

Suggested reading:

Evans RW. Diagnostic testing for migraine and other primary headaches. Neurol Clin.
2009;27:393-415. DOI: 10.1016/j.ncl.2008.11.009. Abstract available at:
http://www.ncbi.nlm.nih.gov/pubmed/19289222

Genizi J, Lahat E, Zelnik N, Mahajnah M, Ravid S, Shahar E. Childhood-onset idiopathic


intracranial hypertension: relation of sex and obesity. Pediatr Neurol. 2007;36:247-249. DOI:
10.1016/j.pediatrneurol.2007.01.002. Abstract available at:
http://www.ncbi.nlm.nih.gov/pubmed/17437908

Copyright 2010 by the American Academy of Pediatrics page 687


2011 PREP SA on CD-ROM

Critique: 214

(Courtesy of D Gilbert)
Magnetic resonance venography demonstrating thrombosis of the left transverse sinus (arrow)
(H=head, F=foot, R=right, L=left).

Copyright 2010 by the American Academy of Pediatrics page 688


2011 PREP SA on CD-ROM

Question: 215

You are called to the newborn nursery to evaluate an infant who has had two dusky episodes
unrelated to feedings. Serum glucose values are normal. Oxygen saturation on pulse oximetry is
98% with the infant sleeping comfortably on her side. Cardiac examination yields normal results.
You notice features consistent with Pierre Robin sequence (PRS) (Item Q215).

Of the following, the series of events that MOST commonly causes PRS is

A. cleft palate -> glossoptosis -> airway obstruction

B. depressed midface -> choanal stenosis -> airway obstruction

C. glossoptosis -> cleft palate -> cleft larynx -> airway obstruction

D. macroglossia -> cleft palate -> airway obstruction

E. retrognathia -> glossoptosis -> cleft palate -> airway obstruction

Copyright 2010 by the American Academy of Pediatrics page 689


2011 PREP SA on CD-ROM

Question: 215

(Reprinted with permission from Taylor MRG. Consultation with the specialist: the Pierre Robin
sequence: a concise review for the practicing pediatrician. Pediatr Rev. 2001;22:125-130)
Features described for the infant in the vignette. Note the micrognathia. In this infant, an oral
airway has been placed.

Copyright 2010 by the American Academy of Pediatrics page 690


2011 PREP SA on CD-ROM

Critique: 215 Preferred Response: E

A sequence is a cascade of events that occurs subsequent to a primary abnormality in the


developing embryo/fetus. One of the best known of these is the Pierre Robin sequence (PRS),
also called "Robin sequence". It is named after the French physician who, in 1933, described the
pattern of micrognathia, glossoptosis (posterior displacement of the tongue), and cleft palate.
The primary cause of PRS is believed to be hypoplasia of the mandible before 9 weeks
gestation. At 9 weeks, the palatal shelves, which "hang down" on either side of the tongue,
normally swing up and close, anteriorly to posteriorly. If the mandible is small (micrognathia) or
posteriorly placed (retrognathia), the tongue is pushed back and mounded up, making it
impossible for the palatal shelves to "clear" the tongue as they try to close over it.
There are myriad causes of PRS, including uterine constraint, single gene disorders,
chromosome abnormalities, teratogenic exposures, disruptions, and unknown reasons. It is
incumbent upon the physician caring for the child who has PRS to investigate underlying causes
because a unifying diagnosis is important in defining a management strategy, the natural history
of the disorder, and recurrence risk.
The clinical presentation of PRS varies. Some affected individuals have relatively mild
micrognathia, a small and soft palatal cleft, and minimal-to-no difficulties with feeding and
breathing. Some have oxygen desaturations that can be treated effectively with side or prone
positioning. Others are severely affected and have marked micrognathia, large palatal clefts, and
severe glossoptosis, leading to airway compromise and frequent cyanotic spells. Furthermore,
there may be swallowing difficulties. Medical intervention is required in these cases and
includes feeding tubes (usually nasogastric) and various methods of maintaining airway
integrity, including tongue-lip adhesion, intubation, tracheostomy, and mandibular traction.
Of note, some studies have shown that infants and children who have PRS may have
oxygen saturations less than 90% for prolonged periods without evidence for airway
obstruction. They are at increased risk for pulmonary hypertension that ultimately can cause cor
pulmonale and failure to thrive. Although no formal recommendations exist for surveillance in
PRS, it may be prudent to monitor oxygen saturations and refer for echocardiography at regular
intervals.

Suggested reading:

Cohen MM Jr. Robin sequences and complexes: causal heterogeneity and


pathogenetic/phenotypic variability. Am J Med Genet. 1999;84:311-315. DOI: 10.1002/(SICI)1096-
8628(19990604)84:4<311::AID-AJMG1>3.0.CO;2-9/. Abstract available at:
http://www.ncbi.nlm.nih.gov/pubmed/10340643

Meyer AC, Lidsky ME, Sampson DE, Lander TA, Liu M, Sidman JD. Airway interventions in
children with Pierre Robin sequence. Otolaryngol Head Neck Surg. 2008;138:782-787. DOI:
10.1016/j.otohns.2008.03.002. Abstract available at:
http://www.ncbi.nlm.nih.gov/pubmed/18503855

Taylor MRG. Consultation with the specialist: the Pierre Robin sequence: a concise review for

Copyright 2010 by the American Academy of Pediatrics page 691


2011 PREP SA on CD-ROM

the practicing pediatrician. Pediatr Rev. 2001;22:125-130. DOI: 10.1542/10.1542/pir.22-4-125.


Available at: http://pedsinreview.aappublications.org/cgi/content/full/22/4/125

van den Elzen AP, Semmerkot BA, Bongers EM, Huygen PL, Marres HA. Diagnosis and treatment
of the Pierre Robin sequence: results of a retrospective clinical trial and review of the literature.
Eur J Pediatr. 2001;160:47-53. DOI: 10.1007/s004310000646. Abstract available at:
http://www.ncbi.nlm.nih.gov/pubmed/11195018

Copyright 2010 by the American Academy of Pediatrics page 692


2011 PREP SA on CD-ROM

Question: 216

A 15-year-old, who wishes to join the school track team, decided to test his endurance with a 3-
hour run. He drank water before and during the run to maintain hydration, but he fainted and was
taken to an emergency department. You advise him about fluids and nutrition and make specific
suggestions for days when he plans to run for more than 1 hour.

Of the following, the MOST appropriate advice for these days is for him to ingest

A. caffeine-containing drinks during and after long runs

B. carbohydrate drinks before, during, and after long runs

C. daily amino acid supplements

D. high-protein meals 3 to 6 hours before long runs

E. increased amounts of water after the run

Copyright 2010 by the American Academy of Pediatrics page 693


2011 PREP SA on CD-ROM

Critique: 216 Preferred Response: B

Optimal exercise performance requires good hydration, with attention given to fluids before,
during, and after exercise (Item C216). Fluid requirements for an athlete vary by the type of
sport; the duration of activity; the ambient temperature; and individual factors such as body
weight, genetics, and sweating rates. In addition to water, electrolyte and carbohydrate intake
becomes important when sports participation lasts longer than 1 hour. Accordingly, the boy
described in the vignette should ingest carbohydrate drinks before, during, and after long runs.
A 1% loss in body weight with exercise can result in an increased heart rate and impaired
heat transfer to the skin and environment. Loss of 1 kg of weight is equivalent to loss of 1 L of
fluid. Dehydration or loss of 2% of body weight impairs performance, cognition, short-term
memory, attention, and visual motor tracking and increases the risk for heat exhaustion and heat
stroke. Water alone is adequate for exercise that lasts 1 hour or less. Excessive water intake
puts the athlete at risk for hyponatremia, which may have been the case for the boy in the
vignette.
The recommended daily allowance for protein is 0.95 g/kg per day in 4 to 13 year olds, 0.85
g/kg per day in 14 to 18 year olds, and 0.8 g/kg per day in adults. No evidence currently
supports the need for additional dietary protein requirements in healthy adolescents who
undertake endurance or resistance exercise. Supplements of amino acids and other proteins do
not enhance performance, and long-term studies on their safety have not been conducted. A
high-carbohydrate, low-fat meal is recommended before an endurance event.
Energy drinks are marketed for their stimulant effect and may contain caffeine, vitamins, and
herbal supplements. They have the same amount of calories as standard soft drinks. Ingestion
of more than 400 mg of caffeine daily by adults has the potential to induce behavioral,
cardiovascular, bone, and reproductive effects, and ingestion of more than 100 mg by
adolescents may result in high blood pressure. Sports drinks are manufactured for those
undertaking physical activity lasting more than 1 hour at a time and contain carbohydrates and
electrolytes. These drinks reduce fatigue by maintaining hydration, electrolyte balance, and blood
glucose concentrations during endurance sports. Use should be at a rate consistent with sweat
loss. The sodium in these drinks helps with water and glucose absorption from the small
intestine and is especially important in maintaining hydration during exercise in the heat.

Suggested reading:

Jardine DS. Heat illness and heat stroke. Pediatr Rev. 2007;28:249-258. DOI:
10.1542/10.1542/pir.28-7-249. Available at:
http://pedsinreview.aappublications.org/cgi/content/full/28/7/249

Kleinman RE. Sports nutrition. In: Pediatric Nutrition Handbook. 6th ed. Elk Grove Village, Ill:
American Academy of Pediatrics; 2009:225-247

Malinauskas BM, Aeby VG, Overton RF, Carpenter-Aeby T, Barber-Heidal K. A survey of energy
drink consumption patterns among college students. Nutr J. 2007;6:35. DOI: 10.1186/1475-2891-
6-35. Available at: http://www.nutritionj.com/content/6/1/35

Copyright 2010 by the American Academy of Pediatrics page 694


2011 PREP SA on CD-ROM

Rodriguez NR, DiMarco NM, Langley S; American Dietetic Association; Dietitians of Canada;
American College of Sports Medicine. Position of the American Dietetic Association, Dietitians of
Canada and the American College of Sports Medicine: nutrition and athletic performance. J Am
Diet Assoc. 2009;109:509-527. Abstract available at:
http://www.ncbi.nlm.nih.gov/pubmed/19278045

Savoca MR, Evans CD, Wilson ME, Harshfiel GA, Ludwig DA. The association of caffeinated
beverages with blood pressure in adolescents. Arch Pediatr Adolesc Med. 2004;158:473-477.
Available at: http://archpedi.ama-assn.org/cgi/content/full/158/5/473

Sawka MN, Burke LM, Eichner ER, Maughan RJ, Montain SJ, Stachenfeld NS. American College
of Sports Medicine position stand: exercise and fluid replacement. Med Sci Sports Exerc.
2007;39:377390. DOI: 10.1249/mss.0b013e31802ca597. Available at:
http://journals.lww.com/acsm-
msse/Fulltext/2007/02000/Exercise_and_Fluid_Replacement.22.aspx

von Duvillard SP, Arciero PJ, Tietjen-Smith T, Alford K. Sports drinks, exercise training, and
competition. Curr Sports Med Rep. 2008;7:202-208. Abstract available at:
http://www.ncbi.nlm.nih.gov/pubmed/18607221

Copyright 2010 by the American Academy of Pediatrics page 695


2011 PREP SA on CD-ROM

Critique: 216

(Reprinted with permission from Kleinman RE. Sports nutrition. In: Pediatric Nutrition Handbook.
6th ed. Elk Grove Village, Ill: American Academy of Pediatrics; 2009:225-247)
Dietary recommendations before, during, and after physical exercise.

Copyright 2010 by the American Academy of Pediatrics page 696


2011 PREP SA on CD-ROM

Question: 217

You have been following a 2-year-old boy who acquired human immunodeficiency virus infection
from his mother. The child has been receiving antiviral therapy, and his last CD4 count was
400/mcL. Over the past month, his mother reports that he has developed a cough and seems to
tire with eating. On physical examination, his respiratory rate is 50 breaths/min and heart rate is
130 beats/min. He is resting in his mothers arms and no retractions or accessory muscle use is
evident. His breath sounds are equal bilaterally, with slight crackles. His oxygen saturation on
room air is 85%, and an arterial blood gas shows a pH of 7.40, a PacO2 of 35 mm Hg, and a
PaO2 of 60 mm Hg. You obtain chest radiography (Item Q217).

Of the following, this patients pneumonia is MOST likely due to

A. cytomegalovirus

B. Mycobacterium tuberculosis

C. Pneumocystis jiroveci

D. Staphylococcus aureus

E. Streptococcus pneumoniae

Copyright 2010 by the American Academy of Pediatrics page 697


2011 PREP SA on CD-ROM

Question: 217

(Courtesy of B Wood)
Findings on chest radiography, as described for the child in the vignette.

Copyright 2010 by the American Academy of Pediatrics page 698


2011 PREP SA on CD-ROM

Critique: 217 Preferred Response: C

The child described in the vignette has a clinical history and presentation consistent with
Pneumocystis jiroveci pneumonia. P jiroveci is the proper nomenclature for human
pneumocystis infections; P carinii has been shown to affect rats only. P jiroveci infection
produces a characteristic syndrome of subacute diffuse pneumonitis with dyspnea, tachypnea,
oxygen desaturation, nonproductive cough, and fever. Chest radiographs often show bilateral
diffuse interstitial or alveolar disease, as seen for this boy (Item C217). Onset can be acute and
fulminant in patients who are immunocompromised due to treatment for solid organ tumors or
leukemia or, as exhibited by the boy in the vignette, chronic because of infection with human
immunodeficiency virus. The mortality rate in immunocompromised patients ranges from 5% to
40% if treated and approaches 100% if untreated. Immunocompromised children are at greater
risk for pneumonia, and the development of fever and changes in respiratory status should
trigger a prompt evaluation that includes physical examination, chest radiography, cultures, and
antibiotic or antiviral therapy.
Cytomegalovirus pneumonia is less common than P jiroveci pneumonia in
immunocompromised patients but may occur. The presentation of the two types of pneumonia is
similar, and coinfection with the two pathogens can be seen. Staphylococcus aureus and
Streptococcus pneumoniae pneumonia typically present with more acute respiratory distress,
including retractions and accessory muscle use, high fever, and productive coughs. Chest
radiography typically demonstrates lobar consolidation, often in association with effusions or
empyema. Mycobacterium tuberculosis infections typically present with a 1- to 6-month history
of fever, weight loss or poor weight gain, cough, night sweats, and chills. Chest radiographic
findings include hilar or mediastinal lymphadenopathy, segmental or lobar infiltrate or atelectasis,
and cavitary lesions (late).

Suggested reading:

American Academy of Pediatrics. Pneumocystis jiroveci infections. In: Pickering LK, Baker CJ,
Kimberlin DW, Long SS, eds. Red Book: 2009 Report of the Committee on Infectious Diseases.
28th ed. Elk Grove Village, Ill: American Academy of Pediatrics; 2009:536-540

Ferris MG, Schutze GE. Natural history and classification of pediatric HIV infection. UpToDate
Online 17.3. 2008. Available at:
http://www.utdol.com/online/content/topic.do?topicKey=pedi_id/11600

Gigliotti F, Wright TW. Pneumocystis carinii (Pneumocystis jiroveci). In: Kliegman RM, Behrman
RE, Jenson HB, Stanton BF, eds. Nelson Textbook of Pediatrics. 18th ed. Philadelphia, Pa:
Saunders Elsevier; 2007:1325-1326

Sectish TC, Prober CG. Pneumonia. In: Kliegman RM, Behrman RE, Jenson HB, Stanton BF, eds.
Nelson Textbook of Pediatrics. 18th ed. Philadelphia, Pa: Saunders Elsevier; 2007:1795-1799

Copyright 2010 by the American Academy of Pediatrics page 699


2011 PREP SA on CD-ROM

Critique: 217

(Courtesy of B Wood)
Pneumocystis pneumonia: There are bilateral central lung infiltrates.

Copyright 2010 by the American Academy of Pediatrics page 700


2011 PREP SA on CD-ROM

Question: 218

On January 13, the father of one of your patients calls to tell you that the tubing on his son's
insulin pump became dislodged during the night. The 75-lb 10-year-old boy has had diabetes for 2
years. According to the father, the boy is feeling nauseated and has a blood glucose of 450
mg/dL (25.0 mmol/L) with large ketones in the urine. The father has replaced the infusion set,
which now seems to be working well, but his driveway was snowed in overnight and he does
not think he will be able to get out of the house for at least 4 to 6 hours. You tell him to try to give
his son sips of ice cold water as well as saltine crackers as tolerated for the next few hours and
check his blood glucose and urine ketones every 2 hours.

Of the following, the MOST appropriate additional suggestion for the father is to

A. administer 4 units of ultrashort-acting insulin subcutaneously using an insulin pen or syringe


and needle and repeat every 2 to 3 hours

B. administer 20 units of glargine insulin subcutaneously

C. continue the insulin pump at its usual infusion rate

D. give the boy hard candy and orange juice as tolerated every hour

E. try to get the police or emergency vehicle to his house and transport his son to the hospital for
intravenous rehydration and insulin therapy

Copyright 2010 by the American Academy of Pediatrics page 701


2011 PREP SA on CD-ROM

Critique: 218 Preferred Response: A

Failure of insulin pump therapy is becoming a leading cause of recurrent diabetic


ketoacidosis (DKA), as described for the boy in the vignette. DKA develops about 6 hours after
the failure of infusion of an ultrashort-acting insulin because there is no depot supply of insulin.
The boy in the vignette has early DKA and will not have access to direct medical care for some
hours. However, his family has the materials on hand to treat him and help him through this
crisis. Although his insulin pump appears to be working and infusing well, this is not a certainty.
Therefore, he should receive insulin by a more direct route. Four units of ultrashort-acting insulin
administered via an insulin pen or syringe and needle every 2 to 3 hours represents
approximately 0.1 unit/kg per dose, which is a relatively low dose of insulin for the treatment of
DKA but should be sufficient because the boy will continue to receive his maintenance insulin
through his pump. If his blood glucose value does not decrease by about 75 to 100 mg/dL per
hour on this regimen, the insulin dose could be increased.
Glargine insulin lasts for 24 hours but is released so slowly that it cannot treat DKA. It can
be used for backup if a pump is not working properly but never should be administered as a
pumped insulin. The insulin pump should be continued at its usual infusion rate, but this is not
sufficient to treat the DKA. Because this boy has hyperglycemia and is not yet able to metabolize
glucose because of insulin deficiency, he does not need hard candy and orange juice. Getting
an emergency vehicle to the boys home to transport him to the hospital appears to be difficult in
this circumstance, but subcutaneous insulin administration has been as effective as intravenous
insulin in the management of DKA.

Suggested reading:

Cooke DW, Plotnick L. Management of diabetic ketoacidosis in children and adolescents. Pediatr
Rev. 2008;29:431-436. DOI: 10.1542/10.1542/pir.29-12-431. Available at:
http://pedsinreview.aappublications.org/cgi/content/full/29/12/431

Hanas R, Lindgren F, Lindblad B. A 2-yr national population study of pediatric ketoacidosis in


Sweden: predisposing conditions and insulin pump use. Pediatr Diabetes. 2009;10:33-37. DOI:
10.1111/j.1399-5448.2008.00441.x. Abstract available at:
http://www.ncbi.nlm.nih.gov/pubmed/18761647

Hanas R, Ludvigsson J. Hypoglycemia and ketoacidosis with insulin pump therapy in children
and adolescents. Pediatr Diabetes. 2006;7 (suppl 4):32-38

Sherry NA, Levitsky LL. Management of diabetic ketoacidosis in children and adolescents.
Paediatr Drugs. 2008;10:209-215. Abstract available at:
http://www.ncbi.nlm.nih.gov/pubmed/18590342

Copyright 2010 by the American Academy of Pediatrics page 702


2011 PREP SA on CD-ROM

Question: 219

A 6-year-old girl is having difficulty in school and is unable to keep up with the first-grade
classroom curriculum. She has problems sounding out words and doing basic math. Her parents
seek your guidance in helping her.

Of the following, the MOST appropriate action is to

A. begin trial of stimulant medication

B. have the girl moved into a special education classroom

C. have the parents hire a private tutor

D. refer the girl for psychoeducational evaluation

E. suggest the girl repeat first grade

Copyright 2010 by the American Academy of Pediatrics page 703


2011 PREP SA on CD-ROM

Critique: 219 Preferred Response: D

The girl described in the vignette is showing academic difficulties that require
psychoeducational evaluation to determine their cause and her eligibility for special education
services. The results of the evaluation may be used to establish an Individualized Education Plan
(IEP) that includes evidence-based educational remediation, accommodations, and modifications.
Special education in each local school district is mandated and regulated under the federal
Education for All Handicapped Children Act (PL 94-142). This act was reauthorized and revised
as PL 101-476 under the title Individuals with Disabilities Education Act (IDEA). The law provides
that every child identified via screening or observation as meeting the definition for
developmental delay be referred for a comprehensive multidisciplinary team assessment. The
team works in collaboration with the family to establish an IEP that contains defined education
and therapy strategies and objectives.
The child in the vignette requires an initial psychoeducational evaluation to determine her
eligibility before being placed in a special education classroom. If she has a learning disability,
repeating the grade will not address the underlying learning issue. The girl has no symptoms of
attentional issues that warrant the use of a stimulant medication. Tutoring may be an adjunct
after determining the girls eligibility for specialized services, but it would not address the
development of an appropriate educational program at school.

Suggested reading:

Committee on Children with Disabilities. The pediatricians role in development and implementation
of an Individual Education Plan (IEP) and Individual Family Service Plan (IFSP). Pediatrics.
1999;104:124-127. Available at: http://pediatrics.aappublications.org/cgi/content/full/104/1/124

Lambros KM, Leslie LK. Management of the child with a learning disorder. Pediatr Ann.
2005;34:275-287. Abstract available at: http://www.ncbi.nlm.nih.gov/pubmed/15871432

Copyright 2010 by the American Academy of Pediatrics page 704


2011 PREP SA on CD-ROM

Question: 220

A 4-year-old boy presents to the emergency department with a 2-day history of fever,
decreased activity, and vomiting and a complaint of increasing headache today. The light has
been bothering his eyes. He attends day camp and participates in swim class 5 days/week. On
physical examination, the sleepy but arousable boy has a temperature of 39.5C, heart rate of
140 beats/min, respiratory rate of 18 breaths/min, and blood pressure of 80/50 mm Hg. His throat
is erythematous, and he has neck pain with flexion. The remainder of his physical examination
findings are normal. Cerebrospinal fluid (CSF) analysis reveals a white blood cell count of
0.32x103/mcL (0.32x109/L) (80% neutrophils, 20% lymphocytes), protein of 55 mg/dL, and
glucose of 45 mg/dL.

Of the following, the additional test on the CSF that is MOST likely to confirm the cause of this
childs illness is

A. arbovirus polymerase chain reaction panel

B. bacterial culture

C. enterovirus polymerase chain reaction

D. herpes simplex virus polymerase chain reaction

E. viral culture

Copyright 2010 by the American Academy of Pediatrics page 705


2011 PREP SA on CD-ROM

Critique: 220 Preferred Response: C

The acute illness with headache, photophobia, nuchal rigidity, and CSF pleocytosis
described for the child in the vignette confirm a diagnosis of meningitis. The lack of toxicity,
normal glucose and minimally elevated protein values, and degree of CSF pleocytosis (white
blood cell count <1.0x103/mcL [1.0x109/L]) are more suggestive of a viral than bacterial
pathogen, although there is sufficient overlap of physical examination and laboratory findings
that treatment with antibiotics pending results of bacterial cultures often is undertaken in this
setting. Polymorphonuclear leukocytes may predominate in the CSF early in the course of viral
meningitis, as noted for this boy.
The presence of pharyngitis, the season (summer months), and potential exposure in a
swimming pool all suggest the diagnosis of enteroviral infection. Enteroviruses are transmitted
primarily by the fecal-oral route. The development of polymerase chain reaction (PCR) assays
has enhanced the ability to diagnose the cause of aseptic meningitis greatly, and such an assay
for enterovirus is the preferred diagnostic test in this case. Viral culture of the CSF is much less
sensitive than PCR for detecting enteroviruses. Isolation of an enterovirus in viral culture from
the pharynx or stool supports the diagnosis of enterovirus aseptic meningitis but is less direct
and takes longer than PCR to provide a result.
Arboviruses (arthropod-borne viruses) refer to a group of viruses transmitted by mosquito
or tick vectors. These infections generally are associated with encephalitis as well as
meningitis, and the affected child may have greater changes in sensorium and seizures at
presentation. West Nile virus infection in children, however, may present as mild aseptic
meningitis.
Herpes simplex infection of the central nervous system in a child of this age would present
as meningoencephalitis, with seizures and greater depression of mental status than described
for this boy. In addition, examination of the CSF should reveal an increase in red blood cells and
greater elevation of protein than reported for this boy. PCR for herpes simplex virus in the CSF is
the test of choice for that condition. Self-limited primary herpes simplex type 2 aseptic meningitis
may present with initial genital herpes infection.

Suggested reading:

American Academy of Pediatrics. Enterovirus (nonpoliovirus) infections. In: Pickering LK, Baker
CJ, Kimberlin DW, Long SS, eds. Red Book: 2009 Report of the Committee on Infectious
Diseases. 28th ed. Elk Grove Village, Ill: American Academy of Pediatrics; 2009:287-288

Nolte FS. Case studies in cost effectiveness of molecular diagnostics for infectious diseases:
pulmonary tuberculosis, enteroviral meningitis, and BK virus nephropathy. Clin Infect Dis.
2006;43:1463-1467. DOI: 10.1086/508871. Available at:
http://www.journals.uchicago.edu/doi/full/10.1086/508871

Copyright 2010 by the American Academy of Pediatrics page 706


2011 PREP SA on CD-ROM

Question: 221

An unimmunized 4-year-old girl presents with malaise, sore throat, and difficulty swallowing. On
physical examination, she has a temperature of 38.0C, bilateral cervical adenopathy, and grayish
exudates over the mucous membranes of her tonsils and pharynx (Item Q221). When you attempt
to remove some of the exudate for culture, bleeding occurs.

Of the following, the MOST appropriate treatment of close contacts of this child is administration
of

A. intramuscular ceftriaxone

B. intravenous antitoxin

C. intravenous immune globulin

D. oral erythromycin

E. oral penicillin

Copyright 2010 by the American Academy of Pediatrics page 707


2011 PREP SA on CD-ROM

Question: 221

(Courtesy of the Red Book Online)


Exudates, as described for the girl in the vignette.

Copyright 2010 by the American Academy of Pediatrics page 708


2011 PREP SA on CD-ROM

Critique: 221 Preferred Response: D

The child described in the vignette has membranous pharyngitis caused by


Corynebacterium diphtheriae, a gram-positive pleomorphic bacillus. Because patients who
have diphtheria can deteriorate rapidly, treatment should be initiated immediately if the disease is
suspected. The mainstay of therapy is diphtheria antitoxin, an equine hyperimmune antiserum
that is administered intravenously (IV) to neutralize diphtheria toxin as rapidly as possible at a
dose that is determined by the site and size of the membrane and the degree and duration of the
illness. Diphtheria antitoxin neutralizes free toxin only and protects against neurotoxicity if given
within the first 1 to 2 days of illness. It likely is of no value for cutaneous diphtheria, but its use
should be considered because toxic sequelae have been reported. Commercially available
immune globulin preparations have not been approved for the treatment of diphtheria. In addition
to antitoxin therapy, antimicrobial therapy is used to stop toxin production, eradicate the
organism, and prevent transmission. Acceptable regimens include erythromycin orally or IV for
14 days, penicillin G procaine intramuscularly (IM) for 14 days, or penicillin G IM or IV for 14
days.
Close contacts of patients who have diphtheria, regardless of their immunization status,
should receive a single IM dose of penicillin G benzathine or 10 days of oral erythromycin.
Diphtheria antitoxin and commercially available immune globulin preparations have not been
approved for the prevention of diphtheria. All exposed persons should have pharyngeal cultures
performed for C diphtheriae and be monitored for 1 week for evidence of disease. Persons
who are pharyngeal carriers should be treated with oral penicillin G or erythromycin for 10 to 14
days or a single IM injection of penicillin G benzathine. Intramuscular ceftriaxone has no role in
the prevention or treatment of diphtheria.
Two follow-up pharyngeal cultures taken at least 24 hours apart should be obtained 24
hours after the completion of antimicrobial therapy. If a culture is positive, a 10-day course of
oral erythromycin should be administered, with follow-up cultures performed again. Previously
immunized exposed persons should receive a booster dose of an age-appropriate preparation
of a diphtheria toxoid-containing vaccine if they have not received one in the previous 5 years.
Close contacts whose immunization status is unknown or those who have not been immunized
fully should receive an age-appropriate preparation of a diphtheria toxoid-containing vaccine.
The respiratory illness caused by C diphtheriae usually develops over 1 to 2 days and is
accompanied by a low-grade fever. The primary lesion of diphtheria is a thick membrane
consisting of fibrin, bacteria, and dead cells that firmly adheres to underlying tissues. Removal of
the membrane results in bleeding. The membrane may be localized or extend widely, causing
soft-tissue edema of the airways.
Severe respiratory diphtheria can result in extensive neck swelling (bull neck) and upper
airway obstruction due to peripharyngeal edema or membrane aspiration. The generalized
symptoms that can occur in patients who have diphtheria, such as myocarditis, renal tubular
necrosis, and peripheral neuropathy, are caused by the potent bacterial exotoxin that is
produced at the site of the local lesion and spreads throughout the body. Less common
manifestations of diphtheria include cutaneous, conjunctival, otic, or vaginal disease.

Suggested reading:

Copyright 2010 by the American Academy of Pediatrics page 709


2011 PREP SA on CD-ROM

American Academy of Pediatrics. Diphtheria. In: Pickering LK, Baker CJ, Kimberlin DW, Long SS,
eds. Red Book: 2009 Report of the Committee on Infectious Diseases. 28th ed. Elk Grove
Village, Ill: American Academy of Pediatrics; 2009:280-283

Feigin RD, Stechenberg BW, Nag PK. Diphtheria. In: Feigin RD, Cherry JD, Demmler-Harrison GJ,
Kaplan SL, eds. Feigin & Cherrys Textbook of Pediatric Infectious Diseases. 6th ed.
Philadelphia, Pa: Saunders Elsevier; 2009:1393-1402

Copyright 2010 by the American Academy of Pediatrics page 710


2011 PREP SA on CD-ROM

Question: 222

A 2-year-old boy who has a history of poorly controlled seizures that are managed with a
ketogenic diet presents with increased fussiness and side pain. A urinalysis reveals:

Specific gravity, 1.020


pH, 6
3+ ketones
2+ blood
Negative for protein, glucose, nitrite, and leukocyte esterase
5 to 10 red blood cells/high-power field (hpf)
Fewer than 5 white blood cells/hpf

Of the following, the MOST likely associated urinary finding in this patient is

A. bacteruria

B. cystinuria

C. hemoglobinuria

D. hypercalciuria

E. hyperoxaluria

Copyright 2010 by the American Academy of Pediatrics page 711


2011 PREP SA on CD-ROM

Critique: 222 Preferred Response: D

The child described in the vignette is receiving a ketogenic diet for poorly controlled
seizures. A urinalysis performed to evaluate fussiness and apparent side pain reveals
microscopic hematuria. The differential diagnosis for microscopic hematuria usually is broken
down into glomerular and nonglomerular hematuria. Glomerular hematuria consists of the various
forms of glomerulonephritis that can be renal-limited or due to systemic disease. The evaluation
usually consists of measurement of blood urea nitrogen, creatinine, albumin, urine protein, and
urine creatinine (to quantify the degree of proteinuria if present). Serologic evaluation also is
undertaken and includes assessment of complement components 3 and 4 (C3 and C4),
antinuclear antibody, and anti-double-stranded DNA antibody. The standard evaluation for
nonglomerular disease is renal bladder ultrasonography to look for structural abnormalities
associated with hematuria such as nephrolithiasis, urolithiasis, renal cysts, or tumors (eg, Wilms
tumor or rhabdomyosarcoma). Some nephrologists also recommend screening patients who
have microscopic hematuria for urine calcium-to-creatinine ratio, although this is not universally
endorsed.
The ketogenic diet consumed by the patient in the vignette places him at risk for the
development of hypercalciuria and kidney stones. Among patients receiving the ketogenic diet
for 2 years, the incidence of kidney stones in children has ranged from 3% to 6%. The risks for
stone formation include hypercalciuria, acidic urine, and hypocitraturia. Hypercalciuria is very
common, increasing from 40% prior to the institution of the ketogenic diet to 75% by 6 months.
Also, analysis of the stones in children following the ketogenic diet shows that they are either
uric acid or mixed calcium/uric acid stones. Patients do not exhibit hyperuricosuria. Recent
evidence supports the empiric prescription of potassium citrate for patients following a ketogenic
diet irrespective of the presence of hypercalciuria. Empiric treatment decreased the incidence of
kidney stones to 0.9%, which was markedly better than a reactive approach of treating only
those who had hypercalciuria, in whom an incidence of 6.7% was noted. Bacteruria, cystinuria,
hemoglobinuria, and hyperoxaluria would not be expected based on the clinical presentation of
the patient in the vignette.
Hypercalciuria has been uncovered in a number of clinical settings. Its pathogenic role is
best understood in children who have nephrolithiasis and urolithiasis. It also has been seen in
children who have gross and microscopic hematuria. In addition, it has been found in a number
of children who have voiding symptoms of dysuria, urgency, and frequency.
The association of hypercalciuria and hematuria has been established for decades, but the
cause-and-effect relationship and the associated mechanism has not been established.
Approximately one third of children who have microscopic hematuria also have hypercalciuria
and no other associated abnormality; 31% of children who have hypercalciuria have hematuria.
Interestingly, the prevalence of hypercalciuria is similar whether patients have microscopic or
gross hematuria, and the magnitude of the hypercalciuria does not correlate with the severity of
the hematuria. Hypercalciuria has been found in approximately 40% of children who have
urolithiasis, but only 5% of children who have hypercalciuria develop stones. Those who have
hypercalciuria, exhibit gross hematuria, and have a family history of stone disease are at highest
risk for developing urolithiasis.
A random urine calcium-to-creatinine ratio usually is the initial screening test for diagnosing

Copyright 2010 by the American Academy of Pediatrics page 712


2011 PREP SA on CD-ROM

hypercalciuria, but the random sample is considered less reliable because calcium excretion can
be altered by diet. Increased intake of sodium or protein leads to increased urinary calcium
excretion. For patients who are continent and able, a 24-hour urine collection for calcium,
creatinine, sodium, potassium, and citrate is recommended. Medical treatment (eg, thiazide
diuretics) generally is avoided unless the patient remains symptomatic and fails therapy of
increased fluid intake and dietary sodium restriction.

Suggested reading:

Furth SL, Casey JC, Pyzik PL, et al. Risk factors for urolithiasis in children on the ketogenic diet.
Pediatr Nephrol. 2000;15:125-128. Abstract available at:
http://www.ncbi.nlm.nih.gov/pubmed/11095028

Massengill SF. Hematuria. Pediatr Rev. 2008;29:342-348. DOI: 10.1542/10.1542/pir.29-10-342.


Available at: http://pedsinreview.aappublications.org/cgi/content/full/29/10/342

McNally MA, Pyzik PL, Rubenstein JE, Hamdy RF, Kossoff EH. Empiric use of potassium citrate
reduces kidney-stone incidence with the ketogenic diet. Pediatrics. 2009;124:e300-e304. DOI:
10.1542/peds.2009-0217. Available at:
http://pediatrics.aappublications.org/cgi/content/full/124/2/e300

Sampath A, Kossoff EH, Furth SL, Pyzik PL, Vining EP. Kidney stones and the ketogenic diet: risk
factors and prevention. J Child Neurol. 2007;22:375-378. Abstract Available at:
http://www.ncbi.nlm.nih.gov/pubmed/17621514

Srivastava T, Schwaderer A. Diagnosis and management of hypercalciuria in children. Curr Opin


Pediatr. 2009;21:214-219. DOI: 10.1097/MOP.0b013e3283223db7. Abstract available at:
http://www.ncbi.nlm.nih.gov/pubmed/19307900

Copyright 2010 by the American Academy of Pediatrics page 713


2011 PREP SA on CD-ROM

Question: 223

You are conducting a school physical examination for a 16-year-old girl. During the discussion,
she admits to having "asthma" but currently only uses a short-acting beta2 agonist (SABA). On
further questioning, she reports that she takes her SABA three times a day as well as prior to
physical activity. She was admitted to the intensive care unit 1 year ago for an asthma
exacerbation, although she failed to keep her outpatient follow-up appointments after being
discharged.

Of the following, the MOST likely outcome of this girls frequent SABA use is

A. decreased risk for asthma mortality

B. hyperkalemia

C. hyponatremia

D. improvement in airway inflammation

E. prolonged QTc interval

Copyright 2010 by the American Academy of Pediatrics page 714


2011 PREP SA on CD-ROM

Critique: 223 Preferred Response: E

The most common adverse reactions to beta2-adrenergic agonists are tremor, tachycardia,
and palpitations. Administration of therapeutic or multiple doses of a beta2-adrenergic agonist
also may result in transient decreases in Pao2, prolonged QTc interval, arrhythmias, and
electrolyte abnormalities. Patients who require continuous beta agonist nebulizations during
hospitalization also may experience hyperglycemia due to glycogenolysis, hypomagnesemia, and
hypokalemia. Hypokalemia results from stimulation of the Na+-K+ pump and may lead to a
decrease in serum potassium of 0.4 to 0.9 mEq/L (0.4 to 0.9 mmol/L). Patients who have pre-
existing hypokalemia due to diuretic therapy may be at higher risk for arrhythmias during
continuous beta2-adrenergic agonist administration.
Hypermagnesemia, hypercalcemia, hyponatremia, and hypernatremia have not been
reported during single or continuous beta2-adrenergic agonist administration. Although beta2-
adrenergic agonists can increase mucociliary clearance, inhibit mediator release from mast cells
and basophils, and inhibit eosinophil and macrophage function, they generally are not considered
anti-inflammatory.
One concern for the patient in the vignette is use of her beta2 agonist three to four times a
day. Frequent use of a beta2 agonist causes downregulation of the beta2 receptor, called
desensitization. The clinical implications can vary. One study found that beta agonist use two to
four times per day among patients ages 12 to 55 years of age who had mild asthma neither
improved nor worsened asthma over 16 weeks. On the other hand, recent studies have
suggested that regular beta2-agonist use can worsen daily peak flows in patients homozygous
for arginine (Arg/Arg) at position 16 of the beta agonist receptor. The use of two or more beta
agonist inhalers per month has been associated with poor asthma control and increased
mortality risk, but the relationship of these outcomes with individual genotypes of the beta-
adrenergic receptor (ie, Arg/Arg, Arg/Gly, or Gly/Gly) still is being determined.

Suggested reading:

Hill VL, Wood PR. Asthma epidemiology, pathophysiology, and initial evaluation. Pediatr Rev.
2009;30:331-336. DOI: 10.1542/10.1542/pir.30-9-331. Available at:
http://pedsinreview.aappublications.org/cgi/content/full/30/9/331

Israel E, Chinchilli VM, Ford JG, et al; National Heart, Lung, and Blood Institute's Asthma Clinical
Research Network. Use of regularly scheduled albuterol treatment in asthma: genotype-
stratified, randomised, placebo-controlled cross-over trial. Lancet. 2004;364:1505-1512. DOI:
10.1016/S0140-6736(04)17273-5. Abstract available at:
http://www.ncbi.nlm.nih.gov/pubmed/15500895

Wood PR, Hill VL. Practical management of asthma. Pediatr Rev. 2009;30:375-385. DOI:
10.1542/10.1542/pir.30-10-375. Available at:
http://pedsinreview.aappublications.org/cgi/content/full/30/10/375

Copyright 2010 by the American Academy of Pediatrics page 715


2011 PREP SA on CD-ROM

Question: 224

A mother brings her 18-month-old child to the emergency department because of increased
sleepiness. She reports that he has had several episodes of vomiting without fever or diarrhea
over the past 24 hours for which she has given him two doses of ondansetron. This afternoon
she had difficulty awakening him from his nap. On physical examination, the child is pale and
arousable but prefers to sleep. His temperature is 37.0C, heart rate is 130 beats/min, respiratory
rate is 24 breaths/min, and blood pressure is 90/60 mm Hg. His pupils are 4 mm and normally
reactive. His lungs are clear, and his abdomen is nontender and mildly distended, with hypoactive
bowel sounds. While you are examining him, he has an episode of bilious emesis.

Of the following, the MOST likely explanation for this patients signs and symptoms is

A. intussusception

B. ondansetron toxicity

C. sepsis

D. viral gastroenteritis

E. volvulus

Copyright 2010 by the American Academy of Pediatrics page 716


2011 PREP SA on CD-ROM

Critique: 224 Preferred Response: A

The child described in the vignette is exhibiting signs and symptoms that suggest the
presence of intussusception, the clinical condition in which a segment of intestine telescopes
into an adjacent portion. The most common form of intussusception occurs when the terminal
ileum "intussuscepts" into the more proximal bowel, but intussusception can occur in any
segment of the intestine. Although intussusception is seen most frequently in male infants
between 3 and 12 months of age (mean incidence, 8 months), it is the most common cause of
bowel obstruction in children between 3 months and 6 years.
Intussusception should be suspected in any child who presents with some combination of
the classic triad of signs and symptoms that includes episodic cramping pain that causes the
patient to draw up the legs, a palpable sausage-shaped mass in the right abdomen, and "currant
jelly" stool (loose stool mixed with dark, clotted blood and mucus). However, only 15% to 20% of
children have all three of these findings, and one third of children have no evidence of
hematochezia. Because of the variability of presentation, a high level of suspicion is needed to
recognize intussusception in children who have more vague abdominal signs and symptoms
such as vomiting and abdominal pain. The diagnosis also should be considered in a child who
presents with unexplained lethargy or pallor, which may be the only presenting sign(s).
Evaluation of the patient in whom intussusception is suspected may include abdominal
radiographs that, in rare cases, demonstrate a soft-tissue mass in the right abdomen or the
"target sign," which is the radiographic view of a cross-section of the intussusception.
Abdominal ultrasonography is more diagnostically useful, with sensitivity and specificity
approaching 100%. In cases where the diagnosis is not in question, contrast or air enema is
both diagnostic and therapeutic (Item C224).
Ondansetron has a very wide therapeutic range, and the likelihood of toxicity is extremely
low. In toxicity studies, no end-organ damage was seen in animals given 30 to 100 times the
recommended dose. In addition, minor adverse effects are uncommon, with headache and
diarrhea reported most frequently. Sepsis is unlikely in the absence of fever. Bilious emesis
should prompt evaluation for more serious causes of abdominal symptoms than viral
gastroenteritis. If bilious emesis and lethargy occur in patients who have volvulus, the
abdominal examination would reveal marked distension and tenderness.

Suggested reading:

King L. Pediatrics, intussusception. eMedicine Specialties, Emergency Medicine, Pediatric.


2009. Available at: http://emedicine.medscape.com/article/802424-overview

Kitagawa S, Miqdady M. Intussusception in children. UpToDate Online 17.3. 2009. Available at:
http://www.utdol.com/online/content/topic.do?topicKey=pedigast/6634&selectedTitle=1~73&sour
ce=search_result

Ko HS, Schenk JP, Trger J, Rohrschneider WK. Current radiological management of


intussusception in children. Eur Radiol. 2007;17:2411-2421. DOI: 10.1007/s00330-007-0589-y.
Abstract available at: http://www.ncbi.nlm.nih.gov/pubmed/17308922

Copyright 2010 by the American Academy of Pediatrics page 717


2011 PREP SA on CD-ROM

Pumberger W, Dinhobl I, Dremsek P. Altered consciousness and lethargy from compromised


intestinal blood flow in children. Am J Emerg Med. 2004;22:307-309. DOI:
10.1016/j.ajem.2004.04.010. Abstract available at:
http://www.ncbi.nlm.nih.gov/pubmed/15258874

Wylie R. Ileus, adhesions, intussusception, and closed-loop obstructions. In: Kleigman RM,
Behrman RE, Jenson HB, Stanton BF, eds. Nelson Textbook of Pediatrics. 18th ed. Philadelphia,
Pa: Saunders Elsevier; 2007:1568-1570

Copyright 2010 by the American Academy of Pediatrics page 718


2011 PREP SA on CD-ROM

Critique: 224

(Courtesy of D Krowchuk)
Fluoroscopic image from an air enema reveals an intussusceptum (arrows show mass). The
mass was fully reduced.

Copyright 2010 by the American Academy of Pediatrics page 719


2011 PREP SA on CD-ROM

Question: 225

You are called to see a 16-year-old girl who underwent scoliosis surgery 6 days ago. She is
receiving parenteral nutrition via peripheral vein and morphine for pain. This morning she began
complaining of severe upper abdominal pain and vomiting. Physical examination demonstrates a
well-developed adolescent in moderate distress whose temperature is 38.7C, heart rate is 90
beats/min, and blood pressure is 130/66 mm Hg. Her abdomen is mildly distended, with moderate
fullness and tenderness in the right upper quadrant and epigastrium. Laboratory data include:

Hemoglobin, 11.5 g/dL (115 g/L)


White blood cell count, 15.5x10 3/mcL (15.5x109/L)
Aspartate aminotransferase, 100 units/L
Alanine aminotransferase, 120 units/L
Gamma-glutamyl transpeptidase, 180 units/L
Amylase, 70 units/L
Lipase, 80 units/L

Of the following, the MOST appropriate diagnostic test is

A. abdominal computed tomography scan

B. abdominal ultrasonography

C. hepatobiliary scintigraphy

D. magnetic resonance cholangiopancreatography

E. upper gastrointestinal endoscopy

Copyright 2010 by the American Academy of Pediatrics page 720


2011 PREP SA on CD-ROM

Critique: 225 Preferred Response: B

During recovery from major surgery or trauma, the acute onset of upper abdominal pain
should alert the clinician to several diagnostic possibilities, including peptic ulcer and
inflammatory pancreatic and gallbladder diseases. The adolescent described in the vignette,
who recently underwent spine surgery, has developed a febrile illness associated with right
upper quadrant fullness and tenderness. Laboratory studies show elevations in the white blood
cell count, gamma-glutamyl transpeptidase, and aspartate and alanine aminotransferases but
near-normal pancreatic enzyme values. Acute calculous or acalculous cholecystitis is the most
likely cause of these findings. No single diagnostic test is universally reliable for evaluating
cholecystitis, and clinical assessment often is the most important factor guiding management.
Once cholecystitis is suspected, however, the diagnostic study of first choice is abdominal
ultrasonography. Typical ultrasonographic findings in acute cholecystitis include identification of
gallstones (in calculous cholecystitis) or biliary sludge; evidence of bile ductular dilatation; and a
distended, thick-walled gallbladder. One study reported that a wall thickness of more than 3.5
mm was a reliable indicator of cholecystitis. In the absence of cholelithiasis or other
ultrasonographic signs of calculous or acalculous cholecystitis, hepatobiliary scintigraphy can
assess biliary excretion and gallbladder function accurately in many cases. Unfortunately,
prolonged fasting and parenteral nutrition are associated with false-positive results. If results of
these tests are inconclusive, other helpful studies include magnetic resonance imaging and
computed tomography scans. These tests may be more helpful than ultrasonography in
determining other sources of abdominal sepsis, but because of its relative reliability and cost,
ultrasonography remains the initial study of choice. Endoscopic intervention (endoscopic
retrograde cholangiopancreatography) may be considered if choledocholithiasis is suspected.
Cholecystitis is defined as inflammation of the gallbladder wall. Both chronic and acute
cholecystitis are diagnosed most frequently in adults as a complication of cholelithiasis, a
condition affecting more than 20 million Americans annually. In the pediatric population,
gallbladder disease, although much less common, occurs with the greatest frequency in
adolescents who have underlying hemolytic disorders leading to gallstone formation and in oral
contraceptive users. In the setting of cholelithiasis, acute cholecystitis is the consequence of
cystic duct obstruction that results in gallbladder inflammation and distension. Production and
release of biliary lysolecithins further exacerbate the inflammatory process.
Although no history of hemolytic disease or other disorder predisposing to gallstone
formation has been cited for the girl in the vignette, her recent surgery, need for parenteral
nutrition support, and ongoing pain management using narcotic analgesia as well as her physical
findings necessitate evaluation for acalculous cholecystitis. This acute inflammatory disorder
has been associated with both medical and surgical conditions, in which intravenous
alimentation, narcotic administration, and prolonged fasting contribute to biliary stasis and
hypofunction. Reports have described the occurrence of acute acalculous cholecystitis in both
adults and children undergoing spinal surgery, but the disorder may develop following any
surgical procedure or systemic illness exhibiting the previously noted risk factors. Following
diagnosis, the condition may be managed medically, employing a combination of antibiotics,
gastric decompression, and parenteral alimentation, but cholecystectomy often is required.
Because the clinical presentation of postoperative acalculous cholecystitis may be

Copyright 2010 by the American Academy of Pediatrics page 721


2011 PREP SA on CD-ROM

nonspecific, significant delays in diagnosis and treatment may occur, resulting in gallbladder
necrosis, perforation, and abscess. For adults who have chronic systemic illness, the mortality
rate associated with these complications has been reported as high as 50%. Frequent findings
among children who have acute cholecystitis include jaundice in approximately 40% and a right
upper quadrant mass in about 25%. Although several laboratory tests yielded abnormal results
for the girl in the vignette, the white blood cell count, serum bilirubin, and liver function profile
may be normal early in the course of disease.

Suggested reading:

Shapiro G, Green DW, Fatica NS, Boachie-Adjei O. Medical complications in scoliosis surgery.
Curr Opin Pediatr. 2001; 13:36-41. Abstract available at:
http://www.ncbi.nlm.nih.gov/pubmed/11176241

Soares-Oliveira M, Mariz C, Carvalho JL, Gomes AT, Estevo-Costa J. Diaphragm of the


gallbladder: a rare cause of acalculous acute cholecystitis. J Pediatr Gastroenterol Nutr.
2000;31:570-571. Available at:
http://journals.lww.com/jpgn/Fulltext/2000/11000/Diaphragm_of_the_Gallbladder__A_Rare_Caus
e_of.22.aspx

Strasberg SM. Acute calculous cholecystitis. N Engl J Med. 2008;358:2804-2811. Extract


available at: http://content.nejm.org/cgi/content/extract/358/26/2804

Copyright 2010 by the American Academy of Pediatrics page 722


2011 PREP SA on CD-ROM

Question: 226

During the health supervision visit of a 4-month-old infant, the mother expresses concern that her
daughter does not respond to her voice. The mother is 20 years old and worked in a child care
center during her pregnancy. The infant was delivered at 38 weeks gestation, weighing 2,800 g.
Otoacoustic emission testing (OAE) was passed bilaterally on postnatal day 2. The infant has had
no medical problems since birth. After searching the Internet, the mother would like to know more
about hearing screening technologies used in infants and whether auditory brainstem response
(ABR) testing would be helpful.

Of the following, the MOST accurate information to provide this mother about hearing screening in
the newborn and infant is that

A. ABR testing does not detect sensory hearing loss

B. ABR testing does not reflect the status of the peripheral auditory system

C. otoacoustic emission (OAE) testing does not detect neural dysfunction

D. OAE testing does not miss isolated frequency region losses

E. OAE testing does not result in failed screening tests due to transient middle ear conditions

Copyright 2010 by the American Academy of Pediatrics page 723


2011 PREP SA on CD-ROM

Critique: 226 Preferred Response: C

Hearing loss affects 2 to 4 per 1,000 infants, and delays in identification of such loss
beyond 6 months of age decrease the ability to intervene in language and speech development.
Selective screening of high-risk infants only identifies 50% of those who have hearing loss.
Universal hearing screening for all infants younger than 1 month of age was endorsed by the
Joint Committee on Infant Hearing in 2007, and more than 38 states have legislation mandating
these programs. The two commonly used noninvasive methods for newborn and infant hearing
screening are otoacoustic emission (OAE) testing and auditory brainstem response (ABR)
testing.
OAE testing effectively assesses the function of the peripheral nervous system, but it is
unable to screen for neural dysfunction associated with the auditory nerve and brainstem. A
probe is placed in the ear canal, a stimulus provided, and the echoes generated by the cochlea
are measured using a microphone. OAE testing can measure sensory hearing loss. This hearing
screen can result in a referral due to ambient noise, vernix in the ear canal, and structural
abnormalities of the outer ear in an infant who has normal hearing.
ABR testing uses surface electrodes to measure neural activity in the cochlea, auditory
nerve, and brainstem in response to a stimulus delivered using an earphone, allowing it to detect
both sensory hearing loss and neural dysfunction. The result is delivered as pass or fail, with
further testing needed to localize the degree and nature of the hearing loss. Unlike OAE testing, it
is unaffected by ambient noise. Both testing methods miss isolated frequency region losses.
Any infant for whom the caregiver expresses concern about hearing loss should have
repeat screening, even if the initial testing was passed, regardless of the screening test used.
The mother of the infant in this vignette is young and worked in a child care center, two risk
factors for congenital cytomegalovirus (CMV) infection in her infant. CMV is believed to be a
leading cause of sensorineural hearing loss. Of note, CMV infection occurs in 0.5% to 1% of
births, with 22% of affected infants developing sensorineural hearing loss independent of
symptomatology at birth.

Suggested reading:

Foulon I, Naessens A, Foulon W, Casteels A, Gordts F. A 10-year prospective study of


sensorineural hearing loss in children with congenital cytomegalovirus infection. J Pediatr.
2008;153:84-88. DOI: 10.1016/j.jpeds.2007.12.049.

Joint Committee on Infant Hearing. Year 2007 position statement: principles and guidelines for
early hearing detection and intervention programs. Pediatrics. 2007;120: 898-921. DOI:
10.1542/peds.2007-2333. Available at:
http://pediatrics.aappublications.org/cgi/content/full/120/4/898

Wrightson AS. Universal newborn hearing screening. Am Fam Physician. 2007;75:1349-1352.


Abstract available at: http://www.ncbi.nlm.nih.gov/pubmed/17508530

Copyright 2010 by the American Academy of Pediatrics page 724


2011 PREP SA on CD-ROM

Question: 227

A previously healthy 11-year-old girl has had a dry hacking cough for 3 months associated with
fatigue, occasional fevers (temperature of 38.4C), and a 4-kg weight loss. On physical
examination, the tired-appearing child has multiple firm, nontender posterior cervical, axillary, and
inguinal nodes; her respiratory rate is slightly elevated; and she has occasional wheezes. Small
nodules are visible along the iris-pupil margin, and an ophthalmologist recently diagnosed anterior
uveitis. Laboratory findings of note include:

Hemoglobin, 10.9 g/dL (109 g/L)


White blood cell count, 16.0x10 3/mcL (16.0x109/L)
Erythrocyte sedimentation rate, 32 mm/hr
Calcium, 12.3 mg/dL (3.1 mmol/L)
Serum angiotensin converting enzyme, 110 units/L (normal, 5 to 89 units/L)

A purified protein derivative test is negative. Chest radiography shows bilateral hilar adenopathy
but no obvious parenchymal disease (Item Q227).

Of the following, the MOST useful test(s) for establishing the diagnosis is(are)

A. antinuclear antibody and rheumatoid factor

B. bone marrow biopsy

C. Epstein-Barr virus serology

D. immunofluorescence and enzyme immunoassay for Mycoplasma pneumoniae

E. lymph node biopsy

Copyright 2010 by the American Academy of Pediatrics page 725


2011 PREP SA on CD-ROM

Question: 227

(Courtesy of B Wood)
Hilar adenopathy (arrows), as described for girl in the vignette.

Copyright 2010 by the American Academy of Pediatrics page 726


2011 PREP SA on CD-ROM

Critique: 227 Preferred Response: E

The differential diagnosis for a child who has persistent symptoms of fever and respiratory
distress is broad and includes infectious, rheumatologic, neoplastic, and rare disorders. The
constellation of findings described for the girl in the vignette suggests the possibility of
sarcoidosis, a rare multisystem disease characterized by the presence of granulomas.
The cause of sarcoidosis is not known, but many reports of familial clusters of the disease
as well as a greater concordance for disease among monozygotic twins than for dizygotic
twins suggest a strong genetic component. Genomic studies have found candidate genes on
chromosome 6 among German families and chromosome 5 among African-American families.
The disease is characterized by chronic inflammation, likely the result of a cellular immune
response to an unknown antigen in genetically susceptible people. Immunologic studies indicate
an interaction of macrophages, Th1 phenotype CD4+ lymphocytes, and their resultant cytokines
and chemokines that causes an inflammatory cascade that produces granuloma. Persistent
antigenic stimulation maintains the process, and the nature of the response suggests an
infectious agent as the source of the antigen. Mycobacterium antigen is a candidate, with some
molecular evidence supporting it as a cause. Propionibacterium acnes also has been
suggested by some, but not all, studies. Interestingly, New York City firefighters who were
present on 9/11/2001 have an increased incidence of sarcoidosis.
The lung is the most common organ involved in adults and older children, but lymph nodes,
eyes, skin, liver, and spleen frequently are affected. Nonspecific systemic symptoms, including
fever, malaise, and weight loss, also are common. Pulmonary symptoms include dry cough,
dyspnea, and chest pain, and the physical examination findings may range from normal to
crackles, wheezes, and diminished breath sounds. Ophthalmologic symptoms are redness, eye
pain, blurry vision, and photophobia, and eye examination may show uveitis with firmly edged
keratitic precipitates, conjunctival granulomas, or peripheral multifocal choroiditis. Skin
manifestations are diverse and may include red, flat-topped papules involving the face; erythema
nodosum; or nonspecific macules, papules, plaques, nodules, ulcers, and subcutaneous tumors.
Pain involving multiple joints is frequent, but range of motion is maintained. Examination may
reveal effusions and boggy tenosynovitis. In adults, typical neurologic findings are facial nerve
palsy, headaches, sensory deficits, and cerebellar signs. In prepubertal children, pituitary
dysfunction may lead to growth failure, diabetes insipidus, and delayed sexual maturation. Heart
involvement may be detected because of heart block, ventricular arrhythmias, or
cardiomyopathy. Parotid gland enlargement occasionally is noted.
Sarcoidosis in children younger than 5 years of age presents distinctly with a triad of
arthritis, rash, and uveitis. In contrast to adults, the first neurologic symptom may be a seizure.
The prognosis may be worse in this age group. Blau disease is an autosomal dominant
granulomatous disease that phenotypically is the same as early-onset sarcoidosis but is not
genetically identical.
The diagnosis of sarcoidosis is based on clinical suspicion and confirmed by biopsy material
demonstrating noncaseating granulomas. Preferred sites for biopsy are peripheral lymph nodes,
skin lesions (not erythema nodosum), enlarged salivary glands, lacrimal glands, or conjunctival
nodules. Bone marrow biopsy is not necessary. Chest radiographs typically show bilateral,
symmetric hilar adenopathy with or without pulmonary infiltrates. Pulmonary function tests most

Copyright 2010 by the American Academy of Pediatrics page 727


2011 PREP SA on CD-ROM

commonly show a restrictive pattern. Bronchoalveolar lavage may demonstrate alveolitis but
often is performed before granulomas form.
There is no single pathognomonic laboratory test for sarcoidosis. Patients may demonstrate
elevated sedimentation rates, anemia, leukopenia, eosinophilia, hypergammaglobulinemia,
hypercalcemia, or hypercalciuria. Angiotensin-converting enzyme (ACE) concentrations are
elevated in more than 50% of patients who have late-onset disease but are not specific; such
assessment is more useful for monitoring the course of disease than for diagnosis. It is
important to note that normal ACE values in children are greater than those for adults. Additional
studies should be performed based on the patients symptoms.
Asymptomatic children who have isolated hilar adenopathy do not need treatment. For those
who have multisystem disease, corticosteroids are the mainstay of treatment. Other
immunosuppressive medications have been tried for patients experiencing intolerable adverse
effects or who are not responsive to steroids. Methotrexate has been used for some adults and
was successful for children in one small study. Other immunosuppressive agents are reserved
for the most difficult cases because of high toxicity. Hydroxychloroquine can be useful in adults
who have disfiguring skin lesions and to help manage hypercalcemia.
Childhood sarcoidosis in the United States is a disease primarily of the southern states, with
80% of cases occurring in Virginia, North Carolina, South Carolina, Arkansas, and Louisiana.
Although a study of Danish children younger than 15 years of age found a prevalence of 0.22 to
0.27 per 100,000, the prevalence of sarcoidosis in United States children is not certain. Most
childhood cases occur between the ages of 13 and 15 years, although there is a rarer and
distinct early childhood-onset form affecting children younger than 5 years and most commonly
presenting during the first postnatal year. In the United States, the incidence is higher for African-
American than for white adults and older children, but the difference is not pronounced for
young children.
Rheumatologic diseases, most notably juvenile idiopathic arthritis and systemic lupus
erythematosus, should be considered in the evaluation of a child who has prolonged fever and
uveitis, and assessing antinuclear antibody and rheumatoid factor may be useful in evaluating
these conditions. However, the predominant symptom of pulmonary disease and the presence of
hilar, paratracheal, and cervical lymph nodes make these rheumatologic conditions less likely for
this child. Epstein-Barr virus and Mycoplasma pneumoniae infections can present with
pulmonary symptoms, lymphadenitis, and fever. However, the time course of this patients
symptoms and the specific ophthalmic findings more strongly suggest sarcoidosis.

Suggested reading:

Baumann RJ, Robertson WC Jr. Neurosarcoid presents differently children than in adults.
Pediatrics. 2003;112:e480-e486. Available at:
http://pediatrics.aappublications.org/cgi/content/full/112/6/e480

DeRavin SS, Naumann N, Robinson MR, et al. Sarcoidosis in chronic granulomatous disease.
Pediatrics. 2006;117:e590-e595. DOI: 10.1542/peds.2005-1349. Available at:
http://pediatrics.aappublications.org/cgi/content/full/117/3/e590

Copyright 2010 by the American Academy of Pediatrics page 728


2011 PREP SA on CD-ROM

Shetty AK, Gedalia A. Childhood sarcoidosis: a rare but fascinating disorder. Pediatr Rheumatol.
2008;6:16. DOI: 10.1186/1546-0096-6-16. Available at: http://www.ped-
rheum.com/content/6/1/16

Shetty A, Gedalia A. Sarcoidosis. eMedicine Specialties, Pediatrics: General Medicine,


Pulmonology. 2009.

Copyright 2010 by the American Academy of Pediatrics page 729


2011 PREP SA on CD-ROM

Question: 228

A 10-year-old boy presents to your office as a new patient for follow-up care after being seen in
the emergency department for acute pharyngitis. On physical examination, the boy, who is obese
(>95% body mass index for age), has enlarged tonsils that are not inflamed and acanthosis
nigricans. Findings on his cardiac, pulmonary, and genital examinations are normal, and his
genitalia are prepubertal. According to his mother, his school performance is declining, he has
prolonged sleep latency, and he has been increasingly irritable, with aggressive behavior at home
and at school.

Of the following, in addition to aggressive weight management efforts, the study MOST likely to
guide therapy for several of this childs concerns is

A. chest radiography

B. electrocardiography

C. electroencephalography

D. pulmonary function testing

E. sleep study

Copyright 2010 by the American Academy of Pediatrics page 730


2011 PREP SA on CD-ROM

Critique: 228 Preferred Response: E

The clinician for the child described in the vignette is challenged by determining which
features of the childs medical and behavioral symptoms are due to his obesity. Behavioral
concerns, especially problems with attention and aggressiveness, seem to correlate with sleep
dysfunction or sleep hygiene, which suggests the need to investigate whether this child is
having problems with sleep. In addition to taking a complete history of the childs sleep pattern
since birth and ascertaining other psychosocial stressors, the clinician should request input from
the parents and the school via questionnaires such as the Pediatric Symptom Checklist, the
Vanderbilt questionnaires, and other instruments. At that point, the clinician can decide on the
risks and benefits of ancillary testing.
Questionnaires that address sleep behaviors may help with gathering more information, but
they are not evidence-based in terms of reliabililty and validity. Parents of children of this age
may have little knowledge of their childrens awakenings and other episodes during the night. A
formal sleep study or polysomnography that includes electrocardiography (ECG), pulse oximetry,
measures of airway resistance, and video capturing of paroxysmal events may provide the
most information that could lead to a diagnosis of sleep apnea.
It should be noted that sleep studies for children may not be widely available in all
communities. Further, interventions for sleep apnea, such as adenotonsillectomy and continuous
positive airway pressure machines, may have unintended consequences such as increased
weight gain, and there is little evidence at this time to support these interventions. Weight
management is the most recommended intervention.
The lack of documented seizure activity for this child obviates the need for
electroencephalography. ECG may be useful in children who are overweight and have
hypertension to provide information about cardiac ventricular function, but it does not predict cor
pulmonale or other sequelae of sleep apnea or other disturbance. Pulmonary function testing and
chest radiography are not helpful in the evaluation of sleep disorders.

Suggested reading:

Barone JG, Hanson C, DaJusta DG, Gioia K, England SJ, Schneider D. Nocturnal enuresis and
overweight are associated with obstructive sleep apnea. Pediatrics. 2009;124:e53-e59. DOI:
10.1542/peds.2008-2805. Available at:
http://pediatrics.aappublications.org/cgi/content/full/124/1/e53

Lumeng JC, Chervin RD. Epidemiology of pediatric obstructive sleep apnea. Proc Am Thorac
Soc. 2008;5:242-252. DOI: 10.1513/pats.200708-135MG. Available at:
http://pats.atsjournals.org/cgi/content/full/5/2/242?ijkey=eca267640bd210fd2febc13f1996f78815
e3d0d2

Marcus CL, Rosen G, Davidson Ward SL, et al. Adherence to and effectiveness of positive
airway pressure therapy in children with obstructive sleep apnea. Pediatrics. 2006;117:e442-
e451. DOI: 10.1542/peds.2005-1634. Available at:
http://pediatrics.aappublications.org/cgi/content/full/117/3/e442

Copyright 2010 by the American Academy of Pediatrics page 731


2011 PREP SA on CD-ROM

Owens JA, Mehlenbeck R, Lee J, King MM. Effect of weight, sleep duration, and comorbid sleep
disorders on behavioral outcomes in children with sleep-disordered breathing. Arch Pediatr
Adolesc Med. 2008;162:313-321. Available at: http://archpedi.ama-
assn.org/cgi/content/full/162/4/313

Paavonen EJ, Raikkonen K, Lahti J, et al. Short sleep duration and behavioral symptoms of
attention-deficit/hyperactivity disorder in healthy 7- to 8-year-old children. Pediatrics.
2009;123:e857-e864. DOI: 10.1542/peds.2008-2164. Available at:
http://pediatrics.aappublications.org/cgi/content/full/123/5/e857

Schechter MS, Section on Pediatric Pulmonology, Subcommittee on Obstructive Sleep Apnea


Syndrome. Technical report: diagnosis and management of childhood obstructive sleep apnea
syndrome. Pediatrics. 2002;109:e69. Available at:
http://pediatrics.aappublications.org/cgi/content/full/109/4/e69

Tarasiuk A, Simon T, Tal A, Reuveni H. Adenotonsillectomy in children with obstructive sleep


apnea syndrome reduces health care utilization. Pediatrics. 2004;113:351-356. Available at:
http://pediatrics.aappublications.org/cgi/content/full/113/2/351

Wijga AH, Scholtens S, Wieringa MH, et al. Adenotonsillectomy and the development of
overweight. Pediatrics. 2009;123:1095-1101. DOI: 10.1542/peds.2008-1502. Available at:
http://pediatrics.aappublications.org/cgi/content/full/123/4/1095

Copyright 2010 by the American Academy of Pediatrics page 732


2011 PREP SA on CD-ROM

Question: 229

You are seeing a 15-year-old girl for a health supervision visit. In reviewing her medical record,
you note that she has had persistent tachycardia over her last several visits, with heart rates in
the range of 120 to 130 beats/min, and she has lost 10 lb. Other than mild anxiety, occasional
episodes of diarrhea, infrequent episodes of double vision, and occasional palpitations, she has
been well. On physical examination, you note a thin, well-developed adolescent who has slight
diaphoresis, tremulousness, thinning hair, and eyelid retraction. Her heart rate is 135 beats/min,
respiratory rate is 18 breaths/min, and blood pressure is 115/65 mm Hg.

Of the following, the MOST likely diagnosis is

A. adrenomedullary tumor

B. anxiety disorder

C. chronic anemia

D. dilated cardiomyopathy

E. hyperthyroidism

Copyright 2010 by the American Academy of Pediatrics page 733


2011 PREP SA on CD-ROM

Critique: 229 Preferred Response: E

Hyperthyroidism in children and adolescents is rare and, therefore, often misdiagnosed.


Palpitations and sinus tachycardia frequently are the presenting complaints that prompt an
evaluation to exclude a cardiac arrhythmia. Other signs and symptoms that usually accompany
the palpitations may aid in making the diagnosis, including tremor, weight loss due to an
increased metabolic rate, emotional lability, fatigue, and muscle weakness. These findings in the
adolescent described in the vignette suggest the diagnosis.
The cause of thyrotoxicosis in adolescents may be Graves disease, subacute
nonsuppurative thyroiditis, or early Hashimoto thyroiditis. Thyroid-stimulating hormone (TSH)-
secreting pituitary tumors are rare, as is an autosomal dominant form of hyperthyroidism
involving TSH receptor gene mutations. Adolescent girls are affected six to eight times more
frequently with Graves disease than are boys. Both Graves disease and Hashimoto thyroiditis
have a genetic basis, and a significant number of affected patients have a positive family
history. The eye and skin manifestations of Graves disease, exophthalmos and pretibial
myxedema, are less common in pediatric patients than adults. The presence of a goiter can be
subtle. Hyperthyroidism usually can be diagnosed by measuring serum thyroxine, 35 3-
triiodothyronine, and TSH and assessing the free thyroxine index.
Adrenomedullary tumors are rare, the most common of which is a pheochromocytoma. This
form of tumor is associated with tachycardia, but invariably there is also systemic hypertension.
Symptoms are caused by excretion of excessive amounts of catecholamines. Approximately
10% of these tumors are malignant. Signs and symptoms of a pheochromocytoma are those of
sympathetic nervous system hyperactivity, including flank pain, elevated heart rate,
hypertension, orthostatic hypotension, palpitations, anxiety, diaphoresis, and pallor. The most
common presentation is headache, excessive sweating, and tachycardia, with the attack
subsiding in less than 1 hour.
Some of the conditions classified as anxiety disorder may have manifestations of
tachycardia, nervousness, and weight loss, but the symptoms generally are intermittent.
Chronic anemia is associated with tachycardia due to the compensatory effect of a
reduction in oxygen-carrying capacity. However, the visual changes, hair thinning, emotional
lability, diaphoresis, and gastrointestinal symptoms reported for this girl would not be expected.
Dilated cardiomyopathy is associated with left ventricular dilation and dysfunction. Among
the numerous causes are infections (eg, viral myocarditis), metabolic and mitochondrial
conditions, abnormalities of the sarcomere, dystrophinopathies (eg, Duchenne muscular
dystrophy), iatrogenic (eg, sequelae of chemotherapy), and idiopathic. Tachycardia is a common
finding, resulting from the attempt to maintain cardiac output in the face of reduced stroke
volume. Affected patients can present with diaphoresis, pallor, shortness of breath, orthopnea,
hepatomegaly, and peripheral edema. In addition, palpitations can be present due to the
myocardial irritability caused by ventricular enlargement. The palpitations can represent either
atrial or ventricular arrhythmias. The findings of visual changes, tremulousness, anxiety, and hair
thinning would be uncommon.

Suggested reading:

Copyright 2010 by the American Academy of Pediatrics page 734


2011 PREP SA on CD-ROM

Sills IN. Hyperthyroidism. Pediatr Rev. 1994;15:417-421. DOI: 10.1542/10.1542/pir.15-11-417.


Available at: http://pedsinreview.aappublications.org/cgi/content/abstract/15/11/417

Zimmerman D, Lteif AN. Thyrotoxicosis in children. Endocrinol Metab Clin North Am.
1998;27:109-126. Abstract available at: http://www.ncbi.nlm.nih.gov/pubmed/9534032

Copyright 2010 by the American Academy of Pediatrics page 735


2011 PREP SA on CD-ROM

Question: 230

A 2-year-old girl presents to the emergency department with a simple febrile seizure. After the
recovering from their shock, the parents, who are medical students, ask you whether genetic
predisposition or particular infectious agents confer risk for febrile seizures. You respond that
several genes appear to increase risk for febrile and nonfebrile seizures, but certain infectious
agents also may be more likely than others to trigger febrile seizures.

Of the following, the agent that is MOST associated with febrile seizures is

A. Aspergillus fumigatus

B. Escherichia coli

C. group A beta-hemolytic Streptococcus

D. human herpesvirus type 2

E. human herpesvirus type 6

Copyright 2010 by the American Academy of Pediatrics page 736


2011 PREP SA on CD-ROM

Critique: 230 Preferred Response: E

Febrile seizures are common in children ages 1 month to 5 years, often presenting between
6 months and 3 years of age. Typically, these are benign. The causes or predispositions are not
fully characterized but include both genetic factors and types of infections. Upper respiratory
tract infections, otitis media, and roseola due to human herpesvirus 6 (HHV 6) are the most
commonly associated illnesses. HSV 2, group A beta-hemolytic streptococcal infection,
aspergillosis, and Escherichia coli infections have not been linked to febrile seizures.
A variety of studies support a role for genetic factors, including higher concordance in
monozygotic versus dizygotic twins and large pedigrees showing autosomal dominant
inheritance patterns. Several linkage sites and gene mutations, including voltage-gated sodium
channels (SCN1A, SCN9A), have been identified as well.

Suggested reading:

Hukin J, Farrell K, MacWilliam LM, et al. Case-control study of primary human herpesvirus 6
infection in children with febrile seizures. Pediatrics. 1998;101:e3. Available at:
http://pediatrics.aappublications.org/cgi/content/full/101/2/e3

Kimia AA, Capraro AJ, Hummel D, Johnston P, Harper MB. Utility of lumbar puncture for first
simple febrile seizure among children 6 to 18 months of age. Pediatrics. 2009;123:6-12. DOI:
10.1542/peds.2007-3424. Available at:
http://pediatrics.aappublications.org/cgi/content/full/123/1/6?

Nakayama J. Progress in searching for the febrile seizure susceptibility genes. Brain Dev.
2009;31:359-365. DOI: 10.1016/j.braindev.2008.11.014. Abstract available at:
http://www.ncbi.nlm.nih.gov/pubmed/19201561

Seltz LB, Cohen E, Weinstein M. Risk of bacterial or herpes simplex virus meningitis/encephalitis
in children with complex febrile seizures. Pediatr Emerg Care. 2009;25:494-497. DOI:
10.1097/PEC.0b013e3181b0a095. Abstract available at:
http://www.ncbi.nlm.nih.gov/pubmed/19633589

Theodore WH, Epstein L, Gaillard WD, Shinnar S, Wainwright MS, Jacobson S. Human herpes
virus 6B: a possible role in epilepsy? Epilepsia. 2008;49:1828-1837. DOI: 10.1111/j.1528-
1167.2008.01699.x. Abstract available at: http://www3.interscience.wiley.com/cgi-
bin/fulltext/120750649/HTMLSTART

Copyright 2010 by the American Academy of Pediatrics page 737


2011 PREP SA on CD-ROM

Question: 231

The parents of a 6-month-old boy in your care are concerned because his development seems to
be slowing. They also say that he has begun to startle easily to loud noises, and he is less
attentive than he once was. On physical examination, he appears normally grown and has no
dysmorphisms. He displays some roving eye movements and has reduced tone compared with
his previous examination at 4 months of age. You refer the infant for an ophthalmology evaluation,
which reveals cherry-red spots of the maculae (Item Q231). Blood analysis of hexosaminidase A
activity reveals that it is undetectable.

Of the following, the MOST accurate statement regarding care/treatment for this child is that

A. bone marrow transplant is curative

B. enzyme replacement therapy is now available

C. gene therapy trials soon will be under way

D. megavitamin therapy is beneficial

E. supportive care is all that can be offered

Copyright 2010 by the American Academy of Pediatrics page 738


2011 PREP SA on CD-ROM

Question: 231

(Courtesy of RG Weaver, Jr)


Macular spot, as described for the infant in the vignette.

Copyright 2010 by the American Academy of Pediatrics page 739


2011 PREP SA on CD-ROM

Critique: 231 Preferred Response: E

The infant described in the vignette has early signs and symptoms of Tay-Sachs disease
(TSD), an autosomal recessive, neurodegenerative disorder due to near-absent-to-absent
hexosaminidase A activity. Hexosaminidase A serves to break down a specific glycolipid, GM2
ganglioside, in the lysosomes of cells in the central nervous system. When this enzyme is
markedly reduced or absent, GM2 ganglioside builds up in the cells, ultimately rendering them
nonfunctional. Historically, 90% of children who have TSD have been of Ashkenazi Jewish
descent due to a carrier frequency of 1 in 30 in this population. However, since the advent of
carrier screening programs and education pertaining to TSD in the Jewish population, a higher
percentage of affected individuals belong to other ethnic groups at this time.
Individuals who have TSD typically appear normal at birth. Beginning between 3 and 6
months of age, they may develop mild motor weakness and myoclonic jerks as well as an
exaggerated startle to loud noise. Between 8 and 10 months of age, they fail to attain new motor
milestones, and they have reduced visual attentiveness, with abnormal eye movements.
Ophthalmologic examination performed at this time reveals a "cherry red spot" caused by
prominence of the fovea centralis due to deposition of GM2 ganglioside in the surrounding
macula. By 12 months of age, there is rapid neurologic deterioration, with loss of voluntary
movements and reduced responsiveness. Seizures commonly appear at this time and are
progressively more severe. By 18 months, the head begins to enlarge due to progressive,
reactive cerebral gliosis. Between 2 and 4 years of age, affected children have swallowing
difficulties, worsening seizures, unresponsiveness, and decerebrate posturing. Death most
commonly occurs due to complications of bronchopneumonia.
The management of TSD is purely supportive and is aimed at providing adequate nutrition,
protecting the airway, and controlling seizures. Bone marrow transplant, enzyme replacement
therapy, and gene therapy are not used to treat TSD, and megavitamin therapy is not known to
be helpful.

Suggested reading:

Kaback MM. Hexosaminidase A deficiency. GeneReviews. 2006. Available at:


http://www.ncbi.nlm.nih.gov/bookshelf/br.fcgi?book=gene&part=tay-sachs

Nyhan WL, Barshop BA, Ozand PT. Tay-Sachs disease/hexosaminidase A deficiency. In: Atlas of
Metabolic Diseases. 2nd ed. London, United Kingdom: Hodder Arnold; 2005:609-615

Copyright 2010 by the American Academy of Pediatrics page 740


2011 PREP SA on CD-ROM

Question: 232

You are evaluating a 15-year-old girl who runs track and has completed orthopedic treatment for
a stress fracture of her tibia. You note that she is shorter than other family members. She has
had a few light periods since menarche 1 year ago. Her mother states that she has always been
a picky eater, but the girl has experienced no weight loss.

Of the following, the MOST appropriate anticipatory guidance for the girl is to

A. begin combined oral contraception

B. do stretching exercises after running

C. increase her calcium intake

D. increase her overall nutritional intake

E. start daily vitamin D supplements

Copyright 2010 by the American Academy of Pediatrics page 741


2011 PREP SA on CD-ROM

Critique: 232 Preferred Response: D

Active individuals need proper diets to meet increased energy and fluid needs and maintain
their body weights, replenish their glycogen stores, and provide protein to build and repair
tissue. The needs depend on sex; body size; and the activity undertaken, its duration, and its
intensity. Dietary components include macronutrients (carbohydrates, protein, and fat) and
micronutrients (fluids, electrolytes, vitamins, and minerals). Most athletes increase their intake to
accommodate these increased needs. If adequate energy is ingested from a variety of foods to
maintain body weight, vitamin and mineral supplements are not needed.
Inadequate intake of energy, protein, and some micronutrients is more common among
participants in sports that have weight requirements; encourage leanness; and require clothing
that is contour-revealing, such as gymnastics, ballet, diving, cheerleading, distance running, and
weight lifting. Female athletes, who are more likely to participate in most of these activities, are at
greater risk for nutritional complications than males. Chronic energy deficits, which the girl
described in the vignette most likely is experiencing, can result in impaired growth and
development in younger athletes, loss of body weight, disruption of endocrine function, loss of
strength and endurance, increased risk for injuries (including stress fractures), and
compromised immune function. Female athletes are at risk for the female athlete triad that
includes the following three interrelated components: disordered eating, amenorrhea, and
osteoporosis. Calcium, vitamin D, and weight-bearing exercises are needed to achieve peak
bone mass in all adolescents. Calcium requirements do not change with athletic participation.
However, if intake of these micronutrients is inadequate and caloric intake is decreased, the risk
for stress fractures increases, as for this girl. Supplementation with calcium, vitamin D, or
estrogen (combined oral contraception) without increasing caloric intake does not reverse bone
changes such as osteopenia or osteoporosis. Stretching after exercise does not affect bone
health.

Suggested reading:

American Dietetic Association; Dietitians of Canada; American College of Sports Medicine,


Rodriguez NR, Di Marco NM, Langley S. American College of Sports Medicine position stand.
Nutrition and athletic performance. Med Sci Sports Exerc. 2009;41:709-731. DOI:
10.1249/MSS.0b013e31890eb86. Available at: http://journals.lww.com/acsm-
msse/Fulltext/2009/03000/Nutrition_and_Athletic_Performance.27.aspx

Kleinman RE. Sports nutrition. In: Pediatric Nutrition Handbook. 6th ed. Elk Grove Village, Ill:
American Academy of Pediatrics; 2009:225-247

Lerand SJ Williams JF. The female athlete triad. Pediatr Rev. 2006;27:e12-e13. DOI:
10.1542/10.1542/pir.27-1-e12. Available at:
http://pedsinreview.aappublications.org/cgi/content/full/27/1/e12

Petrie HJ, Stover EA, Horswill CA. Nutritional concerns for the child and adolescent competitor.
Nutrition. 2004;20:620-631. DOI: 10.1016/j.nut.2004.04.002. Abstract available at:

Copyright 2010 by the American Academy of Pediatrics page 742


2011 PREP SA on CD-ROM

http://www.ncbi.nlm.nih.gov/pubmed/15212744

United States Department of Agriculture. Nutrition and Your Health: Dietary Guidelines for
Americans 2005. Part D: Science Base. Available at:
http://www.health.gov/dietaryguidelines/dga2005/report/HTML/D1_Adequacy.htm

Copyright 2010 by the American Academy of Pediatrics page 743


2011 PREP SA on CD-ROM

Question: 233

You are evaluating a previously healthy 18-year-old girl who reports a 4-day history of
intermittent fevers, chills, and vomiting. She returned 1 month ago from a 6-week humanitarian
educational assistance mission to Kenya that was sponsored by her college. She reports she did
not finish her chemoprophylaxis and that her mosquito bed netting had numerous holes. You
obtain thick and thin blood smears (Item Q233).

Of the following, the patient is MOST at risk for

A. cardiomyopathy

B. high-output renal failure

C. hyperglycemia

D. seizures

E. venous thromboembolism

Copyright 2010 by the American Academy of Pediatrics page 744


2011 PREP SA on CD-ROM

Question: 233

(Courtesy of the Public Health Image Library, Centers for Disease Control and Prevention)
Thin blood smear, as described for the girl in the vignette.

Copyright 2010 by the American Academy of Pediatrics page 745


2011 PREP SA on CD-ROM

Critique: 233 Preferred Response: D

Malaria remains a significant problem worldwide, with an estimated 300 million cases and 1
million deaths annually, most of which are seen in children and young infants. Malaria that is
indigenous to the United States is seen rarely, but there are an estimated 1,000 annual cases
due to affected patients traveling to the United States or travelers who are exposed while
visiting an endemic area.
Malaria classically presents with a high temperature, chills, rigor, sweats, and headache, as
described for the girl in the vignette. Symptoms are often paroxysmal. Other symptoms include
nausea, vomiting, diarrhea, cough, tachypnea, arthralgia, myalgia, and abdominal and back pain.
Hemolysis with resultant anemia, thrombocytopenia, and jaundice may occur. More severe
disease frequently is seen in newly infected, pediatric, pregnant, or immunocompromised
patients.
Plasmodium falciparum, one of several Plasmodium species that infects humans,
produces the most severe disease because it has the potential to affect the highest percentage
of red blood cells. It can produce a variety of clinical syndromes that can be fatal unless
recognized and treated promptly. Cerebral malaria, which can include a decreased level of
consciousness, seizures, retinal hemorrhages, and hemiplegia, is the most common syndrome
seen, with mortality estimated at 20% to 40% if untreated. Other potential life-threatening
symptoms are hypoglycemia, low-output renal failure, noncardiogenic pulmonary edema,
respiratory failure, and shock. Hyperglycemia, high-output renal failure, cardiomyopathy, and
thromboembolic events are not clinical manifestations of malaria.

Suggested reading:

American Academy of Pediatrics. Malaria. In: Pickering LK, Baker CJ, Kimberlin DW, Long SS,
eds. Red Book: 2009 Report of the Committee on Infectious Diseases. 28th ed. Elk Grove
Village, Ill: American Academy of Pediatrics; 2009:438-444

Krause PJ. Malaria (Plasmodium). In: Kliegman RM, Behrman RE, Jenson HB, Stanton BF, eds.
Nelson Textbook of Pediatrics. 18th ed. Philadelphia, Pa: Saunders Elsevier; 2007:1477-1484

Copyright 2010 by the American Academy of Pediatrics page 746


2011 PREP SA on CD-ROM

Question: 234

You are seeing a 16-year-old boy for the first time. He complains that he is always thirsty and
has been getting up to go to the bathroom two or three times a night for the past few weeks. On
physical examination, he has a body mass index of 35 kg/m2 with a central weight distribution,
acanthosis nigricans of the neck and axillae, and a blood pressure of 150/90 mm Hg. He is at
Sexual Maturity Rating 5 puberty. He says that he has always been big-boned and he likes to eat.
His mother and father both have diabetes. His mother had a mild stroke 2 years ago, but is now
"getting around OK." His blood glucose measures 273 mg/dL (15.2 mmol/L).

Of the following, the additional laboratory study that is MOST likely to yield abnormal results for
this boy is a

A. complete blood count

B. high-density lipoprotein cholesterol

C. serum creatinine

D. serum free thyroxine

E. urine microalbumin

Copyright 2010 by the American Academy of Pediatrics page 747


2011 PREP SA on CD-ROM

Critique: 234 Preferred Response: B

The long-term complications of type 2 diabetes include macrovascular disease leading to


myocardial infarction, stroke, and peripheral vascular disease; neuropathy; proteinuria; renal
failure; and retinopathy. The young man described in the vignette likely has type 2 diabetes, like
his parents, and given his body mass index and findings on physical examination, he also has
metabolic syndrome. His hypertension puts him at increased risk for stroke, like his mother.
However, at this point, the most likely additional abnormal laboratory finding would be low high-
density lipoprotein (HDL) and high low-density lipoprotein cholesterol values related to his
metabolic syndrome. The macrovascular complications of diabetes can become apparent
relatively early in the course of type 2 diabetes in adolescents and young people.
His complete blood count result is likely to be normal, and he is unlikely to have sufficiently
severe renal disease to have an abnormal serum creatinine reading. His thyroid function should
be normal. Approximately 10% of individuals who have diabetes mellitus type 1 develop chronic
lymphocytic thyroiditis, but this is not common in type 2 diabetes. Of note, many obese people
have slight elevations in thyroid-stimulating hormone (TSH) concentrations, which have been
attributed to "TSH resistance" of unknown cause. The boy might have an elevated urine
microalbumin reading because of his weight and his poor diabetes control, but low HDL
cholesterol values are much more likely to be found at the time of diagnosis.

Suggested reading:

Pinhas-Hamiel O, Zeitler P. Acute and chronic complications of type 2 diabetes mellitus in children
and adolescents. Lancet. 2007;369:1823-1831. DOI: 10.1016/S0140-6736(07)60821-6. Abstract
available at: http://www.ncbi.nlm.nih.gov/pubmed/17531891

Rosenbloom AL, Silverstein JH, Amemiya S, Zeitler P, Klingensmith GJ. Type 2 diabetes in children
and adolescents. Pediatr Diabetes. 2009;10(suppl 12):17-32

Rosenbloom AL, Silverstein JH, Amemiya S, Zeitler P, Klingensmith GJ; International Society for
Pediatric and Adolescent Diabetes. ISPAD Clinical Practice Consensus Guidelines 20062007.
Type 2 diabetes in the child and adolescent. Pediatr Diabetes. 2008;9:512526. DOI:
10.1111/j.1399-5448.2008.00429.x

Shah AS, Dolan LM, Kimball TR, et al. Influence of duration of diabetes, glycemic control, and
traditional cardiovascular risk factors on early atherosclerotic vascular changes in adolescents
and young adults with type 2 diabetes mellitus. J Clin Endocrinol Metab. 2009;94:3740-3745.
Abstract available at: http://www.ncbi.nlm.nih.gov/pubmed/19723759

Copyright 2010 by the American Academy of Pediatrics page 748


2011 PREP SA on CD-ROM

Question: 235

A 10-year-old boy is undergoing cognitive testing as part of a comprehensive evaluation. He is


suspected of having attention-deficit/hyperactivity disorder because he is very distracted in his
classroom and has trouble staying in his seat. His teachers describe him as well adjusted, with
good self-esteem. He is a star athlete, eats a nutritious diet, and has no problems sleeping. His full-
scale intelligence quotient (IQ) is 87. However, there were a lot of inconsistencies on the various
subtests. He did better in the items that were administered earlier in the session. His parents ask
what factors may have influenced the scores on his test.

Of the following, the factor that likely has the GREATEST influence on the boys test results is

A. lack of breakfast on day of testing

B. lack of motivation

C. level of anxiety

D. level of attentional difficulties

E. the parents IQs

Copyright 2010 by the American Academy of Pediatrics page 749


2011 PREP SA on CD-ROM

Critique: 235 Preferred Response: D

The boy in the vignette is described as being easily distracted, and his level of attention
most likely has the greatest influence on his test scores. Familial factors such as the parents
intelligence quotient (IQ) and education may play a role in a childs IQ. However, this boys test
results are more supportive of an attentional issue, based on his better performance on the
items that were administered earlier during the testing session. He does not appear to be
anxious or lacking motivation, and he typically eats nutritious meals. If the boy is found to have
attention-deficit/hyperactivity disorder, treatment should be initiated and cognitive assessment
repeated.
Intelligence tests are used by schools and psychologists to measure cognitive functioning in
a formal environment. The scores predict the ease with which people learn in formal situations
but do not necessarily predict success in life or occupations. Factors that may affect
performance on intelligence tests include language fluency, access to educational stimulation,
educational resources, motivation, and emotional functioning. IQ also is affected by the childs
genetic makeup and the quality of home and community life. A childs IQ tends to stabilize with
age. A child of approximately 10 years of age tends to have little fluctuation in the IQ score.
However, even an older child may have his or her score compromised when dealing with a life
stress, such as divorce or death of a loved one.

Suggested reading:

Braaten EB, Norman D. Intelligence (IQ) testing. Pediatr Rev. 2006;27:403-407. DOI:
10.1542/10.1542/pir.27-11-403. Available at:
http://pedsinreview.aappublications.org/cgi/content/full/27/11/403

Mahone EM. Psychological assessment. In: Accardo PJ, ed. Capute & Accardos
Neurodevelopmental Disabilities in Infancy and Childhood. Baltimore, Md: Paul H. Brookes
Publishing Co; 2008:261-284

Wechsler D. Manual for the Wechsler Intelligence Scale for Children-Fourth Edition. New
York, NY: Psychological Corporation; 2003

Copyright 2010 by the American Academy of Pediatrics page 750


2011 PREP SA on CD-ROM

Question: 236

The father of one of your patients is planning a business trip to southern China. The family is
concerned about possible exposure to avian influenza and asks for guidance.

Of the following, the BEST advice is for him to

A. avoid eating poultry while traveling

B. avoid visits to markets where live birds are sold

C. receive an additional dose of influenza vaccine prior to departure

D. take amantadine prophylaxis if exposed to a possible case of avian influenza

E. wear a surgical mask when in any public gathering

Copyright 2010 by the American Academy of Pediatrics page 751


2011 PREP SA on CD-ROM

Critique: 236 Preferred Response: B

Avian influenza H5N1 is a highly pathogenic strain in bird and poultry populations that
originated in Southeast Asia in 1997. It is not a human strain, but it has demonstrated the
capacity for transmission to humans who have close contact with infected birds or poultry.
H5N1-infected humans frequently develop severe lower respiratory disease, with a greater than
50% mortality observed in confirmed cases. Accordingly, travelers to areas where avian
influenza H5N1 is endemic should avoid contact with live poultry, including visits to markets
where live birds are sold. H5N1-infected birds and poultry have been detected from Southeast
Asia across to the Middle East into Africa and Europe. Human cases to date have been reported
primarily in Southeast Asia, Iraq, Turkey, and Egypt.
Thorough cooking of food inactivates the virus, although if there are concerns about food
handling or cooking procedures, avoidance of poultry may be appropriate. Limited human-to-
human transmission may occur, based on case reports, but such transmission is rare. There is
no indication for wearing a surgical mask in public as protection against avian H5N1 exposure.
Seasonal influenza vaccines (and the recently approved monovalent H1N1 vaccine) confer
no protection against this strain because of the marked differences in the hemagglutinin and
neuraminidase proteins between this virus and the vaccine strains. A specific H5N1 vaccine has
been developed and approved, but it is presently being stockpiled by the government in case of
an outbreak and is not available for travelers. The H5N1 strain is susceptible to neuraminidase
inhibitors (oseltamivir, zanamivir) but resistant to amantadine and rimantadine. If a person is
exposed to this strain, prophylaxis with a neuraminidase inhibitor drug could be considered.

Suggested reading:

Abdel-Ghafar AN, Chotpitayasunondh T, Gao Z, et al; Writing Committee of the Second World
Health Organization Consultation on Clinical Aspects of Human Infection with Avian Influenza A
(H5N1) Virus. Update on avian influenza A (H5N1) virus infection in humans. N Engl J Med.
2008;358:261-273. Extract available at: http://content.nejm.org/cgi/content/extract/358/3/261

Centers for Disease Control and Prevention. Avian Influenza (Bird Flu). Available at:
http://www.cdc.gov/flu/avian/

World Health Organization. Global Alert and Response: Avian Influenza. 2010. Available at:
http://www.who.int/csr/disease/avian_influenza/en/

Copyright 2010 by the American Academy of Pediatrics page 752


2011 PREP SA on CD-ROM

Question: 237

A 3-year-old girl who attends child care has had 2 days of abdominal pain, bloody diarrhea, and
fever. A stool culture grows Escherichia coli O157:H7. Several other children in the child care
center also are ill with diarrhea. You suspect an outbreak.

Of the following, the BEST advice you can give to the child care center on managing the outbreak
is to

A. close the child care center to new admissions during the outbreak

B. document the number of diarrheal stools each ill child is having

C. keep children with diarrhea in separate rooms from well children

D. treat all children with antibiotic therapy

E. use bottled water in the child care center

Copyright 2010 by the American Academy of Pediatrics page 753


2011 PREP SA on CD-ROM

Critique: 237 Preferred Response: A

An outbreak of diarrhea caused by Escherichia coli O157:H7 should be reported to public


health authorities immediately to prevent further disease transmission. It is important to close the
child care center to new admissions during the outbreak. All symptomatic children should be
excluded from attendance until the diarrhea has resolved and two stool cultures are negative for
E coli O157:H7. It is inappropriate to allow children who have E coli O157:H7 diarrhea to remain
in child care, even if they are separated from well children. Because ill children should not be
attending child care, there is no need for workers at the center to document the number of
stools from each ill child. E coli O157:H7 is transmitted via ingestion of contaminated food,
especially ground beef, or by contact with infected feces. Therefore, the use of bottled water in
the child care center would not prevent disease spread.
There are at least five diarrheagenic E coli pathotypes. Illness due to Shiga toxin-producing
E coli (STEC) strains, of which STEC O157:H7 is the most virulent, usually begins as nonbloody
diarrhea but then becomes diarrhea with visible or occult blood. It can occur in children of all
ages. Severe abdominal pain is common, but fever occurs in fewer than one third of patients.
Infection may progress to hemorrhagic colitis in severe cases, and hemolytic-uremic syndrome
can occur. Whether or not to administer antibiotics to children who have STEC diarrhea remains
controversial, but most experts would not initiate antimicrobial therapy because no benefit has
been proven. Studies have failed to confirm that children who have STEC hemorrhagic colitis
and are treated with antibiotics have a greater risk of developing hemolytic-uremic syndrome.
Patients whose diarrhea is due to enterotoxigenic strains of E coli (ETEC) often have a
moderately severe, self-limited illness characterized by abdominal cramping and watery
diarrhea. Infection caused by this pathotype of E coli accounts for 30% to 40% of cases of
travelers diarrhea. It can occur in persons of all ages who travel to resource-limited countries.
In addition, it is a common cause of diarrhea in children younger than 5 and older than 15 years
of age who reside in these countries. It is uncommon in the United States. ETEC usually is
acquired by ingestion of contaminated food and water, and person-to-person transmission is
uncommon. Therefore, drinking bottled water when traveling to resource-limited countries is
helpful in preventing ETEC diarrhea.
Enteropathogenic strains of E coli (EPEC) produce severe watery diarrhea, predominantly in
children younger than 2 years of age in developing countries. It is not a cause of travelers
diarrhea. The illness can result in significant dehydration and may lead to death. Chronic,
persistent infection can lead to severe wasting.
Enteroinvasive E coli (EIEC) strains can cause mild-to-severe dysentery. Common clinical
manifestations include fatigue, fever, abdominal cramping, and tenesmus. Watery, nonbloody
diarrhea usually progresses to blood-streaked, mucoid diarrhea but rarely becomes grossly
bloody. Fecal leukocytes may be present. The illness usually lasts 1 to 2 weeks.
Lastly, illness caused by enteroaggregative E coli (EAEC) strains usually presents as
watery diarrhea without fever or fecal leukocytes. The diarrhea can occur in children of all ages
in developed and developing countries and can result in a prolonged illness of more than 14
days duration.
Nondiarrheagenic strains of E coli are associated with infections of the gastrointestinal
tract (peritonitis, intra-abdominal abscess, ascending cholangitis), skin (neonatal breast

Copyright 2010 by the American Academy of Pediatrics page 754


2011 PREP SA on CD-ROM

abscess, necrotizing fasciitis), genitourinary tract (urinary tract infection, pelvic abscess), lungs
(neonatal pneumonia), central nervous system (neonatal meningitis), bones and joints (neonatal
osteomyelitis and septic arthritis), and bloodstream (neonate, immunocompromised host).

Suggested reading:

American Academy of Pediatrics. Escherichia coli diarrhea (including hemolytic-uremic


syndrome). In: Pickering LK, Baker CJ, Kimberlin DW, Long SS, eds. Red Book: 2009 Report of
the Committee on Infectious Diseases. 28th ed. Elk Grove Village, Ill: American Academy of
Pediatrics; 2009:294-298

Berry AA, Velarde JJ, Nataro JP. Diarrhea-causing and dysentery-causing Escherichia coli. In:
Feigin RD, Cherry JD, Demmler-Harrison GJ, Kaplan SL, eds. Feigin & Cherrys Textbook of
Pediatric Infectious Diseases. 6th ed. Philadelphia, Pa: Saunders Elsevier; 2009:1525-1540

Copyright 2010 by the American Academy of Pediatrics page 755


2011 PREP SA on CD-ROM

Question: 238

As you are examining a 6-year-old boy during a routine health supervision visit, his mother
expresses concerns about his bedwetting at night. He achieved daytime dryness at age 3 years.
The boy experiences daytime urgency and frequency, but he has no daytime accidents or
associated dysuria, fever, abdominal pain, or constipation. Urinalysis reveals a specific gravity of
1.025; pH of 6; and no blood, protein, nitrite, or leukocyte esterase. Microscopy yields negative
findings.

Of the following, the MOST appropriate treatment for this patients symptoms is

A. desmopressin acetate (DDAVP)

B. enuresis alarm

C. imipramine

D. oxybutynin chloride

E. trimethoprim-sulfamethoxazole

Copyright 2010 by the American Academy of Pediatrics page 756


2011 PREP SA on CD-ROM

Critique: 238 Preferred Response: D

The patient described in the vignette exhibits nocturnal enuresis with daytime symptoms of
urgency and frequency but no daytime incontinence. The negative findings on urinalysis, with a
urine specific gravity greater than 1.020, helps eliminate the likelihood of a secondary cause for
enuresis, such as a urinary tract infection or a urinary concentrating defect, as can be seen in
diabetes insipidus. Most treatments for nocturnal enuresis are withheld until the patient is at least
7 years of age. The pathogenesis of enuresis is likely multifactorial, including genetics, a
functionally reduced bladder capacity, diminished arousal from sleep, and reduced antidiuretic
hormone secretion. Treatment options usually target one or more of these potential abnormalities.
In 2006, the International Childrens Continence Society published standards for terminology
and definitions related to children who have voiding dysfunction, including nocturnal enuresis.
Incontinence is defined as urinary leakage during the day or night at an age when continence
should have been achieved (age 5 years or older). Enuresis is intermittent incontinence of urine
while sleeping. The new classification system subdivides enuresis into monosymptomatic
(nighttime incontinence without daytime symptoms) and nonmonosymptomatic (with lower
urinary tract symptoms that include frequency, urgency, daytime incontinence, and hesitancy).
Primary enuresis is defined as enuresis in a child who has been dry for less than 6 months.
Secondary enuresis is defined as the onset of enuresis in a child who previously was continent
for at least 6 months.
From a clinical perspective, achievement of dryness at night is expected by age 5 years.
Enuresis (nocturnal incontinence) occurs in 15% of 5 year olds, and rates decrease gradually
so that 7% of 10 year olds and 1% of 15 year olds are troubled by this condition. Most children
become bothered by enuresis by about age 7, when it begins to interfere with social activities
(such as "sleep overs").
The evaluation of a child who has enuresis should begin with a history and physical
examination. Inquiry into the number of nights wet per week is an important starting point to
gauge the severity of the condition. Associated symptoms such as daytime incontinence and
voiding symptoms of urgency, frequency, and hesitancy should be sought. Nocturnal fluid
consumption that awakens the child from sleep raises the question of a urinary concentrating
defect. A history of constipation, dysuria, fever, abdominal pain, and prior dryness raises the
possibility of a secondary cause for the urinary incontinence. Family history is very important
because often one or both parents may have suffered from enuresis as a child. A parent who
had primary nocturnal enuresis has a 40% to 45% chance of having an affected child, and
when both parents were affected, the child has approximately a 75% chance of having
enuresis.
Treatment for enuresis consists of behavior modification and pharmacologic therapy.
Behavior therapy can begin by having the child void immediately before bedtime and restricting
fluid consumption 2 hours before bedtime. Bedwetting alarms can be either auditory or silent
(vibratory) and are the most effective strategy for nocturnal enuresis, with success rates
approaching 70%.
Medications tend to be more temporizing measures to treat rather than "cure" enuresis.
Desmopressin acetate effectively reduces the urine volume at night but does little to help
children who have reduced functional bladder capacity, bladder instability, or daytime symptoms.

Copyright 2010 by the American Academy of Pediatrics page 757


2011 PREP SA on CD-ROM

Also, the intranasal preparation should not be used because it is associated with hyponatremic
seizures that were sometimes fatal. Even oral desmopressin should be stopped when the child
develops an intercurrent illness that predisposes him or her to dehydration and electrolyte
disturbance. Anticholinergic agents (eg, oxybutynin chloride) reduce bladder contractions,
making such agents especially useful in treating children who have nocturnal enuresis
associated with daytime symptoms of incontinence, urgency, and frequency, such as the boy
described in the vignette. Imipramine is recommended by some due to its effectiveness, but its
adverse effect profile, especially fatal cardiac arrhythmias with overdoses, makes this agent a
high risk for a nonlife-threatening condition.
The boy in the vignette, who exhibits nonmonosymptomatic enuresis, has daytime symptoms
suggestive of an overactive bladder that could benefit from an anticholinergic agent such as
oxybutynin chloride. Desmopressin acctate and enuresis alarm are unlikely to be beneficial for
the daytime symptoms. There is no evidence of a urinary tract infection in this patient, obviating
the need for antibiotic treatment.

As a result of reviewing this information, do you intend to make a change in practice


to provide better patient care?
Yes No

Suggested reading:

Graham KM, Levy JB. Enuresis. Pediatr Rev. 2009;30:165-172. DOI: 10.1542/10.1542/pir.30-5-
165. Available at: http://pedsinreview.aappublications.org/cgi/content/full/30/5/165

Nevus T, von Gontard A, Hoebeke P, et al. The standardization of terminology of lower urinary
tract function in children and adolescents: report from the Standardisation Committee of the
International Children's Continence Society. J Urol. 2006;176:314-324. DOI: 10.1016/S0022-
5347(06)00305-3. Abstract available at: http://www.ncbi.nlm.nih.gov/pubmed/16753432

Robson WL. Current management of nocturnal enuresis. Curr Opin Urol. 2008;18:425-430. DOI:
10.1097/MOU.0b013e3282fcea9c. Abstract available at:
http://www.ncbi.nlm.nih.gov/pubmed/18520767

Copyright 2010 by the American Academy of Pediatrics page 758


2011 PREP SA on CD-ROM

Question: 239

A 10-year-old girl who has a peanut allergy is in the clinic for her annual follow-up visit. Her
parents explain that when their daughter was 4 years old, she experienced severe anaphylaxis
after eating peanut butter. At age 5, she was referred to an allergist, who performed skin and
blood testing to peanut butter. Her skin test showed a wheal of 15 mm, and her serum
immunoglobulin (Ig) E test result was 52 kU/L. The child knows to avoid peanut products, but her
parents are concerned about their daughters risk for other food allergies.

Of the following, the food MOST likely to cause an allergic reaction in this patient is

A. beans

B. eggs

C. soy

D. tree nuts

E. wheat

Copyright 2010 by the American Academy of Pediatrics page 759


2011 PREP SA on CD-ROM

Critique: 239 Preferred Response: D

The child described in this vignette, who has an immunoglobulin (Ig) E-mediated peanut
allergy, needs to remain vigilant in making food choices and reading food labels because foods
may contain hidden ingredients or may be contaminated with allergenic proteins. Specifically, up
to 50% of children who have peanut food allergies may develop tree nut allergies. Interestingly,
even though peanuts are legumes, only 5% of children who have peanut allergies have
problems with other legumes, such as beans or soy. Accordingly, they do not need to avoid
other legumes unless there is a history of a reaction.
Wheat and egg are common food allergies in children (Item C239) that usually present prior
to age 5 years. Of note, children who initially have IgE-mediated egg allergies are nine times
more likely to develop peanut allergies, although the reverse risk (ie, starting with a peanut food
allergy and subsequently developing an egg allergy) is unknown.

As a result of reviewing this information, do you intend to make a change in practice


to provide better patient care?
Yes No

Suggested reading:

Tatachar P, Kumar S. Food-induced anaphylaxis and oral allergy syndrome. Pediatr Rev.
2008;29:e23-e27. DOI: 10.1542/10.1542/pir.29-4-e23. Available at:
http://pedsinreview.aappublications.org/cgi/content/full/29/4/e23

Waibel KH. Anaphylaxis. Pediatr Rev. 2008;29:255-263. DOI: 10.1542/10.1542/pir.29-8-255.


Available at: http://pedsinreview.aappublications.org/cgi/content/full/29/8/255

Copyright 2010 by the American Academy of Pediatrics page 760


2011 PREP SA on CD-ROM

Critique: 239

Copyright 2010 by the American Academy of Pediatrics page 761


2011 PREP SA on CD-ROM

Question: 240

A mother brings her 18-month-old daughter to the office because she has had a temperature up
to 39.8C for the past 24 hours without any other signs or symptoms. No one at home is ill, but the
girl attends a child care center. Her immunizations are up to date. Acetaminophen has provided
some relief. On physical examination, the child appears well and has a temperature of 39.1C,
heart rate of 140 beats/min, respiratory rate of 28 breaths/min, and blood pressure of 100/60 mm
Hg. The remainder of her examination findings are within normal parameters.

Of the following, based on the history and physical examination findings, the MOST likely cause
of this childs fever is

A. enterovirus infection

B. Escherichia coli urinary tract infection

C. respiratory syncytial virus infection

D. rotavirus infection

E. Streptococcus pneumoniae bacteremia

Copyright 2010 by the American Academy of Pediatrics page 762


2011 PREP SA on CD-ROM

Critique: 240 Preferred Response: A

Among infants and children presenting to medical attention with fever of fewer than 7 days
duration, as many as 40% do not have a localizing source after careful history and physical
examination. Most of such patients have self-limited, infectious causes; a small minority have
bacterial causes that require treatment. It is unlikely that children who have acute fever without
a source have oncologic or noninfectious inflammatory causes for their fevers. The incidence of
fever without a source varies with age and is most common in toddlers.
Because infants younger than 3 months of age who have fever do not localize infection
consistently, their evaluation often includes laboratory testing. As a result, more than 70% have
an infectious agent identified, of which 10% to 12% are bacterial. Those without any source
identified are presumed to have self-limited viral infections. Occult bacterial causes include
sepsis, urinary tract infection, meningitis, bacteremia, osteomyelitis and septic arthritis, and
pneumonia. Commonly isolated pathogens in this age group include group B Streptococcus,
Listeria monocytogenes, Salmonella, Escherichia coli, and Staphylococcus aureus.
Among children 3 to 36 months of age, approximately 60% present with signs or symptoms
attributable to a localized bacterial or viral cause. Most of the remaining 40% are presumed to
have self-limited viral infections. Some children in this age group have occult bacterial infections.
Likely occult sources among the highly febrile (>39.0C) include bacteremia (<1% to 1.5%, varies
with immunization status), urinary tract infection (<1% to 8.3%, varies with sex and age), and
pneumonia (20% in those <5 years who have temperatures >39.0C and white blood cell counts
>20.0x103/mcL [20.0x109/L]).
Epidemiology suggests that the toddler described in the vignette most likely is suffering from
a viral infection, and enterovirus is a common cause of fever without other signs or symptoms.
Although obtaining a urinalysis and urine culture is prudent in this child because urinary tract
infections are a relatively common occult bacterial source and can be found in up to 7% of
febrile girls between 0 and 24 months of age without a localizing source, she is more likely to
have a viral infection. Rotavirus and respiratory syncytial virus are unlikely to cause fever
without other symptoms such as rhinorrhea, wheezing, or diarrhea. Streptococcus pneumoniae
bacteremia is very uncommon, with an incidence of less than 1% in the fully immunized patient.

Suggested reading:

Allen CH. Fever without a source in children 3 to 36 months of age. UpToDate Online 17.3.
2009. Available at:
http://www.utdol.com/online/content/topic.do?topicKey=ped_emer/4803&source=HISTORY

Powell KR. Fever without a focus. In: Kleigman RM, Behrman RE, Jenson HB, Stanton BF, eds.
Nelson Textbook of Pediatrics. 18th ed. Philadelphia, Pa: Saunders Elsevier; 2007:1087-1093

Smitherman HF, Macias CG. Evaluation and management of fever in the neonate and young infant
(less than three months of age). UpToDate Online 17.3. 2009. Available at:
http://www.utdol.com/online/content/topic.do?topicKey=ped_emer/2115

Copyright 2010 by the American Academy of Pediatrics page 763


2011 PREP SA on CD-ROM

Question: 241

You are evaluating a 4-month-old male infant because of worsening postprandial emesis. When
born at term following an uncomplicated pregnancy and delivery, he weighed 3.4 kg. At 4 weeks
of age, he began having frequent episodes of spitting-up after meals, which prompted one visit to
the local emergency department, where abdominal ultrasonography was performed. Since that
time, the baby has continued to have frequent postprandial emesis. His diet is 4 to 5 oz of a
standard cow milk-based formula every 4 hours. On physical examination, the alert, vigorous,
and happy infant is wearing a formula-stained bib and weighs 6 kg. Of note is a small umbilical
hernia.

Of the following, the MOST appropriate next step is

A. administration of lansoprazole 1 mg/kg per day

B. administration of ranitidine 3 mg/kg BID

C. an upper gastrointestinal tract radiographic series

D. thickening of formula with rice cereal

E. trial of a hypoallergenic infant formula

Copyright 2010 by the American Academy of Pediatrics page 764


2011 PREP SA on CD-ROM

Critique: 241 Preferred Response: D

The otherwise healthy and thriving 4-month-old infant described in the vignette presents
with recurrent, postprandial regurgitation that has been present since 1 month of age. Initially,
vomiting prompted an ultrasonographic evaluation, presumably to rule out infantile pyloric
stenosis. Considering the duration of symptoms, the infants healthy appearance, and his
appropriate weight gain, the most likely clinical diagnosis is gastroesophageal reflux (GER). The
primary management objective should be to reduce, if possible, the frequency and severity of
vomiting episodes, and the intervention most likely to achieve this goal is feeding of thickened
infant formula combined with reduced volumes of feedings.
GER during early infancy is a physiologic state associated with the delayed maturation of
gastrointestinal motility and function. Etiologic factors include immaturity of the anatomic
antireflux barrier, reduced esophageal capacitance and gastric compliance, delayed gastric
emptying, and perhaps most prominently, increased transient lower esophageal sphincter (LES)
relaxations. During postnatal life, the frequency and severity of reflux episodes decreases in
parallel with the maturation of LES function, so that by 1 year of age, clinical symptoms of GER
are reported in fewer than 10% of infants.
Regurgitation (the passage of gastric contents into the oropharynx) and vomiting (expulsion
of gastric contents from the mouth) are associated with diverse clinical problems. Clearly, any
infant who has a vomiting history should be assessed carefully for the presence of "red flags"
that might suggest a serious, underlying clinical condition (Item C241A). Regardless of
associated symptoms, any infant whose vomiting episodes commence after 6 months of age
must be evaluated further for a non-GER diagnosis. In general, however, infants who have a
history of chronic postprandial emesis and whose history and physical examination findings do
not encompass the warning signals of a non-GER diagnosis should receive the diagnosis of
uncomplicated infant GER. Accordingly, the infant in the vignette need not undergo further
diagnostic studies. In particular, the use of barium upper gastrointestinal radiographic
examinations should be limited to those cases where concerns of swallowing function or
anatomic integrity (eg, oropharyngeal dysphagia, esophageal stricture, intestinal malrotation, or
atresia) are paramount. Contrast studies are not helpful for evaluating GER.
A recommended approach to the management of uncomplicated infant GER is shown in
(refer to Item C241B). Most importantly, parents should be reassured and educated regarding the
generally benign nature of this problem. For infants who have uncomplicated reflux, neither
dietary nor pharmacologic intervention has been demonstrated to accelerate maturation of the
LES or influence natural history. Nevertheless, several studies have shown that the feeding of
thickened infant formula is an effective clinical strategy that results in reduced frequency and
severity of vomiting episodes. A variety of approaches may be used to achieve formula
thickening, including the use of rice cereal, dietary gums, and commercially available "antireflux"
formulas. The efficacy of thickened feedings may be enhanced further by reducing feeding
volumes. If thickened feedings do not result in symptomatic improvement or if associated atopic
symptoms are demonstrated, a 2-week trial of a hypoallergenic protein hydrolysate formula may
be considered.
Considerable controversy surrounds pharmacologic intervention in GER. Abundant evidence
indicates that the use of so-called prokinetic drugs (eg, metoclopramide) does not alter the

Copyright 2010 by the American Academy of Pediatrics page 765


2011 PREP SA on CD-ROM

clinical course and, therefore, has little, if any, role in management. Furthermore, the adverse
effect profiles of most of these drugs make their use during infancy problematic. Although
studies in preterm infants indicate that erythromycin, a macrolide antibiotic that exerts a motilin
agonist effect in subtherapeutic doses, may be useful in the management of selected cases of
refractory GER, this agent has not been evaluated for uncomplicated reflux. A recent physician
survey found that a considerable number of infants who have presumptive diagnoses of clinical
GER receive empiric trials of acid-reducing medications, either histamine2-receptor antagonists
(eg, ranitidine) or proton pump inhibitors (PPIs) (eg, lansoprazole). These agents have
demonstrated considerable efficacy in the treatment of symptomatic GER and in the healing of
reflux esophagitis in older children and adults, but double-blind, placebo-controlled studies in
infants have failed to show an effect on symptoms such as irritability, back arching, or feeding
refusal. Nevertheless, in cases where suspected esophageal symptoms occur in conjunction
with recurrent emesis, a brief trial of acid-reducing medication may be considered. The use of
these drugs must be tempered by emerging data indicating that long-term PPI use is associated
with an increased incidence of complications, including candidal esophagitis and community-
acquired pneumonia. Clinical trials aimed at determining the safety and efficacy of PPIs in infants
are ongoing. However, current recommendations of the North American Society for Pediatric
Gastroenterology, Hepatology and Nutrition do not support the routine use of these drugs in the
management of infant GER.

Suggested reading:

Craig WR, Hanlon-Dearman A, Sinclair C, Taback SP, Moffatt M. Metoclopramide, thickened


feedings, and positioning for gastro-oesophageal reflux in children under two years. Cochrane
Database Syst Rev. 2004;4:CD003502. DOI: 10.1002/14651858.CD003502.pub2. Available at:
http://www.mrw.interscience.wiley.com/cochrane/clsysrev/articles/CD003502/frame.html

Diaz DM, Winter HS, Colletti RB, et al; NASPGHAN/CDHNF Scientific Advisory Board. Knowledge,
attitudes and practice styles of North American pediatricians regarding gastroesophageal reflux
disease. J Pediatr Gastroenterol Nutr. 2007;45:56-64. DOI: 10.1097/MPG.0b013e318054b0dd.
Available at:
http://journals.lww.com/jpgn/Fulltext/2007/07000/Knowledge,_Attitudes_and_Practice_Styles_of
_North.9.aspx

Horvath A, Dziechciarz P, Szajewska H. The effect of thickened-feed interventions on


gastroesophageal reflux in infants: systematic review and meta-analysis of randomized,
controlled trials. Pediatrics. 2008;122:e1268-e1277. DOI: 10.1542/peds.2008-1900. Available at:
http://pediatrics.aappublications.org/cgi/content/full/122/6/e1268

Michail S. Gastroesophageal reflux. Pediatr Rev. 2007;28:101-110. DOI: 10.1542/10.1542/pir.28-


3-101. Available at: http://pedsinreview.aappublications.org/cgi/content/full/28/3/101

Rudolph CD, Mazur LJ, Liptak GS, et al; North American Society for Pediatric Gastroenterology

Copyright 2010 by the American Academy of Pediatrics page 766


2011 PREP SA on CD-ROM

and Nutrition. Guidelines for evaluation and treatment of gastroesophageal reflux in infants and
children: recommendations of the North American Society for Pediatric Gastroenterology and
Nutrition. J Pediatr Gastroenterol Nutr. 2001;32(suppl 2):S1-S31. Available at:
http://journals.lww.com/jpgn/Fulltext/2001/00002/Guidelines_for_Evaluation_and_Treatment_of.1.
aspx

Wenzl TG, Schneider S, Scheele F, Silny J, Heimann G, Skopnik H. Effects of thickened feeding
on gastroesophageal reflux in infants: a placebo-controlled crossover study using intraluminal
impedance. Pediatrics. 2003;111:e355-e359. Available at:
http://pediatrics.aappublications.org/cgi/content/full/111/4/e355

Copyright 2010 by the American Academy of Pediatrics page 767


2011 PREP SA on CD-ROM

Critique: 241

Copyright 2010 by the American Academy of Pediatrics page 768


2011 PREP SA on CD-ROM

Critique: 241

Copyright 2010 by the American Academy of Pediatrics page 769


2011 PREP SA on CD-ROM

Critique: 241

Copyright 2010 by the American Academy of Pediatrics page 770


2011 PREP SA on CD-ROM

Critique: 241

Copyright 2010 by the American Academy of Pediatrics page 771


2011 PREP SA on CD-ROM

Question: 242

You are called to attend the cesarean section delivery of a 42 weeks gestation infant because of
failure to progress complicated by severe oligohydramnios. Results of maternal screens are
unremarkable, including a negative group B streptococcal test. Rupture of the membranes at the
time of delivery reveals scant fluid that is meconium stained. Upon assessment on the warmer,
the infant appears to be vigorous and has good respiratory effort, a heart rate of greater than
100 beats/min, central cyanosis, and meconium staining of peeling "post dates" skin. You begin
blow-by oxygen, which improves his color, but he develops tachypnea and grunting 5 minutes
after birth. He is transferred to the special care nursery, where he is intubated endotracheally
and an umbilical venous line is placed. Pre- and postductal saturations are 97% while receiving
60% oxygen on the ventilator. His chest radiograph is seen in (Item Q242).

Of the following, the infants clinical presentation is MOST consistent with

A. group B streptococcal pneumonia

B. meconium aspiration syndrome

C. persistent pulmonary hypertension

D. retained fetal lung liquid syndrome

E. transposition of the great vessels

Copyright 2010 by the American Academy of Pediatrics page 772


2011 PREP SA on CD-ROM

Question: 242

(Courtesy of S Izatt)
Radiographic findings, as exhibited by the infant in the vignette.

Copyright 2010 by the American Academy of Pediatrics page 773


2011 PREP SA on CD-ROM

Critique: 242 Preferred Response: B

The history of post dates pregnancy with severe oligohydramnios, the clinical findings of
meconium staining at birth and early-onset respiratory distress, and the chest radiograph
demonstrating patchy opacification and hyperinflation (Item C242A) make the presentation of the
infant described in the vignette most consistent with meconium aspiration syndrome (MAS).
The passage of meconium into amniotic fluid prior to delivery occurs in approximately 13%
of live births and is seen more commonly in post dates infants. Normally, the flow of fetal lung
fluid is up and out of the lungs with in utero breathing. Aspiration of amniotic fluid occurs with
gasping inspiratory breaths that are associated with hypoxia or ischemia, as might be seen with
uteroplacental insufficiency. This suggests that meconium aspiration happens before delivery in
response to stress.
MAS causes lung injury through several mechanisms, including chemical pneumonitis due to
inflammation, mechanical obstruction of the airways, inactivation of surfactant, and
vasoconstriction of the pulmonary vessels. Persistent pulmonary hypertension (PPHN) often is
seen in association with MAS.
The infant who has MAS may appear meconium-stained at delivery and have a barrel chest
due to air trapping. Respiratory distress develops immediately or shortly after birth, with
grunting, flaring, and retractions as well as cyanosis. Auscultation of the lung fields may
demonstrate rales and rhonchi. The chest radiograph classically demonstrates patchy
opacification and hyperinflation, with complete opacification seen in extreme cases. A
pneumothorax or pneumomediastinum may be seen due to the increased risk of air leak from the
ball-valve effect of meconium.
Congenital cyanotic heart disease (eg, transposition of the great vessels) is less likely due
to this infants response to oxygen, although a hyperoxia test to document a Pao2 measurement
greater than 150 mm Hg would help to exclude cyanotic congenital heart disease. Although the
infant is at risk for PPHN, the similar pre- and postductal oxygen saturations do not demonstrate
shunting at the ductal level. The chest radiograph does not demonstrate diffuse parenchymal
infiltrates or fluid in the fissures (Item C242B), which would suggest retained fetal lung fluid
syndrome (previously known as transient tachypnea of the newborn). Underlying group B
streptococcal pneumonia cannot be excluded definitively, but the chest radiograph in this
condition may demonstrate patchy infiltrates and small pleural effusions.

Suggested reading:

Carbine DN, Serwint JR. In brief: meconium aspiration. Pediatr Rev. 2008;29:212-213. DOI:
10.1542/10.1542/pir.29-6-212. Available at:
http://pedsinreview.aappublications.org/cgi/content/full/29/6/212

Dargaville PA, Copnell B for the Australian and New Zealand Neonatal Network. The
epidemiology of meconium aspiration syndrome: incidence, risk factors, therapies, and outcome.
Pediatrics. 2006;117:1712-1721. DOI: 10.1542/peds.2005-2215. Available at:
http://pediatrics.aappublications.org/cgi/content/full/117/5/1712

Copyright 2010 by the American Academy of Pediatrics page 774


2011 PREP SA on CD-ROM

Flidel-Rimon O, Shinwell ES. Respiratory distress in the term and near-term infant. NeoReviews.
2005;6:e289-e297. Available at: http://neoreviews.aappublications.org/cgi/content/full/6/6/e289

Steinhorn RH, Farrow KN. Pulmonary hypertension in the neonate. NeoReviews. 2007;8:e14-
e21. Available at: http://neoreviews.aappublications.org/cgi/content/full/8/1/e14

Copyright 2010 by the American Academy of Pediatrics page 775


2011 PREP SA on CD-ROM

Critique: 242

(Courtesy of S Izatt)
Meconium aspiration: Diffuse patchy opacities, small bilateral pleural effusions, and mild
hyperinflation.

Copyright 2010 by the American Academy of Pediatrics page 776


2011 PREP SA on CD-ROM

Critique: 242

(Reprinted with permission from Aly H. Respiratory disorders in the newborn: identification and
diagnosis. Pediatr Rev. 2004;25:201-208)
Chest radiograph in retained fetal lung fluid syndrome: There are increased pulmonary interstitial
markings and fluid in the interlobar fissures (arrows).

Copyright 2010 by the American Academy of Pediatrics page 777


2011 PREP SA on CD-ROM

Question: 243

A 6-year-old boy complains of achy pain in his calf muscles about one night per week that is
relieved with massage and heat. He has had no fever, rash, fatigue, joint swelling, weight loss, or
other systemic symptoms. The pain is always better in the morning, and he remains very active.
He has had no unusual or compulsive leg movements associated with the pain. Findings on
physical examination, including thorough joint, muscle, and neurologic evaluation, are normal.

Of the following, the MOST appropriate next step in the care of this child is to

A. obtain a bone scan

B. obtain a complete blood count, erythrocyte sedimentation rate, and rheumatoid factor

C. prescribe calcium and vitamin D supplements

D. prescribe muscle stretching, analgesia, and warmth

E. refer him to an orthopedist

Copyright 2010 by the American Academy of Pediatrics page 778


2011 PREP SA on CD-ROM

Critique: 243 Preferred Response: D

Musculoskeletal pain is a frequent presenting complaint to primary care pediatricians,


initiating as many as 6% of visits. Causes for the pain vary considerably by age, with growing
pains being one of the most common diagnoses among children 3 to 12 years old. In a recent
Australian study, 37% of children ages 4 to 6 had symptoms of this condition.
Growing pains is a clinical diagnosis, with specific inclusion and exclusion criteria. Pain is
intermittent rather than persistent, is bilateral rather than unilateral, occurs during the evening or
night, and is absent in the morning. The usual location for the pain is in the muscles of the
anterior thighs, the calves, or the posterior knees; it does not involve joints. Physical examination
yields normal results without evidence of trauma, redness, swelling, heat, or decreased joint
range of motion. For the patient who meets these inclusion and exclusion criteria, such as the
boy described in the vignette, no laboratory or imaging studies are necessary. If such studies
are obtained, any abnormalities suggest another diagnosis.
Traditionally, growing pains have been treated by parents with warm packs and massage,
and physicians often suggest analgesics to help with pain relief, but little evidence addresses
any of these modalities. However, recent evidence demonstrated that muscle stretching,
particularly of the hamstrings and quadriceps, was beneficial. Another small study suggested
that heel wedges may help some children. Growing pains should be managed by families with
the assistance of their primary care physician; orthopedic consultation is needed only for
unusual cases that suggest the presence of another diagnosis. Calcium and vitamin D
supplementation is not necessary.
Studies of children fitting strict diagnostic criteria for restless leg syndrome (RLS) find that
many were diagnosed with growing pains before making the more accurate diagnosis of RLS.
The two conditions share many characteristics, particularly the presence of symptoms in the
evening and night. To prevent delay in diagnosing RLS, clinicians should inquire whether the pain
or unpleasant sensations are accompanied by a strong urge to move and discomfort is relieved
with movement.
Several causes for growing pains have been suggested, but the etiology remains unclear.
Theories have fallen into three general categories: anatomic abnormalities, fatigue, and
psychosocial factors. Recent studies have been unable to associate growing pains with any
anatomic changes in the feet, calling into question an anatomic cause. Studies have
demonstrated a lower pain threshold in children who have growing pains compared with
nonaffected children, suggesting that the condition is a generalized noninflammatory pain
syndrome of childhood. Another study demonstrated decreased tibial bone density in affected
children, raising the question of bone fatigue with activity. Further studies reveal that parents of
affected children note them to have a more intense or negative mood than unaffected children.

Suggested reading:

De Inocencio J. Epidemiology of musculoskeletal pain in primary care. Arch Dis Child.


2004;89:431-434. DOI: 10.1136/adc.2003.028860. Available at:
http://www.ncbi.nlm.nih.gov/pmc/articles/PMC1719931/?tool=pubmed

Copyright 2010 by the American Academy of Pediatrics page 779


2011 PREP SA on CD-ROM

Evans AM. Growing pains: contemporary knowledge and recommended practice. J Foot Ankle
Res. 2008;1:4. DOI: 10.1186/1757-1146-1-4. Available at: www.jfootankleres.com/content/1/1/4

Evans AM, Scutter SD. Prevalence of "growing pains" in young children. J Pediatr. 2004;145:255-
258. DOI: 10.1016/j.jpeds.2004.04.045. Available at:
http://www.jpeds.com/article/PIIS0022347604003622/fulltext

Picchietti D, Allen RP, Walters AS, Davidson JE, Myers A, Ferini-Strambi L. Restless legs
syndrome: prevalence and impact in children and adolescents the Peds REST study.
Pediatrics. 2007;120:253-266. DOI: 10.1542/peds.2006-2767. Available at:
http://pediatrics.aappublications.org/cgi/content/full/120/2/253

Uziel Y, Hashkes PJ. Growing pains in children. Pediatr Rheum. 2007;5:5. DOI: 10.1186/1546-
0096-5-5. Available at: http://www.ped-rheum.com/content/5/1/5

Copyright 2010 by the American Academy of Pediatrics page 780


2011 PREP SA on CD-ROM

Question: 244

A 3-year-old previously healthy boy presents with a 2-day history of a unilateral face and neck
swelling. The child has received all immunizations except hepatitis B and measles-mumps-rubella,
which were declined by the parents because they were concerned about the risk of autism. The
family recently added kittens and puppies to their home. Physical examination reveals diffuse soft-
tissue swelling that crosses the angle of the mandible (Item Q244) with a 2x2-cm tender lymph
node in the submandibular area and some boggy swelling in the preauricular area. The remainder
of the examination yields normal results.

Of the following, the MOST likely cause of the facial swelling and cervical lymphadenopathy in
this child is

A. atypical mycobacterial infection

B. bacterial parotitis

C. Bartonella henselae infection

D. dental abscess

E. viral parotitis

Copyright 2010 by the American Academy of Pediatrics page 781


2011 PREP SA on CD-ROM

Question: 244

(Courtesy of M Rimsza)
Swelling, as described for the boy in the vignette.

Copyright 2010 by the American Academy of Pediatrics page 782


2011 PREP SA on CD-ROM

Critique: 244 Preferred Response: E

Swelling of the face is a relatively common cause for presentation to the physicians office
or emergency department. The differential diagnosis for preauricular swelling includes parotitis,
lymphadenitis, tumor, and lymphosarcoma. Viral parotitis historically was caused by mumps
virus; parainfluenza, influenza, cytomegalovirus, Epstein-Barr virus, enteroviruses, and human
immunodeficiency virus (HIV) also are causative pathogens. Because of the low prevalence of
circulating mumps virus in communities with adequate immunization rates, infection with this
organism is unlikely. In addition, mumps is bilateral in 70% of cases and usually has a prodrome
of fever and malaise. Bacterial parotitis frequently is caused by Staphylococcus aureus, and
patients usually appear ill and may have purulent discharge noted at the Stensen duct. Swelling
of the parotid gland generally crosses the angle of the mandible, a finding that may assist in
differentiating it from preauricular lymphadenitis. Preauricular lymphadenitis is a common finding
in adenoviral infection and often is accompanied by conjunctivitis. It also may occur with otitis
externa or buccal cellulitis.
Rarer causes of preauricular swelling include lymphomas such as Hodgkin disease, non-
Hodgkin lymphoma, and sarcomas. These conditions are marked by gradual increase in size of
the swelling and are usually subacute or chronic rather than acute, as in inflammations of the
parotid gland and preauricular lymph nodes. However, the clinician should be aware that the
consistency of the involved lymph nodes in lymphoma often is "rubbery" and that mediastinal
involvment may obstruct venous return, leading to generalized facial swelling.
The lack of systemic signs for the boy in the vignette excludes a dental abscess. Although
atypical mycobacteria and Bartonella henselae may produce suppurative lymphadenitis, the
more likely cause for the unimmunized boy is mumps. HIV-related parotitis generally is
characterized by a history of risk factors such as exposure perinatally or through transfusion in
a child of this age and often accompanies primary HIV infection.

Suggested reading:

American Academy of Pediatrics. Mumps. In: Pickering LK, Baker CJ, Kimberlin DW, Long SS,
eds. Red Book: 2009 Report of the Committee on Infectious Diseases. 28th ed. Elk Grove
Village, Ill: American Academy of Pediatrics; 2009:468-472

Friedmann AM. Evaluation and management of lymphadenopathy in children. Pediatr Rev.


2008;29:53-60. DOI: 10.1542/10.1542/pir.29-2-53. Available at:
http://pedsinreview.aappublications.org/cgi/content/full/29/2/53

Oral Health Initiative. Parotitis. In: Protecting All Children's Teeth (PACT): A Pediatric Oral
Health Training Program. Elk Grove Village, Ill: American Academy of Pediatrics. Available at:
http://www.aap.org/commpeds/dochs/oralhealth/pact/ch11_sect1d.cfm

Peters TR, Edwards KM. Cervical lymphadenopathy and adenitis. Pediatr Rev. 2000;21:399-405.
DOI: 10.1542/10.1542/pir.21-12-3. Available at:
http://pedsinreview.aappublications.org/cgi/content/full/21/12/399

Copyright 2010 by the American Academy of Pediatrics page 783


2011 PREP SA on CD-ROM

Velez MC. Consultation with the specialist: lymphomas. Pediatr Rev. 2003;24:380-386. DOI:
10.1542/10.1542/pir.24-11-380. Available at:
http://pedsinreview.aappublications.org/cgi/content/full/24/11/380

Copyright 2010 by the American Academy of Pediatrics page 784


2011 PREP SA on CD-ROM

Question: 245

A 9-year-old girl presents to the emergency department because of fever, a macular rash on her
trunk and arms, and pain in her knees and ankles. Her mother explains that she recently had
pharyngitis. You note a murmur on physical examination and order electrocardiography (Item
Q245) and echocardiography. The echocardiogram demonstrates mitral regurgitation.

Of the following, the MOST likely diagnosis for this girl is

A. acute rheumatic fever

B. cat-scratch disease

C. Epstein-Barr viral infection

D. Lyme disease

E. systemic lupus erythematosus

Copyright 2010 by the American Academy of Pediatrics page 785


2011 PREP SA on CD-ROM

Question: 245

(Courtesy of M Lewin)

Copyright 2010 by the American Academy of Pediatrics page 786


2011 PREP SA on CD-ROM

Critique: 245 Preferred Response: A

The girl described in the vignette has three of the five major Jones criteria for the diagnosis
of acute rheumatic fever. The five major criteria are migratory arthritis, subcutaneous nodules,
acute carditis, erythema marginatum, and Sydenham chorea. One mnemonic for the five major
criteria is JONES:

J=joints, signifying migratory polyarthritis


O=obvious reason for diagnosing and treating these patients, which is the potential for
cardiac involvement. Carditis may manifest as valve dysfunction, myocarditis,
pericarditis, and valvulitis
N=nodules, which are subcutaneous, usually pea-sized, or found over the occiput or over
extensor surfaces of the elbows and knees
E=erythema marginatum, a serpentine, slightly raised red or pink rash typically seen on the
trunk and extremities
S=Sydenham chorea

Carditis can lead to chronic cardiac conditions of mitral (and occasionally aortic)
regurgitation and stenosis. Migratory arthritis does not result in permanent joint damage. The
movement disorder associated with Sydenham chorea eventually remits.
The diagnosis of acute rheumatic fever is established when two major criteria are present
and there is evidence of a preexisting group A hemolytic streptococcal infection. The best
evidence of streptococcal infection is elevation of antistreptococcal titers, such as the
antistreptolysin titer or antiDNAase B titer. If two major criteria cannot be demonstrated, the
diagnosis still can be established by the presence of one major criterion and any two of the
following minor criteria: fever, elevated acute-phase reactants (erythrocyte sedimentation rate
or C-reactive protein), and prolonged PR interval on electrocardiography (Item C245).
Oral prednisone is useful in treating severe acute rheumatic carditis, particularly when the
carditis is accompanied by ventricular dysfunction leading to congestive heart failure. Whether
corticosteroid therapy changes the risk for long-term valve pathology is unclear. Similarly, high-
dose salicylate therapy can shorten the duration of acute symptoms, but there are no data that
such treatment reduces the risk of serious lifelong rheumatic heart disease. The most important
chronic therapeutic intervention for the patient who has acute rheumatic fever is antibiotic
prophylaxis to prevent group A streptococcal pharyngitis, which can lead to a recurrent bout of
acute rheumatic fever. Serious valve damage is unusual after the initial attack of acute rheumatic
fever but becomes more likely with subsequent attacks. Administration of intramuscular
benzathine penicillin every 21 to 28 days affords better protection against streptococcal
infection than an oral regimen, likely due to better compliance. Rheumatic fever prophylaxis must
continue into adulthood.
Cat-scratch disease usually is a benign infectious disease caused by the intracellular
bacterium Bartonella. It is found most commonly in children 1 to 2 weeks following a cat scratch
or bite. Common findings are lymphadenopathy, headache, chills, and abdominal pain. The
disease usually resolves spontaneously, with or without treatment, in 1 month. Cardiac valve
involvement and electrocardiographic changes are rare.

Copyright 2010 by the American Academy of Pediatrics page 787


2011 PREP SA on CD-ROM

Epstein-Barr viral infection can cause infectious mononucleosis, which develops when a
person initially is exposed to the virus during or after adolescence. Symptoms of infectious
mononucleosis are fever, pharyngitis, hepatitis, and lymphadenopathy. Usually, laboratory tests
are needed for confirmation. Serologic results include an elevated white blood cell count, an
increased percentage of certain atypical white blood cells, and a positive reaction to a "mono
spot" test. Pericarditis and myocarditis are infrequent findings. The young age of the patient in
the vignette is highly unusual for symptomatic Epstein-Barr virus infection, as are the
electrocardiographic changes, rash, and arthritis.
Lyme disease results in fever, rash, malaise, and muscle soreness. Although neurologic
disorders and arthritis can occur, these are invariably late findings. In addition, the morphology
of the rash distinguishes this condition from acute rheumatic fever, and the Jones criteria are not
fulfilled. Carditis is a rare manifestation of Lyme disease. The most common abnormality is
atrioventricular block of various degrees, although other rhythm abnormalities have been
reported. Pericarditis, myocarditis, cardiomyopathy, and degenerative valvular disease have
been associated with Borrelia burgdorferi infection.
Systemic lupus erythematosus (SLE) can present with rash, carditis, and arthritis. However,
evidence of a preexisting group A hemolytic streptococcal infection would not be expected. Of
course, this laboratory finding could be present in a patient who has SLE, but these two
diagnoses can be distinguished by the additional laboratory findings in SLE, including antinuclear
antibody, anti-double-stranded DNA antibodies, and low complement values.

Suggested reading:

Cilliers AM. Rheumatic fever and its management. BMJ. 2006;333:1153-1156. DOI:
10.1136/bmj.39031.420637.BE. Available at:
http://www.ncbi.nlm.nih.gov/pmc/articles/PMC1676147/?tool=pubmed

Dajani A, Taubert K, Ferrieri P, Peter G, Shulman S, Committee on Rheumatic Fever, Endocarditis,


and Kawasaki Disease of the Council on Cardiovascular Disease in the Young, the American
Heart Association. Treatment of acute streptococcal pharyngitis and prevention of rheumatic
fever: a statement for health professionals. Pediatrics. 1995;96:758-764. Available at:
http://pediatrics.aappublications.org/cgi/content/abstract/96/4/758

Steer AC, Carapetis JR. Acute rheumatic fever and rheumatic heart disease in indigenous
populations. Pediatr Clin North Am. 2009;56:1401-1419. DOI: 10.1016/j.pcl.2009.09.011.
Abstract available at: http://www.ncbi.nlm.nih.gov/pubmed/19962028

Copyright 2010 by the American Academy of Pediatrics page 788


2011 PREP SA on CD-ROM

Critique: 245

(Courtesy of M Lewin)
Prolonged PR interval, as described for the girl in the vignette.

Copyright 2010 by the American Academy of Pediatrics page 789


2011 PREP SA on CD-ROM

Question: 246

The parents of a 9-year-old girl bring her to the office because of 2 days of increasing
restlessness and clumsiness. She previously was healthy but now has irregular, continuous
involuntary movements that are present at rest, worsen with purposeful action, and disappear
during sleep.

Of the following, the diagnostic test that is MOST likely to confirm the diagnosis is

A. anti-double-stranded DNA test

B. antiphospholipid antibody test

C. antistreptolysin O antibody test

D. erythrocyte sedimentation rate

E. throat swab for rapid streptococcal antigen

Copyright 2010 by the American Academy of Pediatrics page 790


2011 PREP SA on CD-ROM

Critique: 246 Preferred Response: C

Chorea is an involuntary, hyperkinetic movement disorder characterized by continuous, jerky


or flowing, unpredictable movement fragments. The direction and timing of movements appear
random, and symptoms generally worsen with action versus rest. The girl described in the
vignette has become gradually restless and clumsy over several days, typical of a child who
has acquired chorea. Most school-age children who present with acute or subacute chorea
have immune-mediated, poststreptococcal chorea, also known as Sydenham chorea, that is
acquired weeks to months after infection with group A beta-hemolytic streptococci (GABHS)
(Item C246). The differential diagnosis includes systemic lupus erythematosus, antiphospholipid
antibody syndrome, hyperthyroidism, drug-induced movement disorders, metabolic diseases
affecting the basal ganglia, and psychogenic movement disorders.
Chorea can cause significant motor impairment, and specialty consultation is advised. In
most cases presenting like the girl in the vignette, blood tests for antistreptolysin O, anti-DNAse
B, and antihyaluronidase antibody titers can confirm the diagnosis. A throat swab for GABHS
should be obtained, although it yields normal results in most cases, making it nondiagnostic.
Other autoimmune causes are less common, but evaluation for other autoimmune diseases with
erythrocyte sedimentation rate, antiphospholipid antibody panel, and anti-double-stranded DNA
should be considered in cases where prior exposure to GABHS is not confirmed.

Suggested reading:

Cardoso F, Seppi K, Mair KJ, Wenning GK, Poewe W. Seminar on choreas. Lancet Neurol.
2006;5:589-602. DOI: 10.1016/S1474-4422(06)70494-X. Abstract available at:
http://www.ncbi.nlm.nih.gov/pubmed/16781989

Gilbert DL. Acute and chronic chorea in childhood. Semin Pediatr Neurol. 2009;16:71-76. DOI:
10.1016/j.spen.2009.03.009. Abstract available at:
http://www.ncbi.nlm.nih.gov/pubmed/19501334

Wild EJ, Tabrizi SJ. The differential diagnosis of chorea. Pract Neurol. 2007;7:360-373. Abstract
available at: http://www.ncbi.nlm.nih.gov/pubmed/18024776

Zomorrodi A, Wald ER. Sydenham's chorea in western Pennsylvania. Pediatrics. 2006;117:e675-


e679. DOI: 10.1542/peds.2005-1573. Available at:
http://pediatrics.aappublications.org/cgi/content/full/117/4/e675

Copyright 2010 by the American Academy of Pediatrics page 791


2011 PREP SA on CD-ROM

Question: 247

The parents of a 12-year-old boy bring him to the clinic because they are concerned about his
lack of interest in socializing and physical activity. He was held back in school this year due to
learning problems, primarily in reading and writing. He has no close friends, and his favorite
activity is watching television. On physical examination, the pleasant and attractive youngster
converses with you easily. His weight is at the 50th percentile and his height at the 95th
percentile. He wears thick glasses due to high myopia. He has mild scoliosis in the thoracic
spine. He shows no signs of puberty. Using a Woods lamp, you identify five caf au lait macules
on his trunk and extremities.

Of the following, the MOST likely diagnosis for this boy is

A. Asperger syndrome

B. Klinefelter syndrome

C. Marfan syndrome

D. neurofibromatosis

E. Stickler syndrome

Copyright 2010 by the American Academy of Pediatrics page 792


2011 PREP SA on CD-ROM

Critique: 247 Preferred Response: B

The combination of poor social interaction with same-age peers, a tendency to be a loner,
difficulties with reading and writing, and tall stature described for the boy in the vignette should
raise suspicion for the diagnosis of Klinefelter syndrome (KS). KS typically is associated with a
47,XXY karyotype, although the term applies to boys who have at least one extra X
chromosome or a mosaic chromosome complement with at least one cell line having more than
one X.
KS is relatively common, with a birth incidence of 1.2 per 1,000 males. There are three times
in the lifespan when KS is likely to be diagnosed: 1) prenatally, on amniocentesis performed due
to advanced maternal age; 2) in adolescence, due primarily to gynecomastia (present in one third
of affected boys); and 3) in adulthood, due to infertility.
Boys who have KS typically do not have dysmorphisms. They are tall compared with their
chromosomally normal siblings, and they often have mild skeletal differences, such as an
increased carrying angle or gynecoid pelvis. Scoliosis is common in boys who have KS. Puberty
usually is delayed, and after puberty, affected boys may have reduced facial or body hair,
gynecomastia, and small testes. Obesity and poor muscular development may be evident.
Elevations in luteinizing hormone and follicle-stimulating hormone are associated with reduced
testicular function and lack of feedback inhibition by testosterone. In adulthood, individuals who
have KS are at increased risk for Leydig cell neoplasia, and a history of gynecomastia is
associated with a 20- to 50-fold increased risk for breast cancer compared with the general
male population.
From developmental and behavioral standpoints, there is wide variation among boys who
have KS, but common problems include delays in motor skills, speech and language, and
reading. A commonly described feature is the "the tip of the tongue" phenomenon, wherein the
boys are unable to retrieve the correct phrase or response to a question. The boys tend to be
immature, to adapt poorly to unfamiliar situations, and possibly to be inappropriately aggressive.
Inattentiveness may be a problem, and social skills are often lacking, although the boys may
relate better to adults than to their peers. Intelligence quotient (IQ) is usually within the normal
range, although the average IQ is typically 10 to 20 points less than in otherwise healthy siblings.
Asperger syndrome (AS) is a multifactorial autism spectrum disorder associated with
deficits in social interaction that involves relative preservation of language and cognitive
function. AS has an estimated incidence of 1 in 5,000. Affected individuals are not typically tall,
and they have an unusual conversational style, sometimes speaking in a monotone, showing
few facial movements, or displaying restricted body movements.
Marfan syndrome is an autosomal dominant connective tissue disorder associated with tall
stature and skeletal abnormalities, including arachnodactyly, pectus deformity, and scoliosis as
well as dilation of the aortic root and subluxed lenses of the eye. Thus far, an association with
learning problems is not documented. Socialization is average for age.
Stickler syndrome is an autosomal dominant connective tissue disorder that may have
skeletal features overlapping with Marfan syndrome in childhood. Cardinal features are
prominent eyes and malar hypoplasia as well as rapidly progressive high myopia. Scoliosis is a
complication of the disorder. Learning problems are not part of the picture, and socialization is
normal.

Copyright 2010 by the American Academy of Pediatrics page 793


2011 PREP SA on CD-ROM

Neurofibromatosis (NF) type 1 is an autosomal dominant neurocutaneous syndrome that has


an incidence of 1 in 3,500. Caf au lait macules (CALMs) typically are the first sign to appear,
and six or more CALMs measuring at least 5 mm in diameter in a prepubertal child should raise
suspicion for this diagnosis. Individuals who have NF1 are at increased risk for intellectual and
learning disabilities. NF1 is not associated with tall stature, and socialization typically is normal.

Suggested reading:

Francomano C, Wilkin DJ, Liberfarb RM. Stickler syndrome. In: Cassidy SB, Allanson JE, eds.
Management of Genetic Syndromes. 2nd ed. Hoboken, NJ: John Wiley & Sons; 2005:539-546

Jones KL. Marfan syndrome. In: Smiths Recognizable Patterns of Human Malformation. 6th ed.
Philadelphia, Pa: Elsevier Saunders; 2006:546-551

Jones KL. Neurofibromatosis syndrome. In: Smiths Recognizable Patterns of Human


Malformation. 6th ed. Philadelphia, Pa: Elsevier Saunders; 2006:590-593

Jones KL. Stickler syndrome. In: Smiths Recognizable Patterns of Human Malformation. 6th ed.
Philadelphia, Pa: Elsevier Saunders; 2006:318-321

Jones KL. XXY syndrome, Klinefelter syndrome. In: Smiths Recognizable Patterns of Human
Malformation. 6th ed. Philadelphia, Pa: Elsevier Saunders; 2006:68-69

Miles JH, McCathren RB. Autism overview. GeneReviews. 2005. Available at:
http://www.ncbi.nlm.nih.gov/bookshelf/br.fcgi?book=gene&part=autism-overview

Schrijver I, Alcorn DM, Francke U. Marfan syndrome. In: Cassidy SB, Allanson JE, eds.
Management of Genetic Syndromes. 2nd ed. Hoboken, NJ: John Wiley & Sons; 2005:335-349

Simpson JL, Graham JM Jr, Samango Sprouse C, Swerdloff R. Klinefelter syndrome. In: Cassidy
SB, Allanson JE, eds. Management of Genetic Syndromes. 2nd ed. Hoboken, NJ: John Wiley &
Sons; 2005:323-333

Viskochil D. Neurofibromatosis type 1. In: Cassidy SB, Allanson JE, eds. Management of Genetic
Syndromes. 2nd ed. Hoboken, NJ: John Wiley & Sons; 2005:369-384

Wattendorf DJ, Muenke M. Klinefelter syndrome. Am Fam Physician. 2005;72:2259-2262.


Available at: http://www.ncbi.nlm.nih.gov/pubmed/16342850

Copyright 2010 by the American Academy of Pediatrics page 794


2011 PREP SA on CD-ROM

Question: 248

A 16-year-old sexually active girl requests a contraceptive method. Other than being a light
smoker, she has no findings of note on her past medical history and physical examination. Her
grandparents are obese and have hypertension and diabetes. She asks about the dangers of
using oral contraceptives.

Of the following, the MOST likely fatal adverse effect of combined oral contraceptives for this girl
is due to the development of

A. breast cancer

B. cholestatic jaundice

C. diabetes

D. hypertension

E. venous thrombosis

Copyright 2010 by the American Academy of Pediatrics page 795


2011 PREP SA on CD-ROM

Critique: 248 Preferred Response: E

Available contraceptive options include barrier and hormonal methods. Other than latex
allergy, condoms have no health risks. Adverse effects of combined (estrogen and
progesterone) oral contraceptives (COCs) include the rare but serious adverse reactions and
the more common but not life-threatening effects. The risk of death is estimated at 1 per 200,000
nonsmoking users younger than age 35 years. Serious acute adverse effects include those
caused by blood clots, which are described in the mnemonic ACHES: abdominal and chest pain,
severe headaches, eye problems (visual changes), and swelling or aching pain in calves/legs.
Myocardial infarction and strokes involve arterial thrombosis, the risk for which increases
substantially over time among those who smoke. Venous thrombosis, on the other hand, is not
affected by smoking but by prothrombotic genetic defects. Such risks, although rare, can be
minimized by screening for a personal or family history of thrombosis. If positive, the patient
should have protein C, protein S, and antithrombin III concentrations evaluated. Women who
have migraine headaches associated with focal signs should avoid using COCs.
Pills containing lower doses of estrogen combined with the newer generation of
progestogens cause no significant changes in blood pressure, carbohydrate and lipid
metabolism, gallbladder disease, or cholestatic jaundice. In those predisposed to gallbladder
disease, the development of stones may be accelerated with the newer lower-dose pills. COCs
protect against benign breast conditions, but the risk of breast cancer with use of these agents
is unclear. Because studies have conflicting results, there is no recommendation to withhold
COCs in women who have family histories of breast cancer. The link between COCs and
hepatocellular adenoma is clear, but the incidence is low.
Minor estrogenic adverse effects of COCs include nausea, dizziness, irritability, weight gain,
and bloating; progestogenic adverse effects include acne, hirsutism, weight gain, loss of libido,
and depression. Such effects can appear in up to 50% of women but generally disappear within
a few months of pill use. Menstrual cycles become lighter and shorter with COC use, and
intermenstrual bleeding may occur.
The transdermal patch and the ring are newer delivery forms of combined hormonal
contraception. The patch can cause local skin reactions at the application site. In addition, users
are more likely to have breast tenderness, vaginal spotting, and dysmenorrhea in the first two
cycles than users of COCs. Ring users have infrequent hormone-related adverse effects.
Fewer than 6% of users complain of vaginitis and vaginal discomfort.
Progesterone-only methods of contraception include the mini-pill, depot
medroxyprogesterone acetate (DMPA) injections, an implantable device, and the levonorgestrel
intrauterine system. Menstrual irregularity, weight gain, depression, and breast tenderness are
the more commonly noted adverse effects of these contraceptive methods. DMPA is more likely
to cause decreased bone density, which reverses upon discontinuation of this method.

Suggested reading:

Cavanaugh RM Jr. Screening adolescent gynecology in the pediatrician's office: have a listen,
take a look. Pediatr Rev. 2007;28:332342. DOI: 10.1542/10.1542/pir.28-9-332. Available at:
http://pedsinreview.aappublications.org/cgi/content/full/28/9/332

Copyright 2010 by the American Academy of Pediatrics page 796


2011 PREP SA on CD-ROM

Gupta N, Corrado S, Goldstein M. Hormonal contraception for the adolescent. Pediatr Rev.
2008;29:386-397. DOI: 10.1542/10.1542/pir.29-11-386. Available at:
http://pedsinreview.aappublications.org/cgi/content/full/29/11/386

Speroff L, Fritz MA. Contraception. In: Clinical Gynecologic Endocrinology and Infertility. 7th ed.
Philadelphia, Pa: Lippincott Williams & Wilkins; 2005:Part III

Trussell J. Contraceptive efficacy. In: Hatcher RA, Trussell J, Nelson AL, Cates W Jr, Stewart FH,
eds. Contraceptive Technology. 19th ed. New York, NY: Ardent Media, Inc; 2007:747-756

Copyright 2010 by the American Academy of Pediatrics page 797


2011 PREP SA on CD-ROM

Question: 249

You are evaluating an 8-year-old girl in the emergency department who was brought in by her
parents after she was bitten by a squirrel that she was chasing at a family picnic. Her parents
ask you if it is likely that their daughter was exposed to rabies.

Of the following, the animals that are MOST associated with human rabies infection in the United
States are

A. bats

B. mice

C. rabbits

D. skunks

E. squirrels

Copyright 2010 by the American Academy of Pediatrics page 798


2011 PREP SA on CD-ROM

Critique: 249 Preferred Response: A

The incidence of rabies in the United States has been steadily decreasing since the 1950s,
with 25 cases reported between 2000 and 2007, 20 of which were acquired indigenously.
Wildlife rabies persists throughout the United States, except in Hawaii, which remains rabies-
free. Wildlife, including bats, raccoons, skunks, foxes, and coyotes, are the most important
potential sources of infection for humans and domestic animals in the United States. Of the 20
indigenously acquired cases, 17 were associated with bat rabies virus variants, although only
one was directly associated with a bat bite.
Rabies in small rodents such as squirrels, rats, and mice as well as lagomorphs (hares and
rabbits) is rare. Worldwide, most rabies cases in humans result from dog bites. No case of
human rabies in the United States has been attributed to a dog, cat, or ferret that has remained
healthy throughout the standard 10-day period of confinement. Although rabies virus has been
demonstrated in human saliva, no documented case due to a human bite has been reported in
the United States.
Rabies is almost always fatal once patients exhibit symptoms. Therefore, control measures
such as educating children to stay away from stray or wild animals and postexposure
prophylaxis with rabies vaccine and rabies immune globulin are critical. The decision to immunize
a potentially exposed person should be made in consultation with the local health department
and in accordance with the guidelines in Item C249.

Suggested reading:

American Academy of Pediatrics. Rabies. In: Pickering LK, Baker CJ, Kimberlin DW, Long SS,
eds. Red Book: 2009 Report of the Committee on Infectious Diseases. 28th ed. Elk Grove
Village, Ill: American Academy of Pediatrics; 2009;552-559

Toltzis. Rabies. In: Kliegman RM, Behrman RE, Jenson HB, Stanton BF, eds. Nelson Textbook of
Pediatrics. 18th ed. Philadelphia, Pa: Saunders Elsevier; 2007:1423-1425

Copyright 2010 by the American Academy of Pediatrics page 799


2011 PREP SA on CD-ROM

Critique: 249

Copyright 2010 by the American Academy of Pediatrics page 800


2011 PREP SA on CD-ROM

Question: 250

A new mother is concerned that her 6-week-old infant is always crying. The infant has gained
weight appropriately and is happy and alert in your office. You reassure her that infants
communicate by crying to have their needs met, but the mother would like more information about
infant crying.

Of the following, the information that you are MOST likely to provide is that

A. changing to a hypoallergenic formula can decrease the crying

B. constantly picking up a crying baby increases the frequency of crying

C. crying often is greater in the morning hours

D. crying tends to peak at 6 weeks of age

E. healthy newborns may cry 6 hours a day

Copyright 2010 by the American Academy of Pediatrics page 801


2011 PREP SA on CD-ROM

Critique: 250 Preferred Response: D

The infant described in the vignette is 6 weeks old, the age at which crying tends to peak.
As noted in a 1962 study by Brazelton, infants may cry up to 3 hours daily at 6 weeks of age;
crying decreases to 1 hour by 12 weeks. Crying is most common between the hours of 3 pm
and 11 pm.
Crying is a normal aspect of psychomotor development in early infancy. It occurs in
response to stresses such as hunger, discomfort, too much or too little stimulation, and
temperature change. Constantly picking up a crying 6-week-old infant will not increase the
frequency of crying. It is not normal for a baby to cry for 6 hours, and this may be a nonspecific
symptom of a medical problem such as an infectious disease or nutritional or metabolic condition.
Crying alone is not an indication to change the infants formula to a hypoallergenic variety.
Children who have allergies to proteins in cows milk generally have small-volume intestinal
bleeding and possibly diarrhea.

Suggested reading:

Bolte R. The crying child: what are they trying to tell you? Part I. Contemp Pediatr. 2007;24(5)74-
81

Carey WB "Colic": prolonged or excessive crying in young infants. In: Levine MD, Carey WB,
Crocker AC, eds. Developmental-Behavioral Pediatrics. 3rd ed. Philadelphia, Pa: WB Saunders
Company; 1999:365-369

Copyright 2010 by the American Academy of Pediatrics page 802


2011 PREP SA on CD-ROM

Question: 251

A 16-year-old patient asks for assistance in preparing a report she is doing on newly described
infectious diseases. She asks you about the severe acute respiratory syndrome (SARS)
outbreak of 2002 through 2004.

Of the following, a CORRECT observation about SARS is that

A. airborne spread is believed to be the primary route of transmission

B. children developed the most severe illnesses during the outbreak

C. mortality from SARS most often was caused by secondary bacterial pneumonia

D. pigs are believed to be the primary reservoir of the SARS coronavirus

E. treatment with ribavirin and corticosteroids is beneficial

Copyright 2010 by the American Academy of Pediatrics page 803


2011 PREP SA on CD-ROM

Critique: 251 Preferred Response: A

Severe acute respiratory syndrome (SARS) burst upon the world scene in late 2002, early
2003. The initial cases appeared in the Guandong province of China and spread rapidly to Hong
Kong and other areas of southeast Asia. Through air travel, the disease subsequently spread
throughout the world, with a significant outbreak in Toronto, Canada. A number of cases also
were reported in the United States. Clinically, SARS is defined as a febrile illness with cough or
difficulty breathing and close contact with a case of SARS or travel or residence in an area of
local transmission of SARS within 10 days of symptom onset. Studies of affected patients led to
the isolation of a new coronavirus (SARS-coronavirus).
Studies of clusters of cases in Hong Kong and Canada suggested that droplet and airborne
spread were the primary routes of transmission of the SARS-coronavirus. Most cases occurred
in adults, with children younger than 15 years of age accounting for a small percentage of
cases. Young children, if infected, developed milder disease than adults. Histologic studies and
cultures from cases did not reveal evidence of bacterial superinfection.
The exact origin of the SARS-coronavirus remains uncertain, but it is related most closely to
a strain found in civets in rural China. Treatment with corticosteroids and the antiviral drug
ribavirin were tried during the SARS outbreak, but there was no evidence that either
monotherapy or a combination of these therapies was clinically beneficial. New cases of SARS
have not been reported since 2004.

Suggested reading:

American Academy of Pediatrics. Coronaviruses, including SARS. In: Pickering LK, Baker CJ,
Kimberlin DW, Long SS, eds. Red Book: 2009 Report of the Committee on Infectious Diseases.
28th ed. Elk Grove Village, Ill: American Academy of Pediatrics; 2009:269-270

Centers for Disease Control and Prevention. Severe Acute Respiratory Syndrome (SARS). 2005.
Available at: http://www.cdc.gov/ncidod/sars/

Stockman LJ, Massoudi MS, Helfand R, et al. Severe acute respiratory syndrome in children.
Pediatr Infect Dis J. 2007;26:68-74. DOI: 10.1097/01.inf.0000247136.28950.41. Abstract
available at: http://www.ncbi.nlm.nih.gov/pubmed/17195709

Copyright 2010 by the American Academy of Pediatrics page 804


2011 PREP SA on CD-ROM

Question: 252

A 2-year-old boy who has been limping and has a temperature of 39.0C presents with a swollen
and warm left knee that is exquisitely tender with any movement. No other findings are of note on
his physical examination. Magnetic resonance imaging of the knee reveals a large joint effusion
but no bony or soft-tissue involvement. Arthrotomy is performed for irrigation and debridement of
the knee joint. The joint fluid contains 100.0x103/mcL (100.0x109/L) white blood cells, with 80%
polymorphonuclear leukocytes and 20% lymphocytes, but the culture is negative after 3 days.
The patient is not improving with intravenous vancomycin therapy.

Of the following, the next BEST step is to

A. add linezolid to the therapy

B. add trimethoprim to the therapy

C. change the therapy to cefotaxime

D. change the therapy to clindamycin

E. continue vancomycin therapy

Copyright 2010 by the American Academy of Pediatrics page 805


2011 PREP SA on CD-ROM

Critique: 252 Preferred Response: C

The child described in the vignette has pyogenic arthritis of the knee, most likely due to
Kingella kingae. Kingella is an increasingly recognized cause of osteoarticular infections in
children younger than 5 years of age, but it is most common in those younger than 2 years. The
distal femur is the most commonly affected bone (Item C252), and the knee, hip, and ankle are
the most commonly affected joints. The clinical presentation often is subacute. The organism
also is an uncommon cause of diskitis, bacteremia, and endocarditis in those who have
underlying cardiac disease, meningitis, and pneumonia.
Kingella is a gram-negative coccobacillus that is difficult to isolate on typical solid culture
media. For beta-lactamase-negative strains, penicillin is the treatment of choice. Third-generation
cephalosporins are very effective in treating the organism, so for this boy, changing the therapy
to cefotaxime would be appropriate. Kingella is resistant to vancomycin, linezolid, and
trimethoprim, and resistance to clindamycin and erythromycin can occur. It generally is
susceptible to gentamicin, chloramphenicol, tetracycline, ciprofloxacin, and trimethoprim-
sulfamethoxazole.
The diagnosis of Kingella requires culture of the organism from bone, synovial fluid, blood,
or other sites of focal disease. The bacteria grow aerobically under enhanced carbon dioxide
conditions. Because Kingella usually fails to grow on routinely used solid media, bone aspirates
and synovial fluid should be inoculated into a blood culture bottle and incubated for at least 7
days.

Suggested reading:

American Academy of Pediatrics. Kingella kingae infections. In: Pickering LK, Baker CJ,
Kimberlin DW, Long SS, eds. Red Book: 2009 Report of the Committee on Infectious Diseases.
28th ed. Elk Grove Village, Ill: American Academy of Pediatrics; 2009:418-419

Kiang KM, Ogunmodede F, Juni BA, et al. Outbreak of osteomyelitis/septic arthritis caused by
Kingella kingae among child care center attendees. Pediatrics. 2005;116:e206-e213. DOI:
10.1542/peds.2004-2051. Available at:
http://pediatrics.aappublications.org/cgi/content/full/116/2/e206

Kuhls TL. Kingella species. In: Feigin RD, Cherry JD, Demmler-Harrison GJ, Kaplan SL, eds.
Feigin & Cherrys Textbook of Pediatric Infectious Diseases. 6th ed. Philadelphia, PA: Saunders
Elsevier; 2009:1773-1776

Copyright 2010 by the American Academy of Pediatrics page 806


2011 PREP SA on CD-ROM

Critique: 252

(Courtesy of D Palazzi)
Magnetic resonance imaging with contrast showing a moderate-sized left knee joint effusion with
abnormally increased synovial enhancement and synovial thickening in a 2 year-old child who
has septic arthritis.

Copyright 2010 by the American Academy of Pediatrics page 807


2011 PREP SA on CD-ROM

Question: 253

The mother of a 6-year-old girl reports during a health supervision visit that her daughter has
nighttime wetting and occasional daytime accidents with urgency. She has no history of
constipation, and no one else in the family has suffered enuresis. Her urinalysis reveals:

Specific gravity, 1.020


pH, 7
2+ blood
Trace protein
Positive for nitrites
3+ leukocyte esterase
5 to 10 red blood cells/high-power field (hpf)
20 to 50 white blood cells/hpf

Of the following, the BEST next diagnostic test to perform for this patient is

A. cystoscopy

B. DMSA (technetium dimercaptosuccinic acid) renal scan

C. magnetic resonance imaging of the lumbosacral spine

D. renal/bladder ultrasonography

E. spiral computed tomography scan of the abdomen

Copyright 2010 by the American Academy of Pediatrics page 808


2011 PREP SA on CD-ROM

Critique: 253 Preferred Response: D

The child described in the vignette exhibits nonmonosymptomatic enuresis, which is defined
as nocturnal incontinence plus daytime voiding symptoms. As is recommended for all patients
who have enuresis, she underwent a urinalysis, which revealed marked pyuria with mild
hematuria and possible bacteruria (nitrite-positive). These findings are strongly suggestive of a
urinary tract infection (UTI). Further evaluation, in addition to a confirmatory urine culture, should
include renal/bladder ultrasonography. This study can help screen for hydronephrosis and renal
stones, which could increase the risk for a UTI. Cystoscopy has a limited role in pediatrics for
enuresis or hematuria; it is most useful to evaluate urinary obstruction and occasionally in the
evaluation of gross hematuria without an underlying cause. DMSA (technetium
dimercaptosuccinic acid) scan is most useful as a follow-up study to evaluate for renal scarring
in a child who has a history of pyelonephritis. Magnetic resonance imaging of the lumbosacral
spine is a useful diagnostic test to evaluate for a possible tethered cord in a child who has an
acquired neurogenic bladder. This is usually a second-line study after initial ultrasonography to
screen for hydronephrosis. Spiral computed tomography (CT) scan of the abdomen is indicated
for a child in whom a renal stone is suspected but who has negative findings on
ultrasonography. The risks of radiation exposure with a CT scan always should be considered
by the clinician, who needs to weigh the risks and the benefits of performing the study.
The search for an underlying organic cause of enuresis should be considered for all
patients, but it is suggested by the history. Pertinent portions of the patient history include
dysuria, frequency, and urgency (UTI); polyuria, nocturia, polydipsia, and nighttime thirst
(diabetes mellitus and urinary concentrating defects such as diabetes insipidus and renal
dysplasia); constipation (dysfunctional voiding); nighttime snoring (sleep apnea due to
adenotonsillar hypertrophy); and new-onset constipation or gait disturbances (tethered cord).
On physical examination, the clinician should examine the oropharynx for tonsillar enlargement,
the abdomen for palpable stool, and the spine for sacral dimples or hair tufts (spina bifida
occulta) as well as check the deep tendon and plantar reflexes. The urinalysis is an inexpensive
and simple test to determine urine concentrating ability. Further evaluations, such as
renal/bladder ultrasonography, are undertaken as clinically indicated.
Treatment of the underlying organic cause often results in marked improvement. Once a UTI
is treated, the urinary symptoms and enuresis should resolve. In a recent study, the incidence of
enuresis was 42% in children who had obstructive airway disease. In these patients, who had
adenotonsillar hypertrophy and nocturnal enuresis, adenotonsillectomy resulted in improvement
in 64% and complete resolution of enuresis in 33%. This patient cohort also showed
improvement in daytime frequency and incontinence with adenotonsillectomy.

Suggested reading:

Firoozi F, Batniji R, Aslan AR, Longhurst PA, Kogan BA. Resolution of diurnal incontinence and
nocturnal enuresis after adenotonsillectomy in children. J Urol. 2006;175:1885-1888. DOI:
10.1016/S0022-5347(05)00935-3. Abstract available at:
http://www.ncbi.nlm.nih.gov/pubmed/16600788

Copyright 2010 by the American Academy of Pediatrics page 809


2011 PREP SA on CD-ROM

Graham KM, Levy JB. Enuresis. Pediatr Rev. 2009;30:165-172. DOI: 10.1542/10.1542/pir.30-5-
165. Available at: http://pedsinreview.aappublications.org/cgi/content/full/30/5/165

Lawless MR, McElderry DH. Nocturnal enuresis: current concepts. Pediatr Rev. 2001;22:399-
407. DOI: 10.1542/10.1542/pir.22-12-399. Available at:
http://pedsinreview.aappublications.org/cgi/content/full/22/12/399

Nevus T, von Gontard A, Hoebeke P, et al. The standardization of terminology of lower urinary
tract function in children and adolescents: report from the Standardisation Committee of the
International Children's Continence Society. J Urol. 2006;176:314-324. DOI: 10.1016/S0022-
5347(06)00305-3. Abstract available at: http://www.ncbi.nlm.nih.gov/pubmed/16753432

Copyright 2010 by the American Academy of Pediatrics page 810


2011 PREP SA on CD-ROM

Question: 254

A 6-year-old boy presents with a 2-year history of frequent pruritic, erythematous eruptions on
his arms and legs. The rash usually worsens during winter but occurs intermittently throughout
the year. His mother has tried various moisturizers, but they have not been effective in controlling
the rash. On physical examination, you note erythematous patches on his antecubital and
popliteal regions bilaterally (Item Q254).

Of the following, the MOST appropriate initial step in management for this patient is

A. food allergy skin testing

B. oral antibiotic therapy

C. oral antihistamine therapy

D. topical calcineurin inhibitor therapy

E. topical corticosteroid therapy

Copyright 2010 by the American Academy of Pediatrics page 811


2011 PREP SA on CD-ROM

Question: 254

(Courtesy of D Krowchuk)
Erythematous patch in the antecubital fossa, as described for the boy in the vignette.

Copyright 2010 by the American Academy of Pediatrics page 812


2011 PREP SA on CD-ROM

Critique: 254 Preferred Response: E

The appearance of pruritic, erythematous patches in the antecubital and popliteal areas
described for the boy in the vignette is most consistent with atopic dermatitis. Atopic dermatitis,
also called eczema, is a common skin condition that affects up to 10% of children and frequently
is the first presentation of atopy (ie, atopic dermatitis, allergic rhinitis, and asthma). Because the
underlying mechanism for atopic dermatitis involves defects in the epidermal barrier function and
subsequent evaporative loss, the mainstay of therapy is topical emollients. These include
moisturizers (eg, creams, lotions, ointments) and occlusive products such as solid vegetable
shortening or petroleum jelly. Because skin hydration also is important, patients should apply a
moisturizer or an occlusive product after bathing for 10 to 15 minutes in warm (not lukewarm or
tepid) water.
Almost all children who have eczema have an overabundance of Staphylococcus aureus
and inflammatory cytokine production. Topical corticosteroids are used to reduce inflammation
when emollients and occlusive creams do not control symptoms adequately. Topical
corticosteroids are available in different preparations (eg, ointment, cream, lotion) and potencies.
Regular use of topical corticosteroids is associated with hypopigmentation, bruising, acne, and
thinning of the skin. The patient should be monitored for secondary infections.
Other therapies targeted at reducing bacteria include topical antibiotics, systemic antibiotics,
and bleach baths. One-eighth to one-half cup of bleach per full bath two to three times a week
can reduce S aureus colonization, but such baths also can be irritating to the skin. Further,
although a few studies have demonstrated improvement in eczema using bleach baths, a recent
Cochrane review did not support its use. Oral antibiotics also failed to demonstrate improvement
when analyzed in a meta-analysis.
Because atopic dermatitis typically involves a cyclical process of itching and scratching,
patients frequently receive oral antihistamines to reduce pruritus. Sometimes these are helpful,
but oral antihistamines often are inadequate and usually are added to therapy when topical
emollients or topical corticosteroids fail to control symptoms.
Topical calcineurin inhibitors (eg, pimecrolimus, tacrolimus) were introduced in 2000 and
approved for the treatment of atopic dermatitis in children older than 2 years. However, their use
has been curtailed significantly after a black box warning was added in 2006, a recommendation
that was not supported by the American Academy of Dermatology or a Task Force from the
American College and Academy of Allergy, Asthma, and Immunology. These medications are not
as effective for eczema as moderate- to high-potency topical corticosteroids, but they are an
alternative to mild-potency corticosteroids and result in less hypopigmentation, thinning of the
skin, or bruising.
Both food and inhalant allergens may play roles in atopic dermatitis. Approximately 30% of
children who have moderate-to-severe atopic dermatitis are ingesting a food that is
exacerbating their eczema via an immunoglobulin E- or T cell-mediated process. Consultation
with an allergist and consideration for skin or patch testing to commonly implicated foods (eg,
milk, egg, wheat, soy, peanut) may be warranted. Some investigators have advocated reducing
dust mite concentrations by encasing the mattress, pillow, and box spring in conjunction with
regular washing of the bed linen. Because the child in the vignette only has mild atopic
dermatitis, pursuing food or inhalant allergen testing is not an appropriate initial step.

Copyright 2010 by the American Academy of Pediatrics page 813


2011 PREP SA on CD-ROM

Suggested reading:

Ashcroft DM, Chen LC, Garside R, Stein K, Williams HC. Topical pimecrolimus for eczema.
Cochrane Database Syst Rev. 2007;4:CD005500. DOI: 10.1002/14651858.CD005500.pub2.
Available at:
http://www.mrw.interscience.wiley.com/cochrane/clsysrev/articles/CD005500/frame.html

Bath-Hextall F, Delamere FM, Williams HC. Dietary exclusions for established atopic eczema.
Cochrane Database Syst Rev. 2008;1:CD005203. DOI: 10.1002/14651858.CD005203.pub2.
Available at:
http://www.mrw.interscience.wiley.com/cochrane/clsysrev/articles/CD005203/frame.html

Birnie AJ, Bath-Hextall FJ, Ravenscroft JC, Williams HC. Interventions to reduce Staphylococcus
aureus in the management of atopic eczema. Cochrane Database Syst Rev. 2008;3:CD003871.
DOI: 10.1002/14651858.CD003871.pub2. Available at:
http://www.mrw.interscience.wiley.com/cochrane/clsysrev/articles/CD003871/frame.html

Copyright 2010 by the American Academy of Pediatrics page 814


2011 PREP SA on CD-ROM

Question: 255

A 12-year-old boy is brought to the emergency department after being struck by a car. On
physical examination, he is unresponsive, moaning, and has a large abrasion over his forehead.
His heart rate is 100 beats/min, respiratory rate is 12 breaths/min, and blood pressure is 130/80
mm Hg. His pupils are unequal.

Of the following, the MOST appropriate initial step is to

A. administer mannitol

B. administer tetanus prophylaxis

C. infuse 20 mL/kg of 0.9% saline

D. obtain head computed tomography scan

E. provide assisted ventilation

Copyright 2010 by the American Academy of Pediatrics page 815


2011 PREP SA on CD-ROM

Critique: 255 Preferred Response: E

The initial evaluation and management of the traumatized patient who has evidence of acute
central nervous system (CNS) injury is the same as for any patient who has serious injury.
Evaluation should be performed methodically and incorporate the components of the "primary
survey" (Item C255), a paradigm that incorporates simultaneous assessment of vital functions
and management of life-threatening injuries.
The outcome of a neurologically injured child is maximized by re-establishing and maintaining
adequate brain perfusion, which is primarily dependent on adequacy of oxygenation, ventilation,
and systemic perfusion. For the boy described in the vignette, a respiratory rate of 12
breaths/min suggests inadequate ventilation and should prompt rapid initiation of assisted
ventilation. Fluid resuscitation with 0.9% saline would not be initially indicated for this patient
because he has no signs of hypovolemic shock. Adjunctive therapies, including oncotic agents
such as mannitol or hypertonic (3%) saline or other diuretics such as furosemide may be
considered once vital functions have been restored. Tetanus prophylaxis should be provided to
any traumatized patient who may not be up-to-date on immunizations once the primary survey
has been completed. Radiographic imaging with a noncontrast computed tomography scan is
very helpful in the evaluation of the brain-injured patient, but it should be obtained only after the
patient has been stabilized.

Suggested reading:

American College of Surgeons. ATLS, Advanced Trauma Life Support. 8th ed. Chicago, Ill:
American College of Surgeons; 2008

Tolias C, Sgouros S. Initial evaluation and management of CNS trauma. eMedicine Specialties,
Trauma, Trauma Management. 2008. Available at:
http://emedicine.medscape.com/article/434261-overview

Vavilala MS, Waitayawinyu P, Dooney NM. Initial approach to severe traumatic brain injury in
children. UpToDate Online 17.3. 2009. Available at:
http://www.utdol.com/online/content/topic.do?topicKey=ped_trau/7290&selectedTitle=26~150&s
ource=search_result

Copyright 2010 by the American Academy of Pediatrics page 816


2011 PREP SA on CD-ROM

Critique: 255

Copyright 2010 by the American Academy of Pediatrics page 817


2011 PREP SA on CD-ROM

Question: 256

A previously healthy 11-year-old boy has developed nocturnal enuresis. He does not have
glycosuria, and a serum glucose concentration is in the normal range. A urinalysis reveals no
evidence of infection.

Of the following, the MOST likely abnormal laboratory finding is the serum concentration of

A. bilirubin

B. calcium

C. creatinine

D. potassium

E. total protein

Copyright 2010 by the American Academy of Pediatrics page 818


2011 PREP SA on CD-ROM

Critique: 256 Preferred Response: B

The acute occurrence of nocturnal enuresis in a child who has no urinary tract infection,
such as the boy described in the vignette, often is a sign of polyuria. The causes of polyuria
include development of diabetes mellitus, renal disease, diabetes insipidus, hyperthyroidism,
hypercalcemia, and hypomagnesemia. This boy does not have an abnormal glucose
concentration and has no evidence of kidney infection. Hypercalcemia is more common in a
previously healthy boy than is hypomagnesemia. It may be the first sign of hyperparathyroidism
or perhaps vitamin D toxicity. Other symptoms of hypercalcemia can include altered mental
status, nausea, vomiting, and coma. Hypomagnesemia usually is related to severe magnesium
losses from the gastrointestinal tract or kidneys and is associated with other chronic illness or
with congenital genetic magnesium loss. Hypokalemia might lead to muscle weakness or
problems with cardiac contractility; hyperkalemia could affect myocardial function. Hypokalemia
can affect renal concentrating ability and might lead to nocturnal enuresis, but few disorders
acutely lead to hypokalemia in an 11-year-old child.
Bilirubin values do not influence renal concentration. An elevated creatinine could reflect
severe renal disease, but polyuria is a fairly late marker of this disorder. The total protein in the
serum does not influence urine volume.

Suggested reading:

Benjamin RW, Moats-Staats BM, Calikoglus A, Savendahl L, Chrysis D. Hypercalcemia in


children. Pediatr Endocrinol Rev. 2008;5:778-784. Abstract available at:
http://www.ncbi.nlm.nih.gov/pubmed/18367999

Claudius IS, Fattal O, Nakamoto J, Pitukcheewanont P. Hypercalcemia. eMedicine Specialties,


Pediatrics: General Medicine, Endocrinology. 2009. Available at:
http://emedicine.medscape.com/article/920955-overview819ClinicalCauses

Kollars J, Zarroug AE, van Heerden J, et al. Primary hyperparathyroidism in pediatric patients.
Pediatrics. 2005;115:974-980. DOI: 10.1542/peds.2004-0804. Available at:
http://pediatrics.aappublications.org/cgi/content/full/115/4/974

Copyright 2010 by the American Academy of Pediatrics page 819

You might also like